You are on page 1of 829

y  ax  b

ba ba P  P  1
100% , 100%
a a
1 180o  n  2 
S L   p  a , S L   Ra
2
Pt  Po 1  r 
t
1 1
V  SB h , V   R 2h
3 3
y2  y1
4 a
S  4 R 2 , V   R 3 x2  x1
3
y  y A yB  y A
4 
S  4 R 2 , V   R 3 x  x A xB  x A
3
a 3 d1 / / d 2  a  a
d a 2 , h
2 d1  d 2  a  a  1
AB   x2  x1    y2  y1  a 3  b3   a  b   a 2 ab  b 2 
2 2

x1 f1  x2 f 2  ...  xn f n
x
f1  f 2  ...  f
I. :

II. :

III. :

IV. :

V. :

VI. :

VII. :

៩។


(C.U)
២០១២
GarmÖkfa
sYsþI elakRKÚGñkRKÚ b¥Ún²sisSanusisS nigmitþGñksikSaTUeTARKb;mCÄdæan CaTIeKarBRsLaj;rab;Gan.
mnusSmñak;²Edl)anekItmkehIy EtgEtcg;)annUvsuPmgÁleTAtammhictarbs;eKerog²xøÜn ehIyedIm,ITTYl
)ansuPmgÁl luHRtaEteKmanPaBeCaKC½ykñúgdMeNIrCIvitrbs;eK. y:agNamij ´k¾minxusEbøkBInak;TaMgenaH
Edr KW´cg;)andMeNIrCIviteCaKC½yeTAéf¶GnaKt. sBVéf¶enH´CaRKUbeRgónKNitviTüamñak; dUcenH´)anxitxM
RsavRCav nigsresremeronEdlTak;TgnwgKNitviTüaCaeRcIn Cak;EsþgdUcCaesovePA EdlGñkkMBugkan;Gan
kñúgédenH KWCaKeRmagFMTI4 ehIyesovePAenHmaneQµaHfa { KNitviTüa Gan> Kit> yl; fñak;TI9 } . crit
lkçN³énesovePAenH KWRbmUlpþMúemeron lMhat; viBaØasa nigcMNucepSg²EdlTak;TgnwgKNitviTüafñak;TI9 .
eTaHbICa´)anRtÜtBinitümuneBle)aHBum< nig)ane)aHBum<sakl,gedIm,IRtÜtBinitüy:agykcitþTukdak;
y:agNak¾edayk¾minGaceFVI[esovePAenHmanPaBl¥\tex©aH)aneLIy dUcenH´eCOCak;favanwgmankMhus enHKW
edaysarcMeNHdwgenAmankRmit TaMgEpñkbec©keTsKruekaslü GkçraviruT§ rcnabf nigbec©keTskMuBüÚT½r b¤erOgrav
bec©keTsepSg²eTot. Et´ KitfakMhusenaH KWtictYcb:ueNÑaHebIeFobnwgTMhMesovePAenH. eTaHbICamankMhus
qÁgy:agdUcemþ:ck¾eday sUmmitþGñkGanemtþaGFüaRs½y GP½yeTas dl;´pg ehIy´sUmrgcaM karriHKn;y:ag
rIkray BIsMNak;mitþGñkGanRKb;mCÄdæanTaMgGs; edIm,I[kare)aHBum< esovePAenHelIkeRkay kan;EtRbesIrCagmun.
´EdlCaGñkeroberogsgÇwmfa elakGñkmitþGñkGannwgeRbIR)as;esovePAenH eFVICaÉksarCMnYykñúg
karsikSaKNitviTüa minEmneRbIR)as;esovePAenH edIm,IebIkcmøgeBlRbLgeLIy. sUmmitþGñkGanRbwgERbg
bnþeTot edIm,IeFVI[KNitviTüakm<úCamanPaBrIkceRmIn edaysarsm,ÚrGñkmancMeNHdwgEpñkKNitviTüa.
sUmmitþGñkGan mansuxPaBl¥ suvtßiPaBRKb;eBl ehIyeronsURt)anBUEk nigTTYl)aneCaKC½ykñgú
dMeNIrCIvit .

sVayerog> éf¶TI 12 Ex FñÚ qñaM 2012


yuvnisSit ÉkeTsKNitviTüa
esckþIEføgGMNrKuN
sUmGrKuNCUndl;elakk«Buk Ca cMerIn nigGñkmþay eBRC esaP½NÐ EdlelakTaMgBIr)anpþl;kMeNIt
dl;kUn ehIybI)ac;EfrkSaciBa©wmkUn CYy[kUn)ansikSarhUt)ankøayCaRKÚbeRgónmñak;. sUmCUnBrelakTaMgBIr
manRBHCnµynW yUr RbkbedaysuxPaBl¥ edIm,ICYypþl;PaBkk;ekþAdl;kUnecACMnan;eRkay. GrKuNdl;bgb¥Ún
RbusRsITaMg 4 nak; Edl)anCYypÁt;pÁg; nigeRCamERCgkareronsURtrbs;´ TaMgfvikar nigsmÖar³ CaBiessKW
kMuBüÚT½r eFVI[´GacBRgwg nigBRgIkcMeNHdwgEpñkviTüasaRsþ)anRbesIrCagmun. CUnBrdl;dl;bgb¥ÚnTaMg 4 man
suxPaBl¥ nigRbkbrbrkak;kbekIncMNUl.
sUmGrKuNdl;elakRKU Kg; Narin sBVéf¶Canaykén GnuviTüal½y eKakRBIg EdlelakRKÚ )anxitxM
RbwgERbgminsþaykmøaMgkñúgkarbgðat;beRgón´ k¾dUcCasisSdéTeTotenAkñúgfñak;. ´Kitfa cMeNHdwgKNitviTüa
kRmitbzmPUmirbs;´CaeRcIn )anedaysarkarRbwgERbg nigBüayamsþab;karbgðat;beRgónrbs;elakRKU.
dUcenH´sUmTTYlsÁal;KuNd¾FMeFgenH Edl´minGacbMePøc)aneTAéf¶GnaKt. sUmCUnBrdl;elakRKU nigRkum
RKÜsar sUmmansuxPaBl¥ nigsuvtßiPaBRKb;eBlevla .
sUmGrRBHKuNdl;RBHetCKuN Pwm suPNÐ CaRKÚecAGFikarvtþéRBrMdYl EdlRBHGgÁ)anGnuBaØati[´
RBHkruNa sñak;GaRs½yenAkñúgvtþ nigeRbIR)as;GKÁisnI kñúgkarerobcMemeron RBmTaMg)anCYypÁt;pÁg;xagPsþúPa
b¤smÖa³epSg². sUmRbeKnBrdl;RBHetCKuNmanRBHCnµyWnyUr.
sUmGrKuNdl;GñkRKÚ Niko CaGñksµ½RKcitþCnCatiCb:un EdlGñkRKÚ)anpþl;kmøaMgcitþ nigelIkTwkcitþdl;
rUb´kñúgkarRbkbrbrCaRKÚbeRgón nigCYy]btßmdl;rbU ´nUvfvikamYycMnYnFM. sUmCUnBrGñkRKÚmansuxPaBl¥
CaBiessenHeBlGñkRKÚeFVIdMeNIrmatuPUminivtþn_eTAkan;RbeTsCb:unvij sUmmansuvtßiPaBx<s;bMput.
sUmGrKuN dl;mCÄmNÐlKruekaslüPUmiPaK k¾dUcCaGnuviTüal½yGnuvtþ Edl)anpþl;Ca TIkEnøg nig
GKÁisnI edIm,IerobcMemeron. sUmGrKuNdl;GñkRKU vNÐI pløa CaRKU]eTsKNitviTüa / GñkRKU Ca suKn§arI
CaRKUKMrUKNitviTüa . GrKuNdl; KrusisSfñak;ÉkeTs KNitviTüa-rUbviTüa rYmCMnan;TI21 man ³
1> elak Gan suxKn§a 2> kBaØa pl RsIFa 3> kBaØa qay suKn§amMu
4> kBaØa pat; suvNÑarI 5> kBaØa sinu suKn§a 6> elak suxa kuslü
7> elak Ém suxsuvNÑ 8> kBaØa Kg; r½tñFI 9> elak kun Git
10 elak yn; m:ar:U 11 elak Gul eGn 12 elak Kwm pan;Na
13 kBaØa v:an; cnßa 14 kBaØa søkw y:at 15> kBaØa sumw sIuNat
16 elak BUk suxsanþ 17> elak etg sarI 18 elak Cwm sar:at;
19 elak raC vutßa 20> elak vn lINa 21 elak mas Parmü
22 kBaØa ey:ak bu)aö 23 elak hYn siuj 24 KrusisS eRbIs dar:a .

CakarBitkarsikSaKNitviTüa vaBMuEmnCakargayRsYlenaHeT CaBiessKWkarsikSaedIm,I[køayxøÜnCasisS


BUEkEtmþg. vaTamTar[eyIgmankarts‘U Büayam GMNt; Gt;Fµt; CaeRcInTaMgkay nigcitþ EtBMuEmnmann½yfa
eyIgminGaceFVI)anenaHeT. eBlsikSaeyIgRtUvEck[dac;fa {etIeyIgeronykecH b¤eronykCab; ?} . kalNa
eyIgcg;køayxøÜneTACasisSBUEkKNitviTüa eyIgRtUveronykecH KWmann½yfaeyIgRtUvyl;[c,as; GMBIemeron
rUbmnþ b¤lMhat;EdleyIg)aneronrYc minEmneronbnøMbgáÚvxøÜnÉgenaHeT. kareronbnøMxøÜnÉg nwgeFV[I eyIg køayeTACa
mnusSsÞak;esÞrI rhUtdl;maneBlxøH minh‘anfaxøÜnÉgxus b¤RtUv k¾man. skmµPaBEbbenH nwgeFVI[eyIgTTYl)an
braC½ykñúgkarsikSa naM[eyIgmanGnaKtminl¥.
kñúgnamCaGñkFøab;qøgkat;karsikSa ehIyk¾Føab;TTYl)anTaMgbraC½y nigeCaKC½y kñúgkarsikSaEpñk
KNitviTüa ´sUmbgðajGMBbI TBiesaFn_braC½yrbs;´dUcteTA ³
-minyl;BIrebobénkarRbLg b¤rebobénkarsresrviBaØasa.
-min)anBRgwgsmtßPaBxøÜnÉg elIlMhat;nImYy²[)anc,as;las; ¬emIlgaylMhat;¦.
-caMEtTTYlkarbgðat;beRgónBIRKU min)ansV½ysikSa min)aneFVIkarRsavRCavelIlMhat;epSg².
-manemaTnPaBRCulelIxøÜnÉg >>>.
skmµPaBTaMgGs;xagelIenHehIy EdleFVI[´TTYl)anbraC½ykñúgkarsikSaKNitviTüa .
sUmkMue)aHbg;ecalkartaMgcitþ enAeBlEdleyIgcab;epþImdMbUg ehIyeyIgCYblMhat;sisSBUEksuT§EtlM)ak
enaHvaCaerOgFmµtaeT eRBaHeyIgminTan;manbTBiesaFn_RKab;RKan;edIm,IedaHRsayva. eyIgRtUvKitkñúgcitþfa
{lMhat;EdleFVIecj KWCalMhat;EdlFøab;CYb } . XøaenHKWcg;mann½yfa sisSBUEkKNitviTüaesÞIrEtTaMgGs;suT§Et
CaGñkmanbTBiesaFn_Føab;CYblMhat;eRcIn dUcenHeFVI[eKGacdwgGMBIviFI l,ic b¤bec©keTsedIm,IedaHRsaylMhat;
TaMgenaH . kalNaeyIgedaHRsaylMhat;sisSBUEk)ankan;EteRcIn enaHeFVI[eyIgdwgBIviFIedaHRsaykan;EteRcIn
Edr. edIm,IkøayxøÜneTACasisSBUEkKNitviTüa GñkminRtUvmanKMnitdUcxageRkam ³
-eKeronecHmkBIeKCakUnGñkman manluyeronKY b¤CYlRKUbeRgón .
-lMhat;eKCYbl¥² TajecjBIGIunF½reNt rIÉeyIgKµanlT§PaBdUceK .
-eKCamnusSmandugtaMgBIkMeNIt .
-suxPaBeKl¥ >>>.
sUmkMumanKMnitEbbenH vaKµanRbeyaCn_GVIsRmab;eyIgeT eRkABIeFVI[eyIgtUccitþelIxøÜnÉg.
cab;BIeBlenHteTAeyIgRtUvxMeronsURtbEnßmeTot RtUvRsavRCav edaHRsaylMhat;[)aneRcIn ecosvag
karx¢il. bisacx¢ilxøackareFVIkargarCaTmøab;Nas; dUcenHsUmkM[u bisacx¢ilcUlxøÜnrbs;eyIg KWeyIgRtUvman
EpnkareronsURt[)anc,as;las; ehIyGnuvtþkareronsURtTaMgenaH[køayeTACaTmøab;.
CacugeRkaysUmCUnBrdl;mti þGñkGanTaMgGs; mansuxPaBl¥ edIm,IeronsURtkøayeTACamnusSmansmtßPaB
nigTTYl)aneCaKC½y edIm,IGPivDÆRbeTsCatieyIg[manPaBrIkceRmIn.
CIvRbvtþisegçbrbs;Gñkeroberog
eRbIs dar:a PROEUS DARA .
³ 04 1988 24 1989 )។

³ ។



5 2 4។
³ 090 250 667 ។
³ 011 50 70 65 / 0977 65 70 50 ។
៩ ³ E-mail : dara_st04@yahoo.com ។
³ ។

2009 ។
³ ។

³ ។

³ 1997-2003 ³
2003-2009 ³

³ (RUPP)

2009-2011 ³

2011- ³ (KU)

99999999999

3
I ³
1> cMnYnGsniTan ............................................001 10> smIkarénbnÞat; ....................................... 008
2> smamaRt ..................................................002 11> RbB½n§smIkardWeRkTI 1manBIrGBaØat ......... 009

3> kenSamBICKNit .........................................003 12> RTwsþIbTBItaK½r ........................................ 009

4> smIkardWeRkTI 1manmYyGBaØat ..................004 13> rgVg; nigbnÞat; .......................................... 010

5> vismIkardWeRkTI 1manmYyGBaØat ..................005 14> lkçN³mMuénrgVg; ...................................... 011

6> bMENgEckeRbkg; ......................................006 15> RTwsþIbTtaEls ....................................... 013

7> mFümsßiti ..................................................007 16> RtIekaNdUcKña ......................................... 014

8> RbU)ab ........................................................008 17> BhuekaN ............................................... 016

9> cm¶ayrvagBIrcMNuc ....................................008 18> sUlIt ..................................................... 017

II ³
-ផ្ផែក ប្បធានលំហាត់ប្បតប
ិ តតិ -ផ្ផែក កំផ្ែលំហាត់ប្បតប
ិ តតិ

1> cMnYnGsniTan ............................................018 1> cMnYnGsniTan ........................................... 037


2> smamaRt ..................................................019 2> smamaRt ................................................. 039

3> kenSamBICKNit .........................................020 3> kenSamBICKNit ........................................ 040

4> smIkardWeRkTI 1manmYyGBaØat ..................021 4> smIkardWeRkTI 1manmYyGBaØat ................. 042

5> vismIkardWeRkTI 1manmYyGBaØat ..................022 5> vismIkardWeRkTI 1manmYyGBaØat.................. 044

6> bMENgEckeRbkg; ......................................023 6> bMENgEckeRbkg; ..................................... 046

7> mFümsßiti ..................................................024 7> mFümsßiti ................................................. 048

8> RbU)ab ........................................................025 8> RbU)ab ........................................................ 050

9> cm¶ayrvagBIrcMNuc ....................................026 9> cm¶ayrvagBIrcMNuc ................................... 051

10> smIkarénbnÞat; ........................................027 10> smIkarénbnÞat; ....................................... 052

11> RbB½n§smIkardWeRkTI 1manBIrGBaØat ..........028 11> RbB½n§smIkardWeRkTI 1manBIrGBaØat ......... 055

12> RTwsþIbTBItaK½r .........................................029 12> RTwsþIbTBItaK½r ........................................ 057

13> rgVg; nigbnÞat; ...........................................030 13> rgVg; nigbnÞat; .......................................... 058

14> lkçN³mMuénrgVg; .......................................031 14> lkçN³mMuénrgVg; ...................................... 060

15> RTwsþIbTtaEls .......................................033 15> RTwsþIbTtaEls ....................................... 062

16> RtIekaNdUcKña ..........................................034 16> RtIekaNdUcKña ......................................... 063

17> BhuekaN ................................................035 17> BhuekaN ............................................... 065

18> sUlIt ......................................................036 18> sUlIt ..................................................... 066


i
III ³
1> cMnYnGsniTan ............................................068 10> smIkarénbnÞat; ....................................... 149
2> smamaRt ..................................................082 11> RbB½n§smIkardWeRkTI 1manBIrGBaØat ......... 162

3> kenSamBICKNit .........................................085 12> RTwsþIbTBItaK½r ........................................ 176

4> smIkardWeRkTI 1manmYyGBaØat ..................097 13> rgVg; nigbnÞat; .......................................... 181

5> vismIkardWeRkTI 1manmYyGBaØat ..................106 14> lkçN³mMuénrgVg; ...................................... 189

6> bMENgEckeRbkg; ......................................115 15> RTwsþIbTtaEls ....................................... 196

7> mFümsßiti ..................................................122 16> RtIekaNdUcKña ......................................... 203

8> RbU)ab ........................................................129 17> BhuekaN ............................................... 217

9> cm¶ayrvagBIrcMNuc ....................................137 18> sUlIt ...................................................... 225

IV ³
ផ្ផែកលំ ....238 ផ្ផែកចម្លយ
ើ ននលំ ផ្ែលបាន ............. 309

V ³
1> DIsbøÚmqñaM 1981 ........................................352 13> DIsbøÚmqñaM 1993 ..................................... 398
2> DIsbøÚmqñaM 1982 ........................................356 14> DIsbøÚmqñaM 1994 ...................................... 400

3> DIsbøÚmqñaM 1983 .......................................360 15> DIsbøÚmqñaM 1995 (មលើកទ១ ី ) ...................... 404

4> DIsbøÚmqñaM 1984 ........................................362 16> DIsbøÚmqñaM 1995 (មលើកទ២ ី ) ..................... 408

5> DIsbøÚmqñaM 1985 ........................................366 17> DIsbøÚmqñaM 1996 (មលើកទី១) ..................... 412

6> DIsbøÚmqñaM 1986 .......................................370 18> DIsbøÚmqñaM 1996 (មលើកទី២) .................... 416

7> DIsbøÚmqñaM 1987 .......................................374 19> DIsbøÚmqñaM 1997 (មលើកទី១)...................... 420

8> DIsbøÚmqñaM 1988 ........................................378 20> DIsbøÚmqñaM 1997 (មលើកទី២) ..................... 420

9> DIsbøÚmqñaM 1989 ........................................382 21> DIsbøÚmqñaM 1998 (មលើកទី១) ...................... 422

10> DIsbøÚmqñaM 1990 ......................................386 22> DIsbøÚmqñaM 1998 (មលើកទី២) ..................... 424

11> DIsbøÚmqñaM 1991 ......................................390 23> DIsbøÚmqñaM 1999 (មលើកទី១) .................... 428

12> DIsbøÚmqñaM 1992 ......................................394 24> DIsbøÚmqñaM 1999 (មលើកទ២ ី ) ...................... 432

ii
25> DIsbøÚmqñaM 2000 .....................................436 32> DIsbøÚmqñaM 2007 ..................................... 466
26> DIsbøÚmqñaM 2001 .....................................440 33> DIsbøÚmqñaM 2008 ..................................... 470

27> DIsbøÚmqñaM 2002 .....................................444 34> DIsbøÚmqñaM 2009 ..................................... 474

28> DIsbøÚmqñaM 2003 ....................................448 35> DIsbøÚmqñaM 2010 ..................................... 478

29> DIsbøÚmqñaM 2004 ......................................454 36> DIsbøÚmqñaM 2011 .................................... 482

30> DIsbøÚmqñaM 2005 ......................................458 37> DIsbøÚmqñaM 2012 ..................................... 486

31> DIsbøÚmqñaM 2006 ......................................462

VI ³
1> sisSBUqñaM 1986 ¬elIkTI1¦ ......................490 19> sisSBUqñaM 1995 ¬elIkTI1¦ ..................... 508
2> sisSBUqñaM 1986 ¬elIkTI2¦ ........................491 20> sisSBUqñaM 1995 ¬elIkTI2¦ ..................... 509

3> sisSBUqñaM 1987 ¬elIkTI1¦ .......................492 21> sisSBUqñaM 1996 ¬elIkTI1¦ ..................... 510

4> sisSBUqñaM 1987 ¬elIkTI2¦ .......................493 22> sisSBUqñaM 1996 ¬elIkTI2¦ ..................... 511

5> sisSBUqñaM 1988 ¬elIkTI1¦........................494 23> sisSBUqñaM 1997 ¬elIkTI1¦..................... 512

6> sisSBUqñaM 1988 ¬elIkTI2¦ ........................495 24> sisSBUqñaM 1997 ¬elIkTI2¦ ..................... 513

7> sisSBUqñaM 1989 ¬elIkTI1¦ ........................496 25> sisSBUqñaM 1998 ¬elIkTI1¦ ..................... 514

8> sisSBUqñaM 1989 ¬elIkTI2¦ ........................497 26> sisSBUqñaM 1998 ¬elIkTI2¦ ..................... 515

9> sisSBUqñaM 1990 ¬elIkTI1¦ ........................498 27> sisSBUqñaM 1999 ¬elIkTI1¦ ..................... 516

10> sisSBUqñaM 1990 ¬elIkTI2¦ ......................499 28> sisSBUqñaM 1999 ¬elIkTI2¦ ..................... 517

11> sisSBUqñaM 1991 ¬elIkTI1¦ .....................500 29> sisSBUqñaM 2000 ¬elIkTI1¦ ..................... 518

12> sisSBUqñaM 1991 ¬elIkTI2¦ .....................501 30> sisSBUqñaM 2000 ¬elIkTI2¦ ..................... 519

13> sisSBUqñaM 1992 ¬elIkTI1¦......................502 31> sisSBUqñaM 2001 ¬elIkTI1¦ .................... 520

14> sisSBUqñaM 1992 ¬elIkTI2¦......................503 32> sisSBUqñaM 2001 ¬elIkTI2¦ .................... 521

15> sisSBUqñaM 1993 ¬elIkTI1¦ .....................504 33> sisSBUqñaM 2002 ¬elIkTI1¦ .................... 522

16> sisSBUqñaM 1993 ¬elIkTI2¦ .....................505 34> sisSBUqñaM 2002 ¬elIkTI2¦ .................... 523

17> sisSBUqñaM 1994 ¬elIkTI1¦ ......................506 35> sisSBUqñaM 2003 ¬elIkTI1¦ .................... 524

18> sisSBUqñaM 1994 ¬elIkTI2¦ ......................507 36> sisSBUqñaM 2003 ¬elIkTI2¦ .................... 525

iii
37> sisSBUqñaM 2004 ¬elIkTI1¦......................526 47> sisSBUqñaM 2009 ¬elIkTI1¦ ..................... 536
38> sisSBUqñaM 2004 ¬elIkTI2¦ ......................527 48> sisSBUqñaM 2009 ¬elIkTI2¦ ..................... 537

39> sisSBUqñaM 2005 ¬elIkTI1¦ ......................528 49> sisSBUqñaM 2010 ¬elIkTI1¦ .................... 538

40> sisSBUqñaM 2005 ¬elIkTI2¦ ......................529 50> sisSBUqñaM 2010 ¬elIkTI2¦ .................... 539

41> sisSBUqñaM 2006 ¬elIkTI1¦......................530 51> sisSBUqñaM 2011 ¬elIkTI1¦ .................... 540

42> sisSBUqñaM 2006 ¬elIkTI2¦......................531 52> sisSBUqñaM 2011 ¬elIkTI2¦ .................... 541

43> sisSBUqñaM 2007 ¬elIkTI1¦ .....................532 53> sisSBUqñaM 2012 ¬elIkTI1¦ .................... 542

44> sisSBUqñaM 2007 ¬elIkTI2¦ .....................533 54> sisSBUqñaM 2012 ¬elIkTI2¦ .................... 544

45> sisSBUqñaM 2008 ¬elIkTI1¦......................534 55> វញ្ញ


ិ ា សាម្ែើប្បឡង្ិ្សពូផ្ក........................ 545

46> sisSBUqñaM 2008 ¬elIkTI2¦......................535 56> វញ្ញ


ិ ា សាធាលប់ប្បឡង្ិ្សពូផ្កតា្មេតត .......... 625

VII ³
1 > EpñkRbFanlMhat;l¥² .................................645 2 > EpñkcemøIyénlMhat;l¥² ............................ 669

VIII ³
>
1 ក្ានតគែិតវទា
ិ ........................................781 >
4 តារាង្វ័យគុែ និងឫ្ទី n ....................... 796

2> ចម្លយ
ើ ននក្ានតគែិតវទា ិ ..........................787 5> តារាងតន្លផលមធៀបប្តមី ោែមាប្ត ............... 798

3> បរមាប្ត
ិ នផៃប្កឡា នង
ិ មាឌរូបធរែីមាប្ត .........793 6> ខ្នែតននរង្វវ្់រង្វវល់មផសងៗ ........................... 799



3
iv
sYsþI¡ elakGñkmitþGñkGanCaTIRsLaj;rab;Gan enAkñúgEpñkenHelakGñknwg)aneXIj
KNitviTüafñak;TI9 Gacniyay)anfaCaEpñksMxan;énemeronnImYy² . cMNucénemeronnImyY ²
erobtamlMdab;lMeday
Epñk vaCYysRmÜldl;

RbsinebIelakGñkmanbBaða b¤cm¶l;Rtg;cMNucNa EdlmanenAkñúgEpñkenH elakGñkGacTak;Tg;eTAkan;


RKU b¤mitþPkþirbs;GñkEdlmansmtßPaB b¤GñkeroberogesovePAenHkñúgeBlevlaKYsm .

i


œ ១ œ
១ ៣៣

x2  a ³
- a0 x a

- a0 x0
៣៤

- a0 x

x3  a x3 a a

៣៥
 : x a 3
³
 3 3
 a

 :
៣.
៣១

៣៦

 a  b  ab a0 b0


 3 a 3 b  3 ab a ,b

៣២ a  a  a , 3 b  3 b2  b
-


a

a
a0 nig b  0  a b  
a  b  a b
b b
-
3
a 3 a
 3
 b0
b
 
b
 3
a3b 3
a2 3
ab  3 b2  a  b

 1 


D ២ E

a c
 ad  bc
b d

1 mile = 1.609 km
1 inch = 2.54 cm
1 feet = 0.3048 m
1 lb = 0.453 kg

២.

a
a b 100%
b

៣. ឬថ

- a b
ba
100%
a
- b a
ba
 100%
b

៤.

Po /r
/ t

- ១ P  P  rP  P 1  r 
1 o o o

- ២ P  P  rP  P 1  r   P 1  r 
2 1 1 1 o
2

- ៣ P  P  rP  P 1  r   P 1  r 
3 2 2 2 o
3

>>>>>>>>>>>>>>>>>>>>>>>>>>>>>>>>>>>>>>>>>>>>>>>>>>>>>>>>>>>>>>>>>>>>>>>>>>
- t P  P 1  r 
t o
t

 2 


‹ ៣ …

១ ២.៤

១ ២ A  x 2  2x  8

a  bc  d   ac  ad  bc  bd A  x 2  2x  8
 
 x 2  2x  1 1  8
 x  1  9
2
 :

1. a  b a  b   a 2  b 2  x  1  3x  1  3
 x  4x  2
2. a  b a  b   a 2  2ab  b 2
A  x 2  2 x  8  x  4x  2
3. a  b a  b   a 2  2ab  b 2
 
4. a  b  a 2  ab  b 2  a 3  b 3
5. a  b a  a ៣
2
 ab  b 2 3
 b3
៣១
២.
³
-

២.១ -

-
ka  kb  k a  b  , k
-

២.២ -

rUbmnþ 1. a 2  b 2  a  b a  b 
៣២
2. a 2  2ab  b 2  a  b a  b 
3. a 2  2ab  b 2  a  b a  b  ³
-
២.៣
-
A  x2   a  b  x  ab

-
2x a
x   ab
x b
ax  bx   a  b  x
A   x  a  x  b 

 3 


„ ៤ 1 „

១ ១ ៤. ១ ២

eyIgGacedaHRsaysmIkardWeRkTI1 manmYy edIm,IedaHRsaysmIkarmanmYyGBaØat EdlGgÁ


GBaØat edayeRbIlkçN³smPaBdUcxageRkam ³ TI1manGBaØatCadWeRkTI2 eKRtUvsresrGgÁTI1
-ebI a  b enaH a  c  b  c CaplKuNktþaéndWeRkTI1 .
-ebI a  b enaH a  c  b  c
៥. ១
-ebI a  b enaH a  c  b  c
-ebI a  b enaH ac  bc , c  0 edIm,IedaHRsaycMeNaTsmIkardWeRkTI1man
mYyGBaØat GñkRtUveFVItamCMhandUcxageRkam ³
 :
-smµtikmµ ³ ¬GVI²EdleK[ b¤eKR)ab;¦
-sMNYr ³ ¬GVIEdleK cg;[eyIgrk¦
-tagGBaØat ³¬eRCIserIsGBaØattagGVIEdleKsYr¦
២.
-KMnitbEnßm ³ ¬ebIcMeNHdwg edIm,IP¢ab;TnM ak;TMng
edIm,IedaHRsaysmIkardWeRkTI1manmYyGBaØat rvagGVIEdleKR)ab; nigGVIEdleKsYr >>>¦
EdlmanPaKEbgCaelx eKRtUv ³ -smIkar ³¬sresrsmIkartamTMnak;TMngxagelI¦
-tRmÚvPaKEbgrYménGgÁTaMgBIrrbs;smIkarrYclub -edaHRsaysmIkar ³ ¬rkb£sénsmIkar¦
PaKEbgrYmenaHecal -epÞógpÞat;cemøIy ³ ¬BinitüeLIgvij¦
-elIktYEdlmanGBaØatmkGgÁmçag nigtYEdlKµan
GBaØateTAGgÁmçageTot ehIytYNabþÚrGgÁ RtUv
bþÚrsBaØatYenaH rYceFVIRbmaNviFI .
៣. ១

BinitütaragplKuNénktþaesµIsUnüxageRkam ³
A B A×B
4 0 0
0 4 0
0 0 0
A×B
A=0 B=0

 4 


Û ៥ 1 Ù

១ ២. ១
២.១
១១ ឬ

ebIeKbUk b¤dkcMnYnEtmYyelIGgÁTaMgBIrén xageRkamCarebobbkRsaycemøIyelIRkab ³


a
vismPaB enaHeK)anvismPaBfµIEdlmanTisedA -krNI x  a ³ (

dUcvismPaBedIm . -krNI x  a ³
a
)
-ebI a  b enaH a  c  b  c
-ebI a  b enaH a  c  b  c -krNI x  a ³ [
a

a
- krNI x  a ³ ]
១២ ឬ
 : eyIgeRbIvg;RkckEbrrkcemøy I ebI
ebIeKKuN b¤EckcMnYn viC¢man EtmYyelIGgÁ vismIkarFMCag b¤tUcCagdac;xat KWmansBaØa  , 
TaMgBIrén vismPaB enaHeK)anvismPaBfµIEdl mann½yfa a minEmnCacemøyI . ebIvismmIkarFMCag
manTisedA dUc vismPaBedIm . b¤esµI b¤ tUcCagb¤esµI eyIgeRbI XñabCMnYsvij.
-cMeBaH c  0 ebI a  b enaH a  c  b  c ២.២

-cMeBaH c  0 ebI a  b enaH a  c  b  c eyIgedaHRsayvismIkar edayeRbIlkçN³vismPaB


៣. ១
១៣ ឬ
edIm,IedaHRsayRbB½n§vismIkardWeRkTI1man
ebIeKKuN b¤EckcMnYn GviC¢man EtmYyelIGgÁ mYyGBaØat eyIgRtUvedaHRsayvismIkarnImYy²
TaMgBIrén vismPaB enaHeK)anvismPaBfµIEdl rYcykcemøIyénvismIkarTaMgenaHRbsBVKañ .
manTisedA pÞúy BIvismPaBedIm .
-cMeBaH c  0 ebI a  b enaH a  c  b  c ៤.

-cMeBaH c  0 ebI a  b enaH a  c  b  c

 5 


¬ ៦ ¬

១ ៣.

CaFmµta enAeBlEdleKRbmUl)anTinñn½yepSg² cMeBaHTinñn½yxøH eKmankarlM)akkñúgkarRbmUlpþúMva


eKEtgEtCYbRbTHfaTinñn½yxøHtUc xøHFM xøHtic xøHeRcIn eTAtameRbkg; edaysarTinñn½ynImYy²BMumancMnYndg
ehIysßitenAkñúgPaBray):ayBIKña EdlCamUlehtu eRcIn. mü:agvijeTotTinñny½ xøHmanKmøatKñaq¶ayeBk
Bi)akkñúgkarsikSaTinñn½yTaMgenaH . rvagRbePTTinñn½yEdltUcCageK nigTinñn½yEdlFM
dUcenH eKRtUvpþúMTinñn½yTaMgenaH eTAtamcMnYndg CageK . dUcenH edIm,I[gayRsYl eKbegáItkarpþMú
rbs;va ehIykareFVIEbbenH ehAfakarRbmUlpþúMTinñn½y Tinñn½yCafñak; .
CaeRbkg; . elIsBIenHeTot eKk¾GaceRbóbeFob RkabssrenACab;²Kña ehAfa GIusþÚRkam .
eRbkg; eTAnwgeRbkg;srub edIm,IKitCaPaKryehAfa enAeBlxøH eKeRbItémøkNþalénfñak; mktag[
eRbkg;eFob . témøénfñak; EdltémøkNþalenH ehAfa p©iténfñak;
vaCamFüméncugfñak;TaMgBIr .
eRbkg;eFobéntémømYyCapleFobrvag
mü:agvijeTot ebIeKP¢ab;p©itfñak;bnþbnÞab;
eRbkg;éntémøenaH nigeRbkg;srub .
enaHeK )anExSkac; ehAfa RkabénBhuekaN .
២.
២ ១.

eRbkg;ekIn CaeRbkg;Edl)aneday bUkeRbkg;


bnþbnÞab;BIelIcuHeRkam .
២ ២. ថ

eRbkg;fy CaeRbkg;Edl)aneday dkeRbkg;


srubCamYyeRbkg;bnþbnÞab; BIelIcuHeRkam .
 : eKtag ³
-eRbkg; eday f -eRbkg;Fob eday f %
-eRbkg;ekIn eday f -eRbkg;ekIneFob eday f 
%

-eRbkg;fy eday f -eRbkg;ថ eFobeday f 


%

 6 


{ ៧ |

១ 

x1 f1  x2 f 2  x3 f3  ...  xn f n
x
f1  f 2  f 3  ...  f n 50  60
50  60  55 )
2
២ -
x1 f1  x2 f 2  x3 f3  ...  xn f n
x
f1  f 2  f 3  ...  f n
-
n
n 1
2
 n
 n
 : me

៣.

៤.

-
x1 f1  x2 f 2  x3 f3  ...  xn f n
x
f1  f 2  f 3  ...  f n
-

 7 


Í ៨ Í

១ ២

P' P P  P' 1

P
cnMYnkrNRIsb
cnMnYkrNGIac

Ê ៩ Ê

១ ២

A  x1 , y1  B  x2 , y2  A  x1 , y1  B  x2 , y2 
AB   x2  x1    y2  y1  x x y  y2 
2 2
M 1 2 , 1 
 2 2 

n ១០ n

១១  xh oy
y ox xh
x  yk ox
 y  ax  b a oy yk

 ២

A  x1 , y1  B  x2 , y2  a
y2  y1 d1 : y  ax  b d 2 : y  ax  b
a
x2  x1
 d1 / / d 2 a  a
១២  d1  d 2 a  a  1

 y  ax  b yy xx
 y  ax

 8 


r ១១ ១ s

១ ៣

 y  ax  b 1

 y  ax  b  2 
1
 2 ៤

edIm,IedaHRsaycMeNaTRbB½ smIkardWeRkTI1man
 :
GBaØat GñkRtUveFVItamCMhandUcxageRkam ³
-
-smµtikmµ ³ ¬GVI²EdleK[ b¤eKR)ab;¦
-

-
-sMNYr ³ ¬GVIEdleK cg;[eyIgrk¦
-tagGBaØat ³¬eRCIserIsGBaØattagGVIEdleKsYr¦
-KMnitbEnßm ³ ¬ebIcMeNHdwg edIm,IP¢ab;TnM ak;TMng

rvagGVIEdleKR)ab; nigGVIEdleKsYr >>>¦
-RbB½ smIkar ³¬sresr smIkar¦
1
-edaHRsay smIkar ³ ¬rk ¦
 2
-epÞógpÞat;cemøIy ³ ¬BinitüeLIgvij¦

; ១២ ;


-
a a
a 3 h
c a h
a 2 a
c  a b
2 2 2

b

- z
y
x, y,z
x t
a
- t  x y z
2 2 2

d a
a d a 2

 9 


? ១៣ ?

១ A
១១
B C

B

o A


C

OBA  OCA  90o


o

AB  AC
១២ ៣

OAB  OAC
COA  BOA


 o

២ B
A

AB  CD  BC  AD o

C
D
o ៥




o

 10 


S ១៤ S

១ ២៤

b b
   b
- AB  AC AOB  AOC a a a

- AOB  AOC AB  AC
B - a b
a
o A a  2b b
2

C

២១ a  90o
a

o

- BAC B A

 o
៤១
C

២២

A
B B
A o
 o
BC
- BAC  D C
2
C
៤២
២៣

A
A
C B
A  C  180o
- A  C o
o
B b  d  180o
B  D D D C

 11 


p  q, r  s

p
s
o
q
r

៦១

A C
I
AB  CD o
I  D
2
B

៦២

A
C
AB  CD o P
P 
2 D
B

B
A
o

X Y
P
- XPA  PBA

 12 


R ១៥ T

A
-  BC  / /  EF 
AE AF EF
-  d1  / /  d2  / /  d3  / /  d4    E F
AB AC BC B C
AB  BC  CD A E d1
B F d2 ៣១

EF  FG  GH C G d3
D H d
4

A A d1
-  d1  / /  d2  / /  d3  B B D C E F
d2
AB AB
 D C
BC BC 
E F

AB BC C C d
 3 A B A B
AB BC 

៣១

EF ABC
AE AF
EF / / BC 
AB AC
AE AF
 EF / / BC
AB AC

A A
E F

B C A
E F

B C E F B C

 13 


¤ ១៦ ¤

១ 2
១១

- AB AC  BC 


-  
-
AB AC BC
ABC ABC
-

 

ABC ABC .
 ABC ABC
A A

B C B C
A A
ABC AB AC BC 1 1
   k
ABC AB AC BC
B C
B   B  B C
១២ BC  BC  B  B
C  C  C  C 

A A

1
B C 2 2 B C

A A
A

- B  B A B C
B C
B C
C  C A A
ABC ABC B C
B C B  C
2 A
A
3 3
B C
 C
AB  AB
B
AB AC
-  A  A
AB AC  AC  AC  AB

AC

BC
AB AC  BC 
ABC ABC BC  BC 

 14 


AC 2  BC  HC AB2  BC  HB

HA2  HB  HC

AB  AC  BC  AH

BC 2  AB 2  AC 2

1 1 1
2
 2

AC AB AH 2

b
c
h

m n
B C
H a

១ AC  BC  HC b2  an
2

AB2  BC  HB c2  am
២ HA  HB  HC h2  mn
2

៣ AB  AC  BC  AH bc  ah
៤ BC  AB  AC a 2  b2  c 2
2 2 2

1 1 1 1 1 1
៥ 2
 2
  2 2
AC AB AH 2 2
b c h

 15 


∕ ១៧ ∕

១ ២

n  n  2 180o
A I
B N ២
K
E J
C
D M L
360o

- 3
- 4 ៤

- 5
- 6 
- 7
- 8  n
- 9
 n  2  180o
- 10 n
- 12
-.......................................................

 16 


១៨

១ ២.

១១ ២១

២២
1
SL   p  a
2
( p: a:

S L   Ra ( R a
-

ST   Ra   R2   R  a  R 

S  4 R2 ( R

១២

1
- V  SB h
3
SB h

1
- V   R2h
3
R h

4
- V   R3
3
R

 17 
sYsþI¡ elakGñkmitþGñkGanCaTIRsLaj;rab;Gan enAkñúgEpñkenHelakGñknwg)aneXIj BIkarEkRKb;
lMhat;RbtibtþiénRKb;emeron EdlmanenAkñúgesovePABum<fñak;TI9 rbs;RksYgGb;rM yuvCn nigkILa e)aHBum<elIk
TI1 qñaM 2011 . lkçN³ enAkñúgEpñkenH KWmanTaMgcemøIylMhat;Rbtibtþi nigmanTaMgRbFanlMhat;
dUcenHelakGñkGacebIkemIlRbFanlMhat; enAkñúgesovePAenH edaypÁÚpÁgelxTMB½rrbs;lMhat;enaH CamYy
elxTMB½rEdlmanP¢ab;BIxagmux énkMENlMhat;RbtibtþinImYy². ral;dMeNaHRsayénlMhat; EtgEteFVIkar
bkRsay[RsbeTAnwgvtßúbMNgénemeronRtg;cMNucenaH .
RbsinebIelakGñkmanbBaða b¤cm¶l;Rtg;cMNucNa EdlmanenAkñúgEpñkenH elakGñkGacTak;Tg;eTAkan;
RKU b¤mitþPkþirbs;GñkEdlmansmtßPaB b¤GñkeroberogesovePAenHkñúgeBlevlasmrmü .

ii


œ ១ œ

25
- 03 : ច 0.04 , 0.01 , 5 ,  1 , , 0.1 ។
36
- 05 : ច 0.008 ,  0.001 , 0.08 ,  0.4 ។
- 06 : 10  0.1 , 3
10  3 0.1 ។
3 3
7 5 10 100
- 07 : ច . ខ. . ឃ. ។
3 3
28 0.2 1 000 0.1

2 81 2
- 08 : ច . ខ. 3 . 12 ឃ. 3
81 ង. 50 ច. ។
8 3 25
1 1 2 2 3 121
- 09 : ច ង . 3 ខ. 3 . 53 ឃ. ។
3 3 25 11 7
- 11 : 5 50  8 32 , 48 x  27 x , 5P  3 45P3  P  0 ,
12 x  12  27 x  27 , 72a  3 98a ។

1 3 2 2 3 3 3
3
- 12 : , , , ។
5 2 12  18 3
53 4 3
43 2

‹‹Â……

 18 


 ២ 

- 19 : ច ច 240 000mile ។ ង

186 000 mile ង ង ច ។

- 21 : ច 1 800 $ 3% ។

- 23 : . ង ង 1 4 800 4 200 ។ ង ងច

ខ. 800 ។ ង 10%។

ច ។

- 25 : ខ 500 000 605 000 ។


ង ។

‹‹Â……

 19 


 ៣ 

- 30 : ច

A  2  x  2    x 2  x  3   2 x  1 x  2 
2

B   x  1  x  2   x  x  1 ។
2 2

- 31 : ច ង

A  6x2  x 1  2x  x  1
B  3  x  1 x  2    x 2  x   x  2  ។
- 33 : ង

A  x2  x 1  4x  x 1  4x  4
B   3x  6  4 x2  1   2 x  4   4 x2  4 x  1 ។
- 34 : ច ង

x 2  4x  5 , x 2  2 x  15 , x 2  11x  30
x 2  2x  8 , x 2  5 x  14 , 2 x 2  5x  3 ។
- 35 : ច x2  6 x  7 , x2  5x  6 ។

1 2 x 2  x 3x  2 2 x 2  x
- 36 : ច 1  ,   ។
x x 1 x 2  x x  1 x x  1
x2 x2  b a2 a3  a 2
- 37 : ច A 2  , B 2  ។
x  25 x 2  20 x  25 a b a  a  2   a  2

‹‹Â……

 20 


o ៤ 1 o

- 43 : ច

. 6x  2  3x  8

ខ. 3x  5  3x  2

. 6 x  4  2  6x  1 ។

3x x x  7 2x  8
- 44 : ច  9   15 ,   4 ។
4 2 6 2
- 45 : ច ង ។

x2  2 x  4 x  8  0 ,  2 x 1   x  3 0 , 3 x 2  9 x  0 , 2 x 2  3x  5  0 ។
2 2

- 47 : . 8 ច ង ។ ង ង 38 ។

ច ។

ខ. 7 ង ច ង 27 ង ច ។ច ច ។

‹‹Â……

 21 


r ៥ 1 s

- 52 : a ង b ច ។ច ab a 2  b2 ។

2x  3  5 x  5 ,
x
- 55 : ច 27 , 0.37x  0.17  3.5 ។
4
 3x 2 4 x  3
2 x  7  6 x  5  4  3  12
- 56 : ច .  ខ. 
 ។
 4 x  11  4  x  2 x  1  3x  4
 2
- 58 : . ច ង 5m ង x m។ x ខ ច -

ង ង 18m ង ច ង 50m ។
2

ខ. ច ច ង 1918 ង 1914 ។ ច ច ង ។

‹‹Â……

 22 


 ៦ 

- 64 : ង ច ង 20 ។
1 2 0 0 1 2 2 1 0 0
4 0 1 1 3 2 1 3 0 1
ច ង ។

- 67 : x ច f ច ច ង ង ង ង ង ង ។

x 0 1 2 3 4
f 4 16 18 6 6

. ច 2 ង ច ។

ខ. ច 2 ង ច។

- 71 : ង 200 kg ។

30-40 40-50 50-60 60-70


ង 35 45 55 65

. ង ង ង ង ។

ខ. ង ង ។

. ច 50 kg ?

‹‹Â……

 23 


 ៧ 

- 77 : ច ង ង ។

x 0 1 2 3 4
y 3 6 5 2 2
- 78 : ច ។

200 300 350 700 840 950


ង 6 2 2 1 1 1

- 79 : ច ង ង ។
10 8 13 12 7 ។

- 82 : ច ង

21-28 28-35 35-42 42-49 49-56 56-63 63-70


ង 3 7 12 15 12 7 3

‹‹Â……

 24 


 ៨ 

- 89 : ង ឃ 4 ងឃ 2។
. ឃ ងច ឃ ។

. ឃ ឃ ច ង ខ ឃ

ច ង។ ច ឃ ។

- 90 : ច 250 ។ ង ខ ច 50
។ ។

- 92 : ង។

.ច ច ខ ច ។

.ច ង ង 10 ។

‹‹Â……

 25 


 ៩ 

- 99 : ច ច ង ច ច

. A  0 , 0 ង B 8 , 15 ខ. A  2 , 6 ង B  3 ,  6
. A  3 ,  5 ង B  1 ,  8 ឃ. A5 , 3 ង B 11 , 11 ។

‹‹Â……

 26 


 ១០ 

x
- 108 : .ច ង y ។
2
1
.ច ង ច ច A 0 , 1 ង ង  ។
3
.ច ច ច A  2 , 0 ង B  1 , 4 ។
x  4
- 110 : ច ង y , y2 ង x4 ង ង ច
2
ខ ង ។

- 111 : ច ច ច  2 , 7 ,  2 , 7 ។

- 113 : ច ច ច ង ។

. ច ច  0 , 0 ង y  3x  1 ។
ខ. ច ច  6 , 3 ង x  3y  9 ។
- 115 : ច ច ច ង ង ។

. A  2 ,  1 ង ង 2 y  x 1  0 ។
 1 
ខ. A , 0 ង ង y  2x ។
 2 
- 117 : ង 12 ង 40o ។ ង ច ង 2o ង

ង។ ង ង 26o ។

‹‹Â……

 27 


 ១១ 

4 x  3 y  4
- 123 : ច  ។
3x  y  2
 y  6  3x 1 3 x  4 y  8 1
- 125 : ច .  ខ.  ។
9 x  2 y  3 2 4 x  y  17  2
 4x  3 y  9 7 x  3  y  3  5  x  y 
- 128 : ច .  ខ.  ។
3x  5 y  7  7  x  1  6 y  5  x  y 
- 131 : . ច ង 50 3 ងច 1 ង2 ង ច 2 ង 60 ។

ច ច ង ។

ខ. ងច ង ង 158m ។ ង ង 5m ច
ង ង ច ង។

‹‹Â……

 28 


 ១២ 

- 138 : ច x cm ង ។ 1
x
1

1
- 139 : ច ង ង ង ង ង 2 , 2 ,1 m ។

- 140 : ង 8m ង ង ។
8m
ង ង 2m ច ង ។

2m
‹‹Â……

 29 


 ១៣ 
B

- ងង ង AB  25 cm ង BQ  6.5 cm Q C
145 : O
P o
ច ង ។ច ង AP ង BC ។
A
M

- 147 : ង ង O ងង A
OA  OB  5.5cm ង MN  9cm ។  o
N
ច ង NP ។ B
P

C

- 148 : ងង O ង ង 2cm ។ A
o
ច ច ងង។ ច ច A ង ងង 

ងច ច B ង O ង ងង
 
l
ង C។ ង AB  OB ។ ច AC ។ A B

- 150 : ST ង RT ង ងង O ST  RT  20 cm ។ TQO
ង TQ  10 cm ច ងង OQ ។ B
P

- 152 : ង ង ABC A  R
o
A។ ច BR  RC ។

Q
C

- 153 : ងង ង r  5cm , r '  3cm ។ OO ' B


 A 
r  O
ងង ង ង A ង B ។ច ង AB ។ O r

- 155 : ងង ខ O ង O ' ។ ងង ង
ច ង ង ង 4cm ង ច ង ង ង 1cm ច ង ។

ច ងង ង ។

1 cm
   
O O O O
4 cm

‹‹Â……

3  30 


( ១៤ )

- 160 : ងង O ង B ច ច AC ។ ច AOB  COB ។


P

- 161: ង ង ងង

C D
B

- 162: ABC ង ងង O ច ច
A
O ABC ។ o

1
ច ABC  AC ង ។ C
2 D
B

- 164: ង ង ង AC ង BD ង I។ C
A 
I
ACB  34o ង CAD  45o ។
ច AIB ។
D
D

E
o
- 167: ងង O AOB  60o ។ ច 
60 o C
ACB , ADB ង AEB ច ង ។
A
B
D

- 169 : ង ងង O ង ង z y
A 
o C
AC AD  DC ។ ច x, y , z។ x

92o
112o
- 171: ង ច a ង b ។
b
a

B
- 173: ច ABCD ង ងង A

AB  BC ។ CD E 76o
ACE  27o ង ADE  76o ។ ច E
o
27 D
C
. ACB ខ. BAD ។

 31 

A

- 174: CMD ង ងង O ច ង ។
60o B
ង CAD  60o ង ADB  75 ។ o
o M
75o

ច CMD ។ D
C

- 175: ច D ង ង ។ 260o
o
S 100o
D

- 178: ងង O ង SET D
ងង ង E។ ង DE  DF ង
FET  40o ។ ច o

. EOF F
40o
ខ. DES ។
S E T

‹‹Â……

 32 


2 ១៥ 2

AP 2
- 182 : P ច ច ង AB ។ ច ង P  ។
AB 5

- 184 : 1 ,  2 ,  3 ,  4 ។ 1 1 1.6

ច x ង y ។ 2 x
1.5

y 3
2
4

- 186 : ច ង A
ង ។

ង 1m B
ង 1m
C
ចង ង ចង ។ F E
ច ។

- 188 : ង ABC  DE  / /  AB  ។ច ង ង ។

AD DC BE EC C
k> 3 2 6 …
x> 7 3 … 5 D E
K> 2 … 3 4

AD DC AC BE EC BC
X> … 8 20 … … 15
A B

g> 12 … 30 9 … …

‹‹Â……

 33 


 ១៦ 
A
- 193 : ABC ង ABC
A y
1 x
ច ច ង ។ 3 4
4
C
ច x, y,z ។ B 7
B z C

- 195 : ABC ង PMN A  80o , B  54o , P  54o ង M  46o ។


.ច ABC ច PMN ។
ខ. ច ច។
A

- 197 : ច ង ង AB ង

CBA ច CDE ។ B 24 m C 32 m D

40 m

- 200 : x ង y ។ E
G
A
3m x
AE  4cm
AD  10cm
I H
E 6 cm B K L
3m 2m
D y C
J

x D 120m B 100m
Q
QR  16cm V
A
RT  10cm 40m
x
R C
S 20cm T
E

- 207 : ង ង x ង y ។ច ង x

z z  x2  y 2 ង ង ង y
x, y ង z ង ។

‹‹Â……

3
 34 


 ១៧ 

- 214 : 8ច ច ង ។ច ច ច ង ង

ច ង ង ង ង។

- 217 : ច ច ង ង ង 2 700 , 3 420o ។


o

- 219 : ច  ង ង ។


2
2

2
12o
2
3

- 220 : ច ង ងច ច

‹‹Â……

 35 


 ១៨ 

- 226 : ច ង ង ង ង ។

10 cm
13 cm
5 cm

55cm
cm
5 cm 2 cm

a) b) c)

- 228 : 28cm ង ង 13cm ។ ច dm3 ។


- 231 : ច ង ច ង ។

12 mm 3 mm
 
o o

a) b)
A
- 232 :  DE  / /  BC  , AD  3cm , AB  5cm ។
ង ADE  6 cm 2 ។
D E
.ច ABC ។
ខ. ច ច DECB ។
B C

‹‹Â……

 36 


œ ១ œ

-TMB½rTI 3 ³ rkb£skaeréncMnYn ³ -TMB½rTI 5 ³ rkb£sKUbéncMnYn ³


k> b£skaerén 0.04 ³ k> b£sKUbén 0.008 ³
eday 0.04  0.2 2
eday 0.2  0.008 3

naM[ 0.04  0.2 naM[ 0.008  0.2


3

dUcenH b£skaerén 0.04 KW 0.2 . dUcenH b£sKUbén 0.008KW 0.2 .


x> b£skaerén 0.01 ³ x> b£sKUbén  0.001 ³
eday 0.01  0.1 2
eday  0.1  0.001 3

naM[ 0.01  0.1 naM[  0.00  0.1


3

dUcenH b£skaerén 0.01 KW 0.1 . dUcenH b£sKUbén  0.001KW  0.1 .


K> b£skaerén 5 ³ K> b£sKUbén 0.08 ³
KµancMnYnsniTanNaEdlkaerrbs;vaesµInwg 5 eT KµancMnYnsniTanNa EdlKUbrbs;vaesµInwg 0.08 eT
naM[ b£skaerrbs;vaCacMnYnGsniTan dUcenH b£sKUbén 0.08 KW 0.08 . 3

dUcenH 5 Cab£skaerén 5 . X> b£sKUbén  0.4


X> b£skaerén  1 ³ KµancMnYnsniTanNa EdlKUbrbs;vaesµInwg  0.4 eT
KµancMnYnsniTanNa b¤GsniTanNa Edlkaerrbs;va dUcenH b£sKUbén  0.4 KW  0.4   0.4 . 3 3

GviC¢maneT naM[vaKµanb£skaereT -TMB½rTI 6 ³ KNnaplKuN


dUcenH cMnYn  1 Kµanb£skaerCacMnYnBiteT . k> 10  0.1  10  0.1  1  1
g> b£skaerén 25
36
³ x> 10  0.1  10  0.1  1  1
3 3 3 3

eday 25  5 
 
2
-TMB½rTI 7 ³ KNnaplEck
36  6  2

k> 7  287  14   12   12
naM[ 25 5

36 6
28  

x>
3
5 5
3  3
25
dUcenH b£skaerén 25
36
KW 5
6
. 3
0.2 0.2
2
1
c> b£skaerén 0.1 ³ K> 10

10

1
   
1
1 000 1 000 100  10  10
KµancMnYnsniTanNa Edlkaerrbs;vaesµInwg 0.1 eT
X>
3
 1 000  3 10  10
100 3 100 3
naM[ b£skaerrbs;vaCacMnYnGsniTan 3
0.1

0.1
3

dUcenH 0.1 Cab£skaerén 0.1 .


 37  -


-TMB½rTI 8 ³ beBa©jmYycMnYnBIr:aDIkal; -TMB½rTI 12 ³ sRmÜl


1
2
1 
5 2 
k> 2

1
   
1
5 2

 5 2 5 2  
8 4 2 2
5 2
x> 81 3
 27  3 33 

3
3
3 52
5 2
K> 12  4  3  22  3  2 3 
3
X> 3
81  3 27  3  3 33  3  33 3 3 2 2 3 3 2 2 3

12  18 2 3  3 2
g> 50  25  2  52  2  5 2

3 2  2 3 2 3 3 2 
c> 2

1 1
2    2 
1
2
2
2

. 2 3  3 2 2 3 3 2 
25 25 5 5 5

3 2  2 3  2

-TMB½rTI 9 ³ beBa©ÚlmYycMnYnkñgú r:aDIkal; 2 3   3 2  2 2

18  12 6  12
k> 3
1
3
 3
1
3
¬RbEhlRbFanKW 3 13 minEmn 3
1
3
¦ 
12  18
30  12 6
1
2

x> 1
3    3
1
3 
1
6
3 3 9 3
 52 6
K> 5 3
2 3 3 3 2 3
25
 5 
25
 125 
2 3
25
 5  2  3 10 ¬eRBaHrUbmnþ a  ba  b  a 2
 b 2 nig

X> 2 3 121 3  2  3 121 3


    
8
3


121 3 8

a  b b  a   a  b 2  a 2  2ab  b 2 .
12111 7

 11 

11 7 7 77
3 3 3 52  3 5  4  3 4 2
-TMB½rTI 11 ³ sRmÜl 3
5 3 4

3
5 3 4  5
3 2
 3 5  4  3 42 
5 50  8 32  5 25  2  8 16  2


3 3 25  3 20  3 16 
 25 2  32 2   7 2  5   4
3
3
3
3

 33 25  33 20  63 2
48x  27 x  16  3x  9  3x
 4 3x  3 3x  3x ¬eRBaHrUbmnþ a  ba 2
 ab  b 2  a 3  b 3  ¦
5P  3 45 P 3  5P  3 9 P 2  5P , P  0 3
3 3

3 3 42  3 4  2  3 22 
 5P  9 P 5P  1  9 P  5P 3
4 3 2

 3
4 3 2  3
42  3 4  2  3 22 
12 x  12  27 x  27  43x  3  93x  3 
3
 
3 3 16  3 8  3 4

 2 3x  3  3 3x  3
 4   2
3
3
3
3

3
3 2 2  2  4 
3 3
 5 3x  3 
42
72a  3 98a  36  2a  3 49  2a 2 6  23 3  3 12
3

 6 2a  21 2a   15 2a 6

 38  -


a ២ a

-TMB½rTI 19 ³ rkry³eBlEdlTTYl)ansBaØaBIPBRBHcnÞ x> rkcMnYnsisSénqñaMenH


tag t Cary³eBlEdlTTYlsBaØaBIPBRBHcnÞ tag n CacMnYnsisSsrubbnÞab;BI)anekIneLIg
tambRmab; eyIg)an ³ bRmab; ³ cMnYnsisSman 800 nak; ehIyekIneLIg 10%
-cm¶ay 186 000 mile eRbIry³eBl 1 s naM[ PaKryénkMeNInKW
- cm¶ay 240 000 mile eRbIry³eBl t s  n  800
100%  10%
800
naM[ 186 000 1
 n  800
240 000 t  10
8
240 000
 t  1.29 s n  800  80
186 000
n  880
dUcenH edIm,ITTYlsBaØaBIPBRBHcnÞeKRtUv dUcenH cMnYnsisSqñaMenHKWman 880 nak; .
eRbIeBl t  1.29 s  1.3 s .
-TMB½rTI 25 ³ rkGRtakarR)ak;kñúgmYyqñaM
-TMB½rTI 21 ³ KNnatémødIEdlRtUvlk; edayeRbIrUbmnþ P  P 1 r 
o
t

tag x CatémødIEdlRtUvlk; ¬KitCa $ ¦ bRmab; ³ R)ak;edIm P  500 000 `


o

tambRmab; R)ak;kéRm 1 800$ RtUvCa 3% énéfødI R)ak;srub P  605 000 `


naM[ 1800$  3%  x ry³eBl t  2 qñaM
3
x  1 800 eyIg)an 605 000  500 000 1  r  2

100
1 800  100 1  r 2  605 000
 x 500 000
3
 60 000 $ 1  r 2  1.21
1  r  1.21 , r  0
dUcenH témødIEdlRtUvlk;KW 60 000$ . 1  r  1.1
r  0.1
-TMB½rTI 23 ³ k> KNnaPaKryéneRbgsaMg)ancuHéfø r  10%
bRmab; ³ eKbBa©úHtémøeRbgsaMgBI 4 800` mk 4 200` epÞógpÞat; ³ 500 000 1  0.1
2

naM[ PaKryéntMhyKW 4 800  4 200  500 0001.1


2
100 %
4 800  500 000  1.21
 0.125 100%
 12.5%
 605 000 Bit
dUcenH \LÚveRbgsaMg)anbBa©úHtémø 12.5% . dUcenH GRtakarR)ak;kñúgmYyqñaMKW r  10% .

 39  -


k ៣ k

-TMB½rTI 30 ³ KNna -TMB½rTI 34 ³ dak;kenSamxageRkamCaplKuNénktþa


 
A  2 x  2   x 2  x  3   2 x  1 x  2 
2
-cMeBaH x 2  4x  5 tamtaragplKuNExVg
  x  2 2 x  x  3   2 x  1 x  2 
2
x 5
  x  2  2 x  2 x  6   2 x  1 x  2 
2
eday x 1
  x  2  2 x  2 x  6  2 x  4 x  x  2
2 2 x  5 x  4 x
  x  2  5 x  4  dUcenH x 2  4 x  5  x  5x  1 .
 5 x  4 x  10 x  8
2
-cMeBaH x 2  2 x  15 tamtaragplKuNExVg
  5 x  14 x  8
2
x 3

B  x  1 x  2  xx  1
2 2
eday x 5
5 x  3x  2 x
 
 x 2  2 x  1 x  2  x x 2  2 x  1  
 x  2x  2x  4x  x  2  x  2x 2  x
3 2 2 3 dUcenH x 2  2 x  15  x  3x  5 .
  2x 2  4x  2 -cMeBaH x 2  11x  30 tamtaragplKuNExVg
x 6
-TMB½rTI 31 ³ dak;kenSamxageRkamCaplKuNktþarYm x 5
A  6 x 2  x  1  2 x x  1
eday  5 x  6 x  11 x

  x  1 6 x 2  2 x  dUcenH x 2  11x  30  x  6x  5 .
 2 x x  13 x  1
-cMeBaH x 2  2x  8 tamtaragplKuNExVg
B  3x  1x  2  x  x x  2  2
 x 4
 3x  1x  2  xx  1x  2
 x  1x  23  x  eday x 2
2 x  4 x  2 x

-TMB½rTI 33 ³ dak;kenSamxageRkamCaplKuNénktþa dUcenH x 2  2 x  8  x  4x  2 .


A  x 2  x  1  4 x x  1  4 x  4 -cMeBaH x 2  5 x  14 tamtaragplKuNExVg
 x 2  x  1  4 x x  1  4 x  1 x 2

  x  1 x  4 x  4 2
 eday x 7
  x  1 x  2  7 x  2x  5x
2

  x  1 x  2  x  2  dUcenH x 2  5 x  14  x  2x  7  .


 
B  3x  6 4 x 2  1  2 x  4 4 x 2  4 x  1   -cMeBaH 2 x 2  5x  3 tamtaragplKuNExVg
 3 x  22 x  12 x  1  2 x  22 x  1 x 1
2

 x  22 x  132 x  1  22 x  1 eday 2x 3


 x  22 x  16 x  3  4 x  2 3x  2 x  5 x
  x  22 x  12 x  5 dUcenH 2 x 2  5 x  3  x  12 x  3 .

 40  -


-TMB½rTI 35 ³ dak;kenSamCaplKuNénktþa -TMB½rTI 37 ³ KNna


x 2  6x  7  x 2  6x  9  9  7 x2 x2  b
A 
 x  3  16 x 2  25 x 2  20 x  25
2

 x  3  4x  3  4 x2 x 2  20 x  25
 
 x  1x  7 
x  5x  5 x2  b



x 2 x 2  20 x  25
eRBaH 16  4 nig a  ba  b  a
2 2
 b2 . 
x  5x  5 x 2  b 
dUcenH KNna)an A  xx5xx205xx 25 b 
25 25 2 2
x 2  5x  6  x 2  5x   6
4 4 2

2
 5 1
x   a2 a3  a2
 2 4 B 2 
a b aa  2  a  2 
 5 1  5 1
  x    x   
 2 2  2 2 
a  2  a 2 a  1
 x  3x  2 a 2b a  2a  1
1

-TMB½rTI 36 ³ KNna b
a  2  0 a  2
1
1

2
manPaKEbgrYmKW xx  1 EdlktþasRmÜl  2
b¤ a  0
x x 1 a  0 
a  1  0 a  1
xx  1  x  1  2 x 

xx  1
x 2  x  x  1  2x dUcenH KNna)an B  b1 .

xx  1
x2 1

xx  1

x 2  x 3x  2 2 x 2  x
 
x 2  x x  1 x x  1
PaKEbgrYm xx  1



x 2  x  x3 x  2   2 x 2  x 
x x  1
x 2  x  3x 2  2 x  2 x 2  x

x x  1
2x 2  2x

x x  1
2 x x  1 x  0 x  0
 ,   
x x  1 x 1  0  x  1
 2

 41  -


o ៤ o

-TMB½rTI 43 ³ edaHRsaysmIkar -TMB½rTI 45 ³ edaHRsaysmIkarxageRkam ³


k> 6x  2  3x  8 x 2  2x  4x  8  0
2 x  2    x  8  x x  2   4 x  2   0
2x  4  x  8 x  2x  4  0
2 x  x  8  4 x  2  0

x  4 x  4  0
 x  2
dUcenH smIkarmanb£s x  4 . x  4

x> 3x  5  3x  2
dUcenH smIkarmanb£s x  2 , x  4 .
3x  5  3x  6
3x  3x  6  5 2 x  12  x  32  0
0x  1 2 x  1  x  32 x  1  x  3  0
x  43x  2  0
Kµantémø x NaEdleFVI[smIkarepÞógpÞat;  x40

dUcenH smIkarKµanb£s . 3 x  2  0
K> 6 x  4  2  6x  1  x4
 x  2 / 3
6x  4  2  6x  6 
6x  4  6x  4
dUcenH smIkarmanb£s x  4 , x  2 / 3 .
6 x  6 x  4  4
0x  0 3x 2  9 x  0
3 x  x  3  0
mantémøén x eRcInrab;minGs;EdlepÞógpÞat;smIkar
 x0
dUcenH smIkarmanb£seRcInrab;minGs; . 
x  3  0
-TMB½rTI 44 ³ edaHRsaysmIkar  x0
 x  3
-cMeBaH 34x  9  2x  15 ¬manPaKEbgrYmKW 4¦ 

3x  36  2 x  60 dUcenH smIkarmanb£s x  0 , x  3 .
3x  2 x  60  36 2 x 2  3x  5  0
x  24 2 x 2  2 x  5x  5  0
dUcenH smIkarmanb£s x  24 . 2 x x  1  5 x  1  0
x  12 x  5  0
-cMeBaH x 6 7  2x2 8  4 ¬manPaKEbgrYmKW 6¦
 x 1  0
x  7  32 x  8  24 
2 x  5  0
x  7  6 x  24  24
7 x  7
x 1
 x  5 / 2
x  1 
dUcenH smIkarmanb£s x  1 . dUcenH smIkarmanb£s x  1 , x  5 / 2 .
 42  -


-TMB½rTI 47 ³
k> rkGayurbs;bUNa
tag x CaGayurbs;bUNa ¬KitCaqñaM¦
naM[ Gayurbs; m:arI KW x  8 ¬KitCaqñaM¦
bRmab; ³ plbUkGayuGñkTaMgBIresµInwg 38 qñaM
eyIg)an ³ x  x  8  38
2 x  8  38
2 x  30
x  15
epÞógpÞat; ³ 15  15  8  38
Bit
38  38

dUcenH Gayurbs;bUNaKW x  15 qñaM .


x> rkcMnYnenaH
tag x CacMnYnEdlRtUvrkenaH
tambRmab;RbFan enaHeyIgsresr)ansmIkarKW ³
2 x  7  3x  27
3x  2 x  7  27
x  34
epÞógpÞat; ³ 2  34  7  3  34  27
Bit
75  75

dUcenH cMnYnEdlRtUvrkenaHKW 34 .

 43  -


r ៥ s

-TMB½rTI 52 ³ cMeBaHvismIkarTI 1 ³ eyIg)an


RsaybBa¢ak;fa ebI a  b eK)an a  b 2 2
2x  7  6x  5 cemøIyCaRkabKW
eday a nig b CacMnYnviC¢manminsUnü naM[ a  0 , b  0 2x  6x  5  7
 4 x  12   
cMeBaH a  b  a  ab i  ¬KuNnwg a ¦
2
x  3 x 3 0 x

ehIy a  b  ab  b ii ¬KuNnwg b ¦


2
cMeBaHvismIkarTI 2 ³ eyIg)an
tam i  nig ii eyIg)an ³ 4x  11  4  x cemøIyCaRkabKW
a  ab  b naM[ a  b
2 2 2 2
4 x  x  4  11  
3x  15 x 0 x
dUcenH ebI a  b enaHeK)an a  b CaR)akd .
2 2
x5
5

-TMB½rTI 55 ³ edaHRsayvismIkar cemøIyrYmCaRbsBVéncemøIyTaMgBIrKW ³


-cMeBaH x
27 cemøIyCaRkabKW    
4 x 3 0 5
x
x
5   dUcenH RbB½n§vismIkarmancemøIy 3 x  5 .
4 x 0 20 x
x  20
 3x 2 4 x  3
 4  3  12 i 
dUcenH vismIkarmancemøIy x  20 . x>  3x  4
 2x 1  ii 
-cMeBaH 2x  3  5x  5 cemøIyCaRkabKW  2
2 x  6  5x  5 cMeBaHvismIkarTI i  ³ eyIg)an
6  5  5x  2 x x

 
3x 2 4 x  3
1 0 x
  cemøIyCaRkabKW
1  3x 3 4 3 12
9x  8  4x  3
x
1
9 x  4 x  3  8
 

3 x 0 1 x
5x  5
dUcenH vismIkarmancemøIy x  1/ 3 . x 1

-cMeBaH 0.37x  0.17  3.5 cemøIyCaRkabKW cMeBaHvismIkarTI ii ³ eyIg)an


3x  4
0.37 x  3.33
 

2x 1 
2
cemøIyCaRkabKW
3.33 x x
x 0 9
4 x  2  3x  4   
0.37 x 2 0 x
x9 4 x  3x  4  2
x  2
dUcenH vismIkarmancemøIy x  9 . cemøIyrYmCaRbsBVéncemøIyTaMgBIrKW ³
-TMB½rTI 56 ³ edaHRsayRbB½n§vismIkar    
x x
7  6 x  5 1 2 0 1
k> 24xx 11  4  x 2
 dUcenH RbB½n§vismIkarmancemøIy  2  x 1 .
 44  -


-TMB½rTI 58 ³ k> rktémø x EdlRtUvyk epÞógpÞat; ³


tambRmab;RbFaneyIgcg)anRbB½n§vismIkar ³ 638  639  640  1918

2x  5  18 2 x  10  18 638  639  640  1914
  
 5 x  50  5 x  50 1917  1918

Bit
 2x  8  x  4 1 1917  1914 Bit
   
5 x  50  x  10 2 dUcenH cMnYnKt;tKñaTaMgbIEdlRtUvrkKW
naM[ cemøIyrYmKW 4  x  10 638 , 639 , 640 .
eyIgGacbkRsay cemøIyCaRkab tamvismIkarnImYy²
cMeBaHvismIkar 1 : x  4 x  x
 

0 4

cMeBaHvismIkar 2 : x  10 x
  
0 10 x

cemøIyrYmCaRbsBVéncemøIyénvismIkar 1 & 2 KW


x
   
0 4 10 x
epÞógpÞat; [ x  5 eyIg)an
25  5  18

20  18
 
Bit
 5  5  50 25  50 Bit
dUcenH vismIkarmancemøIyKW 4  x  10 .
x> rkcMnYnKt;TaMgbIenaH
tag x CacMnYnKt;TI 1
naM[ x 1 CacMnYnKt;TI 2 nig x  2 CacMnYnKt;TI 3
tambRmab;RbFan eyIg)an RbB½n§vismIkar³
 x  x  1   x  2   1918

 x   x  1   x  2   1914
3 x  3  1918
 
3 x  3  1914
3 x  1915
 
3 x  1911
3 x  1915
 
3 x  1911
 x  638.33
 
 x  637
cMnYnKt;EdlenAcenøaH 637  x  638.33 manEtmYy
Kt;KW x  638
naM[ cMnYnTI2KW x  1  639 / cMnYnTI3 x  2  640
 45  -


 ៦ 

-TMB½rTI 64 ³ sg;Rkabssr ³ cMnYnkUn cMnYnRKYsar eRbkg;fy eRbkg;eFob eRbkg;eFob


eyIgmanTinñn½ycMnYnéf¶EdlbuKÁlikmin)anbMeBj x f f f% fy f % 

kargarkñúgry³eBj 20 éf¶ dUcxageRkam ³ 0


1
4
16
50
46
8
32
100
92
1 2 0 0 1 2 2 1 0 0 2 18 30 36 60
4 0 1 1 3 2 1 3 0 1 3 6 12 12 24
4 6 6 12 12
eyIg)antaragbMENgEckeRbkg;dUcxageRkam ³
cMnYnbuKÁlik eRbkg; f eRbkg;eFob f % k> rkcMnYnRKYsarEdlmankUn 2nak;y:ageRcIn
0 6 30 tamtarageRbkg;ekIn cMnYnRKYsarEdlmankUn 2nak;
1
2
7
4
35
20 y:ageRcInKWman 38 RKYsar.
3
4
2
1
10
5
x> rkcMnYnRKYsarEdlmankUn 2nak;y:agtic
srub 20 100 tamtarageRbkg;fy cMnYnRKYsarEdlmankUn 2nak;
tamtaragbMENgEckeRbkg; eyIgsg;)anRkabssr ³ y:agticKWman 30 RKYsar.
eRbkg; f -TMB½rTI 71 ³ sßitim:assisS 200nak;énviTüal½ymYy
7
6 m:as kg 30-40 40-50 50-60 60-70
5
4
eRbkg; 35 45 55 65
3
2
k> sg;tarageRbkg;ekIn nigp©itfñak;
1
fñak;énm:as eRbkg; eRbkg;ekIn p©itfñak;
0 1 2 3 4 cMnYnbuKÁlik 30-40 35 35 35
40-50 45 80 45
-TMB½rTI 67 ³ sg;tarageRbkg;ekIn nigeRbkg;fy 50-60
60-70
55
65
135
200
55
65
eyIgmanTinñn½y Edl x CacMnYnkUn nig f CacMnYnRKYsar
x 0 1 2 3 4
f 4 16 18 6 6
x> sg;BhuekaNeRbkg;ekIn
eyIgsg;)antarageRbkg;ekIn ³ eRbkg;ekIn
cMnYnkUn cMnYnRKYsar eRbkg;ekIn eRbkg;eFob eRbkg;eFob
x f f f% ekIn f %
 200 

0 4 4 8 8 150
135 
1 16 20 32 40
100
2 18 38 36 76 80 

3 6 44 12 88 50 
35 
4 6 50 12 100
eyIgsg;)antarageRbkg;fy ³ 30 40 50 60 70
m:as

 46  -


K> rkcMnYnsisS Edlmanm:aseRkam 50 kg


tamtarageRbkg;ekIn cMnYnsisSEdlmanm:aseRkam
50 kg mancMnYn 35  45  80 nak; .

dUcenH cMnYnsisSEdlmanm:aseRkam 50 kg
man cMnYn 80 nak; .

 47  -


 ៧ 

-TMB½rTI 77 ³ KNnamFüméntaragTinñn½yxageRkam ³ -rkemdüan m ³ e

x 0 1 2 3 4
y 3 6 5 2 2
eyIgerobTinñn½ytamlMdab; 7 8 10 12 13
KNnamFüm Tinñn½ymancMnYntYsrubKW n  5
tamrUbmnþ ³ x  x f fx f f ......fx f / f  y
1 1 2 2 n nnaM[ TItaMgénemdüanKWtY  5 2 1  3
tamTinñn½yerobtamlMdab;rYc tYTI 3 RtUvnwgelx 10
1 2 n

tamtaragTinñn½yxagelIKW ³
x
0  3  1 6  2  5  3  2  4  2 dUcenH rk)anemdüanKW m  10 . e
365 2 2

0  6  10  6  8 30
  1.67
-rkm:Ut m ³ o

18 18
edaym:Ut CatémøEdlmaneRbkg;eRcInCageK .
dUcenH mFüménTinñn½yKW x  1.67 . EtTinñn½yxagelImaneRbkg;esµI1 dUcKñaTaMgGs;
-TMB½rTI 78 ³ rkemdüanénR)ak;ebovtSn_¬KitCaBan;erol¦ dUcenH Tinñny½ enHKµanm:UteT .
R)ak;ebovtSn_ 200 300 350 700 840 950
eRbkg; 6 2 2 1 1 1
-TMB½rTI 82 ³ KNnamFüm emdüan nigm:UténTinñn½y ³
fñak;
eyIgsg;)antarageRbkg;dUcxageRkam ³ 21-28 28-35 35-42 42-49 49-56 56-63 63-70

eRbkg; 3 7 12 15 12 7 3
R)ak;ebovtSn_ x eRbkg; f eRbkg;ekIn
200 6 6
xf
1200
tamtaragTinñn½y eyIgsg;)antarageRbkg;ekIn ³
300 2 8 600 fñak; eRbkg; f p©itfñak; x xf eRb>ekIn
350 2 10 700 21-28 3 24.5 73.5 3
700 1 11 700 28-35 7 31.5 220.5 10
840 1 12 840 35-42 12 38.5 462 22
950 1 13 950 42-49 15 45.5 682.5 37
srub 13 4990 49-56 12 52.5 630 49
56-63 7 59.5 416.5 56
edayTinñn½ymancMnYntYsrub n  13 CacMnYness 63-70 3 66.5 199.5 59

naM[ TItaMgén m KWtY  132 1  142  7


e
srub 59 2684.5

tamtarageRbkg; tYTI 7 RtUvnwgTwkR)ak; 300 Ban;erol -KNnamFüm x ³


dUcenH emdüanénR)ak;ebovtSn_KW m  300 Ban;erol. eyIg)an x  2684
59
.5
 45.5
e

-TMB½rTI 79 ³ rkmFüm emdüan nigm:Ut énTinñn½y dUcenH KNna)anmFüm x  45.5 .


eyIgmanTinñny½ ³ 10 8 13 12 7 -KNnaemdüan m ³ e

-rkmFüm x ³ emdüan CatémøéntYTI 592  29.5


tamTinñn½y x  10  8  135  12  7  505  10
tamtarageRbkg;ekIntYTI 30 sßitenAkñúgfñak; 42-49
dUcenH rk)anmFümKW x  10 . eyIgKNnaemdüantamGaMgETb:ULasüúg
 48  -


naM[ m  42 
49  4229.5  22  45.5
37  22
e

dUcenH KNna)anemdüan me  45 .5 .
-KNnam:Ut m ³ o

m:Ut Catémøp©itfñak;EdlmaneRbkg;eRcInCageK
fñak; 42-49 maneRbkg; 15 eRcInCageK ehIyman
p©itfñak;esµI 45.5
dUcenH KNna)an x  me  mo  45 .5 .

 49  -


 ៨ 

-TMB½rTI 89 ³ fg;mYymanXøIBN’exµA 4 nigXøIBN’s 2 -TMB½rTI 92 ³


1.rkRbU)abEdleQµaH k cab;)anXøIBN’exµA 1. rkRbU)abEdlecjelxdUcKña
-XøIsrubman 6 naM[ krNIGac  6 -RKab;LúkLak;manmux 6 enaHeKGacpÁÜbKUlT§pl
-XøIBN’exµAman 4 naM[ krNIRsb  4 rbs;va)an TaMgGs;cMnYn 6  6  36 rebob
tamrUbmnþ P  cMcMnnYnYnkrNI R sb naM[ krNIGac  36
krNIG ac
-KUEdlmanelxdUcKñaGacCa 1,1 2 , 2 3, 3 4 , 4
P XøIBN’exµA   cMnnY krNIRsb  4  2  66 .67 % 5 , 5 nig 6 , 6  man 6krNI
cMnnY krNIG ac 6 3

dUcenH RbU)ab k cab;)anXøIBN’exµAKW P  66.67% . naM[ krNIRsb  6


2. rkRbU)abEdleQµaH x cab;)anXøIBN’exµA dUcenH P elxdUcKña   cMcMnnYnYnkrNI R sb 6 1
  .
krNIG ac 36 6
edayeQµaH k cab;)anXøIBN’exµAehIymindak;cUlfg;
vij naM[b:HBal;dl;karcab;elIkeRkayrbs;Gñk x 2. rkRbU)abEdlplbUkRKab;TaMgBIresµInwg 10
-XøITaMgGs;sl;Et 5 naM[ krNIGac  5 KUénRKab;TaMgBIrmanplbUkesµI 10 GacCa 4 , 6 ,
5 , 5 , 6 , 4 manbIkrNI
-XøIBN’exµAsl;Et 3 naM[ krNIRsb  3
cMnYnkrNIR sb 3 naM[ krNIRsb  3
P XøIBN’exµA     60 %
cMnYnkrNIG ac 5 cMnYnkrNIR sb
plbUkesµI!0  
3 1
dUcenH P 
cMnYnkrNIG ac

36 12
.
dUcenH RbU)ab x cab;)anXøIBN’exµAKW P  0.6 .
-TMB½rTI 90 ³ rkRbU)abEdleFVIdMeNIredaymFü)aydéT
tag P CaRbU)abGñkeFVIdMeNIredayrfynþpÞal;xøÜn
-Rkumh‘unmanbuKÁlik 250nak; naM[ krNIGac  250
-GñkCiHrfynþpÞal;xøÜn 50nak; naM[ krNIRsb  50
naM[ P  cMcMnnYnYnkrNI R sb

50
krNIG ac 250 5
1
  20 %

tag P CaRbU)abénGñkeFVIdMeNIredaymeFüa)aydéT
Edl P CaRBwtþikarN_bMeBjKñaCamYyRBwtþikarN_ P
naM[ P  1  P  1  15  54  80%
dUcenH RbU)abénGñkeFVIdMeNIredaymeFüa)aydéT
EdlminCiHrfynþpÞal;xøÜnKW P  80% .
 50  -


 ៩ 

-TMB½rTI 99 ³ KNnacm¶ayrvagBIrcMNuc
eyIgGacKNnacm¶ayrvagBIrcMNuc edayeRbIrUbmnþ ³
AB  xB  x A 2   yB  y A 2
k> A0 , 0 & B8 ,  15 

naM[ AB  8  02   15  02


 64  225  289
 17 ÉktaRbEvg .
dUcenH KNna)an AB  17 ÉktaRbEvg .
x> A 2 , 6 & B3 ,  6
naM[ AB  3  2   6  6
2 2

 25  144  169
 13 ÉktaRbEvg .
dUcenH KNna)an AB  13 ÉktaRbEvg .
K> A 3 ,  5 & B1 ,  8
naM[ AB  1  3   8  5
2 2

 49
 13 ÉktaRbEvg .
dUcenH KNna)an AB  13 ÉktaRbEvg .
X> A5 , 3 & B11 , 11
naM[ AB  11 5  11 3
2 2

 36  64  100
 10 ÉktaRbEvg .
dUcenH KNna)an AB  10 ÉktaRbEvg .

 51  -


 ១០ 

-TMB½rTI 108 ³ !> sg;bnÞat;EdlmansmIkar y   2x -TMB½rTI 110 ³ sg;bnÞat;TaMgbIkñúgbøg;EtmYy


x4
cMeBaH x  0 naM[ 0
y 0 Edr CabnÞat;kat;Kl; -cMeBaHbnÞat; y
2
b£ 1
y  x2
2
2
G½kS . bnÞat;enHmanemKuNR)ab;Tis a   12 mann½yfa taragtémøelxén KW 1
y  x2
2
x 0 4
y 2 0

ebI x ekInBIrÉkta enaH y fycuHmYyÉkta. -cMeBaHbnÞat; y  2 CabnÞat;edk kat;G½kS  yy  Rtg; 2


eyIgsg;bnÞat;)an dUcxageRkam ³ -cMeBaHbnÞat; x  4 CabnÞat;Qr kat;G½kS xx Rtg; 4
y sg;bnÞat;TaMgbIkñúgbøg;EtmYydUcxageRkam ³
x y
y x4
2
1
0 y x2
x 2
1 
y2
 
2


0 4 x
2. sg;bnÞat;kat;tamcMNuc A0 , 1 nigmanemKuN
R)ab;TisesµI  13 KNnaRkLaépÞxNÐedaybnÞat;TaMgbIenH
eyIg)anbnÞat;kat;tam A0 , 1 ehIymanemKuNR)ab; bnÞat;TaMgbIpÁúM)anCaRtIekaNEkgEdlman )atesµI 4
ÉktaRbEvg nigkm<s;esµI 2 ÉktaRbEvg
TisesµI  13 mann½yfa ebI x ekInbIÉkta enaH y fycuH
naM[ RkLaépÞRtIekaNEkg S  4 2 2  4 ÉktaépÞ
mYyÉkta.
eyIgsg;bnÞat;)an dUcxageRkam ³ dUcenH épÞRkLaxNÐedaybnÞat;TaMgbIKW S  4 ÉktaépÞ .
y
-TMB½rTI 111 ³ rksmIkarbnÞat;kat;tamBIrcMNuc
¬RbtibtþienHerobcMmin)anl¥ ³ eK[rksmIkarbnÞat;kat;tamBIrcMNuc Et

eBl[cMNuc manEtmYycMNuceTAvij KW  2 , 7 dUcKña¦
x
0 eKGacsg;bnÞat;eRcInrab;minGs;kat;tammYycMNuc
dUcenH minGackMNt;)ansmIkarbnÞat;kat;  2 , 7 .
-TMB½rTI 113 ³ rksmIkarbnÞat;kat;tammYycMNuc ehIy
3. KNnaemKuNR)ab;TisénbnÞat;kat;tamBIrcMNuc RsbeTAnwgbnÞat;mYyepSgeTot
A 2 , 0 nig B 1 , 4
k> kat;tam 0 , 0 ehIyRsbnwgbnÞat; y  3x  1
naM[bnÞat;manemKuNR)ab;Tis tag M x , y  enAelIbnÞat;RtUvrkkat;tam A0 , 0
y y 40
 4 .
4
a 
ehIyRsbnwgbnÞat; y  3x  1 enaHemKuNR)ab;TisesµIKña
B A
x x
B A  1   2  1

 52  -

yM  y A eyIg)an yM  y A 1
eyIg)an xM  x A
3
xM  x A 2
  1

y0 y   1 1
x0
3 naM[ y  3x x2
  1
2
y  1  2x  2
dUcenH y  3x CasmIkarbnÞat;EdlRtUvrk . y  2 x  3

karsg;bnÞat;edIm,IepÞógpÞat; dUcenH smIkarbnÞat;EdlRtUvrkKW y  2 x  3 .


y  3x karsg;RkabedIm,IepÞógpÞat;
y  3x  1

2 y  x 1  0

x> kat;tam 6 , 3 ehIyRsbnwgbnÞat; x  3 y  3


y  2 x  3
tag M x , y  enAelIbnÞat;RtUvrkkat;tam B6 , 3
ehIyRsbnwgbnÞat; x  3 y  3 b¤bnÞat; y  13 x  1 x> tat;tam A 21 , 0  ehIyEkgnwgbnÞat; y  2x
enaHemKuNR)ab;TisesµIKña tag M x , y  enAelIbnÞat;RtUvrkkat;tam  A
1 
, 0

eyIg)an yx  xy  13
M B  2 
M B ehIyEkgnwgbnÞat; y  2x naM[plKuNemKuNR)ab;
y 3 1
x6 3
 TisesµI  1
y y
y  3  x  6 naM[ y  x  1
1 1 eyI g )an x x
M
 2  1
A

M A
3 3
y0
dUcenH y  13 x  1CasmIkarbnÞat;EdlRtUvrk .  1 
 2  1
x  
 2 
karsg;bnÞat;edIm,IepÞógpÞat; 1
2 y  x 
1 2
y x 1
3 1 1
y  x
2 4
1
y x 1
3
dUcenH smIkarbnÞat;EdlRtUvrkKW y   12 x  14 .

-TMB½rTI 115 ³ rksmIkarbnÞat;kat;tammYycMNuc ehIy karsg;RkabedIm,IepÞógpÞat;


EkgeTAnwgbnÞat;mYyepSgeTot y  2x
k> tat;tam A2 ,  1 ehIyEkgnwgbnÞat; 2 y  x  1  0 1
y  x
2
1
4
tag M x , y  enAelIbnÞat;RtUvrkkat;tam A2 ,  1
ehIyEkgnwgbnÞat; 2 y  x  1  0 b¤ y  12 x  12
naM[ plKuNemKuNR)ab;TisesµI  1
 53  -


-TMB½rTI 117 ³ rkeBlevlaEdlsItuNðPaBesµInwg 26 o

tagG½kS xx CaeBlevlaEdlRtUvekIneLIg nigG½kS


 yy  CasItuNðPaBEdlnwgRtUvfycuH
edaysItuNðPaBfycuHkñúgGRta 2 / h mann½yfa
o

ebIeBlekIn 1h enaH sItuNðPaBfycuH 2 o

eyIg)anbnÞat;kat;tamcMNuc 12 , 40  manemKuNR)ab;


esµInwg  2 . ebI M x , y  CacMNucmYyenAelIbnÞat;enH
enaHeyIg)anbERmbRmYlemKuNR)ab;TisKW ³
y  40
 2  y  40  2 x  24
x  12
 y  2 x  64
cMeBaHsItuNðPaB y  26 o

eyIg)an 26  2x  64
2 x  38
x  19 CaeBl 
dUcenH sItuNðPaB 26 RtUvnwgem:ag 19  7 yb; .
o

 54  -


 ១១ 

-TMB½rTI 123 ³ edaHRsayRbB½n§smIkartamRkab i 


x> 34xx  4yy178 ii 
4 x  3 y  4 i  
eyIgmanRbB½n§smIkar 
ii -eyIgedaHRsayedayeRbIviFICMnYs
3x  y  2
eyIgsg;bnÞat; i  & ii enAkñúgbøg;EtmYy tam ii : 4x  y  17  y  17  4x iii
-taragtémøelxén 4x  3 y  4 KW yx 10  24 yk iii  CMnYscUlkñúg i  eyIg)an ³
i  : 3x  417  4 x   8
x 1
-taragtémøelxén 3x  y  2 KW 0
y 1 2
3x  68  16 x  8
19 x  76
sg;RkabTaMgBIrenAkñúgbøg;EtmYy eyIg)an ³ x4

3x  y  2
yk x  4 CMnYscUlkñúg iii  eyIg)an ³
iii : y  17  4  4  1
dUcenH RbB½n§smIkarmanKUcemøIy x , y   4 , 1 .
-TMB½rTI 128 ³ edaHRsayRbB½n§smIkar
k> 43xx35yy97 12
4x  3 y  4


tamtaragbnÞat;RbsBVKñaRtg;cMNuc  2 , 4 eyIgedaHRsayedayeRbIviFIbUkbM)at; enaHeyIg)an
dUcenH RbB½n§smIkarmanKUcemøIy x  2 , y  4 .  4x  3y  9 3

 3 x  5 y  7 4
-TMB½rTI 125 ³ edaHRsayRbB½n§smIkar  12 x  9 y  27
k> 9yx62y 3x3 12 
 12 x  20 y  28
 11y  55  y5
-eyIgedaHRsaytamviFICMnYs cMeBaH y5 ykCMnYskúñg 1 eyIg)an ³
tam 1 : y  6  3x  y  3x  6 3 1 : 4x  3  5  9
yk 3 CYscUleTAkñúg 2 eyIg)an ³ 4 x  24
2 : 9 x  23 x  6   3 x6
9 x  6 x  12  3
3 x  9 dUcenH RbB½n§smIkarmanKUcemøIy x  6 , y  5 .
x  3
yk x  3 CMnYscUlkñúg 3 x> 77xx31y63y  55xx  yy 1
2

3 : y  3   3  6  15 7 x  3 y  9  5x  5 y
eyIg)an 
dUcenH RbB½n§smIkarmanKUcemøIy  xy    153 . 7 x  7  6 y  5 x  5 y
   

 55  -


7 x  3 y  9  5 x  5 y

7 x  7  6 y  5 x  5 y
9 y  2  10 y
y  2
yk y  2 CMnYnskñúg1 eyIg)an
1 : 7 x  3 2  3  5x  2
7 x  15  5 x  10
2x  5
x  5/ 2

dUcenH RbB½n§smIkarmanKUcemøIy x  5 / 2 , y  2 .
-TMB½rTI 131 ³ k> rkcMnYnTaMgBIrenaH
tag x CacMnYnTI! nig y CacMnYnTI@
tambRmab;RbFaneyIgcg)anRbB½n§smIkar
 x  y  50 1

3x  2 y  60 2
eyIgedaHRsaytamviFIbUkbM)at;

 x  y  50 2

3 x  2 y  60
2 x  2 y  100

3 x  2 y  60
5 x  160
x  32
yk x  32 cMnYnkñúg 1 edIm,Irk y eyIg)an ³
1 : 32  y  50  y  18

dUcenH cMnYnTaMgBIrenaHKW cMnYnTI!= 32 nigcMnYnTI@=18 .


x> rkRbEvgTTwg nigbeNþayénctuekaNEkg
tag x CaTTwg nig y CabeNþay 0  x  y xñatEm:t
tambRmab;RbFaneyIgcg)anRbB½n§smIkar
2x  y   158

yx5
edaHRsayedaybUkbM)at;

 x  y  79

 yx5
2 y  84  y  42
cMeBaH y  42  x  42  5  37
dUcenH ctuekaNEkgman TTwg  37m /beNþay  42m .
 56  -


 ១២ 

-TMB½rTI 138 ³ KNnargVas; x KitCa cm énrUb


tag ycm CaGIub:UetnusénRtIekaNEkg i 
-kúñg   i  ³ tamRTwsþIbTBItaK½r
1
y 2  12  12  2
x  ii 
-kñúg   ii ³ tamRTwsþIbTBItaK½r i 
1
x  y 1  2 1  3
2 2 2

1
 x  3 cm

dUcenH rgVas;KNna)anKW x  3 cm .
-TMB½rTI 139 ³ KNnargVas;Ggát;RTUgénRbelBIEb:tEkg
tag y CaRbEvgGgát;RTUgEdlRtUvrk KitCa m
cMeBaH   ABC ³ RTwsþIbTBItaK½r
x 2  22  12  5
cMeBaH   ACD ³ D
y
tamRTwsþIbTBItaK½r 2
x C
1
A
y 2  x 2  22  5  4  9 2

naM[ y  9 3m

dUcenH RbEvgGgát;RtUvKNna)anKW 3 m .
-TMB½rTI 140 ³ rkkm<s;CBa¢aMgenaH
tag h Cakm<s;CBa¢aMg KitCa m
edayCeNþIr CBa¢aMg nigépÞrabpÁúMKña)anCaRtIekaNEkg
tamRTwsþIbTBItaK½r
eyIg)an
82  h 2  2 2  h 2  64  4
 h  60  2 15 m

dUcenH km<s;CeNþIrKW h  2 15 m  7.75 m .

 57  -


 ១៣ 

-TMB½rTI 145 ³ KNnaRbEvg AP nig BC -TMB½rTI 150 ³ KNnakaMénrgVg; OQ S


eK[ AB  25 cm B
eyIgman ST  RT  20 cm 20 cm
r

BQ  6.5 cm P o
Q C
ehIy TQ  10 cm T 
r
10 cm
Q
o

dUcrUbxagsþaM tagkaMrgVg; OS  OQ  r 20 cm
R
A
kñúgRtIekaNEkg TSO man OT CaGIub:Uetnus
-eday AB  OP Rtg; P naM[ P kNþal AB tamRTwsþIbTBItaK½r eyIg)an ³
OT 2  OS 2  ST 2
enaH AP  AB 
25 cm
 12.5 cm
2 2 OQ  TQ2  OS 2  ST 2
-eday BC  OQ Rtg; Q naM[ Q kNþal BC r  102  r 2  202
enaH BC  2BQ  2  6.5 cm  13 cm r 2  20r  100  r 2  400
20r  300
dUcenH KNna)an AP  12.5 cm nig BC  13 cm . r  15

-TMB½rTI 147 ³ rkRbEvg NP dUcenH kaMrgVg;KNna)anKW OQ  r  15 cm .


OA Cacm¶ayBIp©it O eTAGgát; MN
OB Cacm¶ayBIp©it O eTAGgát; NP M -TMB½rTI 152 ³ RsaybMPøWfa BR  RC
eyIgman ABC CaRtIekaNsm)at
B

eday OA  OB  5.5 cm A
 o
P

naM[ MN  NP  9 cm . N naM[ AB  AC A 
o
R
B
P
tamRTwsþIbTbnÞat;b:HEdl 

dUcenH rk)anRbEvg NP  9 cm . KUsecjBIcMNucrYmenAeRkArgVg;


Q
C

-TMB½rTI 148 ³ KNnargVas; AC C


eyIg)an AP  AQ , BP  BR , CQ  CR
eyIgmanrgVg;p©ti O kaM 2 cm o 2 cm

cMeBaH AB  AC enaH AP  BP  AQ  CQ

A CacMNucenAeRkArgVg; 2 cm eday AP  AQ eyIg)an BP  CQ


  BP  BR
B CacMNucb:HrgVg;
A 2 cm B l

tamTMnak;TMng BP  CQ  BR  CR .
AB  OB  OC  2 cm ¬eRBaH OB CakaMrgVg;¦ CQ  CR
kñúg   ABOman OA CaGIub:Uetnus tamRTwsþIbTBItaK½r
dUcenH eyIgbMPøW)anfa BR  RC .
OA  AB2  OB2  22  22  2 2 cm
naM[ AC  OA  OC  2 2  2  4.83 cm -TMB½rTI 153 ³ KNnaRbEvg AB ¬KitCa cm ¦
dUcenH RbEvgKNna)anKW AC  4.83 cm . eyIgman r  5 cm , r   3 cm
eyIgKNnaRbEvg AB tamkrNIdUcxageRkam ³
 58  -


-krNIrgVg;KµancMNucrYm R  r  4

eyIg)an ³ R  r 1
2R  5
OO  AB  r  r 

A B
 R  2.5 cm
 AB  OO  r  r  O r r O

 OO  5  3 cMeBaH R  2.5


 OO  8 enaH r  4  R  4  2.5  1.5 cm
-krNIrgVg;b:HKñaxageRkA A B
dUcenH KNna)an kaMrgVg;TaMgBIrKW
naM[ A RtYtelI B O
 r 
r O
2.5 cm , 1.5 cm .
enaH OO  r  r nig AB  0
-krNIrgVg;kat;Kña)anBIrcMNuc
eyIg)an AB  2r 
B A
r r O
O
b¤ AB  6 cm
-krNIrgVg;b:HKñaxagkñúg
eyIg)an AB  2r B
 
Or Or
A

b¤ AB  6 cm
-krNIrgVg;KµancMNucrYmenAkñúgKña
eyIg)an AB  r  r 
r B A

b£ AB  2 cm . O O r

-TMB½rTI 155 ³ KNnakaMénrgVg;TaMgBIr


tag R CakaMrgVg;FM nig r CakaMrgVg;tUc
-cMeBaHcm¶ayrvagp©itTaMgBIresµI 4 cm

 
O O
4 cm

krNIenH eyIg)an R  r  4 1


-cMeBaHcm¶ayrvagp©itTaMgBIresµI 1 cm
1 cm
 
O O

krNIenH eyIg)an R  r  1 2


tam 1 nig 2 eyIg)anRbB½n§smIkar
 59  -


( ១៤ )

-TMB½rTI 160 ³ bgðajfa AOB  COB -TMB½rTI 164 ³ KNna AIB B

eyIgman rgVg;p©it o nig eyIgmanmMu ACB  34 nig o


A 
I
C

B CacMNuckNþalFñÚ AC CAD  45 .
C B o

mMup©it AOB maFñÚsáat; AB O


 A kñúgRtIekaN ADI man ³ D

mMup©it COB maFñÚsáat; BC CAD  ADB  AIB¬plbUkmMukñúg@esµImMueRkA!¦

eday B CacMNuckNþalFñÚ AC Et ACB  ADB  34 ¬manFñÚsáat;rYm AB ¦


o

naM[ FñÚ AB  FñÚ BC naM[ CAD  ACB  AIB


45o  34o  AIB
eyIg)an mMup©it AOB  COB Edr
79o  AIB
¬mMup©itEdlmanFñÚsáat;esµIKña vaCamMub:unKña¦
dUcenH KNna)an AIB  79 . o

dUcenH eyIgbgðaj)anfa AOB  COB .


-TMB½rTI 167 ³ KNna ACB , ADB , AEB
-TMB½rTI 161 ³ rkcMnYnmMucarwkkñúgrgVg; P
eyIgman AOB  60 naM[ o

tamry³rUbxagsþaM mMucarwkkñúgrgVg; B
ACB  ADB
D

mancMnYnbImMu rYmman ³  AEB E


o
AOB 
C D  60 o C
BPC , BPD , CPD . 2
o A
60
  30o B

-TMB½rTI 162 ³ bgðajfa ABC  12 AC 2


¬mMucarwkkñúgrgVg; nigmMup©itmanFñÚsáat;rYm AB ¦
eyIgman ABC CamMucarwk B

kñúgrgVg;p©it o . dUcenH ACB  ADB  AEB  30o .


A o
tamlkçN³én]TahrN_1 -TMB½rTI 169³ KNnargVas;mMu x, y,z
D

eyIgbgðaj)anfa ³ C
D kñúgRtIekaN ADC man³ A z

o
y
C
x
mMu ABD  2 AD nig mMu CBD  2 CD
1 1
-RCug AD  DC ¬smµtikmµ¦ B

edaydkGgÁ nigGgÁeyIg)an -mMu ADC  90 ¬mMucarwkknøHrgVg;Ggát;p©it AC ¦


o

 1
ABD  AD
naM[ ADC CaRtIekaNEkgsm)atRtg;kMBUl D


 2 vi)ak mMu)at x  y  45 o

CBD  1 CD kñúgRtIekaN ABC man ³



 2
1
ABC  AC -mMu ABC  x  90 ¬mMucarwkknøHrgVg;Ggát;p©it AC ¦
o

2
dUcenH eyIgbgðaj)anfa ABC  12 AC . dUcenH KNna)an x  90 o
, y  z  45 o .
 60  -


-TMB½rTI 171³ KNnargVas;mMu a nig b tamrUb eyIg)an CMD  12  AB  CD  ¬mMukñúgrgVg;¦


eXIjfactuekaNenHCactuekaN 92o

carwkkñúgrgVg; enaHplbUkmMuQmesµI 180 o


112o

b

1
2
 
150o  120o  135o
a

naM[ a  112  180  a  68


o o o
dUcenH KNna)anrgVas;mMu CMD  135 . o

b  92 o  180 o  b  88 o -TMB½rTI 175³ KNnargVas;mMu D énrUbxageRkam ³


dUcenH KNna)anrgVas;mMu a  68 nig b  88 .o o
mMu PDRCamMueRkArgVg;
-TMB½rTI 173³ KNnargVas;mMu naM[ PDR  12  PSR  PR  260o
P

eyIgman ctuekaN ABCD


B o
S 100o
A 1

 260o  100o  D

carwkkñúgrgVg; / AB  BC nigmMu 76o


2
 80o
R

27 o E
ACE  27 o , ADE  76 o dUcenH rgVas;mMuKNna)anKW D  80 .
D
o
C

k> KNnargVas;mMu ACB


ABC  ADC  180o ¬mMuQmctuekaNcarwkkñúgrgVg;¦ -TMB½rTI 178³ KNnargVas;mMu D

ADE  ADC  180 ¬mMubEnßmKña¦


o eyIgman DE  DF nigmMu o

naM[ ABC  ADE  76


F
o
FET  40o 40o

kúñgRtIekaN ABC man AB  BC CaRtIekaNsm)at k> KNnamMu EOF S E T

enaHmMu)at ACB  180 2ABC


o
mMu EOF CamMup©itmanFñÚsáat; EF
180  76 o o naM[ EOF  EF
  52 o

2 edaymMu FET CamMucarwkBiessmanFñÚsáat; EF


eRBaH plbUkmMukñúgénRtIekaNesµI 180 . o

enaH EF  2  FET  2  40  80 o o

dUcenH KNna)anrgVas;mMu ACB  52 . eyIg)an EOF  EF  80


o
o

x> KNnargVas;mMu BAD dUcenH rgVas;mMuKNna)anKW EOF  80 . o

edaymMu BCD  ACE  ACB  27  52  79 x> KNnargVas;mMu DES


o o o

ehIy BCD  BAD  180 ¬mMuQmctu>carwkkñúgrgVg;¦ kñúgRtIekaN EDS man DE  DF


o

naM[ BAD  180  BCD  180  79  101


o o
naM[ RtIekaN EDS CaRtIekaNsm)at Edlman
o o

dUcenH KNna)anrgVas;mMu BAD  101 . mMu EDF  FET  40 ¬mMucarwkmanFñÚsáat;rYm EF ¦


o o

-TMB½rTI 174³ KNnargVas;mMu CMD A nigmMu)at DEF  12 180  40   70 o o o

eyIgman CAD  60 nigmMu o


60
B eday DE  DF enaH DE  DF ¬Ggát;FñÚb:unKñaFñÚesµIKña¦
o
M o
ADB  75o 75o DF
DEF 
mMu CAD manFñÚsáat; CD C D ehIy 2
 DES  DEF  70 o
DE
DES 
naM[ CD  2  CAD  2  60 o
 120o 2

mMu ADBmanFñÚsáat; AB eRBaH DES CamMucarwkBiessmanFñÚsáat; DE


naM[ AB  2  ADB  2  75 o
 150o dUcenH KNna)anrgVas;mMu DES  70 . o

 61  -


2 ១៥ 2

-TMB½rTI 182 ³ sg;cMNuc P A


-TMB½rTI 188 ³ bMeBjtaragxageRkam ³
sg;Ggát; AB rYcsg; kñúgRtIekaN ABC man DE AB
C

knøHbnÞat; Ax  . RkitknøHbnÞat;   P


eyIg)an pleFobsmamaRt D E


Ax  [)an AC  5 ÉktaRbEvg. AD DC
 b¤ AC BC

 BE EC DC EC A B

P¢ab;BI C eTA B ehIyKUsbnÞat;kat; eyIgbMeBjtaragedayeRbIsmamaRtxagelI eyIg)an ³


cMNucÉktaepSg²[Rsbnwg CB .
C B
x
AD DC BE EC
RbsBVénbnÞat;ÉktaTI2 nigGgát; AB Ca cMNuc P k> 3 2 4
6

dUcenH eyIg)an AP 2
 . x> 7 3
35
3
5
AB 5 8
K> 2
3
3 4
-TMB½rTI 184 ³ KNnatémøén x nig y DC AC EC BC
AD BE
eyIgman  ,  ,  ,  CabnÞat;RsbKña
1 2 3 4 X> 12 8 20 9 6 15
enaHeyIg)anGgát;smamaRtKñaKW ³ g> 12
18 30 9
27 45
2 2
1 1.5 y
 
1.6 x 2
1
-cMeBaH 1 1.5
1.6 x

1
2
1.6
x
1.5
naM[ x  1.5 1.6  2.4 y 3
2
-cMeBaH 11.6  2y 4

naM[ y
2
1.6
 1.25

dUcenH témøKNna)anKW x  2.4 , y  1.25 .


-TMB½rTI 186 ³ KNnakm<s;énedImeQI
tamRTwsþIbTtaElscMeBaHRtIekaN
eyIg)anGgát;smamaRt
A

AF CF
 B
BE CE 1m
C
BE  CF
AF 
CE
b¤ AF  CF
CE
F E

Edl CF CaRbEvgRsemaledImeQI nig CE CaRbEvg


BIbegÁaleTAcugRsemal .
 62  -


 ១៦ 

-TMB½rTI 193 ³ KNna x , y , z -TMB½rTI 197 ³ KNnaRbEvgénTTwgTenø AB


eyIgman ABC dUcnwgRtIekaN ABC manpleFob edaybRmab;RbFan
dMNUcesµInwg 14 CBA A
A

3 4 CDE
eyIg)an smamaRtpleFob B 7
C AB CB
 B 24 m C 32 m D
AB AC BC 1 ED CD
   A
40 m
AB AC  BC  4 b¤ AB 24m

naM[ x
y 40m 32m E

24m  40m
AB 1
 
3 1
  x  12 B  z C
enaH AB 
32m
 30m
AB 4 x 4
AC 1 4 1

AC  4
 
y 4
 y  16 dUcenH RbEvgTTwgTenøKNna)anKW AB  30m .
BC 1 7 1
    z  28 -TMB½rTI 200 ³ KNnatémø x nig y
BC  4 z 4
-tamrUb AEB ADC tamlkçxNÐ m>m
dUcenH KNna)an x  12 , y  16 , z  28 .
eday AEBADC

BE AE

CD AD
A

-TMB½rTI 195 ³ k> bgðajfa ABC dUc PMN 6 4


 E 6 cm B
RtIekaN ABC man A  80 , B  54 o o
y 10
D y C

naM[mMu C  180  A  B 


o
 y
6  10
 15 cm AE  4cm
4 AD  10cm
 
 180 o  80 o  54 o  46 o
dUcenH KNna)an y  15 cm .
edayRtIekaN ABC nigRtIekaN MNP man
-mMu C  M  46 o

-tamrUb GIH JLK tamlkçxNÐ m>m


-mMu B  P  54 o

eday GIH  GH  GI
JLK JK JL G

dUcenH ABC NPM tamlkçxNÐ m>m . x 3


3m x

3 2 I H
x> TajrkpleFobdMNUc  x
3 3 9
 m
K
3m 2m
L

eday ABC NPM enaHeyIgTaj)an ³ 2 2 J

ABC AB AC BC
dUcenH KNna)an x  4.5 m .
  
NPM NP NM PM

dUcenH eyIg)anpleFobdMNUc
AB AC BC
 
NP NM PM
.

 63  -


-tamrUb RQV RTS tamlkçxNÐ m>m


eday RQV
RTS

QV QR

TS TR
sUmEk ST BI
20m  20cm
x 16

20 10
16  20
 x  32 m
10

dUcenH KNna)an x  32 m .
x
Q
QR  16cm V

RT  10cm
R

S 20cm T

-tamrUb DAE BAC tamlkçxNÐ m>m


DAE
eday 
DE DA

BAC BC BA
x 220

40 100
220  40
 x  88 m
100
dUcenH KNna)an x  88 m .
D 120m B 100m
A
40m
x
C

-TMB½rTI 207 ³ KNna z nigsg;rUbeLIgvij


cMeBaH z  x  y 2 2

naM[ z  x  y b¤ x  y  z 1
2 2 2 2 2 2

tamTMnak;TMng 1 bBa¢ak;fa RtIekaNEdlmanrgVas;


RCug x , y , z CaRtIekaNEkg ehIyman x CaRbEvg
GIub:Uetnus nig y , z CaRCugCab;mMuEkg .
dUcenH RbEvgRCug z  x , y  x .
sg;rUb ³ x
y z  x2  y 2

 64  -


 ១៧ 

-TMB½rTI 214 ³ P¢ab;cMNucTaMg* R)ab;cMnYnRCug nigGgát;RTUg 2  2    2  12o  2  3  360o

eyIgman 8 cMNuc eyIgP¢ab;)anrUbdUcxageRkam ³ 12  12o  360o


348o

12
  29o

dUcenH eyIgKNna)anmMu   29 . o

BhuekaNenHman 8 RCug nigmanGgát;RTUgcMnYn 20 . -TMB½rTI 220 ³ bBa¢ak;


-)anCactuekaNEkg minEmnCaBhuekaNniy½t eRBaH
-TMB½rTI 217 ³ rkcMnYnRCugénBhuekaN
-cMeBaH BhuekaNEdlmanplbUkmMukñúgesµInwg 2 700 famMuTaMgbYnrbs;vab:un²Kñak¾BitEmn k¾b:uEnþ RCugrbs;vavij
o

ebI n CacMnYnRCugénBhuekaNenaH b:unKñaEtBIr² minEmnb:unKñaTaMgGs;eT.


tamrUbmnþ 180 o n  2  2 700 o -)anCactuekaNesµI minEmnCaBhuekaNniy½t eRBaHfa
n  2  15 RCugTaMgbYnrbs;vab:un²Kñak¾BitEmn k¾b:uEnþ mMurbs;vavij
n  17
minEmnesµIKñaTaMgGs;eT KWesµIKñaBIr²bu:eNÑaH.
dUcenH BhuekaNenHmancMnYnRCug n  17 .
-cMeBaH BhuekaNEdlmanplbUkmMukñúgesµInwg 3 420 o

ebI n CacMnYnRCugénBhuekaNenaH
tamrUbmnþ 180 n  2  3 420
o o

n  2  19
n  21

dUcenH BhuekaNenHmancMnYnRCug n  21 .
-TMB½rTI 219 ³ KNna  énBhuekaNxageRkam ³

2
2

2
12o
2
3

eyIg)an
 65  -


 ១៨ 

-TMB½rTI 226 ³ KNnaépÞRkLaxag nigépÞRkLaTaMgGs; épÞRkLaTaMgGs; ST  S L  S B


-cMeBaHrUb (a) :  30 cm 2  6 3 cm 2
épÞRkLaxag  40.39 cm 2
1
SL 
2
pa 13 cm
-TMB½rTI 228 ³ KNnamaDekaNKitCa dm 3

 5  4 13
1 ekaNenHman km<s; h  28cm
2 5 cm
kaMfas)atmanRbEvg R  13 cm
 130 cm 2
épÞRkLa)at S B  5  5  25 cm 2
tamrUbmnþmaDekaN
h  28 cm
V  R h eyIg)an
1
épÞRkLaTaMgGs;
2
ST  S L  S B 3
 130 cm 2  25 cm 2 1
V   3.14  13 2  28
R  13 cm

 155 cm 2 3
 4952 .83 cm 3
-cMeBaHrUb (b) :
eday 1000 cm  1dm
3 3

épÞRkLaxag sUmEkkaM)atBI 5 m  5 cm
S L  Ra
naM[ 4952 .83 cm  5dm
3 3

 3.14  5  10 dUcenH maDrbs;ekaNKW V  5dm . 3

 157 cm 2
-TMB½rTI 231 ³ sresrpleFobbrimaRt nigpleFobépÞRkLa
10 cm

épÞRkLa)at
S B  R 2 -cMeBaHrUb (a) :
 3.14  5 2
5 cm
pleFobbrimaRt
 78 .5 cm 2 ebI P CabrimaRtrUbFM 12 mm

o
3 mm

o

épÞRkLaTaMgGs; ST  S L  S B P CabrimaRtrUbtUc
 157 cm 2  78.5 cm 2 eyIg)an P  12 mm  4
 235.5 cm 2 P 3m

-cMeBaHrUb (c) : dUcenH pleFobbrimaRtKW P


4 .
épÞRkLaxag P

SL 
1
pa
pleFobépÞRkLa
2
5 cm
ebI S CaépÞRkLarUbFM
 2  6  5  30 cm 2
1
2 S  CaépÞRkLarUbtUc
2 cm

épÞRkLa)at eyIg)an S  12 mm 
 
2

  4 2  16
1
2
 
S B  6  2 2 2  11 

 S   3 mm 

 6 3 cm 2 dUcenH pleFobépÞRkLaKW S
S
 16 .
2 cm

 66  -


-cMeBaHrUb (b) : x> rképÞRkLaénctuekaN DECB


pleFobbrimaRt épÞctuekaN DECB  S  S
ABC ADE

ebI P CabrimaRtrUbFM  S  S
50 50  18
P CabrimaRtrUbtUc  6
3 3
eyIg)an PP  32 
32
cm 2
3

dUcenH pleFobbrimaRtKW P 3

P 2
. dUcenH épÞRkLaénctuekaN DECB KW 323 cm . 2

pleFobépÞRkLa
ebI S CaépÞRkLarUbFM
S  CaépÞRkLarUbtUc
2

eyIg)an SS   32   94
(Y)
 

dUcenH pleFobépÞRkLaKW S 9

S 4
.
smÁal; ³ eKGaceRbIpleFobrUbtUcelIrUbFMk¾)an .
-TMB½rTI 232 ³ eyIgman DE BC ehIy A

AD  3 cm , AB  5cm
D
nigépÞRkLaRtIekaN ADE  6cm 2 E

k> rképÞRkLaRtIekaN ABC B C

tag S CaépÞRkLaénRtIekaN ABC


S  CaépÞRkLaénRtIekaN ADE
naM[eyIg)anpleFobKW ³
2 2
S  AB  S 5
 smmUl
25
   
S
  AD 6 3   9
25  6 50
S  cm 2
9 3

dUcenH épÞRkLaRtIekaN ABC KW S  503 cm . 2

 67  -
sYsþI¡ elakGñkmitþGñkGanCaTIRsLaj;rab;Gan enAkñúgEpñkenHelakGñknwg)aneXIj dMeNaHRsay
lMhat;tamemeronnImYy² EdlmanenAkñúgesovePAKNitviTüafñak;TI 9 rbs;RksYgGb;rMyuvCn nigkILa e)aHBum<
elIkTI 1 qñaM 2011 . enAkñúgEpñkenHmanbgðajTaMgcemøIy nigTaMgRbFanlMhat; eTAtamemeronnImYy²kñgú
esovePArbs;RksYg. eTaHbICa´ )aneXIjesovePAkMENKNitviTüa Edldak;lk; enAelITIpSarCaeRcInehIy
k¾eday k¾´enAEtbnþbegáItEpñkenHeLIg edIm,IbMeBjlkçN³rbs;esovePAenH [kan; Etsm,ÚrEbb saksmCa
esovePAKNitviTüasRmab;RKU nigsisSfñak;TI 9 .
RbsinebIelakGñkmanbBaða b¤cm¶l;Rtg;cMNucNa EdlmanenAkñúgEpñkenH elakGñkGacTak;Tg;eTAkan;
RKU b¤mitþPkþirbs;GñkEdlmansmtßPaB b¤GñkeroberogesovePAenHkñúgeBlevlasmKYr .

iii

1.
k> 9 x> 16 K> 36 X>  64 g>  100 c> 121 q>  144 C> 625

Q> 8 j> 3 3
8 d>  27 z>  64 D> 125 Z> 216 N> 100 ។
3 3 3 3 3

2.

k> 9
16
x> 49
9
K>  81
4
X> 169
49
g> 196
25
c>  400
225

q> 3
1
8
C> 3 
8
27
Q> 3
64
125
j> 3
512
343
d> 3 
216
1000
.
3.
k> 163 x>  3
362 K> 643 X>  3
8 
3
g> 3
273 c> 3
15

q> 3
82 C> 3
642 Q> 3
 27 
2

4.
k> y2 x> x4 K> x2 y 4 X>  y6

g> 16
x2
c> 100
n4
q> 3
8x3 C>  64m3 ។

5.
k>  2x 
2
x> 3
 5 y 
3
K> 4  a
2
X> 3
 x  3
3

g> 16b2  24b  9 c> 9x2  30x  25 q> 4m2  20mn  25n2 C> 49 x2  112 xy  64 y 2 ។

6.

k> 18 x>  48 K> 75 X> 30


49
g> 10
121
c> 3
40

q> 3
54 C>  3
128 Q> 3
192 j> 3
3m
8n 3
d> 3
16x5 ។

7.
k> 36a2b3 x> 27a4b3 K> 72x5 y 2 X>  112a b 3 4
g> 80m4n3

c> 64x 2 y3 q> 3


16m3n3 C> 3
54x4b3 Q>  128a y 3 5 3
j> 3
24 p3q5 ។

8. x, y, m, n p ។

k> 5 x> 2m K> X> 2 g> 2 x c>


3
23 3m

3 3
6 m 5 4
y3 2n

68
9.
k> 3 2 4 2 5 2 3 2 x> 5 2 3 3 6 2 5 3

K> 3 15  4 3  3 15  6 3 X> 4 3  2 17  3 17  3 3  2 3

g> 23 2  83 3  3 2  33 3 c> 83 2  33 3  53 2  23 3 ។

10.

k> 23 27 
3
4
48 x> 14 288  16 72 K> 53 75 
2
3
27

X> 5 3
128  3 3 250 g> 3 81  12 192
3 3
c> 4 3
54  3 3 128 ។

11.
k> 2 8  3 98  2 200 x>  3 50  32  5 200
K> 3 175  2 28  3 63  112 X> 108  2 27  40  5 160
g> 2 16  3 54  2 128
3 3 3
c> 3 81  12 128  3 192  4 54
3 3 3 3

q> 4 54  6 81  4 16  3 24
3 3 3 3
C>  2 40  3 135  5 320  8 5 ។
3 3 3 3

12.
k> 
 2 2 12  18  5 3 32  27    x> 33 40  3 3
 
135  4 3 320  3 40 
K> 2 27
3
 3 48 
4 50 4 18
5

3
X> 23 81  12
3 3
24 
23 135 33 40
3

2

13. a, b, x, y z ។

A  3 32 x  6 8 x B  2 125x2 z  8x 80z

C  7a b3  b 4a2b  4b D  8b 49b  7 9b3  a 4a  a3


E  3xy x 2 y  2 x4 y3 F  3a a3b5  2b a5b3  5 a3b3
G  8a 3 54a  6 3 16a4 H  3 3 x4 y  6 x 3 xy 4  2 3 x4 y 4 ។

14. A a5,b3
A  ab  ab3  9a3b3  a3b ។
15. A  4a  a a2b  b2a  b 9b a  3 , b  2។
16.
k>  2 3 3 2  x>  4 6  2 5  K> 3 5 5 3 
X>  6 2  12 3  g> 3 8  3 48  c>  3 75  2 48 
q>  2 3    12
3 3 
2

C> 3 2 5 15 
3 3
Q>  6 8  3 2  ។
3 3

69
17.
k> 3 5  2 18  3 48  x> 3 3 3 6  3 2  K> 12 3  2 48  3 32 
X> 32 2  2 18  3 48  g> 4 3  2 6  2 5  c> 2 5 3 3  5 2 
3 3 3 3 3 3

q> 3 3 3 8  2 18 
3 3 3
C> 3 5  4 20  2 45  ។
3 3 3

18.
k> 2 3  89  2 5  x> 3 5  2 10  50  2 80 
K>  50  75  32  48  X>  125  75  80  48 
g> 3 18  3 27 2 8  2 12 
3 3 3 3
c>  80  2 27  3 20  3 12 
3 3 3 3

19. a  3 5  2 10 , b  5 7  2 10 , c  3 18  3 27 d  33 6  3 8 ។

k> 3ab x> a 2


 b2 K> a 2
 2b2 X> b 2
 2ab g> 1
2
cd c>
c 2  b2 q> c 2
 2cd ។

20.

k> 33
11
x>  2748 K> 2 75  15 X> 2 84
12
g> 5 7525  5

c> 12 26 q> 5251005 C>  2 728  Q> 63 7 4 3 21


3 3 3 3
 ។
3
3
2 3
3 3
  6 23 3
21.
b2 49 x 2  56 x  16 a 2  16ab  64b2 25b2  10ab  a 2
A B C D ។
b2  14b  49 36 x 2 a 2  10ab  25b2 16b2  24ab  9a 2
22.

k> 3
10
x> 3 333 K> 6
48
X> 8
27
g> 9 184 c> 5 725
q> C> 2 303 Q> j> d> z>
3
9 3 3
18 7m 11 p 3
3 3 3 ។
3
10 3 3
20 36n 49q 4y 2

23.

k> 36 8 6 x> 3 5
3 20
K> 8
2 75  3 50
X> 2 2 3
80  45

g> 92 323 c> 5 84 13 3 q> C>


3 3 3
23 6 23 2

3 3
2 3 27  3 9 3
16  3 12
24.

k> 3
x> 3a
K> 3 x
X> 3
8a
g> 27 3 m
x 8ab 5 2 x3 y 25ab 7 3 m2 n

c> 15
q> 2 C> Q> 73 7 x
3
7 2x 11

2a  b x  3y 3
2m  3 12n 3
x 2  23 y

70
25. a, b c c ។

k> a  3 , b  4 x> a  5 , b  12 K> a  6 , b  8


X> c  21 , b  10 g> b  30 , c  50 ។

26. m3 8 5 n3 8 5 ។


 2mn
k> mn m m x> mm  nn K> n
2 2 2 2

n2
27. 2 3
3 ។
28. A  22  288 ។


r
29. r 628000 cm3 ។
  3.14 ។
2r

30. 217  217  218 ។


x 2 x  2 x3  85 ។

DDCEE

71
1. cUrKNna ³ g> 196

14 2  14 
   
14
2

5
2
k> 9  3  3 2 25 5 5

c> 
2
400 20 2  20  20 4
x> 16  4  4 2
225

15 2
   
 15  15
 
3

K> 36  6  6 2

q> 3
1 3 13 3  1 
    
3
1
23 2
X>  64   8   8 2 8
3
2

23 2
g>  100   10  10 2 C> 3 
8
27
 3 3  3    
3 3
2
3
c> 121  11  11 2
64 3 43 3  4 
3

Q> 3     
4
q>  144   12   12 2 125 53 5 5
3
512 3 83 3  8 
C> 625  25  25 2
j> 3
343

73
   
7
8
7
Q> 8  2  2 3 3 3
216 3 27 3  3 
3

d> 3        
3
.
j>  8   2   2
3 3 3 1000 125  5 5

d>  27   3   3
3 3 3
3. cUrsRmÜlr:aDIkal; ³
z>  64   4   4
3 3 3
k> 16  4   4   4  64
3 2 3 3 2 3

D> 125  5  5
3 3 3
x>  36   6    6   6  6   6
3 2 3 2 2 3 4 3 3 3
6
Z> 216  6  6
3 3 3

K> 64  8   8   8  512
3 2 3 3 2 3

N> 100 minGacbeBa©j)an EtebI


3 3
1000  10

2. cUrKNna ³
X>   8    8   8
3
3
3 3

k> 169  34   34   34


2

2
2
g>  27   27   27
3 3 3 3

72 7
2 c> 1  1  1
3 5 3

x> 49
    
7
3 q> 8  2   2   2  4
2
9 3 3 3 2 3 3 2 3 2 3 2

2
92 9
K> 
81
4
 2     
2 2
9
2 C> 64  4   4   4  16
3 2 3 3 2 3 2 3 2

Q>  27  27  3   3   3 
2
132  13 
X> 169
49

7 2
   
7
13
7
3 2 3 2 3 3 2 3 2 3 2
9

72
4. cUrsRmÜlr:aDIkal; ³ g> 10

1 1
10    10 
10
2

 11 
k> y  y 2 121 121 11

x> x  x   x
4 2 2 2
c> 40  8 5  2  5  2 5
3 3 3 3 3

K> x y  xy   xy 
2 4 2 2 2
xy 2
q> 54  27  2  3  2  3 2
3 3 3 3 3

C>  128   64 2   4  2   4


3 3 3 3 3
2
X>  y   y    y 6 3 2 3

2
Q> 192  64 3  4  3  4 3
3 3 3 3 3

4
g> 16 4
    
4
3
 x j>  1 
2 3
x x x 3m 3 1 3m
3   3m  3    3m 
 2n 
3 3
8n 8n 2n
2
 10 
c> 100
n 4
  2 
n 
10
n2 d> 16x  8x  2x  2x  2x  2x 2x
3 5 3 3 2 3 3 2 3 2

q> 8x  2x  2x 7. cUrbeBa©jmYycMnYnBIr:aDIkal; ³


3 3 3 3

C>  64m   8m  m   8 m m


3 2 k> 36a b  6 a b  b  6 ab b
2 3 2 2 2

5. cUrsRmÜlr:aDIkal; ³ x> 27 a b  3 a  b  3b  3a b 3b
4 3 2 2 2 2 2

k> 2x  2x  2 x 2
K> 72 x y  6 x  y  2x  6x y 2x
5 2 2 2 2 2 2

x>  5 y    5 y
3 3
X>  112 a b   16 a b   7a   4 a b 7a
3 4 2 2 2 2

K> 4  a  4  a 2
g> 80 m n  16m  n  5n  4m n 5n
4 3 2 2 2 2

X> x  3  x  3
3 3
c> 64x y  8 x y  y  8 xy y
2 3 2 2 2

g> 16b  24b  9  4b  3  4b  3


2 2
q> 16m n  2 m n  2  2mn 2
3 3 3 3 3 3 3 3

c> 9x  30x  25  3x  5  3x  5


2 2
C>  54x b   3 x b  2x   3bx 2x
3 4 3 3 3 3 3 3

q> 4m  20mn  25n  2m  5n  2m  5n


2 2 2
Q>  128a y   4 a y  2a   4ay 2a
3 5 3 3 3 3 3 2 3 2

C> 49x 112xy  64 y  7 x  8 y  7 x  8 y


2 2 2
j> 24 p q  2 p q  3q  2 pq 3q
3 3 5 3 3 3 3 2 3 2

6. cUrbeBa©jmYycMnYnBIr:aDIkal; ³ 8. cUrbBa©l
Ú mYycMnYnkñúgr:aDIkal; ³
k> 18  9  2  3  2  3 2 2
k> 5 6  5  6  25 6  150 2

x>  48   16 3   4  3   4 3 2
x> 2m m  2m  m  4m  m  4m 2 2 3

K> 75  25 3  5  3  5 3 2
K> 23

23

23

2
y3 y  3 2 y6
1
X> 30
49

1
49
 30     30 
7
1
7
30
X> 2 3
5  3 23  5  3 8  5  3
40

73
g> 2x 3
4  3 2 x   4  3 8 x 3  4 
3 3
32x 3 x> 14 288 
1
6
72 
1
4
122  2 
1 2
6
6 2

c> 1 1
3
3m 3m 3m  12 2   6 2
3  3
2n 2n3 8n 3 4 6
3 2 2  2 2
9. cUrKNna ³
k> 3 2  4 2 5 2 3 2 K> 53 75 
2
3
27 
3 2
5
5 3 
2 2
3
3 3

 3  4  5  3 2 3 2
 5 3  3 3
 2 5 3
3 32 3  3
x> 5 2 3 3 6 2 5 3
5 2 6 2 3 3 5 3
X> 5 128  3
3 3
250  53 43  2  33 53  2
 5  43 2  3  53 2
 5  6 2   3  5 3
 203 2  153 2  53 2
  22 3

K> 3 15  4 3  3 15  6 3 g> 3 3
81 
13
2
1
192  33 33  3  3 43  3
2
 3 15  3 15  4 3  6 3 1
 3  33 3   43 3
 3  3 15   4  6 3 2
 9 3  2 3  73 3
3 3
 2 3

X> 4 3  2 17  3 17  3 3  2 3
c> 4 3
54  33 128  43 33  2  33 43  2
 4  33 2  3  43 2
 4 3  3 3  2 3  2 17  3 17
 123 2  123 2  0
 4  3  2 3   2  3 17
  3  17 11.cUrKNna ³
g> 23 2  83 3  3 2  33 3 k> 2 8  3 98  2 200

 23 2  3 2  33 3  83 3  2 2 2  2  3 7 2  2  2 102  2
 2  13 2  3  83 3  4 2  21 2  20 2
 33 2  53 3  4  21  20 2   37 2

c> 83 2  33 3  53 2  23 3 x>  3 50  32  5 200

 83 2  53 2  23 3  33 3  3 5 2  2  4 2  2  5 102  2
 8  53 2  2  33 3  15 2  4 2  50 2
 33 2  3 3   15  4  50 2  31 2

10. cUrKNna ³ K> 3 175  2 28  3 63  112

k> 23 27  34 48 
2 2
3
3 3 
3 2
4
4 3  3 5 2  7  2 2 2  7  3 32  7  4 2  7
 15 7  4 7  9 7  4 7
2 3
 3 3   4 3  15  4  9  4 7  16 7
3 4
 2 3 3 3   3

74
X> 108  2 27  40  5 160 x> 33 40  135  4 320  40 
3 3 3 3

 6 2  3  2 32  3  2 2 10  5 4 2 10  33 8  5  27  5   4 64  5  8  5 


3 3 3 3

 6 3  6 3  2 10  20 10  36 5  3 5  44 5  2 5 
3 3 3 3

 6  6 3   2  20 10   22 10  33 5   42 5 


3 3

 93 5  83 5  173 5
g> 2 16  3 54  2 128
3 3 3

 23 23  2  33 33  2  23 43  2 K> 2 27
3
 3 48 
4 50 4 18
5

3
 43 2  93 2  83 2
2 32  3 4 5 2  2 4 32  2
  3 42  3  
 4  9  83 2  53 2 3 5 3
 2 3  12 3  4 2  4 2
c> 33 81 
13
128  33 192  43 54
2  2  12 3  4  4  2
1
 3 3  3  3 43  2  33 43  3  43 33  2
3 3
  10 3
2
 9 3  2 2  123 3  123 2
3 3

 9  123 3  2  123 2
X> 23
3
1
81  3 24 
2
23 135 33 40
3

2
  33 3  143 2 23 3 1 23 33  5 33 2 3  5
 3  3  3 22  3  
3 2 3 2
q> 43 54  63 81  43 16  33 24
 2 3  3  2 5 3 5
3 3 3 3

 43 33  2  63 33  3  43 23  2  33 23  3  2  13 3  2  33 5
 123 2  183 3  83 2  63 3  3
3 3 5
 12  83 2   18  63 3
 43 2  123 3 13. cUrKNnakenSamxageRkam ³
A  3 32 x  6 8 x
C>  23 40  33 135  53 320  83 5
 3 4 2  2 x  6 2 2  2 x
 2 2  5  3 3  5  5 4  5  8 5
3 3 3 3 3 3 3

 12 2 x  12 2 x
 43 5  93 5  203 5  83 5
 0
  4  9  20  83 5  153 5
B  2 125x 2 z  8 x 80 z
12.cUrKNna ³  2 52 x 2  5z  8x 4 2  5z
k>  22 12  18  53 32  27   10 x 5 z  32 x 5 z
 22 2  3  3  2  53 4  2 
2 2 2
3 32
  10  32x 5 z  42 x 5 z
  
 2 4 3  3 2  5 12 2  3 3 
 8 3  6 2  60 2  15 3 C  7 a b 3  b 4a 2 b  4b
  8  15 3  6  60 2  7a b 2  b  b 2 2 a 2  b  2 2  b
 7 3  54 2  7 ab b  2ab b  2 b
 7 ab  2ab  2  b
 9ab  2 b

75
D  8b 49b  7 9b 3  a 4a  a 3 15. cUrKNnatémøelxénkenSam A ³
 8b 7  b  7 3 b  b  a 2  a  a  a
2 2 2 2 2
A  4a  a a 2 b  b 2 a  b 9b
 56b b  21b b  2a a  a a  2 a  a 2 b  b a  3b b
 56  21b b  2  1a a 
 2  b  a  a 2  3b b 
 35b b  3a a
cMeBaH a3,b2 enaHeyIg)an ³
E  3 xy x 2 y  2 x 4 y 3 A  2  2  3  32  3  2  2

 3 xy x 2  y  2 x 4 y 2  y  4 3  15 2

 3x 2 y y  2 x 2 y y dUcenH KNna)an A  4 3  15 2 .
 3  2x y y 
2 2
x y y
16.cUrKNna ³
F  3a a b  2b a b  5 a b
k> 2 3 3 2   2  3 3  2  6 6
3 5 5 3 3 3

 3a a 2 b 4  ab  2b a 4 b 2  ab  5 a 2 b 2  ab
 3a 2 b 2 ab  2a 2 b 2 ab  5ab ab x> 4 6  2 5   4   2 6  5   8 30
  3  2a 2 b 2 ab  5ab ab K> 3 5 5 3   3  5 5  3  15 15
  5a 2 b 2 ab  5ab ab
X> 6 2  12 3   6   12  2  3   72 6
G  8a 3 54a  63 16a 4
g> 3 8  3 48   6 2  12 3    72 6
 8a 3 33  2a  63 2 3 a 3  2a
 24a 3 2a  12a 3 2a c>  3 75  2 48    15 3  8 3   360
 24  12a 3 2a  36a 3 2a
q> 2 3   12
3 3    1 
2   2     3 3  2   3 6
    2 
H  33 x 4 y  6 x3 xy 4  23 x 4 y 4
 33 x 3  xy  6 x3 y 3  xy  23 x 3 y 3  xy
C> 3 2 5 15   3  5 2 15 
3 3 3
15 3 30

 3x3 xy  6 xy 3 xy  2 xy 3 xy Q> 6 8  3 2   12  3 2  


3 3 3
 36 3 2
 3x3 xy   6  2xy 3 xy
 3x3 xy  4 xy 3 xy 17.cUrKNna ³
k> 3 5 2 18  3 48   3 5 6 2  12 3 
14. cUrKNnatémøelxénkenSam A ³  18 10  36 15 >
A  ab  ab  9a b  a b
3 3 3 3

 ab  b ab  3ab ab  a ab x>  3 3 3 
6  3 2  9 18  9 6

 1  b  3ab  a  ab   27 2  9 6 >
cMeBaH a5,b3 enaHeyIg)an ³ K> 12 3 2 48  3 32   3

8 3  12 2 
A  1  3  3  5  3  5 5  3  52 15 > 2

dUcenH KNna)an A  52 15 .  12  6 6 >


76
X> 32 2 2 
18  3 48  3 36 
9
16  6 c>  80  2 27  3 20  3 12 
3 3 3 3

 2 10  6 3 20  3 12 
2
3 3 3
 18  18 6 >
 63 200  63 120  183 20  183 12
g>  4 3 2 6  2 5    8 18  8 15
3 3 3 3 3
  123 25  123 15  183 20  183 12

c> 2 5 3 3  5 2   6 15  10 10
3 3 3 3 3
19. eyIgman a  3 5  2 10 , b  5 7  2 10
q> 3 3 3 8  2 18   3 36  2 9  2 
3 3 3 3 3
c  3 18  3 27 cUrKNna ³ nig d  3 3
6 3 8
 18 3 3  18 3 2 > k>  3ab  33 5  2 10 5 7  2 10 
C>  3 5 4 3 3

20  23 45   12 3 100  63 225  315 35  6 50  10 70  4 100 
  45 35  90 2  30 70  120
cUrKNna ³
x> a  b  3 5  2 10   5 7  2 10 
18. 2 2
2 2

k> 2 3  89  2 5   45  12 50  40 175  20 70  40


 18 3  4 15  72  16 5 >  300  60 2  20 70

x> 3 5  2 10  50  2 80 
K> a  2b  3 5  2 10   25
2 2 2
7  2 10 
2


 3 5 2 10 5 2  8 5 
 45  12 50  40 2175  20 70  40 
 15 10  24 25  10 20  16 50
 85  12 50  350  40 70  40
 15 10  120  20 5  80 2
  305  60 2  40 70
K>  50  75  32  48 
 5 2  5 3 4 2  4 3  X> b 2
 2ab
 20 2  3  2  3    
2
 5 7  2 10  2 3 5  2 10 5 7  2 10  
 202  2 6  3  175  20 70  40
 100  40 6 
 2 15 35  6 50  10 70  4 100 
X>  125  75  80  48   215  20 70  30 35  60 2  20 70  80

 5 5  5 3 4 5  4 3 
 295  40 70  30 35  60 2

 20 5  3  5  3 
g> 12 cd  12  18  27 3 6  8  3 3 3 3

 205  2 15  3
  18  33 6  2 
1
 160  40 15
3 3
2
g> 3 18  3 27 2 8  2 12 
3 3 3 3
 3 108  2 18  9 6  6 
1 3 3 3

 3 18  9 4  2 12 
3 3
2
93 4 3 93 6
 123 18  63 18  12  36  183 12   18  3
2 2
 123 18  36  36  183 12
 123 18  183 12

77
c> c  b   18  27   5 7  2 10 
2 2 3 3
2 2

B
49 x 2  56 x  16  7x  4 
   
7x  4
2

  18  3  5 7  2 10 
 6x 
2
3
2 2 36 x 6x

 324  6 18  9  175  20 70  40 a 2  16ab  64b 2  a  8b  a  8b


3 3 2

C 2    
a  10ab  25b  a  5b  a  5b
2
 3
324  63 18  20 70  206

25b 2  10ab  a 2  5b  a  5b  a
2

q> c 2
 2cd D
16b  24ab  9a
2 2
   
 4b  3a  4b  3a
  18  27   2 18  27 3 6  8 
3 3
2
3 3 3 3

  18  3  2 18  33 6  2
2
3 3 3

 324  6 18  9  23 108  2 18  9 6  6


3 3 3 3 3
22.bM)at;r:aDIkal;BIPaKEbg ³
 33 12  63 18  9  183 4  43 18  183 6  12 k> 103

3 10
10  10

3 10

2
3 10
10
 33 12  23 18  183 4  183 6  3
10

x> 3 333  3 3
33

3

3 11

3 11
11
20.cUrKNna ³ 11 112

k> 1133  1133 ¬GacCa 33



33
 3 ¦ K> 6

6

6

3

3 3

3
11 11 48 4 2  3 4 3 2 3 2 32 2

x>  2748   48

16
 
4
X> 8

8

8

8 3

8 3
27 9 3 27 3 3 3 3 3 3
2 2 9

K> 2 75  15 
2 75
15
2
75
15
 2 5 g> 9 184  92

18

6 6 2
  3 2
32  2 3 2 2 22
X> 2 84
12
 2
84
12
 2 7
c> 5 725  5 725  2 5 10
 
5 10
2 144 12
g> 5 7525  5 5
25 1
 
5

15
9  3 102
q>
3 3 3 3
9 900 900
75 5 3 3   
10  10
3
10 3 3 2 3
103 10
c> 12 26  12
3
6
3  123 3
3
2
C> 2 303  2
3
3
3
3
30
3
2

23 102
10 3 10  3 10 2

3
100
5
q> 5251005  15
3
100 13
3
3  20
5 5
Q> 3
18
3
18 3 9 3 9 3 10 2
  
10 2

3
900

C>  2 728 


3
20 10 10 10
3 3
2 3  9  2 23 18 3
18 20
3
2  
3
2 2
  4 2
j> 3
7m 3 7m 3 6n 2 3 42mn 2
36n

36n

6n 2

63 n 3

3
42mn 2
6n
Q> 63 7 43 21 3 7 1 3 3 3 1
3 3
36
 2   
3
6 23 3 6 2 21 6 6
d> 3
11 p
49q
3
11 p
49q
7q 2 77 pq 2
3 2  3 3 3 
7q 7 q
3
77 pq 2
7q
21. cUrsRmÜlkenSam ³
b  b 
2
b
2 z> 3
3
4y 2
3
3
4y 2
3
2y
2 y
6y
3 3 3 
2 y
36y
2y
A    
b  14b  49 b7 b7
2

78
23.bM)at;r:aDIkal;BIPaKEbg ³ C> 23 2

k> 36 8 6  36 8 6 2 2  36


3
16  3 12
2 2 3

23 2 3 16 2  3 16  12  3 12 2 

 16  12  16  16 12  
4
2 3
3 3 3 3
12 2

18 2  3 2 2  256  192  144 
3 3 3 3

2 
16  12

x> 3 5


3 5  5

15  5  
3
512  384  3 288
3

3 20 3 20  5 3 100 2
15  5 8 3  3 4 3  6  3 2 3  36
3

 
30 2
8  4 6  23 36
3

K> 8

8
2 75  3 50 10 3  15 2

2
 4  23 6  3 36



8 2 3 3 2  bM)at;r:aDIkal;BIPaKEbg EdlGefrCacMnYnviC¢man
 5 2 3 3 2 2 3 3 2  24.

82 3 3 2  82 3 3 2  k> 3x  3x xx  3 x x


 
512  18  30

4 2 3 3 2  3a  2ab
 
15 x> 3a
8ab

8ab  2ab

3a 2ab 3 2ab
4ab

4b
X> 2 3

2 3

2 3
2 80  45 8 5  3 5 5 5
2 3 5 2 15
K> 3 x

3

3

2y 3 2y

2 y 10 xy
5 2x3 y 5 2x 2 y 5x 2 y
 
5 5 5 25
9 3 9 3 2 9 2 6 5a 2b 2
X>
3
g>
3 3 3 3 3
8a 8a
   3
3 
23 32 23 32 3 2 23 64 25ab 25ab 3 5a 2b 2
93 2  3 6

8 8a 3 5a 2b 2 83 5a 2 b 2
 
c> 5 84 13 3  5 84 13 3  132 5ab 5b
3 3 3 3 3

3 3 3
132
mn 2 273 m 2 n 2
g>
3
273 m 273 m
5 4  13  3  13
3 2 3 2   
 73 m 2 n 73 m 2 n 3
mn 2 7mn
3 3
8 13


5 676  3 507
3
c> 15


15 2a  b 
104 2a  b  2a  b 2a  b  
q> 23 6 23 6 6 2  63 9  3 9 2  
23 27  3 9


6  3 9 6 2  63 9  3 9 2   
15 2a  15 b
2a  b
723 6  123 54  23 486

216  9
723 6  363 2  63 18
q> 2 7x 2x3 y  2 7x 2x32y x2 x 3 y 3 y 

207
24 6  123 2  23 18
3 14 2 x  7 6 xy
 
69 4x  3 y

79
C> 11 3
K> a  6 , b  8 rk c ³
3
2m  3 12n
3
11 3 2m   3 2m 12n  3 12n  
2 2 eyIg)an c  a  b  6  8  100
2 2 2 2 2

  

3

2m  3 12n  3 2m   3 2m 12n  3 12n  
2 2 naM[ c  100  10 ÉktaRbEvg .
 



11 3 4m 2  3 24mn  3 144n 2
3
 X> c  21 , b  10 rk a ³
2m  12n
3
11 4m 3 2
 23 3mn  2 18n  3 2 eday c  a  b naM[ a  c  b
2 2 2 2 2 2


2m  12n b¤ a  26 10  576
2 2 2

3
44m 2  23 33mn  23 198n 2

2m  12n dUcenH a  576  24 ÉktaRbEvg .
Q> 73 7 x g> b  30 , c  50 rk a ³
x 2  23 y
eday c  a  b naM[ a  c  b
3
2 2 2 2 2 2

 x 2 x y 4 y 
73 7 x 3 4 3 2 3 2

b¤ a  50  30  1600

 x  2 y  x  2 x y  4 y 
2 2 2
3 2 3 3 4 3 2 3 2


73 7 x 5  143 7 x 3 y  283 7 xy 2 dUcenH a  1600  40 ÉktaRbEvg .
x2  8y
26. eyIgman m  3 8  5 nig n  3 8  5 ³
7 x3 7 x 2  14 x3 7 y  283 7 xy 2

k> KNna mn m m ³
2
x2  8y

mn  m 2 mn  m 
  mn
25.eyIgman a , b , c CavimaRténRtIekaNEkg m m
3 8  5 3 8  5  6 8
Edl c CaRbEvgGIub:Uetnus . KNnaRCugmYy ³
tamRTwsþIbTBItaK½r x> KNna m2  n2
³
c mn
a
a b  c > 2 2 2
m n
2 2
m  n m  n   m  n

b Edl a  0 , b  0 , c  0 mn m  n 
k> a  3 , b  4 rk c ³ 
 3 8 5  3 8 5  2 5   
eyIg)an c  a  b 2 2 2
K> n
2
 2mn
 1
2m
 1 2 
3 8 5
8 5
2
n n 3
c  3  4 b¤ c  25 2 2 2 2
3 8 5  
3 8 5 
1 2
naM[ c  25  5 ÉktaRbEvg . 3 8 5  
3 8 5 
72  6 40  5
1 2
72  5
x> a  5 , b  12 rk c ³ 67  2 77  12 10  

eyIg)an c  a  b  5 12  169 2 2 2 2 2
67
 87  24 10

naM[ c  169  13 ÉktaRbEvg . 67

80
27.cUreRbóbeFobcMnYn 2 nig 3 ³ 3
30.k> bgðajfa 2  2  2 ³
17 17 18

tag A  2 elIkGgÁTaMgBIrCasV½yKuN 6 eyIgBinitü 2  2  2 1  1


17 17 17

A6  2 
6
2 3 2
 23  8 >  217  2
 2171
tag B  3
3 elIkGgÁTaMgBIrCasV½yKuN 6  218
B6  3 3  3 3
6
 
2 3
 32  9
dUcenH 217  217  218 RtUv)anbgðajrYcral; .
eday 9  8 naM[ B  A enaH B  A 6 6

dUcenH eRbóbeFob)an 3  2 . 3 x> kMNt;témøén x Edl 2 x


 2 x  3  85 ³
eyIgman 2  2  8
x x 3 5

28. sRmÜlkenSam ³ 2 x   x 3   2 3   5

A  22  288 2 2 x3  215

 22  144  2 edaysmIkarmaneKal 2 dUcKña enaHeyIg)an


2 x  3  15
 22  12 2
2 x  12
 3 2   2  3
2
2  2  22 x6
 3 2  2 2

dUcenH kMNt;témø)an x  6 .
3 2 2

dUcenH sRmÜlkenSam)an A3 2 2 .


29.KNnakaMénsILu aMg ³
oooo
sIuLaMgmanmaD tageday V  628 000 cm 3

tamrUbmnþ r 

V  r h 2
2r
Et h  2r
naM[ V   r  2r  2 r 2 3

cMeBaH V  2 r Taj)an r  2V


3 3

eday V  628 000 cm nig   3.14 3

eyIg)an r  6282 000


3
cm
3  100000 cm 3 3

3.14
b¤ r  1000 100 cm  10 100 cm
3 3 3 3

dUcenH RbEvgkaMsIuLaMgKW r  10 100 cm . 3 3

81

1. n
30% n 48
25% 28 n
17 n% 85
ឃ n% 150 20
n 30% 400 ។
2. 20 17 ។ ។
3. 25 000 20 000 ។ ។
4. A 2 400 B 3 000 ។
A B ។
5. 350 000 60% ។

6. 60$ 80% ។
7. 4 200 20% ។ ។
8. 16% 4 200 ។
9. 1 12% ។ 600 000
180 000 400 000 14%

10. 8%

70% 80 000 ។

DDCEE

82

1. KNna n Edl ³ g> n esµInwg 30% én 400 ³


k> 30% én n esµInwg 48 ³ eyIg)an n  10030
 400 enaH n  120
eyIg)an 30%  n  48 dUcenH KNna)an n  120 .
b¤ 100
30
 n  48
2. rkPaKryénsMNYrEdleFVIRtUv ³
naM[ n
48 100
enaH n  160
30 sMNYrmancMnYn 20 ehIyeFVIRtUv)ancMnYn 17
dUcenH KNna)an n  160 . -sMNYreFVIRtUv eFobnwgsMNYrTaMgGs;
tagedayRbPaK 17
x> 25% én 28 esµInwg n ³ 20

eyIg)an 25%28  n -PaKryénsMNYreFVIRtUv eFobnwgsMNYrTaMgGs;


enaH n  100 25
 28 b¤ n   28  7
1 tagedayRbPaK 100 n

4
17  100
eyIg)an 100 n

17
enaH n  85
dUcenH KNna)an n  7 . 20 20

dUcenH PaKryénsMNYrEdleFVIRtUvKW 85% .


K> 17 Ca n % én 85 ³
3. rkPaKryénkarlk;bBa©úHtémø ³
eyIg)an n%  85  17
enaH 100 n
 85  17
GavmYyBImunlk;éfø 25 000 erol ehIy\LÚv
lk;éfø 20 000 erol
naM[ n  1785100 enaH n  20
naM[ PaKryénkarlk;bBa©úHtémøKW ³
25000  20000
dUcenH KNna)an n  20 . 25000
100%  20%

X> n % én 150 esµInwg 20 ³ dUcenH PaKryénkarlk;bBa©úHtémøKW 20% .


eyIg)an n% 150  20 4. rkPaKryénsisSsala A eFobnwgsala B
enaH 100 n
 150  20
sala A mansisS 2400 nak; nigsala B man
naM[ n  20150100 enaH n
200 40
15

3
sisS 3000 nak; naM[ PaKryénsisSsala A
eFobnwgsala BKW ³ 3000
2400
100%  80%
dUcenH KNna)an n  403 .
dUcenH PaKryrk)anKW 80% .
83
5. rkcMnYnR)ak;EdlxVHedIm,ITijkg; 8. rktémømunbBa©úHtémø ³
kg;mantémø 350 000 erol ehIyvNÑaKat;snSM tag x CatémømunbBa©úHtémø
)an 60% énéføkg;rYcehIy eKlk;kñúgtémø 4200 erol eRkayBIbBa©úHtémø16%
naM[ Kat;enAxVHR)ak;cMnYn 40% énéføkg;eTot eyIg)an x  4200
x
100%  16%
eyIg)an cMnYnR)ak;xVHKW x  4200
 0.16
40% × 350 000 = 140 000 erol
x
x  4200  0.16 x
0.84 x  4200
dUcenH cMnYnR)ak;enAxVHKW 140 000 erol .
x  5000

6. rktémørbs;m:asIunft ³ dUcenH témøBImunKW 5000 erol .


sux snSM)an 60$ RtUvnwg 80% énéføm:asIuft
9. rkPaBcMeNj b¤xatrbs;mIgsM ³
naM[ R)ak; x $ RtUvnwg 100% énéføm:asIuft
-R)ak;cMNayrbs;mIgsM bg;[FnaKar
eyIg)an smmaRt 60x  100 80
mIgsM x©IR)ak; 1 lanBIFnaKarmankarR)ak; 12%
100
Taj)an x  6080  75 $
naM[ R)ak;cMNay  100 12
1000000  120000 `

dUcenH m:asIunftmantémø 75$ . -R)ak;cMNUlrbs;mIgsM


R)ak; 600 000 ` cMeNj 180 000 ` nigR)ak;
7. rkcMnYnsisSénqñaMcas; ³
400000 cMeNj)an 14% EdlRtUvnwg
tag x CacMnYnsisSénqñaMcas; ¬Edl x  0 ¦
 400000  56000 `
14
-qñaMenHsalamansisScMnYn 4200 nak; ekInCagqñaM 100

muncMnYn 20% naM[ R)ak;cMNUl  180000  56000  236000`


tamry³PaKryénkMeNIn eyIg)an -R)ak;cMeNj = R)ak;cMNUl  R)ak;cMNay
4200  x  236000120000  116000 `
100%  20%
x
4200  x 20
 dUcenH enAdMNac;qñaMmIgsMcMeNj 116000erol
x 100
4200  x
 0 .2 10. rkcMnYnsisSEdlTTYl)anGaharUbkrN_
x
4200  x  0.2 x -sisSRbLgCab;man 70% énsisS 80000 nak;
1.2 x  4200 sisSRbLgCab;  70%80000  56000nak;
x
4200
1 .2
-sisSTTYlGa>man 8% éncMnYnsisSRbLgCab;
x  3500 sisSTTYlGa>  8% 56000  4480nak;
dUcenH cMnYnsisSénqñacM as;KW 3500 nak; . dUcenH sisSTTYlGa>mancMnYn 4480 nak; .
84

1.
A   x  4  x2  4 x  1 B   x  3  x2  4 x  3 C   a  b   a2  2ab  3b2 

D   x  2   x2  5x  1 E   y  5  y 2  3 y  8 F  3n  4 n  5
G   x  3 x  1 H   y  1 y  2 ។
2 2

2. a
x  a x  8  x 2  12 x  32
2 y  6 y  a   2 y 2  8 y  42 ។

3.
k> 2 x  8x
4
x> 7b  21b3
K> 8ax 12a x2 2 3

X> 10 x y 15xy3 2 3
g> 6 x  9 y
2 2
c> 15x  20 y 2

q> 4x  2x  14x
3 2
C> 3a  9a 15
4 2
Q> 2x  3x  4x
3 2

j> 9xy  3x  4xy 2 2


d> 8abc  4b c 12a bc
2 2 2
z> 6x yz  2xy z  4xyz
2 2

D> 12 x y t  4 x yt
4 3 2 3 2
 8 x 2 t  16 xy ។

4.
k> a  x 1  b  x 1 x> z  y  3  2  y  3 K> ab  3a  9b  27
X> ab  7a  4b  28 g> xy  2x  7 y 14 c> ap  2 pk  ya  2 yk
q> am  mb  an  nb C> 12xy  15x  4 y  5 Q> 2ab  8a  3b 12
j> 3ab 12a  b  4 d> xy  8x  3 y  24 ។
5.

A  at  bt  ct  2a  2b  2c B  ax  2ay  3az  4 x  8 y  12 z
C  ax  ay  az  bx  by  bz D  y 2  cy  ay  ac  by  bc ។
6.
A   x  y  z2 B   x  y   z  t 
2 2 2

C   x 2  2 xy  y 2   t 2 D   a  b   a  b ។
2 2

7. a

k> 4 x  3ay  2 x  9 y 2 x  9 y 


2 4 2 2

x> 16 x  5ay  4 x  5 y 4 x  5 y 


2 4 2 2

85
8.
k> x  162
x> y 121 K> y  1 X> 4z  49 g> 3a 12
2 2 2 2

c> 25b  64 2
q> 36x  y C> 84x  21 Q> 3x  75 j> 4m 144 4 2 2 2 2

d> 8x 160x  800 z> 5x y  500 D> 3t z 147 Z> 2 xy  32 x N> 3a b 192a
2 2 2 2 4 2 2 4 3

t> 5ab  20a  30b 120 f> 4cx  4c 12x 12 T> 2xy  2x  4 y  4 ។
2 2 2 2 2 2

9.
k> 9x  1  42 x  3 2 2
x> 4  y  2  y 2 2
K> 3x  1  x  2x 1 2

X> 2 x  5  4 x  25 2 2
g> 16 x  3  x  12 2
c> x  x 3

q> t 1
4
C> 4x 16x
2
Q> x  1  x  1 2 x  2
2 2

j> x  x5
d> x  y 1  4x ។2

10. a P  2 x3  3x 2  8 x  3  x 1  2 x2  ax  3 P
1។
11.
k> x  5x  4
2
x> x  5x  6
2
K> t  8t  15
2
X> x 10x  9 2

g> t 11t  28
2
c> x  7 x  8
2
q> x  x  6
2
C> x 11x 12
2

Q> b  6b  7
2
j> x  3x  4
2
d> y  y  12
2
z> y  2 y  35 2

D> n  4n 12
2
Z> a  3a 18
2
N> x  6x  7
2
t> 5t  12t  7
2

f> 2x 13x  7 2
T> 2x  5x  3
2

12.
k> 2a  24a  70
2
x> 3x  21x  36
2
K> 5a 15a  90
2

X> 2x  4x 160 2
g> 4bc 12bc  40b
2
c> 6 xy  18 xy  168 x
4 2

13.

k> 3x  x x 2 x> 2x3x 5  3x 1 K> y  3yy44


X> a  3  a 5 1 g> b  5  b 5 5 c> 3n  42nn  53 ។

c 2x 2  x  3
14. a, b, c ax  b   ។
x 1 x 1
15.

k> 816xx1  x2x9 10 y  1 2

2
x> 100 y

5y
y  10
2
K> 12111 x  x x 11
2

X> 2t t 3  3t 2 3 g> x 4 x4x16 4  x44 c> 164zz2  z 4 4 ។


2

86
16.
1 3 2 3 1 1 4
   5
k> x y
4
x> x y
5
K> x y
6
X> x
5
 
y x 5y y
8 4 b 3x 7 2b 3
9    2
g> x
6
c> a2 a
2
q> x  49 x  7
2

1 7
C> 5
b4 b
3
 12   2
y a x  7 2 x  14 5b  20 4b  16b
1
Q> a 1
1
j> 1
2

a c
1 c 1
a 1
a 5c
2 x  8x
3
17. A 3 ។ A x  2011 ។
2 x  8x 2  8x

DDCEE

87

1. BnøatkenSamxageRkam ³ 2. rk a edIm,I[ ³

A  x  4 x 2  4 x  1  k> eyIgman x  a x  8  x 2
 12 x  32
 x  4 x  x  4 x  16 x  4
3 2 2
x 2  8 x  ax  8a  x 2  12 x  32
 x 3  15 x  4 x 2   8  a x  8a  x 2  12 x  32


B   x  3 x 2  4 x  3  eyIgpÞwmemKuNRtUvKñaénGBaØat x rbs;GgÁTaMgBIr
 x 3  4 x 2  3 x  3 x 2  12 x  9 eyIg)an  88aa 32 12 naM[ a  4
 x3  7x2  9x  9 


C  a  b  a 2  2ab  3b 2  dUcenH témørk)anKW a  4 .
 a 3  2a 2 b  3ab 2  a 2 b  2ab 2  3b 3
 a 3  3a 2 b  ab 2  3b 3
x> eyIgman 2 y  6 y  a   2 y 2
 8 y  42
2 y 2  2ay  6 y  6a  2 y 2  8 y  42

D  x  2 x 2  5 x  1  2 y 2  2a  6 y  6a  2 y 2  8 y  42
 x 3  5 x 2  x  2 x 2  10 x  2 eyIgpÞwmemKuNRtUvKñaénGBaØat y rbs;GgÁTaMgBIr
 x  3 x  11 x  2
3 2

eyIg)an 2a6a 6 42


8
naM[ a  7

E   y  5 y 2  3 y  8  

 y 3  3 y 2  8 y  5 y 2  15 y  40 dUcenH témørk)anKW a  7 .
 y  2 y  23 y  40
3 2

F  3n  4 n  5
3. dak;kenSamCaplKuNénktþa ³
 3n 2  15n  4n  20 k> 2x  8x  2 xx  4
4 3

 3n 2  19n  20
x> 7b  21b  7bb  3
3 2

G  x  3x  1
2


 x  3 x 2  2 x  1  K> 8ax 12 a x  4ax 2  3ax
2 2 3 2

 x 3  2 x 2  x  3x 2  6 x  3 X> 10 x y 15 xy  5xy 2x  3 y 


3 2 3 2 2

 x 3  x 2  5x  3
g> 6 x  9 y  32 x  3 y 
2 2 2 2

H   y  1 y  2
2


  y  1 y 2  4 y  4  c> 15 x  20 y  53x  4 y 
2 2

 y3  4 y2  4 y  y2  4 y  4
 y  3y  4
3 2
q> 4x  2 x  14 x  2x2x  x  7
3 2 2

C> 3a  9a 15  3a  3a  5


4 2 4 2

88
Q> 2x  3x  4x  x2 x  3x  4
3 2 2
5. dak;kenSamxageRkamCaplKuNénktþa ³
j> 9 xy  3x  4 xy  x9 y  3x  4 y 
2 2 2 A  at  bt  ct  2a  2b  2c
 a  b  c t  2a  b  c 
d> 8abc  4b c  12 a bc  4bc2ac  b  3a 
2 2 2 2  t  2a  b  c 
B  ax  2ay  3az  4 x  8 y  12 z
z> 6 x yz  2 xy z  4 xyz  2 xyz3x  y  2
2 2
 x  2 y  3z a  4x  2 y  3z 
 a  4x  2 y  3z 
D> 12 x y t  4 x yt  8x t  16 xy
4 3 2 3 2 2

C  ax  ay  az  bx  by  bz

 4 x 3x 3 y 3t 2  x 2 yt 2  2 xt  4 y   a  x  y  z   b x  y  z 
 a  bx  y  z 
4. dak;kenSamxageRkamCaplKuNénktþa ³
D  y 2  cy  ay  ac  by  bc
k> ax  1  bx  1  x  1a  b   y  c  y  a  y  c   b y  c 
x> z y  3  2 y  3   y  3z  2   y  c  y  a  b 
K> ab  3a  9b  27  ab  3  9b  3 6. dak;kenSamxageRkamCaplKuNénktþa ³
b  3a  9 
A  x  y   z 2  x  y  z x  y  z 
2

X> ab  7a  4b  28  ab  7   4b  7 


B  x  y   z  t 
2 2

b  7 a  4 
 x  y   z  t x  y   z  t 
g> xy  2 x  7 y  14  x y  2  7 y  2  x  y  z  t x  y  z  t 

 y  2x  7  
C  x 2  2 xy  y 2  t 2 
c> ap  2 pk  ya  2 yk  pa  2k   ya  2k   x  y   t
2 2

 x  y  t x  y  t 
a  2k  p  y  
q> D  a  b   a  b 
2 2
am  mb  an  nb  ma  b   na  b 
 a  b   a  b a  b   a  b 
a  b m  n    a  b  a  b a  b  a  b 
C> 12 xy  15 x  4 y  5  3x4 y  5  4 y  5  2b 2a   4ab

 4 y  53x  1 7. rktémøén a edIm,IepÞógpÞat;smPaBxageRkam ³


Q> 2ab  8a  3b 12  2ab  4  3b  4 k> 4 x  3ay  2 x  9 y 2 x  9 y 
2 4 2 2

b  42a  3  4 x 2  3ay 4  2 x   9 y 2
2
  2

j> 3ab  12 a  b  4  3ab  4  b  4 4 x 2  3ay 4  4 x 2  81y 4

 b  43a  1
eyIgpÞwmemKuNRtUvKñaén y enaHeyIg)an ³ 4

 81
d> xy  8 x  3 y  24   y  8x  3 y  8  3a  81 naM[ a   27
3
  y  8x  3 dUcenH témørk)anKW a  27 .
89
x> 16 x 2  5ay 4  4 x  5 y 2 4 x  5 y 2    t> 5ab 2  20a  30b 2  120
16 x 2  5ay 4  4 x   5 y 2
2
  2
  
 5a b 2  4  30 b 2  4 
16 x 2  5ay 4  16 x 2  25 y 4  5a  30b  4  2

 5a  6 b  2 b  2 


eyIgpÞwmemKuNRtUvKñaén y enaHeyIg)an ³ 4

5a  25 naM[ a 
 25
 5 f> 4cx 2  4c  12x 2  12
5
dUcenH témørk)anKW a  5 .   
 4c x 2  1  12 x 2  1 
 4c  12x  1 2

8. dak;kenSamxageRkamCaplKuNénktþa ³  4c  3x  1x  1

k> x 16  x  4  x  4x  4


2 2 2
T> 2 xy 2  2 x  4 y 2  4
  
 2x y 2  1  4 y 2  1 
x> y 121  y 11   y 11 y  11
2 2 2


 2 x  4  y 2  1 
K> y  1  y  1   y  1 y  1
2 2 2  2 x  2  y  1 y  1

X> 4 z  49  2 z   7  2 z  72 z  7
2 2 2 9. dak;kenSamxageRkamCaplKuNénktþa ³
g> 3a 12  3a  2   3a  2a  2
2 2 2
k> 9x  1  42 x  3
2 2

 32 x  1  2 2 2 x  3
2 2

c> 25b  64  5b  8  5b  85b  8


2 2 2  3x  1  22 x  33x  1  22 x  3
 3x  3  4 x  63x  3  4 x  6
q> 36x  y  6x   y  6x  y6x  y
4 2 2 2 2 2 2
  x  97 x  3
C> 84 x  21  214 x 1  212 x 12x  1
2 2
x> 4 y  2   y 2  2 y  2   y 2 y  2   y 
2

 2 y  2  y 2 y  2  y 
Q> 3x  75  3x  25   3x  5x  5
2 2

 2 y  4  y 2 y  4  y 
j> 4m 144  4m  36   4m  6m  6
2 2   y  43 y  4

d> 8x 160 x  800  8x  20 x  100 


2 2 K> 3x  1  x 2  2 x  1

 8x  10
2 
 3x  1  x 2  2 x  1 
 8x  10x  10  3x  1  x  1
2

 x  13  x  1
z> 5x y 2 2
 500  5 x 2 y 2  100   x  12  x 
 5xy  10 xy  10 
X> 2 x  52  4 x 2  25
D> 3t z 147  3t z  49   3tz  7tz  7
2 4 2 4 2 2
 2 x  5  4 x 2  25
2

 2 x  5  2 x  52 x  5
Z> 2 xy  32 x  2 xy  16   2 x y  4 y  4
2
2 2

 2 x  52 x  5  2 x  5
N> 3a b  192 a  3a b  64 a 
2 4 3 2 4
  102 x  5

90
g> 16  x  3   x  1
2 2
P   x  1  2 x2  5x  3 ehIyeyIgnwg
 4 2 x  3  x  1 dak; 2x  5 x  3 CaplKuNktþatamviFIKuNExVg
2 2
2

 4x  3  x  14x  3  x  1


2 x  1
 4 x  12  x  14 x  12  x  1 2x2  3
x 3
 3x  135x  11 6 x  x  5x

c> x  x  xx 1  xx  1x  1


3 2 eyIg)an 2 x  5x  3  2 x  1x  3
2

naM[ P   x 1 2x 1 x  3


q> t  1  t  1t  1  t 1t  1t  1
4 2 2 2

dUcenH dak;)an P   x 1 2x 1 x  3 .


C> 4 x  16 x  4 xx  4
2

Q> x  1  x  1 2 x  2
2 2 11. dak;kenSamxageRkamCaplKuNktþa ³
 x  1  x  1x  1  2x  1
2 ¬ebItYkNþalKU eyIgKYredaHRsaytamviFIbMeBj
 x  1x  1  x  1  2 nigbnßytYgayRsYlCag EtebItYkNþaless
 x  1x  1  x  1  2
 4x  1
eyIgKY edaHRsaytamviFIKuNExVgeTIbRsYlCag¦
k> x  5x  4 ¬tamviFIKuNExVg¦
2

j> x 5
  
 x  x x 4 1  x x 2 1 x 2 1    x 1
x2  4
 xx  1x  1 x 2  1    x 4
4 x  x  5x
d> x y  1  4 x  x  y  1  4
2
 2

 x y  1  2 y  1  2
dUcenH x 2  5 x  4  x  1x  4 .
 x y  3 y  1
x> x 2
 5x  6 ¬tamviFIKuNExVg¦
10.kMNt;témøén a ³  x 2
x2  6
man P   x 1  2x 2
 ax  3  x 3
3x  2 x  5 x
 2 x 3  ax 2  3x  2 x 2  ax  3
 2 x 3  a  2x 2   3  a x  3 dUcenH x 2  5 x  6  x  2x  3 .
Et P  2x  3x  8x  3 3 2

K> t 2  8t  15 ¬tamviFIbMeBjnigbnßytY¦
eyIgpÞwmemKuNRtUvKñaén P enaHeyIg)an ³  t 2  8t  16  16  15
a  2  3  t  4  1

 3  a  8
naM[ a  5 2

  t  4  1t  4  1
 t  3t  5
dUcenH témøkMNt;)an a 5 .
dUcenH t 2  8t  15  t  3t  5 .
-dak; P CaplKuNktþadWeRkTI 1 ³
cMeBaH a  5 enaH P Gacsresr)anCa
91
X> x 2  10 x  9 ¬tamviFIbMeBjnigbnßytY¦ j> x 2
 3x  4 ¬tamviFIKuNExVg¦
 x 2  10 x  25  25  9 x 4 
x2  4
  x  5  16   x  5  4  x  5  4   x  1
2

  x  9  x  1  x  4 x  3x

dUcenH x 2  10 x  9  x  9x  1 . dUcenH x 2  3x  4   x  4 x  1 .


g> t 2  11t  28 ¬tamviFIKuNExVg¦ d> y 2
 y  12 ¬tamviFIKuNExVg¦
 y  4
2t  7 y2    12
t  28 y 3 
t  4
3y  4 y   y
 4t  7t  11t

dUcenH t 2  11t  28  t  7 t  4 . dUcenH y 2  y  12   y  4 y  3 .


c> x 2
 7x  8 ¬tamviFIKuNExVg¦ z> y 2  2 y  35 ¬tamviFIbMeBjnigbnßytY¦
 y 2  2 y  1  1  35
x 8 
x2  8   y  1  36   y  1  6  y  1  6 
2

 x  1
 x  8x  7 x   y  7  y  5

dUcenH x 2  7 x  8  x  8x  1 . dUcenH y 2  2 y  35   y  7  y  5 .


q> x 2
 x6 ¬tamviFIKuNExVg¦ D> n 2  4n  12 ¬tamviFIbMeBjnigbnßytY¦
x 3   n 2  4n  4  4  12
6
2
x 
 x  2  n  2  16  n  2  4n  2  4
2

 2 x  3x  x  n  6n  2

dUcenH x 2  x  6  x  3x  2 . dUcenH n 2  4n  12  n  6n  2 .


C> x 2
 11x  12 ¬tamviFIKuNExVg¦ Z> a 2
 3a  18 ¬tamviFIKuNExVg¦
 x 12  a  6
x2    12 a2    18
 x  1 a 3 
 x  12x  11x 3a  6a  3a

dUcenH x 2  11 x  12  x  12 x  1 . dUcenH a 2  3a  18  a  6a  3 .


Q> b 2  6b  7 ¬tamviFIbMeBjnigbnßytY¦ N> x 2
 6x  7 ¬tamviFIbMeBjnigbnßytY¦
 b 2  6b  9  9  7  x2  6x  9  9  7

 b  3  16  b  3  4 b  3  4   x  3  16   x  3  4  x  3  4 
2
2

 b  1b  7   x  7  x  1

dUcenH b 2  6b  7  b  1b  7  . dUcenH x 2  6 x  7  x  7 x  1 .


92
t> 5t 2
 12t  7 ¬tamviFIKuNExVg¦ X> 2 x 2  4 x  160  2 x 2  2 x  80  
5t 7 
 2 x 2  2 x  1  1  80 
 
5t 2  7
 t 1  2  x  1  81
2

5t  7t  12t  2 x  1  9  x  1  9 
 2 x  10  x  8
dUcenH 5t 2  12 t  7  5t  7 t  1 .
f> 2x 2
 13x  7 ¬tamviFIKuNExVg¦ dUcenH 2 x 2  4 x  160  2x  10 x  8 .
2 x  1
2x2  7 g> 4bc 2
 12 bc  40 b  4b c 2  3c  10  
x 7
c 5 
14 x  x  13x c2    10
tamviFIKuNExVg c  2
dUcenH 2 x  13 x  7  2 x  1x  7 .
2
 2c  5c  3c

T> 2x  5x  3
2
¬tamviFIKuNExVg¦ dUcenH 4bc 2  12 bc  40 b  4bc  5c  2 .
2 x  1
2x 2 
x 3
3 c> 6 xy 4  18 xy 2  168 x
6 x  x  5x 
 6 x y 4  3 y 2  28  >
dUcenH 2 x 2  5 x  3  2 x  1x  3 . 4y2 7 
y  2   28
tamviFIKuNExVg  y  4
12.dak;kenSamxageRkamCaplKuNénktþa ³  4 y2  7 y2  3y2

k> 2a  24 a  70  2a  12 a  35 
2 2 naM[ 6 xy  18 xy 4 2
 168 x


 2 a 2  12 a  36  36  35  
 6x y 2  7 y 2  4  
 6 x y  7  y  2 y  2
 
2

 2 a  6   1  2a  6  1a  6  1
2

 2a  5a  7  dUcenH 6 xy 4  18 xy 2  168 x

dUcenH 2a  24a  70  2a  5a  7 .


2  
 6 x y 2  7  y  2 y  2 .
x> 3x  21x  36  3x  7 x  12 
2 2
13.KNnakenSamsniTanxageRkam ³
 x 4
x2  12 k> 3x  x x 2  3x  x xx  2
tamviFIKuNExVg  x 3
3x  4 x  7 x 3x 2  x  2

x
dUcenH 3x  21x  36  3x  4x  3 .
2

K> 5a  15a  90  5a  3a  18 


2 2 x> 2x3x 5  3x 1  2x  5 3x3x3x 1
a  6 2 x  5  9 x 2  3x

  18
2
a  3x
tamviFIKuNExVg a 3 
3a  6a  3a 9x  x  5
2

3x
dUcenH 5a 2  15 a  90  5a  6a  3 .
93
K> y  3yy44  y3 y 3y4 4 y  4 15.KNnakenSamsniTanxageRkam ³

3y2  4 y  y  4 k> 816xx1  x2x9
2

3y  4 6x  1 2x
 

3y  5y  4
2
9  x 9  x  x  9
3y  4 6 x  1  2 x9  x 

9  x 9  x 
X> a  3  a 5 1  a  3aa11  5 
6 x  1  18 x  2 x 2
a 2  a  3a  3  5
9  x 9  x 
 2 x 2  12 x  1
a 1 
a  2a  8
2 9  x 9  x 

a 1
10 y  1
x> 
5y

g> b  5  b 5 5  b  5bb55  5 100  y y  10


2

10 y  1 5y
 
b 2  52  5 b 2  20 10  y 10  y  y  10
 
b5 b5 10 y  1  5 y 10  y 

10  y 10  y 
2n  3 3n4n  5  2n  3
c> 3n 
4n  5

4n  5 
10 y  1  50 y  5 y 2
12 n  15 n  2n  3
2 10  y 10  y 
 5 y 2  40 y  1
4n  5 
12 n  17 n  3
2 10  y 10  y 

4n  5
x2
K> 11

121  x 2 x  11
14.kMNt;témø a , b nig c ³ 11 x2
eday ax  b  x c1  ax  bxx11  c  
11  x 11  x  x  11
ax 2  ax  bx  b  c 11  x 2 11  x 


x 1 11  x 11  x 
ax   a  b x   b  c 
2

11  11x 2  x 3

x 1 11  x 11  x 
2x  x  3
2
ehIy ax  b 
c
x 1

x 1 X> t

2
2t  3 3t  3
ax   a  b x   b  c  2 x 2  x  3
naM[
2
 3t  2 2
x 1 x 1 
6t  3
eyIgpÞwmemKuNRtUvKñaén x enaHeyIg)an 3t  4

a  2 a  2 6t  3
 
 a  b  1 naM[ b  1
 b  c  3 c  4
 

dUcenH kMNt;)an a  2 , b 1 nig c  4 .


94
4 x  16 4 4  ab
g> 
x  4x  4 x  4
4 b

a  a 2  4  ab  a
c> a 2

4 x  16  4 x  4  2
 12
2  12a a2 2  12a

x  4x  4 a a
4  ab
4 x  16  4 x  16 
 2a  12a 2
x  4x  4
32 3x 7
 3x

7 
x  4x  4 x 2  49 x  7   x  7  x  7  x  7
q> 1 7 1 7
 
c> 4z  2

4 x  7 2 x  14 x  7 2 x  7 
16  z 2
z4 3 x  7 x  7 
4z  2
 
4
x  7 x  7 
4  z 4  z  4  z 
2 x  7   7  x  7 
4 z  2  44  z  2 x  7  x  7 

4  z 4  z  3 x  7 x  7  2 x  7  x  7 
4 z  2  16  4 z  
 x  7 x  7  2x  7   7x  7 
4  z 4  z  23 x  7 x  49
 14 
 2 x  14  7 x  49
4  z 4  z  20 x  98 20 x  98
 
 5 x  63 63  5 x

16. sRmÜlkenSamsniTanxageRkam ³ 2b 3
 2
1 3 y  3x
 C> 5
b4 b
3
y  3x y y  3x  2
k> x y
4

xy
4

xy
 
4 4x
5b  20 4b  16b
2b  b 2  3b  4
y y
b 2 b  4
2 3 2 y  3x 
 5 3

x> x y

xy

2 y  3x  x  2 y  3x
   5b  4 4bb  4

5

5 xy  5  5y 2b 3  3b  12
b 2 b  4
x x
1 1 yx 
 4b  3
y  x 5y 5 y  x  4bb  4 
K> x y

xy
6
 
xy  6
 
2b 3  3b  12 4bb  4
6 6x

 
b 2 b  4
5y 5y
4b  3
4
5
4  5x


4 2b 3  3b  12 
4  5x y 4 y  5 xy
X> x
5
 x 
5 x
 
5 5x
b4b  3

y y
8 8  9x
9
8  9x y 8 y  9 xy
g> x
6
 x 
6 x
 
6 6x
y y

95
1 1 1
Q> a 1  a 1  a 1
1 1 a
a a 2 a 2
a
1 a 1 a 1
a a
1 1
 3 a  1  a 3 1
a aa a
a 1
2
a 1
2

1 a 1
2
  3
a 1 a
a2 1
 3
a a  1

j> 1
2

1
2
c c
c 1 5  c  c 1
1
5c 5c
2 2
 
1 5c
c c
6 6
5c
2 2
 
6c  5  c 5c  5
6 6
6 12
 2 
5c  5 5c  5

17.KNnatémøén A ³
eyIgman A  2 x 2x 8x 8x 8x
3

3 2



2x x2  4 

2x x2  4x  4 

x  2x  2
x  22
x2

x2
cMeBaH x  2011 enaHeyIg)an ³
x  2 2011 2 2013
A  
x  2 2011 2 2009

dUcenH KNna)an A
2013
2009
.

96
៤ ១

1.
k> 4x  5  2x 14 x> x  7  2x  8
K> 32 y 1  86  y  X>  z  12   52z 1
g> n  8  4n  23n  4 c> 2m  3  2m  5  4mm  4  4
2

q> 5 p  8  p  2 4  3  5 p 13 C>  k  4k  2  2  2k  7


Q> 2  4 2x  31  51  x1  0 j> 3x  2  2 x 2  2x 3  3  x 2 
d> 67 x  8  76x  4  56  7 x  35x 1 z> 83  2 p 161  p  3 p  4  20  3 p ។
2.

k> 53x  98  76x  163 x> 2 y3 1  5 4 y  1  3 y6 2


K> 23z9 8  5z3 3  3z4 8  17 36
X> 3t 8 5  t 1212  1
x4 x4 3x  1
g> 3  5  2  15 c> 1  23n  12  n  1 3 n
q> 3k  5k2 3  37 C> 2.4x  3x 51.6  8.4x2 4.9
Q>  34 x  52    23 x  0.6    127 x  0.3  5.8 j> x  32  4x4 3  1  5x 612
d> 8x  2  3  7 x 
2x 1
3 z> 2 y  12  3 y  22 
y y  2
2 4 6 2

D> z  22  z z  3  8 z  7 z  1  4 ។


3 5 15 5
3.
k> xx  3  0 x> x  5x 1  0 K> x  1  4  0
2

X> 4x  1  2 x  3
2 2
0 g> x  6x  0
2
c> 2x  2  0 ។
2

4.
k> x 2
 2x  3  0 x> x 2  12 x  32  0 K> y 2
 14  5 y X> x 2
 x  3x  12 ។
5. A B 57 km ។
1/3h B 10/3h ។
2km/h ។ ។
6.
288km/h ។ 20
480km/h ។

97
7. 280 ។ ឃ
4 ។ ។
8. A B 50 km ។
A B ។
2.5 ។
9. 40% 20 70%។
70% 50% ?
10. ។ 9%
8 000 000 10% ។
4 600 000 ។
11. 30 22
17
F B
3 ។
22  x x 17  x
F
B 3
F B

12. ។
1
2 1
3 2
4 3
2។
13. 30% 10% ។
184 800 ។ ។

DDCEE

98
៤ 1

1. edaHRsaysmIkar ³ g> n  8  4n   23n  4


n  8  4n  6n  8
k> 4x  5  2x 14
 8  3n  6n  8
4 x  20  2 x  14
 3n  6n  8  8
4 x  2 x  14  20
 9n  0
2x  6
n0
x3

dUcenH smIkarmanb£s x 3 . dUcenH smIkarmanb£s n0 .

x> x  7  2x  8
c> 2m  32  2m  5  4mm  4  4
x  7  2 x  16 4m 2  12m  9  2m  5  4m 2  16m  4
x  2 x  16  7 4m 2  10m  4  4m 2  16m  4
3x  23 4m 2  10m  4m 2  16m  4  4
23  6m  0
x
3 m0

dUcenH smIkarmanb£s x


23
3
. dUcenH smIkarmanb£s m0 .
K> 32 y  1  86  y  q> 5 p  8  p   2 4  3  5 p   13
6 y  3  48  8 y 5 p  8  p  2 4  3  5 p  13 
6 y  8 y  48  3 6 p  8  2 5 p  20 
14 y  45 6 p  10 p  40  8
45 16 p  32
y
14 p  2

dUcenH smIkarmanb£s y   14
45
. dUcenH smIkarmanb£s p  2 .
X>  z  12   52 z  1 C>  k  4k  2  2  2k  7 
 z  12  10 z  5   3k  2  2k  9
 12  5  10 z  z 3k  2  2k  9
 7  11z 3k  2k  9  2
7 k 7
z
11
dUcenH smIkarmanb£s k 7 .
dUcenH smIkarmanb£s z
7
11
.

99
Q> 2  4 2 x  3 1  51  x   1  0 2. edaHRsaysmIkar ³
2  4 2 x  3 1  51  x   1  0
2  4 2 x  3 1  5  5 x   1  0
k> 53x  98  76x  163 ¬PaKEbgrYm 48 ¦
2  4 2 x  12  15 x  1  0 16  5 x 6  9 8  7 x 3  3
  
2  4 17 x  11  0 16  3 6  8 8  6 3 16
80 x  54  56 x  9
2  68 x  44  0
80 x  56 x  9  54
68 x  42
24 x  45
42 21
x  45 15
68 34 x 
24 8
dUcenH smIkarmanb£s x
21
.
34
dUcenH smIkarmanb£s x
15
8
.
j> 
3 x  2  2 x 2  2 x 3  3  x 2 
2 y 1 5  y 3y  2
3x  2 3  2 x 2  2 x 3  6  2 x 2 x> 3

4
 1
6
¬PaKEbgrYm12 ¦
3 x  2 x 3  2 x 2  2 x 2  6  2 3 42 y  1 35  y  12 23 y  2
  
 3 x  6  2 3 43 3 4 12 26
62 3 42 y  1  35  y   12  23 y  2
x
 3 8 y  4  15  3 y  12  6 y  4

x
 6  2 3  3  5 y  19  8  6 y
 3  3  5 y  6 y  8  19
6 3 6 11y  11
x  2 32
3 y  1
dUcenH smIkarmanb£s x  2 32 . dUcenH smIkarmanb£s y  1 .
d> 67 x  8  76 x  4  56  7 x   35x  1
K> 23z9 8  5z3 3  3z4 8  17
42 x  48  42 x  28  30  35x  35x  35 36
0 x  20  65  0 x smIkarmanPaKEbgrYm 36
0 x  45
eRBaH LCM 9 , 3 , 4 , 36   36 eyIg)an ³
Kµantémø x NaEdlKuNnwg 0 esµInwg 45 eT 83z  8 60 z  3 27z  8 17
  
dUcenH smIkarKµanb£s . 36 36 36 36
83 z  8  60z  3  27z  8  17
z> 83  2 p 161  p  3 p  4  20  3 p 24 z  64  60 z  180  27 z  216  17
24  16 p  16  16 p  3 p  12  20  3 p  36 z  116  27 z  199
20  3 p  20  3 p 116  199  27 z  36 z
 3 p  3 p  20  20 63z  315
0p  0 z5
mantémø x eRcInrab;minGs;EdlepÞógpÞat;smIkar
dUcenH smIkarmanb£s z 5 .
dUcenH smIkarmanb£seRcInrab;minGs; .
100
3t  5 t  12 3x  1.6 8.4 x  4.9
X> 8

12
1 ¬PaKEbgrYm 24 ¦ C> 2.4 x 
5

2
33t  5 2t  12 24 24 x 6 x  1.6 58.4 x  4.9
   
24 24 24 10 10 10
33t  5  2t  12  24 24 x  6 x  9.6  42 x  24.5
9t  15  2t  24  24 30 x  9.6  42 x  24.5
11t  24  9  9.6  24.5  42 x  30 x
t
15 14.9  12 x
11
14.9 149
x 
dUcenH smIkarmanb£s t
15
11
. 12 120

dUcenH x
149
Cab£sénsmIkar .
x4 x4 3x  1 120
g> 3

5
 2
15
¬PaKEbgrYm 15¦
5x  4 3x  4 30 3x  1
  
Q>  34 x  52    23 x  0.6    127 x  0.3  5.8
15 15 15 15
5x  4  3x  4  30  3x  1
3 2 7
x  0.4  x  0.6  x  0.3  5.8
4 3 12
5 x  20  3x  12  29  3x
3 2 7
2 x  32  29  3x x  x  x  0.5  5.8
4 3 12
32  29  3x  2 x 9 x  8x  7 x
 5.3
x3 12
10
dUcenH smIkarmanb£s x  3 . x  5.3
12
5.3 12
1 n x
c> 1
2n 1
 n 
3 2 3
¬PaKEbgrYm 6¦ 10
31.8 159
6 4n 3 6n 21  n  x 
    5 25
6 6 6 6 6
6  4n  3  6n  21  n  dUcenH x
159
Cab£sénsmIkar .
3  2n  2  2n 25
 2n  2n  2  3
3 4x  3 5 x  12
0n  1 j> x 
2 4
 1
6
Kµantémø n NaEdlKuNnwg 0 esµInwg 1 eT 12 x 18 34 x  3 12 25 x  12
   
dUcenH smIkarKµanb£s . 12 12 12 12 12
12 x  18  12 x  9  12  10 x  24
27  36  10 x
5k  3
q> 3k 
2
 37 ¬PaKEbgrYm 2¦ 10 x  36  27
6k 5k  3 2  37 9
  x
2 2 2 10
6k  5k  3  74
11k  77 dUcenH smIkarmanb£s x
9
10
.
k 7
dUcenH smIkarmanb£s k  7 .
101
2x 1
d> 8x  2  3  7 x 
2
3 3. edaHRsaysmIkar ³
16x  2  6  14 x  2 x  1  6 k> xx  3  0
16 x  32  6  16 x  7
16 x  16 x  7  26
naM[ xx  30 0 enaH x  0
 x  3
 
0 x  19
Kµantémø x NaEdlKuNnwg 0 esµInwg 19 eT dUcenH smIkarmanb£sBIr x  0 , x  3 .
dUcenH smIkarKµanb£s . x> x  5x 1  0
2 y  1
3 y  2 
2
y y  2
2 naM[ xx 1500 enaH  x  5
x  1
z> 4 6
 
2
 

32 y  1  6 y  2  6 y  y  2
2 2
dUcenH smIkarmanb£sBIr x  5 , x  1 .
  
3 4 y 2  4 y  1  6 y 2  4 y  4  6 y 2  12 y 
12 y 2  12 y  3  6 y 2  24 y  24  6 y 2  12 y K> x  12  4  0
6 y 2  12 y  21  6 y 2  12 y x  1  2x  1  2  0
6 y 2  6 y 2  12 y  12 y  21
x  3x  1  0
x  3  0 x  3
0 y  21 naM[ x 1  0 enaH  x  1
 
Kµantémø y NaEdlKuNnwg 0 esµInwg 21 eT dUcenH smIkarmanb£sBIr x  3 , x  1 .
dUcenH smIkarKµanb£s .
X> 4x  1  2 x  3  0
2 2

D> z 32  zz5 3  8z  157z  1  54 2 2 x  1  2 x  3  0


2 2 2

2x  1  2 x  32x  1  2 x  3  0


smIkamanPaKEbgrYm 15 2 x  2  2 x  32 x  2  2 x  3  0
eRBaH PPCM 3 , 5 , 15   15 4 x  5  0
5
5 z  2 3z z  3 8 z  7 z  1 12 x
2
   4
15 15 15 15
5z  2   3z z  3  8 z  7  z  1  12 dUcenH smIkarmanb£s 5
.
2
x
 
5 z  4 z  4  3z  9 z  8 z  8 z  7 z  7  12
2 2 2 4

5 z  20 z  20  3 z 2  9 z  8 z 2  z  5
2
g> x 2  6x  0
8 z  29 z  20  8 z  z  5
2 2

8 z  8 z 2  29 z  z  5  20
2 x  6x  0
x  6  0 x  6
 28z  15 naM[ x  0 enaH x  0
15  
z
28
x  6
dUcenH smIkarmanb£s x  0 .
dUcenH smIkarmanb£s z
15
28
. 

102
c> 2x 2  2  0 naM[ y  7  0
y  2  0 enaH y  7
 y  2
 
2 x 2 1  0  

x 2 1  0 dUcenH smIkarmanb£s y  7 , y  2 .
x  1x  1  0
naM[ x 1  0
x 1  0 enaH x  1
 x  1
X> x 2  x  3x  12
  x 2  x  3x  12
dUcenH smIkarmanb£s x  1 , x  1 . x 2  4 x  12  0
eyIgdak;CaplKuNktþa tamviFIKuNExVg
4. edaHRsaysmIkar ³  x  6
x2    12
k> x  2x  3  0
2  x 2 
2 x  6 x  4 x
x2  2x  1  1  3  0
x  12  2  0
enaHeyIg)an x  6x  2  0
edayRKb;cMnYnBit x enaH x  1 2
0 naM[  xx  62  00 enaH  yy  62
 
naM[ x  1  2  0
2

dUcenH smIkarmanb£s y  6 , y  2 .
dUcenH smIkarKµanb£s .
5. KNnael,Ónrbs;kaNUtpÞal; ³
x> x 2  12 x  32  0 tag v Cael,Ónrbs;kaNUtpÞal; ¬KitCa km/h¦
x 2  12 x  36  36  32  0
naM[ v  2 Cael,ÓnénclnakaNUt eRBaHvaebIk
x  62  4  0
x  6  2x  6  2  0 tambeNþaycrnþTwkmanel,Ón 2 km/h
x  8x  4  0 tamrUbmnþ el,Óncr × ry³eBlcr = cm¶aycr
x  8  0 x  8
naM[ x  4  0 enaH x  4 eday cm¶ay = 57 km , eBlsrub 10/3 h nig
 
eBlsRmakGs; 1/3 h enaHeyIg)ansmIkar ³
dUcenH smIkarmanb£s x 8, x 4 .
v  2 10  1   57
 3 3
K> y  14  5 y
2
3v  2   57
y 2  5 y  14  0 v  2  19
25 25 v  17
y2  5y    14  0
4 4
 5  81
2 epÞógpÞat; el,ÓnkaNUtKW 17  2  19 km/h
y   0
 2 4 ry³eBleFVIdMeNIr 103  13  3 h
 5 9  5 9
 y    y     0
 2 2  2 2 enaHcm¶aycr 19  3  57 km/h Bit
 y  7  y  2  0
dUcenH el,Ónrbs;kaNUtpÞal;KW v  17 km/h .
103
6. rkry³eBledIm,I[ynþehaH ehaHTan;]T§mÖacRk³ - ry³eBlrbs;GñkCiHm:UtURtUvcMNayKW  250.5v h
tag t Cary³eBlEdlynþehaH ehaHtamTan; edayGñkCiHm:UtUeTAdl; munGñkCiHkg;mYyem:ag
]T§mÖacRk ¬KitCanaTI ¦
eyIg)an  50v  1   250.5v   1
-cm¶aypøÚvEdl ]T§mÖacRkcr)anKW
50 50
d  288 t  20 
1 eRBaH ]T§mÖacRkecjmun 
v 2.5v
2

20 naTI eTIbynþehAeTAtameRkay 2.5  50 50


 
2  2.5v
2.5v 2.5v 2.5v
- cm¶aypøÚvEdl ynþehaHcr)anKW d  480  t
2 125  50  5v
edIm,I[ynþehaH ehaHTan;]T§mÖacRk luHRtaEt ³ 5v  75
v  15
d1  d 2
288t  20  480t naM[ el,Ónm:UtU 2.5v  2.5 15  37.5 km/h
288t  5760  480t
480t  288t  5760 dUcenH kg;manel,Ón 15 km/h nig
192t  5760 m:UtmU anel,Ón 37.5 km/h .
t  30

dUcenH ynþehaHRtUvcMNayeBl 30 naTI 9. kMNt;cMnYnsisSenAkñúgfñak;enaH ³


eTIbehaHtamTan; ]T§mÖacRk . tag V CamaDénsUluysüúgGasIut 70%
-GasIutsuT§ énsUluysüúgGasIut 40% nigman
7. kMNt;cMnYnsisSenAkñúgfñak;enaH ³ maD 20 l KW 40% 20
tag x CacMnYnsisSenAkñúgfñak; ¬KitCanak;¦ - GasIutsuT§ énsUluysüúgGasIut 70% nigman
bRmab; ³ esovePAman 280 k,al Eck[sisS x maD V l  KW 70%V
ehIymñak;²TTYl)anesovePA 4 k,al - GasIutsuT§ énsUluysüúgGasIutfµI 50% nigman
eyIg)an 280 x
4 maDfµI 20  V  l KW 50%  20  V 
enaH x
280
 70 eyIg)ansmmUlGasIutsuT§KW ³
4 40%  20  70%  V   50%  20  V 
dUcenH sisSenAkñúgfñak;enaHmancMnYn 70 nak; . 8
7
V  20  V 
1
10 2
 V  20  V 
80 7 5
8. KNnael,ÓnényanTaMgBIr ³ 10 10 10
tag v Cael,Ónrbs;kg; enaH 2.5v Cael,Ónm:UtU 80  7V  520  V 
7V  5v  100  80
-ry³eBlrbs;GñkCiHkg;RtUvcMNayKW  50v 1h 2V  20  V  10

eRBaH GñkCiHkg;ecjmunGñkCiHm:UtU mYyem:ag dUcenH maDénsUluysüúgGasIut 70% KW 10 l .

104
10.rkR)ak;edIménGRtakarR)ak;nImYy² ³ 12. rkcMnYnemédsrub ³
tag x CaR)ak;edIm EdlKat;TTYl)anGRtakar tag n CacMnYnemédsrubEdlRtUvRbKl;[kILakr
R)ak; 9% ¬KitCaerol¦ tambRmab;RbFan eyIgdwgfa ³
naM[ x  8 000 000 CaR)ak;edIm EdlKat;TTYl -emédRbKl;kñúgéf¶TI1 mancMnYn 2x
)anGRtakarR)ak; 10% ¬KitCaerol¦ x

eday karR)ak;TTYl)anTaMgGs; 4 600 000 ` -emédRbKl;kñúgéf¶TI2 mancMnYn 2 x


2 4
eyIg)an TMnak;TMngénkarR)ak;rbs;Kat;KW ³ x
9% x  10 % x  8 000 000   4 600 000 -emédRbKl;kñúgéf¶TI3 mancMnYn 4 x
2 8
9 x  10  x  8 000 000   4 60 000 000 x
9 x  10 x  80 000 000  4 60 000 000
-emédRbKl;kñúgéf¶TI4 mancMnYn 8  x
19 x  380 000 000 2 16
x  20 000 000 ehIymanemédenAls; 2 eTot
naM[ R)ak;EdlKat;TTYl)anGRtakarR)ak; 10% eyIg)an 2x  4x  8x  16x  2  x
KW 20000000  8000000  28000000 ` 8 x  4 x  2 x  x  32  16 x
15x  32  16 x
dUcenH -R)ak;edImEdlKat;TTYl)anGRta x  32
karR)ak; 9% mancMnYn 20000000 ` dUcenH emédsrubmancMnYn 32 .
-R)ak;edImEdlKat;TTYl)anGRta
karR)ak; 10% mancMnYn 28 000 000 ` 13. rktémø Edl)anlk;BImun ³
tag x Catémølk;BImun ¬KitCaerol¦
11.rkcMnYnsisSEdlelgTaMg)al;Tat; nig)al;e)aH eKbBa©úHtémø 30% éntémøBImun
eyIgmansMNMu Edlman x CacMnYnsisSelgTaMg -naM[ témølk;bnÞab;BbI Ba©úHtémøKW x  30%x
)al;Tat; nig)al;e)aH ehIycMnYnsisS 22  x elg -Bn§Garkénéfølk;KitbEnßmKW 10%x  30% x
Et)al;Tat;suT§ nig 17  x elgEt)al;e)aHsuT.§ -BUsuxcMNayR)ak;srubKW 184 800 `
sisSRbussrub 30 nak; F B
eyIg)an éfølk; + éføBn§ = R)ak;cMNay
sisSminelgTaMg)al; 22  x x 17  x
x  30% x   10%x  30% x   184800
Tat; nig)al;e)aH 3 nak; 3 x  0.3x  0.1x  0.3x   184800
0.7 x  0.10.7 x   184800
eyIg)an 22  x  x  17  x  30  3 0.7 x  0.07 x  184800
 x  39  27
0.77 x  184800
x  12
x  240 000
dUcenH sisSelgTaMg)al;Tat; nig
)al;e)aH mancMnYn 12 nak; . dUcenH TUrsBÞenaHlk;éfø 240 000 ` kalBImun .
105
៥ ១

1.

k. x  5  3 , 3  x  7  2x

x> 5x  10 ,  6x  12 ,  5x  20

K. 2 x   13 ,
x 1

7 4
,  3x 
9
2
X. 5 x  1  4 ,  8x  9  7 , x 1  x  2

g> 3x  5  x  7 ,  2x  11  5x  31
 1
c.  4 x  9  8x  3 ,  2 x  1  2 x  
 2
x 1 2x  3
q> 23 x  3  53 x  27 ,
2

4

C. x  1 42x  1 33x ,
x 1 1 3  2x
5
 
2 2
2.

k> 22xx  33  95 ,


3x  4  8

 3x  4  7
2 x  3  5 x  1 2 x  5  5 x  4
x>  , 
 x  4  3x  2 x  7  2x  3
 3x 2 4 x  3
  2 x  1  0
 4 3 
K> 
2

,  x
2 x  1  3 x 4  2  2  0
 4
3. 40 ។ m2
2 m?
4. ។

5. 5 3 ។ ។

6. 32 12 ។

7. ។ 1 500 20 2 1 000
30 ។ 25 ។ 17 500
20 000 500

106
8. 8 100 ។ A
90 B 80 ។
7 84, 93, 78, 87, 89, 70 81

A
B B។
9. ។ 15 m ។
94 m ។ ។

10. A B A B 5 ។

32 ។ ។

11. 72 ។

12. ។

- 80 000 400 1 km
- 400 000 1 ។

13. ABC AB  12 , AC  3cm ។ BC


3 BC ។

14. 8.
15. 3 cm
15 cm 2
24 cm ។ 2

DDCEE

107
៥ 1

1. edaHRsayvismIkar ³ K2> x 1

7 4
cemøIyCaRkab
k1> x  5  3 cemøIyCaRkab x 1
7  7
7

x 55  35 2 7 4 4
( (
x  2 x
7
4
dUcenH vismIkarmancemøIy x  2 .
dUcenH vismIkarmancemøIy x  74 .
k2> 3  x  7  2x cemøIyCaRkab
 x  2x  7  3 4
) K3>  3x 
9
2
cemøIyCaRkab
x4 3
9 
x
 3  2
2
dUcenH vismIkarmancemøIy x  4 . [
3
x
x1> 5x  10 cemøIyCaRkab 2

x
10
5
2
[
dUcenH vismIkarmancemøIy x   32 .
x2
X1> 5x  1  4 cemøIyCaRkab
dUcenH vismIkarmancemøIy x  2 . 5x  4  1 1
5 (
x2>  6x  12 cemøyI CaRkab x
5
12 x 1
x 2
6
x  2
(
dUcenH vismIkarmancemøIy x  1 .
dUcenH vismIkarmancemøIy x  2 . X2>  8x  9  7 cemøIyCaRkab
x3>  5x  20 cemøIyCaRkab  8 x  7  9
 16 2
 5 x  20 x ]

4
8
5 5 )
x2
x4
dUcenH vismIkarmancemøIy x  2 .
dUcenH vismIkarmancemøIy x  4 . X3> x 1  x  2
x  x  2 1
K1> 2 x   13 cemøIyCaRkab1 0x  3

6
1
x
6
] mantémø x eRcInrab;minGs; EdlepÞógpÞat; 0x  3
dUcenH vismIkarmancemøIy x   16 . dUcenH vismIkarmancemøIyeRcInrab;mni Gs; .
108
x 1 2x  3
g1> 3x  5  x  7 cemøIyCaRkab q2> 2

4
cemøIyCaRkab
3x  x  7  5
6 2x  1 2 x  3
2 x  12 )  0
4 4
x6 2x  2  2x  3
2 x  2 x  3  2
dUcenH vismIkarmancemøIy x  6 . 0 x  5

g2>  2x 11  5x  31 cemøIyCaRkab Kµantémø x NaEdlepÞógpÞat; 0x  5 enaHeLIy


11  31  5 x  2 x 
20
7 dUcenH vismIkarKµancemøIy .
 20  7 x )
 20 1  2 x 1  3x
x
7
C1> x
4

3
12 x 31  2 x  41  3 x 
dUcenH vismIkarmancemøIy x   207 . 12

12

12
12 x  31  2 x   41  3x 
c1>  4 x  9  8x  3 cemøIyCaRkab 12 x  3  6 x  4  12 x
9  3  8x  4 x 6 x  12 x  4  3
1
12  12 x [ 7
x
x 1 18 7
18
dUcenH vismIkarmancemøIy x  1 . cemøIyCaRkab [

c2>  1
 2 x  1  2 x   cemøIyCaRkab dUcenH vismIkarmancemøIy x  187 .
 2
 2x  1  2x  1 x 1 1 3  2x
 2x  2x  1  1

1
2
C2> 5
 
2 2
2 x  1 5 53  2 x 
]
 4x  2  
1 10 10 10
x 2 x  1  5  53  2 x 
2
2 x  2  5  15  10 x
dUcenH vismIkarmancemøIy x   12 . 2 x  3  15  10 x
2 x  10 x  15  3
q1> 2
3
5
x  3  x  27
3
cemøIyCaRkab  8 x  18
9
x
2 x  9  5 x  81 30 4
) 9

9  81  5 x  2 x 4

90  3x cemøIyCaRkab [
x  30

dUcenH vismIkarmancemøIy x  30 . dUcenH vismIkarmancemøIy x


9
4
.

109
2. edaHRsayRbB½n§vismIkar ³ x2> 2x x4335xx21 b¤  3  1  5 x  2 x

 4  2  3x  x
k1> 22xx  33  95 b¤ 22xx  12 b¤ x  6
  2  3x
  2  x  1 b¤ 
6  2 x
cemøIyCaRkab  2
x  

6
-cMeBaH x  6 ³ ]  3
1  x  3
-cMeBaH x  1 ³ [
1 cemøIyCaRkab
-cemøIyénRbB½n§ ³ [
6
]

2
3
-cMeBaH x   23 ³ )
3
dUcenH RbB½n§vismIkarmancemøIy  1  x  6 . -cMeBaH x  3 ³ ]
2

-cemøIyénRbB½n§ ³ 3
)
3

k2> 33xx  44  87 b¤ 3x  12


 b¤ x  4

 3x  3  x  1 dUcenH RbB½n§vismIkarmancemøIy x   23 .
cemøIyCaRkab
4
-cMeBaH x  4 ³ ]  3x 2 4 x  3  9 x 8 64 x  3
1  4  3  2  12  12  12
-cMeBaH x  1 ³ K1>  b¤ 
 42 x  1  3 x  4
[
1
2 x  1  3 x  4
  4
-cemøIyénRbB½n§ ³ [
4
] 4 4
9 x  8  24x  18
b¤ 
dUcenH RbB½n§vismIkarmancemøIy  1  x  4 . 8x  4  3x  4
 8  18  24x  9 x
b¤ 
8x  3x  4  4
10  15x
x1> 2 x  5  5 x  4
b¤ 5  4  5 x  2 x b¤ 
  5x  0
x  7  2x  3  7  3  2 x  x
 2
9  3x x 
b¤  b¤  3
 4  x  x  0
x  3
b£ 
 x  4
cemøIyCaRkab cemøIyCaRkab 2
3

-cMeBaH x  3 ³
3
) -cMeBaH x  23 ³ 0
[
4
-cMeBaH x  4 ³ ( -cMeBaH x  0 ³ (
2
4
-cemøIyénRbB½n§ ³ (
3
) -cemøIyénRbB½n§ ³ 0 3
[

dUcenH RbB½n§vismIkarmancemøIy  4  x  3 . dUcenH RbB½n§vismIkarmancemøIy x  23 .

110
2 x  1  0
 2 x  1

x 
1 4. bgðajfaRCugRtIekaNnImYy²xøICagknøHbrimaRt
K2>  x b¤  b¤  2
 2  2  0  x  4  0  x  4 tag a , b nig c CaRbEvgRCugénRtIekaN
cemøIyCaRkab 
1 naM[ brimaRt P  a  b  c
-cMeBaH x   12 ³
2
) tamvismPaBénRCugrbs;RtIekaN eyIg)an ³
-cMeBaH a  b  c b¤ a  a  b  c  a
4

-cMeBaH x  4 ³ 
1
(
2a  a  b  c
-cemøIyénRbB½n§ ³ 2 4

a
P
1
2
dUcenH RbB½n§vismIkarKµancemøIy . -cMeBaH b  ac b¤ bb  a c b
2b  a  b  c
3. rképÞRkLaEdlGacmanFMbMput ³ b
P
2
tag x CaRbEvgbeNþay nig y CaRbEvgTTwg 2
-cMeBaH c  ab b¤ cc  abc
éncmáar Edl x nig y KitCa m ehIy x  0, y  0 2c  a  b  c
eKdaMkUneQI 40 edImB½T§CMuvji cmáar EdlkUneQI c
P
3
nImYy²XøatBIKña 2 m 2
tam 1 , 2 nig 3 eXIjfaRbEvgRCugnImYy²
naM[ RbEvgbrimaRt P  2  40  80 m
énRtIekaN tUcCagknøHbrimaRt )anbgðajrYc
eyIg)an x  y  P2  802
dUcenH kñúgRtIekaNmYy RbEvgRCugnImyY ²
b¤ x  y  40 ¬elIkGgÁTaMgBIrCakaer¦
x  y 2  402
RtUvtUcCagknøHbrimaRtrbs;va .
x 2  2 xy  y 2  1600
5. rkcMnYnenaH ³
x 2  y 2  1600  2 xy 1
cMeBaH x  0, y  0 enaH x  y 2  0
tag x CacMnYnEdlRtUvrkenaH ³
x 2  2 xy  y 2  0 tambRmab;RbFan 2x  5  3
2 xy  x 2  y 2 2 x
 3  5
eyIgyk 1 CMnYskñúg 2 eyIg)an ³ 2
x
2 xy  1600  2 xy  8
2
2 xy  2 xy  1600
 2  8  2
x
4 xy  1600 2
xy  400 x  16
eXIjfa épÞRkLacmáarKW xy Edl xy  400
dUcenH cMnYnenaH CacMnYnEdlFMCag b¤esµI 16 .
dUcenH cmáarmanépÞRkLaFMbMputKW 400 m . 2

111
6. rkcenøaHqñaMeTAmuxeTot ³ 8. k> etIvisalTTYl)annieTÞs A Edr b¤eT ?
tag t CacenøaHqñaMeTAmuxeTot Edl t  0 tag x CaBinÞúénmuxviC¢acugeRkaymYyeTotEdl
smµtikmµ ³ «BukmanGayu 32 qñaM visalnwgTTYl)an ehIy 0  x  100 .
kUnmanGayu 12 qñaM visal)anRbLg 7 muxviC¢aehIyTTYl)anBinÞú
tambRmab;RbFan eyIg)anRbB½n§vismIkar ³ 84 , 93 , 78 , 87 , 89 , 70 nig 81
32  t  312  t  32  t  36  3t ]bmafa visalRbLg)annieTÞs A ³ eyIg)an

32  t  212  t 
b¤ 32  t  24  2t
  84  93  78  87  89  70  81  x
32  36  3t  t  90
 8
32  24  2t  t 582  x
 90
 4  2t t  2 8
   582  x  720
8  t t  8
x  720  582
eday t  0 enaHeyIg)an 0  t  8 x  138
dUcenH cab;BIbc©úb,nñrhUtdl;ticCag 8 qñaMeTot EttémøénBinÞú 0  x  100
eFVI[Gayu«Buk ticCagbIdg b:uEnþeRcIn dUcenH visalminGacTTYl)annieTÞs A eT .
CagBIrdgénGayurbs;kUn .
x> etIvisalTTYl)annieTÞs B Edr b¤eT ?
7. rkcMnYnRkdasR)ak; 500 ` EdlsuxGacman ³ ]bmafa visalRbLg)annieTÞs B ³ eyIg)an
tag x CacMnYnRkdasR)ak; 500 ` EdlsuxhUt)an 84  93  78  87  89  70  81  x
 80
naM[ suxhUt)anRkdas 1000 ` cMnYn 25  x 8
582  x
 80
Edl x KitCasnøwk 8
582  x  640
suxcg;hUt[)anR)ak; 17 500 ` y:agtic nigR)ak;
x  640  582
20 000 ` y:ageRcIn enaHeyIg)anTMnak;TMng ³ x  58
17500  500x  100025  x   20000
17500  500x  25000  1000x  20000
lT§plenHbBa¢ak;fa visalGacTTYl)annieTÞs B
17500  25000  500 x  20000 eday 0  x  100 niglT§pl x  58
17500  25000  500 x  20000  25000
 7500  500 x  5000
naM[ 58  x  100
 7500  500x  5000
 500

 500

 500
dUcenH visalGacTTYl)annieTÞs B ehIy
15  x  10 edIm,I[TTYl)annieT§s B visal
dUcenH RkdasR)ak; 500 ` EdlsuxGac caM)ac;RtUvyk[)anBinÞúmxu viC¢a
hUt)an mancMnYneRcInCag 10 snøwk cugeRkayticbMputKW 58 .
nigticCag 15 snøwk .
112
9. rkvimaRtGtibrmaénsYnenaH ³ kMNt;témøFbM MputéncMnYnessTaMgBIrtKña ³
11.

tag x CaRbEvgTTwgénsYnenaH ¬KitCa m¦ tag x CacMnYnessTI 1


naM[ RbEvgbeNþayKW 3x 15 eRBaH beNþay naM[ cMnYnessbnÞab;KW x  2
manrgVas;elIs 3dgénTTwgRbEvg 15m eday plbUkéncMnYnessTaMgBIrFMCag 72
edaysYnmanbrimaRtEvgbMputKW 94 m eyIg)an ³ eyIg)an x  x  2  72
2 x  3 x  15  94
2x  70 b¤ x  35
4 x  15  47
4 x  32 edaymancMnYnKt;ess x eRcInrab;minGs;EdlFM
x8 Cag cMnYn 35 enaHcMnYnKt;ess x kMNt;min)an
eday x CacMnYnKt;mantémøGtibrma enaH x  8
dUcenH BIrcMnYnKt;esstKñaFMCageK nigman
naM[ beNþay 3x 15  3 8 15  39 m
plbUkFMCag 72 KWminGackMNt;)an .
dUcenH vimaRtGtibrmaénsYnenaHKW TTwg
12. rkcm¶aypøÚv edIm,IkMu[QñÜlCYlelIsBIR)ak;kk; ³
manRbEvg 8m nigbeNþayman
tag x Cacm¶aypøÚvEdlRtUvebIkbr ¬KitCa km ¦
manRbEvg 39m .
edayR)ak;QñÜl 80 000 ` kñúgmYyéf¶ nig 400`
10. kMNt;RbePTsMbuRtEdlGaclk;)aneRcInbMput ³ kñúgcm¶aypøÚv 1km enaHeyIg)an ³
80 000  400x  400 000
tag x CacMnYnRbePTsMbuRt A ¬KitCasnøwk¦
400x  320 000
naM[ x  5 CacMnYnRbePTsMbuRt B eRBaH x  800
cMnYnRbePTsMbuRt A elIsRbePT B cMnYn 5snøkw dUcenH edIm,IkMu[QñÜlCYlelIsBIR)ak;kk;
cMnYnsMbuRtlk;)ay:ageRcInKW 37 snøkw eyIg)an ³ kñúgmYyéf¶GñkCYlminRtUvebIbrelI
x  x  5  37
2 x  42 cm¶aypøÚvelIsBI 800 km eLIy .
x  21
13. rkRbEvgRCug BC
naM[ cMnYnsMbuRtRbePT A lk;)aneRcInbMputKW
RtIekaN ABC man AB  12cm , AC  3cm
x  21 snøwk nigRbePT B lk;)aneRcIn
tamvismPaB cMeBaHRtIekaN ABC eyIg)an ³
bMputKW x  5  21 5  16 snøwk AB  AC  BC  AB  AC C
?
epÞógpÞat; ³ 2116  37 b¤ 37  37 Bit 12  3  BC  12  3
3

B
A
9  BC  15
9

dUcenH cMnYnsMbuRtlk;)aneRcInbMputKW eday BC CacMnYnKt; nigCaBhuKuNén 3


RbePT A cMnYn 21snøkw nig enaHnaM[ BC  12 cm
RbePT B cMnYn 16 snøwk . dUcenH rk)anRbEvgRCug BC  12 cm .
113
14. rkcMnYnKt;enaH ³
tag x CacMnYnKt;enaH
tambRmab;RbFan 2x  8 b¤ x  4
eXIjfa x CacMnYnKt;FMCagb¤esµI 4
naM[ x  4 , 5 , 6 , 7 , 8 , ...
dUcenH cMnnY Kt;Edlrk)anKW
x  4 , 5 , 6 , 7 , 8 , ... .
15.rkRbEvgbeNþayEdlbUNa RtUveRCIserIs ³
tag x CaRbEvgbeNþay ¬KitCa cm ¦
TTwgmanRbEvg 3 cm ehIyRkLaépÞsßitenA
cenøaHBI 15 cm eTA 24 cm
2 2

tambRmab;RbFan eyIg)anTMnak;TMng ³
15  3 x  24
15 3 x 24
 
3 3 3
5 x8
eday x CacMnYnKt; enaH x6, x7

dUcenH RbEvgbeNþayEdlbUNaRtUv
eRCIserIsKW 6 cm , 7 cm .

114

1. ។
3 2 1 6 1 7 9 7 10 10 10 3 5 7 2 5 1 6 4 8
1 2 7 1 10 3 5 5 3 7 6 2 10 6 4 3 4 5 5 7

10
5?
ឃ 9?

2. ។

x 0 1 2 3 4 5 6 7 8
f 144 195 130 80 58 45 24 6 3

5
7

3. ។

x 1 2 3 4 5 6
f 78 1270 680 320 150 133

x 3 ។
4.
25 22 30 45 28 51 30 32 34 33
18 30 15 20 24 17 24 41 38 27
16 25 28 31 41 28 26 15 25 19
32 36 21 18 34 41 53 25 42 41
8 5 15-20 ,
20-25 , …។

ឃ 25 ។

115
5. 45

10-15

15-20

20-25

25-30

30-35

35-40

40-45
5-10
0-5
4 5 9 12 15 14 10 8 6
45
15

6. 2

5 5-10 10-15 15-20 20-25 25-30 30-35 35-40 40-45

5 1 1 3 2 6 8 12 12

20 30

7. ។

0-5 5-15 15-35 35-45 45-55 55-70 70-85 85-100

1 104 65 45 70 135 195 210


12 3010 3645 1700 1175 1800 2400 1170

ឃ 35

DDCEE

116

1. eyIgmanTinñn½ykMhus GkçraviruT§énsisSmYyfñak; ³ g> sg;RkabssréneRbkg;eFobfyCaPaKry ³


3 2 1 6 1 7 9 7 10 10
f %
10 3 5 7 2 5 1 6 4 8
100%
1 2 7 1 10 3 5 5 3 7 100%
87.5%
6 2 10 6 4 3 4 5 5 7
80%
k> pþµúTinñn½yCataragbgðajBI eRbkg; eRbkg;ekIn
77.5%

65%

nigeRbkg;fy eRbkg;eFob eRbkg;eFobekInnigfy 60% 57%

42.5%

kMhus f f
f
f% 
f % 
f % 40% 32.5%

1 5 5 40 12.5 12.5 100 20%


17.5%
15%
2 4 9 35 10 22.5 87.5 12.5%
3 5 14 31 12.5 35.0 77.5
4 3 17 26 7.5 42.5 65.0 1 2 3 4 5 6 7 8 9 10 kMhus
5 6 23 23 15 57.5 57.5
6 4 27 17 10 67.5 42.5
7 6 33 13 15 82.5 32.5 2. eyIgmantaragsßiti éncMnYnkUnkñúgRKYsar ³
8 1 34 7 2.5 85.0 17.5 cMnYnkUn x 0 1 2 3 4 5 6 7 8
9 1 35 6 2.5 87.5 15.0
10 5 40 5 12.5 100 12.5
cMnYnRKYsar f 144 195 130 80 58 45 24 6 3

srub 40 k> begáIttaragbgðajBIeRbkg; eRbkg;ekInnigfy


x> etIfñak;enH mansisSb:unµannak; ? cMnYnkUn x eRbkg; f eRbkg;ekIn f eRbkg;fy f  

edayeRbkg;srubKW f  40 0
1
144
195
144
339
685
541
dUcenH fñak;eronenHmancMnYnsisS 40 nak; . 2 130 469 346
3 80 549 216
K> -sisSb:unµannak;mankMhsu 10 ? 4 58 607 136
5 45 652 78
edaysisSmankMhus 10 mancMnYneRbkg; f  5 6 24 676 33
dUcenH cMnYnsisSmankMhsu 10 man 5 nak; . 7
8
6
3
682
685
9
3
-ehIytamtarageRbkg;ekIn nigeRkg;ekIneFob srub 685

mansisS 23 nak;RtUvCa 57.5% EdlmankMhus x> -edayeRbkg;srub f  685


y:ageRcIncMnnY 5 . naM[ kñúgtMbn;enHman 685 RKYsar .
X> tamtarageRbkg;fy nigeRbkg;eFobfy -tamtarageRbkg;ekIn man 652 RKYsar Edlman
mansisS 6 nak; RtUvCa 15% EdlmankMhus kUny:ageRcIn 5 nak; nigtamtarageRbkg;fy
y:agticcMnYn 9 . man 9 RKYsarEdlmankUny:agtic 7 nak; .

117
K> sg;RkabssréneRbkg;ekIn ³ -tagTinñn½yxagelICaRkabssréneRbkg;eFob³
f%
f

682 685 50% 48.27%


700 652
676
607
600 40%
549

500 469
30%
400 25.85%
339
300 20%
12.16%
200
144
10%
100 5.70% 5.06%
2.94%

0 1 2 3 4 5 6 7 8 cMnYnkUn 1 2 3 4 5 6 cMnYnkUnsab;

3. taraglT§pléncMnYnkUnsøab;kñúgsRgÁam ³ x> RsaybBa¢ak;elIRkabTaMgBIr Rtg;ssrEdl


cMnYnkUnsøab; x 1 2 3 4 5 6 manGab;sIus x  3 ³
cMnYnRKYsar f
-Rtg;cMNucénRkabssrEdlman x  3 mann½y
78 1270 680 320 150 133

eyIgsg;taragbMENgEckeRbkg; eRbkg;eFob ³
cMnYnkUnsøab; x cMnYnRKYsar f eRbkg;eFob f%
fa man 680 RKYsarEdlmankUnsøab; 3 nak;enA
1 78 2.94 kñúgsRgÁam EdlRtUvCa 25.85% éncMnYnRKYsar
2 1270 48.27
3 680 25.85 TaMgGs;EdlmankUnsøab;kñúgsRgÁam .
4. eyIgmanTinñn½yGayukmµkrenAkñúgshRKasmYy ³
4 320 12.16
5 150 5.70
6 133 5.06 25 22 30 45 28 51 30 32 34 33
srub 2631 18 30 15 20 24 17 24 41 38 27

k> -tagTinñn½yxagelICaRkabssréneRbkg; ³ 16
32
25
36
28
21
31
18
41
34
28
41
26
53
15
25
25
42
19
41
f
1300 1270 k> sresrTinnñ ½yCafñak; mancenøaHfñak;esµI 5 ³
1200 Gayu eRbkg; eRbkg;ekIn eRbkg;eFob eRbkg;eFobekIn
1100
15-20 7 7 17.5 17.5
1000 20-25 5 12 12.5 30
900 25-30 9 21 22.5 52.5
800 30-35 9 30 22.5 75
700
35-40 2 32 5 80
680
40-45 5 37 12.5 92.5
600
45-50 1 38 2.5 95
500
50-55 2 40 5 100
400
320
srub 40

x> sg;GIusþRkam ³
300

200

¬rMlwk ³ GIusþÚRkamCaRkabssrenACab;²Kña¦
150
133
100 78

1 2 3 4 5 6 cMnYnkUnsab; tamtaragbMENgEckeRbkg; eyIgsg;GIusþÚRkam³


118
f
tamtaraglT§pl eyIgsg;)antaragbMENkEck
10
9 9 eRbkg;dUcxageRkam ³
8

eRbkg;eFobekIn

eRbkg;eFobfy
7

eRbkg;eFob
eRbkg;ekIn
eRbkg;fy
ry³eBl

cMnYn)al;
6
5 5

4
0-5 4 4 83 4.82 4.82 100
2 2 5-10 5 9 79 6.02 10.84 95.18
2
1
10-15 9 18 74 10.84 21.68 89.16
15-20 12 30 65 14.46 36.14 78.31
20-25 15 45 53 18.07 54.21 63.85
15 20 25 30 35 40 45 50 55 Gayu 25-30 14 59 38 16.87 71.08 45.78
30-35 10 69 24 12.05 83.13 28.92
K> -eyIgbMeBjtarageRbkg;ekIn nigeRbkg;eFob 35-40 8 77 14 9.64 92.77 16.88
40-45 6 83 6 7.23 100 7.23
ekIn dUckñúgtaragbMENgEckeRbkg;énsMNYr k> srub 83
-KUsRkabrbs;va ³ k> edayeRbkg;srubKW f  83
f% f
40
dUcenH kñúgry³eBl 45 naTI RkumkILakr)an
100% 40
30 32
37 38 100%
82.5% 95% e)aH)al;cUlTI cMnYn 83 RKab; .
75% 30
21
75% 80%
x> tamtarageRbkg;ekIn bgðajfakñúgry³eBl
15 naTIdMbUg RkumkILakr)ane)aH cUlTIcMnYn 18
52.5%
50% 20
12

25% 10
7 30%
17.5%
RKab; ehIyenAenAknøHem:ag cugeRkayRkum
kILakr)ane)aHcUlTI)ancMnYn 83-18 = 65 RKab;
15 20 25 30 35 40 45 50 55 Gayu EdlRtUvnwg 78.31% .
X> bkRsayelIRkabRtg;cMNucEdlman x  25 K> KUsRkabénBhuekaNeRbkg;ekIn ³
tamRkabtageRbkg;ekIn nigeRbkg;eFobekInKW 
f
Rtg;cMNucEdlmanGab;sIusesµI 25 mann½yfa 100
mankmµkry:ageRcIncMnYn 12 nak; EdlmanGayu 83
80 77
25 qñaM EdlRtUvnwg 30% éncMnYnkmµkrsrub . 69

60 59

5. eyIgmantaraglT§ple)aH)al;bBa©ÚlTIeTAtam 40
45

ry³eBlnImYy² kñúgefrevla 45 naTI ³


30

20 18
9
4
10-15

15-20

20-25

25-30

30-35

35-40

40-45
5-10
0-5
ry³eBl

5 10 15 20 25 30 35 40 45 naTI
cMnYn)al; 4 5 9 12 15 14 10 8 6

119
6. eyIgmantaragbgðajBIcMnYndgénkarpSayBaNiC¢kmµ
ry³eBlpSay 7. taraglT§plRbCaCnsøab; nigrgrbYstamGayu ³

85-100
15-35

35-45

45-55

55-70

70-85
5-15
Gayu

0-5
eRkam 5
¬naTI¦

10-15

15-20

20-25

25-30

30-35

35-40

40-45
5-10
Rb>C søab; 1 104 65 45 70 135 195 210

cMnYndg 5 1 1 3 2 6 8 12 12
Rb>C rbYs 12 3010 3645 1700 1175 1800 2400 1170

eyIgsg;)antaragbMENgEckeRbkg;dUcxageRkam³ k> eyIgrkkm<s;GIusþÚRkaménRb>Csøab; nigrbYs


Gayu Rb>C søab; km<s;GIu> Rb>C rbYs km<s;GIu>
ry³eBlpSay

0-5 1 0.2 12 2.4


eRbkg;ekIn
eRbkg;fy
5-15 104 10.4 3010 301
cMnYndg

15-35 65 3.25 3645 182.25


f
35-45 45 4.5 1700 170 h
eRkam 5 5 5 50 45-55
55-70
70
135
7
9
1175
1800
117.5
120
A
5-10 1 6 45 70-85 195 13 2400 160
10-15 1 7 44 85-100 210 14 1170 78
15-20 3 10 43 srub 825 14912
20-25 2 12 40
25-30
30-35
6
8
18
26
38
32
-sg;GIusþÚRkaménTin½yRbCaCnsøab; ³
h
35-40 12 38 24
40-45 12 50 12
15 14
srub 50 13

k> tamtaragbMENgEckeRbkg; eXIjfa eRbkg; 12


10.4
9
srubKW f  50 dUcenH kñúgeBll¶acenH eKpSay 9
7

BaNiC¢kmµ)an 50 dg eRbIeBlGs; 45 naTI . 6 4.5

x> -tamtarageRbkg;fy KWry³eBl 20 naTIcug


3.25
3

eRkay eKpSayBaNiC¢kmµ)an 38 dg . 0.2

15 35 45 55 70 85 100 Gayu
-tamtarageRbkg;ekIn ry³eBlticCag 30 0 5

-sg;GIusþÚRkaménTin½yRbCaCnsøab; ³
naTIdMbUg eKpSayBaNiC¢kmµ)an 18 dg .
K> KUsRkabénBhuekaNekIn nigfy ³ h
301
300
f

f 250
50 50
50
45 44 43 200
40 182.25
40 38 38 170
160
32 150
30 117.5 120
26
f 100
24 78
20
18
50
10 12 12
10
7 2.4
5 6
0 5 15 35 45 55 70 85 100 Gayu
5 10 15 20 25 30 35 40 45 naTI
120
x> begáIttarageRbkg; eRbkg;ekIn ³
Gayu Rb>C søab; eRbkg;ekIn Rb>C rbYs eRbkg;ekIn
0-5 1 1 12 12
5-15 104 105 3010 3022
15-35 65 170 3645 6667
35-45 45 215 1700 8367
45-55 70 285 1175 9542
55-70 135 420 1800 11342
70-85 195 615 2400 13742
85-100 210 825 1170 14912
srub 825 14912

K> rkcMnYnRbCaCnsøab; nigrbYs ³


tamtarageRbkg; RbCaCnsøab; = 825 nak; /
RbCaCnrgrbYs = 14912 nak;
naM[ RbCaCnsøab; nigrgrbYs mancMnYn
825 + 14912 = 15737 nak; .

X> tamtarageRbkg;ekIn RbCaCnEdlmanGayu


ticCag 35 qñaM søab;cMnYn 170 nak; nigmanrbYs
cMnYn 6667 nak; .

aaa

121

1.

22 22 21 21
1 22 23 22 23
2 20 19 21 20
3 22 23 23 23
4 27 27 26
5 20 19 22 20
6 23 24 24 24

2. kg ។
4 6 2 10 8 5 4 4
8 10 5 4 3 7 2 4
6 8 7 4 5 3 6 4

3.
x 16 18 19 20 21 30
f 1 4 9 3 2 1
4. ។

0 2 4 6 8 10 11 12 34 36

0 2 4 6 8 10 11 12 34 36

5 10 15 20 25 30 35

5.
0 0 0 1 3 3 1 1 1 3 0 0 4 3 3 1 1 0 5 6 2 2
3 2 2 4 1 6 5 6 0 1 1 2 3 5 1 3 2 1 1 1 1 2
1 3 3 4 1 1 0 3 1 1 1 3 3 6 0 0 3 4 1 3 2 3
0 2 4 1 3 4 1 5 6 3

122
6. ។

o
C
14o 14o 23o 32o 35o 30o 30o 29o 25o 22o 18o 15o
k> ។


7. 1 8 89 ។ 2
90 ។
8.

mm) 0-5 5-10 10-15 15-20 20-25 25-30 30-35
3 15 72 15 91 35 8

20 mm 35 mm ។
។ ។

9. ។
40-45

45-50

50-55

55-60

60-65

65-70

70-75

75-80

80-85

85-90
g

8 11 31 61 54 58 43 25 17 7


ឃ ។
10. 280 ។ ឃ
4 ។ ។
11. ។
145-150

150-155

155-160

160-165

165-170

170-175

175-180

cm

31 95 131 272 120 77 48




DDCEE

3 123

1. eyIgmantaragsikSacMnYnsisSdUcxageRkam ³ -cMeBaH fñak;TI 2 ³


fñak; cMnYnsisS eyIgmanTinñn½y 19 20 20 21
fñak;metþyü 22 22 21 21
÷mFümKW x  19  20 4 20  21  20
fñak;TI 1 22 23 22 23
fñak;TI 2 20 19 21 20 ÷tYén Me KW 4 2 1  2.5
fñak;TI 3 22 23 23 23
enaHnaM[ Me  20 2 20  20
fñak;TI 4 27 27 26
fñak;TI 5 20 19 22 20 ÷témøm:Ut KW Mo  20 ¬ maneRbkg;elIseK¦
20

fñak;TI 6 23 24 24 24
-cMeBaH fñak;TI 3 ³
rkmFüm emdüan nigm:UténcMnYnsisSkñúgmYyfñak;³ eyIgmanTinñn½y 22 23 23 23
-cMeBaH fñak;metþyü ³ ÷mFümKW x  22  23 4 23  23  22.75
eyIgmanTinñn½y 20 20 21 21
÷tYén Me KW 4 2 1  2.5
÷mFümKW x  20  20 4 21 21  20.5
enaHnaM[ Me  23 2 23  23
÷tYén Me KW 4 2 1  2.5
÷témøm:Ut KW Mo  23 ¬ maneRbkg;elIseK¦
enaHnaM[ Me  20 2 21  20.5
23

-cMeBaH fñak;TI 4 ³
÷Kµanm:UteT eRBaH 20 nig 21 maneRbkg;dUcKña eyIgmanTinñn½y 26 27 27
-cMeBaH fñak;TI 1 ³ ÷mFümKW x  26  273  27  26.67
eyIgmanTinñn½y 21 22 23 23
÷tYén Me KW 3 2 1  2 naM[ Me  27
÷mFümKW x  22  22 4 23  23  22.5
÷témøm:Ut KW Mo  27 ¬ maneRbkg;elIseK¦
÷tYén Me KW 4 2 1  2.5
27

-cMeBaH fñak;TI 5 ³
enaHnaM[ Me  22 2 23  22.5 eyIgmanTinnñ ½y 19 20 20 22
÷Kµanm:UteT eRBaH 22 nig 23 maneRbkg;dUcKña ÷mFümKW x  19  20 4 20  22  20.25
smÁal; ³ -mFüm tageday x ÷tYén Me KW 4 2 1  2.5
-emdüan tageday Me enaHnaM[ Me  20 2 20  20
-m:Ut tageday Mo ÷témøm:Ut KW Mo  20 ¬ maneRbkg;elIseK¦
20

124
-cMeBaH fñak;TI 6 ³ 3. eyIgmanTinñn½y dUcxageRkam ³
eyIgmanTinñn½y 23 24 24 24 x 16 18 19 20 21 30
f 1 4 9 3 2 1
÷mFümKW x  23  24 3 24  24  23.75 -KNnamFüm ³
÷tYén Me KW 4 2 1  2.5 x
16 1  18  4  19  9  20  3  21 2  30 1
1 4  9  3  2 1
enaHnaM[ Me  24 2 24  24 
391
20
 19.55

÷témøm:Ut KW Mo  24 ¬ maneRbkg;elIseK¦
24 -KNnaemdüan ³
2. eyIgmanlT§plbBa©úHm:as ¬kg¦ dUcxageRkam ³ eday tYénemdüanKW  202 1  10.5 ehIytYTI
4 6 2 10 8 5 4 4 10.5 sßitkñúgTinñn½y 19 dUcenH Me  19 .
8 10 5 4 3 7 2 4
-KNnam:Ut ³ edayTinñn½y 19 maneRbkg;elIseK
6 8 7 4 5 3 6 4
eyIgGacsresrsRmÜlCatarag dUcxageRkam ³ dUcenH Mo  19 .
m:as¬kg¦ eRbkg; f x  f f 
4. KNnamFüm emdüan nigm:Ut ³
1
2
1
2
1
4
1
3 k> tamRkaPic eyIgBRgay)anTinñn½y
3 2 6 5 2 , 4 , 4 , 6 , 6 , 6 , 6 , 6 , 6 , 8 , 8 , 10
2  4  2  6  6  8  2  10 72
4
5
6
3
24
15
11
14
-mFüm x
12

12
6
7
3
2
18
14
17
19 dUcenH KNna)an x  6 .
8
10
3
2
24
20
22
24
-KNnaemdüan ³
srub 24 126 tYénemdüanKW 122 1  6.5
-KNnamFüm ³ naM[ Me  6 2 6  6 dUcenH Me  6 .
11  2  2  3  2  4  6  5  3  6  3  7  2  8  3  10  2
x
1 2  2  6  3  3  2  3  2 -KNnam:Ut ³ eday 6 maneRbkg;eRcInCageK
126

24
 5.25 kg
dUcenH Mo  6 .
-KNemdüan ³ x> tamRkaPic eyIgBRgay)anTinñn½y
tYénemdüanKW  242  1  13 ehIytYTI 13 sßitenA 2 , 4 , 4 , 4 , 6 , 8 , 10 , 10 , 10 , 10 , 34 , 34
-mFüm x  2  4  3  6  812 10  4  34  2  136
kñúgfñak;m:as 5 kg dUcenH Me  5 kg . 12

-KNnam:Ut ³ dUcenH KNna)an x  11.33 .


eday m:as 4 kg mancMnYneRbkg;eRcInCageK -KNnaemdüan ³
tYénemdüanKW 122 1  6.5
dUcenH Mo  4 kg .
naM[ Me  8 210  9 dUcenH Me  9 .
125
-KNnam:Ut ³ eday 10 maneRbkg;eRcInCageK 6. k> KNnam:Ut ³
dUcenH Mo  10 . eday sItuNPð aB 14 C nig 30 C maneRbkg;
o o

K> tamRkaPic eyIgBRgay)anTinñn½y esµI 2 eRcInCageK


5 , 5 , 10 , 10 , 10 , 10 , 15 , 15 , 25 , 25 ,
30 , 30 , 30 , 30 , 35 , 35
dUcenH Mo  14 C nig Mo  30 C .
o o

-KNnamFüm ³ x> KNnasItuNðPaBmFüm ³


5  2  10  4  15  2  25  2  30  4  35  2 14  14  23  32  35  30  30  29  25  22  18  15
x
x 12
16 287
  23.92 C
dUcenH KNna)an x  20 .
o

12

-KNnaemdüan ³ rkcMnYnBinÞúEdl sux RtUvbegáIneEfmeTot ³


7.

tYénemdüanKW 162 1  8.5 tambRmab;RbFan ³


naM[ Me  15 2 25  20 dUcenH Me  20 . -BinÞú sux enAqmasTI1 KW 89 8  712 BinÞú
-ehIyBinÞúedIm,I[)anmFümPaK 90 KW
-KNnam:Ut ³ eday 10 nig 30 maneRbkg;eRcIn 90  8  720 BinÞú
CageK dUcenH Mo  10 nig Mo  30 . -naM[ cMnYnBinÞú EdlsuxRtUvbegáInKW
5. k> begáIttarageRbkg; eRbkg;ekIneFobCaPaKry 720  712  8 BinÞú
cMnYnkUn eRbkg; eRbkg;ekIn eRbkg;eFob eRbkg;ekIneFob
0 11 11 14.47 14.47 dUcenH sux RtUvbegáIncMnYn 8 BinÞúeTot .
8. k> begáIttarag eRbkg; eRbkg;ekIn nigfy ³
1 23 34 30.26 44.74
2 9 43 11.84 56.85
3 18 61 23.68 80.26 km<s; f f
p©itfñak; x x f
4 6 67 7.89 88.16 mm f
5 4 71 5.26 93.42 0-5 3 3 239 2.5 7.5
6 5 76 6.58 100 5-10 15 18 236 7.5 112.5
srub 76 10-15
15-20
72
15
90
105
221
149
12.5
17.5
900
262.5
x> KNnaemdüan ³ 20-25 91 196 134 22.5 2047.5
25-30 35 231 43 27.5 962.5
eday eRbkg;srub f  76 30-35 8 239 8 32.5 260

naM[ tYénemdüanKW 762 1  38.5 srub 239 4552.5

x> -tamtarageRbkg;fy cMnYnkUneb:ge)aH Edlman


tamtarageRbkg;ekIn tYTI 38.5 sßitenATinñn½y
km<s;y:agtic 20 mm mancMnYn 134 edIm .
cMnYnkUn 2 dUcenH Me  2 .
-tamtarageRbkg;fy cMnYnkUneb:ge)aH Edlman
km<s;y:agtic 35 mm KWKµan eRBaHtaragbgðaj
RtwmcMnYnkUneb:ge)aHEdlmankm<s;ticCag 35 mm
b:ueNÑaH .
126
K> -KNnam:Ut ³ tamtarageRbkg;srubKW f  315
edayfñak;km<s; 20-25 mancMnYnkUneb:ge)aHeRcIn dUcenH cMnYnTarke)ATwkedaHeKaKW 315 nak; .
CageK naM[ p©itfñak;enHKWCam:Ut x> rkm:asTwkedaHeKaEdlTarke)ACamFümkñúg1éf¶
 64.63 g dUcenH x  64.63 g
20357.5
dUcenH m:UtKNna)anKW Mo  22.5 . x
315
-KNnaemdüan ³ K> KNnam:Ut ³
edayeRbkg;srub f  239 edayfñak; 55-60 mancMnYneRbkg;eRcInCageK
naM[ tYénemdüanKW 239  119.5 naM[ m:UtCap©ti fñak;enH dUcenH Mo  57.5 .
2
tamtarageRbkg;ekIn tYTI 120 sßitenAkñúgfñak; X> KNnaemdüan ³
20-25 mann½yfa fñak;enHCafñak;emdüan tYénemdüanKW 315  157.5
2
eyIgnwgKNna Me tamviFIGaMETb:ULasüúg ³ eday eRbkg; 157.5 sßitenAkñúgfñak; 60-65
25  20119.5  105  20.8
Me  20 
196  105 naM[ Me  60  65  60165 157.5  111  64.31
 111

dUcenH emdüanKNna)an Me  20.8 . dUcenH KNna)an Me  64.31 .


-KNnamFüm ³ x
4552.5
 19.05
-bkRsaytamRkab ³
239

dUcenH KNna)an x  19.05 . 350


300

-eRbóbeFobemdüan nigmFüm ³ 250

eday Me  20.8 nig x  19.05 200


150

dUcenH eRbóbeFob)an Me  x . 100

50
9. k> rkcMnYnTarkEdle)ATwkedaHeKa kñúgmYyéf¶ ³
km<s; p©itfñak; x 40 45 50 55 60 65 70 75 80 85 90 TwkedaHeKa (g )
f 
f 
x f
mm
40-45 8
f
8 315 42.5 340
tamRkabénBhuekaNeRbkg;ekIn nigBhuekaN
45-50 11 19 307 47.5 522.5 eRbkg;fy RkaPicTaMgBIrRbsBVKñaRtg;cMNuc Edl
50-55 31 50 296 52.5 1627.5
55-60 61 111 265 57.5 3507.5 man Gab;sIusesµI 64.31 nigGredaenesµI 157.5 man
60-65 54 165 204 62.5 3375
65-70 58 223 150 67.5 3915 n½yfa Tinñn½ymanemdüan Me  64.31 EdlRtUvnwg
70-75
75-80
43
25
266
291
92
49
72.5
77.5
3117.5
1937.5 eRbkg; f  157.5 .
80-85 17 308 24 82.5 1402.5
85-90 7 315 7 87.5 612.5
srub 315 20357.5

127
10. k> begáIttarageRbkg; eRbkg;ekIn ³ -bkRsayelIRkab ³ eyIgKUsbnÞat;RsbG½kS
km<s; (cm) eRbkg; eRbkg;ekIn Gab;sIus edaykat;G½kSGredaenRtg;eRbkg; 387
145-150
150-155
31
95
31
126
rYceFVIcMeNalEkg éncMNucEdlCaRbsBVrvag
155-160
160-165
131
272
257
529
bnÞat; nigBhuekaNeRbkg;ekIn mkelIGk½ SGab;
165-170
170-175
120
77
649
726
sIus enaHeyIg)antémøkm<s; 162.39 cm
175-180 48 774 KWCaemdüan énTinñn½y.
srub 774

f 
x> sg;RkabénBhuekaNeRbkg;ekIn ³

800 774
726
700
649

600
529

500

400
387 

300
257

200
126
100
31 162.39

145 150 155 160 165 170 175 180 kMBs;sisS

K> KNnaemdüan ³
tYénemdüanKW 774
2
 387

tamtarageRbkg;ekIn tYénemdüansßitenAkñúgfñak;
160-165 mann½yfafñak; 160-165 Cafñak;emdüan

eyIgKNna Me tamviFIGaMgETb:LU asüúg ³


Me  160 
165  160387  257  162.39
529  257

dUcenH KNna)an Me  162.39 cm .

128

1. PHNOM PENH
P H OឬM PឬN
2. 1 21
1 21 ឬ

3. 1 4 // //
4.

5.

182 52 234
254 338 592

1200 760
440
6. 3 A, B, C

A 1420 74 182 1676


B 846 26 122 994
C 570 41 60 671
2836 141 364 3341
A
BឬC
A
BឬC
A BឬC
7. 4
HHHH , HTHH , THHH , TTHH
HHHT , HTHT , THHT , TTHT
HHTH , HTTH , THTH , TTTH
HHTT , HTTT , THTT , TTTT

129
H T
H H
T
8.
2
5
9.
0 1 2 3 4 5 6 7 8
6 1 15 12 24 18 16 10 7

4 6
6
10. 6 4 2
ABCD
X Y

11. 6 A, B, C, D, E F 3
4

12. H
13. 12
1
3

14. 3 2

15. A1
A1 , A2 , B B
A2
A
16. 15
3
5

2
3

130

1. cMnYnGkSrTaMgGs;énBakü PHNOM PENH 2. etIRbU)abéncMnYnEdlsux nigesArkesµIKña b¤eT ?


man 9 GkSr naM[ cMnYnkrNIGac  9 cMnYnenAcenøaHBI 1 dl; 21 KWman 21 cMnYn
eKcab;ykGkSmYyBIBaküenHedayécdnü ³ naM[ cMnYnkrNIGac  21
k> rkRbU)abEdlcab;)anGkSr P ³ -sux kMBugrkcMnYnessenAcenøaHBI 1 dl; 21
eday GkSr P mancMnYn 2 cMnYnessenAcenøaHBI 1 dl; 21 KWman 11 cMnYn
naM[ cMnYnkrNIRsbcab;)anGkSr P KW  2 naM[ P(cMnYness)  11 21
dUcenH P(P)  cMcMnnYnYnkrNI R sb
krNIG ac

2
9
. -esA kMBugrkcMnYnKUenAcenøaHBI 1 dl; 21
cMnYnKUenAcenøaHBI 1 dl; 21 KWman 10 cMnYn
x> rkRbU)abEdlcab;)anGkSr H ³
naM[ P(cMnYnKU)  10
eday GkSr H mancMnYn 2 21

naM[ cMnYnkrNIRsbcab;)anGkSr H KW  2 dUcenH RbU)abéncMnYnEdlsux nigesA


dUcenH P(H)  cMcMnnYnYnkrNI R sb

2
. kMBugnwgrk KWminesµIKñaeT .
krNIG ac 9
3. rkRbU)abénRKab;LúkLak;ecjBIelx 1dl;elx 4
K> rkRbU)abEdlcab;)anGkSr O b¤ M ³
eday RKab;Lkú Lak;manmux 6 enaHkrNIGac  6
eday GkSr O mancMnYn 1
ehIymuxBIelx 1dl; 4 man 4 muxCakrNIRsb
naM[ cMnYnkrNIRsbcab;)anGkSr O KW  1
naM[ P(1dl;elx 4)  64  23
ehIy GkSr M mancMnYn 1
naM[ cMnYnkrNIRsbcab;)anGkSr O KW  1 dUcenH P(1 dl;elx 4)  2
3
.
eK)an P(H b¤ M)  PO  PM   19  19
4. etIRbU)abKb;RBYjelIépÞqUt nigminqUtesµIKñab¤eT ?
dUcenH P(H b¤ M)  92 tag a nig b CavimaRtrbs;ctuekaNEkg
X> rkRbU)abEdlcab;)anGkSr P b¤ N ³ -épÞqUt  12 ab a
// //

eday GkSr N mancMnYn 2 b


naM[ cMnYnkrNIRsbcab;)anGkSr N KW  2 -épÞminqUt 1 b 1 b 1
  a     a    ab
2 2 2 2 2
eK)an P(P b¤ N)  PP  PN   92  92 eday épÞqUt nigépÞminqUtesµIKña dUcenH RbU)abKb;
dUcenH P(P b¤ N)  94 . RBYjelIépÞqUt nigminqUtk¾esµIKñaEdr .
131
5.eyIgmanTinñn½ydUcxageRkam ³ K> rkcMnYncMNtrfynþEdleKRtUveRtómTukCamun
GñkenAkñúgRkug GñkenACayRkug srub -rkcMnYnGñkenAkñúgRkugCiHrfynþ
rfynþ 182 52 234 RbU)abénGñkrs;enAkñúgRkugCiHrfynþKW
m:UtU 254 338 592 P
182

91
182  254 218
k> rkRbU)abénGñkeFVIdMeNIredaym:UtU edayePJóvenAkñúgRkugGeBa©IjcUlrYm 760 nak;
edaym:UtU nigrfynþmansrub 234  592  826 naM[ GñkenAkñúgRkugCiHrfynþmancMnYn
naM[ cMnYnkrNIGacesµInwg 826 91
218
 760  318 nak;

ehIy m:UtUmancMnYn 592 CacMnYnkrNIRsb -rkcMnYnGñkenACayRkugCiHrfynþ


eyIg)an P(GñkCiHm:UtU)  826
592 296

413
 0.72 RbU)abénGñkrs;enACayRkugCiHrfynþKW
52 2
P 
dUcenH RbU)anénGñkeFVIdMeNIredaym:UtUKW 52  338 15

P(GñkCiHm:Ut)U 
296
 0.72 . edayePJóvenACayRkugGeBa©IjcUlrYm 440 nak;
413
naM[ GñkenACayRkugCiHrfynþmancMnYn
x> ¬rebobTI1¦rkRbU)abénGñkeFVIdMeNIredayrynþ 2
15
 440  59 nak;

rfynþmancMnYn 234 CacMnYnkrNIRsb -eyIg)an cMnYnGñkeRbIR)as;rfynþTaMgGs;KW


ehIy cMnYnkrNIGacesµInwg 826 ¬rkxagelI¦ 318  59  377 nak;
eyIg)an P(GñkCiHrfynþ)  826  413  0.28
234 117
naM[ rfynþman 377 eRKÓg
dUcenH RbU)anénGñkeFVIdMeNIredayrfynþKW ehIy RtUvkarcMNt 377 kEnøg
P(GñkCiHrfynþ) 
117
 0.28 . dUcenH eKRtUveRtómcMNtrfynþ 377 kEnøg
413
6. eyIgmantaraglT§plénkarsÞabsÞg;sMeLg ³
¬rebobTI2¦rkRbU)abénGñkeFVIdMeNIredayrynþ KaMRT minKaMRT Kµaneyabl; srub
edayRbU)abénGñkeFVIdMeNIredaym:UtUCaRbU)ab A 1420 74 182 1676
bMeBjnwgRbU)abénGñkeFVIdMeNIredayrfynþ B
C
846
570
26
41
122
60
994
671

naM[ P(GñkCiHm:UtU) + P(GñkCiHrfynþ) = 1 srub 2836 141 364 3341

b¤ P(GñkCiHrfynþ) =1 P(GñkCiHm:UtU) k> rkRbU)abénsMeLgEdlminKaMRTebkçCn A


 1
296 sMeLgminKaMRTsrubmancMnnY 3341 CakrNIGac
413

117
 0.28
sMeLgminKaMRT A mancMnYn 74 CakrNIRsb
413 naM[ P(sMeLgminKaMRTA)  3341 74
 0.0221

dUcenH P(GñkCiHrfynþ)  117413


 0.28 . dUcenH P(sMeLgminKaMRTA)  0.0221 .
132
x> rkRbU)abénsMeLgEdlminKaMRTebkçCn B b¤ C 7. eyIgmanRBwtþikarN_Gacénkare)aHkak; 4 dgKW ³
tamtaraglT§plénkaresÞabsÞg; eyIg)an ³ HHHH , HTHH , THHH , TTHH
HHHT , HTHT , THHT , TTHT
P(sMeLgminKaMRT B b¤ C) 
26 41
 HHTH , HTTH , THTH , TTTH
3341 3341
67 HHTT , HTTT , THTT , TTTT
  0.0201
3341 naM[ cMnYnkrNIGacesµInwg 16
dUcenH P(sMeLgminKaMRT B b¤ C)  0.0201 . k> rkRbU)abEdle)aH)anGkSr H BIr
K> rkRbU)abénkarKµaneyabl;cMeBaHebkçCn A krNIe)aH)anGkSr H BIrman ³
TTHH , HTHT , THHT 
tamtaraglT§plénkaresÞabsÞg; eyIg)an ³  man 6 CakrNIRsb
HTTH , THTH , HHTT 
P(sMeLgKµaneyabl; A) 
182
3341
 0.0545 naM[ P( H BIr)  166  83  0.375
dUcenH P(sMeLgKµaneyabl; A)  0.0545 . dUcenH P( H BIr)  83  0.375
X> rkRbU)abénkarKµaneyabl;cMeBaHebkçCn B b¤ C x> rkRbU)abEdle)aH)anGkSr T BIrmuneK
tamtaraglT§plénkaresÞabsÞg; eyIg)an ³ krNIe)aH)anGkSr T BIrmuneKman ³
P(sMeLgKµaneyabl; B b¤ C) 
122 60

3341 3341 TTHH , TTHT , TTTH , TTTT man 4 krNI


182
3341
 0.0545 naM [ P( T BIrmuneK) 
4 1
  0.25
16 4

dUcenH P(sMeLgKµaneyabl; B b¤ C)  0.0545 dUcenH P( T BIrmuneK)  14  0.25


g> eRbóbeFobRbU)abKaMRT A nigRbU)abKaMRT B b¤ C K> rkRbU)abEdle)aH)anGkSr H mYymuneK
tamtaraglT§plénkaresÞabsÞg; eyIg)an ³ krNIe)aH)anGkSr T BIrmuneKman ³
P(sMeLgKaMRT A) 
1420
 0.4250 HHHH , HTHH , HHHT , HTHT 
3341  man 8 krNI
HHTH , HTTH , HHTT , HTTT 
P(sMeLgKaMRT B b¤ C) 
846 570

3341 3341 naM[ P( H mYymuneK)  168  12  0.5
1416
  0.4238
3341
enaH P(sMeLgKaMRT A)  P(sMeLgKaMRT B b¤ C) dUcenH P( H mYymuneK)  12  0.5

dUcenH eyIgeRbóbeFob)an RbU)abénsMeLg X> rkRbU)abEdle)aH)anGkSr H TaMgbYn ³


KaMRTebkçCn A FMCagRbU)abénsMeLg e)aH)anGkSr H TaMgbYn man 1 krNI KW HHHH
ebkçCn B b¤ C naM[ P( H TaMgbYn)  161  0.0625
dUcenH P( H TaMgbYn)  161  0.0625
133
g> rkRbU)abEdle)aH)anGkSr T mYyy:agtic ³ x> rkRbU)abEdlbuKÁlikenaH )anQb;y:agticmYyéf¶
RbU)abe)aH)anGkSr T mYyy:agtic CaRbU)ab edaycMnYnbuKÁlik )anQb;y:agticmYyéf¶ KW ³
bMeBjnwgRbU)abe)aH)anGkSr H TaMgbYn 1  15  12  24  18  16  10  7  103 nak;

naM[ P( H TaMgbYn) + P( T mYyy:agtic) = 1 ¬GacKNnaedayxøIKW 109  6  103 nak;¦


enaH P( T mYyy:agtic) = 1 P( H TaMgbYn) eyIg)an P(Qb;y:agticmYyéf¶)  109 103

1 15
 1   0.9375
16 16 dUcenH P(Qb;y:agticmYyéf¶)  109
103

dUcenH P( T mYyy:agtic)  16
15
 0.9375 .
K> rkRbU)abEdlbuKÁlikenaH)anQb;cenøaHBI 4eTA 6éf¶
8. rkRbU)abEdlcab;)anXøIBN’ s ³ edaycMnYnbuKÁlik )anQb;cenøaHBI 4 eTA 6éf¶KW ³
edayRbU)abEdlcab;)anXøBI N’s CaRbU)abbMeBj 24 18 16  58 nak; CacMnYnkrNIRsb
nwgRbU)abEdlcab;)anXøBI N’exµA eyIg)an P(Qb;cenøaHBI 4eTA 6éf¶)  109
58

naM[ P(XøIexµA) + P(XøIs) = 1


Taj)an P(XøIs) = 1 P(XøIexµA) dU c enH P(Qb;cenøaHBI 4eTA 6éf¶) 
58
109
tambRmab; P(XøIexµA)  52 K> rkRbU)abEdlbuKÁlikenaH)anQb;eRcInCag 6éf¶
eyIg)an P(XøIs) = 1 52  53 edaycMnYnbuKÁlik )anQb;eRcInCag 6éf¶KW ³
10  7  17 nak; CacMnYnkrNIRsb
dUcenH eyIgrk)an P(XøIs) = 53 .
eyIg)an P(Qb;eRcInCag 6éf¶)  109
17

9. eyIgmantaragéncMnYnGvtþmanrbs;buKl
Á ik ³
dUcenH P(Qb;eRcInCag 6éf¶)  109 17
cM>éf¶Gvtþman 0 1 2 3 4 5 6 7 8

cM>buKÁlik 6 1 15 12 24 18 16 10
eyIgmanXøIexµA 4 nigXøIs 2
7
10.
buKÁliksrub 6 115 12  24 18 16 10  7  109 k> rkRbU)abEdlcab;)anXøBI N’sTaMgBIr
nak; CacMnYnkrNIGac cMnYnkrNIGacKW 4  2  6
k> rkRbU)abEdlbuKÁlikenaH )anQb;y:ageRcInbIéf¶ eyIg)an P(ss) = P(s)×P(s¼s)
edaycMnYnbuKÁlik )anQb;y:ageRcInbIéf¶ KW ³ 2 1 2
   
1

6  1  15  12  34 nak; CacMnYnkrNIRsb
6 5 30 15

eyIg)an P(Qb;y:ageRcInbIéf¶)  109 34 dUcenH RbU)abEdlcab;)anXøIBN’s


TaMgBIrKW P(ss)  151
dUcenH P(Qb;y:ageRcInbIéf¶)  109 34

134
x> rkRbU)abEdlcab;)anXøIBN’exµAmYyy:agtic 12. rkRbU)abEdle)aH)anmux H TaMgbI
edayRbU)abEdlcab;)anXøBI N’exµAmYyy:agtic kak;manmux H nig T enaHlT§plGac)anKW ³
CaRbU)abbMeBjnwgRbU)abcab;)anXøIsTaMgBIr kak;TI1 kak;TI2 kak;TI3 lT§pl
eyIg)an P(ss) + P(exµAmYyy:agtic) = 1 H
H
T
HHH
HHT
naM[ P(exµAmYyy:agtic) = 1 P(ss) H
T
H HTH
T HTT
1 14
 1  H THH
15 15 H
T THT
T
H TTH
dUcenH P( exµAmYyy:agtic )
14
15
T
T TTT
lT§plsrubmancMnYn 8 CacMnYnkrNIGac
11. eyIgmanXøIexµA 4 nigXøIs 2 mux HHH manEt 1 Kt; CacMnYnkrNIRsb
k>
dUcenH P( H TaMgbI)  18
13. k> rkcMnnY énXøIBN’exµA
XøITaMgGs;mancMnYn 12 RKab; CacMnnY krNIGac
tag x CacMnYnXøIBN’exµA EdlCakrNIRsb
naM[ RbU)abcab;)anXøIBN’exµAKW P(exµA) = 12x
Et RbU)abEdlcab;)anXøIBN’exµAesµInwg 13
x 1
eyIg)an 
12 3
b¤ 3x  12 naM[ x  4
dUcenH cMnYnXøIBN’exµAmancMnYn 4 RKab;
x> -¬rebobTI1¦ rkRbU)abEdlcab;)anXøBI N’s
cMnYnXøIBN’s  12  4  8 CacMnYnkrNIRsb
dUcenH s
P( ) 
8 2

12 3

-¬rebobTI2¦ rkRbU)abEdlcab;)anXøIBN’s
eday P(s) CaRbU)abbMeBjnwg P(exµA)
naM[ P(s) = 1  P(exµA)  1  124  128
dUcenH P(s)  128  23
135
14.rkRbU)abEdleKerIs)annarITaMgBIrnak; x> rkcMnYnXøIRkhmEdlRtUvbEnßmcUlkñúgfg;
bursmancMnYn 3 nak; nignarImancMnYn 2 nak; tag y CacMnYnXøIRkhmEdlRtUvbEnßmcUlkñúgfg;
naM[ cMnYnkrNIGacKW 3+2=5 naM[ cMnYnXøsI rubKW 15  x  y  25  y
eyIg)an P(narIBInak;)  P(narI)  P(narI¼narI) cMnYnxøIRkhmekIn)an 15  y
2 1 1
  
5 4 10
naM[ RbU)abcab;)anXøIRkhmfµIKW P  1525  yy
dUcenH P( narIBInak;)  101 EtRbU)abEdlcab;)anXøIRkhmfµIesµInwg 23
eyIg)an 1525  yy  23
15.rkRbU)abEdleKKb;)anGkSr A TaMgBIelIk 225  y   315  y 
eKKb;RBYjBIrdgepSgKña minTak;TgKña 50  2 y  45  3 y
fasEckCabIEpñk enaHcMnYnkrNIGacesµInwg 3 50  45  3 y  2 y
5 y
GkSr A man 2 KW A nig A CacMnYnkrNIRsb
1 2

eyIg)an P(ATaMgBIrelIk)  P(A) × P(A) dUcenH cMnYnXøIBN’RkhmEdlRtUvbEnßm



2 2 4
  cUlkñúgfg;KWmancMnnY 4 RKab; .
3 3 9

dUcenH RbU)abEdlKb;)anGkSr A TaMgBIelIk


KW P(ATaMgBIrelIk)  94
16.k> KNnacMnnY XøIBN’exovkñúgfg;enaH
tag x CacMnYnXøIBN’exovEdlRtUvrk
enaHcMnYnXøITaMgGs;KW 15  x CacMnYnkrNIGac
naM[ P(Rkhm)  1515 x
Et RbU)abEdlcab;)anXøIBN’RkhmesµInwg 53
eyIg)an 1515 x  53
15  5
15  x 
3
x  25  15
x  10

dUcenH cMnnY XøIBN’exovmancMnYn 10 RKab;

136
៩ ចម្ងាយរវាងពីរចំណុច

1. គណនាចម្ងាយរវាងពរី ចំណុច ដែលម្ងនកូអរដោដណែូចខាងដរោម។

k> 9 , 2 nig 6 , 8 x> 5 ,  3 nig  2 , 4


K> 2 , 5 nig 0 , 0 X>  6 ,  3 nig 2 , 1
g> 8 , 7 nig 8 ,  6 c> 3 , 12  nig 7 , 12 
q> 100 ,  203  nig 97 ,  200  C>  3 , 8 nig  4 , 7
2. ចុងទាំងពីរននអងកតផ្
់ ចិតរងវងម ួ ម្ងនកូអរដោដនដរៀងគ្នា 3 , 1 និង  2 , 5 ។ ចូរគណនាោំរងវងរ់ ច
់ យ ួ សង់រងវងដ់ នះ។
3. ចូរគណនាររដវងអងកតរ់ រូងននចតុដោណ PQRS ដែល P 2 , 3 , Q5 , 5 , R6 ,  6 នង
ិ S  3 ,  3 ។
4. ចូររង្ហាញថារតដី ោណKLM ដែល K 2 , 8 , L10 , 11 នង
ិ M 5 , 0  ជារតដី ោណសមបាត ។
5. ចូរគណនាររម្ងរតននរត
ិ ីដោណ ABC ដែល A3 , 7  , B5 , 2 និង C  7 , 3 ។
ចូររង្ហាញថា ABC ជារតីដោណដកង រួចគណនានផ្ៃរកឡាររស់វា ។
6. ចូររកតនមល ី យចម្ងាយរវាងពរី ចំណុចដែលម្ងនកូអរដោដន 4 , 2 នង
y ដែើមបឱ្ ិ 4 , y  ដសមើនង
ឹ 5 ឯកតា ។
7. ចូររកចំណុច C  x , 1 កាុងោរែង់រី 1 ដែើមបឱ្
ី យរតីដោណដែលម្ងនកំពូល A1 , 1 , B4 , 7  និង C  x , 1
ជារតីដោណសមបាតដែល AB  AC ។
8. កំពូលននរតដី ោណមួយម្ងនកូអរដោដន  3 , 2 , 9 , 2 នង
ិ 3 , 10  ។
ក. ចូរគណនាររដវងររុងទាំងរីននរតីដោណ។
ខ. ចូររបារ់ររដេររតីដោណដនះ។
9. កំពូលននរតដី ោណមួយម្ងនកូអរដោដន  1 ,  2 , 5 ,  2 ,  1 , 4 នង
ិ 5 , 4  ។
ក. ចូរគណនាររដវងររុង និងររដវងអងកតរ់ រូងននចតុដោណ។
ខ. ដតើដគសនាិោានែូចដមេចចំដបាះចតុដោណដនះ ?
10. កូអរដោដនកំពូលររស់ចតុដោណមួយម្ងន 2 ,  1 , 6 , 4 , 2 , 9 នង
ិ  2 , 4 ។
ក. ចូរគណនាររដវងររុង និងររដវងអងកតរ់ រូងននចតុដោណ។
ខ. ចូររបារ់ររដេរននចតុដោណដនះ។
11. ចតុដោណមួយម្ងនកំពូល  2 ,  3 ,  3 , 1 , 5 ,  1 នង
ិ 4 , 3 ។ ដតច
ើ តុដោណដនះម្ងនរាងជាអវ?

12. ចូរគណនាកូអរដោដនននចំណុចកណ្ត  
េ លនន AB រួចដៅចំណុចកាុងតរមុយអរតូណរដមតាមករណីនម
ី យ
ួ ៗែូចខាងដរោម
ក. A5 , 8 និង B 5 , 12  ខ. A 5 , 2 និង B 5 , 14  ។
13. ដរររើ រ
ូ មនេកូអរដោដនននចំណុចកណ្ត
េ លអងកត។
់ ចូរគណនាកូអរដោដនចំណុចកណ្ត
េ លននអងកត់ AB កាុងករណី  
នម
ី យ
ួ ៗែូចខាងដរោម ។
ក. A 3 ,  5 និង B7 ,  5 ខ. A 8 ,  6 និង B4 , 2
គ. A3t  5 ,  7  និង Bt  7 , 7  , t  1 ឃ. Aa , b  នង
ិ Ba , c  ។
14. ចូរគណនាកូអរដោដនននចំណុចកណ្ត
េ ល M ននអងកតរ់ វាងពរី ចំណុច A 3 ,  1 នង
ិ B5 , 7  ។
ចូរគណនាររដវង AM និង BM ។
15. េ លននអងកត់ AB  ដែល A 2 , 2 ។ ចូរគណនាកូអរដោដនននចំណុច B ។
ដគឱ្យ M 1 ,  2  ជាចំណុចកណ្ត

137
16. រងវងម
់ យ ិ F  2 , 6  ។ អងកតផ្
ួ ម្ងនផ្ចត ់ ចិតរងវងដ់ នះម្ងនចំណុចចុងមួយរតង់ V 3 , 8 ។ គណនាកូអរដោដនននចំណុច
T ដែលជាចំណុចចុងម្ងាងដរៀតននអងកត់ TV  រួចសង់រងវងដ់ នះ។
17. កូអរដោដនននកំពូលររដលឡូរោមមួយដសមើ 1 , 1 , 2 ,  3 ,  3 ,  1 និង  2 ,  5 ។ ចូរគណនាកូអរ
ដោដនននចំណុចកណ្ត
េ លននអងកតរ់ រូងទាំងពីរ។ ដតើដគសនាិោានបានយ៉ាងែូចដមេច?
18. ចតុដោណមួយម្ងនកំពូល E 5 , 3 , F 5 ,  5 , G 3 ,  3 នង
ិ H  1 , 5 ។ ចូរគណនាកូអរដោដននន
ចំណុចកណ្ត
េ ល A , B , C និង D ននររុង EF  , FG  , GH  និង EH  ។ ដតើចតុដោណ ABCD ម្ងន
រាងជាអវី ?
19. ចតុដោណដកងមួយកំណត់ដោយចំណុច A3 , 2 , B9 , 2 , C 9 , 6 នង
ិ D3 , 6 ។ ចូររាយរញ្ជាក់ថា
ចតុដោណដែលកំណត់ដោយ ចំណុចកណ្ត
េ លននររុងទាំងរួន ររស់ចតុដោណដកងខាងដលើជាចតុដោណដសមើ។
20. ចតុដោណមួយម្ងនកំពូល P 3 ,  2 , Q 1 , 5 , R3 , 1 និង S 5 ,  4 ។ ដរើ A , B , C និង D
ជាចំណុចកណ្ត      
េ លដរៀងគ្នាននររុង PQ , QR , RS នង
ិ SP ។  
ចូររង្ហាញថា ABCD ជាររដលឡូរោម ។
21.  
ដគម្ងនអងកត់ P1 P2 ដែល P1  x1 , y1  និង P2 x2 , y 2  ។ ចូរគណនាកូអរដោដនននចំណុច A និង B ជា
 
ចំណុចដែលដចកអងកត់ P1 P2 ជារីដផ្ាកដសមើៗគ្នា ។
 x  x2 y1  y 2 
22. ដគម្ងនចំណុច P1 x1 , y1  , Pm  1 , ិ P2 x2 , y 2  ។
 នង
 2 2 
ចូររង្ហាញថា P1 Pm  Pm P2  P1 P2 ។ y

23. ដគឱ្យ A1 , 0  និង B3 , 2 រួចសង់រងវងដ់ ែលម្ងនអងកតផ្


់ ចិត AB ។
ក. ចូររកកូអរដោដនននផ្ចិតរងវងដ់ ែលតាងដោយចំណុច I។  B
5 
ខ. M ជាចំណុចមួយដែលម្ងនកូអរដោដន  , y  ចូរកំណត់ y
2  

ដែើមបឱ្
ី យ M ឋត
ិ ដៅដលើរងវង់ រួចរញ្ជាក់ដតើ M ម្ងនរីតាង
ំ រានា
ុ ម ន? 
x O A x

y

DDCEE

138

1. KNnacm¶ayrvagBIrcMNuc EdlmankUGredaen ³ X>  6 ,  3 nig 2 , 1


k> 9 , 2 nig 6 , 8 ebI d Cacm¶ayrvagBIrcMNucenH eyIg)an ³
ebI d Cacm¶ayrvagBIrcMNucenH eyIg)an ³ d 2  62  1  32
d 6  92  8  22  82  4 2

  32  6 2  64  16  80

 9  36  16  5  4 5

 45 dUcenH KNna)ancm¶ay d  4 5 ÉktaRbEvg


 95
3 5 g> 8 , 7 nig 8 ,  6
dUcenH KNna)ancm¶ay d  3 5 ÉktaRbEvg ebI d Cacm¶ayrvagBIrcMNucenH eyIg)an ³
d 8  82   6  72
x> 5 ,  3 nig  2 , 4  0 2   13
2

ebI d Cacm¶ayrvagBIrcMNucenH eyIg)an ³  132  13


d  2  52  4  32
dUcenH KNna)ancm¶ay d  13 ÉktaRbEvg
  7 2  7 2
 49  49 c> 3 , 12  nig 7 , 12 
 2  49
ebI d Cacm¶ayrvagBIrcMNucenH eyIg)an ³
7 2
d 7  32  12  122
dUcenH KNna)ancm¶ay d  7 2 ÉktaRbEvg  42  02  42  4

K> 2 , 5 nig 0 , 0 dUcenH KNna)ancm¶ay d  4 ÉktaRbEvg


ebI d Cacm¶ayrvagBIrcMNucenH eyIg)an ³ q> 100 ,  203  nig 97 ,  200 
d 0  5  0  2
2 2

ebI d Cacm¶ayrvagBIrcMNucenH eyIg)an ³


  52   22
d 97  100 2   200  203 2
 25  4
  32  32
 29
 9  9  18  3 2
dUcenH KNna)ancm¶ay d  29 ÉktaRbEvg dUcenH KNna)ancm¶ay d  3 2 ÉktaRbEvg
139
C>  3 , 8 nig  4 , 7 3. KNnaRbEvgGgát;RTUgénctuekaN PQRS
ebI d Cacm¶ayrvagBIrcMNucenH eyIg)an ³ eyIgman kUGredaenéncMNcu dUcxageRkam ³
d  4  32  7  82 
Q
5
  12   12 P
 3
 11
 2
3 6
2
dUcenH KNna)ancm¶ay d  2 ÉktaRbEvg 5

 3
2. -KNnaRbEvgkaMrgVg; ³ S

edaycugTaMgBIrénGgát;p©itmankUGredaen 6 R

3 , 1 nig  2 , 5 ebI d CaRbEvgGgát;p©it enaH -KNnaRbEvgGgát;RTUg PR ³


d  2  32  5  12 PR  6  22   6  32
 8 2   9 
2
  52  4 2
 64  81  145
 25  16
 41 dUcenH RbEvgGgát;RTUg PR  145 ÉktaRbEvg
naM[ kaMrgVg; R  d2  41
ÉktaRbEvg
2 -KNnaRbEvgGgát;RTUg QS ³
dUcenH kaMrgVg;manRbEvg R  41
ÉktaRbEvg QS   3  52   3  52
2
  82   82
-sg;rgVg;enH ³  64  64  8 2

edIm,I[gayRsYlsg;rgVg;enH eyIgrkp©itrbs;rgVg; dUcenH RbEvgGgát;RTUg QS  8 2 ÉktaRbEvg


p©it I  3 2 2 , 1 2 5  b¤ I  12 , 3 
4. bgðajfa RtIekaN KLM CaRtIekaNsm)at ³
eyIgman K 2 , 8 , L10 , 11 nig M 5 , 0
 L10 , 11
 2 , 5
11

1  
I  , 3 8 K 2 , 8
 2 

3 , 1

M 5 , 0

O 2 5 10

140
-KNnaRbEvg KL ³ naM[ brimaRténRtIekaN ABC KW ³
10  22  11  82 P  AB  AC  BC
KL 
 29  2 29  145
 8 2  32
 3 29  145
 64  9  73 ÉktaRbEvg
-KNnaRbEvg KM ³ dUcenH P  3 29  145 ÉktaRbEvg
KM  5  22  0  82
-bgðajfa ABC CaRtIekaNEkg
 32   8
eday AB  AC   29   2 29 
2
2 2 2 2

 9  64  73 ÉktaRbEvg
 29  4  29  145 >
edayRtIekaN KLM man KL  KM
ehIy BC   145  145
2 2

dUcenH KLM CaRtIekaNsm)at enaHeyIg)an AB  AC  BC


2 2 2

5. eyIgman A3 , 7 , B5 , 2 nig C 7 , 3 tamRTWsþIbTBItaK½r AB  AC  BC


2 2 2

enaH ABC CaRtIekaNEkgRtg; A


A3 , 7
7  dUcenH bgðaj)anfa ABC CaRtIekaNEkg
C  7 , 3
 3
-KNnaépÞRkLaénRtIekaN ABC ³
 B5 , 2 eday ABC CaRtIekaNEkgRtg; A eyIg)an
AB  AC
7 O 3 5 S ABC 
2
29  2 29
-KNnabrimaRténRtIekaN ABC ³ 
2
 29 ÉktaépÞRkLa
AB  5  32  2  72
dUcenH KNna)an S  29 ÉktaépÞRkLa
2 2   5
ABC

2

 4  25  29 ÉktaRbEvg 6. rktémøén y EdleFVI[cm¶ayénBIcMNucesµI 5 ³


AC   7  32  3  72 eyIg)an 4  4   y  2  5
2 2

  102   42  y  22  5


 100  16  116 >  y  22  25
y  2   25
 4  29  2 29 ÉktaRbEvg y  2  5
 7  52  3  22 y  2  5 y  5 2 y  7
BC  naM[  y  2  5   y  5  2   y  3
  
  122  12
 144  1  145 ÉktaRbEvg dUcenH témørk)anKW y  7 , y  3 .
141
7. rkcMNuc C x , 1 kñúgkaRdg;TI 1 ³ x> R)ab;RbePTénRtIekaNenH ³
eyIgman A1 , 1 nig B4 , 7 ehIyman edayRtIekaN ABC enHman
AB  4  12  7  12 AC  BC  10 ÉktaRbEvg dUcKña
 4  12  7  12 dUcenH ABC CaRtIekaNsm)aTmankMBUl A .
 32  6 2  9  36  45
9. eyIgmankMBUlctuekaNtagdUcxageRkam ³
AC  x  12  1  12  x  12
eday AB  AC enaHeyIg)an ³ C  1 , 4 D5 , 4 
x  12  45  

x  12  45
x  1   45
x  3 5  1 O

cMeBaH x  3 5  1  0 minyk eRBaH C x , 1  


A 1 ,  2 B5 ,  2 
enAkñúgkaRdg;TI 1 enaH x  0
dUcenH cMNcu C 3 5  1 , 1 . k> -KNnaRbEvgRCug ³
AB  5  12   2  22 É>RbEvg
 62  6
8. k> KNnaRbEvgRCugTaMgbIénRtIekaN ABC ³ AC   1  1  4  2  6  6 É>RbEvg
2 2 2

eyIgtagkMBUlTaMgbIénRtIekaNenaHeday ³ CD  5  1  4  4  6  6 É>RbEvg
2 2 2

A 3 , 2 , B9 , 2 nig C 3 , 10  eyIg)an


BD  5  5  4  2  6  6 É>RbEvg
2 2 2

AB  9  32  2  22 dUcenH RbEvgRCugKNna)an ¬ÉktaRbEvg¦


 12 2  12 ÉktaRbEvg
AB  6 / AC  6 / CD  6 / BD  6
AC  3  32  10  22 -KNnaRbEvgGgát;RTUg ³
 62  82  100  10 ÉktaRbEvg AD  5  12  4  22  62  62  6 2

BC  3  92  10  22 BC  1  52  4  22   62  62  6 2

  62  82  100  10 ÉktaRbEvg dUcenH RbEvgGgát;RTUgKNna)anKW


dUcenH KNna)anRbEvgRCugTaMbIénRtIekaNKW AD  6 2 ÉktaRbEvg / BC  6 2 ÉktaRbEvg

AB  12 ÉktaRbEvg x> snñdi æancMeBaHctuekaNenH ³


AC  10 ÉktaRbEvg
edayRbEvgRCug AB  AC  BD  CD  6
BC  10 ÉktaRbEvg
nigRbEvgGgát;RTUg AD  BC  6 2
dUcenH ctuekaNenHKWCakaery:agR)akd .
142
10. eyIgtagkUGredaenénkMBUlrbs;ctuekaNeday³ 11. etIctuekaNenH manragCaGVI ?
A2 ,  1 , B6 , 4 , C 2 , 9 nig D 2 , 4 eyIgtagkUGredaenénkMBUlrbs;ctuekaNeday³
C 2 , 9
A 2 ,  3 , B 3 , 1 , C 5 ,  1 nig D4 , 3

6
4 D4 , 3

B6 , 4
B 3 , 1 2
 4 

D 2 , 4

2 2 O 2 4

2 C 5 ,  1
2 O 2 4 6 

A2 ,  1
A 2 ,  3

k> -KNnaRbEvgRCug ³ eyIgKNnaRbEvgRCug nigRbEvgGgát;RTUg ³


AB   3  22  1  32   12  4 2  17
AB  6  22  4  12  4 2  52  41
BD  4  52  3  12   12  4 2  17
AD   2  22  4  12   42  52  41
AC  5  22   1  32  7 2  2 2  53
BC  2  62  9  42   42  52  41
BD  4  32  3  12  7 2  2 2  53
CD   2  22  4  92   42   52  41
AD  4  22  3  32  6 2  6 2  6 2
dUcenH KNna)anRbEvgRCug ¬ÉktaRbEvg¦KW BD  5  32   1  12  82   22  2 17
AB  41 , AD  41 , BC  41 , CD  41 edayctuekaNenHman ³
-RbEvgRCug AB  BD  17 , AC  BD  53
-KNnaRbEvgGgát;RTUgénctuekaN ³
AC  2  22  9  12  10 2  10
-RbEvgGgát;RTUg AD BD
BD   2  62  4  42   82  8
tamRTwsþI ³ ctuekaNEdlmanRCugesµKI ñaBIr² ehIy
dUcenH KNna)anRbEvgGgát;RTUgKW ³ manGgát;RTUgminesµKI ña vaCaRbelLÚRkam .
AC  10 , BD  8 ¬ÉktaRbEvg¦
dUcenH ctuekaNenHmanragCa RbelLÚRkam .
x> R)ab;RbePTénctuekaNenH ³ 12. KNnakUGredaencMNuckNþalén AB rYcedA
edayRbEvgRCug AB  AD  BC  CD  41 kñúgtRmúyGrtUNrem tamkrNInImYy² ³
RbEvgGgát;RTUg AC  BD k> A5 , 8 nig B5 , 12 
tamRTwsþI ³ ctuekaNEdlmanRCugTaMgbYnesµIKña tag I CacMNuckNþalén A nig B eyIg)an ³
 5  5 8  12 
ehIyGgát;RTUgminesµKI ña vaCactuekaNesµI . I ,  b¤ I 5 , 10 
 2 2 

dUcenH ctuekaNenHKWCa ctuekaNesµI . dUcenH cMNcu kNþal A nig B KW I 5 , 10  .


143
-edAcMNuckñúgtRmúyGetUNrem ³ x> eyIgman A 8 ,  6 nig B4 , 2
 B5 , 12 
tamrUbmnþ M  x 2 x , y 2 y 
A B A B

I 5 , 10 
 8 4 6 2 
naM[ b¤ M  2 ,  2
10
M , 
 2 2 
8  A5 , 8

dUcenH kUGredaencMNuckNþalKW M  2 ,  2
6

4
K> eyIgman A3t  5 ,  7 nig Bt  7 , 7
tamrUbmnþ M  x 2 x , y 2 y 
A B A B

naM[ M  3t  52 t  7 ,  72 7  b¤ M 2t  6 , 0


O 2 4 6

dUcenH cMNcu kNþalKW M 2t  6 , 0


x> A 5 , 2 nig B 5 , 14 
tag J CacMNuckNþalén A nig B eyIg)an ³ X> eyIgman Aa , b nig Ba , c
J
  5  5 2  14 
,  b¤ J  5 , 8 tamrUbmnþ M  x 2 x , y 2 y 
A B A B

 2 2 

dUcenH cMNcu kNþal A nig B KW J  5 , 8 . naM [ M


 a  a b  c  b¤  b  c 
 2
,
2 
 M a ,
 2 

-edAcMNuckñúgtRmúyGetUNrem ³ dUcenH cMNcu kNþalKW M  a , b 2 c 


B 5 , 14   14 14. -rkkUGredaenéncMNuckNþal M ³
12
10 eyIgman A 3 , 1 nig B5 , 7
J  5 , 8  8
6
naM[ M   32 5 ,  12 7  b¤ M 1 , 3
4
A 5 , 2  2 dUcenH kUGredaencMNuckNþalKW M 1 , 3 .
6 4 2 O
-KNnaRbEvg AM nig BM ³
13. KNnakUGredaencMNuckNþalén AB³ AM  1  3  3  1  4 2  4 2  2 2
2 2

k> eyIgman A 3 ,  5 nig B7 ,  5 BM  1  5  3  7  


2 2
 42   42 2 2

tamrUbmnþ M  x 2 x , y 2 y 


A B A B dUcenH RbEvgKNna)anKW ³
naM[  3 7 55
M ,  b¤ M 2 ,  5 AM  2 2 ÉktaRbEvg
 2 2 
BM  2 2 ÉktaRbEvg
dUcenH kUGredaencMNuckNþalKW M 2 ,  5
144
15.KNnakUGredaenéncMNuc B ³ 17. KNnakUGredaencMNuckNþalGgát;RTUgTaMgBIr ³
tag Bx , y  CacMNucEdlRtUvrk
B B tagkMBUlTaMgbYnenaHénRbelLÚRkameday
eyIgmancMNuc A 2 , 2 nigcMNuc M 1 ,  2 A1 , 1 , B2 ,  3 , C  3 ,  1 nig D 2 ,  5

eday M CacMNuckNþalénGgát; AB 2

  2  xB A1 , 1

 2 1
 2  x B  2
eyIg)an ³  enaH  4 2
 2  y B  2 2  y B  4
O 2

 2 C  3 ,  1
2
 xB  2  2  xB  4
naM[  b¤  
 y B  4  2  y B  6 4
B2 ,  3

dUcenH kUGredaencMNuc B4 ,  6 . D 2 ,  5


16.KNnakUGredaenéncMNuc T ³ -tag M CacMNuckNþalénGgát;RTUg AD


tag T x , y  CacMNucEdlRtUvrk
T T
naM[ M  1 2 2 , 1 2 5  b¤ M   12 ,  2 
eyIgmancMNuc V 3 , 8 nigp©it F  2 , 6 dUcenH cMNcu kNþal AD KW M   12 ,  2 
eday F CacMNuckNþalénGgát; TV 
 xT  3
 2  2  xT  3  4
-tag N CacMNuckNþalénGgát;RTUg BC
eyIg)an ³  enaH 
 yT  8  6
 2
 yT  8  12 naM[ N  2 2 3 ,  12 3  b¤ N   12 ,  2 
 xT  4  3  xT  7
naM[  b¤  dUcenH cMNcu kNþal BC KW N   12 ,  2 
 yT  12  8  yT  4

dUcenH kUGredaencMNuc T  7 , 4 . -edaycMNuc M   12 ,  2  dUcnwg N   12 ,  2 


-sg;rgVg;enH ³ dUcenH eyIgsnñidæan)anfa cMNucTaMgBIrRtYtsIuKña

V 3 , 8
18. KNnakUGredaenéncMNuckNþal A , B , C , D ³
8 
eyIgmankMBUlénctuekaN E5 , 3 ; F 5 ,  5 ;
F  2 , 6
 6 G 3 ,  3 nig H  1 , 5 ³


T  7 , 4 4 eday A , B , C , D CacMNuckNþalerogKñaén
2
EF  ; FG  , GH  nig EH  eyIg)an ³
 55 35
A ,  b£ A5 ,  1
6 4 2 O 2  2 2 

145
53 53
B , b¤ B1 ,  4
 19. Rsayfa ctuekaNEdlkMNt;)an CactuekaNEkg
 2 2 

C , b¤ C 2 , 1
  3 1  3  5 

eyIgman A3 , 2 ; B9 , 2 ; C9 , 6 ; D3 , 6
 2 2 
 5 1 3  5 
tag K , L , M , N CacMNuckNþalerogKñaén
D
 2
, b¤ D2 , 4
2 

RCug AB , BC  , CD , AD
dUcenH kUGredaenKNna)anKW ³
8
cMNuckNþal EF  KW A5 , 1 D3 , 6 M 6 , 6 C 9 , 6
  
cMNuckNþal FG KW B1 ,  4 6

cMNuckNþal GH  KW C 2 , 1 4 N 3 , 4   L9 , 4

cMNuckNþal EH  KW D2 , 4 2   


A3 , 2 K 6 , 2 B9 , 2

-bBa¢ak;BIragctuekaN ABCD ³ O 2 4 6 8
H  1 , 5

D2 , 4
enaHeyIg)an ³
  39 2 2
4
E 5 , 3

K
 2
,
2 
 b¤ K 6 , 2

99 26
C  2 , 1

2
L
 2
,
2 
 b¤ L9 , 4

93 66
4 2 O 2 4 M
 2
,
2 
 b¤ M 6 , 6

A5 ,  1
 33 2 6
2
N
 2
,
2 
b¤ N 3 , 4


G 3 ,  3
4  eyIgKNnaRbEvgRCugénctuekaN KLMN ³
B1 ,  4

F 5 ,  5 KL  9  62  4  22  32  2 2  13
eyIgKNnaRbEvgRCugénctuekaN ABCD ³ LM  6  92  6  42   32  2 2  13
AB  1  52   4  12   42   32  5 MN  3  62  4  62   32   22  13
AD  2  52  4  12   32  52  34 KN  3  62  4  22   32  2 2  13
CD  2  22  4  12  4 2  32  5 eday ctuekaN KLMN manRbEvgRCug ³
BC   2  12  1  42   32  52  34 KL  LM  MN  KN  13 dUcKña

edayRbEvg AB  CD  5 nig AD  BC  34 tamRTwsþI ³ ctuekaNEdlmanRCugTaMgbYnesµI²Kña


tamRTwsþI ³ ctuekaNEdlmanRCugesµIKñaBIr² vaCa enaHvaKWCactuekaNesµI
RbelLÚRkam dUcenH ctuekaNEdlkMNt;)anBIcMNuckNþal
dUcenH ctuekaN ABCD CaRbelLÚRkam . énRCugctuekaNEkgvaCa ctuekaNesµI
146
20. bgðajfa ABCD CaRbelLÚRkam ³ 21. KNnakUGredaenéncMNuc A nig B ³
eyIgman kMBUlénctuekaN P 3 ,  2 ; Q1 , 5 eyIgman P x , y  nig P x , y 
1 1 1 2 2 2

R 3 , 1 nig S 5 ,  4 Edlman A , B , C , D P1 x1 , y1 



CacMNuckNþalerogKñaRCug PQ ; QR  ; RS  //

A ?  //
nig SP enaHeyIg)ankUGredayen ³ B ? 

//

P2 x2 , y2 
  3  1  2  5  b¤ A  2 , 3 
A ,   
 2 2   2 O
 1 3 5 1 
B ,  b¤ B1 , 3
 2 2  tag Ax , y  nig Bx , y  CacMNucRtUvrk
A A B B
 3
C
 2
, b¤
 3  5 1 4 
2 
 C 4 ,  
 2 eday A nig B Eck P P  CabIEpñkesµIKña 1 2

53 42
D , b¤ D1 ,  3
 eyIg)an P A  AB  BP1 1
 2 2 
naM;[ A CacMNuckNþalén P B
-eyIgKNnaRbEvgRCugénctuekaN ABCD ³ 1

eyIg)an x  x 2 x , y  y 2 y 1
A
1 B
A
1 B

Q 1 , 5

4
ehIy B CacMNuckNþalén AP  2
B1 , 3
 eyIg)an x  x 2 x , y  y 2 y 2
B
A 2
B
A 2

 3 2
A  2 ,  
 2  R3 , 1
yk 1 CMnYskñúg 2 enaHeyIg)an ³
 x1  xB
  x2
4 2 O 2 4 6 x A  x2 2 x  x B  2 x2
 x    1
 3
 C 4 ,  
 B
2 2 4
 2  2 
 y1  y B
P 3 ,  2  y2
 S 5 ,  4  y  y A  y2  2  1
y  y B  2 y2
 4 D1 ,  3 


B
2 2 4
 x1  2 x2
2
4 xB  x1  xB  2 x2 x 
AB  1  22   3  3   9
9 3 5
 enaH 

 B

3 b¤ 3
 2 4 2 4 y 
 B 1 B y  y  2 y 2 
 y  1 2 y2
y
2 

B
3
AD  1  22    3  3   9
81

117
 2 4 2 yk 3 CMnYskñúg 1 ³
2
 x  2 x2
BC  4  12    3  3   9
81

117
 x1  1
3 4 x  2 x2 2 x1  x2
 2  4 2 xA   1 
 2 3 2 3
2 
CD  1  42    3  3   9
9 3 5
  y  1
y  2 y2
4 x  2 x2 2 y1  y 2
 2 4 2 y  1 3  1 


A
2 3 2 3
eday AB  CD  3 25 , AD  BC  117 2
dUcenH kUGredaen A 2 x 3 x 1 2 y1  y 2 
 2
,
ctuekaNenHmanRCugesµIKñaBIr vaCaRbelLÚRkam 3 
 x  2 x2 y1  2 y 2 
dUcenH ctuekaN ABCD CaRbelLÚRkam . kUGredaen B 1
 3
,
3

147
22. bgðajfa P P  P P  P P ³ 1 m m 2 1 2 23. rkkUGredaenénp©itrgVg;tageday I ³
eyIgman P x , y  / P x , y  nig
1 1 1 2 2 2 eyIgman A1 , 0 nig B3 , 2
x x y y 
P  m
1
,  naM[eyIg)anRbEvg ³
2 1 2 sg;rgVg;EdlmanGgát;p©it AB
 2 2 
y
 x  x2  y  y2
2 2
 
P1 Pm   1  x1    1  y1  5 7
M 2  ,1  
 2   2  2 2 

 x  x2  2 x1   y  y 2  2 y1 
2 2

B3 , 2
2
  1   1 
 2   2 


1
x2  x1 2  1  y 2  y1 2 1 
I 2 , 1
4 4

1
x2  x1 2   y 2  y1 2 x O A1

2 2 
x
5 7
M 1  ,1  
x  x2   y  y2 
2 2
 2 
Pm P2   x2  1    y2  1  2

 2   2  y

 2 x  x1  x2   2 y 2  y1  y 2 
2 2

  2
 2
 
  2


edayp©it I CacMNuckNþalénGgát;p©it AB

1
x2  x1 2  1  y 2  y1 2
eyIg)an I  1 2 3 , 0 2 2  b¤ I 2 , 1
4 4

1
x2  x1 2   y 2  y1 2
dUcenH KNna)ankUGredaenp©it I 2 , 1 .
2
eyIgKNnaplbUk P P  Pm P2 ³ x> kMNt; y edIm,I[ M  52 , y  zitenAelIrgVg;
1 m

P1 Pm  Pm P2 
1
x2  x1 2   y2  y1 2  edIm,I[ M zitenAelIrgVg; luHRtaEt MI  AI
2 2
1
x2  x1  2
  y 2  y1 
2 Et MI   2  52   1  y   14  1  y 
2 2

2
P1 Pm  Pm P2  x2  x1 2   y2  y1 2 AI  2 12  1  02  11  2
Et P P  x  x    y  y 
1 2 2 1
2
2 1
2
eyIg)an 1
 1  y   2
2

4
dUcenH bgðaj)anfa P P  P P  P P . 1 m m 2 1 2
1
4
 1  y   2
2

-GñkGaceFVItamrebobmü:ageTotKW ³ 1  y 2  2  1
4
eday P  x 2 x , y 2 y  KWCacMNuckNþal
m
1 2 1 2
1 y  
7
2
énBIrcMNuc P x , y  nig P x , y 
1 1 1 2 2 2 y  1
7
2
naM[ P P  P P  12 P P
1 m m 2 1 2
dUcenH kMNt;)an y  1  27 , y  1  27
enaH P P  P P  12 P P  12 P P  P P
1 m m 2 1 2 1 2 1 2
ehIy M manTItaMgBIrenAelIrgVg;KW
dUcenH bgðaj)anfa P P 1 m  Pm P2  P1 P2 . 5
M  , 1 
7 5
 , M , 1
 
7

2 
.
2 2  2

148
១០

1.

k> a  2 ,  4 , 1 x> a


2
3
, 1 , 5 

2.
k> 3 , 1 , 5 , 4 x> 3 , 0 ,  3 ,  6
K>  2 , 1 ,  4 ,  4 X> 1 , 4 ,  4 ,  2
3.
k>  6,1 , 6 ,  2 x> 3,  4 ,  9 , 2
K>  2,0 , 1 ,  8 X>  4.2,  4 ,  2.2 ,6
g>  2.5, 1 ,  0.5 ,1 c> 0,5 , 3 ,  2.5
4. 0.8%
80% 2000
2008
5. 1992 38%
1992 15.5% 2007

6. m P

k> m  1 , P0 , 2 x> m  1 , P1 , 0 K> m  2 , P3 , 1


X> m  53 , P 1 ,  4 g> m  0 , P2 , 1 c> m   32 , P0 , 3
7.

k> 2x  3 y  11 x> 4x  8 y 1  0 K> y   23 x  1


8.
k> y  x  4 , x  y  5  0 x> y  2x  3 , x  2 y 1  0
K> 3x  y  4  0 , y  2  3x  1 X> 2 y  5x  6 , 5x  2 y 1  0
9.

k> y  3x 1 1
, y x
3
x> y  2 x  5 , x  2y  6  0

K> y  5x  4 , x  5y 1  0

149
10.
k> y  3x  4 5 , 1
x> 3x  2 y  5  0  2 , 4
K> 2x  2 y  9  0  5 
  , 0
 3 
X> x  5 y  6  0 0 , 0
11.
k> y  12 x  4 5 , 0
x> x  y  5  0 0 , 0
K> 8x  3y  1  0  1 , 4
 2
X> y   x  6  4 , 
 3
12. d1 A 3 , 5 2
.
d2: y   m 1 x  2 m d2 d1
m d2 d1 ឬ m
B  6 , 0 A B
yy
13. A  0 , 6 , B  3 , 0 C  6 , 0
AB AC
D C AB
D B AC
14. A  6 , 2 B  2 ,  2
 AB
C  2 , 6 CAB
C2 C AB
C2 M  5 , 3
15. A  6 , 0 , B  6 , 0 C  3 , 9
ABC
ABC
16.  a , 6  2 , a 4x  2 y  b a b
17. 1973
47% 1990 38%
x x 0 1970

3 150
១០

1. sg;bnÞat;kat;tammYycMNuc nigmanemKuNR)ab;Tis 2. rkemKuNR)ab;TisénbnÞat;kat;tamBIrcMNuc ³


k> eyIgman a  2 nigcMNuctageday A 4 , 1
eyIgsg;cMNuc A 4 , 1 rYcKUs 1 ÉktaecjBI A *emKuNR)ab;TisénbnÞat;kat;tamBIrcMNuc
y y
Ax , y  nig Bx , y  KW a  B A
RsbG½kS ox tamTisedAviC¢man nigKUs 2 Ékta A A B B
x x B A

RsbnwgG½kS oy tamTisedAviC¢man enaHeyIg)an


k> manBIrcMNuctageday A3 , 1 , B5 , 4
cMNuc B rYcKUsbnÞat;kat;tamcMNuc A nig B ³
emKuNR)ab;TisénbnÞat;Edlkat;tam A nig B
kMNt;eday a  yx  xy  54 31  32
B A

B A

dUcenH emKuNR)ab;TisénbnÞat;KW a  32 .
B x> manBIrcMNcu tageday C3 , 0 , D 3 ,  6
A emKuNR)ab;TisénbnÞat;Edlkat;tam C nig D
kMNt;eday a  yx  xy   63  30   66  1
D C

D C

x> eyIgman a


2
nigcMNuctageday A1 , 5
3 dUcenH emKuNR)ab;TisénbnÞat;KW a  1 .
eyIgsg;cMNuc A1 , 5 rYcKUs 3 ÉktaecjBI A
RsbG½kS ox tamTisedAviC¢man nigKUs 2 Ékta K> BIrcMNuctageday I  2 , 1 , J  4 ,  4
RsbG½kS oy tamTisedAGviCm¢ an enaHeyIg)an emKuNR)ab;TisénbnÞat;Edlkat;tam I nig J
cMNuc B rYcKUsbnÞat;kat;tamcMNuc A nig B ³ kMNt;eday a  yx  xy   44  12   23  23
J I

J I

dUcenH emKuNR)ab;TisénbnÞat;KW a  23 .

A
X> manBIrcMNuctageday M 1 , 4 , N 4 ,  2

emKuNR)ab;TisénbnÞat;Edlkat;tam M nig M
B

kMNt;eday a  yx  xy  4214  36  2
N M

N M

dUcenH emKuNR)ab;TisénbnÞat;KW a  2 .
151
3. rksmIkarénbnÞat;Edlkat;tamBIrcMNcu ³ K>manBIrcMNuctageday A2 , 0 , B1 ,  8
k>manBIrcMNuctageday A 6 ,1 , B6 ,  2 tag M x , y  CacMNucenAlIbnÞat; AB
naM[ emKuNR)ab;TisénbnÞat; AB KW naM[ emKuNR)ab;TisénbnÞat; AM nig AB esµIKña
a
 2 1  3

6   6 12

1
4
eyIg)an yx  xy  yx  xy
A B A

A B A

tag M x , y  CacMNucenAlIbnÞat; AB y 0 80



x2 1 2
naM[ emKuNR)ab;TisénbnÞat; AM nig AB esµIKña y

8
eyIg)an x y 16   14 x  2 1
y  8x  2
y  8 x  16
y 1  
1
x  6
4
1 3
y   x  1
dUcenH smIkarbnÞat;rk)anKW y  8x  16 .
4 2
1
y x
1 X>BIrcMNuctageday A 4.2,  4 , B 2.2 , 6
4 2
tag M x , y  CacMNucenAlIbnÞat; AB
dUcenH smIkarbnÞat;rk)anKW y   14 x  12 . naM[ emKuNR)ab;TisénbnÞat; AM nig AB esµIKña
x>manBIrcMNuctageday A3 ,  4 , B 9 , 2 eyIg)an yx  xy  yx  xy
A B A

A B A
y   4  6   4 
naM[ emKuNR)ab;TisénbnÞat; AB KW 
x   4.2   2.2   4.2 
2   4 6 1
a   y  4 10
 9  3  12 2 
x  4. 2 2
tag M x , y  CacMNucenAlIbnÞat; AB y  4  5 x  4.2 
y  5 x  21  4
naM[ emKuNR)ab;TisénbnÞat; AM nig AB esµIKña y  5 x  17
eyIg)an y x 34   12
dUcenH smIkarbnÞat;rk)anKW y  5x  17 .
y  4   x  3
1
2
1 3
y   x 4
g> manBIrcMNuctageday A2.5, 1 , B0.5 ,1
2
1
2
5
tag M x , y  CacMNucenAlIbnÞat; AB
y  x
2 2 naM[ emKuNR)ab;TisénbnÞat; AM nig AB esµIKña
dUcenH smIkarbnÞat;rk)anKW y   12 x  52 . eyIg)an yx  xy  yx  xy
A B A

A B A

y   1 1   1

smÁal; ³ bnÞat;BIrRtYtelIKña enaHbnÞat;TaMgBIr x  2 .5 0 .5  2 .5
y 1 2

manemKuNR)ab;TisesµIKña . x  2 .5  2
y  1    x  2 .5 

152
y   x  2.5  1
y   x  1 .5
5. rksmIkarEdlTak;ngw GñkCk;)arI nigcMnYnqñaM ³
tag y  ax  b CasmIkarbnÞat;RtUvrk
dUcenH smIkarbnÞat;rk)anKW y   x  1.5 . Edl x CacMnYnqñaM nig y CacMnYnGñkCk;)arI
c> manBIrcMNcu tageday A0 , 5 , B3 ,  2.5 ÷GñkCk;)arImancMnYn 38% enAqñaM 1992 ³
tag M x , y  CacMNucenAlIbnÞat; AB naM[ y  38% nig x  1992
naM[ emKuNR)ab;TisénbnÞat; AM nig AB esµIKña eyIg)an y  ax  b kat;cMNuc A1992 , 0.38
eyIg)an yx  xy  yx  xy
A B A
÷GñkCk;)arImancMnYn 15.5% enAqñaM 2007 ³
naM[ y  15.5% nig x  2007
A B A
y  5  2.5  5

x0 30
y  5  7.5
eyIg)an y  ax  b kat;cMNuc B2007 , 0.155 

x 3 tag M x , y  CacMNucenAlIbnÞat; y  ax  b
y  4  2.5 x b¤
y  2.5 x  4
naM[ emKuNR)ab;TisénbnÞat; AM nig AB esµIKña
dUcenH smIkarbnÞat;rk)anKW y  2.5x  4 . eyIg)an yx  xy  yx  xy
A B A

4. -sresrsmIkarTak;Tg;ng w karfycuHcMnnY ksikr ³


A B A

y  0.38 0.155  0.38



smIkarEdlRtUvsresrmanrag y  ax  b x  1992 2007  1992
y  0.38  0.225

÷eday cMnYnksikrfycuHCalMdab;eTAtamGRta x  1992 15
y  0.38
0.8% kñúgmYyqñaM enaHeyIg)an a  0.8%  0.015
x  1992
÷ehIyenAqñaM 2000 ksikrmancMnYn 80% y  0.38  0.015 x  1992
y  0.015x  29.88  0.38
ebItag x CaqñaM nig y CacMnYnksikr enaHeyIg)an
y  0.015x  30.26
cMeBaH x  2000 naM[ y  80%
-smIkar y  ax  b GacsresrCa ³ dUcenH smIkarrk)anKW y  0.015x  30.26
80%  0.8%  2000  b
0.8  0.008  2000  b 6. sresrsmIkarbnÞat; sg;bnÞat; rYcrkcMNucRbsBV³
0.8  16  b k> -eyIgman m  1 , P0 , 2
b  16.8
naM[ a  m  1 , x  0 , y  2
dUcenH sresr)ansmIkar y  0.008x  16.8 . bnÞat;RtUvrkmanrag y  ax  b
-eRbIsmIkar):an;sµancMnnY ksikrenAqñaM 2008 ³ eyIg)an 2  1 0  b naM[ b  2
enAqñaM 2008 mann½yfa x  2008 enaHeyIg)an ³ dUcenH smIkarbnÞat;RtUvrkKW y  x  2 .
y  0.008 2008  16.8  0.736 b¤ y  73.6%
-sg;bnÞat; y  x  2 tamrebobemIl
dUcenH enAqñaM 2008 ksikrmancMnYn 73.6% . m  1 , P0 , 2 ¬elOnCag eRbItaragtémøelx¦
153
-sg;bnÞat; y  2x  5 tamrebobemIl
y  x2

emKuNR)ab;Tis m  2 , P3 , 1

y  2 x  5

tamRkabcMNucRbsBVrvagbnÞat; y  x  2 nig ³
-G½kSGab;sIusKW  2 , 0 

-G½kSGredaenKW P0 , 2 .

x> -eyIgman m  1 , P1 , 0 tamRkabcMNucRbsBVrvagbnÞat; y  2x  5 nig ³


naM[ a  m  1 , x  1 , y  0 -G½kSGab;sIusKW 2.5 , 0
bnÞat;RtUvrkmanrag y  ax  b -G½kSGredaenKW 0 , 5 .
eyIg)an 0  11  b naM[ b  1
dUcenH smIkarbnÞat;RtUvrkKW y  x 1 . X> -eyIgman m  53 , P 1 ,  4
naM[ a  m  53 , x  1 , y  4
-sg;bnÞat; y  x 1 tamrebobemIl
bnÞat;RtUvrkmanrag y  ax  b
emKuNR)ab;Tis m  1 , P1 , 0
eyIg)an  4  53  1  b naM[ b   175
y  x  1
dUcenH smIkarbnÞat;RtUvrkKW y
3
5
x
17
5
.

 -sg;bnÞat; y  53 x  175 tamrebobemIl


tamRkabcMNucRbsBVrvagbnÞat; y  x 1 nig ³ emKuNR)ab;Tis m  53 , P 1 ,  4
-G½kSGab;sIusKW P1 , 0 y
3
x
17
5 5
-G½kSGredaenKW 0 , 1 .

K> -eyIgman m  2 , P3 , 1


naM[ a  m  2 , x  3 , y  1 

bnÞat;RtUvrkmanrag y  ax  b
eyIg)an 1  2  3  b naM[ b  5 tamRkabcMNucRbsBVrvagbnÞat; y  x 1 nig ³
dUcenH smIkarbnÞat;RtUvrkKW y  2x  5 . -G½kSGab;sIusKW 17 / 3 , 0
-G½kSGredaenKW 0 , 17 / 5 .
154
g> -eyIgman m  0 , P2 , 1 7. kñúgcMeNambnÞat;bI etIbnÞat;NaRsbKña ?
edayemKuNR)ab;Tis m  0 nig y  1 k> 2x  3 y  11 b¤ y   23 x  113
enaHbnÞat;EdlRtUvrk KWRsbnwgG½kSGab;sIus
ehIymansmIkar y  1 x> 4x  8 y 1  0 b¤ y   12 x  18
K> y   23 x  1
dUcenH smIkarbnÞat;RtUvrkKW y  1 .
edabnÞat; k> y   23 x  113 nig K> y   23 x  1
-sg;bnÞat; y  1 RsbnwgG½kSGab;suIs
manemKuNR)ab;TisesµIKña naM[vaRsbKña
y 1 dUcenH k> 2x  3y  11 Rsbnwg K> y   23 x  1 .
8. rkemKuNR)ab;TisénbnÞat; etIvaRsbKña b¤eT ?
k> y  x  4 manemKuNR)ab;Tis a  1
tamRkabcMNucRbsBVrvagbnÞat; y  1 nig ³ x y5 0 manemKuNR)ab;Tsi a  1
-G½kSGab;sIusKW Kµan eRBaH y  1 RsbG½kS ox edaybnÞat;TaMgBIrmanemKuNR)ab;TisesµIKña
-G½kSGredaenKW 0 , 1 .
dUcenH KUsmIkarbnÞat;TaMgBIrRsbKña .
c> -eyIgman m   32 , P0 , 3
x> y  2x  3 manemKuNR)ab;Tis a  2
naM[ a  m   32 , x  0 , y  3 x  2 y  1  0 manemKuNR)ab;Tis a  
1
bnÞat;RtUvrkmanrag y  ax  b 2
edaybnÞat;TaMgBIrmanemKuNR)ab;TisminesµIKña
eyIg)an 3   32  0  b naM[ b  3
dUcenH KUsmIkarbnÞat;TaMgBIrminRsbKñaeT .
dUcenH smIkarbnÞat;RtUvrkKW y
3
x3 .
2
K> 3x  y  4  0 manemKuNR)ab;Tis a  3
-sg;bnÞat; y   32 x  3 RsbnwgG½kSGab;suIs y  2  3x  1 manemKuNR)ab;Tis a  3
3
y   x3
2
edaybnÞat;TaMgBIrmanemKuNR)ab;TisesµIKña

dUcenH KUsmIkarbnÞat;TaMgBIrRsbKña .

X> 2 y  5x  6 manemKuNR)ab;Tis a  5 / 2
5x  2 y  1  0 manemKuNR)ab;Tis a  5 / 2
tamRkabcMNucRbsBVrvagbnÞat; y   32 x  3 nig ³
edaybnÞat;TaMgBIrmanemKuNR)ab;TisminesµIKña
-G½kSGab;sIusKW 2 , 0
-G½kSGredaenKW 0 , 3 . dUcenH KUsmIkarbnÞat;TaMgBIrminRsbKñaeT .
155
9. etIKUsmIkarNaxøHEdlmanbnÞat;EkgKña ? ehIy y  ax  b kat;tamcMNuc  2 , 4
k> y  3x 1 manemKuNR)ab;Tis a  3 enaH x  2 , y  4
y   x manemKuNR)ab;Tis a  
1
3
1
3
eyIg)an 4  32   2  b naM[ b  7
edaybnÞat;TaMgBIrman a  a  3    13   1 dUcenH smIkarbnÞat;RtUvrkKW y  32 x  7 .
dUcenH KUénsmIkarenH manbnÞat;EkgKña . K> eyIgman 2x  2 y  9  0 nigcMNuc   53 , 0 
x> y  2 x  5 manemKuNR)ab;Tis a  2 bnÞat;RtUvrkmanrag y  ax  b EdlRsbnwg
x  2 y  6  0 manemKuNR)ab;Tis a 
1
2 bnÞat; 2x  2 y  9  0 enaH a  1
edaybnÞat;TaMgBIrman a  a  2  12  1  1 ehIy y  ax  b kat;tamcMNuc   53 , 0 
dUcenH KUénsmIkarenH manbnÞat;minEkgKñaeT . enaH x   53 , y  0
K> y  5x  4 manemKuNR)ab;Tis a  5 eyIg)an 0  1    53   b naM[ b   53
x  5 y  1  0 manemKuNR)ab;Tis a  
1
5 dUcenH smIkarbnÞat;RtUvrkKW y   x  53 .
edaybnÞat;TaMgBIrman a  a  5    15   1
X> eyIgman x  5 y  6  0 nigcMNuc 0 , 0
dUcenH KUénsmIkarenH manbnÞat;EkgKña . bnÞat;RtUvrkmanrag y  ax eRBaHvakat; 0 , 0
10. kMNt;smIkarbnÞat;EdlRsbnwg ³ ehIyRsbnwgbnÞat; x  5 y  6  0 enaH a  15
k> eyIgman y  3x  4 nigcMNuc 5 , 1 dUcenH smIkarbnÞat;RtUvrkKW y  15 x .
bnÞat;RtUvrkmanrag y  ax  b
eday y  ax  b Rsbnwg y  3x  4 enaH a  3 11. kMNt;smIkarbnÞat;EdlEkgnwg ³
ehIy y  ax  b kat;tamcMNuc 5 , 1 k> eyIgman y  12 x  4 nigcMNuc 5 , 0
enaH x  5 , y  1 bnÞat;RtUvrkmanrag y  ax  b EdlEkgnwg
eyIg)an 1  3  5  b naM[ b  14 bnÞat; y  12 x  4 naM[ a  12  1 b¤ a  2
dUcenH smIkarbnÞat;RtUvrkKW y  3x  14 . ehIy y  ax  b kat;tamcMNuc 5 , 0
x> eyIgman 3x  2 y  5  0 nigcMNuc  2 , 4 enaH x  5 , y  0
bnÞat;RtUvrkmanrag y  ax  b EdlRsbnwg eyIg)an 0  2  5  b naM[ b  10
bnÞat; 3x  2 y  5  0 enaH a  32 dUcenH smIkarbnÞat;RtUvrkKW y  2x  10 .
156
x> eyIgman x  y  5  0 nigcMNuc 0 , 0 12. k> kMNt;smIkarénbnÞat; d ³ 1

bnÞat;RtUvrkmanrag y  ax eRBaHvakat; 0 , 0 bnÞat;RtUvkMNt;manrag d : y  ax  b1

ehIy y  ax Ekgnwg x  y  5  0 man a  1 eday d kat;tamcMNuc A 3 , 5


1

naM[ a 1  1 b¤ a  1 naM[ x  3 , y  5
dUcenH smIkarbnÞat;RtUvrkKW y  x . ehIy d manemKuNR)ab;Tis a  2
1

eyIg)an 5   2 3  b naM[ b  1


smÁal; ³ bnÞat;Edlkat;tamKl;tRmúy 0 , 0
dUcenH smIkarbnÞat; d : y  2x 1 .
CabnÞat;EdlmansmIkarrag y  ax . 1

-sg;bnÞat; d : y  2x 1 ³
1

K> eyIgman 8x  3y  1  0 nigcMNuc  1 , 4


bnÞat;RtUvrkmanrag y  ax  b EdlEkgnwgbnÞat; A 3 , 5

8x  3 y  1  0 manemKuNR)ab;Tis a  
8 

3
naM[ a    3   1 b¤ a  8
8 3

ehIy y  ax  b kat;tamcMNuc  1 , 4
enaH x  1 , y  4
eyIg)an 4  83  1  b naM[ b  358 d1 : y  2 x  1

dUcenH smIkarbnÞat;RtUvrkKW y  83 x  358 . x> kMNt;témø m edIm,I[ d // d 2 1

X> eyIgman y   x  6 nigcMNuc   4 ,  23  eyIgman d : y  m 1x  2 nigbnÞat;


2

d : y  2 x  1 >
1
bnÞat;RtUvrkmanrag y  ax  b EdlEkgnwgbnÞat; edIm,I[bnÞat;TaMgBIr d nig d RsbKñaluHRtaEt ³
2 1
y   x  6 manemKuNR)ab;Tis a  1
m 1  2 naM[ m  1
naM[ a  1  1 b¤ a  1
dUcenH témøkMNt;)anKW m  1 .
ehIy y  ax  b kat;tamcMNuc   4 ,  23 
-etImantémø m EdleFVI[ d  d b¤eT ?
enaH x  4 , y   23 2 1

bnÞat; d  d luHRtaEt m  1 2  1


2 1

eyIg)an  23  1  4  b naM[ b  103 b¤  2m  2  1 naM[ m  32


dUcenH smIkarbnÞat;RtUvrkKW y  x  103 . dUcenH mantémø m  32 EdleFV[I d  d1 .
2

157
-sresrsmIkarkarénbnÞat;RtUvnwgtémø m nImYy² 13. kñúgtRmútGrtUNrem eyIgmanbIcMNucKW ³
cMeBaH m  1 enaH d : y  2x  2 A0 , 6 , B 3 , 0 nig C 6 , 0
2

cMeBaH m  32 enaH d  : y  12 x  2
2
AB : y  2 x  6

-sg;bnÞat;TaMgenaH ¬emIlsmIkarbnÞat; rYcsg;Etmþg¦ A0 , 6


1
D : y   x3
1 2
d 2 : y  x2
2
AC : y   x  6
d1 : y  2 x  1
B 3 , 0 C 6 , 0

d 2 : y  2 x  2
D : y   x  3

K> sresrsmIkarbnÞat;kat;tamcMNuc A nig B ³ k> -rksmIkarénbnÞat; AB ³


eyIgman A 3 , 5 nig B6 , 0 tag M x , y  CacMNucenAlIbnÞat; AB
bnÞat;RtUvrkmanrag AB : y  ax  b naM[ emKuNR)ab;TisénbnÞat; AM nig AB esµIKña
tag M x , y  CacMNucenAlIbnÞat; AB eyIg)an yx  xy  yx  xy
A B A

naM[ emKuNR)ab;TisénbnÞat; AM nig AB esµIKña


A B A

y 6 06

eyIg)an yx  xy  yx  xy
A B A x 0 30
y  6  2x
A B A

y 5 05 y  2x  6

x   3 6   3
y 5 5
dUcenH smIkarbnÞat; AB : y  2 x  6 .

x3 9
y  5    x  3
5
9 - rksmIkarénbnÞat; AC ³
5 5
y   x 5 tag N x , y  CacMNucenAlIbnÞat; AC
9 3
5
y  x
10 naM[ emKuNR)ab;TisénbnÞat; AN nig AC esµIKña
9 3
eyIg)an yx  xy  yx  xy
A C A

ebI x0 enaHbnÞat; 5


y  x
10
CYbnwgG½kS A C A
9 3 y6 06

 10 
yy Rtg;cMNuc
0 , 
 3
x0 60
y  6  x
dUcenH bnÞat; 5 10
AB : y   x 
9 3
ehIyCYb y  x  6

nwgG½kS yy
 10 
Rtg;cMNuc
0 ,  . dUcenH smIkarbnÞat; AC : y   x  6 .
 3

158
x> rksmIkarénbnÞat; D ³ -rksmIkarénExSemdüaT½rrbs; AB ³
smIkarbnÞat; D RtUvrkmanrag D : y  ax  b tag d : y  ax  b CasmIkarExSemdüaT½r AB
eday D kat;tam C6 , 0 enaH x  6 , y  0 naM[ d RtUvkat;tam I Edl I kNþal AB
ehIy D EkgnwgbnÞat; AB : y  2x  6 enaH I  6 2 2 , 2 2 2  b¤ I 4 , 0
naM[ a  2  1 enaH a   12 ehIy d : y  ax  b RtUvEkgnwg AB : y  x  4
eyIg)an 0    12   6  b naM[ b  3 naM[ a 1  1 enaH a  1
eyIg)an d : 0  1 4  b enaH b  4
dUcenH bnÞat;rk)anKW D : y   12 x  3 .
dUcenH ExSemdüaT½rén AB KW d : y   x  4 .
K> rksmIkarénbnÞat; D ³
x> bgðajfa CAB CaRtIekaNsm)at ³
smIkarbnÞat; D RtUvrkmanrag D : y  ax  b
eyIgman C 2 , 6 , A6 , 2 nig B2 ,  2
eday D kat;tamcMNuc B 3 , 0
eyIgKNnargVas;RCugénRtIekaN CAB ³
naM[ x  3 , y  0 AC   2  62  6  22  64  16  80
ehIy D RsbnwgbnÞat; AC : y  x  6 AB  2  62   2  22  16  16  32
naM[ a  1 BC   2  22  6  22  16  64  80
eyIg)an 0  1  3  b enaH b  3 edayRtIekaN CAB man AC  BC  80
dUcenH bnÞat;rk)anKW D : y  x  3 . dUcenH CAB CaRtIekaNsm)atkMBUl C .
14.k-rksmIkarbnÞat;kat;tamcMNuc A nig B ³ K> rksmIkarénbnÞat; C ³ 2

eyIgmanBIrcMNuc A6 , 2 nig B2 ,  2 tag C : y  ax  b CabnÞat;RtUvrk


2

tag M x , y  CacMNucenAlIbnÞat; AB eday C kat; C 2 , 6 enaH x  2 , y  6


2

naM[ emKuNR)ab;TisénbnÞat; AM nig AB esµIKña ehIy C Rsbnwg AB : y  x  4 enaH a  1


2

eyIg)an yx  xy  yx  xy
A B A
eyIg)an C : 6  1  2  b enaH b  8
2
A B A

y2 22
x6

26
dUcenH bnÞat;rk)anKW C : y  x  8 .
2

y2
x6
1 X> bgðajfa C kat;tam M  5 , 3 ³
2

y 2  x6 ebIcMNuc M epÞógpÞat;smIkar C enaH C kat; M


2 2
y  x4
cMeBaH M  5 , 3 enaH 3  5  8 b¤ 3  3 Bit
dUcenH smIkarbnÞat; AB : y  x  4 . dUcenH bnÞat; C kat;tam M  5 , 3 .
2

159
15.k> rksmIkarénRCugRtIekaN ABC ³ x> rksmIkarénExSemdüaT½rrbs;RCugRtIekaN ABC
eyIgman A 6 , 0 , B6 , 0 nig C 3 , 9 -smIkarénExSemdüaT½rRCug AB ³
-rksmIkarénRCug AB ³ tag M : y  ax  b CaExSemdüaT½rRCug AB ³
AB

tag M x , y  CacMNucenAlIbnÞat; AB eday M RtUvkatcMNuc I kNþalRCug AB


AB AB

naM[ emKuNR)ab;TisénbnÞat; AM nig AB esµIKña naM[ I   62 6 , 0 2 0  b¤ I 0 , 0 ¬Kl;>¦


AB AB

eyIg)an yx  xy  yx  xy
A B A
enaHsmIkarRtUvrkbþÚrmkCarag M : y  ax AB
A B A

y0

00 ehIy M : y  ax Ekgnwg AB: y  0
x   6 6   6
AB

y0 naM[ 0  ax enaH 0  x


dUcenH smIkarénRCug AB: y  0 CaG½kS xx dUcenH smIkaremdüaT½r M : x  0 CaG½kS yy
AB

-rksmIkarénRCug AC ³ -smIkarénExSemdüaT½rRCug AC ³
tag N x , y  CacMNucenAlIbnÞat; AC tag M : y  ax  b CaExSemdüaT½rRCug AC ³
AC

naM[ emKuNR)ab;TisénbnÞat; AN nig AC esµIKña eday M RtUvkatcMNuc I kNþalRCug AC


AC AC

eyIg)an yx  xy  yx  xy
A C A
naM[ I   62 3 , 0 2 9  b¤ I  23 , 92 
AC AC
A C A

y0

90 ehIy M Ekgnwg AC : y  x  6
x   6 3   6
AC

y  x6
naM[ a 1  1 enaH a  1
eyIg)an 92  1 23   b enaH b  3
dUcenH smIkarénRCug AC : y  x  6 .
dUcenH smIkaremdüaT½r M : y   x  3 .
AC

-rksmIkarénRCug BC ³
tag K x , y  CacMNucenAlIbnÞat; BC -smIkarénExSemdüaT½rRCug BC ³
naM[ emKuNR)ab;TisénbnÞat; BK nig BC esµIKña tag M : y  ax  b CaExSemdüaT½rRCug BC ³
BC

eday M RtUvkatcMNuc I kNþalRCug BC


eyIg)an yx  xy  yx  xy
B C B
BC BC

y0 90
C B
naM[ I  6 2 3 , 0 2 9  b¤ I  92 , 92 
BC BC


x 6 36 ehIy M Ekgnwg BC : y  3x  18
BC
y
 3
x6 naM[ a   3  1 enaH a  1 / 3
y  3 x  18
eyIg)an 92   13  92   b enaH b  3
dUcenH smIkarénRCug BC : y  3x  18 .
dUcenH smIkaremdüaT½r M BC
1
: y  x3
3
.

160
16.kMNt;témø a nig b ³ 17. kMNt;smIkarEdlTak;TgnwgkareRbIfamBl
eyIgmanbnÞat; 4x  2 y  b BIeRbgeTAtamqñaM ³
eday cMNuc a , 6 enAelIbnÞat; 4x  2 y  b tag y  ax  b CasmIkarEdlRtUvkMNt;
eyIg)an 4a  2  6  b b¤ 4a  12  b 1 Edl x CaqñaM nig y CafamBleRbgEdleRbIR)as;
ehIy cMNuc 2 , a  enAelIbnÞat; 4x  2 y  b edayeKkMNt;yk x  0 RtUvnwgqñaM 1970
eyIg)an 4  2  2a  b b¤ 2a  8  b 2 enaH qñaM 1973 kMNt;eday x  3
eyIgpÞwmtémø b énsmIkar 1 nig 2 eyIg)an³ qñaM 1990 kMNt;eday x  20
4a  12  2a  8
eday enAqñaM 1973 famBleRbgeRbIR)as;KW 47%
4a  2a  8  12
2a  4 enAqñaM 1990 famBleRbgeRbIR)as;KW 38%
a  2 naM[ x  3 RtUvnwgfamBleRbIR)as; y  47%
cMeBaH a  2 ykeTACMnYskñgú 1 eyIg)an ³ x  20 RtUvnwgfamBleRbIR)as; y  38%
1 : 4a  12  b
4 2  12  b
edayemKuNR)ab;TisénbnÞat; CapleFobén
b4 bERmbRmYlén y nig bERmbRmYlén x
dUcenH témøkMNt;)anKW a  2 , b  4 . eyIg)an a  38%20473 %
0.38  0.47 0.09
a 
-sg;bnÞat;enaH ³ 17 17

cMeBaH b  4 enaHeyIg)an 4x  2 y  4 naM[ 47%  


0.09
17
3 b

b¤ 2x  y  2 naM[ y  2x  2 0.47  


0.09
17
3  b
eyIgsg;bnÞat; edayemIl y  2x  2 b  0.47 
0.27

¬mincaM)ac;eRbItaragtémøelx ¦ 8.26
17
b
17
smIkarGacsresr y
0.09
17
x
8.26
17
b¤ y
9
1700
x
826
1700

dUcenH smIkarTMnak;TMngkMNt;)anKW
y
9
1700
x
826
1700
.
y  2 x  2

161
១១ ១

1. ឬ

x  y  8 2 x  y  3
k>  3 ,  5 x>  2 ,  1
 x  y  2 x  2 y  4
 y  3x x  y  3
K>  0 , 0 X>  5 , 2
x  y  3 2 x  2 y  6
2.

k> xx  4yy14 x> xy  32  00 K> 2x x23yy813 X> xy  y3x 4
   
x  y  1 x  y  3
2 x  y  6
g>  c> 
x y q> 
 7  2x C> xy  7y  3x
 y  8  2x  2  2  2  y  2 

3.

k> 52xx  22yy  92 x> 22xx  2y y58 K> 34xx  33yy  916 X> 8x7 x 3 y6 y 17
2
   

g> 37xx 10 y  1


2 y  13
c> 84xx  23 yy  1510 x  8 y  1
q> 210 x  4 y  16
C> 5 x  3 y  9  0

3x  4 y  17  0
   

Q> 6x2x552yx67 y j> 43x x35y y97 d> 37xx  49 yy  01 z> 46x x23yy1.5
19
   
4.

k> 4x x2 yy  16 x> 4x x23yy64 K> 2yx34yx135 X> 53xx  yy  15
23
 1   

g> 22x x3 yy  62 c> 4x x5 yy  112 q> 32xx  33yy 18
11
C> 54xx  32yy  14
   
 x  10 y x  7  y
5 x  7 y  3 3x  6 y  6  
Q>  j>  d> 1 z> 1
2 x  14 y  2 2 x  2 y  2 x  3 y  2  x 1  y
4
5. m
4 x  6 y  12 12x  3my  1
 
2 x  my  5 4 x  2 y  3
m n

mx  ny  10  6 x  y  30

 n  1x  1 y  n  m
 4x  3 y  5 
 3

162
6. 2 3 21 000 ៛
3 2 19 000 ៛

7. 8 000 ៛ 11
500 ៛ 1 000 ៛
8.
7m
1m
9. 110 m 5m 2m
2
150 m
10. 20 $ H P
H6 P4 1.5 $
H2 P5 1$
H4 P4
11. 22 246
3
12. 490
7
5
13. 324
4
14. ABC 192 m BC , CA , AB
2.5 , 2 1.5
25
15. 36
81
3
16. 64 m
5

17. Ax  By  9  2 , 1  3 , 3 A B
2 2 1
x  y  2

18. 
1  5  3
 x y 4
19. A B

2008 2009 2010 2011


A 2.3 2.7 3.1 3.5
B 2.9 3.3 3.7 4.1
A B ឬ

DDCEE

3
163
១១ 1

1. kUGredaenEdl[CacemøIyrbs;RbB½n§smIkarb¤eT? X> 2x xy2y3 6 b¤ xx  yy  33 b¤ x  y  3


 
k> xx  yy  82 eyIgmankUGredaen 5 , 2

eyIgmankUGredaen 3 ,  5 mann½yfa cMeBaH x  5 , y  2 eyIg)an ³
mann½yfa cMeBaH x  3 , y  5 eyIg)an ³ 5  2  3 b¤ 3  3
3   5  8 3  5  8 8  8 eXIjfa kUGredaenHvaepÞógpÞat;RbB½n§smIkar
 b¤  b¤ 
3   5  2 3  5  2  2  2
eXIjfa kUGredaenHvaepÞógpÞat;RbB½n§smIkar dUcenH 5 , 2 CakUGredaenrbs;RbB½n§smIkar
dUcenH 3 ,  5 CakUGredaenrbs;RbB½n§smIkar 2. edaHRsayRbB½n§smIkartamRkab ³
 x 1
2 x  y  3 x  4 y  1 4 y  x  1 y 
x>  k>  b¤  b¤  4
x  2 y  4  x  y  4  y  x  4    4
 y x
eyIgmankUGredaen 2 ,  1 eyIgeRbItaragtémøelxedIm,Isg;Rkab ³
mann½yfa cMeBaH x  2 , y  1 eyIg)an ³ -cMeBaH y  x 4 1 ³ xy  13 15
2  2   1  3 4  1  3 3  3
 b¤  b¤ 
 2  2   1  4 22  4  04 - cMeBaH y   x  4 ³ xy  22 0
4
eXIjfa kUGredaenHminvaepÞógpÞat;RbB½n§smIkar eyIgsg;RkabénRbB½n§smIkar)an ³
dUcenH 2 ,  1 minCakUGredaenrbs;Rb>smIkar x 1
y
4
K> xy  y3x 3

eyIgmankUGredaen 0 , 0
mann½yfa cMeBaH x  0 , y  0 eyIg)an ³
0  3  0 0  0

00 3
b¤ 
03
y  x  4
 
edayRkabTaMgBIrkat;KñaRtg;cMNuc  3 , 1
eXIjfa kUGredaenHminvaepÞógpÞat;RbB½n§smIkar
mann½yfa x  3 , y  1 CacemøIyénRbB½n§
dUcenH 0 , 0 minCakUGredaenrbs;Rb>smIkar
dUcenH RbB½n§smIkarmanKUcemøIy xy  31 .

164
x> xy  32  00 b¤ xy  23 edayRkabTaMgBIrkat;KñaRtg;cMNuc 2 ,  3
 
mann½yfa x  2 , y  3 CacemøIyénRbB½n§
y  2 CabnÞat;edkkat;G½kSGredaenRtg; 2

x  3 CabnÞat;Qr kat;G½kSGab;sIusRtg;  3 dUcenH RbB½n§smIkarmanKUcemøIy xy  23 .



eyIgsg;RkabénRbB½n§smIkar)an ³
X> xy  y3x 4 b¤  yy  3xx  4
 

y2
tagragCMnYyedIm,Isg;Rkab ³
-cMeBaH y  3x x 0 1
y 0 3

-cMeBaH y   x  4 x 0 1
y 4 3
eyIgsg;Rkab)an ³
x  3
edayRkabTaMgBIrkat;KñaRtg;cMNuc  3 , 2 y  3x

mann½yfa x  3 , y  2 CacemøIyénRbB½n§

 x  3
dUcenH RbB½n§smIkarmanKUcemøIy  .
y  2
y  x  4
 2 x  13
 y  3
K> 2x x23yy813 b¤ 
 y  x 8
 2
eyIgeRbItaragtémøelxedIm,Isg;RkabénbnÞat; edayRkabTaMgBIrkat;KñaRtg;cMNuc 1 , 3
-cMeBaH y  2 x 3 13 xy 23 51 mann½yfa x  1 , y  3 CacemøIyénRbB½n§
-cMeBaH y
x 8 x 2 4 dUcenH RbB½n§smIkarmanKUcemøIy xy 13 .
2 y 3 2 
2 x  13
y
3 g> 2yx8y2x6 b¤  yy  86  22xx
 

x 8
tagragCMnYyedIm,Isg;Rkab ³
y
2 -cMeBaH y  6  2 x xy 22 03

-cMeBaH y  8  2 x x 3 4
y 2 0
eyIgsg;Rkab)an ³

165
x  y  3  y  x  3
 
q>  7  2x b¤  7
y  6  2x  y  2  y   x  2
eRbItaragtémøelxedIm,Isg;Rkab ³
-cMeBaH y   x  3 xy 03 12
-cMeBaH y   x  72 xy 70/ 2 51/ 2
y  8  2x

tamRkabbnÞat;TaMgBIrRsbKña ¬eRBaHemKuNR)ab;
Tis énbnÞat;TaMgBIrKWesµI -2 dUcKña ¦
dUcenH RbB½n§smIkarKµanKUcemøIy . tamRkab bnÞat;TaMgBIrRsbKña ¬eRBaHmanemKuN
x  y  1
 y  x 1
R)ab;TisesµI -1 dUcKña¦

c> x y b¤ 
 2  2  2  y  x  4
dUcenH RbB½n§smIkarKµanKUcemøIy .
taragtémøelxRtUvKñaén x nig y edIm,Isg;Rkab
-cMeBaH y   x  1 xy 10 10 C> xy  7y  3x b¤  yy  x x3 7
 

-cMeBaH y   x  4 x 0 2 taragtémøelxRtUvKñaén x nig y ³


y 4 2
-cMeBaH y   x  7 xy 34 43
-cMeBaH y  x3
x 0 1
y 3 4

y  x3
y  x  4
y  x  1

y  x  7

tamRkab bnÞat;TaMgBIrRsbKña ¬eRBaHmanemKuN


R)ab;TisesµI -1 dUcKña¦
tamRkabbnÞat;TaMgBIRbsBVKñaRtg;cMNuc
dUcenH RbB½n§smIkarKµanKUcemøIy . dUcenH x  2 , y  5 CaKUcemøIyénRbB½n§smIkar
166
3. edaHRsayRbB½n§smIkartamviFIbUkbM)at; ³ X> 8x7x 3y6y 17 2 2
eyIg)an ³
k> 52xx  22yy  92 12


16 x  6 y  34 1

 7 x  6 y  2 2
2 x  2 y  2
 9 x  36
naM[ x  369  4
5 x  2 y  9naM[ x  77 b¤ x  1
7x  7 yk x  4 CMnYskñúgsmIkar 2 eyIg)an ³
yk x  1 CMnYskñúgsmIkar 1 eyIg)an ³  7  4   6 y  2
2 1  2 y  2 6 y  2  28
2 y  2  2 30
y
6
4
y y 5
2
y  2
dUcenH RbB½n§smIkarmanKUcemøIy xy  45 .

dUcenH RbB½n§smIkarmanKUcemøIy xy 12 .

g> 37xx 10 y  1
2 y  135
2 x  2 y  8 x  y  4 1 
x>  b¤ 
2 7 x  10 y  1 1
2 x  y  5 2 x  y  5 
x  y  4 15x  10 y  65 2 naM[ x  2266  3
 22 x  66
2 x  y  5naM[ x  93  3
3x  9 yk x  3 CMnYskñúgsmIkar 2 eyIg)an ³
yk x  3 CMnYskñúgsmIkar 1 eyIg)an ³ 15   3  10 y  65
10 y  65  45
3 y  4
 20
y  43 y
10
y 1 y  2

dUcenH RbB½n§smIkarmanKUcemøIy xy  13 . dUcenH RbB½n§smIkarmanKUcemøIy xy  32 .


 
1
K> 34xx  33yy  916 2 c> 84xx  23 yy  1510   2
 
3x  3 y  9
  8 x  6 y  30 1
4 x  3 y  16 
7 x  7
naM[ x  77  1 8 x  2 y  10 2 naM[ y  840  5
8 y  40
yk x  1 CMnYskñúgsmIkar 1 eyIg)an ³ yk y  5 CMnYskñúgsmIkar 2 eyIg)an ³
3 1  3 y  9 b¤ 3 y  9  3 8 x  2 5  10
12 8 x  10  10
y 4
3 x0

dUcenH RbB½n§smIkarmanKUcemøIy xy  41 . dUCenH RbB½n§smIkarmanKUcemøIy xy  05 .


 

167
q> 2x10x8y 4y116  5 j> 4x3x 3 y5 y 9 7
4
3
 
10 x  40 y  5 1 12 x  9 y  27 1
 
 10 x  4 y  16 2 naM[ y  11 
1  12 x  20 y  28 2naM[ y  11
55
5
44 y  11 44 4 11y  55

yk y
1
4
CMnYskñúgsmIkar 1 eyIg)an ³ yk y  5 CMnYskñúgsmIkar 1 eyIg)an ³
12 x  9  5  27
1
 10 x  4   16 12 x  27  45
4
 10 x  16  1 x
72
12
b¤ x6

x
15
b¤ x
3
 10 2 dUcenH RbB½n§smIkarmanKUcemøIy xy  65 .

 x  3 / 2
dUcenH RbB½n§smIkarmanKUcemøIy  .
y  1/ 4   7 
d> 37xx  49 yy  01
3

C> 53xx  34 yy  17
90
0
b¤ 5 x  3 y  9

3x  4 y  17
4
3  21x  28 y  7 1
  
20 x  12 y  36 1  21x  27 y  0 2 naM[ y  71  7

9 x  12 y  51 2
y7
29 x  87
naM[ x  2987  3 yk y  7 CMnYskñúgsmIkar 2 eyIg)an ³
21x  27 7   0
yk x  3 CMnYskñúgsmIkar 1 eyIg)an ³ 21x  189
20 3  12 y  36
12 y  36  60 x
189
21
b¤ x9

y
24
12
b¤ y2
dUcenH RbB½n§smIkarmanKUcemøIy xy  97 .

 x  3
dUcenH RbB½n§smIkarmanKUcemøIy  .
y  2
z> 4x6x 2 y3 y 119.5
  2
3

6 x  5  2 x  7 y  4x  7 y  5
Q>  b¤  12x  6 y  57 1
 2x  5 y  6 2 x  5 y  6   2 
12x  6 y  3 2  60  5
naM[
x 
 4 x  7 y  5 1 24x  60 24 2

 4 x  10 y  12 2 naM[ y  17 1 yk x
5
CMnYskñúgsmIkar eyIg)an ³
2
17 y  17 17 2
 5
yk y  1 CMnYskñúgsmIkar 1 eyIg)an ³ 12    6 y  3
 2
 2 1
4x  7 1  5 b¤ 4x  5  7 enaH x   6 y  3  30
4 2 9
y
2
 1
x  
dUcenH RbB½n§smIkarmanKUcemøIy  2 . dUcenH RbB½n§smIkarmanKUcemøIy xy  95/ 2/ 2 .
 y  1 

168
edaHRsayRbB½n§smIkartamviFICMnYs ³ 23 1
4. X> 53xx  yy  15 2
1 
k> 4x x2 yy  16
1 2 tam 1 : 5x  y  23 enaH y  5x  23

tam 1 : x  2 y  16 b¤ x  16  2 y yk y  5x  23 CMnYs 2 eyIg)an ³
yk x  16  2 y CMnYskñúg 2 eyIg)an ³ 2 : 3x  5x  23  15
3x  5 x  23  15
2 : 416  2 y   y  1
 2 x  23  15  x  4
64  8 y  y  1
9 y  63 b¤ y  7 cMeBaH x  4 enaH y  5x  23

cMeBaH y  7 enaH x  16  2 y  5 4  23  3 >


 16  2 7 
2 dUcenH RbB½n§smIkarmanKUcemøIy xy  34 .

x  2
dUcenH RbB½n§smIkarmanKUcemøIy  .
 y  7 g> 2x2x 3yy62 12

x> 4x x23yy64 12 tam 1 :  2x  y  2 b¤ y  2  2x

tam 1 : x  2 y  6 b¤ x  6  2 y yk y  2  2x CMnYskñúg 2 eyIg)an ³
2 : 2 x  32  2 x   6
yk x  6  2 y CMnYskñúg 2 eyIg)an ³ 2x  6  6x  6
2 : 46  2 y   3 y  4 8x  0  x0
24  8 y  3 y  4
 5 y  20 b¤ y4
cMeBaH x0 enaH y  2  2x  2  2  0  2

cMeBaH y  4 enaH x  6  2 y  6  2  4  2 dUcenH RbB½n§smIkarmanKUcemøIy xy  02 .



 x  2
dUcenH RbB½n§smIkarmanKUcemøIy .
c> 4x x5 yy  112 12

y  4

K> 2yx34yx135 12 tam 1 : x  5 y  11 enaH x  11 5 y

yk 1 CMnYskñúg 2 enaHeyIg)an ³ yk x  11 5 y CMnYskñúg 2 enaHeyIg)an ³
2 : 411  5 y   y  2
2 : 2 x  3 4 x  5  13
44  20 y  y  2
2 x  12 x  15  13
 42
28  21y  42  y  2
14 x  28  x 2  21
14
cMeBaH x  2 enaH y  4x  5 cMeBaH y2 enaH x  11 5 y

 42  5  3 >  11  52  1 >


dUcenH RbB½n§smIkarmanKUcemøIy x  2
 . dUcenH RbB½n§smIkarmanKUcemøIy xy 12 .

 y  3

169
11 1 cMeBaH x  1 enaH 7 y 1  x
q> 32xx  33yy 18 2
 7 y  1   1
tam 1 : 2 x  3 y  11 enaH 3 y  11 2x 7y  2
yk 3 y  11 2x CMnYskñúg 2 enaHeyIg)an ³ y
2
2 : 3x  11  2 x   18 7

x  18  11  x  7
dUcenH RbB½n§smIkarmanKUcemøIy xy  21/ 7 .
cMeBaH x  7 enaH 3 y  11 2x 
11  2  7
y
3
 1
j> 32xx 62yy  6 b¤ xx  2yy  2 iii
 
dUcenH RbB½n§smIkarmanKUcemøIy xy  71 . yk ii : x  y CMnYskñúg i  enaHeyIg)an ³

i  : y  2y  2  y
2
1 3
C> 54xx  32yy  14 2
 dUcenH RbB½n§smIkarmanKUcemøIy x  y  23 .
tam 1 : 5x  3 y  4 enaH x  4 53 y 3
 x  10 y
yk 3 CMnYskñúg 2 enaHeyIg)an ³  
d> 1
x  3 y  2 * *

2 : 4 4  3 y   2 y  1 2
 5 
16  12 y
yk * : x  10 y CMnYskñúg * * enaHeyIg)an ³
 2y 1
5 * * : 1 10 y  3 y  2
2
16  12 y  10 y  5
5y  3y  2  y 1
 11 1
 22 y  11  y  
 22 2 tam * : x  10 y  10 1  10
yk y  12 CMnYskñúg 3 eyIg)an ³
dUcenH RbB½n§smIkarmanKUcemøIy xy 10 .
1 1 
4  3
3 : x 2  83  1
5 2 2 x  7  y
5 1
z> 
1 b¤ xx  4y 47y 2
dUcenH RbB½n§smIkarmanKUcemøIy x  y  12 . 
4
x 1  y 

yk 1 : x  y  7 CMnYskñúg 2 enaHeyIg)an ³


5 x  7 y  3 5 x  7 y  3 1
Q>  b¤  2 : y  7  4  4 y
2 x  14 y  2  x  7 y  1 2 3  3y
tam 2 : x  7 y  1 enaH 7 y 1  x y 1
yk 7 y 1  x CMnYskñúg 1 enaHeyIg)an ³ tam 1 : x  y  7  1  7  8
1 : 5 x  1  x   3
4 x  4 dUcenH RbB½n§smIkarmanKUcemøIy  xy   18 .
4    
x  1
4
170
5. k> rktémø m edIm,I[RbB½n§smIkarKµancemøyI ³ naM[ m  8 nig n  6
-eyIgman 42xx  6myy 125 dUcenH rk)antémø m  8 nig n  6 .

 6 x  y  30
rMlwk ³ lkçN³énRbB½n§smIkardWeRkTI!manBIrGBaØat -eyIgman n 1x  1 y  n  m

RbB½n§smIkardWeRkTI!manBIrGBaØatmanrag aaxx  bbyy  cc  3
 edIm,I[RbB½ns§ mIkarmanKUcemøIyeRcInrab;luHRta ³
-ebI aa  bb  cc RbB½n§smIkarKµanKUcemøIy 6 1
 
30
b¤ 6
3
30
n 1 1 n  m n 1 nm
-ebI aa  bb  cc RbB½n§>manKUcemøIyeRcInrab;minGs; 3

a b eyIg)an n 6 1  3 nig n30 m  3


-ebI 
a  b
RbB½n§>manKUcemøIy EtmYyKt;.
naM[ n  1  2 b¤ n  3
edIm,I[RbB½ns§ mIkarKµanKUcemøIyluHRtaEt ³ ehIy m  n  10 enaH m  10  n  10  3  7
4 6 12
 
2 m 5
enaHeyIgTaj)an ³ dUcenH témørk)anKW m  7 nig n  3 .
4 6
 b¤ 4m  12 naM[ m   3
12
2 m 4 6. rktémøesovePATaMgBIrRbePT ³
dUcenH témøkMNt;)anKW m  3 . -tag x CatémøesovePAesþIg ¬KitCaerol¦
y CatémøesovePARkas; ¬KitCaerol¦
x  3my  1
-eyIgman 12 -tambRmab;RbFan ³
4 x  2 y  3
edIm,I[RbB½ns§ mIkarKµancemøIyluHRtaEt ³ ÷suxTijesovePAesþIg 2k, nigRkas; 3k, Gs;
12  3m 1  3m 1
4

2

3
b¤ 3
2

3
R)ak;cMnYn 21 000 ` naM[ 2x  3 y  21000 1
 3m
eyIg)an 3  2 b¤  3m  6 b¤ m  2 ÷esATijesovePAesþIg 3k, nigRkas; 2k, Gs;
R)ak;cMnYn 19 000 ` naM[ 3x  2 y  19000 2
dUcenH témøkMNt;)anKW m  2 . -tam 1 nig 2 eyIg)anRbB½n§smIkar ³
2 x  3 y  21000   2
x> rktémø m nig n edIm,I[RbB½n§smIkar 
3x  2 y  19000  3
eRbIviFIbUkbM)at;

manKUcemøIyeRcInrab;minGs; ³  4 x  6 y  42000

-eyIgman mx4xny3 y 105  9 x  6 y  57000
5 x  15000
naM[ x  15000  3000
 5
edIm,I[RbB½ns§ mIkarmanKUcemøIyeRcInrab;luHRta ³ cMeBaH x  3000 enaH 2  3000  3 y  21000
m n 10

4 3 5
 b¤ m n

4 3
2 naM[ 3y  15000 enaH y  15000
3
 5000

eyIg)an m4  2 nig n3  2 dUcenH esoePAesþIgtémø 3000` nigRkas; 5000`


171
7. rkcMnYnRkdas;R)ak;RbePTnImYy² ³ eyIg)an 2x  1  y 2
-tag x CacMnYnRkdas;R)ak; 500` ¬KitCasnøwk¦ -tam 1 nig 2 eyIg)anRbB½n§smIkar ³
y CacMnYnRkdas;R)ak; 1000` ¬KitCasnøwk¦  x  1  y  7 1
  x  y  6 i 
-tambRmab;RbFan ³ x b¤ 
 2  1  y 2  x  2  2 y ii 
÷kñúgkarbUbmanRkdas;R)ak;BIrRbePTcMnYn 11 s> yk i  CMnYskñúg ii eyIg)an ³
eyIg)an ³ x  y  11 i  ii  : y  6  2  2 y  y 8
÷R)ak;srubkñgú kabUbmancMnYn 8000 ` cMeBaH y 8 CMnYskñúg i  eyIg)an ³
eyIg)an ³ 500 x  1000 y  8000 ii i  : x  y  6  8  6  14 >
-tam i  nig ii eyIg)anRbB½n§smIkar ³
 x  y  11 i  dUcenH ExSBYrmanRbEvg 14 m nig
 eRbIviFICMnYs edImetñatmankm<s; 8 m .
500x  1000 y  8000 ii 
tam i  : x  y  11 enaH x 11 y
yk x 11 y CMnYskñúgsmIkar ii eyIg)an ³ rkRbEvgbeNþay nigTTwgéndIenaH ³
9.

ii  : 50011  y   1000 y  8000 -tag x CaRbEvgbeNþay ¬KitCa Em:Rt¦


5500  500 y  1000 y  8000 y CaRbEvgTTwg ¬KitCa Em:Rt¦
500 y  8000  5500
2500
-smµtikmµ ³ dImanragCactuekaNEkg
y 5
500 -tambRmab;RbFan ³
cMeBaH y  5 enaH x  11 y  11 5  6 ÷dImanbrimaRt 110 m
dUcenH Rkdas;R)ak; 500 ` mancMnYn 6 snøwk eyIg)an 2x  y   110 b¤ x  y  55 1
Rkdas;R)ak; 1000 ` mancMnYn 5 snøwk . ehIymanépÞ S  xy
÷ebIbEnßm 5m elIbeNþay nig 2m elITTwg enaH
8. rkRbEvgExSBYr nigkm<s;edImetñat ³
épÞekInelIsBImun 150 m 2

-tag x CaRbEvgExSBYr ¬KitCa Em:Rt¦ eyIg)an x  5 y  2  xy  150


y Cakm<s;edImetñat ¬KitCa Em:Rt¦
b¤ xy  2x  5 y 10  xy  150
-smµtikmµ ³ bMBg; nigExSbMBg;manRbEvg 1 m 2 x  5 y  140 2 >
-tambRmab;RbFan ³ -tam 1 nig 2 eyIg)anRbB½n§smIkar ³
÷ebIKat;minbt;ExSCaBIr enaHKat;bgðÚtbMBg;mkdl;  x  y  55   2
 eRbIviFIbUkbM)at;
dI ehIyenAsl;ExS 7m eTot 2 x  5 y  140
 2 x  2 y  110
eyIg)an ³ x  1  y  7 1 
2 x  5 y  140 naM[ y  10
÷ebIKat;bt;ExSCaBIr Kat;bgðÚtmkdl;dlI µm 3 y  30

172
yk y  10 CMnYskñúg 1 eyIg)an ³ 11. rkBIrcMnYnenaH ³
1 : x  10  55 >
 x  55  10  45 -tag x nig y CaBIrcMnYnenaH Edl x  y
dUcenH RbEvgbeNþayKW 45m , TTwg 10m . -bRmab;RbFan ³
÷pldkénBIrcMnYnesµnI wg 22
10.ebITijnM H 4 nignM P 4 etIeKRtUvGab;R)ak;b:unµan? eyIg)an x  y  22 1
edIm,IrkR)ak;Gab; eyIgRtUvdwgtémønMnImyY ²sin ÷BIrdgéncMnYnFMbUkbIdgéncMnYntUcesµInwg 246
-tag x CatémønM H ¬KitCa $¦ eyIg)an 2x  3 y  246 2
y CatémønM P ¬KitCa $¦ -tam 1 nig 2 eyIg)anRbB½n§smIkar ³
-smµtikmµ ³ eKmanR)ak; 20$  x  y  22 3
eRbIviFIbUkbM)at;

2 x  3 y  246
-tambRmab;RbFan ³ 
3x  3 y  66
÷ebITijnM H 6 nignM P 4 eKGab;vij 1.5$ 
2 x  3 y  246
eyIg)an ³ 6x  4 y  20 1.5 naM[ x  312
5 x  312 5
 62.4

b¤ 6x  4 y  18.5 1 yk x  62.4 CMnYskñúg 1


1 : 62.4  y  22
÷ebITijnM H 2 nignM P 5 eKGab;vij 1$ y  62.4  22
eyIg)an ³ 2x  5 y  20 1 y  40.4
b¤ 2x  5 y  19 2 dUcenH BIrcMnYnEdlRtUvrkKW 62.4 nig 40.4 .
-tam 1 nig 2 eyIg)anRbB½n§smIkar ³
6 x  4 y  18.5   1
eRbIviFIbUkbM)at; 12. KNnacMnYnTaMgBIrenaH ³

2 x  5 y  19 3
 -tag x nig y CaBIrcMnYnEdlRtUvKNnaenaH
 6 x  4 y  18.5

6 x  15 y  57
-bRmab;RbFan ³
11y  38.5
naM[ y  3811.5  3.5 ÷BIrcMnYnmanplbUkesµI 490 KW x  y  490 1
cMeBaH y  3.5 CMnYskñúg 2 eyIg)an ³ ÷BIrcMnYnmanpleFobesµInwg 73 KW xy  73 2
2 : 2 x  53.5  19
2 x  19  17.5 -tam 1 nig 2 eyIg)anRbB½n§smIkar ³
x
1.5  x  y  490  x  y  490
 
2 x 3 b¤
x 9 tamlkçN³
x  0.75 y  7  3  7

ebITijnM H 4 nignM P 4 enaHeKRtUvGab;R)ak;KW ³ smamaRt x y x  y 490
    49
3 7 3  7 10
20  4  0.75  4  3.5  20  3  14   3 >
naM[ x  3 49  147 nig y  7  49  343
dUcenH eKnwgGab;R)ak;[eyIgvijcMnYn 3$ . dUcenH cMnYnTaMgBIrenaHKW 147 nig 343 .
173
13.KNnacMnYnnImYy² ³ -tam 1 nig 2 eyIg)anRbB½n§smIkar ³
-tag x nig y CaBIrcMnYnEdlRtUvKNnaenaH BC  CA  AB  192

 BC CA AB
-bRmab;RbFan ³ 
 2.5

2

1.5
÷pleFobénBIrcMnYnesµInwg 54 KW ³ xy  54 tamlkçN³énsmamaRt eyIg)an ³
BC CA AB BC  CA  AB 192
b¤Gacsresr 5x  4y 1      32
2.5 2 1.5 2.5  2  1.5 6

÷kaerénpldkrbs;vaesµInwg 324 ³ enaH BC


2.5
 32  BC  2.5  32  80

eyIg)an x  y   324 b¤ x  y   324


2 CA
 32  CA  2  32  64
2
naM[ x  y  18 2 b¤ x  y  18 3 AB
 32  AB  1.5  32  48
-tam 1 nig 2 eyIg)anRbB½n§smIkar ³ 1.5

x y
 
dUcenH KNna)anRbEvgRCugénRtIekaN ABC
5 4 tamlkçN³énsmamaRt eyIg)an ³ KW BC  80m , CA  64m , AB  48m .
 x  y  18
x y x  y 18
    18 KNnacMnYnnImYy² ³
5 4 54 1 15.
naM[ x  5 18  90 nig y  4 18  72 -tag x nig y CaBIrcMnYnEdlRtUvKNnaenaH
-tam 1 nig 3 eyIg)anRbB½n§smIkar ³ -bRmab;RbFan ³
x y
 
tamlkçN³énsmamaRt eyIg)an ³ ÷pldkénBIrcMnYnesµnI wg 36 KW x  y  36 1
5 4 2
 x  y  18 x
÷kaerénpleFobesµInwg 25 KW   
25
x y x  y  18  
81 y 81
    18
5 4 54 1
enaH x

25
naM[
x 5

x
b¤
5
naM[ x  5  18   90 nig y  4  18   72 y 81 y 9 y 9
x y
b¤Gacsresr x y
 2 b¤
 3
dUcenH cMnYnnImYy²KW 90 nig 72 5 9 5 9
-tam 1 nig 2 eyIg)anRbB½n§smIkar ³
b¤cMnYnnImYy²KW -90 nig -72 .  x  y  36

x y tamlkçN³énsmamaRt eyIg)an³
14.KNnaRbEvgRCug BC , CA nig AB ³  5  9

-bRmab;RbFan ³ x y x  y 36
    9
5 9 59 4
÷RtIekaN ABC manbrimaRt 192m ³ naM[ x  5   9  45 nig y  9   9  81
eyIg)an BC  CA  AB  192 1 -tam 1 nig 3 eyIg)anRbB½n§smIkar ³
÷smamaRtRCugTaMgbIerogKñaKW 2.5 2 nig 1.5 ³  x  y  36

eyIg)an BC 
CA AB
 2 x  y tamlkçN³énsmamaRt eyIg)an³
2.5 2 1.5  5  9

174
x  y x  y 36 18
5

9
  
5  9 14 7
yk A2 CMnYskñúg 2 eyIg)an ³
18  162 2 :  2  B  3 enaH B  5
naM[ 18 90
x  5 
7 7
nig
y   9 
7 7
dUcenH kMNt;)an A  2 nig B  5 .
dUcenH cMnYnnImYy²KW -45 nig -81
b¤cMnYnnImYy²KW 907 nig  162
7
. 18. edaHRsayRbB½n§smIkar ³
2 2 1  1 1 1
x   2  x  2  y  2
16.KNnaRbEvgén)atnImYy² énctuekaNBñay ³ eyIgman 

y 2 b¤ 

1  5 3
 1  5 1  3
-tag B CaRbEvg)atFM nig b CaRbEvg)attUc 
x y 4 
x y 4

-bRmab;RbFan ³ tag X  1x nig Y  1y enaHeyIg)anRbB½n§fµIKW ³


÷pleFob)atctuekaNBñayesµI 53 KW Bb  53 
 2 X  2Y 
1
4 X  4Y  1   1


2 b¤ 
b¤Gacsresr)an b3  B5 1  X  5Y  3

4 X  20Y  3
 4
÷plbUk)atrbs;vaesµInwg 64 KW b  B  64 2 
 4 X  4Y  1
4 X  20Y  3 naM[ 2
b¤ Y  121
-tam 1 nig 2 eyIg)anRbB½n§smIkar ³ 24Y  2
Y
24
b B
 
3 5 tamlkçN³énsmamaRt eyIg)an ³ cMeBaH Y  121 eyIgman X  5Y  34
b  B  64
b B b  B 64
enaH X  34  5Y  34  125  912 5  124  13
   8
3 5 35 8 eday X  1x nig Y  1y enaHeyIg)an ³
enaH b  3  8  24m nig B  4  8  40m
  x  3 nig
1 1 1 1
  y  12
dUcenH RbEvg)attUc 24m nig)atFM 40m . x 3 y 12

kMNt; A nig B ³ dUcenH RbB½n§smIkarmanKUrcemøIy  xy   12


3 
 .
17.    
eyIgmanbnÞat; Ax  By  9 19. etIplitpl A ekInCagplitpl B b¤eT?

-ebIbnÞat; Ax  By  9 kat;tamcMNuc 2 , 1 -kMeNInplitpl A tamqñaMnImYy²KW ³


eyIg)an 2 A  B  9 1 2.7  2.3  3.1  2.7  3.5  3.1  0.4 ¬efr¦

-ebIbnÞat; Ax  By  9 kat;tamcMNuc  3 , 3 -kMeNInplitpl B tamqñaMnImYy²KW ³


eyIg)an  3A  3B  9 b¤  A  B  3 2 3.3  2.9  3.7  3.3  4.1  3.7  0.4 ¬efr¦

-yk 1 - 2 eyIg)an ³ dUcenH plitpl A minGaclk;ekInCaplit


2 A  B  9
 pl B eT eRBaH GRtakMeNInénka
 A  B  3 naM[ A
6
b¤ A  2
3A  6 3 lk;tamqñaMesµIKña .
175
១២

1. x

10 m x

6m
2. ABC ឬ

k> AB  7 , BC  9 , AC  12

x> AB  52.8 , BC  45.5 , AC  69.7

K> AB  83 , BC  49 , AC  67
3. ABC A

15 cm 41cm

B D C
12 cm

4. O 5 cm
 AB
O
8 cm
OM A
//
M
//
B

5. 3 cm
h

6. x y m
x
y
12
50
x 38
6

8 16

7.
18cm

12 cm

176
8. 1 cm
A
9. cm
AB 16

AH
BH HC B H C
20

10. O x
o
Ox Oy
O 100 km

100 km y

DDCEE

177
១២

1. KNna x kñúgrUbxageRkam ³ AC 2  BC 2  AB2


67 2  49 2  832
eyIgmanRtIekaNEkg Edlman x CaRCugCab; 4489  2401  6889
mMuEkg tamRTwsþIbTBItaK½r eyIg)an ³ ¬minBit¦
6890  6889
x  6  10
2 2 2

x 2  100  36 10 m x
dUcenH ABC minEmnCaRtIekaNEkgeT .
x  64
x 8 m 3. KNnaépÞRkLaénRtIekaN ABC ³
6m
eyIgmanRtIekaN ABC dUcrUbxageRkam
dUcenH KNna)an x 8 m . A

2. bBa¢ak;fa ABC CaRtIekaNEkgb¤eT ? 15 cm 41cm

k> AB  7 , BC  9 , AC  12
B C
cMeBaHkrNIenH ebI ABC CaRtIekaNEkgluHRta ³ 12 cm
D

AB 2  BC 2  AC 2 eyIgdwgfa S  12  BC  AD
ABC
7  9  12
2 2 2

49  81  144 -cMeBaHRtIekaNEkg ADBEkgRtg; D eyIg)an ³


¬minBit¦
130  144 AD  DB  AB b¤ AD  AB  DB
2 2 2 2 2 2

AD2  152  122


dUcenH ABC minEmnCaRtIekaNEkgeT .  225  144  81
naM[ AD  81  9 cm
x> AB  52.8 , BC  45.5 , AC  69.7
cMeBaHkrNIenH ebI ABC CaRtIekaNEkgluHRta ³ -cMeBaHRtIekaNEkg ADC EkgRtg; D eyIg)an ³
DC  DA  AC b¤ DC  AC  DA
2 2 2 2 2 2
AB2  BC 2  AC 2
52.82  45.52  69.7 2
DC 2  412  9 2
2787.84  2070.25  4858.09  1681 81  1600

4858.09  4858.09 ¬Bit¦ naM[ DC  1600  40 cm


enaH BC  BC  DC  12  40  52 cm
dUcenH ABC KWCaRtIekaNEkg .
eyIg)an S  12  52  9  234cm
ABC
2

K> AB  83 , BC  49 , AC  67
cMeBaHkrNIenH ebI ABC CaRtIekaNEkgluHRta ³ dUcenH KNna)an S ABC  234 cm 2 .

178
4. KNnaRbEvg OM kñúgrUbxageRkam ³ 6. kMNt;témø x nig y énrUbxageRkamKitCa m ³
-eyIgmanrgVg;p©it O mankaMRbEvg 5 cm nig -eyIgmanrUb ³
AB  manRbEvg 8 cm tamRTwsþIbTBItaK½r y
12
-eday MA MB naM[ O x 2  6 2  82
 36  64  100 6 x
M CacMNuckNþal AB  // //
M A B
naM[ x  100  10
eyIg)an OM  AM Rtg; M 8
ehIy y  12  x cMeBaH x  10
2 2 2

¬eRBaH kaMrgVg;EkgnwgGgát;FñÚRtg;cMNuckNþal¦
enaH y  144  100  44
2

-kñúgRtIekaN OMA EkgRtg; M


b¤ y  44  4 10  2 10
tamRTwsþIbTBItaK½r OM  AM  OA
2 2 2

naM[ OM  OA  AM
2 2 2 dUcenH KNna)an x  10 nig y  2 10 .
eday OA  5 cm nig AM  AB 2
8
  4 cm
2
-eyIgmanrUb ³ x
E

eyIg)an OM  5  4  25  16  9
2 2 2 eyIgKUsbEnßm CD D C
16
enaH OM  9  3cm Edl CD // AB 38
50

tamRTwsþIbTBItaK½r
dUcenH KNna)anRbEvg OM  3cm . x 2  CD 2  CE 2
A B 16

 CD 2  BE  BC 
2

5. KNnaépÞRkLaénRtIekaNsm½gS ³  16 2  50  38


2

-ebIRtIekaNsm½gSmYyman  256  144  400


rgVas;RCug a enaHkm<s;rbs; h naM[ x  400  20  20 2

vakMNt;eday h  a 2 3 dUcenH KNna)an x  20 .


Taj)an a  2h3 b¤ a  2 33h 7. KNnakm<s;énekaN rYcKNnamaDénekaNenaH ³

-épÞRkLaénRtIekaNsm½gSKW ³ tamRTwsþIbTBItaK½r
h 2  182  122
S  ah b¤ S  
1 1 2 3h 3
h  h 2
 324  144  180
18cm
2 2 3 3
eday h  3 cm naM[ h  180  6 5 cm
12 cm

naM[ S  33   3   33  3  3 cm
2 2 ehIy V  13 S h  13  r h
b
2

b¤ V  13   12  6 5  288
2
5 cm3
dUcenH KNna)anépÞRkLaénRtIekaN
sm½gSKW S  3 cm . 2 dUcenH KNna)an km<s; h  6 5 cm
nigmaD V  288 5 cm 3
.
179
8. KNnaRbEvgGgát;RTUgénKUbenaH ³ K> KNna BH nig HC ³
eyIgmanKUbEdlman A 1cm B -kñúgRtIekaNEkg AHB EkgRtg; H eyIg)an ³
rgVas;RCugesµI 1cm D C BH 2  AB2  AH 2
2
 48 
RbEvgGgát;RTUgKW AG E F  12 2     144 
 5 
2304 1296
25

25
tamRTwsþIbTBItaK½r H G
 36 
2

naM[ BH 
1296
   
36
cm
-cMeBaH RtIekaN EHG EkgRtg; H eyIg)an ³ 25  5  5
EG2  GH 2  EH 2
 1 1  2
2 2
dUcenH KNna)an BH 
36
5
cm  7.2 cm .
-cMeBaH RtIekaN AEG EkgRtg; G eyIg)an ³ -kñúgRtIekaNEkg AHC EkgRtg; H eyIg)an ³
AG  EG  AE
2 2 2

 2 1  3
2
naM[ AG  3 cm HC 2  AC 2  AH 2
2
 48  2304 4096
 16 2     256  
dUcenH KUbEdlmanRCugesµI 1cm  5  25 25

vamanGgát;RTUg 3 cm .
2
 64 
naM[ HC 
4096
25
   
 5 
64
5
cm

9. KNnargVas;RCugénrUbKitCa cm ³ dUcenH KNna)an BC 


64
cm  12.8 cm .
A 5
16
¬manviFIRsYlCagenH Et´cg;[GñkGnuvtþRTwsþIbTBItaK½r¦
B H C
10. KNnacm¶aypøÚvEdlynþehaHnImYy² ehaH)an ³
k> KNna AB ³ 20 edayynþehaHmanel,ÓnesµIKña ehIyehaHecjBI
kñúgRtIekaN ABC EkgRtg; A eyIg)an cMNucEtmYyenAem:agEtmYy naM[ cm¶aypøÚvesµIKña
AB2  BC 2  AC 2 -tag x Cacm¶aypøÚvénynþehaHnImYy² ¬km¦
 202  162  400  256  144 tamRTwsþIbTBItaK½r eyIg)an ³
naM[ AB  144  12  12cm 2 x 2  x 2  1002
o x
2 x 2  10000
dUcenH KNna)an AB  12cm x 2  5000 100 km
x  5000
x> KNna AH ³
x  50 2 km y
tamlkçN³énRtIekaNEkg ABC mankm<s; AH x  70.71km
eyIg)an AH  BC  AB AC dUcenH ynþehaHTaMgBIrehaH)ancm¶ay
naM[ AH  ABBC
 AC
b¤ AH  122016  485 cm esµIKñaRbEvg 70.71 km .
dUcenH KNna)an AH 
48
5
cm  9.6cm

180
កំណែលំហាត់តាមមមមរៀន គែិតវិ ទ្យា អាន. គិត. យល់

១៣

1. O PT T៖
P
T
O ao
bo
o
50 O
O 40 o b o P
T ao
T
P
a a b b

O A
 O P
bo  130o
65 o
a o  bo
ao ao O
P co bo B
T S T
o
35 P T

a b a,b c AOT BOP


a b
2. PT PV O T V

T T
22o c

( ) O
a
P ( ) O b 42o P
b
c a

V V
T b
25o T
a
b c
( ) O P ( ) 
a 21o
P
X
O
c
V

3. O PA PB A B
r d cm
A A
12
5
17 d
O P P O
r
15
B B A
4. O PA PB 2
b
A B O

៖ P c
cm
9 B

181
កំណែលំហាត់តាមមមមរៀន គែិតវិ ទ្យា អាន. គិត. យល់

b c
OAPB OAPB

A
10
5. AB , BC AC F
D,E F
D
ABC cm 5
C
B E 15

6. PA PB O
A
3 cm AP  82.4 cm AOB  140o 82.4
3 P
៖ o
O  140
APOB
 22  B
AOB   
 7 
A
7. O AT BT 20 cm O
Q
TQO TQ = 10 cm 10

T 20 B

8. O 4.5 cm
A
TA TB A B 4.5 9

TA = 9 cm O T
OATB
B
OATB

9. A B D C
4 cm A
CB CD B D A 
B
B DA DC
A C ABCD

10. TA TB O A
5 cm ATB  60o ៖
T 60 o O
AOB
5
 22 
AOB    B
 7 

182
កំណែលំហាត់តាមមមមរៀន គែិតវិ ទ្យា អាន. គិត. យល់

A
11. O 8 cm 10 cm
AB T T O

AB ( : OA OB )
B

12. MA , MB MC C
A
O O’ MA = MC
 B 
O O

13. EF EG H
F
JEF  22 ៖
o

EGH E
J H
22 o
HGF 5

FGE G

DDCEE

183
កំណែលំហាត់តាមមមមរៀន គែិតវិ ទ្យា អាន. គិត. យល់

មេម ៀនទី១៣ ង្វង្ន


់ ិង្បន្ទាត់

1. kñúgrUbnImYy²énrgVg;p©it O man PT CabnÞat;b:H ³ X> rktémøénmMu a nig b ³


k> rktémøénmMu a ³ eday OT  TP O

eday OT  TP O naM[ OTP 90 o


a o
bo

S
naM[ OTP  90 enaH a90
T
o
50 o o
35 o
 55 o o
P 35o

¬eRBaH bnÞat;b:HRtUvEkg T a o
ehIy b 2STP
o
¬mMup©itesµIBIrdgmMuBiess¦
P
nwgkaMrgVg;Rtg;cMNucb:H¦ enaH b 23570
o o o

eyIg)an a  180  50  90   40


o o o o o
dUcenH rk)antémømMu a  55 nig b  70 . o o

dUcenH rk)antémømMu a  40 . o
g> rktémøénmMu a , b nig c ³
eday OTTP O

65o
x> rktémøénmMu a nig b ³ P naM[ OTP 90 P oa o
c o
bo
T
eday OT  TP a o

bo
enaH a90
o
 65o
 25 o T o

naM[ aOTP 90 o


O
ehIy c o 65

o
32 .5 32 3 0 o o

2
smµtikmµ ³ PT  OT ¬eRBaH mMucarwkesµIknøHmMup©itEdlmanFñÚsáat;rmY ¦
naM[ PTO CaRtIekaNEkgsm)at eday cb
o o
OTP  90 o

enaH mMu)atnImYy²manrgVas;esµIKñaesµInwg 45 o

naM[ b  90  c  90  32.5  57.5


o o o o o o

eyIg)an b  45 o

dUcenH a  25 , b  57.5 , c  32.5 .


o o o

dUcenH rk)antémømMu a  90 nig b  45 .


o o

c> rktémøénmMu a nig b ³


mMu a  1302  65
o

K> rktémøénmMu b ³ o A o
P
o
130o
 bo
a
eday OT  TP ¬eRBaH mMucarwkesµIknøH B T
O

naM[ OTP  90 o O
40 o b o P mMup©itEdlmanFñÚsáat;rYm¦
eyIg)an b  90  40
o o o
T ehIymMu b  180  OTP  TOP  Edl
o o

b¤ b  50
o o
OTP  90 nig TOP  180  130  50
o o o o

naM[ b  180  90  50   40


o o o o o

dUcenH rk)antémømMu b  50 . o

dUcenH rk)an a  65 nig b  40 . o o o

184
កំណែលំហាត់តាមមមមរៀន គែិតវិ ទ្យា អាន. គិត. យល់

1. eyIgmanrgVg;p©it O ehIy PT nig PV CabnÞat;b:H eday PT b:HrgVg;Rtg; T naM[ OTP  90o

k> rkrgVas;mMuEdlminsÁal;KW a , b nig c ³ kñúgRtIekaNEkg OTP man


T
22o 
b  180 o  90 o  25 o  65 o 
O
a
P
RtIekaN OTV CaRtIekaNsm)at ¬ OT  OV ¦
man TOV  2b  2  65  130
b
o o
c
V
naM[ a  c  180  130  2  25o o o

eday PT nig PV CaknøHbnÞat;BIrKUs


ecjBIcMNuc P EtmYy dUcenH a  25 , b  65 , c  25 .
o o o

eTAb:HnwgrgVg;erogKñaRtg; T nig V X> rkrgVas;mMuEdlminsÁal;KW a,b nig c ³


enaH OP CaknøHbnÞat;BuHmMu TPV nig TOV b
T
naM[ c  22 nigmMu OTP  90
o o
c
21o
eyIg)an a  b  180  90  22   68
a
o o o o X  P
O

dUcenH a  68 , b  68 , c  22 .
o o o

kñúgRtIekaNEkg OTP man OTP  90 o

x> rkrgVas;mMuEdlminsÁal;KW a,b nig c ³ naM[ a  180  90  21   69


o o o o

T
c
ehIy TOX CaRtIekaNsm)at ¬ OT  OX ¦
O b 42o P naM[ mMu)at b  c
a
cMeBaH RtIekaN TOX man b  c  a
V
¬plbUkmMukñúgBIr esµInwgmMueRkAmYyminCab;nwgva¦
eday PT nig PV CaknøHbnÞat;BIrKUsecjBIcMNuc o

P EtmYyeTAb:HnwgrgVg;erog KñaRtg; T nig V


naM[ b  b  a b¤ 2b  a enaH b  a2  692
naM[ a  c  90 o
dUcenH a  69 , b  34 .5o , c  34 o 30 .
cMeBaHctuekaN OTPV manplbUkmMukñúgesµI 360 o

enaH b  360  90  90  42   138


o o o o o 2. -KNnargVas;kaM r KitCa cm ³
dUcenH a  90 , b  138 , c  90 .
o o o
eday PB b:HrgVg;Rtg; B A

naM[ OBP CaRtIekaN O 17  P

K> rkrgVas;mMuEdlminsÁal;KW a , b nig c ³ EkgRtg; B 15


r

a
T
25o tamRtwsþIbTBItaK½r cMeBaHRtIekaNEkg OBP
r  17  15 b¤ r  64 enaH r  8 cm
b
2 2 2 2
O P

c
V dUcenH KNna)anrgVas;kaM r  8 cm .
185
កំណែលំហាត់តាមមមមរៀន គែិតវិ ទ្យា អាន. គិត. យល់

-KNnargVas; d KitCa cm ³ K> KNnabrimaRténctuekaN OAPB ³


A POAPB  OA  AP  PB  OB
12
5  2  b  9  c  2  9  9  2  22 cm
d
P O

dUcenH KNna)anbrimaRt P OAPB  22 cm .


B

eday PA b:HrgVg;Rtg; A naM[ OAP CaRtIekaN X> KNnaépÞRkLaénctuekaN OAPB ³


EkgRtg; A eday OAP nig OBP CaRtIekaNEkgBIb:unKña
tamRTwsþIbTBItaK½r cMeBaHRtIekaNEkg OAP ehIy S  S  S
OAPB OAP OBP

eyIg)an OP  12  5  144  25  169


2 2 2 naM[ S  2S  2  12 9  2  18 cm
OAPB OAP
2

naM[ OP  169  13 cm dUcenH KNna)anépÞRkLa S OAPB  18 cm 2 .


eday OP  d  OA ¬eRBaH OA CakaMrgVg;¦
eyIg)an d  OP  OA  13  5  8 cm 4. KNnaépÞRkLaén RtIekaN ABC KitCa cm ³ 2

dUcenH KNna)an d  8 cm . A
10
F
3. eyIgmanrUbdUcxageRkam manrgVas;KitCa cm ³
D
A 5
b
2 C
B E 15
O
P c

9 B

k> KNnargVas; b ³ eday ABC CaRtIekaNEkgRtg; B ³


eday PA nig PB CaknøHbnÞat;BIrKUsecjBIcMNuc naM[ S  12 AB  BC
ABC

P EtmYy eTAb:HnwgrgVg;erogKñaRtg; A nig B ehIy BE  BD  5cm ¬ D nig E CacMNucb:H¦


naM[ PA PB b¤ b  9 cm nig AD  AF  10cm ¬ D nig F CacMNucb:H¦
enaH AB  AD  BD  10  5  15 cm
dUcenH KNna)an b  9 cm . BC  BE  EC  5  15  20 cm >
x> KNnargVas; c ³ eyIg)an S  12 15 20  150 cm
ABC
2

eday OA  OC ¬kaMrgVg;Etp©it O mYy¦ dUcenH KNna)anépÞRkLa S  150 cm 2 .


ABC
naM[ c  2 cm
dUcenH KNna)an c  2 cm .

186
កំណែលំហាត់តាមមមមរៀន គែិតវិ ទ្យា អាន. គិត. យល់

5. k> KNnaépÞRkLactuekaN APBO KitCa cm ³ 2


7. k> rkbrimaRténctuekaN OATB ³
A A
82.4
9
3 P 4.5
o
O  140 O T

B B

eday OAP nig OBP CaRtIekaNEkgBIb:unKña brimaRt P  OA  AT  TB  OB


OATB

ehIy S APBO S S
OAP naM[ OBP
eday OA  OB  4.5 cm ¬kaMénrgVg;EtmYy¦
S OAPB  2S OAP  2 
1
 AO  AP ehIy TA  TB  9 cm ¬eRBaH A nig B Ca
2
cMNucb:HrgVg;KUsecjBIcMNuc T EtmYy ¦
 2
1
3  82.4  247.2 cm 2
2 eyIg)an P  4.5  9  9  4.5  27 cm
OATB

dUcenH épÞRkLa S  247 .2 cm . APBO


2
dUcenH KNna)anbrimaRt P  27 cm . OATB

x> KNnaépÞRkLaceRmókfastUc AOB ³


x> rképÞRkLaénctuekaN OATB ³
tamrUbmnþ épÞRkLaceRmókfasEdlmanmMu 

eday OAT nig OBT CaRtIekaNEkgBIb:unKña
nigrgVas;kaM R KW S  R  360
AOB
2
o
ehIy S  S  S naM[
OATB OAT OBT

eday   140 , R  3cm ehIyyk   227


o
S OATB  2 S OAT  2 
1
OA  AT 
2
naM[ S  227  3  140
o
 11 cm2 2
AOB
360 o
 2
1
4.5  9  40.5 cm 2
2
dUcenH épÞRkLaceRmókfas S  11 cm 2 .
AOB
dUcenH épÞRkLa S  40 .5 cm OATB
2
.
6. rkrgVas;énkaMrgVg; ³ 8. rképÞRkLaénctuekaN ABCD ³
A
eday TB b:HrgVg;Rtg; O 
D C

B enaH TBO CaRtI


Q
10
ctuekaN ABCD A B  

ekaNEkgRtg; B T 20 B
man CB  CD eRBaH B
tamRTwsþIbTBItaK½r cMeBaH RtIekaNEkg TBO nig D CacMNucb:HénrgVg;p©it A nig DA  DC
TO 2  TB 2  BO2
10  r 2  202  r 2
eRBaH A nig C CacMNucb:H énrgVg;p©it B ehIy
100  20r  r 2  20 2  r 2 mMu ADC  90 ¬eRBaHbnÞat;b:HEkgnwgkaMrgVg;¦
o

20r  400  100 rgVg;p©it A nigrgVg;p©it B manrgVas;kaMesµIKña 4 cm


300
r
20
 15 naM[ ABCD CakaerEdlmanRCugesµI 4 cm
dUcenH KNna)anrgVas;kaMrgVg; r  15 cm . dUcenH épÞRkLa S  4  16 cm .ABCD
2 2

187
កំណែលំហាត់តាមមមមរៀន គែិតវិ ទ្យា អាន. គិត. យល់

9. k> KNna AOB ³ 11. bgðajfa MA  MC ³


A
-eyIgman MA  MB M

T 60 o O ¬eRBaH MA nig MB A C
5

B
CabnÞat; KUsecjBI 
O
B O

cMNucEtmYyehIy
edayctuekaN OATB manmMu OAT  90 nig o
b:HrgVg;p©it O Rtg; A nig B
OBT  90 ¬eRBaH TA nig TB CabnÞat;ehIy
o
-mü:ageTot MB  MC ¬eRBaH MB nig MC Ca
EkgnwgkaMrgVg;Rtg; A nig B ¦ bnÞat;KUsecjBIcMNucEtmYy ehIyb:HrgVg;p©it O
naM[ AOB  360  90  90  60   120
o o o o o
Rtg; B nig C
dUcenH KNna)an AOB  120 . o eXIjfa MA  MB nig MB  MC
dUcenH MA  MC RtUv)anbgðaj .
x> KNnaépÞRkLaénceRmókfatUc AOB ³
tamrUbmnþ épÞRkLaceRmókfasEdlmanmMu  12. k> KNna EGH ³ F

nigrgVas;kaM R KW S  R  360 AOB
2
o
E
J H
22 o
eday   120 , R  5 cm ehIyyk   227
o 5

G
naM[ S  227  5  120
o
 26 .19 cm 2 2
AOB
360 o
eday EG CaknøHbnÞat;b:HnwgrgVg;p©it H
dUcenH épÞRkLaceRmókfas S AOB  26 .19 cm 2 enaH EG  GH naM[ EGH  90 o

10. KNnargVas; AB ³ dUcenH KNna)an EGH  90 . o

eyIgKUsP¢ab; OA nig OB bEnßm x> KNna HGF ³


eday AB b:HrgVg;tUc A
eday EF  EG nig HF  HG ¬kaMrgVg;EtmYy¦
naM[ AT  TO > T 
O naM[ EH CaemdüaT½rén FG enaH EH  GF
tamRTwsþIbTBItaK½r eyIg)an HGF  GEH  22 ¬eRBaH mMuman
o

B
AT  OA  OT >
2 2 2
RCugEkgerogKñaKW EG  GH nig EH  GF ¦
 10  8  36 enaH AT  36  6 cm
2 2
dUcenH KNna)an HGF  22 . o

mü:ageTot AB  2 AT  2  6  12 cm
K> KNna FGE ³
¬eRBaHGgát;FñÚEkgnwgkaMrgVg;Rtg;cMNuckNþal¦
FGE  EGH  HGF  90  22  68 >
o o o

dUcenH KNna)anrgVas; AB  12 cm . dUcenH KNna)an FGE  68 . o

188
១៤

B
1. O AB = AC A
65 o

ABC  65o ACB ABO o D

A
D
ACD , BAE
E
2.
130o
BDC 45 o C
B

A B
38 o
3. O AB CD
BCD , ADC ABC
C D

A
C
y 60 o
34 o P
4. x y x
D
B
A

5. O AC BD I
40 o 60 o B
CAB  40 , AIB  60
o o
I
. ACD ខ. CAD
D C

6. A, B, C O o

AOC  120o ABC 120o

A
C
B
C

25 o 45 o
7. O AD BC D o
ACB  45 , ACD  25
o o

. ADC ខ. OAB
A B

189
A

8. A, B, C D O 70 o
B
AB CD P CBD  35o , o
35 75 o
CAP  70 o
DBP  75 o

P
. ACD ខ. APC . DAB D
C

C
9. O AC BA
CD T AT = AC ATD  20o D o

20 o
T
. ADB ខ. BOC BDC A B

10. O QS PR M 134o

PR POQ  134o o

. PRQ ខ. RPQ RSQ


R M P
S
S

11. O PQ R
RS ROQ  48o o
48 o
. OQR ខ. RST TRQ T

P R Q

A
110o D
12. O ABC BC x
40 o
y z
C AE D x, y z B  E
o C

B A
13. ABCD AB = BC
ACE  25o CD E ADE  80o 80 o E
. ABC ខ. BAD 25 o D
C T

L K 56 o J
14. TKXM SNXL 40 o

JKL TKJ  56o NJK  40o X

. MLN ខ. LMN . KLN


M N

3 S

190
១៤

1. KNnargVas;mMu ACB nig ABO ³ 3. KNnargVas;mMu BCD , ADC nig ABC ³


A B
38 o
B
A
65 o

o D

C D
C

-eyIgman AC  AB nigmMu ABC  65 o mMu BCD  BAD  38 ¬manFñÚsáat;rYm BD¦ o

naM[ ABC CaRtIekaNsm)atEdlmanmMu)at mMu ADC  BAD  38 ¬mMuqøas;kñúgeRBaH AB// CD ¦


o

ACB  ABC  65 > o mMu ABC  ADC  38 ¬manFñÚsáat;rYm AC ¦


o

ehIymMu ACB  ADB  65 ¬FñÚsáat;rYm AB ¦ o


dUcenH BCD  ADC  ABC  38 o

-eday OB  OD ¬kaMrgVg;EtmYy¦ enaH OBD


CaRtIekaNsm)at Edlman ³ 4. rkrgVas;mMu x nig y ³
OBD  ADB  65 ¬mMu)atRtIekaNsm)at¦o
rebobTI1 ¬rktameKalbMNgénemeron¦
nig ABD  90 ¬mMucarwkknøHrgVg;Ggát;p©it AD ¦
o A
C
naM[ ABO  ABD OBD  90  65  25 o o o
y 60 o
34 o P
x
D
dUcenH rgVas;mMu ACB  65 nig ABO  25 o o
B

2. rkrgVas;mMu ACD , BAE nig BDC ³


eyIgman mMu x  y ¬eRBaHmanFñÚsáat;rYmKña CD ¦
A eday mMueRkArgVg;  12 ( FñÚsáat;FM  FñÚsáat;tUc )
D

45 o
130o
E

C
b¤Gacsresr P  12  AB  CD
B
naM[  CD  AB  2  P
-mMu ACD  ABD  45 ¬mMumanFñÚsáat;rYm AD ¦
o
eday  AB  2  ACB  2  60  120 o o

-mMu BAE  130  45  85 ¬eRBaH RtIekaN


o o o
ehIy P  34 o

ABE manplbUkmMukñúgBIresµImMueRkAmYy¦ eyIg)an  CD  120  2  34  52 o o o

-mMu BDC  BAC  BAE  85 o


naM[ x  y  12  CD  12  52  26 o o

¬mMumanFñÚsáat;rYm BC ¦
dUcenH KNna)anrgVas;mMu x  y  26 o .
dUcenH ACD  45 , BAE  BDC  85
o o

191
rebobTI2 ¬rktamcMeNHdwgmanRsab;¦ 6. KNnargVas;mMu ABC ³
A
C
y 60 o
34 o P o

120o
x
D C
A
B
B
kñúg BCP manmMu x  60  34  26 ¬eRBaHo o o
eyIgman 1
ABC   CA Edl CA  COA
plbUkmMukñúgBIrénRtIekaNesµImMueRkAmYy¦ 2
Et mMuqk COA  360 120  240
o o o

eyIgman mMu x  y ¬eRBaHmanFñÚsáat;rYmKña CD ¦


naM[ ABC  12  CA  12  240  120 o o

dUcenH x  y  26 . o

dUcenH KNna)an ABC  120o .


5. k> KNnargVas;mMu ACD ³
A 7. k> KNnargVas;mMu ADC ³
C
o
40 60 o B 25 o 45 o
I
D o
D C
kñúgRtIekaN ABI man IAB  40 , AIB  60 o o A B

naM[ ABI  ABD  180  40  60   80 o o o o eday AD// BC naM[ ADC  BCD  180 o

enaHmMu ACD ABD  80 ¬FñÚsáat;rYm AD ¦ o Et BCD  BCA  ACD  45  25  70 o o o

enaHeyIg)an ADC  180  BCD o

dUcenH KNna)anrgVas;mMu ACD  80 . o

 180  70  110 > o o o

x> KNnargVas;mMu CAD ³ dUcenH KNnargVas;mMu ADC  110 . o

tamsmµtikmµ AB  BC enaH ABC sm)at


vi)ak mMu)at ACB  CAB  40 o x> KNnargVas;mMu OAB ³
ehIy ADB  ACB  40 ¬FñÚsáat;rYm AB ¦ o eday OA  OB ¬kaMrgVg;EtmYy¦ enaH OAB Ca
kñúgRtIekaN ADI manplbUkmMu RtIekaNsm)at Edlman OAB  OBA
ADB  CAD  AIB ¬mMueRkArgVg;¦
ehIy AOB  2ACB  2  45  90 o o

naM[ CAD  AIB  ADB ¬eRBaH mMup©itesµIBIrdgénmMucarwkmanFñÚsáat;rYm¦


eyIg)an OAB  OBA  180 2 90  45
o o

 60  40  20 >
o
o o o

dUcenH KNna)an CAD  20 . o


dUcenH KNna)an OAB  45o .
192
8. k> KNnargVas;mMu ACD ³ 9. k> KNnargVas;mMu ADB ³
A C

70 o
B D o
35 o 75 o
20 o
T B
P A
D
C
eday AT  AC enaH ATC CaRtIekaNsm)at
edaymMu ABD CamMuCab;bEnßmnwgmMu DBP vi)ak ATD  ACT  20 o

naM[ ABD  180  DBP  180  75  105


o o o o
ehIy DBA  ACD  20 ¬FñÚsáat;rYm AD ¦ o

ehIy ABCDCactuekaNcarwkkñgú rgVg;enaHvaman mü:ageTot ADC  90 ¬mMucarwkknøHrgVg;¦


o

plbUkmMuQmKñaesµIngw 180 o
enaHmMu ADT  ADC  90 ¬mMubEnßmKña¦ o

eyIg)an ACD  ABD  180 o


cMeBaH ADT ³ TAD  180  ATD  ADT  o

enaH ACD  180  ABD  180 105  75


o o o o
b¤ TAD  180  20  90   70
o o o o

dUcenH KNna)an ACD  75 . o Et ADB  DBA  TAD ¬mMueRkA ABD¦


b¤ ADB  TAD  DBA  70  20  50 o o o

x> KNnargVas;mMu APC ³


dUcenH KNna)an ADB  50 . o

eday CAB  CDB  180 ¬mMuQm ABCD¦ o

naM[ CDB  180  CAB


o
x> KNnargVas;mMu BOC ³
 180  70  110 >
o o o
eyIgman BOC sm)at eRBaH OB  OC
kñúg BDP man DBP  APC  CDB enaHmMu)at BCA  OBC  ADB  50 o

¬eRBaHplbUkmMukñúgBIrénRtIekaNesµInwgmMueRkAmYy¦ ¬eRBaH BCA nig ADB manFñÚsáat;rYm AB ¦


naM[ APC  CDB  DBP kñúg BOC man BOC  180  BCA  OBC  o

 110  75  35 >
o o o
naM[ BOC  180  50  50   80
o o o o

dUcenH KNna)anrgVas;mMu APC  35 . o


dUcenH KNna)anrgVas;mMu BOC  80 . o

K> KNnargVas;mMu DAB ³ K> KNnargVas;mMu BDC ³


man CAD  CBD  35 ¬manFñÚsáat;rYm CD ¦
o
eday BDC  ADC  ADB
ehIy DAB  CAB  CAD  70  35  35 o o o
naM[ BDC  90  50  40
o o o

dUcenH KNna)anrgVas;mMu DAB  35 . o


dUcenH KNna)anrgVas;mMu BDC  40 . o

193
10. k> KNnargVas;mMu PRQ ³ x> KNnargVas;mMu RST ³
Q
eday RST  12 ROT  12  48  24 o o

o
134o
¬eRBaH mMucarwkesµIknøHmMup©itEdlmanFñÚsáat;rmY TR ¦
R M P
dUcenH KNna)anrgVas;mMu RST  24 . o

S
eday 1
2
1
PRQ  POQ   134o  67o
2
K> KNnargVas;mMu TRQ ³
¬eRBaH mMucarwkesµInwgknøHmMup©itEdlmanFñÚsáat;rYm¦ eday TRQ  RST  24 ¬mMucarwk nigmMu o

Biess EdlmanFñÚsáat;rYm TR ¦
dUcenH KNna)anrgVas;mMu PRQ  67 . o

dUcenH KNna)anrgVas;mMu TRQ  24 . o

x> KNnargVas;mMu RPQ ³


eday QS RbsBVnwg PR Rtg;cMNuckNþal M 12. KNnargVas;mMu x , y nig z ³
naM[ QS CaemdüaT½rén PR eFVI[ QR  QP A
110o D
eyIg)an PQR CaRtIekaNsm)at kMBUl Q 40 o
x
y z
 E
vi)akmMu)atTaMgBIr RPQ  PRQ  67 o
B o C

dUcenH KNna)anrgVas;mMu RPQ  67 . o

eday ABCD CactuekaNe):ag carwkkñúgrgVg;


K> KNnargVas;mMu RSQ ³ enaHplbUkmMuQmrbs;va RtUvEtesµI 180 naM[ o

eday RSQ  RPQ  67 ¬manFñÚsáat;rYm QR ¦


o B  ADC  180o
 ADC  180o  B  180o  40o  140o
dUcenH KNna)anrgVas;mMu RSQ  67 . o
ehIy A  BCD  180 o

11. k> KNnargVas;mMu OQR ³  BCD  180 o  A  180 o  110 o  70 o >


S
naM[ x  180o  ADC  180o  140o  40o

y  180 o  BCD  180 o  70 o  110 o >


o cMeBaHRtIekaN CDE man x  y  z  180 o

48 o

T
naM[ z  180 o
 x  y 

P R Q
 180o  40o  110o  
 180o  150o  30o
eday PQ b:HrgVg;Rtg; R enaH ORQ Ca  
naM[ OQR  180  90  48   42
o o o o dUcenH KNna)anrgVas;mMudUcteTA ³
x  40 o , y  110 o , z  30 o .
dUcenH KNna)anrgVas;mMu OQR  42 . o

194
13. k> KNnargVas;mMu ABC ³ eyIgman MKN  NJK  40 ¬eRBaH mMu o

B A
Biess nigmMucarwkmanFñÚsáat;rYm KN énrgVg;FM¦
ehIy MLN  MKN  40 ¬eRBaH mMucarwk
o

25 o
80 o
D
E
manFñÚsáat;rYm MN énrgVg;tUc¦
C
dUcenH KNna)anrgVas;mMu MLN  40 . o

eday ADC CamMuCab;bEnßmnwgmMu ADE


naM[ ADC  180  ADE  180  80  100
o o o o x> KNnargVas;mMu LMN ³
ehIy ABCD CactuekaNe):ag carwkkñúgrgVg; eday JKL enAelIbnÞat;EtmYy nig TKXM Ca
enaHplbUkmMuQmrbs;va RtUvEtesµI 180 o bnÞat;b:H enaHeyIg)anmMuTl;kMBUl
naM[ ABC  ADC  180 o LKM  TKJ  56 > o

b¤ ABC  180  ADC  180 100  80


o o o o naM[ LKN  LKM  MKN
b¤ LKN  56  40  96
o o o

dUcenH KNnargVas;mMu ABC  80 . o

mü:ageTot LMNK CactuekaNcarwkkñúgrgVg;tUc


x> KNnargVas;mMu BAD ³ naM[ plbUkmMuQm LMN  LKN  180 o

eday AB  BC enaH ABC CaRtIekaNsm)at LMN  180o  LKN


 180o  96o  84o
vi)ak ACB  BAC  12 180  ABC o

dUcenH KNna)an LMN  84o .


b¤ ACB  12 180  80   50 o o o

naM[ BCD  ACB  ACD K> KNnargVas;mMu KLN ³


b¤ BCD  50  25  75 o o o eyIgman MKN  LNK  NJK  40 o

eday BCD  BAD  180 ¬mMuQmKña¦ o ¬ mMuBiess nigmMucarwkmanFñÚsáat;rYm KN énrgVg;FM¦


b¤ BAD  180  BCD  180  75  105
o o o o ehIy LKN  LKM  MKN
b¤ LKN  56  40  96
o o o

dUcenH KNnargVas;mMu BAD  105 . o

kñúg LKN manplbUkmMukñúgesµIngw 180 enaHo

14. k> KNnargVas;mMu MLN ³ KLN  180o  LKN  LNK 


T  180o  96o  40o   44o
L K 56 o
40 o
J
dUcenH KNna)anrgVas;mMu KLN  44 . o

M N

195
១៥

1. AB AB  6 cm 5
2. AB AB  11 cm 3 4 5
3. AB AB  12 cm 3 5 7
3
4. AB  7 cm C AB
5
5
5. AB  9 mm C D AB
2

6. AB / /CD / / EF / /GH
. DF FH AC  6 cm , CE  12cm A B
EG  4cm BD  9cm C D
ខ. BD , DF FH AC  6 cm ,
CE  12cm , EG  4cm HB  33cm E F
. DF CG AC  6 cm , CE  12cm G H
DB  8cm FH  9cm

7.  AB / /  MN  x
A

A A
8 x
9 8 x
M M
N M N
16 x 6
3 4
B 4 N
C B C B C
12

A
4
8. BE / /CD 8
E
12
ED CD B D
cm 2
C

6
9. a b a
b
cm 4
2
8

196
10. 13
kaMTI13
2.40 m 3.40 m A
. ABC B
1 muxQrénkaM
AB
20
BC
ខ. AB kaMTI1
BC C
B 3.40m

1
. AB BC 1 cm 20 cm
20
11. OX OY OX I J OI = 2 cm IJ = 3 cm OY
I’ , J’ OI’ = 3 cm I’J’ = 4.5 cm II’ JJ’
12.

1.65 m 6.50 m
10 m
.
ខ.
13. ABCD E E BC
AB F E CD AD G
. AGE ADC AFE ABC
ខ. FG // BD

DDCEE

197
១៥

1. EckGgát; AB  6 cm Ca 5 cMENkesµIKña ³ eyIgKUsbnÞat; d  rYcRkit[)an AC  3 /


A   B  CD  5 nig DE  7 rYcKUsP¢ab; EB bnÞab;mk
/
CM , DN [Rsbnwg EB enaH AB  12 cm
/
/
/
/

x RtUv)anEckCaGgát;smamaRtnwg 3 5 7 .
eyIgKUsknøHbnÞat; Ax rYcedARbeLaHRkit 5Ékta 4. rkcMNuc C EdlEckGgát; AB tampleFob 53 ³
esµI²KñaelI Ax rYcKUsP¢ab;BIcMNcu 5 ÉktaeTAkan; A
C
B 

cMNuc B ehIyKUsGgát;bnþbnÞab;[RsbKña. /
/
/
M /

tamRTwsþIbTtaEls AB  6 cm RtUv)anEck x
/
/
/
/

Ca 5 cMENkesµI²Kña . N

2. EckGgát; AB  11 cm CaGgát;smamaRtnwg eyIgKUsknøHbnÞat; Ax rYcRkit 8 RbeLaHesµI²Kña


3 4 5 ³ Edl AM  3 , MN  5 ehIyKUsP¢ab; NB nig
A / /
M

N B 
KUs MC [Rsbnwg NB enaHeyIg)ancMNuc C
C
/ /
/ /
/ / EdlEckGgát; AB tampleFob 53 .
D / /
/ /
d 
5. kMNt;cMNuc C nig D EdlEck AB tampleFob
E 5
2
eyIgKUsbnÞat; d  rYcRkit[)an AC  3 / M l 
\
\ \
N
CD  4 nig DE  5 rYcKUsP¢ab; EB bnÞab;mk
\ \
\ \ \
\
\ \
\ \
C B
\ \
CM , DN [Rsbnwg EB enaH AB  11 cm
A D  
/ /
/ /
/ /

RtUv)anEckCaGgát;smamaRtnwg 3 4 5 . P
Q d  /

3. EckGgát; AB  12 cm CaGgát;smamaRtnwg -KUs d  edA[)an AP  2 , PQ  2 rYcP¢ab;


3 5 7 ³ QB nig PC [RsbKña enaHeyIg)an C Eck

A 
M

N B
Ggát; AB tampleFob 52 .
/ /
/ /
C / / /
/
-KUs l edA[)an AN  9, AM  6 rYcP¢ab;
D / /
NB nig MD [RsbKña enaHeyIg)an D Eck
/ /
/ /
/ d 
E
Ggát; AB tampleFob 156  52 .
198
6. eyIgmanrUb Edl AB// CD // EF // GH ³ K> KNna DF nig CG ³
A B eday AC  6 cm , CE  12cm , DB  8cm
C D nig FH  6 cm tamRTwsþIbTtaElseyIg)an ³
E F
BD DF

AC CE
b¤ 8 DF

6 12
G H naM[ DF  8 612  16cm
CE  DH
k> KNna DF nig FH ³ ehIy DH
CG CE

DF
b¤ CG 
DF
tamRTwsþIbTtaElseyIg)an Et DH  DF  FH
BD DF FH

AC CE EG
 eday AC  6 cm , naM[ CG  CE  DFDF
 FH 

CE  12 cm , EG  4cm nig BD  9 cm 12  16  6


CG   16.5 cm
16
naM[ 96  DF 
FH
12 4 dUcenH DF  16cm nig CG  16.5 cm .
eyIg)an 6  12 enaH DF  9 612  18cm
9 DF
7. KNnaRbEvg x énrUbnImYy²xageRkam ³
ehIy 96  FH4 enaH FH  9 6 4  6cm eday  AB //MN  enaH AM 
AN
MB NC
dUcenH KNna)an DF  18cm / FH  6cm -cMeBaHrUb A
9 8
x> KNna BD / DF nig FH ³ 16
M N
x
tamRTwsþIbTtaElseyIg)an B C
BD DF FH

AC CE EG
 eday AC  6 cm , naM[ 9 8
 naM[ x
8 16 128
  14.22
16 x 9 9
CE  12 cm , EG  4cm nig HB  33 cm
dUcenH KNna)an x
128
 14.22 .
tamlkçN³smamaRteyIg)an ³ 9
BD DF FH BD  DF  FH
  
AC CE EG AC  CE  EG
-cMeBaHrUb A

BH 33 3
   x
6  12  4 22 2 8
3 AC 3  6
naM[BD 3

AC 2
BD b¤ 2

2
 9cm M N
3 4
3CE 3 12
DF 3

CE 2
DF b¤ 2

2
 18cm B C
8  4 32
FH 3
 FH b¤ 3EG 3  4
  6cm naM[ 8 x

3 4
naM[ x
3

3
 10.67
EG 2 2 2

dUcenH BD  9cm , DF  18cm , FH  6cm dUcenH KNna)an x


32
3
 10.67 .
199
-cMeBaHrUb A eyIg)an a6  24 naM[ a  2 4 6  3 cm
M
x
ehIy b8  6 6 4 naM[ b  810 6  4.8 cm
6
B 4 N

12
C dUcenH a  3 cm nig b  4.8 cm .
eday  AB //MN  enaH BC
AC MC

NC 10. eyIgmanrUbdUcxageRkam ³
Et AC  x , MC  6 , NC  12  4  8 kaMTI13
A
naM[ 12x  86 enaH x  6 812  728  9
muxQrénkaM
dUcenH KNna)an x 9 .
8. KNnaRbEvg ED nig CD KitCa cm ³ kaMTI1
A B C
3.40m
4
8
12 E k> sg;RtIekaN ABC EkgRtg; B ³
B D
2 eyIgmanbøgm; anmaRtdæan 201
C
naM[ RCugQr AB  2.40m RtUvnwgkñúgbøg;
eday BE// CD tamRTwsþIbTtaElseyIg)an ³ 2.40m
AB   0.12m  12 cm
Ggát;smamaRt BC AB AE
 20
ED
ehIy RCugedk BC  3.40m RtUvnwgkñúgbøg;
b¤ 2  ED naM[ ED  4 8 2  1 cm
8 4
3.40m
BC   0.17m  17 cm
mü:ageTot ACAB BE

20
CD sg;rUb RtIekaN ABC EkgRtg; B ³
b¤ 108  CD
12
naM[ CD  12 810  15 cm A

dUcenH ED  1 cm nig CD  15 cm . 0.12m

9. KNnaRbEvg a nig b KitCa cm ³ B 0.17m C

6
x> etIeKRtUvEck AB nig BC Cab:unµanEpñk ?
a
b -eKRtUvEck AB Ca 13 Epñk edIm,IkMNt;TItaMg
2
4
km<s;énkaMCeNþIrnImYy² eRBaHCeNþIrman 13 kaM
8
-eKRtUvEck BC Ca 13 Epñk edIm,IkMNt;TItaMg
tamrUbxagelI eyIgmanGgát;edkTaMgBIrRsbKña muxQrénkaMCeNþIrnImYy² eRBaHCeNþIrman 13kaM
200
K> eFVIviFIEck AB nig BC ³ 12. eyIgmanrUbdUcxageRkam ³
y
N A

B C k> bkRsaysßanPaBenH edayrUbFrNImaRt ³


A
M
x

-eyIgEck AB  0.12m  12 cm eday ³ P


1.65m
KUsknøHbnÞat; By rYcedA[)an 13 RbeLaHRkit B
6.50m
C
Q
esµI²Kña ehIyP¢ab; NA nigP¢ab;Ggát;bnþbnÞab;[ 10m

Rsbnwg NA enaH AB RtUvEckCa 13 EpñkesµIKña. tag AB Cakm<s;r)ar / PQ Cakm<s;rbs;suxa


-eyIgEck BC  0.17m  17 cm eday ³ Edlman AB // PQ .
KUsknøHbnÞat; Bx rYcedA[)an 13 RbeLaHRkit tag BQ Cacm¶ayénsuxaKitBIeCIgbegÁal
esµI²Kña ehIyP¢ab; MC nigP¢ab;Ggát;bnþbnÞab;[ BC Cacm¶aymøb;énr)arKitBIeCIgbegÁal
Rsbnwg MC enaH BC RtUvEckCa 13 EpñkesµIKña. x> rkkm<s;rbs;r)ar ³
11. etIbnÞat; II  nig JJ  RsbKñab¤eT ? tamRTwsþIbTtaEls eyIg)anpleFobsmamaRt
J
X AB BC

PQ QC
b¤ AB

10
1.65 10  6.5

I 

O /

naM[ AB  10 3.15.65  4.714m


/
/
I /

dUcenH r)armankm<s; .
/
/
/
4.714m
J
/

eday OOII   23cm  nig


cm 2 IJ

3cm

2 13. tambRmab;RbFaneyIgKUsrUb)an ³
3 I J  4.5cm 3 B
naM[ OI   I J  ¬manGgát;smamaRtKña¦
OI IJ
F
tamRTwsþIbTRcastaEls enaH II  Rsbnwg JJ  A
E C

G
dUcenH bnÞat; II  Rsbnwg JJ  .
D

201
¬edIm,IgayRsYl ´sUmcmøgrUbdEdlkmvij¦
B

F
E C
A
G

k> sresrRTwsþIbTtaElskñúgRtIekaN ³
-cMeBaH AGE nig ADC ³
eday AGE nig ADC man GE // DC
dUcenH eyIg)an AG AE GE
 
AD AC DC
.
-cMeBaH AFE nig ABC ³
eday AFE nig ABCman FE // BC

dUcenH eyIg)an AF AE FE
 
AB AC BC
.
x> bgðajfa FG// BD ³
tamsMeNrénRTwsþIbTtaElsTaMgBIrxagelI
AG AE

AD AC
ni g AF AE

AB AC
naM[ AG 
AD AB
AF

tamRTwsþIbTRcastaEls enaH FG// BD


dUcenH FG// BD RtUv)anbgðajrYcral; .

¤
202
១៦

1.
N
F
G T
P M A

B E S
R
C

2.
A P L
B 51o R T
H 70 o 34o
82o
K
40 o
C
57 o 80 o 70 o 64o
E S M N
D F G

3. ABC ABC , PMN PM N 


A N N
M
B C
M
B C A P P
(A) (B)

4.

40 27
14 8 16 16 10
32 20 30
6 7 36
12
(A) (B) (C)

5. ABC GEF
A
G
B C
C G E
A
E
F B F

AB BC ...? AB ...? BC
   
GE ...? FG EG EF ...?
203
6.
A
77 o N
348m 290m
156.6m 130.5 m
58o 58o 45o
B C M P
400m 180m

7. ABCD AD BC F FD : FA  8: 5
FB  2.25cm BC
8. 35 dm 56 dm
10 dm
A R C

9. ABC BAC  90o


S
RS  BC ABC SRC
B

a
10. C D MN
b
a 3 a 1
. MN  11 dm ,  ខ. MN  6 dm , 
b 5 N
b 3
C
M
D

11. ABC AB   , AC   , BC   BAC


BC I BC J
. ABI AJC
ខ. AI  AJ
. BAJ  CBJ
. JB 2  JA  JI

12. A B B
A
C
AC D  AC CD D D E
DE // AB DE
AB CA = 1.8 km ,
C
CD = 90 m DE = 150 m AB

13. ABC A AB AC
M N
. ABC AMN
ខ. AB  AM  AC  AN

204
14. CD AB
. AD = 4 dm , DB = 9 dm CD
ខ. AB = 29 dm , CD = 10 cm AD DB
15. aha  bhb  chc

16. A ABC BC D
E ADB AEB AB2  AD  AE
17. 10 m ខ P
3m P
A
18. AB A
E
AE = 3 cm
BA’ EA’=9dm
EB B

19. b c h
h a b
D C
20. ABCD I
AD  IB   AD I
12 cm
. IB
ខ.
A 10 cm B

21. NMQ PNQ


N

. MQ 5 cm
3 cm
ខ. NP
Q
M

22. CD AB D
I
I AI  15 cm , IC  12 cm ,
C
ID  4 cm , IB  5 cm , BD  3 cm
. IAC IBD
ខ. AC
A

205
A
23. P A
C D 
. PAC PAD
P D
ខ. AP2  PC  PD C

24. 10 cm

5 cm

 
4 cm
8 cm

25.
.
3 3
v h r
    v V
V H  R
h H
ខ. v  6.28 cm3
h  2 cm , H  4 cm
r R
V

DDCEE

206
១៦

1. sresreQµaH nigRCugRtUvKña ³ -cMeBaH rUb¬K¦


-cMeBaHKURtIekaNdUcKña ³ P
34o
L
82o
N K

64o
M N
P M A

B eday PKLman L  180  34 o o



 82 o  64 o

C dUcenH PKL MKN


÷sresreQµaH ³ NPM  ACB eRBaH vamanmMBu Ib:unerogKña .
÷manRCugRtUvKña ³ NP 
NM PM
 .
AC AB CB 3. eRbóbeFobRtIekaN rYcsresrpleFobdMNUc ³
-cMeBaHKURtIekaNdUcKña ³ -cMeBaH rUb(A)
F A
G T
B C

E S B C A
R

÷sresreQµaH ³ EFG  TRS tamsmµtikmµénrUb ABC nig ABC man ³


÷manRCugRtUvKña ³ EF FG EG
  . A  A nig C  C
TR RS TS

2. KURtIekaNxageRkamdUcKñab¤eT? ehtuGVI ? dUcenH ABC ABC tamlkçxNÐ m>m


-cMeBaH rUb¬k¦ B 51 o
A eday  ABCABC 
naM[ AB

AC

BC
AB AC  BC 
C
57 o
-cMeBaH rUb(B)
D E N N
M

RtIekaNTaMgBIrKW ABCnig ADE mindUcKñaeT M

eRBaH vamanEtmMumYyb:unKña KWxuslkçxNÐTaMgbI . P P

-cMeBaH rUb¬x¦ tamsmµtikmµénrUb PMN nig PM N  man ³


R T
M  M  nig P  P
H 70 o

40 o

dUcenH PMN  PM N  tamlkçxNÐ m>m


o
80 o 70 S
F G

RtIekaNTaMgBIrKW HFG nig TSR mindUcKñaeT eday 


PMN
naM[ PPM 
PN

MN
PM N  M  PN  M N 
eRBaH vamanEtmMumYyb:unKña KWxuslkçxNÐTaMgbI .
207
4. KURtIekaNxageRkamdUcKñab¤eT? ehtuGVI ? 5. -bMeBjpleFobdMNUc ³
-cMeBaH rUb(A) A

B
C G

14 8 16 E
F
6 7
12
edayRtIekaN ABC dUcnwgRtIekaN GEF
RtIekaNTaMgBIrCa RtIekaN dUcKñay:agR)akd eyIgbMeBj)an ³ GE
AB BC CA
 
EF FG
.
eRBaH RtIekaNTaMgBIrmanpleFobRCug ³ -bMeBjpleFobdMNUc ³
 enaH
6 1 7 1 8 1 6 7 8 G
 ,  ,   C
12 2 14 2 16 2 12 14 16 E

mann½yfa vabMeBj[lkçxNÐdMNUc C>C>C . A

B F

-cMeBaH rUb(B)
16
edayRtIekaN ABC dUcnwgRtIekaN GEF
10
32 20
eyIgbMeBj)an ³ EG AB AC BC

EF GF
 .
6. RsaybBa¢ak;fa RtIekaNdUcKña rYcTajrkvi)ak ³

RtIekaNTaMgBIrCa RtIekaN dUcKña A


77 o N
348m

eRBaH RtIekaNTaMgBIrmanpleFobRCug ³
290m
156.6m 130.5 m
58o 58o 45o
B C M P
180m

 enaH ehIyRCug
400m
16 1 10 1 16 10
 , 
32 2 20 2 32 20
smamaRterogKñaTaMgBIrenHenAGmnwgmMub:unKñamYy edayRtIekaNTaMgBIrxagelImanpleFobRCug ³
NM 156.6m
  0.45
KWmMuTl;kMBUl vabMeBj[lkçxNÐdMNUc C>m>C . AB 348m
NP 130.5m
  0.45
-cMeBaH rUb(C) AC 290m
MP 180m
40 27
  0.45
BC 400m
36
30
naM[ NM NP MP

AB AC BC

RtIekaNTaMgBIrCa RtIekaN dUcKña dUcenH ABC NMP tamlkçxNÐ C>C>C


eRBaH RtIekaNTaMgBIrmanpleFobRCug ³ -Tajvi)ak ³
A  N  77
eday  ABC
o
40 4 36 4
 ,  enaH
40 36
 ehIyRCug nam[ 
C  P  45 
o
30 3 27 3 30 27 NMP
smamaRterogKñaTaMgBIrenHenAGmnwgmMub:unKñamYy
KWmMuTl;kMBUl vabMeBj[lkçxNÐdMNUc C>m>C . dUcenH N  77 o , C  45 o .

208
7. KNnaRbEvg BC ³ AB

AC

BC
AB AC  BC 
tambRmab;RbFan eyIgKUsrUb)andUcxageRkam ³ eyIgeRbIlkçN³smamaRt enaHeyIg)an ³
F AB AC BC AB  AC  BC
  
AB AC  BC  AB  AC   BC 
A B Et AB  AC  BC  P  35 dm

D C
AB  AC   BC   P  56 dm >
naM[ AAB 
AC

BC 35 5
  1
eday ABCD CactuekaNBñayenaH AB// CD B AC  BC  56 8

naM[ FAB  FDA ¬mMuRtUvKña¦ mü:ageTot AMB AM B tamlkçxNÐ m>m
ehIy FBA FCD ¬mMuRtUvKña¦ ¬eRBaH B  B nig M  M   90 ¦ o

eyIg)an FAB FDC tamlkçxNÐ m>m vi)ak  AMB


AM B

AB

AM
AB AM 
vi)ak  FDC
FAB

FC FD

FB FA
b¤ AAB 
B  H
h
2

Et FD: FA  8 : 5 mann½yfa FD 
FA 5
8
tam 1 nig 2 eyIg)an Hh  85
nig FB  2.25 m ehIy FC  FB  BC ehIybRmab;km<s;RtIekaNtUc h  10 dm
eyIg)an FBFB  BC 8

5
eyIg)an 10Hdm  85 b¤ H  8 105 dm  16 dm
2.25  BC 8
b¤ 2.25

5 dUcenH km<s;énRtIekaNFMKW 16 dm .
8
2.25  BC   2.25
5 9. RsaybBa¢ak;fa ABC SRC ³
BC  3.6  2.25
BC  1.35 A R C

dUcenH KNna)anRbEvg BC  1.35 m . S

8. rkRbEvgkm<s;mYyénRtIekaNtUc ³ B

eyIgtagRtIekaNtUcFM dUcrUbxageRkam ³ eday ABC nig SRC man ³


A A -mMu ACB  SCR ¬mMurYm¦
h
-mMu BAC  RSC  90 CamMuEkgdUcKña
o

B
M
C H dUcenH ABC SRC tamlkçxNÐ m>m
B C
M

edayRtIekaNTaMgBIrdUcKña eyIg)ansmamaRt ³
209
10. edAcMNuc C nig D edIm,IEckGgát; MN xagkñúg x> eyIgman MN  6 dm , ba  13 ³
nigxageRkA tampleFob ba ³ N
Q
k> eyIgman MN  11 dm , ba  53 ³ M C

N D A
Q
P
C l
M B

D
PA
B
l ¬eyIgedaHRsaydUclMnaM lMhat; k> Edr ¦
eyIgKUsbnÞat; l kat;tam M rYcedA[)an ³
¬eXIjrUbdUclM)ak Ettamdan[c,as;KWFmµta¦ MA  1, AB  3 Edl A nig B enAmçagén M
eyIgKUsbnÞat; l kat;tam M rYcedA[)an ³ KUsP¢ab; BN rYcKUs AC [Rsbnwg BN
MA  3 , AB  5 Edl A nig B enAmçagén M
Edl C CaRbsBVrvag AC nig BN
KUsP¢ab; BN rYcKUs AC [Rsbnwg BN tamRTwsþIbTtaElseyIg)anpleFob
Edl C CaRbsBVrvag AC nig BN CM AM
 b¤ CM 1

tamRTwsþIbTtaElseyIg)anpleFob CN AB CN 3
CM AM
 b¤ CM 3
 dUcenH C Eck MN xagkñúgtampleFob
CN AB CN 5
CM 1
 .
dUcenH C Eck MN xagkñúgtampleFob CN 3
CM 3

CN 5
. -enAelIbnÞat; l edA[)ancMNuc ³
MQ  1 , MP  2 Edl M enAcenøaH QP
-enAelIbnÞat; l edA[)ancMNuc ³ KUsP¢ab; PN rYcKUs QD [Rsbnwg PN
MQ  3 , MP  2 Edl M enAcenøaH QP
Edl D CaRbsBVrvag QD nigbnøayén MN
KUsP¢ab; PN rYcKUs QD [Rsbnwg PN enaH DMQ  NMP tamlkçxNÐ m>m
Edl D CaRbsBVrvag QD nigbnøayén MN
vi)ak DM 
MQ
b¤ DM 1

enaH DMQ  NMP MN MP
MN DM  MN DN
MN 2

vi)ak DM 
MQ
b¤ DM 3

b¤ DM 1

2

1 2

3
MN MP MN 2
b¤ DM 
MN DM  MN DN
 
b¤ DM 1

DN
3
eyI g )an DM 1
DN 3

3 2 3 2 5
b¤ DM 
DN
eyI g )an DM 3
 dUcenH D Eck MN xageRkAtampleFob
3 5 DN 5
DM 1
 .
dUcenH D Eck MN xageRkAtampleFob DN 3
DM 3

DN 5
. ¬´bkRsayEvg EbbBnül;mUlehtudl;GñkGan>>>¦

210
11. tambRmab;RbFaneyIgKUsrUb)an ³ naM[ JB  JA  JI
2

A
dUcenH JB  JA  JI RtUv)anRsaybBa¢ak; .
2

12. KNnacm¶ay AB ³
I
B C
A B
J
D E
k> RsaybBa¢ak;fa ABI dUcnwg AJC ³
eday ABI nig AJC man ³ C

eday DE // AB enaHeyIg)anmMuRtUvKña ³
-mMu ABC  AJC ¬mMumanFñÚsáat;rmY AC ¦
CDE  CAB nig CED  CBA
-mMu BAI  JAC ¬eRBaH AJ CaknøHbnÞat;BuH¦
naM[ CDE  CAB
dUcenH ABI  AJC tamlkçxNÐ m>m . vi)ak  CDE 
CD DE

CAB CA AB
x> KNnaplKuN AI  AJ ³ naM[ AB  DECD CA  150 m901m.8 km
eday  ABI 
AI

AB
AJC AC AJ
KNna)an AB  3 km
naM[ AI  AJ  AB AC Et AB   , AC  
enaH AI  AJ   dUcenH RbEvgKNna)an AB  3 km .
13. tambRmab;RbFaneyIgKUsrUb)an ³
dUcenH KNna)anplKuN AI  AJ   .
A
K> bgðajfa BAJ  CBJ ³ N

eday BAJ  JAC ¬eRBaH AJ CaknøHbnÞat;BuH¦ M

ehIy JAC  CBJ ¬mMucarwkFñÚsáat;rYm JC ¦ B C

naM[ BAJ  CBJ k> RbdUc ABCnig AMN ³


dUcenH BAJ  CBJ RtUv)anbgðaj . eday ABCnig AMN man
-mMu ABC  PAC  ANM
X> RsaybBa¢ak;fa JB  JA  JI
2

eRBaH ABC  PAC mMucarwkmanFñÚsáat;rYm AC


eday ABJ nig BIJ man ³
nig PAC  ANM mMuqøas;kñúg
-mMu BAJ  CBJ  IBJ ¬bgðajxagelI¦
-mMu ACB  QAB  AMN
-mMu BJA  IJB ¬mMurYmEtmYy¦
eRBaH ACB  QAB mMucarwkmanFñÚsáat;rYm AB
naM[ ABJ  BIJ tamlkçxNÐ m>m
ehIy QAB  AMN CamMuqøas;kñúg
vi)ak  ABJ 
JB AJ

BIJ JI JB dUcenH ABC AMN tamlkçxNÐ m>m .
211
x> RsaybBa¢ak;fa AB AM  AC  AN ³ -eyabl;´ ³ RbsinebIeyIgEkRbFanmkCa
eday  ABC 
AB AC
 AB  29 dm , CD  10 dm vijenaHeyIg)an ³
CD 2  AD   AB  AD
ANM AN AM
Taj)an AB AM  AC  AN 10 2  AD  29  AD
100  29 AD  AD2
dUcenH Rsay)anfa AB AM  AC  AN .
AD2  29 AD  100  0

14. tambRmab;RbFaneyIgKUsrUb)an ³ eyIgedaHRsayedayeRbIviFIKuNExVg ³


 AD  25
C AD2  100
 AD  4 
 4 AD  25 AD  29 AD
eyIg)an AD  4AD  25   0
B
A
D naM[ AD  4  0 enaH AD  4 dm
eday C CacMNucénknøHrgVg;Ggát;p©it AB AD  25  0 enaH AD  25 dm
enaH ABC CaRtIekaNEkgRtg; C mankm<s; CD -cMeBaH AD  4 dm ³ DB  AB  AD
eyIg)an CD  AD DB b¤ CD  AD  DB
2
 29  4  25dm >
k> KNna CD ebI AD  4 dm , DB  9 dm ³ -cMeBaH AD  25 dm ³ DB  29  25  4dm
naM[ CD  4dm 9dm  36dm  6 dm 2

dUcenH AD  4 dm nig DB  25dm


dUcenH KNna)an CD  6 dm . b¤ AD  25 dm nig DB  4dm .
x> KNna AD nig DB ³ 15. RsaybMPøW[eXIjfa ah a  bhb  chc ³
eyIgman AB  29 dm , CD  10 cm  1dm A >
B
eday CD  AD DB
2
c C b
CD  AD   AB  AD
2
hb h a h c
1  AD  29  AD
2
B A C
1  29 AD  AD2 a

AD2  29 AD  1  0 kñúgRtIekaN AAB nigRtIekaN CCB man ³


¬CasmIkardWeRkTI2manb£sminEmnCacMnYnKt; dUcenH -mMu A  C  90 ¬ A nig C  CaeCIgkm<s;¦
o

sisSBMuGacedaHRsay)aneLIy . ´eXIjmanesovkMEN -mMu B CamMurYm


CaeRcIndak;lk;enAelITIpSar ecHEt edaHRsayy:agRsYl naM[ AAB  CCB tamlkçxNÐ m>m
edaymin)anKitBIxñat dm nig cm eFVI[cemøIyrbs;eK
xusKW 25dm b¤ 4dm rebobedaHRsayenAfñak;TI 10 vi)ak  ACACBB  CACA  CB
AB
b¤ hh  ac a

ehIycemøIyEdlRtUvKW AD  29  2 837 dm ¦. Taj)an ah  ch ¬k¦


a c

212
kñúgRtIekaN AAC nigRtIekaN BBC man ³ 17. KNnaedImb£sSIBIKl;eTAcMNuc)ak; P ³
-mMu A  B  90 ¬ A nig B CaeCIgkm<s;¦
o P

-mMu C CamMurYm x
10  x

naM[ AAC  BBC tamlkçxNÐ m>m


B
vi)ak  ABABCC  BABA  BC
A 3m
AC
b¤ hh  ba a

b
tag x CaRbEvgb£sSIBIKl; A eTAcMNuc)ak; P
Taj)an ah  bh ¬x¦
a b
tamRTwsþIbTBItaK½rcMeBaHRtIekaNEkg PAB ³
tamTMnak;TMng ¬k¦ nig ¬x¦ 32  x 2  10  x 2  
eyIg)an ah  bh  ch
a b c 9  x  100  20 x  x 2
2

x 2  x 2  20 x  100  9
dUcenH ah  bh  ch RtUv)anRsaybMPøW .
a b c 20 x  91
91
x
16. eRbóbeFobRtIekaN ADB nigRtIekaN AEB³ 20
m

A
dUcenH BIKl;eTAcMNcu )ak;manRbEvg 4.55 m .
18. KNnaCeRmATwk EB ³
D
B C
A
E A

RtIekaN ADB nigRtIekaN AEB man ³ E

-mMu BAD EAB ¬mMurYm¦ B


-mMu ABC  AEB eRBaHedaysarTMnak;TMng
ABC  ACB mMu)aténRtIekaNsm)at ABC
edayépÞTwkEkgnwgpáaQUkQr
ACB  AEB mMucarwkmanFñÚsáat;rYm AB
naM[ EAB EkgRtg; E ehIytamBItaK½r ³
AB 2  AE 2  EB2 >
dUcenH ADB  AEB tamlkçxNÐ m>m . Et AB  AB  AE  EB ¬edImQUkEtmYy¦
-rYcTajbBa¢ak;fa AB  AD AE ³ 2 eyIg)an  AE  EB   AE  EB 2 2 2

eday  ADB 
AB AD

eday AE  3dm , AE  9 dm
ABE AE AB
enaH 3  EB   9  EB 2 2 2

Taj)an AB AB  AD AE 9  6 EB  EB2  81  EB2


b¤ AB  AD  AE2 6 EB  81  9
72
EB   12 dm
dUcenH AB2  AD  AE RtUv)anbBa¢ak; . 6

dUcenH CeRmATwkKW EB  12 dm .
213
19. KNna h eTAtamtémøén b nig c ³ x> KNnaépÞRkLaénRbelLÚRkam ³
eday ABCD CaRbelLÚRkamman)at AD nig
a
h mankm<s; IB enaHeyIg)an ³
b S ABCD  AD  IB  12  8  96 cm 2
tamRTwsþIbTBItaK½r RbEvgGIub:UetnusKW a 2  b2 ³
dUcenH KNna)an S ABCD  96 cm 2 .
tamlkçN³énRtIekaNEkg eyIg)an ³
h   a  b  a  b
2 2
21. eyIgmanrUbdUcxageRkam ³
ab
h P
a b
2 2

CaFmµta eKminEdlTukr:aDIkal;enAPaKEbg N

naM[ h  ab  aba a b b
2 2 5 cm
3 cm
2 2
a2  b2 M Q

dUcenH KNna)antémø h  aba a b b .


2 2
k> KNnargVas; MQ ³
2 2
tamRTwsþIbTBItaK½rcMeBaHRtIekaNEkg MNQ ³
20. eyIgmanrUbdUcxageRkam ³ eyIg)an MN  MQ  NQ
2 2 2

D C MQ2  NQ 2  MN 2
MQ  NQ 2  MN 2
I
12 cm
 52  32  16  4 cm
A 10 cm B
dUcenH KNna)an MQ  4 cm .
k> KNna IB ³
eyIgman ABCDCaRbelLÚRkam nig I CacMNuc x> KNnargVas; NP ³
kNþal AD ehIy IB    AD  enaHeyIg)an ³ tamsmµtikmµénrUb MNQ nig NPQ man ³
IA 
AD BC 12 cm
   6 cm NMQ  PNQ  90 nig PQN  NQM
o

2 2 2
tamRTwsþIbTBItaK½rcMeBaH RtIekaNEkg AIB naM[ MNQ  NPQ tamlkçxNÐ m>m
eyIg)an IA  IB  AB 2 2 2 vi)ak  MNQ
NPQ

MN MQ

NP NQ
naM[ IB  AB  IA2 2 2
Taj)an NP 
MN  NQ
MQ
IB  AB2  IA2
3  5 15
 10 2  6 2  100  36
NP    3.75 cm
4 4
 64  8 cm
dUcenH KNna)an NP  3.75 cm .
dUcenH KNna)an IB  8 cm .
214
22. eyIgmanrUbdUcxageRkam Edlman ³ x> Taj[eXIjfa AP  PC  PD 2

AI  15cm , IC  12cm , ID  4cm , IB  5cm > eday  PAC 


PA PC

PDA PD PA
nig BD  3 cm B
D
Taj)an PA  PC  PD
2

I
C dUcenH Taj)anfa PA  PC  PD . 2

A 24. etIekaNTaMgBIrdUcKñab¤eT ?
k> eRbóbeFobRtIekaN IAC nig IBD ³
eday IAC nig IBD man ³ 10 cm

 nig
IB 5 cm 1 ID 4 cm 1 5 cm
  
IA 15cm 3 IC 12cm 3  
ehIy BID  CIA ¬mMuTl;kMBUl¦ 4 cm
8 cm

dUcenH IBD IAC tamkrNI C>m>C . ekaNTaMgBIrCaekaNdUcKña luHRtaEtpleFob


FatuRtUvKñaesµIKña
x> KNnargVas; AC ³ eday Dd  84cm  ehIy
cm 1 h

5cm 1

eday IBD IAC enaH IB 
BD 1

2 H 10cm 2
IA AC 3 eyIg)an pleFob Dd  Hh  12
b¤ BD 1
AC 3
 naM[ AC  3BD
eyIg)an AC  3  3cm  9 cm dUcenH ekaNTaMgBIrCaekaNdUcKña .
dUcenH KNna)an AC  9 cm . 25. eyIgmanekaNBIrdUcKña dUcrUbxageRkam ³
23. eyIgmanrUbdUcxageRkam ³
A
h H

P
C
D r R

k> eRbóbeFobRtIekaN PAC nigRtIekaN PAD³


3 3
v h r
k> bgðajfa    
V H  R
³
eday PACnig PAD ³ -maDekaNtUc 1
v  r 2 h
-mMu P CamMurYm 3

-mMu PAC  ADP mMucarwkmanFñÚsáat;rYm AC -manekaNFM 1


V  R 2 H
3
eyIgeFVIpleFobmaDekaNTaMgBIr enaHeyIg)an ³
dUcenH PAC PAD tamlkçxNÐ m>m .
215
1 2
r h
v 3 r 2h
  2
V 1 2
R H R H
3
edayekaNTaMgBIrdUcKña enaHpleFob Rr  Hh
2 2
naM[ Vv  Rr rR b¤ Vv  Hh hH
2 2

3 3
eyIg)an Vv  Rr b¤ Vv  Hh
3 3

3 3

Gacsresr Vv   Rr  h


 
H
3 3
v r h
dUcenH bgðaj)anfa    
V R H 
.
x> KNnamaD V énekaNFM ³ ¬esovePABum<dak;RcLM ³
sUmEkBI v  6.28cm eTACa v  6.28cm ¦
2 3

3
v h
eyIgman  
V H
eday v  6.28 cm , h  2cm , H  4cm
3

3
6.28  2 
naM[ V
 
4
3
6.28  1 
 
V 2
6.28 1

V 8
V  6.28  8
V  50.24 cm3

dUcenH KNna)an V  50 .24 cm 3 .

216
១៧

1. 7 8 9 10 12
2. 1800o , 2 700o 4860o
3. 51300
4. 540o

5. x

x 100o 2x 2x
3x 80 o x 50 o x x
x 2x 2x 160 o
2x

6. y
2y 2y
2y
y
y 2y 3y
80o y
y 3y 2y
70 o y 2y
2y

7. 150o , 160o 175o


8. 3240o
9. 900o

10. n 168o n
11.
. n6
ខ. n5
. n 8 .
. 12
12. 900o
13.
12
o
14. 27 n
n

217
n  n  3
15. n
2
:
16.
17. n 30o
n

DDCEE

218
១៧

1. KNnaplbUkrgVas;mMukñúgénBhuekaN ³ -krNIBhuekaNmancMnYnRCugesµI 12 ³
-krNIBhuekaNmancMnYnRCugesµI 7 ³ eday plbUkmMukñúg  n  2180 nig n  12
o

eday plbUkmMukñúg  n  2180 nig n  7


o
naM[ plbUkmMukñúg  12  2180  1800
o o

naM[ plbUkmMukñúg  7  2  180  900


o o
dUcenH BhuekaNEdlmancMnYnRCgesµI 12
dUcenH BhuekaNEdlmancMnYnRCgesµI 7 vamanplbUkmMukñúgesµI 1800 o
.
vamanplbUkmMukñúgesµI 900 o
.
2. rkcMnYnRCugénBhuekaN edaysÁal;plbUkmMukñúg ³
-krNIBhuekaNmancMnYnRCugesµI 8 ³
-krNIplbUkmMukñúgesµI 1800o

eday plbUkmMukñúg  n  2180 nig n  8


o

ebIBhuekaNenHmancMnYnRCugesµInwg n eyIg)an ³
naM[ plbUkmMukñúg  8  2180  1080
o o
n  2180o  1800o
dUcenH BhuekaNEdlmancMnYnRCgesµI 8 n2
1800o
180o
vamanplbUkmMukñúgesµI 1080 . o
n  10  2
n  12
-krNIBhuekaNmancMnYnRCugesµI 9 ³
dUcenH BhuekaNEdlmanplbUkmMukñúg
eday plbUkmMukñúg  n  2180 nig n  9
o

esµI 1800 vamancMnYnRCugesµI 12 .


o

naM[ plbUkmMukñúg  9  2180  1260


o o

dUcenH BhuekaNEdlmancMnYnRCgesµI 9 -krNIplbUkmMukñúgesµI 2700o

vamanplbUkmMukñúgesµI 1260 o
. ebIBhuekaNenHmancMnYnRCugesµInwg n eyIg)an ³
n  2180o  2700o
-krNIBhuekaNmancMnYnRCugesµI 10 ³ 2700o
n2
eday plbUkmMukñúg  n  2180 nig n  10
o
180o
n  15  2
naM[ plbUkmMukñúg  10  2180  1440
o o

n  17

dUcenH BhuekaNEdlmancMnYnRCgesµI 10 dUcenH BhuekaNEdlmanplbUkmMukñúg


vamanplbUkmMukñúgesµI 1440 o
. esµI 2700 vamancMnYnRCugesµI 17 .
o

219
-krNIplbUkmMukñúgesµI 4860 o
eyIg)anplbUkmMukñúg
ebIBhuekaNenHmancMnYnRCugesµInwg n eyIg)an ³ x  3x  x  80o  4  2180o

n  2180o  4860o 5 x  80o  360o

4860o 5 x  360o  80o


n2
180o 280o
x  56o
n  27  2  29 5

dUcenH BhuekaNEdlmanplbUkmMukñúg dUcenH témøKNna)anKW x  56o .


esµI 4860 vamancMnYnRCugesµI 29 .
o

-cMeBaHrUb ¬x¦ ³ CaBhuekaNmancMnYnRCug 5 ³


3. rkBhuekaNEdlmanplbUkmMukñúgesµI 5130 o
100o
ebI n CacMnYnRCugénBhuekaN enaH n CacMnYnKt; x 50 o
2x 2x
eyIg)an n  2180  5130
o o

n2
5130 o eyIg)anplbUkmMukñúg
180 o
100o  x  2 x  2 x  50o  5  2180o
n  28 .5  2  30 .5
5 x  150o  540o
tamlT§plbgðajfa cMnYnRCugminEmnCacMnYnKt; 5 x  540o  150o

dUcenH KµanBhuekaNNaEdlman x
390o
 78o
5
plbUkmMukñúgesµI 5130 eLIy .
o

dUcenH témøKNna)anKW x  78o .


4. rkcMnYnRCugénBhuekaNe):agenaH ³
ebI n CacMnYnRCugénBhuekaNenaH eyIg)an ³ -cMeBaHrUb ¬K¦ ³ CaBhuekaNmancMnYnRCug 6 ³
n  2180o  540o 2x 2x
x x
540o
n2 160 o
2x
180o
n  3 2  5
eyIg)anplbUkmMukñúg
dUcenH BhuekaNe):agenaHmancMnYnRCugesµI 5 . 160o  x  2 x  2 x  x  2 x  6  2180o
8 x  160o  720o
5. rktémø x énrUbBhuekaNnImyY ² ³ 8 x  720o  160o
-cMeBaHrUb ¬k¦ ³ CaBhuekaNmancMnYnRCug 4 ³ 560o
x  70o
8
x
3x 80 o
dUcenH témøKNna)anKW x  70 o .
x

220
6. KNnatémø y tamrUbnImYy²xageRkam ³ 7. KNnacMnYnRCugénBhuekaNniy½t ³
-cMeBaHrUb ¬k¦ ³ manplbUkmMueRkAesµI 360 o
-BhuekaNniy½tEdlmanrgVas;mMukñgú esµI 150
o

y 2y ebI n CacMnYnRCugénBhuekaNniy½t eyIg)an


y n  2180o  150o
2y n
180o n  360o  150o n
eyIg)anplbUkmMueRkA 180o n  150o n  360o
y  2 y  y  2 y  360 o 30 o n  360o
6 y  360 o 360o
n  12
y  60 o 30o

dUcenH témøKNna)an y  60 o . dUcenH BhuekaNniy½tenaHmancMnYnRCug 12 .


-cMeBaHrUb ¬x¦ ³ manplbUkmMueRkAesµI 360 o
-BhuekaNniy½tEdlmanrgVas;mMukñgú esµI 160
o

y ebI n CacMnYnRCugénBhuekaNniy½t eyIg)an


80o y n  2180o  160o
70 o
y n
eyIg)anplbUkmMueRkA 180o n  360o  160o n
70o  80o  y  y  y  360o 180o n  160o n  360o
3 y  150o  360o 20 o n  360o
360o  150o 360o
y  70o n  18
3 20 o

dUcenH témøKNna)an y  70 . o dUcenH BhuekaNniy½tenaHmancMnYnRCug 18 .


-cMeBaHrUb ¬K¦ ³ manplbUkmMueRkAesµI 360 o

2y 2y -BhuekaNniy½tEdlmanrgVas;mMukñgú esµI 175


o

2y 3y ebI n CacMnYnRCugénBhuekaNniy½t eyIg)an


3y
n  2180o  175o
2y
2y n
180o n  360o  175o n
eyIg)anplbUkrgVas;mMueRkA 180o n  175o n  360o
3 y  2 y  2 y  3 y  2 y  2 y  2 y  360o 5 o n  360o
16 y  360o 360o
n  72
360o 5o
y  22.5 o
16
dUcenH BhuekaNniy½tenaHmancMnYnRCug 72 .
dUcenH témøKNna)an y  22 .5o .

221
8. KNnargVas;mMueRkAnImYy²énBhuekaNniy½t ³ 10. rkcMnYn n ³
plbUkrgVas;mMukñúgesµInwg 3240 o
eday BhuekaNniy½tEdlman n RCug nigman
ebI n CacMnYnRCugénBhuekaNniy½t eyIg)an rgVas;mMukñúgmYyesµI 168 enaHeyIg)an ³
o

n  2180o  3240o n  2180o  168o


3240 o n
n2 180o n  360o  168o n
180o
n  18  2 180o n  168o n  360o
n  20 12o n  360o
eyIgdwgfa cMnYnRCug = cMnYnmMueRkA n
360o
 30
12o
naM[ cMnYnmMueRkA n  20
edaymMueRkAnImYy² énBhuekaNniy½tmanrgVas; dUcenH cMnYnEdlRtUvrkKW n  30 .
esµI²Kña nigmanplbUkesµI 360 o
11. KNnargVas;mMueRkAnImYy²énBhuekaNniy½t ³
eyIgdwgfa mMeu RkAnImYy²énBhuekaNniy½t
o
eyIg)an rgVas;mMueRkAnImYy²  36020  18 o

kMNt;eday 360n Edl n CacMnYnRCug


o

dUcenH rgVas;mMueRkAnImYy² énBhuekaN


niy½tenaHKW 18 o
. k> qekaNniy½t n  6
eyIg)an 360n  3606  60
o o
o

9. rkcMnYnRCug énBhuekaNniy½tenaH ³ dUcenH mMueRkAnImYy²esµInwg 60 . o

ebI n CacMnYnRCugénBhuekaNniy½tenaH eyIg)an


-plbUkrgVas;mMukñúgesµInwg n  2180 o x> bBa©ekaNniy½t n  5
eyIg)an 360n  3605
o o

-plbUkrgVas;mMueRkA 360 o  72 o

eday plbUkrgVas;mMukñúgrYmnwgplbUkrgVas;mMueRkA dUcenH mMueRkAnImYy²esµInwg 72 . o

esµInwg 900 naM[eyIgGacsresr)an ³


o

n  2180o  360o  900o K> GdæekaNniy½t n  8


n  2180o  900o  360o eyIg)an 360n  3608
o o
 45 o
o
540
n2
180o
n  3 2
dUcenH mMueRkAnImYy²esµInwg 45 . o

n5
X> BhuekaNniy½tEdlmanRCug 12
dUcenH cMnYnRCugénBhuekaNniy½tenaHKW 5 . eyIg)an 360n  360
o
 30
o
o

12

dUcenH mMueRkAnImYy²esµInwg 30 . o

222
12.rkcMnYnRCug énBhuekaNenaH ³ eXIjfa BhuekaN ABCDEFGHJKLM man³
ebI n CacMnYnRCugénBhuekaNenaH eyIg)an -mMu ABC  ABI  IBC  60  90  150
o o o

-plbUkrgVas;mMukñúgesµInwg n  2180 o
naM[ mMukñúgnImYy²esµIKña esµInwg 150 o

-plbUkrgVas;mMueRkA 360 o
-RCug AB  BC  CD  ...  MA
eday plbUkrgVas;mMukñúg nigmMueRkAesµIngw 900 o
eyIgrkcMnYnRCugéBhuekaNtamTMnak;TMng ³
naM[ eyIgGacsresr)an ³ n  2180o  150o
n
n  2180o  360o  900o 180o n  360o  150o n
n  2180o  900o  360o 180o n  150o n  360o
540o 30 o n  360o
n2
180o 360o
n  12
n  3 2  5 30o

dUcenH cMnYnRCugénBhuekaNenaHKW 5 . dUcenH BhuekaNEdlekItCaBhuekaN


niy½tmancMnYnRCugesµI 12 .
13.RsaybBa¢ak;farUbEdlekItCaBhuekaNniy½t
EdlmancMnYnRCugesµInwg 12 14. RsayfaBhuekaNEdlman n RCugenaHcMnYn
A M Ggát;RTUgrbs;vaesµInwg nn2 3 ³
B L
eKGacKUsBhuekaN)anluHRtaEtcMnYnRCugrbs;
C I K
vaRtUvFMCagb¤esµI 3
D 120o J
-cMeBaH n  3 vaCaRtIekaN
E
F G
H cMnYnGgát;RTUg 332 3  0
eyIgP¢ab;kMBUlkaerenACab;²Kña enaHeyIgnwg)an -cMeBaH n  4 vaCaRtIekaN
BhuekaN ABCDEFGHJKLM cMnYnGgát;RTUg 442 3  2
eyIgman qekaNniy½tEdlrgVas;mMukñúgnImYy² -cMeBaH n  5 vaCaRtIekaN
esµInwg 6  26180  120 cMnYnGgát;RTUg 552 3  5
o
o

naM[ AIB  360  120  90  90 


o o o o
-cMeBaH n  6 vaCaRtIekaN
 360o  300o cMnYnGgát;RTUg 662 3  9
 60o
kñúgRtIekaN AIB man IA IB nig AIB  60 o
>>>>>>>>>>>>>>>>>>>>>>>>>>>>>>>>>>>>>>>>>>>>
naM[ AIB CaRtIekaNsm½gS dUcenH BhuekaNman n RCugenaHcMnnY
vi)ak AIB  IBA  IAB  60 o Ggát;RTUgesµI nn2 3 , n  3 .
223
15.KNnamMukñúg nigmMueRkAénqekaNniy½t ³
-plbUkmMukñúgénqekaN KWesµInwg
6  2180 o  4 180 o  720 o
edaymMunImYy²manrgVas;esµIKña
o
naM[ mMukñúgnImYy²esµInwg 7206  120 o

-plbUkmMueRkAénqekaN KWesµInwg 360 o

o
naM[ mMueRkAnImYy²esµInwg 3606  60 o

dUcenH kñúgqekaNniy½t KNna)an ³


mMukñúgesµInwg 120 / mMueRkAesµInwg 60 .
o o

16. KNnacMnYnRCug n énBhuekaNniy½t ³


-plbUkmMueRkAénqekaN KWesµInwg 360 o

o
naM[ mMueRkAmYyénqekaNniy½tKW 3606  60 o

-eKmanmMueRkAmYyénBhuekaNniy½t manrgVas;
30 eRcInCag rgVas;mMueRkAmYyénqekaNniy½t
o

naM[ mMueRkAénBhuekaNniy½tenaHesµInwg
60  30  90
o o o
>
eyIg)an cMnYnRCugénBhuekaNniy½tenaHesµInwg
360 o
n 4
90 o

dUcenH BhuekaNniy½tenaHmancMnYnRCug
n  4 ¬BhuekaNniy½tenaHCakaer¦ .

224
១៨

1.
S
S S

14 cm
9 cm
14cm
8 cm
o
15 cm 8 cm

(a) (b) (c)

2.
k> R  5cm , h  12cm x> R  3.4 cm , h  8.9 cm K> R  3.7 cm , h  7 cm
X> R  5.2 cm , h  11cm g> R  5 mm , h  13.4 mm c> R  7 dm , h  16 dm
3.
k> R  3 cm x> D  18 dm K> R  3.7 cm X> D  23.2 cm
4.

k> S
36
25
 cm 2 x> S  16
46
m 2
K> S
12
5
 dm 2 X> S  1764 cm

5. S
. V  288 cm
3

32
ខ. V   m3
16 6 cm
6.
4.5 cm
.
o

ខ.

A B
BE 1
7. AB // CD  E
CE 2
CED 16 cm 2
C D
AEB
A B

E F
8. DEFG ABCD G
D C

M
N

225
A I B
9. ABCD I AB
o
BD CI O BIO
DOC C
D

10. 3 cm 9 cm 2 6m
2
m
11. 144 cm 2 1 256 cm 2 2
18 cm 1 cm 2
1
12. 9 600 m 2 cm 2
2 500
x
13. 6
ខ 6 cm 10 cm
10 4
. x
ខ. 2

14. SABCD 60 cm  SO


80 cm EFGK OO’= x
. EFGK x
ខ. x  30 cm ខ ABCD EFGK
2
15. BA  6 , SA  8 , SA '  SA S
3
cm
. A’B’
D C
ខ. A’B’C’D’ H
O
2 A B
2
A’B’C’D’     ABCD
3 D C
2
. SO '  SO O
 H
3
3 A
2 B
SA’B’C’D’     SABCD
3 S
1
16. SM '  SM , SO  10 cm , OM  6 cm
2
. O'M '
2
o M
 O '  
1
 O
2
ខ.
1
SO '  SO o M
2
3
. O’ 1
S O  
2

226
1
17.
4

9
18. -
10
7
10

19. R h
R h
h
.

ខ.
R
S

20. R h
h
R A o B
2
.
ខ.

C o D
R R
2

21. 2a
a 2a
.
2a
ខ.
2a

22. 2 cm r 
r
r
 3 2 cm

23. 1 cm

1 cm
1 cm 1 cm

DDCEE

3 227
១៨

1. rképÞRkLaxag nigépÞRkLaTaMgGs; ³ ÷KNnaépÞRkLaTaMgGs; ³


-cMeBaHrUb (a) S tamrUbmnþ S  S  S
T L B

EtépÞRkLa)atmanragCaqekaNniy½t eyIg)an ³
 
9 cm
1 
S B  6  8 82  4 2 
2 
15 cm  24 48  96 3 cm 2
÷KNnaépÞRkLaxag ³ 8 cm

tamrUbmnþ S  12 pa  12  4 15cm  9 cm
L naM[ S  336 cm  96 3 cm  502 .28 cm
T
2 2 2

KNna)an S  270 cm
L
2
dUcenH RkLaépÞTaMgGs;KW S  502 .28 cm . T
2

dUcenH épÞRkLaxagKW S  270 cm . L


2
-cMeBaHrUb (c) S

÷KNnaépÞRkLaTaMgGs; ³ 14 cm

tamrUbmnþ S  S  S
T L B

EtépÞ)at S  15 cm 15 cm  225 cm


B
2 o
8 cm

naM[ S  270 cm  225 cm  495 cm


T
2 2 2

÷KNnaépÞRkLaxag ³
dUcenH épÞRkLaTaMgGs;KW S  495 cm . T
2
tamrUbmnþ S  Ra  3.14  8 cm 14 cm
L

KNna)an S  351 .68 cm 2

-cMeBaHrUb (b) S L

dUcenH épÞRkLaxagKW S  351 .68 cm . L


2

÷KNnaépÞRkLaTaMgGs; ³
14cm
tamrUbmnþ S  S  S
T L B

EtépÞ)at S  R  3.14  8 cm  200 .96 cm


8 cm

÷KNnaépÞRkLaxag ³ B
2 2 2

tamrUbmnþ S  12 pa  12  6  8 cm 14cm
L
naM[ S  351 .68 cm  200 .96 cm
T
2 2

KNna)an S  336 cm
L
2 KNna)an S  552 .64 cm
T
2

dUcenH épÞRkLaxagKW S  336 cm . L


2 dUcenH épÞRkLaTaMgGs;KW S  552 .64 cm . T
2

228
2. KNnamaDekaNtamkrNIdUcxageRkam ³ g> eyIgman R  5 mm , h  13.4 mm
k> eyIgman R  5cm , h  12cm tamrUbmnþ V  13 S h b¤ V  13 R h
B
2

tamrUbmnþ V  13 S h b¤ V  13 R h
B
2
naM[ V  13  3.14   5 mm 13.4 mm 2

naM[ V  13  3.14  5 cm 12 cm


2
KNna)an V  70.127 mm 3

KNna)an V  314 cm 3
dUcenH manDekaNKW V  70.127 mm . 3

dUcenH manDekaNKW V  314 cm . 3

c> eyIgman R  7 dm , h  16 dm
x> eyIgman R  3.4 cm , h  8.9 cm tamrUbmnþ V  13 S h b¤ V  13 R h
B
2

tamrUbmnþ V  13 S h b¤ V  13 R h
B
2 naM[ V  13  3.14  7 dm 16 dm2

naM[ V  13  3.14  3.4 cm  8.9 cm


2 KNna)an V  820 .59 dm 3

KNna)an V  107 .685 cm 3 dUcenH manDekaNKW V  820 .59 dm . 3

dUcenH manDekaNKW V  107 .685 cm . 3


3. rképÞRkLaEsV‘ nigmaDb‘UltamkrNInmI Yy² ³
k> eyIgman R  3 cm
K> eyIgman R  3.7 cm , h  7 cm -épÞRkLaEs‘V S  4R  4  3.14  3 cm
2 2

tamrUbmnþ V  13 S h b¤ V  13 R h
B
2
KNna)an S  113 .04 cm 2

naM[ V  13  3.14  3.7 cm  7 cm2


dUcenH épÞRkLaEs‘VKW S  113 .04 cm . 2

KNna)an V  100 .302 cm 3

-maDb‘Ul V  43 R  43  3.14  3 cm


3 3

dUcenH manDekaNKW V  100 .302 cm . 3


KNna)an V  113 .04 cm 3

X> eyIgman R  5.2 cm , h  11cm dUcenH épÞRkLaEs‘VKW V  113 .04 cm . 3

tamrUbmnþ V  13 S h b¤ V  13 R h 2
B
x> eyIgman D  18 dm
naM[ V  13  3.14  5.2 cm 11cm 2

naM[ R  D2  182dm  9 dm
KNna)an V  311 .32 cm 3

-épÞRkLaEs‘V S  4R  4  3.14  9 dm


2 2

dUcenH manDekaNKW V  311 .32 cm . 3


KNna)an S  1017 .36 dm 2

dUcenH épÞRkLaEs‘VKW S  1017 .36 dm . 2

229
-maDb‘Ul V  43 R  43  3.14  9 dm
3 3 36

eyIg)an R
1 25  1

36 1 6 3
  
KNna)an V  3052 .08 dm 3 2 2 25 2 5 5

dUcenH épÞRkLaEs‘VKW V  3052 .08 dm . 3 dUcenH KNna)ankaMEsV‘KW R  53 cm .

k> eyIgman R  3.7 cm x> eyIgman S  1646


m 2

-épÞRkLaEs‘V S  4R  4  3.14  3.7 cm


2 2
tamrUbmnþ épÞEs‘V S  4R 2

KNna)an S  171 .95 cm 2


naM[ R  4S enaH R  4S
2

1 S
2 
dUcenH épÞRkLaEs‘VKW S  171 .95 cm . 2
46

enaH R
1 16  1 46  46  0.848 m

-maDb‘Ul V  R 3   3.14  3.7 cm
4
3
4
3
3 2 2 16 8

KNna)an V  212 .07 cm 3 dUcenH KNna)ankaMEsV‘KW R  46


8
m .
dUcenH épÞRkLaEs‘VKW V  212 .07 cm . 3
K> eyIgman S  125  dm 2

tamrUbmnþ épÞEs‘V S  4R 2

X> eyIgman D  23.2 cm


naM[ R  4S enaH R  4S
2

1 S
2 
naM[ R  D2  23.22 cm  11.6 cm 12

-épÞRkLaEs‘V S  4R  4  3.14  11 .6 cm
2 2
enaH R
1
2
5


1 12
2 5

60
10
KNna)an S  1690 .08 cm 2

KNna)an R
60

15
dm
dUcenH épÞRkLaEs‘VKW S  1690 .08 cm . 2 10 5

dUcenH KNna)ankaMEsV‘KW R  15
dm .
-maDb‘Ul V  R 3   3.14  11.6 cm
4
3
4
3
3 5

KNna)an V  6534 .95 cm 3


X> eyIgman S  1764 cm
dUcenH épÞRkLaEs‘VKW V  6534 .95 cm . 3
sUmEkBI S  1764 cm eTACa S  1764  cm 2

eRBaH S CaépÞ minGaceRbIxñatRbEvg)aneT ¡¡¡


4. rkkaMEs‘VedaysÁal;épÞRkLadUcxageRkam ³ tamrUbmnþ épÞEs‘V S  4R 2

k> eyIgman S  36 25
 cm 2
naM[ R  4S enaH R  4S
2

tamrUbmnþ épÞEs‘V S  4R 2

enaH R  1764 
 441  21 cm
4
naM[ R  4S enaH R  4S  12 S
2

dUcenH KNna)ankaMEsV‘KW R  21 cm .
230
5. KNnakaMénb‘UledaysÁal;maD ³ naM[ V   3.14  4.5 cm  6 cm
1
C
3
2

k> eyIgman V  288 cm 3


KNna)an V  127 .17 cm
C
3

tamrUbmnþ maDb‘Ul V  43 R 3
ehIy V  43 R
S
3

naM[ R3 
3V
4
enaH R3
3V
4 naM[ V  43  3.14  4.5 cm  381.51 cm
S
3 3


eyIg)an R  34V  3  288
3
4
3 eyIg)an V  127.17  12  381.51
b¤ R  216  6 cm
3 KNna)an V  317 .925 cm 3

dUcenH kaMb‘lU KNna)anKW R  6 cm . dUcenH maDsUlItKW V  317 .925 cm . 3

x> eyIgman V  1632


 m b¤ V  2 m3 3 x> rképÞRkLaTaMgGs;énsUlIt ³
tamrUbmnþ maDb‘Ul V  43 R 3 épÞénsUlIt S  S  12 S Edl S CaépÞRkLa
L S L

xagénekaN nig S CaépÞénEsV‘


naM[ R3 
3V
enaH R3
3V S

4 4 eday S  Ra
L
3V 3 3  2
eyIg)an R3
4

4
Et tamRTwsþIbTBItaK½r a  R  h 2 2

b¤ R3
3 3 12 3 12
  m
enaH S  R R  h
L
2 2

2 8 2
b¤ S  3.14  4.5  4.5  6
L
2 2

dUcenH kaMb‘lU KNna)anKW R  .


3
12
m  14.13  56.25
2
 14.13  7.5  105.975 cm 2

6. eyIgmanekaN nigknøHEs‘VdUcrUb ³ ehIy S  4R  4  3.14  4.5


S
2 2

S KNna)an S  254 .34 cm


S
2

eyIg)an S  105.975  12  254.34


6 cm
KNna)an S  233 .145 cm 2

4.5 cm
o
dUcenH épÞénsUtItKW S  233 .145 cm . 2

k> rkmaDTaMgGs;énsUlIt ³
maDénsUlIt V  V  12 V C S
rMlwk ³ -maDBIr:amIt ³ V  13 S h B

Edl V CamaDekaN nig V CamaDEsV‘


C S
-maDekaN ³ V  13 R h 2

eday V  13 S h  13 R h
C B
2
-maDEsV‘ ³ V  43 R 3

231
7. rképÞRkLaén AEB ³ 9. etIépÞRkLa BIO tUcCagépÞRkLa DOC
A B b:unµandg ? A I B
E o

D C
C D

eday AB// CD enaH AEB CED enaH eday AB// DC enaH BIO DOC enaH
pleFobépÞRkLa esµIkaerénpleFobFatuRtUvKña pleFobépÞRkLa esµIkaerénpleFobFatuRtUvKña
2 2

eyIg)an SS   CE
BE 
AEB


naM[ SS   DC
BIOBI 


CED DOC

b¤ S AEB  1 
 
2
Et AB  DC  2BI eRBaH I kNþal AB
16 2 2

naM[ 16
S AEB   4 cm 2
eyIg)an SS   2BIBI  b¤ SS  14
BIO BIO

DOC DOC
4
naM[ S BIO
1
 S DOC
dUcenH rk)anépÞRkLa S AEB  4 cm 2 . 4

dUcenH eRbobeFob)an S BIO 


1
4
S DOC .
8. etIépÞRkLakaer DEFG tUcCagépÞRkLakaer
ABCD b:unµandg ? 10. rképÞRkLaBitR)akdénCBa¢aMgKitCa m ³ 2

A B eyIgdwgfa CBa¢aMgmYyRtUv)aneKftbRgÜm[eTA
E F CarUbft Edlmankm<s; 3cm nigmanépÞRkLa
D G C 9 cm ehIyCBa¢aMgmankm<s; 6 m
2

M
mann½yfa rUbfténCBa¢aMg nigCBa¢aMgCarUbdUcKña
N -ebItag S CaépÞCBa¢aMg enaHeyIg)anpleFob
eday DEFG nig ABCD CakaerdUcKña épÞRkLa CakaerénpleFobFatuRtUvKñaKW ³
2

eyIg)an SS   DG 
2
S  6m 

DEFG
  
DC 
ABCD 9 cm 2
 3 cm 
Et DG DM

DC DN

DG 9

DC 15
DG 3

DC 5
b¤ S

36 m 2
2
9 cm 2 9 cm 2
S DEFG  3 
naM[  
S ABCD  5 

S DEFG

S ABCD 25
9
S
9 cm 2  36 m 2
9 cm 2
enaH 9
S DEFG  S ABCD
25
S  36 m 2

dUcenH eRbobeFob)an S DEFG 


9
25
S ABCD . dUcenH CBa¢agM manépÞBitR)akdKW 36 m . 2

232
11. etIvimaRtenH esµIb:unµan m ebIenAkñúgEpnTITIBrI ? 13. k> KNna x ³
eyIgdwgfa épÞRkLaénTIFøasalamYymanBIr ebI
eKdak;vaenAkñúgEpnTIBIrepSgKña C2
C1
6
x

mann½yfa épÞRkLaTaMgBIrénsala CarUbdUcKña C3


10 4
-ebI x CavimaRtmYyenAkñúgEpnTITIBIrenaH 2
eyIg)anpleFob ³
144 cm 2  18 cm 
 
2
edaymuxkat;Rsbnwg)at enaHekaNTaMgbIdUcKña
256 cm 2  x 
eyIg)an pleFob ³
naM[ 18 cm
x

144 cm 2
256 cm 2 3
1 2
R1 x 3
V1  x   x 
18 cm 12 cm
 
V2  x  4 
b¤ 3 
 


R2 x  4 
1 2 x 4

x 16 cm 3
2
18 cm 3 R12  x 
  
x 4 R22  x  4 
18 cm  4 R1 x
x 
3 R2 x  4
x  24 cm
eday R  D2  62  3 nig R  D2  102  5
1
1
2
2

dUcenH vimaRtenHesµI 24 cm enAkñúgEpnTITIBIr naM[ x x 4  53 b¤ 5x  3x 12 enaH x  6


12. rképÞRkLaénTIFøaCa cm ebIvaenAkñúgbøg;mYy ³
2
dUcenH KNna)an x  6 cm .
-EpnTImanmaRtdæan 2500
1
mann½yfa 1m enA
x> rkmaDénekaNenaH ³
elIEpnTI RtUvnwg 2500m énrUbBit
eyIgmanpleFob ³
-eyIgdwgfa épÞRkLaénTIFøaBit 9600 m 2

V1  x 
3

  , x6
-ebItag S CaépÞRkLaénTIFøaenAelIEpnTIenaH V3  x  4  2 

eyIg)anpleFob ³
3
V1  1 
 
2
V3  2 
S  1  V3  8V1
 
 2500 
2
9600 m
S 1 Et V  13 R x  13  3.14  3 cm  6 cm
1 1
2 2


9600  10000 cm 2
6250000
2
KNna)a V  56.52 cm
1
3

naM[ S
96 000 000 cm
6 250 000
 15.36 cm 2
naM[ V  8  56 .52  452 .16 cm
3
3

dUcenH épÞRkLaénTIFøaKW S  15 .36 cm 2 . dUcenH KNna)anmaDénekaNKW


452 .16 cm 3
.
233
14. k> rkRCugénkaer EFGK tamtémø x ³ b¤ V   13  60  1
2 
 80     37 .5 2  50 
 3 
tambRmab; RbFaneyIgKUsrUb)an ³  96000  23437.5
S  72562.5 cm3

dUcenH maDkMNat;BIr:amItKW 72562 .5 cm . 3

K G
80 cm O
E F
x
15. k> KNna AB ³
D


C
eyIgmanrUb Edlman BA  6 cm , SA  8 cm
O
A
60 cm B ehIy SA  23 SA S
edayeKkat;Rsbnwg)at enaHBIr:amItTaMgBIrdUcKña D C

eyIg)anpleFob ³ A
O
B
H

VSEFGK  80  x 
3
D C
 
VSABCD  80  O
 H

S EFGK 80  x 
1 A B
 80  x 
3
3  
1
S ABCD  80  80  edayBIr:amIt SABCDdUcBIr:amIt SABCD
3
eyIg)a pleF[bRtUvKñaesµIKñaKW ³
 80  x 
2 2
a EFGK
  2
a ABCD  80 
2
AB SA AB 3
SA
AB 2
a EFGK 80  x 
AB SA 6
b¤ 
SA
b¤ 6

3

6 2
a ABCD 80
naM[ AB   4 cm
eday a  AB  60 cm
ABCD
3

naM[ a 60  8080 x b¤ a 60
EFGK EFGK
 1
x dUcenH KNna)an AB  4 cm .
80
enaH a  60  34 x cm
EFGK x> KNnaépÞRkLaénkaer ABCD
dUcenH RbEvgRCugénkaer EFGK KW eday S  AB  4cm  16 cm
ABCD
2 2 2

60  x cm 
3
. dUcenH KNna)an S  16 cm . ABC D
2

4
2
2
x> KNnakMNat;BIr:amIt ebI x  30 cm ³ -epÞógpÞat;fa S ABCD     S ABCD
3
naM[ BIr:amIt EFGK man ³ eday S  AB  6 cm  36
ABCD
2 2

-RCug EF  60  34  30  37.5 cm 2

naM[ 16   23   36 b¤ 16  16 Bit


-km<s; h  80  30  50 cm
eday maDkMNat;BIr:amIt V  V  V SABCD SEFGK
dUcenH BitCaRtwmRtUvcMeBaHkar[epÞógpÞat; .
234
K> bgðajfa SO  23 SO ³ eday ekaNTaMgBIrdUcKña eyIg)anpleFob ³
1
eyIgman pleFobRtUvKña SSOO  AAB
B 
OM  SM 

OM  2
b¤ 
SM
OM  1
 b¤
OM SM OM SM OM 2
eday AB  4 cm nig AB  6 cm naM[ OM  
OM
enaH 6
OM    3 cm
naM[ SSOO  64 b¤ SO  23 SO 2 2

dUcenH KNnna)an OM   3 cm .


dUcenH bgðaj)anfa 2
SO  SO
3
. 2
1
3
-epÞógpÞat;fa S     S
2
³
-TajbBa¢ak;fa V   23 
SABCD  VSABCD
S   OM  
2

eyIgman pleFob 
S  OM 

eyIgmanpleFobRtUvKñaKW
b¤ SS   63  enaH
2 2
1
2 
3
S     S
VSABCD  SO 
3
VSABCD  3
SO  2

VSABCD  SO 
 b¤ 
VSABCD  SO 

2
1



 dUcenH epÞógpÞat;)an S     S
2
.
3
VSABCD  2 
naM[  
VSABCD  3 
3
x> bgðajfa SO  12 SO ³
2
Taj)an VSABCD     VSABCD
3 eyIgman pleFob SSOO  OOM M 

dUcenH bBa¢ak;)anfa enaH SSOO  63 naM[ SO  12 SO


3
2
VSABCD     VSABCD
3
. dUcenH bgðaj)anfa 1
SO  SO .
2

16. k> KNna OM  ³ K> RsaybBa¢ak;fa 1


VO   VO    
3

³
2
eyIgmanrUbEdlman SM   12 SM , SO  10 cm VO   OM   3
eyIgmanpleFob  
nig OM  6 cm S
VO   OM 
3
3
b¤ VO 
VO 
 
6
3
M 1
o b¤ VO 
VO 
 
2
3
1
o M naM[ VO      VO 
2

tag S CaépÞRkLafasmanp©it O nig S  CaépÞ 1


3

dUcenH bgðaj)anfa VO   VO     .


RkLafasmanp©it O 2

235
17. rkCeRmIsrbs;GñkTijEpø«Lwk edIm,I[cMeNj³ 19. k> KNnamaDekaN nigmaDsIuLaMg ³
÷tag V CamaD«LwktUcmYy mankaM r
1

V CamaD«LwkFMmYy mankaM R
2

eyIgcat;Tukfa «LwkTaMgtUcTaMgFMmanragCaEsV‘ h
dUcKña enaHeyIg)anpleFobRtUvKña ³ 
3
V r
   EteyIgdwgfa r  R
1 1 R
V R 4
2

1 
3 -maDekaN Edlmankm<s; h nigrgVas;kaM R KW
V1  4  V  S h  R h  1.05R h ÉktamaD
R 3 1 1
V1  1 
eyIg)an 
V2  R 
 b¤  
V2  4  3
B
3
2 2

 
 
dUcenH maDekaNKW V  1.05R h ÉktamaD 2

naM[ V1

V2 64
1
b¤ 1
V1  V2
64
-maDsIuLaMg Edlmankm<s; h nigrgVas;kaM R KW
enaH 1
2V1  2  V2 b¤ 1
2V1  V2
64 32 V   S h  R h  3.14 R h ÉktamaD
B
2 2

tampleFob eyIgeXIjfa maD«LwktUcBIrEpø


tUcCag maD«LwkFMmYycMnnY 32 dg dUcenH maDsIuLaMgKW V   3.14R h ÉktamaD 2

dUcenH GñkTijRtUvyk«LwkFM eTIbcMeNj . x>eRbóbeFobmaDekaN nigmaDénsIuLaMg ³


2
eyIg)an VV  13..0514RR hh  0.33
18. rkCeRmIsénm:asuInEdlcMeNjCag ³ 2

tag V CamaDm:asIuntUc nig V CamaDm:asIunFM


1 2
dUcenH eRbóbeFob)an V  0.33V  .
eday m:asIunTaMgBIrmanragsIuLaMgdUcKña man
20. k> KNnamaDénsIuLaMgenaH ³
pleFobvimaRtRtUvKñaénm:asIunTaMgBIresµI 109
3
S
V1  9 
eyIg)an  
V2  10 
b¤ V1

729
V2 1000

naM[ V1 
729
1000
b¤ V  0.729V
V2 1 2
h
A o B
Et témøma: sIuntUc  0.7 éntémøm:asIuFM
eday maDm:asIuntUcesµI 0.729 énmaDm:asIunFM 
C
Et témøm:asIuntUcesµI 0.7 éntémøm:asIunFM R
o
R
D

2
dUcenH GñkTijRtUveRCIserIsykm:asIuntUc edayRtIekaN SAB dUcnwgRtIekaN SCD
eTIbcMeNjCag . R
2
eyIg)an SSOO  CD
AB
Et AB
CD

2R
2 1
2

236
SO  1
naM[ SO 2
 b¤ SO  12 SO  12 h 22. kMNt;témø r edIm,I[maDKUbesµIBIrdgmaDEsV‘ ³
2
R
enaHmaDsIuLaMg V      OO
2 
r
2
naM[ V 
R
4
 SO  SO 
2 cm
R  1  R 2 h
2
b¤ V 
4 
 h  h 
2 

3.14 R 2 h
4 2
tag V CamaDKUb nig V  CamaDEsV‘
KNna)an V
8
 0.39 R 2 h
eyIg)an V  2V 
4
dUcenH maDsIuLaMg V  0.39R 2 h . 2 3  2  r 3
3
,  3

8  8r 3

x> KNnapleFobmaDénsIuLaMg nigmaDekaN ³ r3 1  r 1


eday maDekaNKW V   13 R h 2

dUcenH kMNt;)antémø r  1 cm .
1 2
R h
naM[ V
8
V 1 2

3
23. eRbóbeFobCaPaKryénépÞRkLakaer nigépÞ
R h 8
3 RkLaénRtIekaNsm½gS ³
dUcenH KNna)anpleFobKW V 3

V 8
.
21. k> KNnamaDénsIuLaMgenaH ³ 1 cm 1 cm

-épÞRkLakaer S  1cm   1 cm 2
2

maDsIuLaMgKW
2a
-épÞRtIekaNsm½gS 1
S   a  h
2
V  S B h  R 2 h
2
1
Et R  a nig h  2a 2a h 1   
2 2

2
3
4
naM[
h
3
4

2
3

naM[ V    a  2a  6.28a ÉktamaD


2 3

enaH 1
S    1cm 
3
cm 
3 2
cm
2 2 4
dUcenH maDsIuLaMgKW V  6.28a ÉktamaD 3
3
S S
naM[  4 b¤

3
S  b¤3
S
x> KNnapleFobrvagmaDsIuLaMg nigmaDénKUb S 1 S 4 4

maDKUbKW V   2a   8a ÉktamaD


3 3 eday 4
3
 0.4330  43 .30 %

naM[eyIg)anpleFobKW ³ enaH S   43.30%S


V 6.28 a 3
  0.785
V 8a 3 dUcenH eebóbeFob)an S   43.30%S .
dUcenH KNna)anpleFob V
V
 0.785 .
237
sYs¡þI elakGñkmitþGñkGanCaTIRsLaj;rab;Gan enAkñgú EpñkenHelakGñknwg)aneXIj
288

“No part of this test may be reproduced in any form without permission”

RbsinebIelakGñkmanbBaða b¤cm¶l;Rtg;cMNucNa EdlmanenAkñúgEpñkenH elakGñkGacTak;Tg;eTAkan;


RKU b¤mitþPkþirbs;GñkEdlmansmtßPaB b¤GñkeroberogesovePAenHkñúgeBlevlasmKYr .

iv
1) Calculate and simplify.
 x 2  6 x  8   x 2  2 x  8 x 2
 6 x  8   x 2  2 x  8

2) Solve the following inequality.


3  x  2   5x  2 3  x  2   5x  2

3) Express a in terms of and b . a b


 2  a  b  2  a  b

4) Find the equation of the straight line that  0, 3


passed through point  0, 3 with a slope 7។
of 7.

5) Find x when l m n as shown in the x l m n


figure at right.
l l
3cm 4cm 3cm 4cm
m m
5cm xcm 5cm xcm

n n

6) Expand and simplify.


 2x  1   2 x  3 2 x  5  2x  1   2 x  3 2 x  5
2 2

7) Solve the following equation.


 x 1  0
2
 x 1 0
2

238
8) Calculate and simplify.
5 7 5 5  
10
5  5 7  5 5   10
5

9) Find the range of y when the domain is y


2  x  1 regarding the quadratic 1 2
1 2  x  1 y x ។
function y  x 2 . 2
2

10) The width and diagonal of a rectangle are


5 cm and 35 cm, respectively. Find its 5 cm 35 cm។
length.

11) Factorize the following expression.


xyz  xy  xz  x xyz  xy  xz  x

12) Given the quadratic inequality 5x2  19 x  4  0 ។


5x2  19 x  4  0 . ① ។
① Solve the quadratic inequality. ② x ①។
② Draw the range of x found in ① on a
number line.

3
13) Find cos A given that sin A  for ABC cos A ABC
5
3
with C  90o . sin A  C  90o ។
5

14) How many different ways are there to


create a team consisting of one male and
one female from among 4 boys and 5 girls?
4 5 ។

15) Given that the geometric sequence below.


125 , 25 , A , 1 , B , … 125 , 25 , A ,1, B ,…
① Find A . ① រក A ។
② Find B . ② រក B ។

239
16) Expand and simplify the following
expression.
 x  y  4x  y    2x  y  2x  y 
 x  y  4x  y    2x  y  2x  y 

17) Express 2010 as the product of prime 2010 ។


numbers.

18) Find x satisfying the following equation. x


4 x2  5x  8  0
4 x2  5x  8  0

19) Calculate and simplify the


following

expression.  52 2  5 2 
2 5

2  52 2  5 2   2 5
2

20) Find the value of a when the parabola ) a y  ax 2


y  ax 2 passes through point  2 ,  8 .  2 ,  8 ។

21) In the diagram on the right, find x when x DE BC :


DE BC . C C

E E
6 6
4 4

A B A B
x D 3 x D 3

22) The diagram on the right shows a


right-angled triangle with BC=3cm , BC = 3cm , AC = 7cm C = 90o ។
AC = 7cm and C=90o . Find the length, AB cm ។
in cm, of side AB.
A A

7cm 7cm

B C B C
3cm 3cm

240
23) Expand and simplify the following
expression.  x  6 y   x2  6 xy  36 y 2   x  6 y   x2  6xy  36 y 2 

24) Factor the following expression.


x8  1
x8  1

25) Calculate the following expression.


Rationalize the denominator if your answer
is a fraction.

2 2 2  2  22 

26) Assume that the universal set consists of ) 1 9។


integers from 1 to 9. When A = {1 , 3 , 7 , 8 }
A = {1 , 3 , 7 , 8 } and B = {1 , 4 , 6 , 7 }
find the elements of A  B . B = {1 , 4 , 6 , 7 } A B ។

27) Consider the quadratic inequality : 2 x 2  3x  9  0


2 x 2  3x  9  0 . ① x ។
① Solve for x in the quadratic inequality.
② x ① ។
② Show the range of x in ① on a number
line.

28) Calculate A  3B when A  3x 2  2 x and A  3B


B  x2  x 1 . A  3x  2 x 2
B  x2  x 1 ។

29) Find  that satisfies sin   cos for  sin   cos


0o    180o . 0    180 ។
o o

30) Calculate the following combinations. )


① 10 C2
① 10 C2
② 10 C8
② 10 C8

241
31) Expand and simplify the following ៣១)
expression.  x  3 y    x  y  x  5 y 
2
 x  3 y    x  y  x  5 y 
2

32) Factor the following expression. ៣២)


4 x2  225
4 x2  225

33) Solve for x in the following equation. ៣៣) x


x 2  10 x  6  0
x 2  10 x  6  0

34) Simplify the following expression. ៣៤)


5 3 2 5  15
5 
5 3 2 5   15
5

35) y is directly proportional to the square of x ៣៥) y x y  45


and y  45 when x  3 . Express y in terms x  3។ y
of x .
x។

36) In the diagram on the right, find x when ៣៦) x m


m. x x

4 4
m 1 m 1
2 2

37) For a right-angled triangle, Find the length )


of the hypotenuse when the other two sides 5 cm
are 5 cm and 12 cm.
12 cm ។

242
38) Expand and simplify the following ៣៨)
expression.  x  2 y   x 2  2 xy  4 y 2   x  2 y   x2  2xy  4 y 2 

39) Factor the following expression. ៣៩)


x3  6 x 2 y  12 xy 2  8 y 3 x3  6 x 2 y  12 xy 2  8 y 3

40) Solve the following linear inequality. ៤០)


3x 12  9x 18
3x 12  9x 18

41) Find the coordinates of the vertex of the ៤១)


parabola y  x 2  8 x  15 . y  x 2  8 x  15 ។

42) ① Solve the quadratic inequality ៤២) ① ២


5 x 2  14 x  3  0 .
5 x 2  14 x  3  0
② Show the range of x in ① on the
② x ។
number line.

43) Find the value of 10 P3 . ៤៣) 10 P3 ។

44) A bag contains6 white balls and 2 red balls. ៤៤) 6


If three balls are chosen at random without 2។
replacement, what is the probability that
only one ball is red. ។

5 5
45) If  is an acute angle and cos   , ៤៥)  cos   ។
7 7
① find the value of sin  . ① sin  ។
② find the value of tan  . ② tan  ។

243
46) Expand and simplify the following ៤៦)
expression. x  x  2   x 1 x  3 x  x  2   x 1 x  3

47) Factor the following expression. ៤៧)


9x 2  y 2 9x 2  y 2

48) Solve for x in the following equation. ៤៨) x


x2  5x  6  0
x2  5x  6  0

49) Simplify the following expression. ៤៩)

1  5 
2
 20
1  5 
2
 20

50) y is directly proportional to the square of y x y 1


x and y  1 when x  2 . Express y in x  2។ y
terms of x .
x។

51) Find the length of the side of a square ៥១)


whose length of the diagonal is 4 cm.
4 cm ។

52) In the diagram on the right, find the value ៥២) x m


of x when m.
x 2 x 2

6 4 6 4
m m

53) Expand and simplify the following ៥៣)


expression.  x  y   x2  xy  y 2 
 x  y   x2  xy  y 2 

244
54) Factor the following expression. ៥៤)
x3  3x 2 y  3xy 2  y 3 x3  3x 2 y  3xy 2  y 3

55) Simplify the following expression. ៥៥)


4
3 1
 
 1 3 3  3  4

 1 3 3  3  
3 1

56) For the parabola y   x 2  ax  b , find the ៥៦) y   x 2  ax  b


value of a and b such that the coordinate of a b 1 , 2 ។
the vertex is 1 , 2 .

57) How many different three-member teams ៥៧)


can be formed fromed nine students?

58) Solve for x in the inequality ៥៨) x


2 x2  5x  3  0 . 2 x  5x  3  0 ។
2

1 1
59) Answer the following when sin   for ៥៩) sin  
2 2
0o    90o . 0o    90o
① find the value of cos  . ① cos  ។
② find the value of tan  .
② tan  ។

60) Answer the following for two sets, ៦០)


A = { 2 , 3 , 5 , 7} and B = { 1 , 2 , 3 , 4}.
A = { 2 , 3 , 5 , 7} B = { 1 , 2 , 3 , 4} ។
① Find the elements of set A  B and list
them. ① A B
② Find the number of elements in set ។
A B . ② A B ។

245
61) Simplify the following expression. ៦១)
a)   8   4  10 a)   8   4  10
b)  56   8   3 b)  56   8   3
c)  1   2    2 
4 3
c)  1   2    2 
4 3

7 7 1 1
d)     7 7 1 1
d)    
8 9 16 2 8 9 16 2
e) 6  2  3 2  e) 6  2  3 2 

f ) 10 2 5  10   1
10
 5  10  
f ) 10 2 5  10   1
 5  10 
10
g ) 8x  2  5x  7  g ) 8x  2  5x  7 
h) 6  0.6 x  0.4   0.3  2 x  5  h) 6  0.6 x  0.4   0.3  2 x  5 
i) 2  3x  4 y   5  4 x  3 y  i) 2  3x  4 y   5  4 x  3 y 
3x  5 y x  5 y 3x  5 y x  5 y
j)  j) 
8 6 8 6
k ) 32 x y  4 x y  8 x
3 2 2
k ) 32 x y  4 x y  8 x
3 2 2

2 2
 3   6  3   3
2
  6  3 
2
l )   x3 y 2     x 4    y 2  l )   x3 y 2     x 4    y 2 
 5   5  2   5   5  2 

62) Expand the following expression. ៦២)


a)  4 x  3 3 x  2  a)  4 x  3 3 x  2 
b)  x  3 y  x  5 y    x  4 y 
2
b)  x  3 y  x  5 y    x  4 y 
2

63) Factorize the following expression. ៦៣)


a) 16 x 2  25 a) 16 x 2  25
b) 9 x 2  24 xy  16 y 2 b) 9 x 2  24 xy  16 y 2

64) Solve the following equations. ៦៤)


a) 4 x  9   x  6 a) 4 x  9   x  6
b) 0.5 x  3.2  0.8 x  1.6 b) 0.5 x  3.2  0.8 x  1.6
c) x 2  14 x  45  0 c) x 2  14 x  45  0
d ) x  8x  9  0
2
d ) x2  8x  9  0

246
65) Solve the following simultaneous equations. ៦៥) ( )
7 x  5 y  1
a) 
3x  2 y  1 7 x  5 y  1
a) 
1 3
3x  2 y  1
 x  y 1
b)  4 2
0.5 x  0.5 y  0.1 1 3
 x  y 1
b)  4 2
0.5 x  0.5 y  0.1

66) Find the value of a3  b2 when a  2 and ៦៦) a3  b 2 a  2


b 3.
b 3។

67) Solve the following in equation. ៦៧)


4x  11  2x  15
4x  11  2x  15

68) y is directly proportional to x and y  4 ៦៨) y x y  4


when x  8 . Find the value of y when x 8។ y
x  4 .
x  4 ។

69) y is a linear function of x . Express y as a ៦៩) y x។ y


function of x when the graph passes x
through point  2 , 4 with slope of 3.
 2 , 4 3។

70) Is 221 a prime number? ៧០) 221 ?

71) Find degree measure of each exterior angle ៧១)


of a regular nonagon (regular 9-sided, 9-
( 9 , 9 )។
angled, polygon).

247
72) In the accompanying diagram, find x when ៧២) x m
m

1cm 1cm
xcm xcm
4cm 4cm
8cm 8cm
m m

73) In the rectangle ABCD with 6 cm length ៧៣) ABCD 6 cm


and 3 cm width, find the length of the
3 cm ។ AC ។
diagonal AC.

A 6cm A 6cm
D D
3cm 3cm

B C B C

74) Calculate. ៧៤)


a )  13   15    4  a )  13   15    4 
b)  8  5  20   5  b)  8  5  20   5 
c)  2   42 c)  2   42
3 3

5 1
d )    0.75    0.2 5 1
d )    0.75    0.2
6 4 6 4
e)  18  72  2 e)  18  72  2
f)
3
5
2 5 5  4 5  f)
3
5
2 5 5  4 5 
g ) 7  3x  2   9  2 x  5  g ) 7  3x  2   9  2 x  5 
4x  1 5x  3 4x  1 5x  3
h)  h) 
6 8 6 8
i) 3  9 x  y   5  x  2 y  i) 3  9 x  y   5  x  2 y 
j ) 0.3  x  8 y   8  0.4 x  0.2 y  j ) 0.3  x  8 y   8  0.4 x  0.2 y 
k ) 18 x y   2 xy   6 x y
2 3
k ) 18 x 2 y   2 xy   6 x 3 y
2 2
 5  1   18 
2 2
l )   xy    xy 2     y   5  1   18 
l )   xy    xy 2     y 
 9  6   5   9  6   5 

248
75) Expand and simplify the following ៧៥)
a )  7 x  5 y  3 x  4 y   2 y  10 y
expressions.
a )  7 x  5 y  3 x  4 y   2 y  10 y
b)  x  6    x  5  x  8 
2

b)  x  6    x  5  x  8 
2

76) Factor the following expressions. ៧៦)


a) x 2  3x  54 a) x 2  3x  54
b) x 2 y  4 y 3 b) x 2 y  4 y 3

77) Solve for x in the following equations. ៧៧) x


a ) 6 x  16  9 x  11 a ) 6 x  16  9 x  11
7 x  3 3x  1 7 x  3 3x  1
b)  b) 
8 4 8 4
c) x  4 x  21  0
2
c) x  4 x  21  0
2

d ) x2  8x  2  0 d ) x2  8x  2  0

78) Solve the following system of equations. ៧៨)


8 x  3 y  30 8 x  3 y  30
a)  a) 
5 x  9 y  3 5 x  9 y  3
0.8 x  0.3 y  0.8 0.8 x  0.3 y  0.8
 
b)  2 3 9 b)  2
 5 x  2 y  5
3 9
 5 x  2 y  5

79) Find the value of 9 x 2  2 xy when x  3 ៧៩) 9 x 2  2 xy


and y  1 . x  3 y  1។

249
80) A large dice and a small dice are both ៨០)
numbered 1 to 6. When they are rolled,
1 6។
what is the probability that the product of
the two numbers on the top faces is 9?
9។

81) Make y the subject of the formula ៨១) y 8x  3 y  7 ។


8x  3 y  7 . ( y 8x  3 y  7 )

82) y is inversely proportional to x and y  8 y x y  8


when x  3 . Find the value of y when
x  3។ y
x  2 .
x  2 ។

83) y is directly proportional to the square of x y x y  32


and y  32 when x  4 . Express y in x  4។ y
terms of x .
x ។

84) Find the measure of each exterior angle of a )


regular octagon in degrees. An octagon is ។ 8។
an 8-sided polygon.

85) In the diagram on the right, find x when x l m


l m.
l 41o l 41o
95o 95o

m x m x

250
86) In the diagram, point A, B, C and D lie on A, B, C D O។
the circumference of circle O. Find x
x BD
when BD is the diameter of the circle and
DBC  38o . DBC  38o ។
D D
A A
x O x O
38o 38o
B C B C

87) Calculate.
a )  12    13   4  a )  12    13   4 
b)  2  5  12   6  b)  2  5  12   6 
c)  4   2
2
c)  4   24
4 2

5 5
d )   0.2   0.5 5 5
d )   0.2   0.5
8 16 8 16
e)  2  8  18 e)  2  8  18
f ) 4  3x  5   8  2 x  3 f ) 4  3x  5   8  2 x  3
5x  3 x  1 5x  3 x  1
g)  g) 
2 3 2 3
h) 4  6 x  7 y   8  2 x  4 y  h) 4  6 x  7 y   8  2 x  4 y 
i ) 0.2  5 x  10 y   2  2 x  0.5 y  i ) 0.2  5 x  10 y   2  2 x  0.5 y 
j ) 28 x y   5 xy   35 xy
2 2 3
j ) 28 x 2 y 2   5 xy   35 xy 3
2
5   5   2  5   5
2
  2 
k )  x 2 y     xy 3     y 3  . k )  x 2 y     xy 3     y 3 
6   9   15  6   9   15 

88) Expand and simplify the following )


expressions. a )  x  7 y  x  7 y 
a )  x  7 y  x  7 y 
b)  x  4    x  2  x  8 
2

b)  x  4    x  2  x  8 
2

251
89) Factor the following expressions.
a) x 2  8 x  12 a) x 2  8 x  12
.
b) ax 2  4axy  5ay 2 b) ax 2  4axy  5ay 2

90) Solve for x in the following equations. x


a ) 9 x  19  11x  13 a ) 9 x  19  11x  13
7 x  4 5x  4 7 x  4 5x  4
b)  b) 
8 6 8 6
1
c) x 2   0 1
c) x 2   0
4 4
d ) x  2x 1  0
2
d ) x  2x 1  0
2

91) Solve the following systems of equations.


x  y  1
a)  x  y  1
3 x  2 y  18 a) 
3 x  2 y  18
2 x  y  3
 2 x  y  3
b)  1 1 
b) 
0.5 x  8 y  2 1 1
0.5 x  8 y  2

92) Find the value of 3 xy  6 xy 2 when x  5 3 xy  6 xy 2


and y  2 . x  5 y  2។

93) Express 2010 as the product of prime 2010 ។


numbers.

252
1 1
94) Solve for h in the equation mgh  mv 2 , ) h mgh  mv 2
2 2
where m  0 and g  0 . m0 g 0 ។

95) y is directly proportional to x and y  5 y x y5


when x  2 . Find the value of y when
x  2 ។ y
x  4.
x4 ។

96) y is directly proportional to the square of x y x y  3


and y  3 when x  3 . Express y in terms x  3។ y
of x . x ។

97) Find the measure of each interior angle of a )


regular octagon in the degrees. An octagon
។ 8។
is 8-sided polygon.

98) In the diagram on the right, find x when ) x m៖


m.
x x
2 2
6 6
4 4
m m

99) In the diagram on the right, points A, B, C A, B, C D


and D lie on the circumference of a circle. ។ x
Find x when BC : CD  1: 2 and
BC : CD  1: 2 CAD  46o ។
CAD  46o .
A A
o o
x 46 x 46

D D
B B
C C

253
100) Simplify.
a)  5  2  8  4 a)  5  2  8  4
b) 18  30   6  b) 18  30   6 
c)  5   32  4
2
c)  5   32  4
2

2
2 5 1 2 5 1
2
d)     d)    
3 6 3 3 6 3
e)  2  3  2 2  24 e)  2  3  2 2  24
  10
2
5 1 
  10
2
f) f) 5 1 
5 5
g ) 2 8 x  5  6  3x  2  g ) 2 8 x  5  6  3x  2 
h) 0.8  0.6 x  5   0.5  0.9 x  3  h) 0.8  0.6 x  5   0.5  0.9 x  3 
i) 3  4 x  8 y   7  2 x  6 y  i) 3  4 x  8 y   7  2 x  6 y 
3x  5 y 4 x  y 3x  5 y 4 x  y
j)  j) 
6 9 6 9
k )  28 xy  7 xy 2
3
k )  28 xy  7 xy 2
3

2
5 5  4 5  4
2
l ) x3 y   x 2 y   x 5
l ) x3 y   x 2 y   x
6 3  3 6 3  3

101) Expand and simplify the following


a )  3 x  4 y  3x  4 y 
expressions.
a )  3 x  4 y  3x  4 y 
b)  x  6  4 x  3   2 x  5 
2

b)  x  6  4 x  3   2 x  5 
2

102) Factor the following expressions.


a) x 2  4 x  12 a) x 2  4 x  12
b)  x  y   12  x  y   36
2
b)  x  y   12  x  y   36
2

254
103) Solve for x in the following equations. x
a ) 7 x  4  4 x  10 a ) 7 x  4  4 x  10
x  5 3x  7 x  5 3x  7
b)  1 b)  1
4 2 4 2
c ) 5 x 2  40  0 c ) 5 x 2  40  0
d ) x2  8x  4  0 d ) x2  8x  4  0

104) Solve the following systems of equations.


2 x  5 y  2 2 x  5 y  2
a)  a) 
 x  3 y  12  x  3 y  12
0.3x  0.4 y  0.1 0.3x  0.4 y  0.1
 
b)  6 4 7 b)  6 4 7
 5 x  3 y   15  5 x  3 y   15

105) Find the value of 2ab  b2 when a  4 2ab  b2 a4


and b  2 .
b  2 ។

106) Two dice, A and B, are both numbered A B


1 to 6. When they are rolled, find the 1 6។
probability that the sum of the numbers on
the top faces is 4. 4។

107) Solve for y in the equation 5x  2 y  3 . y 5x  2 y  3 ។

255
108) y is inversely proportional to x and y  9 y x y9
when x  4 . Find the value of y when x  4។ y
x  6 .
x  6 ។

109) y is directly proportional to the square of ) y x y 8


x and y  8 when x  4 . Express y in x  4។ y
terms of x . x។

110) Find the sum of the measure of interior ) ។


angle of a dodecagon. A dodecagon is a
12។
12-sided polygon.

111) In the diagram on the right, find x when x m


m.
62o 62o

x x
46o 46o
m m

112) In the diagram on the right, four points A, B, C D


A, B, C and D lie on the circumference of O។ x
circle O. Find x when AB = AC and
AB = AC ADB  64o ។
ADB  64o .
A A
x x
o
64 D 64o D
O O
 

B C B C

256
113) Simplify.
a )  8    10    16  a )  8    10    16 
b) 16  24   8  b) 16  24   8 
c )  3     2 
2 4
c )  3     2 
2 4

2
5 1 1
2
d)   4  5
d)   4 
9 3 9 3
e) 18  98  2 e) 18  98  2
  12
2
f ) 6 2 
  12
2

6 f ) 6 2 
6
g ) 7  4 x  3  5  5 x  4  g ) 7  4 x  3  5  5 x  4 
h) 0.6  3 x  0.7   2  0.3 x  0.2  h) 0.6  3 x  0.7   2  0.3 x  0.2 
i) 4  9 x  6 y   9  x  2 y  i) 4  9 x  6 y   9  x  2 y 
5x  3 y 2x  5 y 5x  3 y 2x  5 y
j)  j) 
12 9 12 9
k )  63 x y  9 xy 2
2 3
k )  63 x y  9 xy 2
2 3

2
 5  35 7 2 .  5  35
2
l )  xy   x 2 y 3  y l )  xy   x 2 y 3 
7 2
y
 6  48 20
 6  48 20

114) Expand and simplify the following


expressions. a )  5 x  y  6 x  y 
a )  5 x  y  6 x  y 
b)  x  7    x  8  x  6 
2

b)  x  7    x  8  x  6 
2

115) Factorize the following expressions.


a) x 2  100 a) x 2  100
b) ax 2  6ax  9a b) ax 2  6ax  9a

257
116) Solve for x in the following equations. x
a) 9 x  5  6 x  7 a) 9 x  5  6 x  7
b) 0.2  x  2   0.8  2 x  3 b) 0.2  x  2   0.8  2 x  3
c) x  3x  54  0
2
c) x 2  3x  54  0
d ) x2  8x  9  0 d ) x2  8x  9  0

117) Solve the following systems of equations.


2 x  7 y  11 2 x  7 y  11
a)  a) 
5 x  2 y  8 5 x  2 y  8
1.2 x  3.4 y  1.5 1.2 x  3.4 y  1.5
 
b)  2 4 1 b)  2
 3 x  9 y  3
4 1
 3 x  9 y  3

118) Find the value of 9a 2  ab2 when a  3 9a 2  ab2


and b  4 .
a 3 b  4 ។

119) Two dice, A and B, are both numbered A B


1 to 6. When they are rolled, find the 1 6។
probability the both number facing up
even. ។

120) Which number is the greatest among


3 3 , 2 7 and 5.
3 3,2 7 5។

258
121) y is directly proportional to x and y  18 ) y x y  18
when x  9 . Find the value of y when x  9 ។ y
x  2 .
x  2 ។

122) y is proportional to the square of x and ) y x y  4


y  4 when x  4 . Express y in terms x  4។ y
of x .
x។

123) Find the measure of each interior angle of ) ។


a regular pentagon. A pentagon is a
5-sided polygon. 5។

124) In the figure on the right, find x when x m


m.
107o 107o

85o x 85o x
m m

125) In the figure on the right, three points A, A, B C


B and C lie on the circumference of circle
O។ x AB  AC
O. Find x when AB  AC and
BAC  68 ។ o
BAC  68o .

C C
A o
A o
68 68
O O
x x

B B

259
126) For each equation, y   x  1 , y  x  1 , ១២៦) y   x 1 , y  x  1 ,
y  2 x  2 , y  3x  3 , . Its slope is y  2 x  2 , y  3x  3 ,
equal to its y-intercept. These straight
x  0។
lines pass through a certain point. Find the
coordinates of this point. ។

127) The figure shows trapezoid ABCD with ១២៧) ABCD


AD||BC and AE = DE where E is the
AD||BC AE = DE E
intersection point of AC and BD.
AC BD ។
A D A D
E E
B C B C
a) Prove that EBC is an isosceles
EBC ។
triangle.
b) Prove that AB = DC. AB = DC ។

128) Solve x 4  289 . Note that x is a positive ១២៨) x 4  289 ។ x


real number. Write only the answer. ។ ។

129) Three points A, B and C lie on circle O. ១២៩) A, B C O។


Each point is connected to the others. ។
Given that AB : BC : CA  3 : 4 : 5 .
AB : BC : CA  3 : 4 : 5 ។
A A

 
O O
B B
C C

a) Find CAB , ABC and BCA .


CAB , ABC BCA ។
Write only the answer.
b) Express AB : BC in the simplest ratio. ។
AB : BC ។

260
1 2 3 1 2 3
130) Given a parabola y  x  x  1 . Find ១៣០) y x  x 1។
2 2 2 2
the vertex, intersection points with
coordinate axis, and then draw its graph.

131) Each surface of cube ABCD-EFGH, as ១៣១) ABCD-EFGH


shown in the figure at right, has a distinct
label starting from 1 to 6. Each vertex,
from A to H, is labeled with a number that 1 6។ A H
is the sum of numbers on the surfaces
sharing the vertex. For example, vertex C
។ C
is labeled with a number that is the sum of
numbers on surfaces ABCD, BFGC and ABCD, BFGC CGHD។
CGHD. It is possible to label the surfaces
so that the numbers labeled on the vertices
8 ។
occur as eight consecutive integers.
Find the eight consecutive integers. ។

A D A D
B B
C C

E E
H H
F G F G

132) Simplify. ១៣២)


5x  2 y 5x  2 y   5x  y 
2
5x  2 y 5x  2 y   5x  y 
2

261
133) Solve the following equation. ១៣៣)
x2  1  5x x2  1  5x

134) Let y be directly proportional to the ១៣៤) y x


square of x and y  20 when x  2 . y  20 x  2 ។
Express y as a function of x . x។
y

135) Find the length of one side of a rhombus ១៣៥)


whose diagonals measure 10 cm and 6 cm.
10 cm 6 cm ។

136) Factorize the following expression. ១៣៦)


xz  x  2 y  2 yz xz  x  2 y  2 yz

137) In the accompanying figure, how many ១៣៧)


different shortest paths are there from
A B។
point A to point B.

B B

A A

262
3 3
138) Find  that satisfies sin   where ១៣៨)  sin  
2 2
0o    180o . 0o    180o ។

139) Find the coordinates of the center of mass ១៣៩) ABC


for ABC that is created by connecting A(-2 , 6) ,
three points A(-2 , 6) , B(7 , -9) and
C(1 , 0). B(7 , -9) C(1 , 0) ។

140) In the diagram shown on the right, ១៤០) ABCD


quadrilateral ABCD is inscribed in a ។ AB = DC
circle. Prove that AB = DC when AD||BC.
AD||BC ។
A D A D

B C B C

141) Using the equality 8051  72  902 , ១៤១) 8051  72  902


express 8051 as the product of prime 8051 ។
number. Write only your answer.

142) The equality  m2  n2 is satisfied for ១៤២)  m2  n 2


three positive integers , m and n . Prove ,m n។
that can be factored as the product of
prime number when n  m  1.
n  m 1 ។

263
143) The diagram on the right shows rectangular ១៤៣)
prism ABCD-EFGH. Answer the following
ABCD-EFGH ។
when AE = 19 cm, EF = 32 cm and
FG = 25 cm. AE = 19 cm, EF = 32 cm
Find the length of diagonal AG. FG = 25 cm.
AG ។
D C
D C
A
B
A
19 cm H B
G
25 cm 19 cm H G
E 32 cm F 25 cm
E 32 cm F

144) Find the remainder when the polynomial ១៤៤)


x3  2 x2  4 x  8 is divided by x  2 . x3  2 x 2  4 x  8 x2 ។

145) Find the value of sin135o . ១៤៥) sin135o ។

146) Find the value of tan 1  2  when ១៤៦) tan 1  2 
tan 1  2 and tan  2  3 . tan 1  2 tan  2  3 ។

១៤៦) ២ 3x  4 x  8  0
2
147) When the quadratic equation
3x 2  4 x  8  0 has two roots,  and  ,     ។
find the value of    .

264
148) The diagram on the right shows a ១៤៨)
semicircle whose center is O and diameter
O AB ។
is line segment AB. Point P lies on arc AB
and point H lies on AB such that PH is P AB H
perpendicular to AB, AH = 3 cm and AB PH AB ។
BH = 7 cm.
AH = 3 cm BH = 7 cm ។

P P

O O
A   
B A   
B
3cm H 7cm 3cm H 7cm

a) Find the lengths of line segments OP OP PH ។


and PH. Write only your answer. ។
b) Find the ratio of the lengths of line
segment PA to line segment PB, PA PB
PA:PB. PA:PB ។

១៤៩) ២ x xa 0
2
149) Consider the quadratic equation
x 2  x  a  0 where a is a constant. If one a ។ -
of the roots of the quadratic equation is 5,
5 a ។
find the value of a and the other root.
Write only your answer. ។

150) For two consecutive odd numbers, prove ១៥០)


that the square of the smaller odd number
subtracted from the square of the larger
odd number is equal to 4 times the even 4
number between the two odd number. ។

265
151) Let x cm be the radius of a sector whose ១៥១) x cm
perimeter is 20 cm.
20 cm ។
a) Express the length of the arc using x .
Write only your answer. x។
b) Find the radius of the sector such that ។
the area of the sector has the maximum
value. Also find the maximum area.
)

152) A die numbered 1 to 6 is rolled 3 times. ១៥២) 1 6


Find the probability that the same number
3 ។
is on the top face on the 1st and 2nd rolls
and a different number is on the top face ១ ២
on the 3rd roll. ៣។

153) For ABC, AB=2 , AC=3 and ១៥៣) ABC, AB=2 , AC=3
15 15
sin A  for 90o  A  180o . sin A  90o  A  180o ។
4 4
a) Find the area of ABC . Write only ABC ។
your answer.
b) Find the length of side BC. ។
BC ។

154) Expand and simplify the following ១៥៤)


express expression.
 x  3 y   x2  3xy  9 y 2 
 x  3 y   x2  3xy  9 y 2 

155) Factor the following expression. ១៥៥)


x3  3x 2 y  3xy 2  y 3
x3  3x 2 y  3xy 2  y 3

266
156) Simplify the following expression. ១៥៦)
52 6 52 6

157) Find the value of cos  when tan   3 ១៥៧) cos  tan   3
for 90o    180o . 90o    180o ។

158) Six students are divided into 2 rooms, A ១៥៨) 6 A B


and B, containing 3 students each. How

many different ways can the students be
divided ? ។

159) For two sets, A = { 1 , 4 , 7 , 10 } and ១៥៩) A = { 1 , 4 , 7 , 10 }


B = { 1 , 3 , 5 , 7 , 9 }, find the elements
B = { 1 , 3 , 5 , 7 , 9 }។
of set A  B and list them.
A B ។

160) Find the value of a such that the ១៦០) a x3  3x2  ax  4


polynomial x3  3x2  ax  4 is divisible x 2។
by x  2 .

161) Find the shortest distance between point ១៦១) (0 , 1)


(0 , 1) and the straight line x  3 y  7  0 . x  3y  7  0 ។

267
162) Find all sets of integer,  x , y , z  , ១៦២)  x , y , z
satisfying the following equality. Write ។
only your answer.

x 6  y 6  z 6  3xyz
x 6  y 6  z 6  3xyz

163) A sector has a perimeter 20 cm and its ១៦៣) 20 cm


central angle is less than 360o . Find the 360 ។o

range of the radius of the sector such that


the range of the area, A , is given by A។
16 cm 2  A  24 cm 2 . 16 cm  A  24 cm ។
2 2

164) Let P and Q be the points of intersection ១៦៤) P Q


between the circle x 2  y 2  5 and the x  y 5
2 2
x  y 1
straight line x  y  1 , where the value of
P
the x -coordinate of point P is less than of
point Q. Q។
a) Find the coordinate of points P and Q. P Q។
b) Let R be the point of intersection R
between the tangent line to the circle
at P and the tangent line to the circle at P Q។
Q. Find the coordinates of point R. R។

165) Find all sets of integer,  x, y, z  , ១៦៥)  x, y, z 


satisfying the following equality. Write ។
only your answer.

x 4  y 4  z 4  3xyz
x 4  y 4  z 4  3xyz

268
166) Consider the following proposition for ១៦៦) n។
integer n .
n  2n
2
n
“If n2  2n is an odd number, then n is an
odd number.” ។”
a) Give the contrapositive of the ។
proposition. Write only your answer. ។
b) Prove that the proposition is true using
your answer in (a).

167) When polynomial P  x  is divided by ១៦៧) P  x x 3


x  3 the remainder is 3. When P  x  is 3។ P  x x4
divided by x  4 the remainder is 1. Find 1។
the remainder when P  x  is divided by
P  x  x  3 x  4 ។
 x  3 x  4 .

168) A rectangular piece of land has length ១៦៨)


100m and width 50m. This land is drawn
100m 50m ។
1
on a scale map. 1
1000 ។
1000
Find the area, in cm 2 , of the piece of land
drawn on the map. You don’t need to cm 2 ។
write your steps. Write only your answer. ។

169) The surface area of a sphere of radius ១៦៩) r cm


r cm can be express as 4 r 2 cm2 , where 4 r cm
2 2

 is the ratio of the circumference of a

circle to its diameter.
a) Find the surface area of a sphere when 5 cm។
the radius is 5 cm. You don’t need to
write your steps. Write only your ។ ។
answer.
b) Find the radius of a sphere when the
surface area is 81 cm 2 . 81 cm 2 ។

269
170) Expand and simplify the following ១៧០)

 x  4 y   x2  4xy  16 y 2 
expression.
 x  4 y   x2  4xy  16 y 2 

171) Factor the following expression. ១៧១)


125x3  75x 2  15x  1
125x3  75x 2  15x  1

172) Find the volume of a sphere with radius of ១៧២) 6។


6.

173) A is an acute angle. Find the value of ១៧៣) A ។ tan A


2
tan A when cos A  . cos A 
2

3 3

174) How many six-digit integers can be ១៧៤)


formed using all of the following six
number 0, 1, 2, 3, 4 and 5 ?
0, 1, 2, 3, 4 5?

175) Find the coordinates of the vertex of the ១៧៥)


parabola y   x 2  8 x  5 . y   x2  8x  5 ។

270
176) Given the parabola y  2 x 2  4kx  3k  9 . ១៧៦) y  2 x 2  4kx  3k  9 ។
① Find the range of k such that the k
parabola and the x -axis intersect at ។
two distinct points.
② Show the range of k in ① on the k ។
number line.

177) On the x - y plane, find the equation of the ១៧៧) x-y


circle that has its center at the origin and
passes through point (5, 12).
(5, 12) ។

178) Solve for x in the following equation. ១៧៨)


4 x3  8x2  11x  3  0
4 x3  8x2  11x  3  0

179) Find the range of  that satisfies the ១៧៩) 


1 1
inequality cos   , where 0o    360o . cos   0o    360o ។
2 2

180) Given the two points A(-3) and B(3) on a ១៨០) A(-3) B(3)
number line. ។
a) Find the coordinate of point P which
divides internally the line segment AB P AB
such that the ratio of AP to PB is 1:2. AP:PB=1:2 ។
b) Find the coordinate of point Q which
Q AB
divides externally the line segment AB
such that the ratio of AQ to QB is 1:2. AQ:QB=1:2 ។

271
181) How many pages are there in x percent of ១៨១) x
a 120-page book?
120 ។

182) Express y as a function of x , given that ១៨២) y x y


y is directly proportional to x and y  8 x y  8
when x  2 .
x  2 ។

183) Find the value of y when x  2 , given ១៨៣) y x2 y


that y is inversely proportional to x and x y2
y  2 when x  4 .
x  4។

184) Square tiles with 30 cm sides are arranged ១៨៤) 30 cm


without spaces on the floor of Robert’s
class.
a) What is the area of one tile in cm 2 ? Robert ។
Include units in your answer. cm 2
b) The area of Robert’s class is 54 m 2 . ។
How many tiles are used in total.
Robert 54 m 2 ។

185) There is 20 g of sugar dissolved in a ១៨៥) 20 g 160 g។


160 gram beverage.
a) In decimal, what is the weight ratio of
sugar to the beverage? ។
b) What is the percentage of sugar in the
beverage?

272
186) In the accompanying diagram of ១៨៦) ABCD ។
quadrilateral ABCD, answer the following

with units. You may use a ruler.

D D

A C A C

B B
a) What is the area of ACD in cm 2 ? ACD cm 2
Answer this question after measuring AD
the length of sides AD and CD. Round
off your answer to one decimal place. CD ។
b) Find the area of quadrilateral ABCD ។
in cm 2 . Round off your answer to one ABCD cm 2 ។
decimal place.

187) There are 50 identical nails. The total ១៨៧) 50


weight is 80 g.
80 g ។
a) In grams, what is the weight of 100
nails? 100
b) Some nails are put in a 150 g box such ។
that the total weight is 790 g. How
many nails must be in this box?
Explain your steps leading to the 150 g ។
answer. 790 g។

188) Answer the following. ១៨៨)


a) How many m is 5 km? m 5 km?
b) How many m2 is 30 000 cm 2 ?
m 2
30 000 cm 2 ?
c) How many cm3 is 10 ?
cm3 10 ?

273
189) In parallelogram ABCD shown on the ១៨៩) ABCD
right, let the intersection of the diagonals
O
be O and the intersection points of sides
AB and CD with the straight line that AB CD
passes through point O be M and N, O M N ។
respectively. Answer the following to
BM = DN ។
prove BM = DN.

A D A D
N N
M O M O

B C B C

a) To prove BM = DN, which triangles BM = DN


should be shown to be congruent in ?
the simplest way ?
b) Explain in words the congruence
condition in your answer for a). There ។ ។
is no need to prove this.

190) Find the volume of the solids below and ១៩០) ។


answer with units. Note that (a) is a ) )
rectangular prism and (b) is a solid of
combined rectangular prism. ។
(a). )

6cm 6cm
8cm 8cm
10cm 10cm

(b). 12cm ) 12cm


6cm 6cm
3cm 3cm

10cm 10cm
3cm 3cm
6cm 6cm
6cm 6cm

274
191) The table shows the results of mathema- ១៩១)
tics examination for five students A, B, C,
A, B, C, D E។
D and E. The table shows the difference of
each student’s score to 72, which is the
average score for the five students. When 72
the score exceeds 72, it is express as a
។ 72
positive number and when it is less than
72, it is expresses as a negative number. 72

Name A B C D E
Difference to the A B C D E
average score +4 -1 -8 +5
+4 -1 -8 +5
a) Find the score for A.
b) Find the difference between B’s score A។
and C’s score as a positive number.
B C
c) Find the score for D.

D។

192) The figure shows the rectangle ABCD ១៩២) ABCD


with AB = a cm and BC = b cm. Include AB = a cm BC = b cm ។
units in your answer.

A D
A D

a cm
a cm

B C
B C
b cm
b cm
a) What is the length of the perimeter of
the rectangle ABCD in term of a and ABCD
b? a b។
b) What is the area of rectangle ABCD in
terms of a and b ? ABCD
a b។

193) Find the area, in cm 2 , of a circle with ១៩៣) cm 2


diameter d cm . Express your answer in
d cm ។
terms of d and  , the ratio of the
circumference of a circle to its diameter. d 

275
194) Isosceles triangle ABC with AB = AC is ១៩៤) ABC AB=AC
rotated clockwise about vertex A comes to
A
the extension of side BC. Let the rotated
vertices A and B be A’ and B’, BC ។ A B
respectively. Consider a special case when A’ B’ ។
three points A, B’ and A’ are on the same
A, B’ A’
straight line.

A
A
B
B

B C A'
B C A'
a) How many angles are there that are ABC ?
equal to ABC ?
b) Indicate all the angles that are equal to BAC ។
BAC . ABC ។
c) Find the size of ABC and answer
with unit.

១៩៥) 8
195) Linda bought 8 cakes consisting of some 90
90 pesos lemon cakes and some 120 pesos
fruitcakes. She paid 870 pesos for the 120
cakes. Let x and y be the number of ។ 870
lemon cakes and fruitcakes she bought ។ x y -
respectively.
a) Write simultaneous equation in terms ។
of x and y . x
b) Find the number of lemon cakes and y។
fruitcakes she bought.

១៩៦) (PGCD)
196) Find the greatest common factor (GCF) of (14, 35)
each set of numbers.
a) (14, 35) (26, 39, 52)
b) (26, 39, 52)

276
197) The diagram shows rectangular piece of ១៩៧)
paper ABCD with AB = 4 cm and
ABCD AB = 4 cm
AD = 5 cm. Point P lies on side AB and
the paper is folded along DP so that vertex AD = 5 cm។ P AB
A comes to side BC. Let A’ be the image DP A
of vertex A with DP as the reflection axis.
BC។ A’
Include units in your answer.
A DP ។
5cm ។
A D
5cm
4cm A D
P
4cm
B A' C P

B A' C
a) What is the length in cm of segment
A’C? A’C cm ?
b) What is the length in cm of segment PB cm ?
PB?

198) There is one melon and one apple. The ១៩៨) ។


melon weighs 1.05 kg and the apple 1.05 kg 0.3 kg ។
weighs 0.3 kg.
a) How many kilograms heavier is the ?
melon than the apple?
b) How many times more does the melon
weigh than the apple?

199) One year ago, the price of a digital camera ១៩៩) 8400 ។
was 8400 pesos. Today, the price is 5040
5040 ។
pesos. By what percent has the price of
the digital camera decreased?

277
200) Kevin and Linda are talking about the ២០០) -
properties of their favorite quadrilaterals.

Kevin: “I like quadrilateral in which the
four side are the same length and :
the four angles are all 90o .”
Linda: “I like quadrilateral in which the 90o ។
two opposite sides are the same
length and the opposite angles are :”
equal in measure. But the lengths
of the adjacent sides are not equal
and all angle are not 90o .
a) What kind of quadrilaterals does
Kevin like? Write your answer in the 90o ។
most appropriate name.
b) What kind of quadrilaterals does
Linda like? Write your answer in the ។
most appropriate name.

201) Lucy bought a bag of snacks. It contained ២០១) ។


15% more than the normal bag and 15%
weighed 276 g. What is the weight, in
grams, of the normal bag of snacks? 276 g ។
Include units in your answer. ។

202) Consider the integers from 50 to 150. ២០២) 50 150 ។


a) How many integers are a multiple of 6? 6?
b) How many integers are multiples of
both 6 and 8?
6 8?

278
203) There is a triangle in which the ratio of the ២០៣)
base to the height is 5:8. Answer the
5:8 ។
following questions and include units in
your answer? ។
a) Find the height, in cm, of the triangle cm
when the base is 40 cm.
40 cm ។
b) Find the base and height, in cm, of the
triangle when the area is 320 cm 2 . cm
320 cm 2 ។

1 1
204) Ellie bought a 1 liter bottle of juice. ២០៤) 1
2 2
2 ។
Yesterday, she drank liters of juice.
5 2 2
2 ។
Today, she drank of the rest of the juice. 5 5
5 ។
a) How many liters of juice did she drink
today? Include units in your answer.
b) How many times more juice did she ។
drink today than she did yesterday?
c) How many liters of juice are left?
Include units in your answer.

205) Answer the following. ២០៥)


3 3
a) How many minutes are of an hour?
4 4
b) How many mm is 8 cm? mm 8 cm ?
c) How many m2 is 400 cm 2 ?
m2 400 cm 2 ?

206) Find the values of following expression ២០៦) x  1


when x  1and y  5 . y5
a) 7 x  9 y  4 7x  9 y  4
b) 3x 2  2 xy 3x 2  2 xy

279
207) The table below contains information of ២០៧)
the heights of five mountains. The

numbers show the difference in meters
between 500 m and the actual height. 500 m ។
Positive numbers indicate heights higher 500 m។
than 500 m. Negative numbers indicate
500 m។
heights lower than 500 m.

Mountain A B C D E A B C D E
Height
(difference 99 258 -339 -106 -92
between ( 99 258 -339 -106 -92
500 m) 500 m)

a) How many meters higher is mountain E D


E than mountain D?
b) Find the difference of the height
between the highest mountain and the ។
lowest mountain. Write your answer ។ ។
as a positive number. Write the steps
leading to your answer.

208) There is a cylinder-shaped log with base ២០៨)


diameter 20 cm and height 45 cm. Answer 20 cm 45 cm ។
the following question and include units in
your answer. Use 3.14 for the ratio of the
circumference of a circle to its diameter. ។ 3.14
20cm ។


20cm

45cm
45cm


2
a) Find the total surface area, in cm , of
the log.
cm 2 ។
b) There is paint that can 2000 cm 2 per 2
liter. How many liters of this paint are 2000 cm
required to paint the whole surface? ។
You only need to apply one coat of
paint.

280
209) Mae’s father gave her x pesos for her one ២០៩) x -
3 3
day trip. She thought she would spend ។
5 5
of the money for transportation fees and
2
food and of the money for souvenirs but 2
5
she actually spent 120 pesos more for 5
souvenirs than she expected. Therefore, 120
she had 120 pesos less to spend for ។
transportation fees and food than she
120
originally planned. Answer the following
questions and include units in your
answer. ។
a) Express the actual cost for souvenirs

in terms of x .
b) Express the actual cost for transporta- x។
tion fees and food in terms of x .
c) Let’s assume that the actual cost for
the souvenirs was equal to the actual x។
cost for the transportation fees and
food. Find the value of x . Write the ។
steps leading to your answer.
x។

abc abc
210) Solve for a in the equation s  . ២១០) a s ។
3 3

211) Find the value of y when x  4 in the ២១១) y x4


linear function y  2 x  5 . y  2x  5 ។

212) Find the measure of each interior angle, in ២១២)


degrees, of a regular decagon. A decagon

is a 10-sided polygon.
10 ។

281
213) In the diagram, find x when m. ២១៣) x m។

58o 58o

m m
32o 32o
x x

214) The table on the right shows the ២១៤)


ingredients needed to make chocolate chip

cookies.
<Chocolate chip cookies> < >
Ingredient Amount
Butter 120 g 120 g
Sugar 50 g
Egg 1 50 g
Cake flour 150 g 1
Chocolate chips 60 g 150 g
a) Express the weight ratio of the sugar 60 g
to cake flour in simplest form.
b) Tracy has 40 g of chocolate chips in
the kitchen. If she uses all the
chocolate chips to make chocolate ។
chip cookies, how much butter in g
does she need? Include units in your 40 g
answer. ។

215) Two different number are selected from ២១៤) 1, 2, 3, 4


1, 2, 3, 4 and 5 to make two-digit integers.
5 ។
a) How many two-digit integers can be
created when 1 is in the tens place?
b) How many two-digit integers can be 1 ។
created in total?

282
216) The figure on the right has the axis of ២១៦) ។
symmetry .

A G A G
H H

C E C E
B D F B D F

a) Find the vertex that is symmetric to A


vertex A with respect to line . ។
b) Find the side that is symmetric to side
EF with respect to line . EF

217) Shelly bought three sheets of drawing ២១៧)


paper and one color pen set. The total cost ។ 340 ។
was 340 pesos. One sheet of drawing
paper cost x pesos and one color pen set x
cost 100 pesos. 100 ។
a) Write the total cost in terms of x . x។
b) Find the price, in pesos, of one sheet
of drawing paper. Include units in
your answer. ។ ។

218) The figure shows square ABCD with a ២១៨) ABCD 8 cm ។


side 8 cm. A semicircle with a diameter of AB
AB is in the square. Include units in your
answer. Use  for the ratio of the circum- ។ ។
ference of a circle to its diameter. 
A D ។
A D
8 cm
8 cm
B C

B C
a) Find the perimeter, in cm, of the
shaded region. cm ។
b) Find the area, in cm 2 , of the shaded cm2 ។
region. Write the steps leading to your ។
answer.

283
219) The figure shown on the right is rhombus ២១៩)
ABCD with acute angle A . Points E and
ABCD A ។
F are located such that CE = CF. Let G be
the intersection point between BF and DE. F F CE = CF ។ G
BF = DE will be proved by using congr- BF DE ។ BF = DE
uent triangle in the simplest way.
A ។

B G D
E F
B G D
E F
C
a) Which two triangles should be
shown to be congruent? C
b) Explain in words the condition for
congruent triangles in your answer
for a).

c) Find EGF when ABC  117o
and CBF  46o . Include units in EGF ABC  117o
your answer. CBF  46o ។ ។

220) The figure on the right shows right-angled ២២០) ABC


triangle ABC with AB = 8 cm, BC = 6 cm
AB = 8 cm, BC = 6 cm B  90o ។
and B  90o . Point P moves on side AB.
P AB ។ x cm
Let x cm be the length of AP and let y
AP y cm 2
PBC ។
cm 2 be the area of PBC .

A A
x cm x cm
8 cm P 8 cm P

B C B C
6 cm 6 cm

a) Find the length, in cm, of PB in terms cm PB x។


of x . Include units in your answer.

b) Express y in terms of x .
y x។
c) When the area of PBC is a cm 2 ,
express x in terms of a . Write the PBC a cm 2 ។
steps leading to your answer. x a។

284
221) In the diagram on the right, the straight b
២២១) y  ax y
line y  ax and the retangular hyperbola x
b
y  intersect at point A(4, 3). Point B A(4, 3) ។ B
x x -2 ។
lies on the rectangular hyperbola and its
x -coordinate is -2. The straight line O B ។
passing through points O and B is drawn. y
b y  ax
y y
y  ax x A
b 
y
x A x
 O
x
O B

B

a។
a) Find the value of a . b។
b) Find the value of b . O B។
c) Find the equation of the line that
passes through points O and B.

222) The cone shown on the right has base ២២២) 6 cm


radius 6 cm and height 12 cm. Answer the 12 cm។
following questions and include units in
your answer. Express the answer in terms ។
of  which is the ratio of circumference 
of a circle to its diameter. ។

12 cm 12 cm

6 cm 6 cm

a) Find the area, in cm 2 , of the base.


cm 2 ។
b) Find the volume, in cm3 , of the cone.
cm3 ។

285
223) Frank went to a store to buy foods for a ២២៣)
party. One rice meal set and one burger

set cost 220 pesos altogether. When he
paid 3000 pesos for 10 rice meal sets 15 220
burger sets, the change was 200 pesos. Let ។ 3000
x and y pesos be the costs of one rice
10 15
meal set and that of one burger set,
respectively. 200 ។ x y
a) Write the system of equations in terms
of x and y .

b) Find the cost of one rice meal set and
the cost of one burger set. Include x y។
units in your answer. ។

224) The figure on the right shows a right- ២២៤) ABC


angled triangle ABC with BAC  90o BAC  90 o
ABC  60o ។ AH
and ABC  60o . AH is perpendicular to
BC BH = 2 cm ។
side BC and BH = 2 cm. Answer the
following questions and include units in
your answer. ។
A A

60o 60o
B C B C
2 cm H 2 cm H

a) Find the length, in cm, of line segment


cm AH ។
AH. Write the steps leading to your
answer. ។
b) Find the length, in cm, of line segment cm CH ។
CH.
cm 2
ABC ។
c) Find the area, in cm 2 , of ABC .

225) Find the volume, in cm3 , of a cube with ២២៥) cm3


edges of length 10 cm. 10 cm ។

286
226) Find the value of the following expression ២២៦) x 5
when x  5 and y  2 .
y  2 ។
a) 2 x  3 y
2x  3 y
b) xy  2 y 2
xy  2 y 2

227) y is inversely proportional to x and ២២៧) y x y  2


y  2 when x  6 . Find the value of x  6 ។ y
y when x  3 . x 3 ។

228) Solve for x in the following equations. ២២៨) x


a) 3x 13  11x  3 3x 13  11x  3
4 x  5 3x  2
b)  4 x  5 3x  2
3 4 
3 4

229) Find the greatest common (GCF) of each ២២៩) (GCD)


set of numbers. (14, 21)
a) (14, 21)
b) (10, 15, 20) (10, 15, 20)

230) For triangle ABC, the measures of the ២៣០) ABC A , B


three angle A , B and C are x o , 2 x o C o
x , 2x o
3x o ។
and 3xo , respectively. x។
a) Find the value of x ?
b) What kind of triangle ABC ? Write the ABC
most appropriate name. ។

287
231) Kevin saw an advertisement for an ២៣១)
apartment with the following information
written on it:
“15 minutes walk from station A.” 15 A
The distance from station A to the apart- A 1200 m ។
ment is 1200 m.

a) Find the average speed, in meters per
minute, of the “walk” written on the ។
advertisement. 20
b) It took 20 minutes for Kevin to walk
A ។
form station A to the apartment. Find
Kevin’s average speed, in meters per ។
minute.

232) When computing 3.14 × 25, some people ២៣២) 3.14 × 25


do it using the way shown in figure 1.
1។ 2
Figure 2 displays the detailed version of
Figure 1 which shows the decimal 1
position. Only look at figure 2 and answer ។ 2
the following questions.

3.14 3.14 3.14 3.14


 25  25  25  25
1570 15.70 1570 15.70
628 62.80 628 62.80
78.50 78.50 78.50 78.50
Figure 1 Figure 2 rbUTI 1 rbUTI 2
a) What number is multiplied by 3.14 to 3.14 15.70
give 15.70 in ①? ①?
b) What number is multiplied by 3.14 to
3.14 62.80
give 62.80 in ②?
c) Express 3.14×25 as a single expression ②?
in terms of 3.14 and the two numbers 3.14×25
from questions a) and b).
3.14

288
233) Find area, in cm 2 , of each the following ២៣៣) cm 2 ។
figures. Include units in your answer. ។
a) Triangle

3 cm 3 cm

6 cm 6 cm

b) Parallelogram

4 cm 4 cm
5 cm 5 cm

4 cm 4 cm

234) Consider the integers form 1 to a given ២៣៤) 1


integer added together like such,
1  2, 1  2  3 , 1  2  3  4
1  2, 1  2  3 , 1  2  3  4
Let’s take the ratio of their sums. For
example, the ratio of 1  2 to 1  2  3 is ។
3: 6  1: 2 . 1 2 1 2  3 3: 6  1: 2 ។
a) Express the ratio of 1  2  3 to
1 2  3
1  2  3  4 in simplest form.
b) The ratio of 1  2  3  4  5  6 to a 1 2  3  4 ។
certain sum is 3: 4 . Find the sum and 1 2  3  4  5  6
write it in addition form.
3: 4 ។

235) The figure on the right has axes of ២៣៥)


symmetry and a point of symmetry. ។

a) Construct the point of symmetry using


only a ruler. Label the point O.
។ O។
b) Find the total number of axes of
symmetry. ។

289
236) Charlie collected 400 plastic bottle caps ២៣៦) 400
that weighed 1000 g in total. A recycle
1000 g។
company buys 400 plastic bottle caps for
$1.00. Another company sends polio 400 $1.00 ។
vaccines to countries in need. Vaccine for -
one person is sent for every $2.00

donation from the recycle company.
Answe the following questions. $2.00
a) How many plastic bottle caps does ។
Chalie need to collect to send a polio
vaccine for one person?
b) Suppose each plastic bottle cap has the
same weight. Find the weight, in gram, ។
of one plastic bottle cap. Include units
in your answer. ។
c) When burning 400 plastic bottle caps, ។
3150 g of carbon dioxide ( CO 2 ) are ។
emitted. How many times greater is
400
the amount of CO 2 emitted when
( CO 2 ) 3150 g ។ CO 2
burning 400 plastic bottle caps than
the weight of 400 plastic bottle caps?
400
400 ។

237) The product of integers from 1 to n is ២៣៧) 1 n n !។


express as n ! . For example, 4!  1 2  3 4  24
4!  1 2  3 4  24 and
5!  1 2  3 4  5  120
5!  1 2  3 4  5  120 .
a) Calculate 6! 6! ។
b) Calculate  6! 5!   6! 4!   6! 3! .  6! 5!   6! 4!   6! 3! ។

238) Brian wants to make a formula to find the ២៣៨)


area of a circle with circumference a cm a cm a។
in terms of a . Answer the following

questions and use  for the ratio of the
circumference of a circle to its diameter. ។
a) Find the radius, in cm, of the circle. cm ។
Include units in your answer.

b) Let S cm 2 be the area of the circle.

2
Express S in terms of  and a . Write S cm
the steps leading to your answer. S  a។

290
239) Answer the following. ២៣៩)
a) How many hours is 20 minutes?
20
b) How many g is 0.3 kg?
c) How many m2 is 20000 cm2 ? g 0.3 kg?
m 2
20000 cm 2 ?

240) Solve the following systems of equations. ២៤០)


3x  2 y  11 3x  2 y  11
a)  
 x  y  5  x  y  5
 y  6x  3  y  6x  3
b)  
 y  4x  7  y  4x  7

241) Solve for x in the following equations. ២៤១) x


a) 2 x  4  x  6 2x  4  x  6
b) 0.4x  5  0.9 x  5
2 x  1 3x  2 0.4x  5  0.9 x  5
c)  2 x  1 3x  2
3 4 
3 4

242) Find the measure of each exterior angle of ២៤២)


a regular 18-sided polygon in degrees. 18 ។

243) In the diagram below, find x when m. ២៤៣) x m។

52o 52o
x x

43o 43o
m m

291
244) Patrick and Kim play a game using a ២៤៤)
spinner labeled from A to D as shown on
A D
the right. Both players start the game with
10 points. A player spins the spinner and ។
computes their score according to the 10 ។
following rules. When spinning the spinner
more than once, each operation is done
after computing the previous score. ។

A: Add 3 ។
B: Subtract 3
 A: 3
C: Multiply by 2
D: Multiply by -2 B: 3 
C: 2
D: -2
a) When Patrick spun the spinner, it
landed on B. Find Patrick’s score.
b) When Kim spun the spinner two B។ ។
times, she got the letters A and D in
this order. Find Kim’s score.
c) When spinning the spinner two times, A D ។
find the lowest possible score. ។

245) For parallelogram ABCD shown in the ២៤៥) ABCD


figure, diagonals AC and BD intersect at O.
AC BD O។
A D A D
O O
79 o 79 o

31o 31o
B C B C

a) AOB and COD are a pair of angles AOB COD


with a certain name. What is this pair of ។
angles called?
b) ABO and CDO are a pair of angles ABO CDO
with a certain name. What is this pair of ។
angles called? CAD DBC  31o
c) Find CAD when DBC  31o and
DOC  79o ។
DOC  79o . Include units in your
answer. ។

292
246) The product of integers from 1 up to n is ២៤៦) 1 n
expressed as n ! . For example,
n !។
3!  1 2  3  6 and 4!  1 2  3 4  24 .
a) Compute 5!. 3!  1 2  3  6 4!  1 2  3 4  24
b) Find the value of integer n satisfying 5!។
the following equality. n
 n! 5! n! 4!   n! 3!  2010
 n! 5! n! 4!   n! 3!  2010

247) Linda does not know why the addition ២៤៧)


method is true when solving systems of

equations. Explain the reason why
a  c  b  d is satisfied when a  b and ac bd
cd. ab cd។
ab ab
) c  d ) c  d
ac bd ac bd

248) For triangle ABC, the measures of the ២៤៨) ABC A, B
three angles A, B and C are C o
x , 2x o
3x o

x o , 2 x o and 3xo , respectively. Find the
x ។
value of x .

249) For quadrilateral ABCD, the measures of ២៤៩) ABCD A, B,
the four angles A, B, C and D are C D y , 2 y , 3 yo
o o

y o , 2 y o , 3 y o and 4 y o , respectively. Find


4 yo ។ y ។
the value of y .

250) For pentagon ABCDE, the measures of ២៥០) ABCDE


the five exterior angles A, B, C, D A, B, C, D E
and E are z o , 2 z o , 3z o , 4 z o and 5z o , z o , 2 z o , 3z o , 4 z o 5z o ។
respectively. Find the value of z .
z ។

293
251) For parallelogram ABCD shown in the ២៥១)
figure, diagonals AC and BD intersect at O.
AC BD O។
A D A D
O O

B C B C

a) AOB and COD are a pair of angles AOB COD


with a certain name. What is this pair ។
of angles called?
b) ABO and CDO are a pair of angles ABO CDO
with a certain name. What is this pair ។
of angles called? DAB  ABC ។
c) Find the measure of DAB  ABC
in degrees. Include units in your ។
answer.

252) A triangle has base 6a cm and height 5 cm. ២៥២) 6a cm 5 cm។


Express the area, in cm2, using a . cm 2
a ។

253) Kevin weighs 52.5 kg. ២៥៣) 52.5 kg.


a) Kevin’s brother weighs 1.2 times more
1.2
than Kevin. Find Kevin’s brother’s
weight, in kg. Include units in your ។ kg។
answer. ។
b) Kevin’s sister weighs 42 kg. How
42 kg ។
many times heavier is Kevin’s sister
than Kevin?

254) There are 80 students in Jim’s grade. ២៥៤) 80 ។


a) 36 students in Jim’s grade are girls.
36 ។
What percent of students in Jim’s
grade are girls? ។
b) In Jim’s grade, 15% of students belong , 15%
to a soccer club. How many students

belong to the soccer club?

294
255) In the following figures, find angles X and ២៥៥) X Y
Y by calculating. Include units in your
។ ។
answer.
a) Quadrilateral

72o 72o
X X

116o 116o

b) Line segments AD and BC cross at E. AD BC E។

A C A C

74o E Y 74o E Y

43o 43o
35o 35o
D D
B B

256) Find the volume, in cm3, of each of the ២៥៦) cm3 ។


following figures. Include units in your

answer.
a) Cube

8 cm 8 cm

b) Solid made up of rectangular prisms.

10 cm 10 cm

2 cm 2 cm
6 cm 6 cm
5 cm 5 cm

9 cm 9 cm

295
257) There are several identical nails in a box. ២៥៧)
50 nails weigh 140 g.
។ 50 140 g។
a) Find the weight, in g, of 200 nails.
Include units in your answer. g 200 ។
b) The total weight of all nails in the box ។
is 2800 g. How many nails are in the
2800g។
box? Write the steps leading to your
answer.

258) Charlie gives candies to his friends. When ២៥៨) ។


he gives 8 candies to each of them, 11 8
candies are left over. Let x be the number
of friends he has. 11 ។ x ។
a) Express the total number of candies x ។
using x . 8
b) When he gives 8 candies to each of his
friends, 11 candies are left over. When 11 ។
he gives 10 candies to each of his 10 5 ។
friends, he is 5 candies short. Write
x ។
the equation using x .
c) Find the number of friends he has. ។

259) In the diagram on the right, each number ២៥៩) 1 16


from 1 to 16 is assigned to each of the 16 16 ។
squares so that the sum of the 4 number in
any row, column or diagonal is the same. 4
(This kind of square is called a magic ។
square) Seven numbers are already
)
assigned.
a 14 15 b ។
a 14 15 b
12 c 6 d
g 12 c 6 d
e f 10
e f 10 g
h i 3 16
h i 3 16
a) Find the sum of four numbers in a
row, column or diagonal.
b) Find the numbers for a and h . ។
a h។

296
260) In the diagram on the right, point C lies on ២៦០) C AB
segment AB such that AB = 12 cm and
AB = 12 cm AC = 8 cm ។
AC = 8 cm. There are three semicircles
whose diameters are segments AB, AC AB, AC BC
and BC, respectively. Include units in ។
your answer and use  for the ratio of the

circumference of a circle to its diameter.

12 cm 12 cm
A B A B
8 cm C 8 cm C

a) Find the area, in cm2, of the semicircle cm2


of diameter 12 cm. 12 cm ។
b) Find the area, in cm2, of the shaded
part. Write the steps leading to your cm2 ។
answer. ។

261) In the diagram below, find x when ២៦១) x m។


m.

63o 63o

x x
o
38 38o
m m

262) There are two fish tanks, A and B. Tank ២៦២) A B។ A


A can hold 27 L of water and tank B can
27 L B 36 L
hold 36 L of water.
a) Write the ratio of the volume of tank ។
A to the volume of tank B in simplest A
form.
B ។
b) 35 fish are divided into the two tanks
such that the ratio of the number of 35
fish in tank A to tank B is equal to the A
ratio of their volumes. Find the
B ។
number of fish in tank A and in tank B.
A B។

297
263) Becky walks at a speed of 80 m per ២៦៣) 80 m ។
minute. Answer the following and include
units in your answer.
a) It takes 8 minutes for Becky to go to ។
school from her home. Find the 8
distance, in m, to school from her
។ m
home.
b) The distance to a station form her ។
home is 2 km. How long, in minutes, 2 km។
does it take for Becky to go to the
-
station from her home?

264) The figure on the right has axis of ២៦៤) ។


symmetry .

A A

B I B I
C H C H

D G D G
E F E F

a) Find the vertex that is symmetric to C ។


vertex C with respect to . GF
b) Find the side that is symmetric to side

GF with respect to .

265) 17 added to a number is equal to 15 ២៦៥) 17 15


subtracted from that number then
5។ x
multiplied by 5. Let that number be x and
answer the following.
a) Write the equation in terms of x . x ។
b) Find the value of x .
x ។

298
266) In the diagram on the right, There is a ២៦៦) 8 cm
semicircle of diameter 8 cm in a sector
8 cm
whose radius is 8 cm and its central angle
is 90o . Include units in your answer and 90o ។
use  for the ratio of the circumference of 
a circle to its diameter. ។

8 cm 8 cm

a) Find the area, in cm2, of the sector cm2


whose radius is 8 cm and its central
8 cm 90o ។
angle is 90o .
b) Find the area, in cm2, of the shaded cm2 ។
part. Write the steps leading to your ។
answer.

267) In the diagram on the right, the straight ២៦៧) y  2x  6


line y  2 x  6 , denoted by , passes
y x A B ។
through the y -axis and x -axis at A and
C (2, 1) ។ m
B, respectively. The coordinates of point
C is (2, 1). Let m be the straight line that A C។
passes through points A and C.
y y
m m
 A  A

 C  C
B B
O x O x

a) Find the coordinates of point B. B។


b) Find the equation of straight line m . m ។
Write the steps leading to your answer.

299
268) The table below contains information on ២៦៨)
the highest and lowest temperatures for
២០
each day from July 20th to July 24th in
Osaka city. The numbers show the ២៤
difference in temperature between 30 o C ។
and the actual temperature. Positive 30 oC ។
numbers indicate temperatures higher than
30 oC
30 oC . Negative numbers indicate
temperatures lower than 30 oC .
30 oC ។
July 20th 21st 22nd 23rd 24th ២០ ២១ ២២ ២៣ ២៤
Highest +3.9 +4.2 +5.3 +6.0 +5.6 +3.9 +4.2 +5.3 +6.0 +5.6
Lowest -3.6 -3.8 -3.8 -2.5 -2.7
-3.6 -3.8 -3.8 -2.5 -2.7
a) Find the actual lowest temperature on
the 20th. Include units in your answer. ២០។
b) Which day has the greatest difference ។
between the highest and lowest
temperatures?

269) The diagram on the right shows a


rectangular prism of length 5 cm, width ២៦៩)
5 cm and height a cm. Include units in 5 cm 5 cm a cm ។
your answer.

5 cm 5 cm

5 cm 5 cm
a cm a cm

a) Find the volume, in cm3, of the


rectangular prism. cm3 ។
b) Find the surface area, in cm2, of the cm 2

rectangular prism.

300
270) There are two numbers, x and y . The ២៧០) x y ។ x y
sum of x and y is 30. The sum of 7 times 30 ។ 7 x 2 y 95 ។
x and 2 times y is 95.
x y។
a) Write a system of equation using x
and y . x y។
b) Find the value of x and y .

271) In the graph shown on the right, the ២៧១) y  ax 2


parabola y  ax 2 has its vertex at O and O A(1, 2) ។
passes through point A(1, 2). Point B lies B y  ax 2
on y  ax 2 and its x -coordinate is 2.
2។ H
Point H lies on the y -axis and line BH is
perpendicular to the y-axis. Point C lies on BH ។ C
line BH such that HB = BC. The parabola BH HB = BC ។
y  bx 2 has its vertex at O and passes y  bx 2
O
through point C.
C។
y y

y  ax 2 y  bx 2 y  ax 2 y  bx 2
     
H B C H B C

A A
 

x x
O O

a) Find the value of a . a ។


b) Find the y -coordinate of point B.
B។
c) Find the value of b . Write the steps
leading to your answer. b ។

301
272) The figure on the right shows trapezoid ២៧២)
ABCD with AD || BC and AD  DC .
AD || BC AD  DC ។ H
Point H lies on side BC and line AH is
perpendicular to side BC. Answer the BC AH BC ។
following when AB = 6 cm, BC = 7 cm AB = 6
and AD = 5 cm.
cm, BC = 7 cm AD = 5 cm ។
5 cm
A D 5 cm
A D
6 cm
6 cm

B H C
7 cm B H C
7 cm
a) Find the length, in cm, of AH.
b) Find the area, in cm2, of trapezoid cm AH ។
ABCD. 2
cm
ABCD ។

273) Consider the following six integers. ២៧៣)


6,  5,  1, 2, 4, 5
6,  5,  1, 2, 4, 5
a) If you choose two integers and add
them, find the least possible sum of
the two integers. ។
b) If you choose two integers and
multiply them, find the least possible
product of the two integers. ។
c) Three difference integers are assigned
to each of the letters, A, B and C in the A, B C
expression below. Find the set of
integers, A, B and C such that the ។ A, B C
expression takes the minimum value. ។
A
C A
C
B B

1 2 1 2
274) Consider the function y  x . ២៧៤) y x ។
2 2
a) Find the value of x when y  4 . x y  4។
b) For the problem “Find the range of y y 2  x  4 ”
in the interval 2  x  4 ”
“2 y 8” ។
Nick’s answer was “ 2  y  8 ”. Is his
answer correct? If it is correct, write ”
“Correct” on your answer sheet. If it is ។ ។
not correct, give your reason.

302
275) Find the volume, in cm3, of each of the ២៧៥) cm3 ។
following solids. Include units in your

answer and use  for the ratio of the
circumference of a circle to its diameter. ។
a) Cylinder

2r cm 2r cm
r cm r cm
 

b) Cone

2r cm 2r cm

r cm r cm

 

276) 2 students are selected form 4 students, A, ២៧៦) 2 4 A,


B, C and D, at random. B, C D ។
a) How many different ways can 2
students be selected?
b) What is the probability that student A 2 ។
will be selected? A
c) After selecting 2 students, a president
and a vice president will be selected
from the 2 students at random. What is 2
the probability that student A will be
president?
2 ។ A

277) Circles are arranged according to a rule as ២៧៧)


shown below.
1st 2nd 3rd 4th …
១ ២ ៣ ៤

...

Find the number of circles in the 8th


៨ ។
arrangement.

303
278) The table on the right shows the number ២៧៨)
of doctors per 1000 people for the top four
1000 ។
countries. Japan is added to the table for
comparison.

Number of doctor
Country
per 1000 people
Cuba 6.4 1000
Greece 5.4 6.4
Belarus 4.9
Russia 4.3 5.4
Japan 2.1 4.9
4.3
a) How many times more is the number of
doctors per 1000 people in Cuba than 2.1
in Russia? Round your answer off to
1000
one decimal place.
b) To find the number of doctor per 1000
people in a country, we need two ។
values, “the total number of doctors” 1000
and “population” of the country.
Let x be the total number of doctors ” ” ។
and let y be the population of a x y
country. Express the number of doctors ។
per 1000 people using x and y .
1000
c) Suppose that the population of Japan is
x y ។
130,000,000. Find the total number of
doctors in Japan.
130,000,000 ។

279) The figure on the right shows two right- ២៧៩) ABC
angled triangles ABC and CDE, where
CDE E
point E is the midpoint of side AC. AB = 2
cm, BC = 4 cm and DE = 5 cm. Include AC ។ AB = 2 cm, BC = 4 cm DE = 5
units in your answer. cm ។ ។
D D
5 cm 5 cm
A A
E E
2 cm 2 cm

B C B C
4 cm 4 cm

a) Find the length, in cm, of side AC. cm AC ។


b) Find the length, in cm, of side CD. cm CD ។
c) Find the sum of the areas, in cm2, of
cm2 ABC
ABC and CDE .
CDE ។

304
280) The diagram on the right shows equilateral ២៨០) រូបខាងស្តាំបង្ហាញពីររីកោណសម័ងស ABC ។
triangle ABC. Points D and E lie on side
ចំណុច D នង
ិ E កៅកលើរជុង AB នង
ិ AC ករៀងគ្នា
AB and AC, respectively such that
AD = CE. ACD  CBE will be proven ដែល AD = CE ។ ACD  CBE នឹងររូវ
in the simplest way using congruent បង្ហាញកោយោរករបើលក្ខខណឌប៉ន
ុ គ្នាននររីកោណ។
triangles.
A A

D D
E E

B C B C

1) Which two triangles should be shown ១) ករគ


ើ ូររកី ោណណា ដែលគួបង្ហាញថាវាប៉ន
ុ គ្នា?
to be congruent? ២) ករល
ើ ក្ខខណឌណា ដែលររូវយក្មក្បញ្ជាក្់ចំកោះ
2) Which conditions are required to prove
that the two triangles in your answer ររីកោណទាំងពីរ ដែលថាវាប៉ន
ុ គ្នាក្ាុងចកមលយ
ើ របស់
for 1) are congruent? Choose three អ្ាក្សរាប់សំណួរទី ១)។ ចូរករជស
ើ ករសលក្ខ
ើ ខណឌ
conditions from the following and បនី នបណា
ា លក្ខខណឌខាងករោម ក យ
ើ សរកសរ
write the corresponding letters.
(a) AD = CE អ្ក្សរដែលររូវគ្នានឹងលក្ខខណឌកោះ។
(b) AC = CB ក្. AD = CE
(c) CD = BE
ខ. AC = CB
(d) ADC  CEB
(e) ADC  CEB គ. CD = BE
(f) CAD  BCE ឃ. ADC  CEB
3) Explain in words the condition for
ង. ACD  CBE
congruent triangles in your answer for
1). ច. CAD  BCE
៣) ពនយល់ជាោក្យ នូវលក្ខខណឌប៉ន
ុ គ្នាននររីកោណ
ក្ាុងចកមលយ
ើ របស់អ្ាក្ សរាប់សំណួរទី ១)។

281) In the figure on the right, a square with ២៨១) ក្ាុងរូបខាងស្តាំ ោករមួយានរង្ហវស់រជុងរបដវង 4 cm
sides of length 4 cm is equally divided គរឺ រូវដចក្កចញជាោកររូច 4  4 កសមើៗគ្នា ដែលរជុង
into 4  4 small squares, each with a side
of length 1 cm. ានរង្ហវស់របដវង 1 cm ។

In this figure, construct a line segment of ក្ាុងរូបកនះ ចូរែឹក្បោារ់ដែលានរបដវង 5 cm


length 5 cm using only a ruler. You can កោយោរករបរើ ាស់ដរបោារ់បកុ៉ ណា
ោ ះ។ អ្ាក្អាចករបើ
only use the ruler for drawing lines.
ានដរបោារ់បកុ៉ ណា
ោ ះកែើមបគ
ី ូសបោារ់។

305
282) In the accompanying figures, each ២៨២) ក្ាុងរូបនម
ី យ
ួ ៗបង្ហាញពអ្ ំ ស
ី ំពូល៥ ដែលជន ួ ឱ្យកលខ។
shows 5 bulbs to represent a number.
អ្ំពូលអ្គគស
ិ នន
ី ម
ី យ
ួ ៗអាចកឆះភលឺ ឬមន
ិ កឆះភល។
ឺ រូបទ១

Each electric bulb can be either on or
off. Figure 1 shows all electric bulbs are បង្ហាញពីអ្ំពូលអ្គគិសនីទាង ិ កឆះដែលជំនស
ំ អ្ស់មន ួ ឱ្យ
off and it represents 0. When the electric កលខ 0 ។ កៅកពលដែលអ្ំពូលអ្គគិសនីកឆះភលឺ ែូចក្ាុង
bulbs are on as in Figure 2 to Figure 6,
they represent 1, 2, 4, 8 and 16, ី ែល់ទី៦ ដែលពួក្វាជំនស
រូបទ២ ួ ឱ្យកលខ 1, 2, 4, 8
respectively. Some othe numbers can និង 16 ករៀងគ្នា។ កលខកផសងកទៀរអាចររូវានជំនស

also be represented by a combination of កោយោរផសំចូលគ្នាននអ្ំពូលទាំង៥កនះ។ ជាឧទា រណ៍
these 5 bulbs. For example, Figure 7 is a
រូបទី ៧ ជាបនសំននរូបទី 2, 5 នង
ិ 6។ ែូចកនះរូបទី ៧
combination of Figure 2, 5 and 6. Thus
Figure 7 represents 1 + 8 + 16 = 25. ជំនស
ួ ឱ្យកលខ 1 + 8 + 16 = 25 ។
Answer the following. ចូរកឆលើយនឹងសំ ណួរខាងករោម។
Figure 1 រូបទី 1
Figure 2 រូបទី 2
Figure 3 រូបទី 3
Figure 4 រូបទី 4
Figure 5 រូបទី 5
Figure 6 រូបទី 6
Figure 7 រូបទី 7
a) What number is represented when all ក្. ករកើ ៅកពលអ្ំពូលទាំងអ្ស់កឆះភលឺ វាជំនស
ួ ឱ្យកលខអ្វ?

bulbs are on? ខ. រូបននអ្ំពូលទាំងកនះជំនស
ួ ឱ្យកលខ 10។ ចូរគូសរូប
b) Configure these bulbs to represent 10.
Draw your figure using and just របស់អ្ក្
ា កោយករបរើ ាស់ និង ែូចរូបខាងកលើ។
like the figure above.
283) In the rectangle shown on the right, points ២៨៣) ក្ាុងចរុកោណដក្ងបង្ហាញក្ាុងរូបខាងស្តាំ, ចំណុច M
M and N are the midpoints of sides AD និង N គឺជាចំណុចក្ណា
ា លករៀងគ្នាននរជុង AD និង
and BC, respectively. BD is the diagonal.
MBD  NDB will be proved in the BC ។ BD គជា
ឺ អ្ងកររ់ ទូង។ MBD  NDB
simplest way by using congruent triangles. នឹងររូវ បង្ហាញកោយោរករបើលក្ខខណឌប៉ន
ុ គ្នានន ររី-
A M D កោណ។
A M D

B N C
B N C
1) Which two triangles should be shown ១) ករើគូររីកោណណា ដែលគួបង្ហាញថាវាប៉ន
ុ គ្នា?
to be congruent? ២) ករើលក្ខខណឌណា ដែលររូវយក្មក្បញ្ជាក្់ចំកោះ
2) Which conditions are required to ររីកោណទាំងពីរ ដែលថាវាប៉ន
ុ គ្នាក្ាុងចកមលយ
ើ របស់
prove congruency of the two triangles
អ្ាក្សរាប់សំណួរទី ១)។ ចូរករជស
ើ ករសលក្ខ
ើ ខណឌ
in your answer for 1)? Choose three
conditions from the following and បនី នបណា
ា លក្ខខណឌខាងករោម ក យ
ើ សរកសរ
write the corresponding letters. អ្ក្សរដែលររូវគ្នា។
a) AM  CN b) DM  BN ក្. AM  CN ខ. DM  BN
c) BD  DB d) MBD  NDB គ. BD  DB ឃ. MBD  NDB
e) A  C f) MDB  NBD ង. A  C ច. MDB  NBD
3) Explain in words the condition for con- ៣) ពនយល់ជាោក្យ នូវលក្ខខណឌប៉ន
ុ គ្នាននររកី ោណ
gruent triangles in your answer for 1).
ក្ាុងចកមលយ
ើ របស់អ្ាក្ សរាប់សំណួរទី ១)។

306
284) The diagram shows rectangle ABCD. Let ២៨៤) រូបបង្ហាញពីចរុកោណដក្ង ABCD ។ កគឱ្យ M ជា
M be the midpoint of side AB. When
ចំណុចក្ណា
ា លននរជុង AB ។ កៅកពលដែលកយង
ើ ភ្ជាប់
connecting point M to vertices C and D,
AMD  BMC will be proven in the ចំណុច M កៅក្ំពូល C និង D, AMD  BMC
simplest way using congruent triangles. នឹងររូវបង្ហាញកោយោរករបើលក្ខខណឌប៉ន
ុ គ្នាននររីកោណ

A D A D

M M

B C B C

1) Which two triangles should be shown to ១) ករើគូររីកោណណា ដែលគួបង្ហាញថាវាប៉ន


ុ គ្នា?
be congruent? ២) ករើលក្ខខណឌណា ដែលររូវយក្មក្បញ្ជាក្់ចំកោះ
2) Which conditions are required to prove ររកី ោណទាំងពរី ដែលថាវាប៉ន
ុ គ្នាក្ាុងចកមលយ
ើ របស់
that the two triangles in your answer for
1) are congruent? Choose three conditions អ្ាក្សរាប់សំណួរទី ១)។ ចូរករជស
ើ ករសលក្ខ
ើ ខណឌ
from the following and write the បីននបណា
ា លក្ខខណឌខាងករោម ក ើយសរកសរ
corresponding letters. អ្ក្សរដែលររូវគ្នា។
a) AD  BC b) MD  MC ក្. AD  BC ខ. MD  MC
c) AM  BM d) AMD  BMC គ. AM  BM ឃ. AMD  BMC
e) MAD  MBC f) ADM  BCM ង. MAD  MBC ច. ADM  BCM
3) Explain in words the condition for con- ៣) ពនយល់ជាោក្យ នូវលក្ខខណឌប៉ន
ុ គ្នាននររកី ោណ
gruent triangles in your answer for 1).
ក្ាុងចកមលយ
ើ របស់អ្ាក្ សរាប់សំណួរទី ១)។

285) In the figure on the right, a square with ២៨៥) ក្ាុងរូបខាងស្តាំ ោករមួយានរង្ហវស់រជុងរបដវង 8 cm
sides of length 8 cm is equally divided
គឺររូវដចក្កចញជាោកររូច 8  8 កសមើៗគ្នា ដែលរជុង
into 8  8 small squares, each with a side
of length 1 cm. ានរង្ហវស់របដវង 1 cm ។

In this figure, construct a line segment of ក្ាុងរូបកនះ ចូរែឹក្បោារ់ដែលានរបដវង 10 cm


length 10 cm using only a ruler. You can កោយោរករបរើ ាស់ដរបោារ់បកុ៉ ណា
ោ ះ។ អ្ាក្អាចករបើ
only use the ruler for drawing lines. You
don’t need to write your steps leading to ានដរបោារ់បកុ៉ ណា
ោ ះកែើមបគ
ី ូសបោារ់។ អ្ាក្មន
ិ ចាំ
your answer. ាច់សរកសរជំហានននចកមលយ
ើ របស់អ្ាក្កទ។

307
286) Three marbles are picked from a jar ២៨៦) ក្ូនឃលីបីគឺររូវានករជស
ើ ករសព
ើ ីក្ុ ងរក្ឡមួ
ា យ ដែល
containing 6 white marbles and 3 red
ានផាុក្ក្ូនឃលស
ី 6 នង
ិ ក្ូនឃលរី ក្ ម 3 ។ តាង A ជា
marbles. Let A be the event “three
marbles picked are red”. រពឹរាិោរណ៍ “ក្ូនឃលីទាង
ំ បីានករជើសករសគ
ើ ឺរក្ ម”។
Three marbles are picked from a jar ក្ូនឃលីបីគឺររូវានករជើសករសព
ើ ីក្ុ ងរក្ឡមួ
ា យ ដែល
containing 7 white marbles and 3 red
ានផាុក្ក្ូនឃលស
ី 7 នង
ិ ក្ូនឃលរី ក្ ម 3 ។ តាង B ជា
marbles. Let B be the event “three marbles
picked are red”. រពឹរាិោរណ៍ “ក្ូនឃលីទាង
ំ បីានករជើសករសគ
ើ ឺរក្ ម”។
How many time more is the probability of ករើបោ
ុ៉ ម នែង ដែលរបូាបកក្ើរានកឡង
ើ ននរពឹរាិ-
occurring event A than that of event B?
ោរណ៍ A ករចន
ើ ជាងរបូាបននរពរ
ឹ ោ
ាិ រណ៍ B ?

287) Cube A has a side of length a . Cube B is ២៨៧) គូប A ានរទនុង a ។ គូប B គជា
ឺ គូបដែលាន
a cube that is obtained by extending each
កោយក្វើោរពរងីក្រទនុងនីមយ
ួ ៗននគូប A របដវង b។
side of cube A by length b .
a) Write the volume of cube B in terms ក្. ចូរសរកសរាឌននគូប B ជាអ្នុគមន៍នន a និង b។
of a and b . Write only your answer សរកសរចកមលយ
ើ របស់អ្ាក្ជាទរមង់ពោលរ។
in expanded form.
ខ. ករើបោ
ុ៉ ម នែងននាឌគូប B ករចើនជាងាឌននគូប A?
b) How many times more is the volume
of cube B than that of cube A? បញ្ជាក្់ចកមលយ
ើ របស់អ្ាក្ជាអ្នុគមន៍នន t ,
Express your answer in term of t , b
ដែល t  ។
b a
where t  .
a

288) Prove x  y  z  a when x  y  z  3a ២៨៨) បង្ហាញថា x  y  z  a កៅកពលដែល


and x 2  y 2  z 2  3a 2 , where a, x, y and x  y  z  3a នង
ិ x  y  z  3a ដែល
2 2 2 2

z are real numbers.


a, x, y និង z គឺជាចំនន
ួ ពិរ។

លំហាត់ត្តឹមលលខ ២៨៨ លេះ ខញំបា


ុ េបកប្ត្បជាភាសាជាតិ

េិងលោះត្សាយរកចលមលើយ ជូេដល់មិតអ្
ត នកអាេរួចរាល់ល ើយ។

សូមមមើលដំមោះស្រាយបន្ទាប់ពីទំព័រមេះ

មដើមបីម្ាៀងផ្ទាត់ជាមួយចមមលើយរបស់អ្នកមស្រោយពីមោះស្រាយរួច។

3
308
1) 6)
x 2
 6 x  8   x  2 x  8
2
 2 x  1   2 x  3 2 x  5
2

 x2  6 x  8  x2  2 x  8  4 x 2  4 x  1   4 x 2  10 x  6 x  15 
 4 x  16  4 x 2  4 x  1  4 x 2  4 x  15
 8 x  16
2)
3 x  2  5x  2 7)
3x  6  5 x  2
 x 1 0 ២
2

3 x  5 x  2  6
x 1  0 x 1
 2 x  8
x1  x2  1 ។
2 x 8

2 2
x4 8)

 57  5 5   10
5
3) a b
 2 a  b  5  5 5  7 5  35  2 5
  30
 ab
2
9) y
b  a
2 1 2
 2b 2  x  1 y x
a ។ 2
2 x2  0 x y0
2  x  0 0 y2
4) 1
y  ax  b 0  x 1 0 y
2
 0, 3 x  0, y  3
0 y
1
0 y2
a7 2
0 y2 ។
y  ax  b 3  7  0  b
b  3
10)
y  7x  3 ។

2 2 5 cm
5) x
2
 35  5 35 cm

l m n
2
 30
3 cm 4 cm 3 4 20 30 cm
 ឬ  x ។
5 cm x cm 5 x 3 30 cm ។

309

11) 14)
xyz  xy  xz  x
 x  yz  y  z  1 4 5
 x  y  z  1   z  1  4! 5!
C  4, 1  C  5, 1  
 x  y  1 z  1 3! 1! 4! 1!
 4  5  20

12) ២ 5x  19 x  4  0
2 20 ។

① ២
f  x   5x 19x  4
2
f  x  0 15)
125 , 25 , A ,1, B ,…
f  x   0   5x2 19x  4  0
① រក A
   19  4  5 4  361  80  441
2
U 2 25 1
q  
  19   441 19  21 U1 125 5
x1    4
2  5 10 A
1
  25  5 A 5 ។
  19   441 19  21 1
5
x2    ② រក B
2  5 10 5
1 1 1
B  1  B  ។
5 5 5
1
x  4 
5 16)
f  x  0 - 0 + 0 -
 x  y  4 x  y    2 x  y  2 x  y 
x  4 ឬ x 
1  4 x 2  xy  4 xy  y 2   4 x 2  y 2 
5
 4 x 2  3 xy  y 2  4 x 2  y 2
② x   3 xy
1
x  4 ឬ x 
5
17) 2010
) (
x 4 01 x 2010  2  3  5  67
5
13) cos A 18) x
ABC C 4 x2  5x  8  0
sin 2 A  cos2 A  1    5  4  4   8  25  128  153
2
B
cos 2 A  1  sin 2 A
  5   153 5  3 17
cos A   1  sin A 2 C A x1  
2 4 8
A cos A  0   5   153 5  3 17
3 x2  
cos A  1  sin 2 A sin A  2 4 8
5
3 25  9
2 5  3 17
cos A  1     
4 x ។
8
5 25 5

310

19) 23)
 x  6 y   x 2  6 xy  36 y 2 
 52 2  5 2 
2 5

2  x3   6 y 
3

 5  10  2 10  4  10
 x3  216 y 3
 1

24)
20) a
x8  1   x 4  1 x 4  1
y  ax 2  2 ,  8
x  2 , y  8   x 2  1 x 2  1 x 4  1

8  a   2 
2
8  4a   x  1 x  1  x 2  1 x 4  1
a  2 ។
25)

 
C 2
21) x 2 22 
E 22

4
6

2  2 2 
A B
  2  2
22
D
2  2  2
x 3

DE BC EAD CAB   2 2
24
AD ED x 4
 ឬ 
AB CB x3 6
26) A B
6x  4x  12 x6 ។
A = {1 , 3 , 7 , 8 }
B = {1 , 4 , 6 , 7 }
22) AB cm
A  B   1, 3, 4, 6, 7, 8 
A
U   1, 2, 3, 4, 5, 6, 7, 8, 9 

7cm A  B   2, 5, 9  ។

B C
3cm 27) ២ 2 x 2  3x  9  0
① ២
ABC C f  x   2x2  3x  9 f  x  0
AB  BC  AC
2 2 2
f  x   0  2x2  3x  9  0
  32  4  2   9  9  72  81
BC = 3cm , AC = 7cm
2
AB2  32  7  9  7  16cm2 3  81 3  9 3
x1   
AB   16cm  4cm 2
2  2 4 2
AB  0 AB 3  81 3  9
x1    3
2  2 4
AB  4 cm ។

311

3 33) x
x  3 
2 x 2  10 x  6  0
f  x  0 + - 0
 '   5   6   25  6  31
0 + 2

x  3 ឬ x 
3   5   31
2 x  5  31
1
② x
3 34)
x  3 ឬ x 
2

5 3 2 5   15
5
) (
x 3 0 3 x  3 5  10  3 5
2  10
28) A  3B
A  3x  2 x 2
B  x2  x 1 35) y x
A  3B  3x 2  2 x  3  x 2  x  1 y  ax 2

y  45 x 3
 3x 2  2 x  3x 2  3x  3
 5x  3 45  a  32 a5
y  5x ។
2

29)  sin   cos


sin   cos cos  36) x
tan   1 x
l
  45  180 k , k 
o o

4
0o    180o   45o ។ 1
m
2
30)
2 1
10! 10  9  8! m 
① 10 C2  C 10, 2     45 24 x
8! 2! 8! 2
10! 10  9  8! ឬ 2x  6 x 3 ។
② 10 C8  C 10,8    45
2! 8! 2  8!

37)
31)
5 cm 12 cm
 x  3 y    x  y  x  5 y 
2

 x 2  6 xy  9 y 2   x 2  5 xy  xy  y 2  2
  5cm   12cm 
2 2

 x 2  6 xy  9 y 2  x 2  6 xy  y 2  25cm2  144cm2
 8 y2  169cm2
 169cm2  13 cm
32)
4 x 2  225   2 x   152
2 13 cm ។

  2 x  15  2 x  15 

312

38)
 x  2 y   x 2  2 xy  4 y 2  x  
1
3 
5
 x  2 y
3 3

f  x  0 + 0 - 0 +
 x  8y3 3

1
  x3
39) 5
x3  6 x 2 y  12 xy 2  8 y 3 ② x
  x  2y
3
1
  x3
5
40) [ ]
x 10 x
3x  12  9 x  18  3
5
3 x  9 x  18  12
 6 x  6 43) 10 P3
6 x 6 10! 10  9  8  7!
 P  P 10,3 
10 3   720
6 6 7! 7!
x 1
44)
41) 6 2

y  ax 2  bx  c 8!
 C  8 ,3   56
 b b  4ac 
2
5! 3!
- S  , 
 2a 4a 

- x
b  C  6 , 2  C  2 ,1
2a 6! 2!
   30
4! 2! 1!1!
y  x 2  8 x  15 30 15
 
 8  8   4 115  56 28
2

 , 
 2 1 4  1 
 
45) ① sin 
 4 , 1 ។
sin 2   cos2   1
sin   1  cos2  , 
42) ① ២
5
cos  
5 x 2  14 x  3  0 7
f  x   5x2 14x  3 f  x  0 5 49  25 2 6
2

sin   1     
f  x   0  5x2 14x  3  0 7 49 7
 '   7   5   3  49  15  64
2
② tan 
  7   64 7  8 1 2 6
x1    sin  7  2 6
5 5 5 tan   
cos  5 5
  7   64 7  8
x2   3 7
5 5
313

46) 53)
x  x  2    x  1 x  3  x  y   x2  xy  y 2   x3  y 3 ។

 x  2 x   x  3 x  x  3
2 2

 x2  2 x  x2  2x  3  3 54)
x3  3x2 y  3xy 2  y3   x  y  ។
3

47)
9 x 2  y 2   3x   y 2  3x  y 3x  y 
2
55)
4
3 1

 1 3 3  3  
48) x
x2  5x  6  0 
4  3 1  3 3 3 3 3
S  5  2  3
 x2 , x3  3 1  3 1 
P  6  2  3  2 3 22 3  2

49)
56) a b
1  5 
2
 20 y   x 2  ax  b
 1 2 5  5  2 5  6 1 , 2
y  ax 2  bx  c
50) y x  b b2  4ac 
 , 
y  ax 2  2a 4a 
y 1 x2  b
 2a  1  b  2 a
1  2
1  a  22 a  2
4  b  4ac  2 b  4ac  8a
1 2  4a
y x ។
4 a  2  1 a  2
 2  2
a  4  1 b  8  1 a  4b  8
51)
a d a2 b 1 ។
d
d a 2 a
2 57)
d  4 cm 9
4 cm
a  2 2 cm 3
2 9 3
C  9,3
52) x m
9!
m C  9,3 
x 2 6! 3!
x 6 9  8  7  6!
 6   84
2 4 4 6! 3  2
x 3 m
84 ។

314

58) x
2 x2  5x  3  0 a )   8   4  10  2
f  x   2x2  5x  3 f  x  0 b)  56   8   3  10
f  x   0  2 x 2  5x  3  0 c)  1   2    2    10
4 3

 '   5  4  2   3  25  24  49


2
7 7 1 1
d )      1
  5   49 5  7 1 8 9 16 2
x1 
2 2

4

2 e) 6  2  3 2  2 
  5   49 5  7
x2 
2 2

4
3 
f ) 10 2 5  10   1
10
 5  10 
21 2
1   11
x   3  2
2 g ) 8 x  2   5 x  7   3x  5
f  x  0 + 0 - +
h) 6  0.6 x  0.4   0.3  2 x  5   3 x  0.9
0

x
1
ឬ 3 x i ) 2  3 x  4 y   5  4 x  3 y    14 x  7 y
2
3x  5 y x  5 y 13 x  35 y
j)  
59) ① cos  8 6 24
1 k ) 32 x y  4 x y  8 x  y
3 2 2

sin   0o    90o 2 2
2  3   6  3  2
sin   cos2   1
2 l )   x3 y 2     x 4    y 2   x 2
 5   5  2  3
cos  1  sin 2  , 0o    90o
1 4 1
2
3 62)
cos   1     
2 4 2 a)  4 x  3 3x  2 
② tan   12 x 2  8 x  9 x  6
1  12 x 2  x  6
sin  1 3
tan    2   b)  x  3 y  x  5 y    x  4 y 
2

cos  3 3 3
2  x 2  5 xy  3 xy  15 y 2   x 2  8 xy  16 y 2 
 x 2  8 xy  15 y 2  x 2  8 xy  16 y 2
60) ① A B   y2

A = { 2 , 3 , 5 , 7} B = { 1 , 2 , 3 , 4} 63)
A  B   1, 2,3, 4,5,7 ។ a) 16 x 2  25
  4 x   52
2
② A B
A  B   2 , 3   4 x  5 4 x  5
A B b) 9 x 2  24 xy  16 y 2
  3x   2  3x  4 y   4 y 
2 2

n  A  B  2 ។
  3x  4 y 
2

61)

315

64) 66) a3  b 2
a) 4 x  9   x  6 a  2 b3
5 x  15 a3  b2   2  32  8  9  1
3

x  3
b) 0.5 x  3.2  0.8 x  1.6
67)
5 x  32  8 x  16
4 x  11  2 x  15
 3 x  48
2x  4
x  16
x2
c) x  14 x  45  0
2

 S  14  5  9 68) y
 y  ax
 P  45  5  9
x5 , x9 ។ y  4 x 8
1
d ) x  8x  9  0
2 4  a  8 a
2
 '   4  9  16  9  7
2
1
y x
  4   7
2
x  4 7 1
1 x  4 y     4   2
2
65)
69) y x
7 x  5 y  1
a)  y  ax  b
3x  2 y  1
D  14  15  1  2 , 4 x2, y4
Dx 3 a 3
Dx  2  5  3  x  3
D 1 4  3 2  b b  2
D 4
Dy  7  3  4  y y   4 y  3x  2 ។
D 1

x  3 , y  4 ។ 70) 221 ឬ ?

1 221  1317
 x  6 y  4 1
3
 x  y 1
b)  4 ឬ 
2 ។
5 x  5 y  1  2 
221
0.5 x  0.5 y  0.1

71)
1 x  6 y  4 3
n
3  2  n  2  180o
 2 : 5  6 y  4   5 y  1 n9 )
n
35 y  21  y  
3

 9  2  180o
 140o
5 9
 3 2
 3 x  6    4   180  140o  40o
o

 5 5
2 3
x ,y ។ 40o ។
5 5

316

72) x 76)
1cm
m a) x 2  3 x  54  x 2  9 x  6 x  54
xcm
4cm  x  x  9  6  x  9
8cm
1 8 x m   x  9  x  6 

b) x 2 y  4 y 3  y  x 2  4 y 2 
4 x
32
ឬ x  32  4 x  x
5  y  x  2 y  x  2 y 

73) AC
A 6cm
D 77) x
a) 6 x  16  9 x  11
AC 2  AB 2  BC 2 3cm
 3x  27
AC  AB 2  BC 2 B C
x  9
AB  3 cm , BC  AD  6 cm 7 x  3 3x  1
b) 
AC  32  62  45  3 5 cm 8 4
7x  3  6x  2
74) x5
a)  13   15    4    2
c) x 2  4 x  21  0
b)  8  5  20   5    44
 S  4   7   3
c)  2   4   24
3 
 P  21   7   3
2

5 1 15
d )    0.75    0.2  x  7 , 3 ។
6 4 8
e)  18  72  2  4 2 d ) x2  8x  2  0
 '   4    2   16  2  18
2

f)
3
5
 
2 5 5  4 5  6 5
  4   18
x  43 2 ។
g ) 7  3 x  2   9  2 x  5   3 x  59 1
4x  1 5x  3 x  13
h)  
6 8 24 78)
i ) 3  9 x  y   5  x  2 y   22 x  13 y 8x  3 y  30
a) 
j ) 0.3  x  8 y   8  0.4 x  0.2 y   3.5 x  0.8 y 5x  9 y  3
k ) 18 x 2 y   2 xy   6 x 3 y   6 y 24 x  9 y  90

 5  1
2
  18 
2
40  5x  9 y  3 x  3
l )   xy    xy 2     y    29 x  87
 9  6   5  y
x  3 8  3  3 y  30
75)
3 y  30  24  y  2
a)  7 x  5 y  3x  4 y   2 y 10 y x  3, y  2 ។
 21x 2  28 xy  15 xy  20 y 2  20 y 2
0.8 x  0.3 y  0.8
 21x  13xy
2  8x  3 y  8
b)  2 3 9 ឬ 
 x y  4 x  15 y  18
 x  6    x  5 x  8
2
b) 5 2 5
 x 2  12 x  36   x 2  8 x  5 x  40 
1 4
x ,y ។
2 3
  9 x  76

317

79) 9 x 2  2 xy 85) x
x  3 y 1 l m l 41o
9 x2  2 xy  9  3  2  31  81  6  87 x  95o  41o
2
95o
 54o m
80) 9 x
6
86) x
 6  6  36 D
x  BDC A
9 1 x O
3 BC )
38o
BDC  90  38  52
o o o B C
1

36 x  BDC  52o ។
81) y
8x  3 y  7 87)
3 y  7  8x a )  12    13   4    3
7  8x b)  2  5  12   6    12
y
c )  4   2 4  0
3 2

82) y
5 5 1
a d )   0.2   0.5 
y 8 16 2
x
e)  2  8  18  0
y  8 x 3
f ) 4  3x  5   8  2 x  3   4 x  4
a 24
8   a  24 y 5 x  3 x  1 13 x  11
3 x g)  
24 2 3 6
x  2 y  12 ។
2 h) 4  6 x  7 y   8  2 x  4 y   8 x  4 y
i ) 0.2  5 x  10 y   2  2 x  0.5 y   5 x  y
83) y x j ) 28 x 2 y 2   5 xy   35 xy 3   4 x 2
y  ax 2 2
5   5   2  1
y  32 x4 k )  x 2 y     xy 3     y 3   x 3 y 2
6   9   15  6
32  a  4 2
 a  2
y  2 x 2 ។
88)
a)  x  7 y  x  7 y   x 2  49 y 2
84)
 x  4    x  2  x  8 
2
b)
n
 x 2  8 x  16   x 2  8 x  2 x  16 
 n  2  180o
n 8 )
n  x 2  8 x  16  x 2  10 x  16


 8  2  180o
 135o
 18 x
8
 180  135o  45o
o
89)

45o ។
a) x 2  8 x  12   x  2  x  6 
b) ax 2
 4axy  5ay 2  a  x  y  x  5 y 

318

90) x 93) 2010


a ) 9 x  19  11x  13 2010  2  3  5  67
 2 x  32
x  16 94) h
7 x  4 5x  4 1
b)  mgh  mv 2
8 6 2
21x  12  20 x  16 1 2
mv
x  4 v2
h 2 
1 mg 2g
c) x 2   0
4
1 95) y
x2 
4 y  ax
1
x y5 x  2
2
5 5
d ) x  2x 1  0
2 5  a   2   a   y x
2 2
 '  12   1  1  1  2 5
x4 y    4   10 ។
1  2 2
x  1 2
1
91) 96) y x
y  ax 2
x  y  1
a)  y  3 x 3
3x  2 y  18
1
2 x  2 y  2 3  a  32  a  
 3
3x  2 y  18 x4 1 2
5 x  20 y x ។
3
x  4 : 4  y  1  y  3
x  4 , y  3 ។ 97)
2 x  y  3
 2 x  y  3 n
b)  1 1 ឬ   n  2  180 o
0.5 x  8 y  2 4 x  y  4 n 8 )
n
2 x  y  3  8  2  180o
   135o ។
4 x  y  4 x
7 8
6x  7 6
7 7 2
x  : 2  y  3  y  98) x
6 6 3
7 2 x
x , y ។ 2
6 3
6
4
92) 3 xy  6 xy 2 m
x  5 y2
3xy  6 xy 2  3  5 2   6  5  22   90 m
x 6
 x 3 ។
2 4

319

99) x 103) x
A
BC : CD  1: 2 a ) 7 x  4  4 x  10
3 x  6
o
CAD x 46
BAC 
2 x  2
D
46 o
x  5 3x  7
ឬ x   23o B b)  1
2 C 4 2
x  5  6 x  14  4
100)  5 x  15
a)  5  2  8  4   7 x3
b) 18  30   6   23 c) 5 x 2  40  0
c)  5   32  4   11
2 5 x 2  40
2 x2  8
2 5 1 2
d)       x  2 2
3 6 3 3
  d ) x  8x  4  0
2
e)  2 3  2 2  24  4  6
 '   4   4   16  4  20
2

  10
2
f) 5 1   6
5   4   20
x  42 5
g ) 2  8 x  5   6  3 x  2   34 x  2 1
h) 0.8  0.6 x  5   0.5  0.9 x  3  0.03 x  2.5
104)
i ) 3  4 x  8 y   7  2 x  6 y   26 x  66 y
2 x  5 y  2
3 x  5 y 4 x  y x  17 y a) 
j)    x  3 y  12
6 9 18
k )  28 xy  7 xy   4 y
3 2
D  6  5  11
2 Dx 66
5 5  4 2 Dx  6  60  66  x   6
l ) x3 y   x 2 y   x  D 11
6 3  3 5y
D 22
Dy  24  2  22  y y   2
D 11
101)
a )  3 x  4 y  3 x  4 y   9 x 2  16 y 2 x  6 , y  2 ។

 x  6  4 x  3   2 x  5 
2
b) 0.3x  0.4 y  0.1
 3x  4 y  1
 4 x 2  3 x  24 x  18   4 x 2  20 x  25  b)  6 4 7 ឬ 
 x y  18x  20 y  7
5 3 15
 4 x 2  21x  18  4 x 2  20 x  25
  x  43 D  60  72  12
Dx 8 2
Dx  20  28  8  x   
102) D 12 3
D 3 1
a) x 2  4 x  12 Dy  21  18  3  y  y  
D 12 4
 x 2  2 x  6 x  12   x  2  x  6 
2 1
 x  y   12  x  y   36 x ,y ។
2
b)
3 4
  x  y  6
2

320

105) 2ab  b2 111) x


62o
a4 b  2 m
2ab  b2  2  4   2    2    20
2
x
x  62  46 o o
46o
m
106) 4  108o
A
A B
x
 6  6  36 112) x 64o
O D
4 3 AB = AC 

1 , 3 ;  2 , 2 ; 3 , 1 ABC  ACB C
B
3 1 x  180o  2ACB
  ។
36 12 ACB  ADB  64o AD
x  180o  2  64o  52o
107) y
5x  2 y  3 113)
2 y  3  5x
a)  8    10    16    2
3  5x
y b) 16  24   8   13
2
c)  3   2    7
2 4

2
108) y 5 1
d )   4     1
a 9 3
y
x e) 18  98  2   5 2
y9 x4
 12

2

a 36 f ) 6 2   10  2 6
9  a  36 y 6
4 x g ) 7  4 x  3  5  5 x  4   3 x  1
36
x  6 y  6 ។ h) 0.6  3 x  0.7   2  0.3 x  0.2   2.4 x  0.02
6
i ) 4  9 x  6 y   9  x  2 y   45 x  6 y
109) y x 5 x  3 y 2 x  5 y 23 x  29 y
j)  
y  ax 2 12 9 36
k )  63 x y  9 xy   7 xy
2 3 2
y 8 x4 2
 5  35 7 2 y
1 x2 l )  xy   x 2 y 3  y 
8  a  42  a  y  6  48 20 3
2 2
114)
110)
a)  5 x  y  6 x  y 
n
 30 x 2  5 xy  6 xy  y 2
 n  2 180 o

 30 x 2  xy  y 2
n  12
 x  7    x  8 x  6 
2
b)
 x 2  14 x  49   x 2  6 x  8 x  48 
12  2 180o  10 180o  1800o
 x 2  14 x  49  x 2  14 x  48  1

321

115) 12 x  34 y  15

a ) x  100 
2
 x  10  x  10  12 x  8 y  6 1
y
b) ax 2  6ax  9a  a  x 2  6 x  9  42 y  21 2
1 1
 a  x  3
2
y 6 x  4    3
2 2
1
ឬ 6 x  1  x  
116) x 6
a) 9 x  5  6 x  7 1 1
x , y ។
3 x  12 6 2
x  4
118) 9a 2  ab2
b) 0.2  x  2   0.8  2 x  3 
a 3 b  4
x  2  8 x  12
9a 2  ab 2  9  32  3   4 
2

 7 x  14
 81  48  33
x2
c) x 2  3 x  54  0
119)
x 2  6 x  9 x  54  0
A B
x  x  6  9  x  6  0
 6  6  36
 x  6  x  9   0 9
x  6  0 x  6  2, 2  ;  2, 4  ;  2 , 6  ;  4, 2  ;  4, 4  ;
 x  9  0 ឬ  x  9
   4 , 6  ;  6, 2  ;  6, 4  ;  6 , 6 
ឬ x  6 , 9 ។
9 1
  ។
d ) x  8x  9  0
2
36 4
 '   4  9  16  9  7
2

120) ?
  4   7
x  4 7 3 3,2 7 5
1
3 3  32  3  27
117)
2 7  2 2  7  28
2 x  7 y  11
a)  5  52  25
5 x  2 y  8
D  4  35  39 27  27  28
78 2 7 ។
Dx  22  56  78  x  2
39
39
Dy  16  55  39  y 1 121) y
39
y  ax
x  2 , y 1 ។
y  18 x  9
1.2 x  3.4 y  1.5 18  a  9  a  2 y  2 x
 12 x  34 y  15
b)  2 4 1 ឬ  x  2 y  2  2  4
  x y  6 x  4 y  3
 3 9 3 y4 ។

322

122) y x 127) EBC


y  ax 2
A D
y  4 x4
E
1
4  a  42  a   B C
4
1 BCA  CAD
y   x2 ។ AD||BC
4
CBD  ADB
AE = DE CAD  ADB
123)
BCA  CBD
n
E AC BD
 n  2  180o
EBC ។
n
n5 AB = DC

 5  2  180 o

 108o ។ EBC EC = EB
5
 AE  DE

 EC  EB
124) x AC  DB
107o
m
ABCD
85o x AC  DB
x  180o  107 o m
AB = DC ។
 73 o

128)
125) x C x 4  289
A o
68
O x 2   289  x 2  17
x x 2  17 x
x 2  17 x   17
B
x   17 x )
180o  68o
ABC   56o x  17 ។
2
180o  2  68o
OBC   22o
2 129) CAB , ABC BCA
A
x  ABC  OBC  56o  22o  34o AB : BC : CA  3 : 4 : 5

126) 
AB BC CA O
y   x 1 , y  x  1 ,   B
3 4 5 C
y  2 x  2 , y  3x  3 ,

AB BC CA AB  BC  CA 360o
2x  2  3x  3 x  1 y0      30o
3 4 5 3 45 12
x  1, y  0 ។

323

AB -
 30o  AB  90o
3
BC
 30o  BC  120o
4
CA
 30o  CA  150o
5

AB 90o
BCA    45o
2 2
BC 120o
CAB    60o
2 2
CA 150o
ABC    75o
2 2
BCA  45o , CAB  60o , ABC  75o

AB : BC
131)
AB : BC  3: 4 AB : BC  3 : 4 ។ 1 6
130) - A D
1 3 ABCD = 1 , ABFE = 2
y  x2  x  1 EFGH = 5 , CDHG = 3
B
2 2 C
ADHE = 4 , BCGF = 6
 b b2  4ac 
S  ,  E H
 2a 4a 
F G
  3
2
1 
    4   1 
3
  A = ABCD+ ABFE +ADHE =1+2+4 = 7
S  2 , 
2 2 
 1 1  B = ABCD+ ABFE+ BCGF = 1+2+6 = 9
 2 2  4   C = ABCD+ CDHG+BCGF = 1+3+6 = 10
   2 
D = ABCD+ CDHG+ADHE = 1+3+4 = 8
 9 
3 4 2  3 1
E = ABFE + EFGH+ ADHE = 2+5+4=11
S  ,   S  ,  F = ABFE + EFGH+ BCGF = 2+5+6 = 13
2 2   2 8
G = EFGH +CDHG+ BCGF = 5+3+6 = 14
 
H = EFGH +CDHG+ ADHE = 5+3+4 = 12
 3 1
 , 
 2 8
7, 8, 9, 10, 11, 12, 13, 14
-
y0 132)
1 2 3
x  x  1  0 ឬ x 2  3x  2  0  5 x  2 y  5 x  2 y    5 x  y 
2

2 2
a bc  0  25 x 2  4 y 2   25 x 2  10 xy  y 2 
c 2
x1  1 , x2    2  25 x 2  4 y 2  25 x 2  10 xy  y 2
a 1
  5 y 2  10 xy
1, 0  ;  2, 0  ។

324

133) 138) 
x  1  5x ឬ x  5x  1  0
2 2
3
sin  
   5  4 1  1  25  4  29
2
2
  60  360o k ឬ   120o  360o k
o

  5  29 5  29
x  ។ k
2 1 2
0o    180o
134) y x   60o ,   120o ។
y  ax 2
y  20 x  2
139) ABC
20  a   2   a  5
2
G
y  5x ។ A xA , yA  ; B  xB , yB  ; C  xC , yC 
2

x x x y y y 
G A B C , A B C 
135)  3 3 
10 cm 6 cm ABC
A(-2 , 6) , B(7 , -9) C(1 , 0)

6 cm
 2  7  1 6   9   0 
G ,  ឬ G  2 ,  1 ។
 3 3 
10 cm
a 140) AB = DC A D
2 2 AD||BC
 6 cm   10 cm 
a 
2
  
 2   2  AB  DC B C

a 2  9 cm2  25 cm 2 AB  DC
a 2  34 cm2
a  34 cm 141) 8051
8051  72  902
34 cm ។
8051  902  7 2
8051   90  7  90  7 
136)
xz  x  2 y  2 yz 8051  83  97
83 97
 x  z  1  2 y 1  z 
8051  83  97
 1  z  x  2 y 

142)
137)  m2  n 2 ឬ  n 2  m2
B
  n  m n  m
n  m 1
A ( ។
A B។

325

143) AG 146) tan 1  2 


D tan 1  tan  2
C tan 1   2  
1  tan 1  tan  2
A
B tan 1  2 tan  2  3
23 5
19 cm H G tan 1   2     1
25 cm 1  2  3 5
E 32 cm F
147)  
EG 2
3x 2  4 x  8  0
EG 2  EF 2  FG 2
 
  32 cm    25 cm 
2 2

b 4 4
 1024 cm 2  625cm 2   S    ។
a 3 3
 1649 cm 2
AG 2  EG 2  AE 2 148) OP PH
 1649 cm  19 cm 
2 2
P

 1649 cm 2  361cm 2
 2010 cm 2
O
AG   2010 cm , AG  0 A   
B
3cm H 7cm
AG  2010 cm ។
AB 3cm  7 cm
OP  OA    5cm
144) 2 2
OP  OA  5 cm
f  x OH  OA  AH  5 cm  3 cm  2cm
x  R R  f   PH 2  OP2  OH 2
( ១០)
PH  OP2  OH 2 , PH  0

x3  3x 2  4 x  8 x2 PH  52  22 cm
PH  21 cm
R   2  3  2  4  2   8
3 2

R  8  12  8  8 PA PB
R4 AP 2  AH 2  PH 2 , AP  0

o
AP  AH 2  PH 2  32  212  30 cm
145) sin135
AB  PH 10  21 2100
sin 180o     sin  PB     70 cm
PA 30 30
sin135o  sin 180o  45o  PA 30 3
 
 sin 45 o
PB 70 7
2

2 PA : PB  3 : 7 ។

326

149) a 
a  30 , x2  6  20  2 x
   20  2x )
360 2 x
x xa 0
2
x1  5

5  5  a  0  a  30
2

o
a  30 x2  x  30  0 S   R2 
360
x1 x2  P  30 20  2 x
S   x2 
30 30 2 x
x2    6 ។
x1 5  10  x  x
  x 2  10 x
   x 2  10 x  25   25
150)
2n  1 2n  1
   x  5   25
2

2n
 x  5  0    x  5  0
2 2

n
ឬ   x  5  25  25
2

 2n  1   2n 1
2 2

S
 4n  4n  1   4n  4n  1
2 2
 x  5
2
0 ឬ x 5 *
 8n  4  2n 25 cm 2
 2n  1   2n 1  4  2n
2 2
x  5cm ។

4 152) ១ ២

។ ៣

- 6 3
151) x 6  6  6  216
- ១ ២
6
20 cm 
x cm - ៣
2x  20 5
 20  2 x  cm  - 6  5  30
30 5
 ។
216 36

153) ABC
0  20  2x  2 x
3 15
0  20  2 x  x  10 SABC 
4
10
20  2 x  2 x  x 1
 1 SABC   AB  AC  sin A
10 2
 x  10 * 1 15 3 15
 1 S ABC   2  3 
2 4 4

327

BC 158)
BC 6 A B
BC  AB  AC  2  AB  AC  cos A
2 2 2

sin 2 A  cos2 A  1 6 3
cos A  1  sin A
2 2
, 90  A  180
o o

cos A   1  sin 2 A , cos A  0 6! 6  5  4  3!


C  6,3    20 ។
2 3! 3! 3! 3  2
 15  16  15 1
  1      
 4  16 4
159) A B
 1
BC 2  22  32  2  2  3     A = { 1 , 4 , 7 , 10 }
 4
B={1,3,5,7,9}
BC 2  13  3  BC  4
A  B   1 , 7 ។
154)

 x  3 y   x 2  3xy  9 y 2   x3  27 y 3 160) a
f  x   x3  3x2  ax  4
155)
x2 0
x  3 x y  3 xy  y   x  y  f  2  0
3 2 2 3 3

2  3  22  a  2  4  0
3
156)
8  12  2a  4  0
52 6  3 2 6  2
2 2
a  12
 
2
 3 2  3 2
161)
157) cos 
d  x1 , y1 
sin 
tan   3 ឬ  3 ax  by  c  0
cos 
sin  ax1  by1  c
cos   d
3 a 2  b2
cos A  1  sin 2 A
2
, 90o  A  180o
(0 , 1) x  3y  7  0
cos A   1  sin A , cos A  0 2
1  0  3 1  7 10
sin  d   10
  1  sin 2  12   3
2
10
3
sin   3 1  sin 2 
sin 2   9  9sin 2  162)  x , y , z
10sin 2   9 x 6  y 6  z 6  3xyz
3
sin   , sin   0
10
3
x , y , z    0, 0, 0  , 1,1,1 , 1, 1, 1 ,

cos  
sin 
 10  
1
 
10  1,1,  1 ,  1,  1,1
3 3 10 10

328

163) x  1 y  1   1  2
x cm x2 y  1  2  1
 20  2x cm P  1 , 2  Q  2 ,  1
o
360
R
x  y2  5
2

0  20  2x  2 x
O  0 , 0
0  20  2 x  x  10
10 - OP O  0 , 0 P  1 , 2
20  2 x  2 x  x
 1 y  2 x P
10
 x  10 * y  ax  b P  1 , 2
 1 1
20  2 x a
A   x2  2
2 x 1 5
 10  x  x 2  1  b  b 
2 2
  x 2  10 x  cm 
2
P y  x
1 5
16 cm 2  A  24 cm 2 2 2
- OQ O  0 , 0 Q  2 ,  1
 x2  10x  16 1
1
 x2  10x  24  2 y x
2
Q
1 : x2  10 x  16  0 y  a'x b' Q  2 , 1
 x  2 x  8  0  2  x  8 3 a2
 2  : x 2  10 x  24  0 1  2  2  b  b  5
 x  6  x  4   0  x  4, 6  x  4  Q y  2x  5
* , 3  4 -
10 P Q R
 x4 , 6 x8
 1 1 5
x   2x  5
10 10 2 2
2  )
5  1 x  5  4 x  10
cm  x  4 cm , 6 cm  x  8 cm ។  3x  15
10
 1
x5
164) P Q x  5 : y  25  5  5
R  5 , 5 ។
 x2  y 2  5 1

x  y  1  2 165)  x, y, z 
x 4  y 4  z 4  3xyz
 2 : x  y  1  y  1 x 1
1 : x 2  1  x   5
2

x , y , z    0, 0, 0  , 1,1,1 , 1, 1, 1 ,


2x2  2x  1  5
 1,1,  1 ,  1,  1,1
x2  x  2  0
 x  2  x  1  0  x  1 , 2

329

166) 168) cm 2
n 2  2n 50 cm 2
n ។” 1
-
pq 1000
qp 1 cm 1000 cm
n -
n 2  2n ”។ 100 m = 10000 cm
10 cm 50 m = 5000 cm
5 cm
n n  2k  10cm  5cm  50cm2
k
n2  2n   2k   2  2k 
2
169) 5 cm
 4k  4k
2

20 cm 2
 2  2k  2k 
2

4 r 2 cm2
2k  2k
2
n  2n
2
2
r  5 cm 4 5 cm2  20 cm 2

81 cm 2
។ 4 r 2 cm 2  81 cm 2
4r 2  81
167) P  x  x  3 x  4
81
Q  x ax  b r
4
P  x  x  3 x  4 9
r , r 0
P  x    x  3 x  4 Q  x   ax  b 2
- P  x x 3 3 9
r  cm ។
P  3  3 3a  b  3 1 2

- P  x x4 1
170)
P  4  1 4a  b  1  2
- 1  2  x  4 y   x2  4 xy  16 y 2   x3  64 y 3

3a  b  3

 4a  b  1 171)

125 x 3  75 x 2  15 x  1   5 x  1
3
D  3  4  7
Da 2 2
Da  3  1  2  a
 
D 7 7 172) 6
D 15 15
Db  3  12  15  b  b   4
D 7 7 V   r3 r 6
3
2 15
 x ។ 4
7 7 V    63  288 ។
3

330

173) tan A 2 x 2  4kx  3k  9  0


2 '  0
cos A 
 2k   2   3k  9  0
2
3
sin A  cos2 A  1
2
A
 2k   2   3k  9   0
2

sin A  1  cos2 A
4k 2  6k  18  0
94
2
2 5 2k 2  3k  9  0
sin A  1     
3 9 3
f  k   2k 2  3k  9 f k   0
5 f  k   0  2k  3k  9  0 2
sin A 5
tan A   3  ។   32  4  2   9  9  72  81
cos A 2 2
3 3  81 3  9
k1    3
2 2 4
3  81 3  9 3
174) k2   
2 2 4 2
0, 1, 2, 3, 4 5
១ 3
k  3 
2
- ១ 5 - ៤ 3 f k   0 + 0 - 0 +
- ២ 5 - ៥ 2 3
k  3 ឬ k
- ៣ 4 - ៦ 1 2

k
5  5  4  3  2  1  5  120  600 ។ 3
k  3 ឬ k
2
175) ) (
k' 3 0 3 k
y  ax 2  bx  c 2
177)
 b b 2  4ac 
- S  , 
 2a 4a 
b x2  y 2  R2 , R
- x
2a (5, 12)
52  122  R2 ឬ R 2  169
y   x2  8x  5 x 2  y 2  169
  8   4   1  5 
2
8
 ,  178)
 2   1 4   1 
 
4 x3  8x2  11x  3  0
 4 , 21 ។
x  3
 x  3  4 x2  4 x  1  0
176) k
 x  3 2 x  1 0
2

y  2 x  4kx  3k  9
2

1
y0 x  3 ,
2

331

179)  182) y x
1 y  ax
cos   0o    360o
2 y  8 x  2
cos  
2 8  a   2  a  4
2
y  4x ។

45o 183) y
a
cos  y
2 x
2
y2 x4
315 o a 8
2  a 8 y
4 x
45o    315o ។ x2
8
y 4 y4 ។
2
180) P
184) cm 2 ?
A a B b 30 cm
- P  x AB 30 cm  30 cm  900 cm 2
mb  na
m:n x ។
mn
54 m 2  540 000 cm 2
- Q  x AB
mb  na 540000 cm2
m:n x ។   600
mn 900 cm2

A(-3) B(3) P 185)


AB AP:PB=1:2 20 g 160 g
P
1 3  2   3 3 20 g
x   1 P  1  0.125
1 2 3 160 g

Q
Q AB
AQ:QB=1:2 Q 20 g 100%
 12.5%
1 3  2   3 9
x   9 Q  9  160 g
1 2 1
186) ACD cm 2
181) x
x AD CD
x 120 6 x 2។
120  ។
100 5

332

187) 100
50 80 g OBM ODN
80 g - ABD  CDB
1   1.6 g
50
- OB  OD ( O BD)
100
- BOM  DON
1.6 g 100  160 g
OBM ODN

150 g 790 g 190)


 790 g  150 g  640 g

6cm
640 g
 400 ។ 8cm
1.6 g 10cm

188) m 5 km? V  10 cm  8 cm  6 cm  480 cm3

1 000 m 1 km 12cm
5 000 m 5 km។ 6cm
3cm

m2 30 000 cm 2 ?
10cm
10 000 cm 2 1 m2 3cm
6cm
30 000 cm 2 3 m2 ។ 6cm

cm3 10 ? V   6cm  6cm  10cm    6cm  3cm 10cm 


1  1dm3 1 000 cm3  360cm3  180cm3  540cm3
10 10 000 cm3 ។ 191) A
A 72  4  76 ។
189)
B C
A D B 72 1  71
N C 72  8  64
M O 71  64  5

B C D
x D
MB OBM DN
E 72  5  77
ODN
72
76  71  64  x  77
OBM ODN  72
5
។ D x  360  288  72 ។

333

192) ABCD 195)


A D x  y  8

90 x  120 y  870
a cm

B C x  y  8 x  y  8 1
 ឬ 
b cm
90 x  120 y  870 3x  4 y  29  2
2  a  b  cm
ABCD
1 x  y  8  y  8  x 3

ABCD 3  2
3x  4  8  x   29
ABCD  a  b  cm2
3x  32  4 x  29
193) cm 2 x3

d cm x 3 3 : y  8  x  8  3  5
d2 3  5  8
cm2 
4 90  3  120  5  870
3, 5។
194) ABC
196) (PGCD)
A
(14, 35)
B
17  2  7 35  5  7
PGCD(14, 35) = 7 ។
B C A'
(26, 39, 52)
ABC 4 26  2 13
ACB , CB ' A ' , B ' CA ' BAA ' 39  3 13
52  22 13
BAC
PGCD( 26, 39, 52) = 13 ។
BAC 3
B ' A ' C , CAB ' , ACB ' ។ 197) A’C cm
5cm
ABC A ' C  A ' D  CD
2 2 2 A D

BAC  CAB ' A ' C 2  AD 2  AB 2 4cm


ABC
BAC   52  4 2  9 P
ABC  BAA ' 2
x ABC ABC  A ' C  3 cm B A' C

x 5 PB cm
xx  180o  x  180o  x  72o
2 2 PA  PA' BA '  BC  A ' C  2 cm
ABC  72 ។
o
PB  PA  4  PA  4  PB

334

PBA ' 202) 6


PB 2  BA '2  PA '2 50 150
PB 2  BA '2  PA2 6 9 6 25
PB 2  22   4  PB   25  9  1  17
2
6
PB  4  16  8 PB  PB
2 2

8 PB  12 6 8
3 6 8 24
 PB  cm
2 50 150
24 3 24 6
198) 24  6  3 1  4
1.05 kg 0.3 kg
1.05kg  0.3kg  0.75kg 203) cm
b h
b 5
1.05 kg
 3.5 b : h  5:8 ឬ 
0.3 kg h 8
40 cm
40  8
199) h  64 cm
5
8400 5040
cm
320 cm 2
8400  5040  100%  40% 1
8400 bh  320 ឬ bh  640
2
b  bh 5  640
 ឬ b  400
2

200) h 8
b  400  20 cm , b  0
។ 640 640
h   32 cm ។
b 20

204)


2 1 2 23 2
1      
5 2 5 52 5
201)
2 15  4 2 11
x    
5 10 5 10
x  15%x  276 ឬ 1.15x  276 11

276 25
x  240 g
1.15

335

208) cm 2
11 20cm
25  11  5  11  1.1

2 25 2 10
5
45cm

1  2 11  3 10  11 3 21
1        
2  5 25  2 25 2 25
75  42 33
 
50 50 d2  d 
S  2S B  S L  2    2   h
 4  2
d2   202 
205)
3    dh     20  45 
4 2 2
 200  900  1100
1 60
3 3   3.14
 60  45
4 4 S  1100  3.14  3454 cm 2
mm 8 cm ?
1 cm 10 mm
1 2000 cm 2
8 cm 8 10  80 mm
3454
 1.727 ។
2000
m2 400 cm 2 ?
1 m2 10 000 cm 2
209) x
400
400 cm 2  0.04 m 2 2
10000 x
5
206)
x  1 y5 120
7 x  9 y  4  7  1  9 5  4  34 2
x  120 ។
3x2  2 xy  3  1  2  1 5  13 5
2

207) E D
D 500  106  394 m x
3
E 500  92  408 m x
5
D E 408  394  14 m
120
B C 3
x  120 ។
258   339  597 m 5

336

x
60 g 120 g
40 g
2 3 40 g 120 g
x  120  x  120  80 g
5 5 60 g
1
x  240
5
x  1200 215)
1 1
210) a 4
abc
s 1 4  4 ។
3
a  b  c  3s
a  3s  b  c 5

211) y 4

y  2x  5 5  4  20 ។

x4 y  2 4  5  3
216) A
212)
A G
H

10  2  180o  144o


10
C E
213) x m B D F

58o A G។
EF
m
32o EF CB ។
x

217) x
x  58o  32o  26o x 3 x  100

214)
340

50 g 1 3x 100  340
 ។
150 g 3 3x  240
x  80

337

218) cm 220) cm PB x
A D A
x cm
8 cm
8 cm P
B C
B C
6 cm
  8cm
8cm  3   24  4  cm  PB  AB  AP
2
PB  8 cm  x cm
cm2
PB   8  x  cm

y x
1   82
8  8  
2 4
 64  8  cm 2
y PBC
1
219) y   BC  PB
2
A 1
y   6  8  x    24  3x  cm 2
2

x a
B G D
F
PBC a cm 2 ya
E
a  24  3x
C 3x  24  a
BF = DE 24  a
x cm
3
BCF DCE ។
221) a
y
b y  ax
BCF DCE y
x A

- CE = CF
x
- C O

- BC = DC B
BCF DCE
y  ax A(4, 3)
EGF 3
3  a4  a 
ABC  117 o
CBF  46 o
4
BAD  180  117o  63o
o
b
ABG  117o  46o  71o b
y
BGD  360   2  71  63   155
o o o o A(4, 3)
x
b
EGF  BGD  155o 3  b  12
4

338

O B 224) cm AH
12 A
- b  12 y
x
12
y B 2
x 60o
12 B C
y  6 B  2,  6 2 cm H
2
- O  0 , 0 ABH

y  ax 30o , 60 o 90o

y  ax B  2,  6 BH : AH : AB  1: 3 : 2

6  a   2  a  3 BH  3
BH : AH  1: 3 AH 
1
y  3x ។
AH  2cm  3 AH  2 3 cm
2
222) cm
cm CH
CAH
30o , 60 o 90o
12 cm
AH : CH : AC  1: 3 : 2
AH  3
6 cm AH : CH  1: 3 CH 
1
S B   R 2     6 cm   36 cm2
2
CH  2 3 cm  3

cm3 CH  6 cm
1 1
V   S B  h   36 12  144 cm3 cm 2 ABC
3 3
1
223) x y SABC   BC  AH
2
1
   BH  CH   AH
 x  y  220 2
 1
10 x  15 y  3000  200    2  6  2 3
2
 8 3 cm 2
 x  y  220

2 x  3 y  560
225) cm3
D  3  2  1 , Dx  660  560  100
V  a3 a
Dy  560  440  220
10 cm
x  100
V  10 cm   1 000 cm3
3

y  120

339

226) ABC
x 5 y  2 ABC
2 x  3 y  2  5  3  2  10  6  16 x o , 2 x o ,3x o 30o , 60o ,90o

xy  2 y 2   5 2  2  2   5  8  3
2
ABC ។

227) y 231)
a d
y vm  d
x t
y  2 x  6 t
2 
a
 a  12 y
12 d  1 200 m t  15
6 x 1 200
12 vm   80
x 3 y  4 15
3
d  1 200 m t  20
228) x
1 200
vm   60
3x 13  11x  3 20
8x  16
232) 3.14 15.70
x  2
3.14 15.70
4 x  5 3x  2 15.70
  5 ។
3 4 3.14
16 x  20  9 x  6
7 x  14 3.14 62.80

x2 3.14 62.80


62.80
 20 ។
3.14
229) (GCD)
3.14×25
(14, 21) 14  2  7 21  3 7 3.14
GCD 14, 21  7
3.14  25  3.14  5  3.14  20
(10, 15, 20)
10  2  5 , 15  3  5 , 21  22  5
233) cm 2
GCD 10, 15 , 20   5
3 cm
1
230) x S   6cm  3cm
2 6 cm
ABC x o , 2 x o ,3x o  9 cm 2
xo  2 xo  3xo  180o
6 x o  180o
S  4cm  4cm
x  30o 5 cm
4 cm

 16 cm 2

4 cm

340

234) 237) 6!
1 2  3 1 2  3  4 6!  1 2  3  4  5  6  720
1 2  3 6 3
 
1  2  3  4 10 5
 6! 5!   6! 4!   6! 3!
-
 6! 5!   6! 4!   6! 3!
  720  120    720  24    720  6 
S
 600  696  714
1  2  3  4  5  6 21  2010

S S
3: 4 238) cm
21 4
a  cm 
21 3
 S  28
S 4 3
r
-
a
28  1  2  3  4  5  6  7 2 r  a r  cm  ។
2

235) S  a
2
 a  a2 a2
S r      2 
2

 2  4 4
 cm2 

O 239)
20
1 60
1 20 1
6 20 
60 3

g 0.3 kg?
1 kg 1000 g
O
0.3 1000
0.3 kg  300 g
1
236) m2 20000 cm 2 ?
400 $1.00 10000 cm 2 1 m2
$2.00 20000 1
20000 cm 2  2 cm2
800 10000
240)

1000 3 x  2 y  11
 2.5 
400 3x  2 y  11 3x  3 y  15
 ឬ
CO 2  x  y  5 y4
CO 2 x  y  5  4  5  1
3150
 3.15 ។
x  1 , y  4 ។
1000

341

 y  6x  3 244)

 y  4x  7 B( 3)
6x  3  4x  7 10  3  7
2 x  4
x  2
x  2 y  4  2  7  15 A D
x  2 , y  15 ។ A: 3 D: -2
10  3   2  26
241) x
2x  4  x  6 ១ C
x2 ២ D

0.4x  5  0.9 x  5 10  2   2  40


4 x  50  9 x  50 245) AOB COD
 5 x  100
A D
x  20
O
2 x  1 3x  2 79 o

3 4 31o
B C
8x  4  9 x  6
 x  2 AOB COD

x2
ABO CDO
242) ABO CDO
18 ។

CAD
18  2  180o  160 o
CAD  ADB  79o
18
ឬ CAD  DBC  79o
CAD  31o  79o
180 160  20
o o o

CAD  48o
246) 5!
243) x
m 5!  1 2  3  4  5  120
n
52o
x  n ! 5!   n! 4!   n! 3!  2010
n ! 120  n ! 24  n ! 6  2010
3  n ! 150  2010
o
43
m
n !  720
720  1 2  3 4  5  6  6!
x  180o   52o  43o   85o n6 ។
n!  6!

342

247) ac bd 251) AOB COD


- A D

O
ab ac bc
cd c d B C

AOB COD
ac bd ។

248) x ABO CDO


ABC A, B ABO CDO
C o
x , 2x o
3x o

x  2 x  3x  180
o o o o

6 x o  180o DAB  ABC

x  30 ABCD AB / /CB

DAB  ABC  180o


249) y AB
ABCD
A, B, C D 252) cm2 a
yo , 2 yo , 3 yo 4 yo 6a cm 5 cm
y o  2 y o  3 y o  4 y o  360 1
S   6a  5  15a cm2
2
10 y o  360o
y  36 253) kg
52.5 kg
250) z 1.2
ABCDE
A, B, C, D E 52.5 1.2  63 kg
z o , 2 z o , 3z o , 4 z o 5z o
z o  2 z o  3z o  4 z o  5z o  360o
15 z o  360o
z  24o 52.5 kg
42 kg

42 kg
 0.8 ។

o
360 52.5 gk

343

254) 256) cm3


80
36
8 cm
36 100%
 45%
80
V  a3  8 cm3   512 cm3
15%

10 cm
15  80
15%  80   12
100 2 cm
6 cm
5 cm
255) X Y
9 cm
72o V  10  4  6    5 10  2 
X
 240  100

116o  340 cm3

257) g 200
X  116  90  72  360
o o o o
50 140 g
X  278  360o o
200
X  82 o
140 g  200
 560 g
50
AD BC E

A C 50 140 g
140 g
74o E Y 1  2.8 g
50
43o 2800 g
35o
2800 g
D  1000 ។
B 2.8 g

258) x
8
74  35  Y  43
o o o
11
109  Y  43
o o

Y  66o 8 x  11

344

x cm2
10
5 1    4 2  1    82 
18     
2 4  2 4 
8 x  11  10 x  5 1 1
 18   4   16 
2 2
 18  2  8
 12 cm 2
x
8 x  11  10 x  5 261) x
8 x  10 x  5  11 63o
 2 x  16
x8 x
8 ។ 38o
m

259) ឬ m

a 14 15 b x  180o   38o  63o 

12 c 6 d  180o  101o  79o


e f 10 g
262)
h i 3 16
27 L 3
 ឬ 3: 4 ។
36 L 4
15  6  10  3  34 A B
a, b A, B
a h
.......... a  b  35 1

a  1 , h  13 a 3
   2
b 4

260) 12 cm
1 : b  35  a 3  2
a 3

35  a 4
4a  105  3a
12 cm 7a  105
A B
8 cm C a  15
a  15 3
12 cm b  35 15  20
1   122  1
   36  18 cm ។
2
 A 15
2 4  2
B 20 ។

345

263) m 266) cm2


d
v d  vt
t
v  80 m / mn t  8 mn
d  80  8  640 m

8 cm
d d
v t 8 cm 90o
t v
d  2 km  2000 m v  80 m / mn  90o
S r      8 cm  
2 2

2000 m 360o 360o


t  25 mn 1
80 m / mn     8 cm    16 cm 2
2

25 ។
4

cm2
264) C 1 82 
16       16  8  8 cm 2
2 4
A

267) B
B I
C H
y
m
D G  A
E F

C H។  C
B
O x
GF
y  2x  6
GF DE ។
B y0
2x  6  0 x  3
265) x
B  3, 0  ។

17  x   x  15  5 m
A x0
x y  2 0  6  6 A  0 , 6
m A 0 , 6 ; C  2,1
17  x   x  15   5 y  6 1 6

17  x  5 x  75 x0 20
5
4 x  92 y6   x
2
x  23
5
y   x6
2
346

268) ២០ 271) a y
២០ ២១ ២២ ២៣ ២៤
y  ax 2 y  bx 2
+3.9 +4.2 +5.3 +6.0 +5.6
  
-3.6 -3.8 -3.8 -2.5 -2.7 H B C

២០

30   3.6   26.4 oC A

x
O

២២
y  ax 2 A(1, 2)
2  a 1  a  2
2
a2 ។
5.3   3.8  9.1 oC
២២ ។ B
3 B y  ax 2 ឬ y  2 x 2
269) cm
B x2
5 cm
y  2 2  8
2
y 8 ។

5 cm b
a cm
C BH yC  yB  8
HB = BC xC  2 xB  2  2  4

V  5  5  a  25a cm3 C C  4 , 8
y  bx 2 C  4 , 8
2
cm
1
8  b  42 b ។
S  2  5  5  4  5  a   50  20a  cm2  2
272) cm AH
270) x y 5 cm
A D
AH 2  BH 2  AB2
6 cm
 x  y  30

7 x  2 y  95 AH 2  AB2  BH 2
AH 2  62  22 B C
H
x y
 36  4  32 7 cm

AH  32  4 2 cm AH  0
 x  y  30

7 x  2 y  95 cm2 ABCD
D  2  7  5 ABCD
35  AD  BC   AH
Dx  60  95  35  x7 S
5 2
115
Dy  95  210  115  y   23

 5  7   4 2  24 2 cm2
5
2
x  7 ; y  23 ។

347

273) 276) 2
6,  5,  1, 2, 4, 5 4 A, B, C D 2
6   5  11 AB, AC, AD, BC, BD, CD
6 ។

 65  30 A
AB, AC, AD
A, B C
3 1
A A 
C 6 2
B
5 A
  6   11 1
1 A
A  5 , B  1 , C  6 ។ 2
1
A
274) x 2
1 2 A
y x y4
2 1 1 1
   25% ។
1 2 2 4
4  x 2 ឬ x 2  8 ឬ x  2 2 ។
2
277)
y
“2 y 8” ។
១ ២ ៣ ៤
1
y  x2 2  x  4
2 ...
x0 x0
1 1 1  02  12
y   02  0 ។
2 2 5  12  22
“2 y 8” ។ 3 13  22  32
275) cm3 4 25  32  42
......................................
8 7 2  82  49  64  113 ។
2r cm 278)
r cm

6.4
V  S B h   r 2  2r  2 r 3 cm3  1.5 ។
4.3
1000
2r cm x y
x
r cm
y

x
1 1 2 1000 1000 ។
V  S B h    r 2  2r   r 3 cm3 y
3 3 3
348

280) ១) តរើគូររីតោណណា ដែលគួបង្ហាញថាវាប៉ន


ុ គ្នា?

A
y  130 000 000
x D
1000  2.1
y E
x
1000  2.1 C
130 000 000 B
x  273000 ។
គូររតី ោណដែលគួបង្ហាញថាវាប៉ន
ុ គ្នាគៈឺ

279) cm AC ACD នង
ិ CBE ។
D
5 cm ២) តរជើសតរសលក្ខ
ើ ខណឌទាំងបដី ែលររម
ឹ ររូវៈ
A
អក្សរដែលររូវនឹងលក្ខខណឌទាំងបីររឹមររូវគឺៈ
E
2 cm
ក្, ខ និង ច ។

B
4 cm
C ៣) ពនយល់ជាពាក្យ នូវលក្ខខណឌប៉ន
ុ គ្នាននររីតោណ
ររីតោណទាំងពីរប៉ន
ុ គ្នាតាមលក្ខខណឌទី២ ជ.ម.ជ គឺៈ
មុម
ំ យ
ួ ប៉ន
ុ គ្នា អមតោយរជុងពរី ប៉ន
ុ តរៀងគ្នា។ (តរពាះ
ABC
ររីតោណ ACD និង CBE មាន៖
AC 2  AB 2  BC 2
-រជុង AD = CE បរមាប់របធាន
AC 2  22  42  20
-មុំ CAD  BCE មុនំ នររតី ោណសម័ងស
AC  2 5 cm , AC  0 -រជុង AC = CB រជុងននររីតោណសម័ងស)

cm CD 281) គូសអងករដ់ ែលមានរបដវង 5 cm ក្ាុងរូប៖


CDE
CD  ED  CE
2 2 2

AC 2 5
CE    5 cm
2 2
តរពាះតាមរទស
ឹ ីប
ត ទពតា
ី គ័រ 3  4  5
2 2 2
2
CD  5  5  20
2 2

CD  20 , CD  0 282) ក្. តពលអំពូលទាំងអស់តឆះភលឺ វាជំនស


ួ ឱ្យតលខអវី?

តពលអំពូលទាំងអស់តឆះភលឺ វាជាបនសំននអំពូលចាប់ពី
CD  2 5 cm
ី ែលទី៦ តនះតលខដែលវាររូវជំនស
រូបទ២ ួ គឺ៖
ABC CDE 1  2  4  8  16  31
S  SABC  SCDE
ំ ស
ខ. គូសរូបននអំពូលទាំងតនះជន ួ ឱ្យតលខ 10
1 1
  AB  BC    CE  CD  ក្ាុងចំតណាមចំនន
ួ ពី 1, 2, 4, 8, 16 មានជតរមស
ើ ដរមួយ
2 2
1
  2  4 
2
1
2

52 5  គរ់ ដែលបូក្តសមើ 10 គឺ 2  8  10 ររូវជាបនសំននអំពូល
ក្ាុងរូបទី 3 និងរូបទី 5 ែូចតនះតយើងគូសរូបបាន៖
 4  5  9 cm 2

349

283) ១) តរើគូររីតោណណា ដែលគួបង្ហាញថាវាប៉ន


ុ គ្នា? 285) គូសបន្ទារ់ដែលមានរបដវង 10 cm ក្ាុងរូប
A M D

B N C
គូររតី ោណដែលគួបង្ហាញថាវាប៉ន
ុ គ្នាគៈឺ

MBD នង
ិ NDB ។

២) តរជស
ើ តរសលក្ខ
ើ ខណឌទាំងបដី ែលររម
ឹ ររូវៈ តរពាះតាមរទឹសី តបទពីតាគ័រ 6  8  10
2 2 2

អក្សរដែលររូវនឹងលក្ខខណឌទាំងបរី រម
ឹ ររូវគៈឺ
286) រក្ចំនន
ួ ែង ដែលរបូបាបតក្រ
ើ មានត ង
ើ ននរពរ
ឹ ោ
តិ រណ៍
ខ, គ និង ច ។ A តរចន
ើ ជាងរបូបាបននរពឹរតិោរណ៍ B

៣) ពនយល់ជាពាក្យ នូវលក្ខខណឌប៉ន
ុ គ្នាននររតី ោណ -របូបាបននរពឹរតិោរណ៍ A គឺ P  A ដែលៈ
ររីតោណទាំងពីរប៉ន
ុ គ្នាតាមលក្ខខណឌទី២ ជ.ម.ជ គឺៈ C  3 , 3 1
P  A  
មុម
ំ យ
ួ ប៉ន
ុ គ្នា អមតោយរជុងពីរប៉ន
ុ តរៀងគ្នា។ (តរពាះ C  9 , 3 84
ររតី ោណ MBD នង
ិ NDB មាន៖
-របូបាបននរពរ
ឹ ោ
តិ រណ៍ B គឺ P  B  ដែលៈ
-រជុង DM  BN ជាពាក្់ក្ណា
ត លរជុងរសបគ្នា C  3 , 3 1
P  B  
-មុំ MDB  NBD មុឆ្ល
ំ ល ស់ក្ុ ង
ា C 10 , 3 120
-រជុង BD  DB ជារជុងរួម) 1
P  A 120 10
-ន្ទំឱ្យចំនន
ួ ែង  84   ែង
284) ១) តរើគូររីតោណណា ដែលគួបង្ហាញថាវាប៉ន
ុ គ្នា? P  B 1 84 7
A D
120

M
287) ក្. សរតសរមាឌននគូប B ជាអនុគមន៍នន a នង
ិ b
B C តោយមាឌននគូប B មានរបដវងរទនុងគឺ a b
គូររតី ោណដែលគួបង្ហាញថាវាប៉ន
ុ គ្នាគៈឺ ន្ទំឱ្យ មាឌ VB   a  b 
3

AMD នង
ិ BMC ។ ែូចតនះ VB  a3  3a 2b  3ab 2  b3 ។

២) តរជើសតរសលក្ខ ខណឌទាំងបដី ែលររម ខ. រក្ចំនន


ួ ែងននមាឌគូប B តរចន
ើ ជាងគូប A
ើ ឹ ររូវៈ
តោយគូប A មានមាឌគឺ VA  a
3
អក្សរដែលររូវនឹងលក្ខខណឌទាំងបីររឹមររូវគឺៈ
តយើងបានផលត ៀបរវាងមាឌគូប B និងគូប A គឺៈ
ក្, គ និង ង ។
VB a 3  3a 2b  3ab 2  b3

៣) ពនយល់ជាពាក្យ នូវលក្ខខណឌប៉ន
ុ គ្នាននររីតោណ VA a3
2 3
ររីតោណទាំងពីរប៉ន
ុ គ្នាតាមលក្ខខណឌទី២ ជ.ម.ជ គឺៈ b b b
 1 3  3     
មុម
ំ យ
ួ ប៉ន
ុ គ្នា អមតោយរជុងពរី ប៉ន
ុ តរៀងគ្នា។ (តរពាះ a a a
b
ររីតោណ MBD និង NDB មាន៖ ើ ជំនស
តយង ួ t ន្ទំឱ្យតយង
ើ បានៈ
a
-រជុង AD  BC រជុងរសបគ្នាតសមើគ្នា VB
-មុំ MAD  MBC  90o មុដំ ក្ងែូចគ្នា  1  3t  3t 2  t 3
VA
-រជុង AM  BM , M ក្ណា
ត លរជុង AB

350

288) បង្ហាញថា x  y  z  a តរពាះ x  y  z  3a និង x  y  z  3a


2 2 2 2

ចំតពាះ
ពិនិរយ  x  a   y  a   z  a
2 2 2

 x 2  2ax  a 2  y 2  2ay  a 2  z 2  2az  a 2 x  a  0 x  a


 
 x 2  y 2  z 2  2ax  2ay  2az  3a 2 តយើងបាន  y  a  0 ន្ទំឱ្យ  y  a
z  a  0 z  a
 x 2  y 2  z 2  2a  x  y  z   3a 2  
 3a 2  2a  3a  3a 2 ែូចតនះ តយង
ើ បញ្ជាក្់បានថា x yza ។
0

351
sYsþI¡ elakGñkmitþGñkGanCaTIRsLaj;rab;Gan enAkñúgEpñkenHelakGñknwg)aneXIj BIvBi aØasaFøab;
ecjRbLgDIsbøÚm P¢ab;CamYycemøIyRKb;qñaM KWcab;BIqñaM 1981 rhUtdl;qñaM 2012 . CaFmµtaviBaØasa nig
cemøIyrbs;va énqñaMnImYy² RtUv)anerobcMCa 4 TMBr½ dUcenHebIviBaØasaxøHxøI eFVI[sl;TMB½rTMenr.
elakGñkGaceFVIkarkMNt;TuknUvlkçN³énviBaØasaEdl)anecjRbLgrYcehIyenH edIm,IeFVIkar):an;sµan
nUvviBaØasaEdlnwgecjCabnþbnÞab;enAqñaMxagmux.
RbsinebIelakGñkmanbBaða b¤cm¶l;Rtg;cMNucNa EdlmanenAkñúgEpñkenH elakGñkGacTak;Tg;eTAkan;
RKU b¤mitþPkþirbs;GñkEdlmansmtßPaB b¤GñkeroberogesovePAenHkñúgeBlevlasmKYr .

v
9

sm½yRbLg ³ 06 kkáda 1981


viBaØasa ³ KNitviTüa ry³eBl ³ 60 naTI BinÞú ³ 10

I. BICKNit
1> rktémø a edIm,I[RbPaK F  3a3a 0.213a2a 26 ³
k> esµInwgsUnü .
x> Kµann½y .
2> edaHRsayRbB½n§smIkartamRkaPic  yy  2xx4500 12 .

3> mnusSmñak;Gayu 51 qñaM ehIykUnrbs;Kat;manGayu 21 qñaM . etIb:unµanqñaMeTAmuxeTot eTIbGayu«Buk
esµInwg 2 dgGayukUn .

II. FrNImaRt
1> rgVg;BIrmanp©ti O nig O kat;KñaRtg; A nig B . eKKUsGgát;p©it AC  kat;tam O nigGgát;p©it
AD kat;tam O . RsaybMPøW[eXIjfa cMNucTaMgbI C , B nig D sßitenAelIbnÞat;EtmYy
ehIybnÞat; AB   CD .
2> MN CargVas;BI)atdIdl;páaQUk nigEpñkput N
A B
BITwk AN  10 cm . eRkamGMNacxül;bk;
mYy enaHedImQUkeRTtRbkan;yksßanPaB
MB ¬emIlrUb¦ eday AB  30 cm
M
rkCeRmATwk MA .
ENnaM ³ kñúgRtIekaNsm)at MNB KUskm<s;ecjBIkMBUl rYceRbóbeFobRtIekaN .

352
9

cemøIy
I. BICKNit
y  2x  5  0
1> rktémø a edIm,I[RbPaK F ³
k> esµInwgsUnü
eyIgman F  3a3a 0.213a2a 26 ³
3a  0.07  2a  3 y x40

3a  3a  2
2a  0.07a  3 tamRkaPic ³ cMNucRbsBVénbnÞat;TaMBIrKW 3,1

a  3a  2
eyIgGacKNna F )ankalNavamann½y dUcenH RbB½n§smIkarmanKUcemøIy x  3, y  1 .
ehIyvamann½yluHRtaEtPaKEbgxusBIsUnü ³ 3> rkcMnYnqñaMeTAmuxeTot
naM[ aa  32  00 b¤ aa  23 tag x CacMnYnqñaMeTAmuxeTot EdlRtUvrk
 
-edIm,I[ F  0 luHRtaEt PaKykrbs;vaesµIsnU ü tambRmab;RbFan eyIgsresr)ansmIkar ³
51  x  221  x 
eyIg)an 2a  0.07 a  3  0 51  x  42  2 x
51  42  2 x  x
naM[ aa  30.070 0 b¤ aa  30.07 x9
 

dUcenH ebI F  0 enaHtémø a  0.07 , a  3 . epÞógpÞat; ³ 51  9  221  9  60  60 Bit


x> Kµann½y dUcenH cMnYnqñaMteTAmuxKW ³ 9 qñaMeTot .
RbPaK F Kµann½ykalNa PaKEbgrbs;vaesµI 0 II. FrNImaRt
naM[ aa  32  00 b¤ aa  23 1> RsayfacMNuc C , B nig D enAelIbnÞat;EtmYy
  A

dUcenH ebI F Kµann½y enaH a  3 , a  2 .  


O
O
2> edaHRsayRbB½n§smIkartamRkaPic ³ C B D

eyIgman  yy  2xx4500 12 eday ABˆC  90 ¬mMucarwkknøHrgVg;Ggát;p©it AC ¦


o


ABˆ D  90 ¬mMucarwkknøHrgVg;Ggát;p©it AD ¦
o

eyIgeRbItaragtémøelxedIm,Isg;bnÞat;TaMgenH ³
x 1 2
naM[ ABˆC  ABˆD  CBˆD  90  90  180 o o o

y  2x  5  0
y  3 1 eXIjfa CBˆ D  180 CamMurab
o

x 2 3
y x40
y 2 1
dUcenH cMNcu C , B nig D enAelIbnÞat;
eyIgsg;RkaPic)an ³ EtmYy ehIy AB   CD .
353
9

2> rkCeRmATwk MA smÁal; ebIGñkmineFVItamkarENnaMRsYlCagKW ³


tamENnaMeyIgKUsrUbbEnßm)an ³ edayépÞTwkEkgnwgpáaQUkQr
N H
naM[ ABM EkgRtg; A ehIytamBItaK½r ³
A B MB2  AB2  AM 2 >
Et MB  MN  AM  AN ¬edImQUkEtmYy¦
eyIg)an  AM  AN   AB  AM
2 2 2

M
eday AN  10cm , AB  30cm
eyIgeRbóbeFob ABN nig HMN enaH  AM  10   30  AM
2 2 2

AM 2  20 AM  100  900  AM 2
eday ABN nig HMN man ³ 20 AM  800
-mMu BAˆ N  MHˆ N  90 eRBaH épÞTwkEkgnwg
o

AM 
800
 40 cm
páaQUkQr nig MH Cakm<s;RtUvnwg)at BN 20

-mMu ANˆ B  HNˆ M ¬mMurYm¦ dUcenH CeRmATwkKW AM  40cm .


dUcenH ABN HMN tamlkçxNÐ m>m
vi)ak ABNHMN

AN BN
HN MN

Taj)an MN  BNAN HN 1


eday AN  10 cm
-tamRTwsþIbTBItaK½rcMeBaH ABN EkgRtg; A
BN 2  AN 2  AB2 ,  AB  30 cm
 102  302  100  900  1000
BN  1000  10 10 cm
-km<s; MH RtUvnwg)at BN énRtIekaN
sm)at MNB naM[ MH k¾CaemdüanEdr
naM[ HN  12 BN  12 10 10  5 10 cm
tam 1 eyIg)an MN  10 1010 5 10
 50 cm

naM[ CeRmATwk AM  MN  AN
 50cm 10cm  40cm >
dUcenH TwkmanCeRmA AM  40cm .

354
9

355
9

sm½yRbLg ³ 29 mifuna 1982


viBaØasa ³ KNitviTüa ry³eBl ³ 60 naTI BinÞú ³ 10

I. BICKNit
1> dak;CaplKuNktþa abx  y   xya  b  .
2 2 2 2

2> RkumsamKÁIbgábegáInplmYy lk;RsUvCUnrdæTaMgGs;cMnYn 0.825 t . RsUvenaHmanbIRbePTKW páaxJI


nagkuk nigRsUvBeRgaH. témø 1 kg RsUvnagkukesµIBak;kNþaltémøsrubén 1 kg RsUvpáaxJI nig
1 kg RsUvBeRgaH ehIyéføelIstémø 1 kg énRsUvBeRgaHcMnYn 0.50 erol .

k> témø 1 kg RsUvpáaxJI 1 kg RsUvnagkuk nig 1 kg RsUvBeRgaHrYmKñaéfø 3.00 erol . KNnatémø


1 kg énRbePTRsUvnImYy² .

x> eKdwgfaRkumsamKÁIlk;RsUvnagkuk 250 kg nig RsUvpáaxJI RBmTaMgRsUvBeRgaHTaMgGs;)anR)ak;


837.50 erol. KNnaTm¶n;RsUvpáaxJI nigRsUvBeRgaH .

II. FrNImaRt
eK[RtIekaN ABC carwkkñúgrgVg;. knøHbnÞat;BuH BAC kat;RCug BC  Rtg; I nigFñÚ BC Rtg; J .
1> RsaybBa¢ak;fa AIB nig AJC dUcKña .
2> KNnaplKuN AI  AJ .
3> bgðajfa BAJ nig CBJ b:unKña .
4> RsaybBa¢ak;fa JB  JA  IJ .
2

356
9

cemøIy
I. BICKNit dUcenH témøRsUvkñúg 1kg Edlrk)anKW ³
1> dak;CaplKuNktþa ³ RtUvpáaxJI mantémø 1.50 erol
 
ab x 2  y 2  xy a 2  b 2  
 abx  aby  a xy  b xy
2 2 2 2 RtUvnagkuk mantémø 1.00 erol
  
 abx 2  a 2 xy   b 2 xy  aby 2  RtUvBeRgaH mantémø 0.50 erol
 axbx  ay   bybx  ay 
 bx  ay ax  by 
x> KNnaTm¶n;RsUvpáaxJI nigRsUvBeRgaH
tag x CaTm¶n;RsUvpáaxJI ¬KitCa kg ¦
dUcenH dak;CaplKuNktþa)an
y CaTm¶n;RsUvBeRgaH ¬KitCa kg ¦
  
ab x 2  y 2  xy a 2  b 2 
 bx  ay ax  by  tambRmab;RbFaneyIg)an ³
 x  250  z  825
2> k> KNnatémø 1 kg énRbePTRsUvnImYy² ³ 
1.5x  1 250  0.5z  837.5
tag a CatémøRsUvpáaxJI 1kg ¬KitCaerol¦ ¬eRBaHTm¶n;RsUv 0.825 t  825kg ¦
b CatémøRsUvnagkuk 1kg ¬KitCaerol¦  x  z  825  250

c CatémøRsUvBeRgaH 1kg ¬KitCaerol¦ 1.5 x  0.5 z  837.5  250
 x  z  575
- témø 1 kg RsUvnagkukesµIBak;kNþaltémøsrub 
1.5 x  0.5 z  587.5
én 1 kg RsUvpáaxJI nig 1 kg RsUvBeRgaH  x  z  575 1

naM[ b  12 a  c b¤ a  c  2b 1 3 x  z  1175 2 

- ehIytémø 1 kg RsUvnagkuk éføelIstémø 1 kg eyIgdkGgÁngi GgÁénsmIkar 2  1 ³


3x  z  1175

énRsUvBeRgaHcMnYn 0.50 erol  x  z  575
naM[ b  c  0.5 2 2 x  600

- témø 1 kg RsUvpáaxJI 1 kg RsUvnagkuk nig naM[ x  300

1 kg RsUvBeRgaHrYmKñaéfø 3.00 erol


cMeBaH x  300 CMnYskñúg 1 ³
1 : x  z  575
naM[ a  b  c  3 3 z  575  x
tam 1 , 2 nig 3 eK)anRbB½n§smIkar ³  575  300  275

a  c  2b

a  c  2b

a  c  2b

epÞógpÞat; 300  250  275  825
b  c  0.5  b  c  0.5  b  c  0.5
a  b  c  3

a  c   b  3

2b  b  3

825  825 Bit
dUcenH RsUvpáaxJI manTm¶n; 300 kg
a  c  2b a  c  2  1 a  0.5  2

 b  c  0.5

 1  c  0.5

 c  0.5 RsUvBeRgaH manTm¶n; 275 kg .
3b  3 b  1 b  1
  

357
9

II. FrNImaRt vi)ak ABJ 


JB JA

B
BIJ JI JB
J
I
Taj)anBIpleFob JB  JB  JA JI
dUcenH RsaybBa¢ak;)anfa JB  JA  IJ .
2

A C

1> RsaybBa¢ak;fa AIB nig AJC dUcKña


eday AIB nig AJC man ³
-mMu ABˆ I  AJˆC ¬mMucarwkmanFñÚsáat;rYm AC ¦
-mMu BAI  JAC ¬ AJ knøHbnÞat;BuH BAC ¦
dUcenH AIB AJC tamlkçxNÐ m>m .
2> KNnaplKuN AI  AJ
eday AIB AJC ¬sRmayxagelI¦
eK)an AIB ACJ

AI AB

AC AJ
Taj)an AI  AJ  AB AC
dUcenH KNna)anplKuN AI  AJ  AB AC .
3> bgðajfa BAJ nig CBJ b:unKña
eday BAˆ J  CAˆ J ¬ AJ knøHbnÞat;BuH BAC ¦
Et CAˆ J  CBˆ J ¬mMucarwkmanFñÚsáat;rYm CJ ¦
naM[ BAˆ J  CAˆ J  CBˆ J 1
dUcenH bgðaj)anfa BAJ  CBJ .
4> RsaybBa¢ak;fa JB  JA  IJ
2

eyIgeRbóbeFob ABJ nig BIJ


eday ABJ nig BIJ man ³
-mMu BAˆ J  IBˆ J eRBaH
tam 1 : BAˆ J  CAˆ J  CBˆ J  IBˆ J
-mMu AJˆB  BJˆI ¬mMurYm¦
dUcenH ABJ BIJ tamlkçxNÐ m>m
358
9

359
9

RksYgGb;rM yuvCn nigkILa elxbnÞb; ³ >>>>>>>>>>>>>>>>>>>>>>>>>>>>>>>>>>>>>>>


RbLgsBaØabRtmFümsikSabzmPUmi cMeNHTUeTA nigbMeBjviC¢a elxtu ³ >>>>>>>>>>>>>>>>>>>>>>>>>>>>>>>>>>>>>>>>>>>>
eQµaH nightßelxaGnurkS sm½yRbLg ³ >>>>>>>> >>>>>>>>> 1993 mNÐlRbLg ³ >>>>>>>>>>>>>>>>>>>>>>>>>>>>>>>>>>
1> >>>>>>>>>>>>>>>>>>>>>>>>>>>>>>>>>>>>>> namRtkUl nignamxøÜn ³ >>>>>>>>>>>>>>>>>>>>>>>>>>>>>>>>>
2> >>>>>>>>>>>>>>>>>>>>>>>>>>>>>>>>>>>>>> éf¶ExqñaMkMeNIt ³ >>>>>>>>>>>>>>>>>>>>>>>>>>>>>>>>>>>>>>>>>>> GkSrsm¶at;
htßelxa ³ >>>>>>>>>>>>>>>>>>>>>>>>>>>>>>>>>>>>>>>>>>>>>>>>>>>
 ebkçCnminRtUveFVIsBaØasmÁal;Gm VI YyelIsnøwkRbLgeLIy. snøwkRbLgNaEdlmansBaØasmÁal;RtUv)anBinÞúsUnü .
--------------------------------------------------------------------------------------------------------------------------
viBaØasa ³ KNitviTüa ry³eBl ³ 120 naTI BinÞú ³ 100
esckþIENnaM ³ GkSrsm¶at;
1> ebkçCnRtUvbt;RkdasenHCaBIr rYcKUsExVgEpñkxagelIénTMB½rTI2 [b:unRbGb;EpñkxagelI
énTMB½rTI1 EdlRtUvkat;ecal. hamsresrcemøIyelIkEnøgKUsExVgenaH .
2> ebkçCnRtUvKUsbnÞat;bBaÄr[cMBak;kNþalTMB½rTI2 nigTMB½rTI3 sRmab;sresrcemøIybnþ.

I. ¬10 BinÞú¦

360
9

cemøIy

361
9

sm½yRbLg ³ 03 kkáda 1984


viBaØasa ³ KNitviTüa ry³eBl ³ 60 naTI BinÞú ³ 10

I. BICKNit
1> KNna a  b  ca  b  c  ab  ac  bc .
2 2 2

2> edaHRsaysmIkar 2x  3x  3x  2  4xx  1  x  11  x .


3> sRmÜlkenSam aabb cc . rYcKNnatémøelxénkenSamenH kñúgkrNIEdl ³
2 2

a  35.4 , b  48.6 nig c  29.6 .

II. FrNImaRt
1> eK[BIcMNuc O nig O . eKKUsrgVg;p©it O nig O EdlmankaM OO ehIykat;KñaRtg; A nig
B . bnÞat;mYykat;tam A kat;rgVg; O  Rtg; I nigkat;rgVg; O  Rtg; J .

k> bgðajfaRtIekaN AOO nig BOO CaRtIekaNsm½gS .


x> R)ab;RbePTRtIekaN BIJ .
2> eK[RbelLÚRkam ABCD BC  //AD . eKKUsrgVg;mYykat;tam A nig B rgVg;enHkat;
RCug AD Rtg; P nig BC  Rtg; Q . RsaybMPøWfa ctuekaN CDPQ carwkkñúgrgVg;EtmYy .
3> eK[rgVg;p©it O nigGgát;FñÚ AB. C CacMNucmYyelIFñÚtUc AB . eKbnøay AC  [)anGgát;
CM  Edl CM   CB . rkcMNuc M kalNa C rt;elIFñÚtUc AB .
smÁal; ³ sMNYrenH minTak;TgKñaeT .

362
9

cemøIy
I. BICKNit II. FrNImaRt
I A
1> KNna J
a  b  c a 2
 b  c  ab  ac  bc
2 2
 O
 
 a  ab  ac  a b  a c  abc 
3 2 2 2 2
O
a b  b  bc  ab  abc  b c 
2 3 2 2 2

a 2 c  b 2 c  c 3  abc  ac 2  bc 2 B
 a 3  b 3  c 3  3abc 1>k> bgðajfa AOO nig BOO Ca  sm½gS
rgVg;TaMgBIrmankaM OO EtmYy enaHrgVg;TaMgBIr
2> edaHRsaysmIkar ³ CargVg;b:unKña
2 x  3 x  3x  2   4 xx  1  x  11  x 
2 x  3x 2  6 x  3x  6  4 x 2  4 x  1  x 2 eday AOO man OA  OO  OA
 x2  7x  6  1 x2 ¬CakaMrgVg;BIrb:unKña¦
 x2  x2  7x  1 6
7 x  5 dUcenH AOO CaRtIekaNsm½gS .
5
x
7 ehIy BOO man OB  OO  OB
dUcenH smIkarman x   75 Cab£s . ¬CakaMrgVg;BIrb:unKña¦
dUcenH BOO CaRtIekaNsm½gS .
3> sRmÜlkenSam a  b 2  c 2
abc x> R)ab;RbePTRtIekaN BIJ
a  b  c  a  b  c a  b  c 
2 2

abc abc eday AOO nig BOO CaRtIekaNsm½gS


 abc naM[ AOˆ B  AOˆ O  BOˆ O  60  60  120
o o o

Edl kenSam a bc  0


AOˆ B  AOˆ O  BOˆ O  60 o  60 o  120 o

dUcenH sRmYl)an a  b 2  c 2  abc . RtIekaN BIJ man ³


abc
ˆ
-mMu IJB  AJB  AO2B  1202
o
 60 o
-KNnatémøelxénkenSamenH
- mMu JIB  AIB  AO2ˆ B  1202  60
o

cMeBaH a  35.4 , b  48.6 nig c  29.6 o

eday aabb cc  a  b  c ¬mMucarwkelIrgVg;esµkI nøHmMup©ti EdlmanFñÚsáat;rYm¦


2 2

 35.4   48.6  29.6 eXIjfa BIJ manmMuBIr IJB  JIB  60 o

 42.8
naM[ BIJ CaRtIekaNsm)at EdlmMu)atesµI 60 o

dUcenH témøKNna)an aabb cc


2 2
 42 .8 . dUcenH RtIekaN BIJ CaRtIekaNsm½gS .
363
9

2> RsaybMPøWfa ctuekaN CDPQ carwkkñúg naM[ CMB CaRtIekaNsm)at


rgVg;EtmYy vi)ak CBˆ M  CMˆ B ¬mMu)atRtIekaNsm)at¦
A P D ehIy CBˆ M  CMˆ B  ACˆB ¬mMueRkA CMB ¦
B
b¤ 2CMˆ B  ACˆB naM[ CMˆ B  AC2ˆB
C 
Q
Et ACˆ B 
AB
¬mMucarwkFñÚsáat;FM AB ¦
-bRmab; ABCD CaRbelLÚRkam BC //AD 2

AB
naM[ ABˆ Q  PDˆ C 1 ¬mMuQménRbelLÚRkam¦ eyIg)an ˆ
CMB 
ACˆB
 2 

AB
ehIy BC //AD man PQ Caxñat; 2 2 4

enaH APˆ Q  PQˆ C 2 ¬mMukñúgrYmxag¦ b¤Gacsresr CMˆ B  AMˆ B 


AB
4
-rgVg;mYykat; A nig B ehIyrgVg;enHkat;RCug eday AB efr enaH FñÚFM AB k¾efrEdr
AD Rtg; P nig BC  Rtg; Q ehIy C rt;elIFñÚtUc AB enaH M k¾rt;elIFñÚén
mann½yfa A , B , Q, P enAelIrgVg;EtmYy rgVg;mYyRbkbedaymMuefrKW AMˆ B 
AB

naM[ ctuekaN ABQP carwkkñgú rgVg;mYy 4

vi)ak plbUkmMuQm ABˆ Q  APˆ Q  180 3 o


-ebI C RtYtelI A enaH M RtYtelI M 
tam 1, 2 , 3 eyIg)an ³ Edl AB  AM 
ABˆ Q  PDˆ C
-ebI C RtYtelI B enaH M RtYtelI B Edr
APˆ Q  PQˆ C  PDˆ C  PQˆ C  180 o mann½yfa M rt;elIFñÚ BM  énrgVg;mYy
ABˆ Q  APˆ Q  180 o
dUcenH kalNa C rt;elIFñÚtUc AB enaH M
eday ctuekaN CDPQ manplbUkmMuQm CasMNMucMNucrt;elIFñÚ BM  énrgVg;mYy
PDˆ C  PQˆ C  180 enaHvacarwkkñúgrgVg;
o

RbkbedaymMuefr AMˆ B 
AB
.
dUcenH ctuekaN CDPQ carwkkñgú rgVg;EtmYy . 4

3> rkcMNuc M kalNa C rt;elIFñÚtUc AB


M

M
C
A //
B
O

tambRmab;RbFan CM   CB

364
9

365
9

sm½yRbLg ³ 25 mifuna 1985


viBaØasa ³ KNitviTüa ry³eBl ³ 60 naTI BinÞú ³ 10

I. BICKNit
1> k> dak;kenSamxageRkamCaplKuNktþadWeRkTI1 én x ³
A  x  3x  2  9x  3
2

B  2 x  4  x  1
2 2

x> KNnatémøelxén C  BA cMeBaHtémø x  3 , x  1 nig x  53 .


K> etI D  x 31xx5 5 nig C smmUlKña b¤eT ?
2> k> tagRkaPicénTMnak;TMngxageRkam kñúgtRmúyGrtUNremedayyk 1 cm CaÉkta ³
y  2 x  4 nig y  x  1 .

x> edaHRsayRbB½n§smIkar  yy  2x x1 4 . yklT§plxagelIedIm,IedaHRsayRbB½n§smIkar



 1 2
 y  1  2 x  5  4
 1 1
.
  1
 y  1 2 x  5

3> k> KNna A  10 48  6 108  4 12 nig B


1

1
11  3 11 11  3 11
.
x> edaHRsaysmIkar 2 x3  7  x  1 .
II. FrNImaRt
manrgVg; O p©it O Ggát;p©it AB. O CacMNuckNþalén  AB nigCap©itrgVg; O Edl
kat;tam A nig B . M nig N CacMNucénFñÚFM  AB rbs;rgVg; O . bnÞat; MA , MB  CYbrgVg;
O Rtg; C nig D ehIy NA nig NB CYbrgVg; O Rtg; E nig F .
1> RbdUcRtIekaN MAD nig NBE .
2> bgðaj[eXIjfa MAD nig NBE CaRtIekaNEkgsm)at .
3> cMNuc M cl½telIFñÚFM  AB énrgVg; O bgðajfa CD manrgVas;efr ehIyEdlnwgRtUv
KNnaCaGnuKmn_én R CargVas;kaMrgVg;Ggát;p©it AB. rksMNMucMNuckNþal I én CD .

8
366
9

cemøIy
I. BICKNit K> etI D  x 31xx5 5 nig C smmUlKña b¤eT ?
1> k> dak;kenSamCaplKuNktþadWeRkTI1 én x eyIgman C  x x33x35xx5 1
A  x  3x  2  9x  3
2


 x  3x  2  9
2
 eXIjfa ebIsRmYl C nwg x  3 enaH C dUc D
 x  3x  2  3x  2  3
 x  3x  5x  1
dUcenH -ebI x  3 enaH C nig D smmUlKña
-ebI x  3 enaH C nig D minsmmUlKña .
dUcenH A  x  3x  5x  1 .
B  2 x  4  x  1
2 2 2> k> tagRkaPicénTMnak;TMngxageRkam ³
 2 x  4  x  12 x  4  x  1 eyIgeRbItaragtémøelxedIm,Isg;bnÞat;
 x  33x  5 y  2x  4 nig y  x 1

dUcenH B  x  33x  5 . x
y 4 2
0 1 x
y 1 2
0 1

x> KNnatémøelxén C  BA
eK)an C  BA  x x33x35xx5 1 y  x 1

-cMeBaH x  3
C
3  33  53  1  0   2  4  0
3  33  3  5 04 0

dUcenH C mancemøIyeRcInrab;minGs; . y  2x  4

-cMeBaH x  1 x> edaHRsayRbB½n§smIkar  yy  2x x1 4


C
 1  3 1  5 1  1   4 60  0 
 1  3 1  3  5  4 8 eyIgedaHRsayeday dkGgÁnigGgÁ
dUcenH mancemøIyEtmYyKt;KW 0 .  y  2x  4
C 
eyIg)an y  x 1 naM[ x 1
-cMeBaHx
5 3x  3
3
 5  5  5    4   10  8 
cMeBaH x  1
  3   5   1    
C  3  3  3   3  3  3 
 naM[ y  x  1  y  1 x  11  2
 5  5  4
  3  3   5   0
 3  3   3 
dUcenH RbB½n§smIkarmanKUcemøIy
manPaKyk  0 ÉPaKEbg  0 . x 1 , y  2 .
dUcenH C Kµann½y kñúgkarKNnatémøelx . ¬ebIepÞógpÞat;tamRkaPicKWRtUvKña ¦
367
9

K> yklT§plxagelIedIm,IedaHRsayRbB½n§ x> edaHRsaysmIkar ³


 1 2 2 x 7
 y  1  2 x  5  4  x 1
smIkar ³  1 1
3
  1 2 x 7  3 x 3
 y  1 2 x  5
 1 1 4 x
 y  1  2  2 x  5  4 edayRKb;cMnYnBit x enaH x  0 Et  4  0
Gacsresr  1 1
  1
 y  1 2 x  5 dUcenH smIkarKµanb£sCacMnYnBiteT .
tag Y  y 1 1 nig X  2x1 5
II. FrNImaRt
Edl y  1 , x  52 M N

eyIg)anRbB½n§fµI YY2XX 14


 O  D
E
tamlT§plxagelIKW X  1 , Y  2 C
I
F
naM[ Y  y 1 1  2  y 1 1  y   12 A 
B
O
1 1 I I 
X  1  x3
2x  5 2x  5
O 
dUcenH RbB½n§smIkarmanKUcemøIy
1> RbdUcRtIekaN MAD nigRtIekaN NBE
x3 , y 
1
2
. edayRtIekaN MAD nigRtIekaN NBE man ³
3> k> KNna -mMu AMˆ B  ANˆB ¬mMucarwkmanFñÚsáat;rYm AB
A  10 48  6 108  4 12 énrgVg;p©it O ¦
 40 3  36 3  8 3 -mMu ADˆ M  BEˆN  90 eRBaH CamMuCab;bEnßm
o

 12 3 énmMu ADˆ B , AEˆB EdlCamMucarwkknøHrgVg;p©it O


dUcenH KNna)an A  12 3 . dUcenH MAD NBE tamlkçxNÐ m>m .
1 1
B 
11  3 11 11  3 11 2> bgðaj[eXIjfa MAD nig NBE Ca

11  3 11 11  3 11 RtIekaNEkgsm)at
11  3 1111  3 11 kñúgRtIekaN MAD nig NBE man ³
22

11  9 11
2 -mMu ADˆ M  BEˆN  90 ¬manbBa¢ak;xagelI¦
o

ˆ
-mMu AMˆ B  ANˆ B  AO2B  902  45 eRBaH
22 o
 1 o
22

dUcenH KNna)an B 1 . kñúgrgVg;p©it O man AMˆ B , ANˆ B CamMucarwk ehIy


368
9

AOˆ B
2
R 2
mMu
AOˆ B CamMup©it enaH AMˆ B  ANˆ B 
2

OI  R  
2 2

R2 R2
 R  2  2
2

 2 
mü:ageTot AOˆ B  90 edaysarvaCamMucarwk
o
R2 R R 2
OI   
knøHrgVg;EdlmanGgát;p©it AB . 2 2 2
eday O CacMNucnwg enaH sMNuMcMNuc I rt;enA
dUcenH MAD nig NBE CaRtIekaNEkgsm)at
elIrgVg;kaM OI
3> bgðajfa CD manrgVas;efr Et M cl½tenAelIFñÚFM AB
edaymMu AMˆ B  45 ehIyCamMueRkArgVg;p©it O
o 
Edl FñÚFM AB  360  FñÚtUc AB o

 
AB CD 
naM[ AMˆ B 
2
¬ FñÚtUc AB  mMup©it AOˆ B  90 ¦ o


 
CD  AB 2 AMˆ B naM[ FñÚFM AB  360  90  270 o o o

 180o  2  45o  90o eday M cl½tenAelIFñÚFM 270 enaH I k¾cl½t o


eday CD  90 efr enaHnaM[ CD k¾efrEdr
o
enAelIFñÚ I II   270 Edr o

dUcenH bgðaj)anfa CD manrgVas;efr . -ebI M RtYtelI A enaH I RtYtelI I 


-ebI M RtYtelI B enaH I RtYtelI I 
-KNna CD CaGnuKmn_én R

eday CD  90 enaH mMup©it COˆ D  90 Edr
o o dUcenH sMNcuM MNuc I rt;enAelIFñÚ I II   270 o

naM[ RtIekaN COD CaRtIekaNEkgRtg; O énrgVg; O mankaM OI  R 2 2 .


tamRTwsþIbTBItaK½r CD  OC  OD 2 2 2

eday OC  OD  R CakaMrgVg;Ggát;p©it AB rUbsRmÜlkaremIl


naM[ CD  R  R  2R
2 2 2 2 M N

CD  2 R 2  R 2 /
O  D
dUcenH KNna)an CD  R 2 . E
I
F
C
- rksMNMucMNuckNþal I én CD A 
B
O
eday I CacMNuckNþal CD I I 

naM[ OI  CD ¬kaMrgVg;EkgnwgGgát;FñÚRtg; O 
cMNuckNþal ¦
ehIy CD  R 2 efr enaH OI k¾efrEdr
2

Edl OI  OC   CD
2

2 


2

369
9

sm½yRbLg ³ 07 kkáda 1986


viBaØasa ³ KNitviTüa ry³eBl ³ 60 naTI BinÞú ³ 10

I. BICKNit
1
a b 
1> sRmÜlr:aDIkal; 1
ab Edl 0ba .
1
a2  b2
2> eK[kenSam A  5x  4  x  4x  4  6  3xx  3 .
2 2

k> sresr A CaBhuFabRgÜm nigerobtamsV½yKuNcuHén x .


x> sresr A CaplKuNktþadWeRkTI 1 .
K> edaHRsaysmIkar A  0 nig A  2 .
X> eK[RbPaK F  3x  1A x  4 . rktémø x EdlnaM[RbPaKmann½y rYcsRmÜlRbPaK.
3> kñúgtRmúyGrtUNrem ¬Éktþa sg;TIEm:t¦
k> sg;bnÞat;rbs;GnuKmn_ y  x  2 D  nig y   x  4 D  .
1 2

x> kMNt;kUGredaenéncMnucRbsBV D  nig D  tamkarKNna rYctamRkaPic .


1 2

K> bgðajfa D  nig D  EkgKña .


1 2

II. FrNImaRt
eK[rgVg;p©it O Ggát;p©it AB. elIbnÞat;Edlb:HrgVg;Rtg; A eKedAcMNuc C mYy . bnÞat; CB
kat;rgVg; O Rtg; D .
1> eRbóbeFobRtIekaN CAD nigRtIekaN CBA rYcbgðajfa CA  CB  CD . 2

2> H CaeCIgbnÞat;EkgKUsecjBI A eTAbnÞat; OC  . eRbóbeFobRtIekaN CAO nigRtIekaN


CHA rYcbgðajfa CH  CO  CB  CD .
3> eRbóbeFobRtIekaN CDH nig COB rYcbBa¢ak;fa O , B , D , H sßitenAelIrgVg;EtmYy.
4> ]bmafa AB  a , AC  a2 KNna BC , CD , BD , AD CaGnuKmn_én a .
5> bnÞat;Edlb:HrgVg; O Rtg; D nig B . RbsBVKñaRtg; M . bgðajfa M sßitenAelIrgVg; C 
bBa¢ak;p©itrbs;va rYcKUsrgVg;enH . bgðajfacMNuc A , M , H rt;Rtg;Kña .

8
370
9

cemøIy
I. BICKNit b¤ 3x  7 x  0
2
 x3x  7   0

1> sRmÜlr:aDIkal; ³ naM[ 3xx07  0 x  0


 
x  0
 
3x  7 x  7 / 3
a b 
1  a b  
a  b 1

ab  ab
1
dUcenH cMeBaH A  0 enaH 1
x2, x
1 a  b2 1
2
3
a  b2
2
a2  b2 cMeBaH A  2 enaH x0, x
7
3
.
a  b 1
2 2
a b 2 2
 
ab a2  b2 1 K> rktémø x EdlnaM[RbPaKmann½y ³

a2  b2

 a b  ab  a b F
A

x  23x  1
ab ab 3x  1 x  4 3x  1 x  4

a b 
1 edIm,I[ F luHRtaEtPaKEbgrbs;vaminEmnsUnü
dUcenH sRmÜl)an 1
a b  a b . naM[ 3xx1400  3xx41  xx  14/ 3
1   
a2  b2
dUcenH F mann½ykalNa x  13 , x  4 .
2> eyIgman A  5x  4 x  4x  4  6  3xx  3
2 2

k> bRgÜmBhuFa A nigerobtam sV½yKuNcuHén x sRmÜl F ³ F  3xx  123xx14  4x  2x


 
A  5 x 2  4  x 2  4 x  4  6  3 x x  3
Edl ktþasRmÜl 3x 1  0 b¤ x  13
 5 x 2  20  x 2  4 x  4  6 x  18  3 x 2  9 x
 3x 2  7 x  2 x2
dUcenH sRmÜl)an F
4 x
.
dUcenH KNna)an A  3x  7 x  2 . 2

3> sg;bnÞat;rbs;GnuKmn_ ³
x> sresr A CaplKuNktþadWeRkTI 1 eyIgman y  x  2 D  nig y   x  4 D 
 
A  5 x 2  4  x 2  4 x  4  6  3x x  3
1 2

 5x  2x  2  x  2  3x  2x  3


2 eyIgeRbItaragtémøelxedIm,Isg;bnÞat;TaMgBIr
 x  25x  2  x  2  3x  3
x 0 2
D1  : y  x  2
2
 x  25 x  10  x  2  3x  9
y 0
x 0 4
 x  23x  1  D2  : y   x  4
y 4 0
dUcenH dak;CaplKuNktþa A  x  23x  1 y  x  2 D1 
K> > edaHRsaysmIkar ³
cMeBaH A  0 smmUl x  23x  1  0 3, 1

naM[ 3xx2100  3xx21  xx  12/ 3 y   x  4 D2 


  
cMeBaH A  2 smmUl 3x  7 x  2  2 2

371
9

x> kMNt;kUGredaenéncMnucRbsBV D  nig D  1 2 -mMu ADˆ C  BAˆ C  90 eRBaH bnÞat;b:H AC  AB


o

eyIgman y  x  2 D  nig y   x  4 D 
1 2 nig AD  BC edaysar ADˆ B  90 CamMu
o

eyIgpÞwmsmIkarGab;sIusénbnÞat;TaMgBIr carwkknøHrgVg;EdlmanGgát;p©it AB
eyIg)an ³ x  2   x  4 dUcenH CAD CBA tamlkçxNÐ m>m
2x  6  x  3 /
cMeBaH x  3 : y  x  2  3  2  1 vi)ak CADCBA

CA CD

CB CA

dUcenH kUGredaenéncMNucRbsBVKW 3 , 1 . Taj)an CA  CB  CD


2

-tamRkaPiceXIjfa bnÞat;TaMgBIrRbsBVKñaRtg; dUcenH bgðaj)anfa CA  CB  CD 1 .


2

cMNucmankUGredaen 3 , 1 edaykareFVIcMeNal 2> eRbóbeFobRtIekaN CAO nigRtIekaN CHA


EkgBIcMNucRbsBVmkelIG½kSTaMgBIr eday RtIekaN CAO nigRtIekaN CHA man ³
dUcenH kUGredaenéncMNucRbsBVKW 3 , 1 . -mMu OCˆA  ACˆH ¬mMurYm¦
K> bgðajfa D  nig D  EkgKña
1 2
-mMu OAˆ C  AHˆC  90 ¬eRBaH H CaeCIgkm<s;¦
o

bnÞat;BIrEkgKñaluHRtaplKuNemKuNR)ab;TisKW ³ dUcenH CAO CHA tamlkçxNÐ m>m


a  a  1 > kñúg CAO EkgRtg; A nigman AH Cakm<s;
eday y  x  2 D  nig y   x  4 D 
1 2
tamlkçN³énRtIekaNEkg CAO ³
manplKuNemKuNR)ab;Tis a  a  1 1  1 eyIg)an CA  CH  CO 2
2

dUcenH bnÞat; D  EkgKñanwgbnÞat; D  .


1 2 eyIgpÞwm 2 nig 1 naM[ CH  CO  CB  CD
II. FrNImaRt dUcenH bgðaj)anfa CH  CO  CB  CD 3 .
tambRmab;RbFaneyIgsg;rbU )an ³ 3> eRbóbeFobRtIekaN CDH nigRtIekaN COB
C
M eday RtIekaN CDH nigRtIekaN COB man ³
D -mMu DCˆH  OCˆB ¬mMurYm¦
C 
H -tam 3 : CH  CO  CB  CD
A  B Taj)anpleFobRCug CH 
CD
O CB CO

O dUcenH CDH COB tamlkçxNÐ C>m>C


1> eRbóbeFobRtIekaN CAD nigRtIekaN CBA vi)ak DHˆC  OBˆC
eday RtIekaN CAD nigRtIekaN CBA man ³ mü:ageTot DHˆ C  DHˆ O  180 ¬mMuCab;bEnßm¦
o

-mMu ACˆD  BCˆA ¬mMurYm¦ naM[ OBˆ C  DHˆ O  180 o

372
9

eXIjfakñúgctuekaN OBDH manplbUkmMuQm M


C
KW OBˆ C  DHˆ O  180 enaHvacarwkkñúgrgVg;mYy
o
D
C 
dUcenH O , B , D , H sßitenAelIrgVg;EtmYy H

4> KNna BC , CD , BD , AD CaGnuKmn_én a A


O

B

eyIgmankar]bma AB  a , AC  a2 O
-kñúg ABC EkgRtg; A ³ tamRTwsbþI TBItaK½r 5> bgðajfa M sßitenAelIrgVg; C 
BC 2  AB2  AC 2
2
bRmab; MB  nig MD  CabnÞat;b:HrgVg;
a a 2 5a 2
 a     a2 
2

2 4

4
naM[ MBˆ O  MDˆ O  90 o

enaH BC  5a 2 a 5

¬eRBaH bnÞat;b:HEkgnwgkaMrgVg;¦
4 2
edaykñúgctuekaN MBOD manplbUkmMuQmKW
-tam 1 ³ CA 2
 CB  CD
2
MBˆ O  MDˆ O  90 o  90 o  180 o >
a
 
naM[ CD 
CA 2
  
2 a2

2 enaH ctuekaN MBOD carwkkñgú rgVg; C 
CB a 5
2
4 a 5
EdlmanGgátp; ©it OM

a

a 5 dUcenH bgðaj)anfa M sßitenAelIrgVg; C 
2 5 10
ehIyp©itrgVg; C  CacMNuckNþal OM .
-eday BD  BC  CD 
a 5 a 5
2

10
5a 5  a 5 2 a 5
-bgðajfacMNuc A , M , H rt;Rtg;Kña
 
10 5 tamlT§pl O , B , D , H sßitenAelIrgVg;EtmYy
-kñúg ACD EkgRtg; D ³ tamRTwsbþI TBItaK½r nig ctuekaN MBOD carwkkñúgrgVg; C 
AC 2  AD2  CD 2
AD2  AC 2  CD 2
naM[ BhuekaN MBOHD carwkkñgú rgVg; C 
 a   a 5 
2 2
a 2 5a 2
EdlmanGgátp; ©it OM EtmYy
     
 2   10  4 100 enaH MHˆO  90 ¬mMucarwkknøHrgVg;Ggát;p©it OM ¦
o


a 2 a 2 5a 2  a 2 4a 2 a 2

4 20

20

20

5
ehIy AH  CO naM[ OHˆ A  90 o

enaH AD 
a 2

a a 5

edayplbUkmMu MHˆO  OHˆA  90  90  180 o o o

5 5 5
Et MHˆ O  OHˆ A  MHˆ A
dUcenH KNnaCaGnuKmn_én a )andUcteTA ³ mann½yfa mMu MHˆ A  180 vaCamMurab
o

a 5 a 5
BC 
2
; CD 
10 dUcenH bgðaj)anfacMNuc A , M , H rt;Rtg;Kña .
BD 
2a 5
5
; AD 
a 5
5
.
373
9

sm½yRbLg ³ 07 kkáda 1987


viBaØasa ³ KNitviTüa ry³eBl ³ 60 naTI BinÞú ³ 10

I. BICKNit
1> k> eK[kenSam ³ F  2x  1x  3x  3 nig G   3x  4x  3x  3 .
KNna ³ A  F  G nig B  F  G .
x> eK[ C  BA . sRmÜlrYcKNnatémøelxénkenSamenH cMeBaHtémø x dUcteTA ³
x , x5 , x 2 .
3
5
2> edaHRsayvismIkar 3x2 4  2x3 5  x 4 8 rYcbkRsaytamRkaPic .
3> cUrsg;bnÞat;tamTMnak;TMngTaMgbIxageRkamenAkñúgtRmúyEtmYy ³
y  x  4 , y   x  1 , y  4 x rYcKNnakUGredaenéncMNucRbsBVTaMgbI .
2
3

II. FrNImaRt
Ggát; AB nig CD CaGgát;p©itEkgKñaénrgVg; O Edlman O Cap©it nigkaMmanrgVas; R . I
CacMNuckNþalén OA . CI  CYbrgVg; O Rtg; E . tam E eKKUsbnÞat;Rsbnwg AB Edl
CYbnwgrgVg; O Rtg; F . CF  CYb AB Rtg; J . H CacMeNalEkgén E elI CF  .
1> R)ab;RbePTRtIekaN CEF nigctuekaN CIDJ .
2> RbdUcRtIekaN COI nig CED rYcKNna CI , CE nig IE CaGnuKmn_én R .
3> RbdUcRtIekaN CEH nig JDF rYcKNnapleFob CH CJ
.

374
9

cemøIy
I. BICKNit 2> edaHRsayvismIkar ³
3x  4 2 x  5 x  8
1> k> KNna ³ A  F  G nig B  F  G 2

3

4
eyIgman F  2x  1x  3x  3 63 x  4   42 x  5  3x  8
nig G   3x  4x  3x  3 18x  24  8 x  20  3 x  24
10 x  3 x  24  4
A  F G /
 2 x  1x  3x  3   3x  4x  3x  3 7 x  28
 x  3x  32 x  1   3x  4 x4
 x  3x  3 x  5
dUcenH vismIkarmancemøIy x  4
dUcenH KNna)an A  x  3x  3 x  5

B  F G /
-bkRsaytamRkaPic ³ x 0 4

x

 2 x  1x  3x  3   3x  4x  3x  3 EpñkminqUtCacemøIyénvismIkar .


 x  3x  32 x  1   3x  4
 x  3x  35 x  3
3> sg;bnÞat;TaMgbIkñúgtRmúyEtmYy ³
dUcenH KNna)an B  x  3x  35x  3 eyIgeRbItaragtémøelxedIm,Isg;bnÞat;TaMgbI ³
4 0
x> sRmÜlcMeBaH C  BA ³ y  x4
x
y 0 4
A  x  3x  3 x  5  x  5
C   2 x 3 0
B x  3 x  35 x  3 5 x  3 y   x 1
3 y 1 1
x  3  0 x  3
Edl x  3  0 b¤
 x  3 y  4 x
x 0 1
  y 0 4
A  x5
dUcenH sRmÜl)an C 
B 5x  3
. eyIgsg;bnÞat;TaMgbI)andUcxageRkam ³
-KNnatémøelxénkenSam C ³ y  4 x y  x4

cMeBaH x  53 ³ C
3 22 22
 5
C 5
3
 25  25
33 0
Kµann½y A
5  3
5
B
55 0
cMeBaH
x5 C  ³ 
5  5  3 22
 0 2
y   x 1
3
cMeBaH x 2 ³ -rkkUGredaenéncMNucRbsBVTaMgbI
C
 2 5  2 5 5 2 3

  

5 2  3 5 2  3 5 2  3   tag A CaRbsBVén y  x  4 , y   32 x  1
 10  3 2  25 2  15 5  22 2

25  2  9

41
pÞwmsmIkarGab;sIusénbnÞat;TaMgBIr ³
375
9

2
x  4   x 1
3
b¤ 3x  12  2 x  3 naM[ CD CaemdüaT½rén AB
3x  2 x  3  12 EtGgát;FñÚ EF  //AB enaH CD k¾CaemdüaT½r
5 x  15
x  3 énGgát;FñÚ EF  Edr vi)ak CE  CF
cMeBaH x  3 : y  x  4  3  4  1 dUcnH RtIekaN CEF CaRtIekaNsm)at .
dUcenH kUGredaenéncMNucRbsBV A 3 , 1 . -R)ab;RbePTctuekaN CIDJ ³
tag B CaRbsBVén y   23 x  1 , y  4x eyIgman I nig J enAelI AB enaH IJ // EF
pÞwmsmIkarGab;sIus ³  23 x 1  4x naM[ CEˆF  CIˆJ nig CFˆE  CJˆI ¬mMuRtUvKña¦
Et CEˆF  CFˆE ¬mMu)atRtIekaNsm)at CEF ¦
b¤  2x  3  12x  10x  3  x  103 eyIg)an CIˆJ  CJˆI enaH CIJ Ca  sm)atEdr¦
cMeBaH x  103 : y  4x  4  103   65 ebI CD CaemdüaT½rén EF  enaH CD k¾Ca
dUcenH kUGredaenéncMNucRbsBV B 103 ,  65  emdüaT½rén IJ  Edr enaH OI  OJ
edayctuekaN CIDJ manGgát;RTUg CD  IJ
tag C CaRbsBVén y  x  4 , y  4x Rtg;cMNuckNþal O enaHvaCactuekaNesµI
pÞwmsmIkarGab;sIus ³ x  4  4x dUcenH ctuekaN CIDJ CactuekaNesµI .
b¤ 5x  4  x   54
2> RbdUcRtIekaN COI nig CED
cMeBaH x   54 : y  4 x  4    54   165 eday RtIekaN COI nig CED man ³
dUcenH kUGredaenéncMNucRbsBV  4 16 
C  , 
-mMu COˆ I  CEˆD  90 eRBaH CD  IJ Rtg; O
o

 5 5 nig CEˆ D CamMucarwkknøHrgVg;Ggát;p©it CD


II. FrNImaRt -mMu OCˆI  ECˆD ¬mMurYm¦
tambRmab;RbFaneyIgsg;rbU )an ³ dUcenH COI CED tamlkçxNÐ m>m
C
vi)ak COICED

CO CI

CE CD ED

OI
1

A
I 
O H
B
-KNna CI , CE nig IE CaGnuKmn_én R
J
eday CO  OA  R ¬eRBaH CakaMrgVg;dUcKña¦
 ¬eRBaH I kNþal OA ¦
E F OA R
OI 
D O 2 2

1> R)ab;RbePTRtIekaN CEF kñúgRtIekaNEkg COI ³ tamRTwsþIbTBItaK½r


CI 2  CO 2  OI 2
eday AB  CD Rtg;cMNuckNþal O 2
R R 5
CI  CO 2  OI 2  R2    
2 2

376
9

CE  JF
tamvi)ak 1 xagelI CO 
CE CD
CI
eyIg)an CH 
JD
Taj)an CE  COCI CD 4 5R 3 5R 4 5R 3 5R
 
CH  5 10  5 10
eday CO  R , CD  2R , CI  R 2 5 R 5 R 5
2 2
naM[ CE  R  2R  4R  4 5R 2
12  5  R 2
6R 2

R 5 R 5 5
2  5 10  5
R 5 R 5
eday IE  CE  CI 2 2
2
4 5R R 5 6R 2 12 R 12 5 R
     
5 2 5 R 5 5 5 25
8 5 R  5 R 5 3R 5
 
10 10
naM[pleFob CH KW ³
dUcenH eyIgKNnaCaGnuKmn_én R )an ³ CJ
12 5 R
R 5 4 5R 3 5R CH 12 5 R 2 24
CI  , CE  , IE   25   
2 5 10 CJ R 5 25 R 5 25
2
3> RbdUcRtIekaN CEH nig JDF dUcenH KNna)anpleFob CH 24
 .
eday RtIekaN CEH nig JDF man ³ CJ 25

-mMu CHˆ E  JFˆD  90 eRBaH EH Cakm<s;én


o
sUmKUsrUbEtmYy)anehIy
CEF nig CFˆ D CamMucarwkknøHrgVg;Ggát;p©it CD C

-mMu CEˆH  HDˆ F eRBaH CamMumanRCugRtUvKñaRsb


erogKña edaysar CI // HD ¬ CIDJ ¦ nig A
I 
O H
J B
EH  CF
DF  CF
 EH // DF . E F
D O
dUcenH CEH JDF tamlkçxNÐ m>m .
CEH
vi)ak JDF

CE CH

JD JF
CE  JF
Taj)an CH 
JD
eday CE  4 55R , JD  CJ  CI  R 2 5
eRBaH JD , CI CaRCugénctuekaNesµI
ehIy CE  CF
CI  CJ
 JF  IE 
3 5R
10

377
9

sm½yRbLg ³ 07 kkáda 1988


viBaØasa ³ KNitviTüa ry³eBl ³ 60 naTI BinÞú ³ 10


I. BICKNit

1> sRmÜlkenSam ³ E  a  b  a aa  b bb .
2> kMNt;témø m nig n edIm,I[smIkar ³ mx  m  4x  3n  2  0 epÞógpÞat;cMeBaH
2

x  2 nig x  5 .

3> k> sg;bnÞat; D  tagsmIkar 3x  32 y  12 nigbnÞat; D tagsmIkar 5x  2 y  6


kñúgtRmúyGrtUNrem xoy  .
x> rkkUGredaenéncMNcu RbsBV I rvag D  nig D .
K> bMPøWfabnÞat;  tagsmIkar m  1x  2m 1y  12  6m kat;tamcMNuc I
¬ m Ca):ar:aEm:Rt ¦.
II. FrNImaRt
1> eKmanrgVg;myY p©it O nigcMNuc M mYyenAeRkArgVg; . tam M KUsbnÞat;b:H MA  nig MB 
eTAnwgrgVg;Edl A nig B CacMNucb:H. bnÞat;Ekgnwg OA Rtg; O kat; MB  Rtg; E .
RsaybMPøWfa EO  EM .
2> eKmanctuekaNBñay ABCD EdlmanmMu A nigmMu D CamMuEkg ehIyGgát;RTUg AC  Ekgnwg
RCugeRTt BC  .
k> eRbóbeFobRtIekaN ABC nigRtIekaN CAD nigbBa¢ak;fa AC  AB  CD .
2

x> eKKUsrgVg;EdlmanGgátp; ©it BC  ehIykat; AB Rtg; E . R)ab;eQµaHctuekaN AECD.


K> ehA AI  CaemdüanénRtIekaN ABC . P CacMNucmYyén BI  . tam P KUsbnÞat; 
Rsbnwg AI  .  kat; AB Rtg; M ehIykat;knøHbnÞat; CA Rtg; N . eRbóbeFob
pleFob AM AB
ni g AN
AC
.
smÁal; ³ sMNYrFrNImaRt k> x> K> minGaRs½yKñaeT .

8
378
9

cemøIy
I. BICKNit 3> k> sg;bnÞat; D  nig D kñúgtRmúy xoy 
1> sRmÜlkenSam ³ eyIgeRbItaragtémøelxedIm,Isg;bnÞat;TaMgenH
a a b b x 3 4
E  a  b   D  ³ 3x  2 y  12 y 9 0
a b 3
a  b    
2
a  b  a a b b
D  ³
 x 0 2
5x  2 y  6
a b y 3 2
a a  a b b a b b a a b b
 eyIgsg;bnÞat;TaMgBIr)andUcxageRkam ³
a b


a b b a

 a  b ab 
 ab D   : 5 x  2 y  6
a b a b 4.5
EdlktþaEdlsRmÜl a  b  0
dUcenH sRmÜlkenSam)an E  ab . D  : 3x  2 y  12
3
2> kMNt;témø m nig n 3
eyIgman mx  m  4x  3n  2  0
2

x> rkkUGredaenéncMNucRbsBV I
-cMeBaH x  2 ³ eyIg)an  2
m  2 2  m  42  3n  2  0 / eyIgmansmIkarbnÞat;TaMgBIr 3x  3 y  12

5 x  2 y  6
4m  2m  8  3n  2  0
9 x  2 y  36
6m  3n  6  0 
5 x  2 y  6
6m  3n  6 1  x
42
3
14 x  42
-cMeBaH x  5 ³ eyIg)an 14

m 5  m  4 5  3n  2  0
2 cMeBaH x  3 ³ 5  3  2 y  6  y
9
2
25m  5m  20  3n  2  0
20m  3n  22  0
dUcenH kUGredaenéncMNucRbsBV I  3 , 92  .
20m  3n  22 2
K> bMPøWfabnÞat;  kat;tam I
edayyk 2  1 edIm,IbM)at; n enaHeyIg)an ³
20m  3n  22
eyIgman  ³ m  1x  2m  1y  12  6m

6m  3n  6  m  2 ebI  kat;tam I  3 , 92  enaHvaepÞógpÞat;smIkar
14m  28 m  1x  2m  1y  12  6m
yk m  2 CMnYskñúg 1 eyIg)an ³ m  1  3  2m  1  9  12  6m
6  2  3n  6 2
 18 3m  3  9m  9  12  6m
3n  6  12  n   6
3 Bit
12  6m  12  6m
dUcenH kMNt;)antémø m  2 , n  6 . dUcenH bnÞat;  BitCakat;tam I Emn .
379
9

II. FrNImaRt M vi)ak ABC



AC AB
 ¬ CA  AC ¦
CAD CD AC
1> RsaybMPøWfa EO  EM
E dUcenH Taj)an AC  AB  CD .
2

B x> R)ab;eQµaHctuekaN AECD


A 
O
edayctuekaN AECD man ³
eday MA  nig MB  -mMu EAˆ D  ADˆ C  90 ¬mMuEkgctuekaNBñay¦
o

CabnÞat;b:HKUsecjBI M -mMu AEˆC  90 eRBaH CE  AB Rtg; E


o

naM[ MO  CaknøHbnÞat;BuHmMu AMˆ B edaysar E enAelIrgVg;Ggát;p©it BC  Edlman


enaHeyIg)an AMˆ O  BMˆ O 1 mMu CEˆB  90 CamMucarwkknøHrgVg; .
o

mü:ageTot OA  AM ¬kaMrgVg;EkgnwgbnÞat;b:H¦ eXIjfa ctuekaN AECD manmMuEkgcMnYnbIKW


OA  OE ¬smµtikmµ¦ EAˆ D  ADˆ C  90 nig AEˆ C  90
o o

naM[ AM // OE dUcenH ctuekaN AECD CactuekaNEkg .


eyIg)an AMˆ O  MOˆ E 2 ¬mMuqøas;kñúg¦
tam 1 nig 2 enaH BMˆ O  MOˆ E K> eRbóbeFobpleFob AM AB
ni g AN
AC

naM[ RtIekaN EMO CaRtIekaNsm)at -kñúgRtIekaN ABI man AI // MP


vi)ak EO  EM tamRTwsþIbTtaEls AM AB IB

IP
1

dUcenH RsaybMPøW)anfa EO  EM . -kñúgRtIekaN CNP man AI // NP


tamRTwsþIbTtaEls AN 
IP
b¤ AN IP
 2
2> tambRmab;RbFaneyIgKUsrUb)an ³ AC IC AC IB

D C eRBaH AI  CaemdüanénRtIekaN ABC Edl


 I
naM[ IB  IC enaHeKGacCMnYs IC eday IB .
A
P tamTMnak;TMng 1 nig 2 naM[ AM 
AB AC
AN

ME B
N
k> eRbóbeFobRtIekaN ABC nigRtIekaN CAD dUcenH eRbóbeFob)an AM AN

AB AC
.
eday ABC niig CAD man ³
-mMu ACˆB  CDˆ A  90 ¬smµtikmµ CamMuEkg¦
o

-mMu ACˆD  BAˆ C ¬CamMuqøas;kñúg eRBaH


AB// CD )atrbs;ctuekaNBñay ABCD¦

dUcenH ABC CAD tamlkçxNÐ m>m


380
9

381
9

sm½yRbLg ³ 07 kkáda 1989


viBaØasa ³ KNitviTüa ry³eBl ³ 60 naTI BinÞú ³ 10

I. BICKNit
1> dak;kenSamxageRkamCaplKuNktþa ³
A  x2  x  2
B  x  22 x  1  x  4x  2  x  2
2

2> sRmYlkenSam 2
a b

1

2 b
a  b a b
.
3x  2 y  0 1
3> k> edaHRsayRbB½n§smIkartamRkaPic  .
3x  y  7  0 2
x> rYcepÞógpÞat;tamEbbKNna .
II. FrNImaRt
1> eKmanrgVg;BIrkat;KñaRtg; A nig B . tam A eKKUsbnÞat;b:HeTAnwgrgVg;TaMgBIr Edlkat;rgVg;
TaMgBIrRtg; C nig D . RsaybMPøWfa CBˆ A  DBˆ A .
2> eKmanbnÞat;nwg D  mYy. enAelIbnÞat;enHeKedAcMNcu nwg A , B , C tamlMdab;enH . tam A
nig B eKKUsbnÞat;ERbRbYlBIr  nig  EkgKñaRtg; E rYceKKUs D Ekgnwg D  Rtg;
C . bnÞat; D  kat;   nig   erogKñaRtg;cMNuc M nig N .

k> bMPøWfabnÞat; BM  nig  AN  EkgKñaRtg; F . rkGrtUsg;énRtIekaN BMN .


x> B CacMNucqøHú én B cMeBaH MN  . bMPøWfacMNuc B enAelIrgVg;carwkeRkARtIekaN AMN .
rksMNMucMNucp©it O énrgVg;enH .
K> bMPøWfa CM  CN  CA CB .

382
9

cemøIy
I. BICKNit eyIgsg;RkaPic)an ³
1> dak;kenSamxageRkamCaplKuNktþa ³ 3x  2 y  0
A x x22

 x 2  x  2x  2 7
3
 xx  1  2x  1
 x  1x  2
14

dUcenH CaplKuNktþa A  x  1x  2 . 3x  y  7  0 9

B  x  22 x  1  x  4x  2  x  2
2
tamRkaPic cMNucRbsBVrvagbnÞat;KW   149 , 73 
 x  22 x  1  x  4  x  2 x> rYcepÞógpÞat;tamEbbKNna ³
 x  22 x  1
eyIgman 33xx  2y y70 0 12
dUcenH CaplKuNktþa B  x  22x  1 . 
eyIgdkGgÁngi GgÁedIm,IbM)at; x eyIg)an ³
2> sRmYlkenSam 3x  2 y  0 1

2

1

2 b 3x  y  7  0 2 7
a b a  b a b 3y  7  0  y
3
manPaKEbgrYmKW  a  b  a  b   a  b cMeBaH y
7
CMnYskñúgsmIkar 1 ³
3
eK)an ³ a 2 b  a 1 b  a2 bb 1 : 3x  2 y  0
2 a  b   a  b  2 b
2
x y
   3
a2  b2 a2  b2 a b
2 7 14
x . 
2 a 2 b  a  b 2 b 3 3 9

a b


a b dUcenH cMNucRbsBVKW   149 , 73  BitEmn .
a b

dUcenH 2

1

2 b

a b II. FrNImaRt
a b a  b a b a b
1> RsaybMPøWfa CBˆ A  DBˆ A
3> k> edaHRsayRbB½n§smIkartamRkaPic ³ ¬sUmemIlkarbkRsayenATMB½rbnþ¦
eyIgman 33xx  2y y70 0 12
 A
eyIgeRbItaragtémøelxedIm,Isg;RkaPic ³ D
x 0 2
1 : 3x  2 y  0 :
y 0 3
B
x 3 2
2 : 3x  y  7  0 : C
y 2 1

383
9

RsaybMPøWfa CBˆ A  DBˆ A km<s;TIbI RtUvEtkat;tam B nigEkgnwg  AN 


A mann½yfa BM  kat;tam B nig BM    AN 
D
O dUcenH bnÞat; BM  nig  AN  EkgKñaRtg; F .

O B -rkGrtUsg;énRtIekaN BMN
C kñúgRtIekaN BMN man BM    AN  Rtg; F
tag O nig O Cap©iténrgVg;TaMgBIr ehIy    Rtg; E naM[ ME   BN 
edaykñúg ACB nig DAB man ³ enaH  AN  nig ME  Cakm<s;TaMgBIrén BMN
-mMu ACˆB  DAˆ B ¬mMucarwk nigmMuBiessmanFñÚ eday km<s;  AN  nig ME  RbsBVKñaRtg; A
sþat;rmY AB énrgVg;p©it O ¦ dUcenH GrtUsg;én BMN KWCacMNuc A .
-mMu CAˆ B  ADˆ B ¬mMucarwk nigmMuBiessmanFñÚ
x> bMPøWfacMNcu B enAelIrgVg;carwkeRkA AMN
sþat;rmY AB énrgVg;p©it O ¦
eday M nig N enAelI D Edl D  D 
dUcenH ACB DAB tamlkçxNÐ m>m
ehIy BC  BC ¬eRBaH B CacMNucqøúHén B ¦
vi)ak mMuTIbIKW CBˆ A  DBˆ A naM[ M nig N sßitenAelIemdüaT½rén BB
dUcenH RsaybMPøW)anfa CBˆ A  DBˆ A . naM[ MB  MB nig NB  NB
2> lMhat;enH eKykeTARbLgqñaM 1991 mþgeTot enaH BMB nig BNB CaRtIekaNsm)at
tambRmab;RbFaneyIgsg;rbU )an ³ naM[ emdüaT½r MN k¾mannaTICaknøHbnÞat;BuHmMuEdr
 
eK)an BNˆ C  BNˆ C nig BMˆ C  BMˆ C
M
mü:ageTot BNˆ C  MAˆ C nig BMˆ C  NAˆ C
E

O ¬eRBaH BYkvamanRCugRtUvKñaEkgerogKña¦
B ˆ ˆ
D 
A
B
C eXIjfa BNˆ C  BNˆ C  BNˆ C  MAˆ C 1
  F N
BNC  MAC
BMˆ C  B Mˆ C
D   B Mˆ C  NAˆ C 2 
BMˆ C  NAˆ C
k> bMPøWfabnÞat; BM  nig  AN  EkgKñaRtg; F eyIgbUkGgÁnigGgÁén 1 nig 2 ³
kñúg AMN man ³  BNˆ C  MAˆ C

 BMˆ C  NAˆ C
EN  AM ¬eRBaH   nig   EkgKñaRtg; E ¦
BNˆ C  BMˆ C  MAˆ N
AC  MN ¬eRBaH D  nig D  EkgKñaRtg; C ¦ edaykñúg NMB manplbUkmMukñúgKW ³
naM[ EN nig AC Cakm<s;TaMgBIrén AMN 
ˆ  BMˆ C  MBˆ N  180o
BN
C
 
MAˆ N
ehIy EN nig AC RbsBVKñaRtg; B enaH MAˆ N  MBˆ N  180o

384
9

eXIjfactuekaN AMBN manplbUkmMuQm ³


MAˆ N  MBˆ N  180 enaH AMBN CactuekaN
o

carwkkñúgrgVg;
mann½yfa B enAelIrgVg;carwkeRkA AMN
dUcenH B enAelIrgVg;carwkeRkA AMN .
- rksMNMucMNucp©it O énrgVge; nH
eday A, B , C CacMNucnwg ¬smµtikmµ¦
enaH B CacMNucqøúHén B k¾CacMNucnwgEdr
naM[ AB CaGgát;FñÚEdlmanRbEvgefr
ehIyrgVg;p©it O kat;tam A nig B
naM[ OA  OB ¬CakaMrgVg;EtmYy¦
eK)an O CacMNuccl½t EdleFVI[ OA  OB
mann½yfa O RtUvsßitelIemdüaT½rén AB
dUcenH sMNMucMNuc O sßitelIemdüaT½rén AB
K> bMPøWfa CM  CN  CA CB
eyIgeRbóbeFob ACN nig MCB
eday ACN nig MCB man ³
-mMu ACˆN  MCB ¬eRBaH D  D Rtg; C ¦
-mMu NAˆ C  BMˆ C ¬mMumanRCugRtUvKñaEkgerogKña¦
dUcenH ACN MCB tamlkçxNÐ m>m
vi)ak ACN MCB

CN CA

CB CM
Taj)an CM  CN  CA CB
Et CB  CB ¬eRBaH B CacMNucqøúHén B ¦
eK)an CM  CN  CA CB
dUcenH bMPøW)anfa CM  CN  CA CB .

385
9

sm½yRbLg ³ 06 kkáda 1990


viBaØasa ³ KNitviTüa ry³eBl ³ 60 naTI BinÞú ³ 10

I. BICKNit
1> eK[kenSam ³ A  x  16  3x  1x  4 nig B  x
2 2
 8x  16 .
k> dak;knSam A nig B CaplKuNktþadWeRkTImYyén x .
x> sRmÜlRbPaKsniTan F  BA .
2> KNnakenSam P  5 5 3  5 3 3 ; Q  a aa  b bb nig R  2  2  3 .
3> kñúgtRmúyGrtUNrem xoy  sg;bnÞat; D  rbs;smIkar y  2x  3 nig D : y  5x  6 .
k> rkkUGredaenéncMNucRbsBV I rvagbnÞat;TaMgBIr rYcepÞógpÞat;edayKNna .
x> rksmIkarbnÞat;  Edlkat;tamcMNuc A2 ;  3 ehIyRsbnwgbnÞat; D  .

II. FrNImaRt
1> eK[ctuekaNBñay ABCD )attUc AB. bnøayRCugeRTt AD nig BC  Edlkat;KñaRtg; E .
k> eK[ ED  a ; EA  b ; EB  c . KNna BC .
x> Ax  CaknøHbnÞat;BuH DAB nig Dy  CaknøHbnÞat;BuH ADC . knøHbnÞat;TaMgBIrCYbKñaRtg;
J . R)ab;eQµaHRtIekaN AJD .

2> rgVg;BIrmanp©it O nig O kat;KñaRtg; A nig B . KUsGgát;p©ti AD nig AE  .


k> RsaybMPøWfabIcMNcu D , B , E sßitenAelIbnÞat;EtmYy .
x> M CacMNuccl½tenAelIrgVg; O . tam M KUsbnÞat;  AM  kat;rgVg; O Rtg; N .
RsaybMPøWfa MBˆ N mantémøefrkalNa M rt;CMuvij A .
K> I CacMNuckNþal AM  rksMNMucMNuc I kalNa M rt;CMuvji A elIrgVg; O .

386
9

cemøIy
I. BICKNit Q
a a b b
a b
1> mankenSam A  x  16  3x  1x  4
2
a a b b a  b 

nig B  x  8x  16
2
 a b a b  
k> dak; A nig B CaplKuNktþadWeRkTImYyén x 
a a b b  a b 
A  x 2  16  3 x  1x  4  a2  b2
 x  4 x  4   3 x  1x  4 
a 2  b ab  a ab  b 2
 x  4 x  4  3x  1 a b
 x  4 5  2 x  a  b  a ab  b ab
2 2

a b
dUcenH kenSam A  x  45  2 x  . 
a  b a  b   a  b  ab
a b
B  x 2  8 x  16


a  b  a  b  ab 
 x 2  2  x  4  42 a b
 x  4  a  b  ab , a  b  0  a  b
2

 x  4x  4
dUcenH KNna)an Q  a  b  ab .
dUcenH kenSam B  x  4x  4 .
R  2 2 3
x> sRmÜlRbPaKsniTan F
A
B  42 3 
2
3  2 3  12
A  x  4 5  2 x 
F 
B  x  4  x  4 
 3  1  3  1

2


5  2x
, x4 0  x  4 dUcenH KNna)an R  3  1 .
x4

dUcenH sRmÜl)an F
A 5  2x
 . 3> sg;bnÞat; D  nig D
B x4
D  ³ y  2 x  3 ³
x 0 1
y 3 5
2> KNnakenSam ³
5 3 ³ xy 11
D   : y  5 x  6
2
P  4
5 3 5 3
  
5 5  3 3 5  3  eyIgsg;)andUcxageRkam ³


5 3 5 3   D  : y  2x  3
5 5 5 3 3 5 3 3
 I 3 , 9 
5 2  32
8 52 3 D : y  5x  6
 4 5 3
2

dUcenH KNna)an P4 5 3 .

387
9

x> rkkUGredaenéncMNucRbsBV I Et AD  ED  EA enaH BC  EB  ED


EA
 EA

tamRkaPic ³ ebIeyIgeFVIcMeNalEkgBIrcMNuc ehIy ED  a ; EA  b ; EB  c ¬smµtikmµ¦


RbsBVeTAelIG½kSTaMgBIr enaHeyIg)an I 3, 9 . BC 
EB  ED  EA ca  b  ac  bc
 
-epÞógpÞat;tamkarKNna ³ EA b b
ac  bc
eyIgman D  ³ y  2x  3 nig D : y  5x  6 dUcenH KNna)an BC 
b
.
eyIgpÞwmsmIkarGab;sIusénbnÞat;TaMgBIr ³ x> R)ab;eQµaHRtIekaN AJD
2 x  3  5x  6
3x  9  x3 eday AB//CD ¬)atTaMgBIrctuekaNBñay¦
cMeBaH x  3 enaH y  2x  3  2  3  3  9 naM[ DAˆ B  CDˆ A  180 ¬mMubEnßmKña¦
o

Et DAˆ B  2DAˆ J ¬eRBaH Ax  BuHmMu DAB¦


dUcenH KNna)an I 3, 9 .
CDˆ A  2 ADˆ J ¬ eRBaH Dy  BuHmMu ADC ¦
x> rksmIkarbnÞat;  ³ naM[ 2DAˆ J  2 ADˆ J  180 o

smIkarbnÞat;Edlrkmanrag  : y  ax  b 2DAˆ J  ADˆ J   180 o

eday  : y  ax  b kat;tam A2 ;  3 DAˆ J  ADˆ J  90o

eyIg)an  3  a  2  b  b  3  2a eXIjfakñúg AJD man DAˆ J  ADˆ J  90 o

ehIy  RsbnwgbnÞat; D  enaH a  a  2 naM[ AJˆD  90 ¬CamMuEkg¦


o

cMeBaH a  2 ³ b  3  2a  3  2  2  7 dUcenH RtIekaN AJD CaRtIekaNEkgRtg; J .


dUcenH smIkarbnÞat;  : y  2x  7 . 2> tambRmab;RbFaneyIgsg;rUb)an ³
N
II. FrNImaRt A
O 
1> tambRmab;RbFaneyIgsg;rUb)an ³ I

E
E O
M B

A B y D
k> RsaybMPøfW a D , B , E sßitenAelIbnÞat;EtmYy
J
D C edayrgVg;Ggát;p©it AD nig AE  RbsBVKñaRtg;
x
cMNuc A , B enaHcMNuc A , B enAelIrgVg;TaMgBIr
k> KNna BC
-rgVg;Ggát;p©it AD man B enAelIrgVg;
eday)atctuekaNBñay ABCD man AB//CD
naM[ ABˆD  90 ¬mMucarwkknøHrgVg;Ggát;p©it AD ¦
o

tamRTwsþIbTtaEls eyIg)anGgát;smamaRtKW ³
EA EB EB  AD -rgVg;Ggát;p©it AE  man B enAelIrgVg;
  BC 
AD BC EA naM[ ABˆE  90 ¬mMucarwkknøHrgVg;Ggát;p©it AE  ¦
o

388
9

eday ABˆD  ABˆE  90  90  180


o o o

N
Et ABˆD  ABˆE  DBˆE A

eXIjfa ABˆˆ D  ABˆˆ E  180ˆ  DBˆ E  180 O
o
o 
I
ABD  ABE  DBE E
O
mann½yfa DBˆ E CamMurab b¤ bnÞat;Rtg; M B

dUcenH D , B , E sßitenAelIbnÞat;EtmYy . D

x> RsaybMPøWfa MBˆ N mantémøefrkalNa


M rt;CMuvij A

kñúg MNB eTaHbI M rt;CMuvij A y:agNak¾eday


k¾eyIgenAEtTTYl)an ³
-cMeBaHrgVg;p©ti O ³ AMˆ B  12  AB efr
-cMeBaHrgVg;p©ti O ³ ANˆ B  12  AB efr
naM[ MBˆ N  180  AMˆ B  ANˆ B  k¾efrEdr
o

dUcenH MBˆ N mantémøefrkalNa M rt;CMuvij A


K> rksMNMucN M uc I kalNa M rt;CMuvij A
elIrgVg; O
eyIgman IM  IA ¬eRBaH I kNþal AM  ¦
enaHeyIg)an OI  AM
¬eRBaHkaMrgVgE; kgnwgGgát;FñÚRtg;cMNuckNþal¦
eday cMNuc O , A minERbRbYl
eyIg)an OIˆA  90 mantémøefr
o

edIm,I[ OIˆA  90 efr luHRta I sßitenAelI


o

rgVg;EdlmanGgát;p©it OA
-ebI M RtYtelI D enaH I RtYtelI O
-ebI M RtYtelI A enaH I RtYtelI A
dUcenH sMNcMu MNuc I enAelIrgVg;Ggát;p©it OA .

389
9

sm½yRbLg ³ 09 mifuna 1991


viBaØasa ³ KNitviTüa ry³eBl ³ 60 naTI BinÞú ³ 10

I. BICKNit
1> dak;kenSamxageRkamCaplKuNktþa ³
A  3 x  5  16
2

B  x  32 x  1  x  3  x  3
2

rYcedaHRsaysmIkar 1 3
B A
 0 ; A0 ; B0 .
2> k> BnøatkenSam P  4  5 3  10  .
x> KNnakenSam Q  6  4 2 .
3> tamTMnak;TMng x  2  y  2 KNna xy rYcKNna x nig y edaydwgfa x  y  1 .
4> kñúgtRmúyGrtUNrem xoy  manÉktaCasg;TIEm:Rt cm .
k> sg;bnÞat;   : y  2x  3 nig   : y  3x  7 .
1 2

x> bnÞat;TaMgBIrRbsBVKñaRtg;cMNuc A . KNnakUGredaenéncMNuc A .


K> sresrsmIkarbnÞat; D  Edl D   ehIykat;tamcMNuc I 4 , 3 .
1

II. FrNImaRt
eKmanbnÞat;nwg D  mYy. enAelIbnÞat;enHeKedAcMNucnwg A , B , C tamlMdab;enH . tam A
nig B eKKUsbnÞat;ERbRbÜlBIr  nig  EkgKñaRtg; E rYceKKUs D Ekgnwg D  Rtg; C .
bnÞat; D kat;  nig  erogKñaRtg;cMNuc M nig N .
k> bMPøWfabnÞat; BM  nig  AN  EkgKñaRtg; F . rkGrtUsg;énRtIekaN BMN .
x> B CacMNucqøHú én B cMeBaH MN  . bMPøWfacMNuc B enAelIrgVg;carwkeRkARtIekaN AMN .
rksMNMucMNucp©it O énrgVg;enH .
K> bMPøWfa CM  CN  CA CB .

8
390
9

cemøIy
I. BICKNit -edaHRsaysmIkar cMeBaH B  0
1> dak;kenSamxageRkamCaplKuNktþa ³ eK)an x  3x  5  0
A  3 x  5  16
naM[ xx  53  00  xx  53
2

 3 x  5  4 2  
2

 3 x  5  43 x  5  4 
 3 x  13 x  9 
dUcenH ebI B  0 enaHb£s x  3 , x  5 .
 33 x  1 x  3 2> k> BnøatkenSam P ³
B  x  32 x  1  x  3  x  3
P  4  5 3  10 
2

 x  32 x  1  x  3  x  3
2
 12  4 10  3 5  50
 x  32 x  1  x  3  1
 12  4 10  3 5  5 2
 x  3x  5

dUcenH A  33x  1x  3 nig B  x  3x  5 dUcenH Bnøat)an P  12  4 10  3 5  5 2

-edaHRsaysmIkar ³ cMeBaH B1  A3  0 x> KNnakenSam Q ³


Q  6  4 2  4  2  2  2  22
eK)an x  31 x  5  33x  13x  3  0  2  2 2
 2 2
1 1
 0
x  3x  5 3x  1x  3 dUcenH KNna)an Q  2 2 .
tRmÚvPaKEbgrYm x  3x  53x  1 rYclubecal
x  3  0  x  3
3> KNna xy
 
Edl x  5  0  x  5 eyIgman TMnak;TMng x  2  y2
3x  1  0  x  1 / 3
 
Taj)an xy  22  x
 2
eK)an 3x  1  x  5  0 y

2 x  6  0  x  3 / dUcenH KNna)an xy  2 .
cMeBaH x  3 minyk eRBaHvaCalkçxNÐ -KNna x nig y cMeBaH x  y  1
dUcenH smIkar 1 3
 0
B A
KµancemøyI . x  2  y  2 enaH
x
2

y
2
¬tamsmamaRt¦
x y 2 2 2 2
-edaHRsaysmIkar cMeBaH A  0 x
2

y
 
1

2 2  2 2  2 22  22

2
eK)an 33x  1x  3  0 naM[ x 2 2
  x  2 2
2 2
naM[ 3xx3100  xx  13/ 3 y

2 2
 y
2 2  22
 2 1
 
2 2 2

dUcenH ebI A  0 enaHb£s x   13 , x  3 dUcenH KNna)an x  2  2 / y  2 1 .


391
9

4> k> sg;bnÞat;   nigbnÞat;  


1 2 II. FrNImaRt
eyIgeRbItaragtémøelxedIm,Isg;bnÞat;TaMgenH tambRmab;RbFaneyIgsg;rbU )an ³
 
  : y  2 x  3 ³
x 0 1 M
1
y 3 1
E
 2  : y  3x  7³ y 4 1 x 1 2

O

D  B B
eyIgsg;bnÞat;TaMgBIr)andUcxageRkam ³  
A C
F N
1  : y  2 x  3  2  : y  3x  7 D 

D : y  2 x  11 k> bMPøWfabnÞat; BM  nig  AN  EkgKñaRtg; F


kñúg AMN man ³
EN  AM ¬eRBaH   nig   EkgKñaRtg; E ¦

AC  MN ¬eRBaH D  nig D  EkgKñaRtg; C ¦


A
naM[ EN nig AC Cakm<s;TaMgBIrén AMN
x> KNnakUGredaenéncMNuc A ehIy EN nig AC RbsBVKñaRtg; B enaH
eyIgman   : y  2x  3 nig   : y  3x  7
1 2
km<s;TIbI RtUvEtkat;tam B nigEkgnwg  AN 
eyIgpÞwmsmIkarGab;sIusénbnÞat;TaMgBIr ³ mann½yfa BM  kat;tam B nig BM    AN 
eyIg)an  2x  3  3x  7
dUcenH bnÞat; BM  nig  AN  EkgKñaRtg; F .
3x  2 x  7  3  x  4 /
cMeBaH x  4 : y  2x  3  2  4  3  5 -rkGrtUsg;énRtIekaN BMN
kñúgRtIekaN BMN man BM    AN  Rtg; F
dUcenH kUGredaenéncMNucRbsBVKW A4 ,  5 . ehIy    Rtg; E naM[ ME   BN 
K> sresrsmIkarbnÞat; D  enaH  AN  nig ME  Cakm<s;TaMgBIrén BMN
smIkarbnÞat;EdlRtUvrkmanrag D : y  ax  b eday km<s;  AN  nig ME  RbsBVKñaRtg; A
eday D   naM[ a  a  2
1
dUcenH GrtUsg;én BMN KWCacMNuc A .
ehIykat;tamcMNuc I 4 , 3
x> bMPøWfacMNcu B enAelIrgVg;carwkeRkA AMN
naM[ 3   2 4  b  b  3  8  11
eday M nig N enAelI D Edl D  D 
dUcenH smIkarbnÞat; D : y  2x  11 . ehIy BC  BC ¬eRBaH B CacMNucqøúHén B ¦
naM[ M nig N sßitenAelIemdüaT½rén BB
naM[ MB  MB nig NB  NB
enaH BMB nig BNB CaRtIekaNsm)at
392
9

naM[ emdüaT½r MN k¾mannaTICaknøHbnÞat;BuHmMuEdr K> bMPøWfa CM  CN  CA CB


eK)an BNˆ C  BNˆ C nig BMˆ C  BMˆ C eyIgeRbóbeFob ACN nig MCB
mü:ageTot BNˆ C  MAˆ C nig BMˆ C  NAˆ C eday ACN nig MCB man ³
¬eRBaH BYkvamanRCugRtUvKñaEkgerogKña¦ -mMu ACˆN  MCB ¬eRBaH D  D  Rtg; C ¦
ˆ ˆ
eXIjfa BNˆ C  BNˆ C  BNˆ C  MAˆ C 1 -mMu NAˆ C  BMˆ C ¬mMumanRCugRtUvKñaEkgerogKña¦
BNC  MAC
dUcenH ACN MCB tamlkçxNÐ m>m
BMˆ C  B Mˆ C
 B Mˆ C  NAˆ C 2  vi)ak ACN 
CN CA

ˆ
BMC  NAC ˆ
MCB CB CM
eyIgbUkGgÁnigGgÁén 1 nig 2 ³ Taj)an CM  CN  CA CB
 BNˆ C  MAˆ C
 Et CB  CB ¬eRBaH B CacMNucqøúHén B ¦
 BMˆ C  NAˆ C
BNˆ C  BMˆ C  MAˆ N
eK)an CM  CN  CA CB
edaykñúg NMB manplbUkmMukñúgKW ³ dUcenH bMPøW)anfa CM  CN  CA CB .
ˆ  BMˆ C  MBˆ N  180o
BN
 C
 
MAˆ N

MAˆ N  MBˆ N  180o rUbdEdl edIm,I[RsYlemIlkñúgkarbkRsay


eXIjfactuekaN AMBN manplbUkmMuQm ³   M
MAˆ N  MBˆ N  180 enaH AMBN CactuekaN
o

E
carwkkñúgrgVg; 
O
B
mann½yfa B enAelIrgVg;carwkeRkA AMN D 
A
B
C
  F N
dUcenH B enAelIrgVg;carwkeRkA AMN . D 
- rksMNMucMNucp©it O énrgVge; nH
eday A, B , C CacMNucnwg ¬smµtikmµ¦
enaH B CacMNucqøúHén B k¾CacMNucnwgEdr
naM[ AB CaGgát;FñÚEdlmanRbEvgefr
ehIyrgVg;p©it O kat;tam A nig B
naM[ OA  OB ¬CakaMrgVg;EtmYy¦
eK)an O CacMNuccl½t EdleFVI[ OA  OB
mann½yfa O RtUvsßitelIemdüaT½rén AB
dUcenH sMNMucMNuc O sßitelIemdüaT½rén AB
393
9

sm½yRbLg ³ 14 kkáda 1992


viBaØasa ³ KNitviTüa ry³eBl ³ 60 naTI BinÞú ³ 10

I. BICKNit
1> k> bMEbkkenSam A CaplKuNktþaEdl A  2 x  3x  4  4 x  9  2 x  3 .
2 2

x> etI x RtUvmantémøesµIb:unµan edIm,I[ A  0 ?


K> KNnatémøelxénknSam A cMeBaH x  1 2 .
2> sRmYlkenSam F  2xx32x7x 49 4x6x972xx23 .
2 2

3> kñúgtRmúyGrtUNrem xoy  EdlmanÉktþaCa sg;TIEm:Rt ¬ cm ¦ .


k> sg;bnÞat; D  : y  2x  4 nigbnÞat; D  : y  x  1 .
1 2

x> bnÞat; D  nig D  RbsBVKñaRtg;cMNuc I . KNnakUGredaenéncMNuc I .


1 2

K> sresrsmIkarbnÞat; D  Ekgnwg D  nigkat;tamcMNuc A0 ,1 .


2

II. FrNImaRt
eK[rgVg; C  p©it O viCÄmaRt AB Edl AB  4R nigrgVg; C  p©it O viCÄmaRt OA .
tam B KUsbnÞat;b:H BM  eTAnwgrgVg; C  Rtg; M . BM  kat;rgVg; C  Rtg; E .
1> bgðajfa AE  Rsbnwg OM  . rYcTajbBa¢ak;fa AM  CaknøHbnÞat;BuHén BAE.
2>  AM  kat;rgVg; C  Rtg; N . bgðajfa M CacMNuckNþalén AN  .
3> bnÞat; ON  kat; BM  Rtg; G . etI G CaGVIcMeBaHRtIekaN ANB .
KNna OG CaGnuKmn_én R .
4> bnÞat; AE  kat; BN  Rtg; P . bgðajfa MP  AB  .

394
9

cemøIy
I. BICKNit 3> k> sg;bnÞat; D  nigbnÞat; D 
1 2

1> k> bMEbkkenSam A CaplKuNktþa eyIgeRbItaragtémøelxedIm,Isg;bnÞat;TaMgBIr


 
A  2 x  3x  4  4 x 2  9  2 x  3
2
D  : y  2 x  4 ³
x 0 2
 2 x  3x  4  2 x  32 x  3  2 x  3
1
2 y 4 0
 2 x  3x  4  2 x  3  2 x  3 D2  : y  x  1 ³ xy 10 12
 2 x  3x  4  2 x  3  2 x  3
 2 x  3x  2 eyIgsg;bnÞat;TaMgBIr)andUcxageRkam ³
dUcenH bMEbk)an A  2x  3x  2 .
D2  : y  x  1
x> rktémø x edIm,I[ A  0
eyIgman A  2x  3x  2
ebI A  0 enaH 2x  3x  2  0
 3 D1  : y  2 x  4
2 x  3  0 2 x  3 x
naM[ 
 x  2  0   x  2   2
 
 x  2 x> KNnakUGredaenéncMNcu I
dUcenH ebI A  0 enaH x  32 , x  2 . eyIgman D  : y  2x  4 b¤ y  2x  4
1

D  : y  x  1 b¤ y  x  1
K> KNnatémøelxénknSam A cMeBaH x  1 2
2

eyIgpÞwmsmIkarGab;sIusénbnÞat;TaMgBIr ³
eyIgman A  2x  3x  2 cMeBaH x  1 2 eyIg)an  2x  4  x  1
eyIg)an A  21  2  31  2  2 3x  3  x 1 /
 
 2 2 1 3  2 
 6 2  43 2 cMeBaH x  1 enaH y  x  1  1  1  2
 5 2 1
dUcenH kUGredaenéncMNucRbsBV I 1, 2 .
dUcenH cMeBaH x  1 2 enaH A  5 2 1 .
K> sresrsmIkarbnÞat; D 
2> sRmYlkenSam F ³
2x  3x  4  4 x 2  9  2x  32
smIkarbnÞat; D  EdlRtUvrkmanrag y  ax  b
F
x  27 x  9  6 x  7 x  2 eday D   D   a  a  1
2


A
x  27 x  9  6 x  7 x  2 Et D  : y  x  1 man a  1 naM[ a  1
2


2 x  3x  2 ehIy D  kat;tamcMNuc A0 ,1
x  27 x  9  6 x  7 
2x  3
eyIg)an 1  a  0  b  b  1
 , x  2  0  x  2
x2
2x  3
dUcenH smIkarbnÞat; D  ³ y   x  1 .
dUcenH sRmÜl)an F
x2
.
395
9

II. FrNImaRt rgVg;p©it O man AN CaGgát;FñÚ nig AN  MO


tambRmab;RbFaneyIgsg;rbU )an ³ naM[ MA  MN eRBaH
P kaMEkgnwgGgát;FñÚRtg;cMNuckNþalCanic©
N dUcenH M CacMNuckNþalén AN  .
E M
G 3> etI G CaGVIceM BaHRtIekaN ANB
  B
A O O
kñúg ANB mancMNuc M kNþal AN nig O
C

C  kNþal AB naM[ Ggát; BM nig ON Caemdüan


1> bgðajfa AE  Rsbnwg OM  TaMgBIrénRtIekaNenH .
rgVg;viCÄmaRt AB man E enAelIrgVg;enH edayemdüan BM nig ON RbsBVKñaRtg; G
¬eRBaH BM  kat;rgVg; C  Rtg; E ¦ enaH emdüanTIbI k¾kat;tamcMNuc G enHEdr .
naM[ ABE EkgRtg; E Edl AE  EB dUcenH G CaTIRbCMuTm¶n;énRtIekaN ANB .
ehIy BM  CabnÞat;b:HrgVg;p©it O Rtg; M
-KNna OG CaGnuKmn_én R
naM[ OM  MB b¤ OM  EB
tamlkçN³emdüan OG  13  ON
eXIjfa OAEMEBEB  AE//OM 
Et ON  OA  AB 2

4R
2
 2 R ¬kaMrgVg;dUcKña¦
dUcenH bgðaj)anfa AE  Rsbnwg OM  .
naM[ OG  13  2R  23 R
-TajbBa¢ak;fa AM  CaknøHbnÞat;BuHén BAE
eday AE //OM  ¬sRmayxagelI ¦ dUcenH KNna)an OG 
2
3
R .
naM[ OMˆ A  EAˆ M 1 ¬mMuqøas;kñúg¦ 4> bgðajfa MP  AB 
ehIy OAM CaRtIekaNsm)at eK)an AP  BE nig BP  AN eRBaH E nig N
¬eRBaH OA  OM kaMrgVg;p©it O dUcKña¦ enAelIrgVg;viCÄmaRt AB vaCamMcu arwkknøHrgVg;
naM[ OMˆ A  OAˆ M 2 ¬mMu)at  sm)at¦ kñúg APB man AP  BE nig BP  AN
pÞwm 1 nig 2 eyIg)an ³ enaH BE nig AN Cakm<s;TaMgBIrénRtIekaNenH
EAˆ M  OAˆ M b¤ EAˆ M  BAˆ M
ehIy M CacMNucRbsBVénkm<s; BE nig AN
dUcenH AM  CaknøHbnÞat;BuHén BAE . enaHkm<s;TIbI RtUvkat;tam M ehIyEkgnwg
2> bgðajfa M CacMNuckNþalén AN  RCugTIbIénRtIekaNenH mann½yfa MP  AB
rgVg;viCÄmaRt OA man M enAelIrgVg;enH dUcenH bgðaj)anfa MP  AB  .
naM[ mMu OMˆ A  90 enaH AM  MO
o

b¤ AN  MO ¬eRBaH N Cabnøayén AM ¦
396
9

397
9

sm½yRbLg ³ 14 kkáda 1993


viBaØasa ³ KNitviTüa ry³eBl ³ 60 naTI BinÞú ³ 10


I. BICKNit
1> eK[kenSam Ax  x  2x  3  2  x5  x  4  x . 2

k> BnøatkenSamxagelIenH rYcsresrtamlMdab;sV½yKuNcuHén x .


x> sresr Ax CaragplKuNktþa .
K> sRmYlRbPaKsniTan F x  5x  2A xxx  2 . kMNt;témø x edIm,I[ F x   0 .
2> sg;bnÞat; D  nig D  tagGnuKmn_ y  x  4 nig y  5  x .
1 2

rkkUGredaenéncMNucRbsBVrvagbnÞat;TaMgBIr .
II. FrNImaRt
eK[rgVg;p©it O kaM R . KUsbnÞat; D  b:HnwgrgVg;Rtg; P . A CacMNucmYyén D  . rgVg;Ggát;
p©it OA kat;knøHbnÞat;BuH OAP Rtg; H . bnøayén OH  kat;bnÞat; D  Rtg; M .
1> bgðajfargVgG; gát;p©it OA kat;tamcMNuc P .
2> R)ab;RbePTRtIekaN AOM .
3> KUskm<s; MK  énRtIekaN AOM . RsaybMPøWfa MK  R .

8
cemøIy
I. BICKNit x> sresr Ax CaragplKuNktþa
1> eKman Ax  x  2x  3  2  x5  x  4  x 2 Ax   x  2x  3  2  x 5  x   4  x 2

 x  2x  3  x  25  x   x 2  4 
k> BnøatkenSam nigsresrtamlMdab;sV½yKuNcuH  x  2x  3  x  25  x   x  2x  2
Ax   x  2x  3  2  x 5  x   4  x 2  x  2x  3  5  x  x  2
 x 2  3x  2 x  6  10  2 x  5 x  x 2  4  x 2  x  2x  4
 x 2  5 x  6  10  7 x  x 2  4  x 2
 x 2  2x  8 dUcenH dak; Ax CaplKuNktþa)anKW ³
dUcenH BnøatkenSam)an Ax  x 2
 2x  8 . Ax   x  2x  4 .

398
9

K> sRmYlRbPaKsniTan F x ³ II. FrNImaRt


eKman F x  5x  2A xxx  2 tambRmab;RbFaneyIgsg;rbU )andUcxageRkam ³
b¤ F x  xx  225x  4x  5x  4x O

K
Edl x  2  0  x  2 H
D 
dUcenH sRmÜl)an F x  5x  4x . A P M
1> bgðajfargVg;Ggát;p©it OA kat;tamcMNuc P
- kMNt;témø x edIm,I[ F x   0 bnÞat;b:H D  EkgnwgkaMrgVg; OP Rtg;cMNucb:H P
ebI F x   0 enaH 5x  4x  0 naM[ mMu OPA  90 ¬eRBaH A enAelI D  ¦
o

eK)an 5x  x4  00  xx  54 ehIy OA Ggát;p©itrgVg; enaHmann½yfa


  OPA CamMucarwkknøHrgVg; Ggát;p©it OA 

dUcenH ebI F x   0 kMNt;)antémø x  4 . dUcenH rgVg;Ggát;p©it OA kat;tamcMNuc P .


2> sg;bnÞat; D  nig D 
1 2
2> R)ab;RbePTRtIekaN AOM
eyIgeRbItaragtémøelxedIm,Isg;bnÞat;TaMgBIr eday H CacMNucenAelIrgVg;Ggát;p©it OA
x 4 0
D  ³ y  x  4
1 naM[ AH  OH b¤ AH  OM
y 0 4

D2  ³ y  5 x
x 0 5 ¬eRBaH M enAelIbnøay OH ¦
y 5 0
eday AOM man AH CaknøHbnÞat;BuHpg nig
D1  : y  x  4 Cakm<s;pg luHRtaEt AOM CaRtIekaNsm)at
dUcenH RtIekaN AOM CaRtIekaNsm)at .
D2  : y  5  x
vi)ak AM  AO
3> RsaybMPøWfa MK  R
-rkkUGredaenéncMNucRbsBVrvagbnÞat;TaMgBIr eday OPA  90 enaH OP Cakm<s;én AOM
o

eyIgpÞwmsmIkarGab;sIusénbnÞat;TaMgBIr ehIybRmab; MK  Cakm<s;énRtIekaN AOM


eyIg)an x  4  5  x kñúg AOM man OP  nig MK  Cakm<s;erogKña
b¤ 2x  1  x
1
2
RtUvnwgRCug AM nig AO Edl AM  AO
cMeBaH x  2 : y  5  x  5  2  2
1 1 9 naM[ MK  OP  R ¬eRBaH OP  R kaMrgVg;¦
dUcenH cMNcu RbsBVénbnÞat;TaMgBIrKW  12 , 92  . dUcenH bMPøW)anfa MK  R .

399
9

sm½yRbLg ³ 12 kkáda 1994


viBaØasa ³ KNitviTüa ry³eBl ³ 60 naTI BinÞú ³ 10

I. BICKNit
1> k> eRbóbeFobcMnYn x  3 5 nig y  2 11 .
x> sRmYlkenSam E  5 5 3  5 3 3 .
2> eK[kenSam ³ Ax  x  4x  4  x  36  3x  5x  4 .
2 2

k> BnøatkenSam Ax rYcerobtamlMdab;sV½yKuNcuHén x .


x> dak; Ax CaplKuNktþadWeRkTI1 .
K> edaHRsaysmIkar Ax  0 / Ax  2 .
X> eK[RbPaK F x  3x A1x4  x rktémø x EdleFVI[ F x mann½y rYcsRmYl F x .
3> k> kñúgtRmúyGrtUNrem xoy  sg;bnÞat; D  : y  x  2 nig D  : y  4  x .
1 2

x> rkkUGredaenéncMNucRbsBV I rvagbnÞat;TaMgBIr .


II. FrNImaRt
eK[rgVg;p©it O EdlmanGgátp; ©itBIrKW AB nig CD EkgKña . E CacMNucmYyén OA .
1> R)ab;eQµaHénRtIekaN CED .
2> tamcMNuc C eKKUsbnÞat;Ekgnwg CE  ehIytam D eKKUsbnÞat;Ekgnwg DE  . bnÞat;TaMg
BIrenHRbsBVKñaRtg;cMNuc F .
k> eRbóbeFobRtIekaN CEF nigRtIekaN DEF .
x> bgðajfa F enAelIbnÞat; AB .
3> H CacMNucqøúHéncMNuc E eFobnwgcMNuc O .
k> R)ab;eQµaHctuekaN CEDH .
x> RsaybBa¢ak;fa H CaGrtUsg;énRtIekaN CDF .

8
400
9

cemøIy
I. BICKNit Ax   3x 2  7 x  2
 3x 2  x  6 x  2
1> k> eRbóbeFobcMnYn x  3 5 nig y  2 11  x3x  1  23x  1
eyIgman x  3 5 nig y  2 11  3x  1x  2

naM[ x  3 5  3  5  9  5  45
2
dUcenH plKuNktþa Ax  3x  1x  2 .
y  2 11  22 11  4 11  44
K> edaHRsaysmIkar ³
eday 45  44  x  y eyIgman Ax  3x  1x  2
dUcenH eRbóbeFob)an x  y . -cMeBaH Ax  0 enaH 3x  1x  2  0
 1
x> sRmYlkenSam E ³ naM[ 3xx2100  3xx21   x  3
 
x  2
eKman E  5 5 3  5 3 3

5

3 dUcenH ebI Ax  0 enaH x
1
3
, x2 .
5 3 5 3


  
5 5  3 3 5  3  -cMeBaH Ax  2 enaH 3x  7x  2  2 2


5 3 5 3   b¤ 3x  7 x  0  3x  7x  0
2

5 5 5 3 3 5 3 3
  7
2
5  3
2
naM[ 3xx07  0  3xx07   x  3
 
8 52 3 x  0

53

24 5 3  dUcenH ebI Ax   2 enaH x
7
3
, x0 .
 4 5 3
2
X> rktémø x EdleFVI[ F x mann½y
dUcenH sRmÜl)an E4 5 3 .
eyIgman F x  3x A1x4  x
2> eK[kenSam ³ RbPaK F x mann½y luHRtaEtPaKEbgxusBI 0

Ax   x 2  4 x  4  x  36  3x   5 x 2  4   1
3x  1  0 3x  1 x
k> Bnøat Ax rYcerobtamlMdab;sV½yKuNcuHén x eK)an 
4  x  0   x  4   3
  
   x 4
Ax   x 2  4 x  4  x  36  3 x   5 x 2  4
 x  4 x  4  3 x  3 x  18  5 x  20
2 2 2 dUcenH F x  mann½y kalNa x  13 , x  4 .
 3x 2  7 x  2
sRmÜl F x  3x A1x4  x
dUcenH Bnøat)an Ax  3x  7 x  2 . 2

3x  1x  2  x  2

x> dak; Ax CaplKuNktþadWeRkTI1 3x  14  x  4  x
x2
dUcenH sRmÜl)an F x  
4 x
.
401
9

3> k> sg;bnÞat; D  nig D  kñúgtRmúy xoy 


1 2
2> k> eRbóbeFob CEF nig DEF
eyIgeRbItaragtémøelxedIm,Isg;bnÞat;TaMgBIr RtIekaNTaMgBIrman EC  CF nig ED  DF
D1  : y  x  2
x 0 2 enaHmMu ECˆF  EDˆ F  90
o

y 2 0
x 0 4
naM[ CEF nig DEF CaRtIekaNEkgTaMgBIr
 D2  : y  4  x
y 4 0 Edlman ³ - RCug EC  ED ¬bBa¢ak;xagelI¦
-GIub:UetnusrYm EF
D1  : y  x  2 dUcenH CEF DEF tamkrNI G>C .
vi)ak CF  DF
x> bgðajfa F enAelIbnÞat; AB
D2  : y  4  x tamvi)akxagelI CF  DF mann½yfa F RtUv
sßitenAelIemdüaT½r énGgát; CD
x> rkkUGredaencMNucRbsBV I énbnÞat;TaMgBIr ehIyemdüaT½rén CD KW AB
eyIgman D  : y  x  2 nig D  : y  4  x
1 2
dUcenH eXIjfa F enAelIbnÞat; AB .
edaypÞwmsmIkarGab;sIusénbnÞat;TaMgBIr 3> k> R)ab;eQµaHctuekaN CEDH .
eK)an x  2  4  x  2x  6  x  3 H CacMNucqøúHéncMNuc E eFobnwgcMNuc O
cMeBaH x  3 enaH y  x  2  3  2  1 naM[ OE  OH Edl E nig H enAelI AB
dUcenH cMNucRbsBVrvagbnÞat;TaMgBIr I 3,1 . ehIy OC  OD ¬kaMrgVg;p©it O dUcKña¦
nigman EH   CD Rtg;cMNuckNþal O
II. FrNImaRt
enaH CEDH CactuekaNmanGgát;RTUgEkgKña
tambRmab;RbFaneyIgsg;rbU )an ³ Rtg;cMNuckNþal ehIyvaKµanmMuEkg
C
dUcenH ctuekaN CEDH CactuekaNesµI .
A E O
H
B F
vi)ak ³ CE // DH nig ED// CH
x> RsaybBa¢ak;fa H CaGrtUsg;én CDF
D eday CE // DH
CE  CF
 DH  CF
1> R)ab;eQµaHénRtIekaN CED ED // CH
edayGgát;p©it AB nig CD EkgKña ED  DF
 CH  DF

ehIy E CacMNucmYyén OA enaH E sßitenA ehIy FH  CD eRBaH F nig H enAelI AB
elIemdüaT½r AB énGgát; CD naM[ CDF man DH , CH , FH Cakm<s;
naM[ EC  ED dUcenH H CaGrtUsg;én CDF .
dUcenH RtIekaN CED CaRtIekaNsm)at .
402
9

403
9

sm½yRbLg ³ 13 kkáda 1995


viBaØasa ³ KNitviTüa ¬elIkTI1¦ ry³eBl ³ 60 naTI BinÞú ³ 10


I. BICKNit
1> edaHRsaysmIkar ³ 1 x2x  2 xx 3 .
2> edaHRsayvismIkar ³ 2  1  3x  2x  3 rYcbkRsaycemøIyenAelIG½kS .
3> RsaybBa¢ak;fa ³ x 5 3  xx 29  2xx16  x 2x4 x937 .
2

2 2

4> sRmYlRbPaK ³ A  x    .
2 2
2
 2x  6  x 2  2x  2
x2  4
x 2  3x  2
5> KNnatémøelxénRbPaK ³ F
x2 1
cMeBaH x  1 ; x  3 .
6> enAkñúgtRmúyGrtUNrem xoy  sg;bnÞat; D  : y  1  2x ; D  : y  2x  3
1 1 2 2

rYcrkkUGredaenéncMNucRbsBVrvagbnÞat;TaMgBIr .

II. FrNImaRt
1> tamkMBUlénRtIekaNsm)at DEF eKKUsbnÞat;mYyEdlkat;)at EF  Rtg; G nigkat;rgVg; C 
carwkeRkARtIekaN DEF Rtg; H . eRbóbeFobRtIekaN DEG nigRtIekaN DEH rYcbgðajfa
DE  DG  DH .
2

2> eK[RbelLÚRkam MNPQ EdlmMu Nˆ  120 nigknøHbnÞat;BuHmMu M̂ kat; NP


o

Rtg;cMNuckNþal I .
k> kMNt;ragRtIekaN NMI rYceRbóbeFob MN nig NP .
x>tag J CacMeNalEkgén I elI MQ  . KNnargVas;mMuénRtIekaN MIJ .
K>tag I  CacMNucqøúHén I eFobnwg MQ  . kMNt;ragRtIekaN MII  rYceRbóbeFob IJ nig
MI  .

8
404
9

cemøIy
I. BICKNit 
10 x  30

2x  4

x 2  4x  3

2 x2  9 2 x2  9  2 x2  9   
1> edaHRsaysmIkar ³ 10 x  30  2 x  4  x 2  4 x  3

eyIgman 1 x2x  2 xx 3 2 x2  9  
x 2  4 x  37

smIkarmann½yluHRtaEt x  0 
2 x2  9 
eyIg)an 1 x2x  2 xx 3 eXIjfalT§plénGgÁTI1 dUcGgÁTI2CaR)akd
1  2x  2x  3 ¬lubPaKEbgecal¦ dUcenH 5 x2
 2 
x  1 x 2  4 x  37
 .
1  2x  2x  3 x  3 x  9 2x  6 2 x2  9  
 2 x  2 x  3  1
 4 x  4
4> sRmYlRbPaK ³
x 1 A
x  
2
 2x  6  x 2  2x  2
2
 2

x2  4
x 2  2 x  6  x 2  2 x  2x 2  2 x  6  x 2  2 x  2
dUcenH smIkarmanb£s x  1 . 
x2  4


4 x  42 x 2  8
2> edaHRsayvismIkar ³ x2  4
4 x  1  2x 2  4 
eyIgman 2  1  3x  2x  3 
x2  4
1  3x  2 x  3  8 x  1
x  4 Edl x  4  0  x  4  x  2
2 2

dUcenH vismIkarmancemøIy x  4 . dUcenH sRmÜl)an A  8x  1 .


-bkRsayelIG½kS x  4 x 5> KNnatémøelxénRbPaK F cMeBaHtémø x ³
0
 cMeBaH kmµviFIcas; 
eyIgman F  x x 3x1 2
2

2
EpñkminqUtCacemøIyénvismIkar .
-cMeBaH x  1
x 4 0 x
cMeBaH kmµviFfI µI  eK)an F  1 1 311 2  02  0
2

EpñkKUsDit CacemøIyénvismIkar .
3> RsaybBa¢ak;fa ³ dUcenH cMeBaH x  1 enaH F  0 .
5

x2

x 1

x 2  4 x  37 -cMeBaH x  3
x  3 x 2  9 2x  6 2 x2  9   2

BinitüGgÁTI1 ³ ¬erobcM[dUcGgÁTI2¦ eK)an F  3  3  3  2  5  43 3 2


3 1
5 x2 x 1
 2  53 3
x  3 x  9 2x  6 dUcenH cMeBaH x  3 enaH F .
2  5   x  3 2x  2  x  1x  3 4
  
 
2x  3x  3 2 x  9 2x  3x  3
2

405
9

6> sg;bnÞat; D  nig D  kñúgtRmúy xoy 


1 2 -vi)ak DEG 
DE DG

DHE DH DE
eyIgeRbItaragtémøelxedIm,Isg;bnÞat; Taj)anBIsmamaRt DE  DG  DH 2
x 0 1
D1  : y1  1  2 x :
y 1 1
x 1 2
dUcenH eyIgbgðaj)anfa DE  DG  DH . 2

D2  : y 2  2 x  3 :
y 1 1 2> tambRmab;RbFaneyIgKUsrUb)an ³
D2  : y2  2 x  3 N I
// // P
o
120

M J Q

D1  : y1  1  2 x I
-rkkUGredaenéncMNucRbsBVrvagbnÞat;TaMgBIr ³ k-kMNt;ragRtIekaN NMI
eday D  : y  1  2x nig D  : y  2x  3 bRmab; Nˆ  120 enaH NMˆ Q  60 ¬eRBaH
1 1 2 2
o o

eyIgpÞwmsmIkarGab;sIusénbnÞat;TaMgBIr ³ plbUkmMuCab;RCugEtmYyénRbelLÚRkamesµI 180 ¦ o

1  2x  2x  3 ehIy MI BuHmMu NMˆ Q  60 o

4x  4  x 1
naM[)an NMˆ I  IMˆ J  30 o

cMeBaH x  1 enaH y  1  2x  1  2 1  1
edaykñúg NMI man Nˆ  120 nig NMˆ I  30 o o

dUcenH kUGredaenéncMNucRbsBVKW 1 ,  1 . naM[ NIˆM  30 ¬plbUkmMukñúgén  esµI 180 ¦


o o

II. FrNImaRt dUcenH RtIekaN NMI CaRtIekaNsm)at .


1> tambRmab;RbFaneyIgKUsrUb)an ³
D vi)ak MN  IN
ehIy I CacMNuckNþalén NP enaH IN  12 NP
G
E F
dUcenH eyIgeRbóbeFob)an MN  12 NP .
H

eRbóbeFobRtIekaN DEG nigRtIekaN DEH x- KNnargVas;mMuénRtIekaN MIJ


edayRtIekaN DEG nigRtIekaN DEH man ³ eday J CacMeNalEkgén I elI MQ 
-mMu EDG  EDH ¬mMurYm¦ naM[ IJˆM  90 o

-mMu DEG  DHE eRBaH ehIy IMˆ J  30 ¬sRmaybBa¢ak;xagelI¦


o

mMu DEG  DFE ¬mMu)atén  sm)at¦ eK)an MIˆJ  60 ¬plbUkmMukñúgén  esµI 180 ¦
o o

Et DFE  DHE ¬mMumanFñÚsáat;rYm DE ¦ dUcenH rgVas;mMuénRtIekaN MIJ KW ³


dUcenH DEG DEH tamlkçxNÐ m>m . IJˆM  90 , IMˆ J  30 , MIˆJ  60 .
o o o

406
9

K- kMNt;ragRtIekaN MII 
eday J CacMeNalEkgén I elI MQ 
naM[ II   MJ Rtg; J
ehIy I  CacMNucqøúHén I eFobnwg MQ 
naM[ JI  JI 
eK)an M CacMNucenAelIemdüaT½r MJ
enaHnaM[ MI  MI 
 MI 
-edaykñúg MII  man MIMIˆI   MIˆJ  60 o

mann½yfa vaCaRtIekaNsm)atmanmMumYyesµI 60 o

dUcenH RtIekaN MII  CaRtIekaNsm½gS .


vi)ak MI   II 
ehIy JI  JI  mann½yfa J kNþalGgát; II 
enaH IJ  12 II  b¤ IJ  12 MI 
dUcenH eRbóbeFob)an IJ  12 MI  .

407
9

sm½yRbLg ³ 22 sIha 1995


viBaØasa ³ KNitviTüa ¬elIkTI2¦ ry³eBl ³ 60 naTI BinÞú ³ 10

I. BICKNit
1> eK[ ³ A  x  1  3x  2  x  1  3x  2  .
2 2 2

k> dak;kenSam A CaplKuNktþadWeRkTI 1 én x .


x> sRmYlkenSam B  2x A 2 .
2

K> KNnatémø x kalNa B  3 .


2> edaHRsayvismIkar ³ 5x5 4  6x  2  4  x 3 5 . rYcbkRsaytamRkaPic .
3> k> edaHRsayRbB½n§smIkartamRkaPic ³ 33xx  2y y7 0 0 .

x> rYcepÞógpÞat;cemøIytamkarKNna .

II. FrNImaRt
eK[RtIekaNEkgsm)at ABC EkgRtg; A ehIy AB  AC  a . DAC k¾CaRtIekaNEkg
sm)at EkgRtg; D . RtIekaNTaMgBIrenHsßitenAsgxagRCugrYm AC  .
1> kMNt;RbePTctuekaN ABCD .
2> KNna BC ; AD ; BD CaGnuKmn_én a .
3> Ggát;RTUg AC  nig BD RbsBVKñaRtg; O . KNna OA , OC , OB , OD CaGnuKmn_én a .
4> bgðajfa AOD nig BOC CaRtIekaNdUcKña . rYcKNnapleFobdMNcU .

408
9

cemøIy
I. BICKNit 2> edaHRsayvismIkar ³
1> man A  x  1  3x  2  x  1  3x  2 
2 2 2
eyIgman 5x5 4  6x  2  4  x 3 5
k> dak;kenSam A CaplKuNktþadWeRkTI 1 én x eyIgtRmÚvPaKEbgrYm 15 rYclubPaKEbgrYmecal
2

A  x  1  3x  2  x  1  3x  2
2 2
 15x  12  90 x  30  60  5 x  25
 x  1  2x  13x  2  3x  2  75x  42  5 x  85
2 2

 x  1  3x  2  70 x  43
2 2

 2x  13x  2 x


43
70
dUcenH A  2x  13x  2 CaplKuNktþa . dUcenH vismIkarmancemøyI x
43
.
70
x> sRmYlkenSam B  2x A 2 2 - bkRsaytamRkaPic ³
eday A  2x  13x  2
43

x 70 0 x

eK)an B   2x2x132x  2
 cMeBaH kmµviFIcas; 
2

 2x  13 x  2 
EpñkminqUtCacemøIyénvismIkar


2 x2 1  x

43

 2x  13 x  2 
 cMeBaH kmµviFIfµI 
0 x 70


2x  1x  1 EpñkKUsDitCacemøIyénvismIkar .
 3 x  2 

x  1
, x 1  0  x  1 3> k> edaHRsayRbB½n§smIkartamRkaPic ³
dUcenH smRmÜl)an B. 3x  2 eyIgman ³ 33xx  2y y7 0 0
x 1 
K> KNnatémø x kalNa B  3 eRbItaragtémøelxedIm,Isg;RkaPicénbnÞat;TaMgBIr
smIkar B mann½ykalNa x 1 0 b¤ x  1 
3 x  2 y  0
x 0 2
 y 0 3
eK)an 3   3xx1 2 
3 x  y  7  0 x  2 1
3 x  3  3 x  2  y 1 4
3x  3x  2  3 eyIgsg;RkaPicdUcrUbxageRkam ³
 
3 3 x  2 3
3x  2 y  0
2 3
x
3 3 7

x
2  3  3 3  3

 3  3 3 3  
14

2 3 633 3 95 3 3x  y  7  0 9
x 
39 6
tamRkaPic
dUcenH KNna)antémø x  9  65 3
. dUcenH bnÞat;TaMgBIrRbsBVKñaRtg;  14 7 
 ,  .
 9 3

409
9

x> epÞógpÞat;cemøIytamkarKNna ³ 2> KNna BC ; AD ; BD CaGnuKmn_én a


eyIgman 33xx  2y y7 0 0 b¤ 33xx  2y y07 eday ABC CaRtIekaNEkgsm)at EkgRtg; A
 
man AB  AC  a tamRTwsþIbTBItaK½r eK)an³
edaydkGgÁnigGgÁ énsmIkarTaMgBIr BC 2  AB2  AC 2
3 x  2 y  0
 BC  AB2  AC 2
eK)an 3 x  y  7 naM[ y
7
3y  7 3  a 2  a 2  2a 2  a 2

cMeBaH y
7
3
enaH 3x  y  7 dUcenH KNna)an BC  a 2 .
7
3x   7
3
eday DAC CaRtIekaNEkgsm)at EkgRtg; D
3x  
14
 x
14 naM[ AD  CD
3 9
tamRTwsþIbTBItaK½r AC  AD  CD
2 2 2

dUcenH  x   149 , y  73  dUcKNnatamRkaPic . AC 2  2AD2 /


II. FrNImaRt Taj)an AD 2 
AC 2
 AD 
AC
2 2
tambRmab;RbFaneyIgsg;rbU )andUcxageRkam ³
B
Et AC  a naM[ AD 
a
2

a 2
2

dUcenH KNna)an AD  a 2 2 .
O kñúgRtIekaNEkg BCD EkgRtg; C
A C
¬eRBaH BCD man DCˆB  90 bBa¢ak;xagelI¦
o

D tamRTwsþIbTBItaK½r eK)an ³
1> kMNt;RbePTctuekaN ABCD BD 2  BC 2  CD 2
eday ABC CaRtIekaNEkgsm)at EkgRtg; A BD  BC 2  CD 2

naM[ mMu)at ABˆ C  ACˆB  45 o


BD     a 
a 2 
2

2

 2
ehIy DAC CaRtIekaNEkgsm)at EkgRtg; D
a2 5a 2 10a 2 a 10
naM[ mMu)at DAˆ C  DCˆA  45 o BD  2a 2 
2

2

4

2
eK)an DCˆB  DCˆA  ACˆB  45  45  90 o o o

dUcenH KNna)an BD 
a 10
.
naM[ BC  CD Rtg; C 2

-kñúgctuekaN ABCD manmMuEkg DCˆB  90 o


3> KNna OA , OC , OB , OD CaGnuKmn_én a
AD  CD eday AC  nig BD RbsBVKñaRtg; O
nig BC  CD
 AD // BC

dUcenH ABCD CactuekaNBñayEkg . ehIy AD// BC ¬sRmayxagelI¦

410
9

tamRTwsþIbTtaEls eyIg)anGgát;smamaRtKña vi)ak OAD



OA OD AD
 
OA OD AD OCB OC OB CB
  a
OC OB CB
tamlkçN³smamaRt eK)an ³ Et OA a 3 1
 3   
OC 2a 3 2a 2
OA OC OA  OC AC 3
   
AD CB AD  CB AD  CB
naM[ OA OD AD 1
 
OC OB CB 2

a a a 2
   
a 2  2a 2 3a 2 3
a 2
2
a 2
2 2
dUcenH pleFobdMNUcKNna)anKW 12 .
a 2
2
naM[ OA
AD 3

2
 OA 
2 AD
3

3
2 a
3 rUbdEdl RKan;EtRsYsemIlb:ueNÑaH ¡¡¡¡
2  a 2 2a
ehIy OC
CB

3
2
 OC 
2 BC
3

3

3
B

dUcenH KNna)an OA  a3 nig OC  2a3 .


O
-kñúgRtIekaNEkg ABO EkgRtg; A A C

tamRTwsþIbTBItaK½r OB  AB  OA2 2 2
D
OB  AB  OA 2 2

2
a a2 10a 2 a 10
 a     a2 
2
 
3 9 9 3

- eday OD  BD  OB  a 210  a 310


3a 10  2a 10

6
a 10

6

dUcenH OB 
a 10
3
nig OD  a 610 .
4> bgðajfa AOD nig BOC CaRtIekaNdUcKña
eday AOD nig BOC man³
-mMu OAˆ D  OCˆB  45 ¬mMu)atRtIekaNsm)at¦
o

-mMu AOˆ D  COˆ B ¬mMuTl;kMBUl¦


dUcenH AOD BOC tamlkçxNÐ m>m .

411
9

sm½yRbLg ³ 01 kkáda 1996


viBaØasa ³ KNitviTüa ¬elIkTI1¦ ry³eBl ³ 60 naTI BinÞú ³ 10


I. BICKNit
1> KNnakenSam E   3   2  5 .
3 4 2

2> sresrBhuFaxageRkamCaplKuNktþa ³
k> x 18x  18
2

x> 4x  25 .2

3> eK[smamaRt ba  dc bgðajfa ac  22ca 33db .


4> eK[bnÞat;BIr EdlmansmIkar D : y  x  5 nig D : 2 y  3x .
k> sg;bnÞat; D  nig D kñúgtRmúyGrtUNrem xoy  .
x> R)ab;emKuNR)ab;TisénbnÞat;nImYy² .
K> KNnakUGredaencMNucRbsBV P rvag D  nig D .
5> edaHRsaysmIkar 2 x3  7  x  1 .
II. FrNImaRt
eK[RtIekaN ABC EkgRtg; A .
1> I CacMNuckNþalén BC  . R)ab;eQµaHRtIekaN IAC .
2> O CacMNuckNþalén AB . R)ab;eQµaHénctuekaN OICA .
3> KNna BC ebI AB  3cm ; AC  4cm .
4> rgVg;Ggát;p©it AB kat;rgVg;Ggát;p©it AC  Rtg;cMNucmYyeTot M .
k> bBa¢ak;fa M sßitenAelI BC  .
x> bgðajfa AB  BM  BC .
2

412
9

cemøIy
I. BICKNit -eyIgsg; D  nig D )andUcrUbxageRkam ³
1> KNnakenSam E ³  D  : 2 y  3 x
eyIgman E   3   2 3 4
5 2

 27  16  25
D  : y  x  5
 36

dUcenH KNna)an E  36 .


2> sresrBhuFaxageRkamCaplKuNktþa ³ x> R)ab;emKuNR)ab;TisénbnÞat;nImYy²
k> x 18x  81  x  2  x  9  9
2 2 2 bnÞat; D : y  x  5 b¤ D : y   x  5
 x 2  2  x  9  92
dUcenH emKuNR)ab;TisénbnÞat; D  KW a  1
 x  9  x  9x  9
2

dUcenH x  18 x  81  x  9 x  9 
2
. bnÞat; D   : 2 y  3 x b¤ D   : y  3 x
2

x> 4 x 2
 25  2 x   5 2
2
dUcenH emKuNR)ab;TisénbnÞat; D KW a  32 .
 2 x  52 x  5 / K> KNnakUGredaencMNucRbsBV P
dUcenH 4 x 2  25  2 x  52 x  5 . eyIgman D : y   x  5 nig D : y  32 x
3> bgðajfa ac  22ca 33db edaypÞwmsmIkarGab;sIusénbnÞat;TaMgBIr ³
eyIgmansmamaRt ba  dc enaH ac  db eyIg)an  x  5  32 x
3
tamlkçN³smamaRt eyIgGacsesr)an ³ 2
xx5
a b 2a 3b 2a  3b 3 
      1 x  5
c d 2c 3d 2c  3d 2 
a 2a  3b
Taj)an
c 2c  3d
5
x5  x2
2
dUcenH eyIgbgðaj)anfa a 2a  3b
 . cMeBaH x  2 enaH y  x  5  2  5  3
c 2c  3d

4> k> sg;bnÞat; D  nig D kñúgtRmúy xoy  dUcenH kUGredaencMNucRbsBV P2 , 3 .
eyIgeRbItaragtémøelxedIm,Isg;bnÞat;TaMgenH 5> edaHRsaysmIkar 2 x3  7  x  1
D  : y  x  5 ³
x 1 2
y 4 3 smIkar enHmanGBaØat x enAeRkamr:aDIkal;
smIkarmann½yluHRtaEt x  0
D   : 2 y  3 x ³ x 0 2
y 0 3 eK)an ³
413
9

2 x 7
 x 1 3> KNna BC ebI AB  3cm ; AC  4cm
3
2 x 7  3 x 3
eday RtIekaN ABC CaRtIekaNEkgRtg; A
3 x  2 x  7  3 tamRTwsþIbTBItaK½r eK)an ³
x  4  0 BC 2  AB2  AC 2
BC  AB2  AC 2
dUcenH smIkarKµanb£s .  32  4 2  25  5 cm

II. FrNImaRt dUcenH KNna)an BC  5 cm .


tambRmab;RbFaneyIgsg;rbU )an ³ 4> k> bBa¢ak;fa M sßitenAelI BC 
B
// M
-edayrgVg;Ggát;p©it AB man M enAelIrgVg;
3 cmO 
I
naM[ mMu AMB  90 ¬CamMucarwkknøHrgVg;¦
o

//
C
-edayrgVg;Ggát;p©it AC  man M enAelIrgVg;
A 4 cm
naM[ mMu AMC  90 ¬CamMucarwkknøHrgVg;¦
o

-edaymMu BMˆ C  AMˆ B  AMˆ C  90  90


o o

1> R)ab;eQµaHRtIekaN IAC ³ naM[ BMˆ C  180 CamMurab


o

-eday I CacMNuckNþalén BC  enaH AI Ca mann½yfa M sßitenAelI BC 


emdüan én  Ekg ABC RtUvnwgGIub:Uetnus BC 
tamRTwsþI eK)an IA  IB  IC dUcenH eyIgbBa¢ak;)anfa M sßitenAelI BC  .
-edayRtIekaN IAC man IA  IC x> bgðajfa AB  BM  BC
2

dUcenH RtIekaN IAC CaRtIekaNsm)at . eday AMB  90 o

naM[ BM CacMeNalEkgénGgát; ABelI BC 


2> R)ab;eQµaHénctuekaN OICA ³ tamTMnak;TMngkñúgRtIekaNEkg ABC eK)an ³
-eday I CacMNuckNþalén BC  AB  BM  BC
2
/
nig O CacMNuckNþalén AB
naM[ OI KWCa)atmFüménRtIekaNEkg ABC dUcenH eyIgbgðaj)anfa AB  BM  BC .
2

EkgRtg; A
vi)ak AC// OI
-eXIjfa ctuekaN OICA man AC// OI
nigmMu BAˆ C CamMuEkg
dUcenH ctuekaN OICA CactuekaNBñayEkg .
414
9

415
9

sm½yRbLg ³ 19 sIha 1996


viBaØasa ³ KNitviTüa ¬elIkTI2¦ ry³eBl ³ 60 naTI BinÞú ³ 10

I. BICKNit
1> BnøatplKuN ³ 2x  3x  2 .
2> sresrkenSamxageRkamCaplKuNktþa ³
k> 5x  2  42  x
2 2

x> 2 x  5  4 x  10 x  2  4 x  25
2 2
.
3> KNna 20  80  245 .
4> k> edaHRsaysmIkar 1x  x 2 1  x 3 x .
2

x> edaHRsayvismIkar 42  x  3  2x 1  4 .


5> edaHRsayRbB½n§smIkar 2yx 2 yx 6 .

II. FrNImaRt
eK[rgVg;p©it O manGgát;p©it AB nigcMNuc P enAeRkArgVg;enH . bnÞat; PA nig PB  kat;rgVg;
O erogKñaRtg;cMNuc M nig N enAmçagénbnÞat;  AB  .

1> RsaybMPøWfa AN  nig BM  Cakm<s;énRtIekaN PAB .


2> H CacMNucRbsBVrvag AN  nig BM . RsaybMPøWfa PH   AB .
3> bgðajfactuekaN HMPN carwkkñúgrgVg;mYy .
4> RsaybBa¢ak;fa AN  BP  BM  AP .
5> cMNuc I cl½tenAelIFñÚtUc AM ehIy E CacMNuckNþalén AI . rksMNMcu MNuc E kalNa
I ERbRbYl .

416
9

cemøIy
I. BICKNit eyIg)an 1

2
 2
3
x x 1 x  x
1> BnøatplKuN ³ 1

2

3
x x  1 x x  1
eyIgman 2 x  3x  2  2 x 2
 4 x  3x  6
x  1  2x  3
 2x 2  x  6 / x2
x  2
dUcenH Bnøat)an 2x  3x  2  2x  x  6 2
.
2> sresrkenSamxageRkamCaplKuNktþa ³ dUcenH smIkarmanb£s x  2 .
k> 5x  2  42  x
2 2 5> edaHRsayRbB½n§smIkar ³
 5 x  2   22  x 
2 2
eyIgman 2yx 2 yx 6 smmUl xx  yy  32
 5 x  2  22  x 5 x  2  22  x   
 5 x  2  4  2 x 5 x  2  4  2 x  RbB½n§smIkarmanpleFobemKuN a b c
 
a  b c 
 7 x  6 3 x  2 
KW 11   11  32 mann½yfa vaKµanKUcemøIy
dUcenH 5x  2  42  x  7 x  63x  2
2 2

x> 2 x  5  4 x  10 x  2  4 x  25
2 2
dUcenH RbB½n§smIkarKµanKUcemøIy .
 2 x  5  22 x  5x  2  2 x   5  II. FrNImaRt
2 2 2

 2 x  5  22 x  5x  2  2 x  52 x  5


2

 2 x  52 x  5  2 x  4  2 x  5
tambRmab;RbFaneyIgsg;rbU )an ³
P
 2 x  52 x  14
 22 x  5x  7  I M
E
N
E
dUcenH 2 x  5  4 x  10 x  2  4 x
2 2
 25 H
A  B
 22 x  5x  7 . O

3> KNna 20  80  245


 4  5  16  5  49  5 1> bMPøWfa AN  nig BM  Cakm<s;én PAB
 2 5 4 5 7 5 kñúgrgVg;p©it O manGgát;p©it AB ehIycMNuc M
5 5
nig N enAelIrgVg; naM[ ANˆ B nig AMˆ B CamMu
dUcenH KNna)an 20  80  245  5 5 . carwkknøHrgVg; mann½yfa ANˆB  90 / AMˆ B  90
o o

4> k> edaHRsaysmIkar 1x  x 2 1  x 3 x 2


vi)ak AN  BN b¤ AN  BP
x  0 AM  BM b¤ AP  BM
x  0
smIkarmann½yluHRtaEt 
x  1  0  
x 2  x  0

 x  1 dUcenH AN  nig BM  Cakm<s;én PAB .
417
9

2> RsaybMPøWfa PH   AB 5> rksMNMucNM uc E kalNa I ERbRbYl


bRmab; H CacMNucRbsBVrvag AN  nig BM  eday E CacMNuckNþalén AI 
mann½yfa H sßitenAelIkm<s;TaMgBIrén PAB naM[ EA  EI
naM[ H CaGrtUsg;énRtIekaN PAB enaH H eyIg)an OE  AI
k¾sßitenAelIkm<s;TIbIEdr ¬eRBaH Ggát;FÚñEkgnwgkaMrgVg;Rtg;cMNuckNþal¦
eK)an PH  Cakm<s;TIbIRtUvnwgRCug AB edaycMNuc A , O CacMNucnwg
dUcenH PH   AB . enaHnaM[mMu AEˆO  90 efr
o

edIm,I[ AEˆO  90 efr luHRtaEtcMNuc E RtUv


o

3> bgðajfactuekaN HMPN carwkkñúgrgVg;mYy sßitenAelIknøHrgVg;Ggát;p©it AO


tamsRmayxagelI ANˆB  90 nig AMˆ B  90
o o

-ebI I RtYtelI M enaH E RtYtelI E Edl E


naM[ ANˆP  90 ¬mMubEnßmCamYymMu ANˆB  90 ¦
o o

CacMNuckNþalén AM 
PMˆ B  90 ¬mMubEnßmCamYymMu AMˆ B  90 ¦
o o

-ebI I RtYtelI A enaH E k¾RtYtelI A Edr


edayctuekaN HMPN manmMuQm ³
-mMu HNˆP  ANˆP  90 o dUcenH sMNMucMNuc E Ca  AE énrgVg;
-mMu PMˆ H  PMˆ B  90 o Ggát;p©it AO .
naM[plbUkmMQ u m HNˆP  PMˆ H  90  90  180
o o o

dUcenH ctuekaN HMPN carwkkñúgrgVg;mYy .


4> RsaybBa¢ak;fa AN  BP  BM  AP
kñúgRtIekaN PAB man ³
-km<s; AN RtUvnwg)at BP
naM[ S  12 AN  BP
PAB

-km<s; BM RtUvnwg)at AP
naM[ S  12 BM  AP
PAB

-edayvaCaépÞénRtIekaNEtmYy enaHeKpÞwm)an ³
1 1
AN  BP  BM  AP
2 2

AN  BP  BM  AP

dUcenH Rsay)anfa AN  BP  BM  AP .
418
9

419
9

RksYgGb;rM yuvCn nigkILa elxbnÞb; ³ >>>>>>>>>>>>>>>>>>>>>>>>>>>>>>>>>>>>>>>


RbLgsBaØabRtmFümsikSabzmPUmi cMeNHTUeTA nigbMeBjviC¢a elxtu ³ >>>>>>>>>>>>>>>>>>>>>>>>>>>>>>>>>>>>>>>>>>>>
eQµaH nightßelxaGnurkS sm½yRbLg ³ >>>> >>>>>>>>>> 1997 mNÐlRbLg ³ >>>>>>>>>>>>>>>>>>>>>>>>>>>>>>>>>>
1> >>>>>>>>>>>>>>>>>>>>>>>>>>>>>>>>>>>>>> namRtkUl nignamxøÜn ³ >>>>>>>>>>>>>>>>>>>>>>>>>>>>>>>>>
2> >>>>>>>>>>>>>>>>>>>>>>>>>>>>>>>>>>>>>> éf¶ExqñaMkMeNIt ³ >>>>>>>>>>>>>>>>>>>>>>>>>>>>>>>>>>>>>>>>>>> GkSrsm¶at;
htßelxa ³ >>>>>>>>>>>>>>>>>>>>>>>>>>>>>>>>>>>>>>>>>>>>>>>>>>>
 ebkçCnminRtUveFVIsBaØasmÁal;Gm VI YyelIsnøwkRbLgeLIy. snøwkRbLgNaEdlmansBaØasmÁal;RtUv)anBinÞúsUnü .
--------------------------------------------------------------------------------------------------------------------------
viBaØasa ³ KNitviTüa elIkTI1 ry³eBl ³ 60 naTI BinÞú ³ 100
esckþIENnaM ³ GkSrsm¶at;
1> ebkçCnRtUvbt;RkdasenHCaBIr rYcKUsExVgEpñkxagelIénTMB½rTI2 [b:unRbGb;EpñkxagelI
énTMB½rTI1 EdlRtUvkat;ecal. hamsresrcemøIyelIkEnøgKUsExVgenaH .
2> ebkçCnRtUvKUsbnÞat;bBaÄr[cMBak;kNþalTMB½rTI2 nigTMB½rTI3 sRmab;sresrcemøIybnþ.
RbeTsmanPaBvwkvr mankar)aj;KñaenAraCFanIPñMeBj kalBIéf¶TI 05 nig 06 Ex kkáda qñaM 1997 .
dUcenH kñúgqñaM 1997 enHBMmu ankarRbLgeLIy .

420
9

cemøIy

421
9

sm½yRbLg ³ 24 sIha 1998


viBaØasa ³ KNitviTüa ¬elIkTI1¦ ry³eBl ³ 60 naTI BinÞú ³ 100


I. RsaybMPøWfa E  3 2 1 2 1  2 1 CacMnYnKt; .


2

II. eK[KUbBIrRbePTcMnYnsrub 11. KUbmYyRbePTmanRTnugesµI 3 cm nigKUbmYyRbePTeTotmanRTnug


esµI 5 cm . eKmindak;KUbRtYtKñaeT . eKtMerobKUbTaMg 11 CamYyCYrEdlmanRbEvg 47 cm .
rkcMnYnKUbénRbePTnImYy² .
III. eK[tRmúyGrtUNrem xoy  cUrsg;bnÞat;EdlmansmIkar y  2x  1 nig y  2x  2 . rYcKNna
cMNucRbsBVrvagkUGredaenéncMNuc EdlCaRbsBVrvagbnÞat;TaMgBIr .
IV. eK[knøHrgVg;p©it O Ggát;p©it AB. cMNuc M cl½tenAelIknøHrgVg;enH .
1> R)ab;RbePTRtIekaN AMB .
2> KNna AM ebI OA  2cm , BM  7cm .
3> rksMNMucMNuc P CacMNuckNþalén AM  .

8
cemøIy
I. RsaybBa¢ak;fa E CacMnYnKt; ³ naM[ D  53  2

eyIgman E  3 2 1 2 1  2 1 2


Dx  55  47  8  x 
Dx 8
 4
  2  13 2  1  2  1
D 2
D y 14
  2  12 2  2
D y  47  33  14  y   7
D 2
 2 2  1 2  1  22  1  2
dUcenH KUbmanRbEvgRTnugesµI 3cm mancMnYn 4
dUcenH E  2 CacMnYnKt; . KUbmanRbEvgRTnugesµI 5cm mancMnYn 7 .
II. rkcMnYnKUbénRbePTnImy Y ²³ III. sg;bnÞat;kñúgtRmúyGrtUNrem  xoy 
tag x CacMnYnKUbEdlmanRbEvgRTnugesµI 3 cm eyIgeRbItaragtémøelxedIm,Isg;bnÞat;TaMgenH
y CacMnYnKUbEdlmanRbEvgRTnugesµI 5 cm -cMeBaH y  2x  1 ³ xy 10 22
tambRmab;RbFaneyIg)anRbB½n§smIkar ³
 x  y  11 -cMeBaH y  2x  2 ³ x 0 2


3x  5 y  47
eyIgedaHRsaytamedETmINg; ³ y 2 1

422
9

-eyIgsg;bnÞat;TaMgBIr)andUcxageRkam ³ 2> KNna AM ebI OA  2cm , BM  7cm

y
x
1
eday AMB CaRtIekaNEkg Rtg; M
2
tamRTwsþIbTBItaK½r
AB2  AM 2  BM 2
AM 2  AB2  BM 2
x
y  2 AM  AB2  BM 2
2
AM  2OA2  BM 2 , AB  2OA

-KNnakUGredaenéncMnucRbsBVrvagbnÞat;TaMgBIr AM  2  22  7
2

eyIgman y  2x  1 nig y  2x  2 AM  16  7  9  3 cm

edaypÞwmsmIkarGab;sIus enaHeK)an ³ dUcenH KNna)an AM  3 cm .


x
x
2
1 
2
2 3> rksMNMucNM uc P CacMNuckNþalén AM 
x x
  2 1 eday P CacMNuckNþalén AM 
2 2
x 1 naM[ PA PM
cMeBaH x  1 eyIg)an OP  AM
enaH y  2x  1  12  1  2 ¬eRBaH Ggát;FÚñEkgnwgkaMrgVg;Rtg;cMNuckNþal¦
edaycMNuc A , O , B CacMNucnwg
dUcenH cMNcu RbsBVrvagbnÞat;TaMgBIrKW enaHnaM[mMu APˆ O  90 efr
o

mankUGredaen  x  1 , y  2  . edIm,I[ APˆ O  90 efr luHRtaEtcMNuc P RtUv


o

sßitenAelIknøHrgVg;Ggát;p©it AO
IV. tambRmab;RbFaneyIgsg;rUb)an ³ -ebI M RtYtelI B enaH P RtYtelI O
M
P
-ebI M RtYtelI A enaH P RtYtelI A Edr
A  B dUcenH sMNMucMNuc P CaknøHrgVg;Ggát;p©it AO .
O
1> R)ab;RbePTRtIekaN AMB
edayRtIekaN AMB man AB CaGgát;p©it nig
M CacMNuccl½telIknøHrgVg; enaH AMB Ca

RtIekaNcarwkknøHrgVg; Edl AMˆ B  efr  90 o

dUcenH AMB CaRtIekaNEkg .

423
9

sm½yRbLg ³ 09 tula 1998


viBaØasa ³ KNitviTüa ¬elIkTI2¦ ry³eBl ³ 60 naTI BinÞú ³ 100

I. eKEckR)ak;[mnusS 3 nak;. GñkTImYy)an 40% énR)ak;srub ehIyGñkTIBrI TTYl)an 15 énR)ak;srub
ÉGñkTIbITTYl)anR)ak; 12 000 erol . rkcMENkR)ak;rbs;GñkTImyY nigcMENkR)ak;rbs;GñkTIBIr .
II. bnÞat;b:HrgVg;p©it O Rtg; M kat;bnøayénGgát;p©it CD Rtg; P . bgðajfaRtIekaN PMC nig
RtIekaN PDM dUcKña rYcTajbBa¢ak;fa PM  PC  PD .
2

III. cUrsresrBakü {xus} b¤ {RtUv} kñúgRbGb;enAxagmuxGMNHGMNagnImYy²xageRkamenH ³


□ a 2  b 2  2ab

□ 2 1 
1
2 1

□ bnÞat; y  2x 1 nig y  2x  2 RbsBVKñaRtg;mYycMNcu .


IV. cUrKUssBaØa  kñúgRbGb;enAxagmuxcemøIyEdlRtwmRtUvmanEtmYyKt; ³
1> etIkenSamNaEdlesµInwg x  2 ³
2

k> □ x  2x  4
2
x> □ x  4x  4
2

K> □ x  4x  2
2
X> □ x  4x  4
2

2> etIcMnYnNa Cab£sénsmIkar ³ x x 2 x  0 .


2

k> □ 0 x> □  2
K> □ 2 X> □ 0 nig 2 .
V. ABC CaRtIekaNEkgRtg; A ehIy x  0 . eKdwgfa BC  3x  2 nig AB  2 x  1 .

KNna AC CaGnuKmn_én x .
2

VI. eK[cMNuc A1 , 2 nig B3 , 1 . rksmIkarbnÞat; AB .

424
9

cemøIy
I. rkcMENkR)ak;rbs;GñkTImYy nigGñkTIBIr ³ vi)ak PMC 
PM PC

PDM PD PM
tag x CacMnYnR)ak;srubTaMgGs; Taj)an PM  PM  PC  PD
naM[ R)ak;rbs;GñkTI1 KW 40% x b¤ PM  PC  PD 2

R)ak;rbs;GñkTI2 KW 15 x
dUcenH TajbBa¢ak;)anfa PM  PC  PD . 2

R)ak;srubCaplbUkR)ak;GñkTaMgbI eyIg)an ³
1 III. sresrBakü {xus} b¤ {RtUv} kñúgRbGb; ³
x  40%  x   x  12000
5 RtUv a  b  2ab eRBaH
2 2
5 x  200% x  x  60000
5 x  2 x  x  60000
a 2  b 2  2ab
2 x  60000
a 2  2ab  b 2  0
x  30000
a  b2  0
naM[ -cMENkR)ak;rbs;GñkTI1 KW BitcMeBaHRKb;cMnYnBit a nig b .
40% x 
40
100
 30000  12 000 erol RtUv 2 1 
1
eRBaH
2 1
-cMENkR)ak;rbs;GñkTI2 KW 1
2 1 
1 1
x   30000  6 000 erol 2 1
5 5
 2 1  2 1  1
dUcenH cMENkR)ak;GñkTI1 KW 12 000 erol 2
2  11  1
cMENkR)ak;GñkTI2 KW 6 000 erol . 11 Bit
II. tambRmab;RbFaneyIgKUsrUb)andUcxageRkam³
bnÞat; y  2x  1 nig y  2x  2
xus

M
RbsBVKñaRtg;mYycMNuc . eRBaH
bnÞat;TaMgBIrmanemKuNR)ab;TisesµI 2 dUcKña
P C O

D naM[ bnÞat;TaMgBIrRsbKña KµancMNucRbsBVmYy .
IV. KUssBaØa  kñúgRbGb;muxcemøIyEdlRtwmRtUv ³
bgðajfa PMC nig PDM dUcKña 1> etIkenSamNaEdlesµInwg x  2 ³ 2

eday PMC nig PDM man ³ x> ☑ x  4x  4 2

-mMu PMC  PDM ¬mMucarwkFñÚsáat;rYm MC ¦ eRBaH x  2  x  2  x  2  2  x  4x  4


2 2 2 2

-mMu MPC  DPM ¬mMu P rYmKña ¦ 2> etIcMnYnNa Cab£sénsmIkar ³ x x 2 x  0


2

eXIjfa PMC nig PDM manmMuBIrb:unerogKña K> ☑ 2 eRBaH cMeBaH x  2


dUcenH PMC PDM tamlkçxNÐ m>m . 2  22 2
0 
44
 0  0  0 Bit
2 2

425
9

V. tambRmab;RbFaneyIgKUsrUb ³ C

3x  2

KNna AC CaGnuKmn_én x A 2x 1 B
2

tamRTwsþIbTBItaK½r cMeBaH ABC EkgRtg; A


eK)an BC  AB  AC
2 2 2

naM[ AC  BC  AB
2 2 2

AC 2  BC 2  AB 2
 3 x  2   2 x  1
2 2

 3 x  2  2 x  13 x  2  2 x  1
 x  15 x  3
 5x 2  8x  3

dUcenH KNna)an AC 2  5x 2  8x  3 .
V. rksmIkarbnÞat; AB
eyIgmancMNuc A1 , 2 nig B3 , 1
smIkarbnÞat;EdlRtUvrkmanrag AB  : y  ax  b
-eday AB  : y  ax  b kat;tam A1 , 2
eK)an 2  a 1  b  a  b  2 1
-eday AB  : y  ax  b kat;tam B3 , 1
eK)an 1  a  3  b  3a  b  1 2
-edayyk smIkar 2  1 eK)an ³
3a  b  1

a  b  2 naM [ a
1
2a  1 2
cMeBaH a   2 enaHtam 1 a  b  2
1

naM[ b  2  a  2    12   2  12  52


dUcenH smIkarbnÞat;  AB : y   1 x  5
2 2
.

426
9

427
9

sm½yRbLg ³ 07 sIha 1999


viBaØasa ³ KNitviTüa ¬elIkTI1¦ ry³eBl ³ 60 naTI BinÞú ³ 100

C
I. cUrsresrBakü {xus} b£ {RtUv} kñúgRbGb;xagmuxGMNHGMNagnImYy²xageRkam ³
1> □ x  6 Cab£sénsmIkar 2x  6  3x  19 . 6

2> □ BC  5 ebIeK[ ³


C
2 1.5
B
  
BC B C , AB  1 , AC  2 , CC  6 , BC  1.5 . A 1 B

II. cUrKUssBaØa  kñúgRbGb;enAxagmuxcemøIyEdlRtwmRtUv ³


1> eK[ A 1 , 1 nig B3 , 4 enaHcm¶ay AB manRbEvg ³
k> □ AB  13 x> □ AB  6
K> □ AB  5 X> □ AB  4 .
2> KNnakenSam E  5 12  4 3  48 KW ³
k> □ E  8 3 x> □ E  9 3
K> □ E  10 3 X> □ E  11 3 .
3> eKRkLúkRKab;LúkLak;BIr rkRbU)abEdlGac[eKRkLúk)anBinÞúsrub 7 BinÞú .
k> □ P   365
7 x> □ P   56
7

K> □ P   16
7 X> □ P   13 .
7

III. sYnbEnømYyragctuekaNEkg manbeNþayelIsTTwg 12 m ehIydIenaHmanépÞRkLa 160 m . 2

rkbrimaRténsYnbEnøenaH .
IV. enAkñúgtRmúyGrtUNrem xoy  .
k> sg;bnÞat; D  EdlmansmIkar y  2x  3 .
1

x> sresrsmIkarbnÞat; D  Edlkat;tamcMnuc P5 , 1 ehIyRsbnwg D . rYcsg; D  kñúg


2 1 2

tRmúyxagelI .
V. kñúgrgVg;p©it O Ggát;p©it BC  Edl BC  5cm . eKKUsGgát;FñÚ BA Edl BA  3cm .
k> R)ab;RbePTRtIekaN ABC .
x> KNnaRbEvg AC .
428
9

K> KNna cos ABˆ C nig sin ABˆ C .


X> eKKUsemdüan BM  énRtIekaN ABC Edlbnøayrbs;vakat;rgVg; O Rtg; D .
bgðajfa MB MD  MA . 2

8
cemøIy
I. sresrBakü {xus} b£ {RtUv} kñúgRbGb; ³ 1, 6 ; 2 , 5 ; 3, 4 ; 4 , 3 ; 5 , 2 ; 6 ,1
1> xus x  6 eRBaH cMeBaH x  6 mancMnYn 6 krNI enaHcMnYnkrNIRsb  6
eK)an 26  6  3  6  19 naM[ P   krNI
7
Rsb 6 1

krNIGac 36 6
 .
0  1 minBit III. rkbrimaRténsYnbEnøenaH
2> xus BC  5 eRBaHebIeK[ ³ BC BC C tag x CaRbEvgTTwg Edl x  0 KitCa m
tamlkçN³bnÞat;RsbeK)an ³ 6 naM[ x 12 CaRbEvgbeNþay sYn
AC BC

AC  B C 
C
tambRmab;RbFan eK)an ³
x x  12   160
2 1.5
AC   BC B
160 m 2
B C   A 1 B
x
AC x  12 x  160  0
2


2  6 1.5  12  6 man   6   160   196
2
x  12

2 2
II. KUssBaØa  kñúgRbGb;enAxagmuxcemøIyRtwmRtUv ³ naM[ x   6 1 196  6  14  0 minyk
1

1> enaHcm¶ay AB manRbEvg ³ K> ☑ AB  5 x2 


 6  196
 6  14  8 m
eRBaH eyIgman A 1 , 1 nig B3 , 4 1
naM[RbEvgbeNþay x  12  8  12  20 m
naM[ AB  3   1  4  1
2 2

-ebItag P CabrimaRténsYnbEnøenaH
 4  3  25  5 .
2 2

eK)an P  28  20   56 m
2> KNnakenSam E  5 12  4 3  48 KW ³
K> ☑ E  10 3 dUcenH brimaRténsYnbEnøenaHKW P  56 m .
eRBaH E  5 12  4 3  48
 10 3  4 3  4 3  10 3 .
IV. k> sg;bnÞat; D  EdlmansmIkar y  2x  3
1

3> rkRbU)abEdleKRkLúk)anBinÞúsrub 7 BinÞú ³ eyIgeRbItaragtémøelxedIm,I sg;bnÞat;enH


K> ☑ P   16 eRBaH RKab;LúkLak;nImYy²man
7
cMeBaH D  ³ y  2x  3 xy 03 24
1

mux 6 dUcKña naM[cMnnY krNIGacKW 6  6  36 eyIgsg;bnÞat; D  kñúgtRmúyGrtUNrem xoy 


1

ehIyplbUkRKab;LúkLak;)anBinÞú 7 BinÞú rYmman ³ )andUcrUbxageRkam ³


429
9

x> KNnaRbEvg AC
D1  : y  x  3
2 eday ABC CaRtIekaNEkgRtg; A
tamRTwsþIbTBItaK½r AC  BC  AB
2 2 2

D2  : y  1 x  3
2 2
eday BC  5 cm , AB  3 cm
naM[ AC  5  3  16  4 cm
2 2

x> sresrsmIkarbnÞat; D  2
dUcenH KNna)an AC  4 cm .
smIkarbnÞat;EdlRtUvsresrKW D  : y  ax  b
K> KNna cos ABˆ C nig sin ABˆ C
2

-eday D  : y  ax  b kat;tam P5 , 1


kñúgRtIekaNEkg ABC manRbEvgRCug ³
2

eK)an 1  a  5  b  b  1  5a 1


BC  5 cm , AB  3 cm nig AC  4 cm
-eday D //D  enaH a  a
2 1

Et D  ³ y  2x  3 naM[ a  a  12 naM[ cos ABˆ C  BC


AB 3

5
1

-edayyk a  12 CMnYskñúg 1 ³ ehIy sin ABˆ C  BC


AC 4

5

eK)an b  1  5a  1  5  12  2 2 5   32 dUcenH cos ABˆ C 


3
5
nig sin ABˆ C  54 .
dUcenH sresr)anbnÞat; D  : y  12 x  32 .
2
X> bgðajfa MB MD  MA 2

eyIgeFVIkareRbóbeFob ABM nig DCM


-sg; D  kñúgtRmúyxagelI
2
edayRtIekaN ABM nig DCM man ³
taragtémøelx D  : y  12 x  32 ³ xy
2
1 3
1 0 -mMu ABˆ M  DCˆM ¬mMucarwkmanFñÚsáat;rYm AD ¦
V. tambRmab;RbFaneyIgsg;rUb)an ³ -mMu AMˆ B  DMˆ C ¬mMuTl;kBM Ul¦
A
// D
dUcenH ABM DCM tamlkçxNÐ m>m .

M // vi)ak ABMDCM

MB MA
MC MD

B C
O
Taj)an MB MD  MA MC
eday BM  CaemdüanénRtIekaN ABC
k> R)ab;RbePTRtIekaN ABC naM[ MA  MC
eday ABC man BC CaGgát;p©it nig A eK)an MB MD  MA MA
CacMNucenAelIrgVg; enaH ABC CaRtIekaN b¤ MB MD  MA 2

carwkknøHrgVg;
dUcenH eyIgbgðaj)an MB MD  MA . 2

dUcenH ABC CaRtIekaNEkgRtg; A .


430
9

431
9

sm½yRbLg ³ 09 sIha 1999


viBaØasa ³ KNitviTüa ¬elIkTI2¦ ry³eBl ³ 60 naTI BinÞú ³ 100


I. cUrKUssBaØa  kñúgRbGb;enAxagmuxcemøIyEdlRtwmRtUvmanEtmYyKt; ³
1> KNna 18  8 ³
k> □ 10 x> □ 5 2
K> □ 2 X> □ 5 g> □ 3 .
2> kñúgfg;mYymanb‘ícRkhm 2 edIm nigb‘ícexov 4 edIm. eKlUkcab;b‘íc 1 edImBIkñúgfg;edayécdnü.
rkRbÚ)abénRBwtþikarN_cab;)anb‘ícexovmYyedIm ³
k> □ 16 x> □ 12
K> □ 13 X> □ 23 g> □ 1 .
II. kMNt;témø x edIm,I[RbPaK F  x  51x  6 Kµann½y .
2

III. rksmIkarbnÞat;kñúgrUbxageRkam ³
y

2 B

A
x O 1 x


y

IV. lT§plénkarRbLgKNitviTüa)an[dwgfa ³ sisS 1 nak;)anBinÞú 100 sisS 4 nak;)anBinÞú 80


sisS 10nak;)anBinÞú 70 sisS 30 nak;)anBinÞú 50 nigsisS 13 nak;)anBinÞú 40 . cUrbMeBjtaragenH .
BinÞú eRbkg; eRbkg;ekIn
100
80
70 M
50
40

V. b£sSImYyedImQrRtg;EkgnwgépÞdI ehIyRtUvxül;vay)ak;Rtg;
cMNuc M dUcrUbxagsþaMenH. rkRbEvgTaMgGs;énedImb£sSI . A
60 o
S
3m

432
9

VI. ABC CaRtIekaNsm½gS . cMNuc D sßitenAelIbnÞat; AB . A Cap©itrgVg; .


C

60 o

D A G B

1> KNna GDˆ E rYcbgðajfa FBD CaRtIekaNEkgRtg; F .


2> eRbóbeFob DEG nig EFC .

8
cemøIy
I. KUssBaØa  kñúgRbGb;xagmuxcemøIyEdlRtwmRtUv³ III. rksmIkarbnÞat;kñúgrUbxageRkam ³
1> KNna 18  8 ³ K> ☑ 2 y

2 B
eRBaH 18  8  3 2  2 2  2 . 
A
2> rkRbU)abénRBwtþikarN_cab;)anb‘ícexov 1edIm ³ x O

1 x


X> ☑ 23 ³ eRBaH kñúgfg;manb‘ícRkhm 2 edIm y

nigb‘ícexov 4 edIm smIkarbnÞat;EdlRtUvrkmanTRmg; y  ax  b


enaHkrNIGac 2  4  6 nigkrNIRsb  4 -edaybnÞat;kat;G½kS yy Rtg; B0 , 2
naM[ P(exov)  krNI Rsb 4 2
  . naM[ b  y  2
krNIGac 6 3
-ehIypleFobénbERmbRmYl y nig x BIcMNuc
II. kMNt;témø x edIm,I[RbPaK F Kµann½y ³
A1, 0 eTAcMNuc B 0 , 2  CaemKuNR)ab;Ts
i én
eyIgman F  x  51x  6
2
bnÞat;kat;tam A nig B
edIm,I[RbPaK F Kµann½yluHRtaEt PaKEbgesµI 0 eK)an a  yx  xy  20  10  21  2
B A

eK)an x  5x  6  0
2 B A

x 2  2 x  3x  6  0
dUcenH rk)ansmIkarbnÞat; y  2x  2 .
xx  2  3x  2  0
x  2x  3  0
eKGaceRbIviFImü:ageTot
x  2  0 x  2 -edaybnÞat;kat;tam A1, 0 nig B0 , 2
naM[ x  3  0  x  3
  eK)anRbB½n§smIkar 02  aa 10bb  ba  22
 
dUcenH kMNt;témø)an x2, x3 . dUcenH smIkarbnÞat;rk)anKW y  2x  2 .
433
9

IV. tambRmab;RbFaneyIgGacbMeBjtarag)an ³ -bgðajfa FBD CaRtIekaNEkgRtg; F


BinÞú eRbkg; eRbkg;ekIn eday FBD man ³
100
80
1
4
1
5
-mMu BDF  GDE  30 TajBIxagelIo

70
50
10
30
15
45
-mMu DBF  ABC  60 ¬mMuRtIekaNsm½gS¦
o

40 13 58 EtplbUkmMukñgú én FBD esµI 180 KW ³ o

IV.rkRbEvgedImb£sSITaMgGs; ³ M
BDF  DBF  BFD  180o
BFD  180o  BDF  DBF 
RbEvgedImb¤sSITaMgGs;KW 
 180o  30o  60o 
AM  MS EdlRtUvrk ³  90 o

edaykñúg AMS EkgRtg; A 60 o


dUcenH FBD CaRtIekaNEkgRtg; F .
A S
man tan 60  AM
o 3m
AS 2> eRbóbeFob DEG nig EFC
naM[ AM  AS  tan 60  3  o
3 3 3 m
-eday DEG man DG CaGgát;p©it nig E enA
ehIy tamRTwsþIbTBItaK½r ³ elIrgVg; enaH DEG carwkknøHrgVg;
eK)an MS  AM  AS
2 2 2

naM[ DEG EkgRtg; E


MS  AM 2  AS 2
vi)ak DEG  90 nig DGE  60
o o

 3 3   3
2 2

 27  9  36  6 m
-eday DEG nig EFC man ³
enaH AM  MS  3 3  6  5.2  6  11 .2 m mMu DEG  EFC  90 o

¬eRBaH EFC CamMubEnßmCamMu BFD  90 ¦ o

dUcenH RbEvgedImb¤sSITaMgGs;KW 11.2 m . mMu DGE  ECF  60 o

V. eyIgmanrUb ³ C
¬eRBaHmMu ECF  60 CamMu  sm½gS ABC ¦
o

E
F
eXIjfa DEG nig EFC manmMuBIrb:unerogKña

60 o
dUcenH DEG EFC tamlkçxNÐ m>m .
D A G B

1> KNna GDˆ E


tamTMnak;TMngmMup©it nigmMucarwkEdlmanFñÚsáat;rYm
ˆ o
naM[ GDˆ E  EA2G  602  30 o

eRBaH GDˆ E nig EAˆ G manFñÚsáat;rYm EG


dUcenH KNna)anmMu GDˆ E  30 . o

434
9

435
9

sm½yRbLg ³ 24 kkáda 2000


viBaØasa ³ KNitviTüa ry³eBl ³ 120 naTI BinÞú ³ 100


I. cUrKUssBaØa  kñúgRbGb;enAxagmuxcemøIyEdlRtwmRtUvmanEtmYyKt; ³
rktémø m EdlnaM[bnÞat; y  m  2x  3 RsbnwgbnÞat; y  3x 1 .
k> □ m  5 x> □ m   73
K> □ m  1 X> □ m  53 .
II. tamrUbxagsþaM cUrbMeBjcenøaHxageRkam[)anRtwmRtUv ³ D
1
OA  ............ cm O C
OB  ............ cm 2 1
OC  ............ cm 1
I
OD  ............ cm A 1 B

eday AB  1 cm ; BI  1 cm ; IC  1 cm ; CD  1 cm nig OI  2 cm . ¬sresrcemøIyCab£skaer¦


III. RtIekaN ABC mYymanrgVas;RCug AB  x , AC  x  1 nig BC  x  2 Edl x  0 .
rktémø x edIm,I[RtIekaN ABC CaRtIekaNEkgRtg; A .
IV. kñúgtRmúyGrtUNrem xoy  eKmancMNuc A1,1 nig B5 ,  1 .
k> cUrkMNt;kGU redaenéncMNuckNþal M rbs; AB .
x> cUrsresrsmIkarbnÞat; AB .
V. kñúgfg;mYymanXøIelOg 6 nigXøIexov 5 . eKlUkcab;ykXøImþgbI . rkRbU)abénRBwtþikarN_ ³
k> cab;)anXøIexovmYyy:agtic .
x> cab;)anXøIelOgBIr nigXøIexovmYy.
VI. eK[RtIekaNEkgsm)at ABC Edlman Aˆ  90 , AB  AC  a nig AH Cakm<s;énRtIekaN .
o

k> KNna AH CaGnuKmn_én a .


x> O Cap©itrgVgc; arwkkñúgRtIekaN ABC . bgðajfa O sßitenAelI AH  . rYcKNna AO
CaGnuKmn_én R ¬kaMrgVg; O ¦ .
K> KNna R CaGnuKn_én a .

8
436
9

cemøIy
I. KUssBaØa kñúgRbGb;enAmuxcemøIyEdlRtwmRtUv ³ IV. k> kMNt;kUGredaenéncMNuckNþal M én AB 

rktémø m ³ K> ☑ m  1 eyIgmancMNuc A1,1 nig B5 ,  1


eRBaH y  m  2x  3 Rsbnwg y  3x 1 naM[ M  x 2 x , y 2 y  A B A B

kalNa a  a  m  2  3  m  1 .   1  5 1   1 
M , 
II. bMeBjcenøaHedayKNna OC , OB , OA, OD D  2 2 
M 2 , 0
tamRTwsþIbTBItaK½r 1
 OC  OI  IC
2 2
O
2 1
2C dUcenH kUGredaenéncMNuckNþal M 2 , 0 .
OC  OI 2  IC 2 I
 2  1  5 cm
2 2
A 1 B
1
x> sresrsmIkarbnÞat; AB ³
 OB  OI  IB
smIkarbnÞat;EdlRtUvrkmanrag AB : y  ax  b
2 2 2

OB  OI 2  IB 2
 2 2  12  3 cm
-eday y  ax  b kat;tam A1,1
 OA 2  OB 2  AB 2 eK)an 1  a   1  b   a  b  1 1
OA  OB 2  AB 2
2
-eday y  ax  b kat;tam B5 ,  1
 3  12  2 cm
 OD 2  OC 2  CD 2 eK)an 1  a  5  b  5a  b  1 2
OD  OC 2  CD 2 -edayyk 2  1 eK)an ³
2
 5  12  6 cm 5a  b  1

dUcenH eyIgbMeBj)an OA  2 cm  a  b  1 naM[ a
2

1
OB  3 cm 6a  2 6 3
OC  5 cm -yk a
1
3
CMnYskñúg 1 :
OD  6 cm
1 :  a  b  1 1
 b  a 1   1 
2
III.rktémø x edIm,I[ ABC EkgRtg; A ³ 3 3

eKman AB  x , AC  x  1 nig BC  x  2 dUcenH smIkarbnÞat;  AB : y   1 x  2 .


3 3
tamRTwsþIbTBItaK½r ³ B
x2
x  22  x 2  x  12 x V. kñúgfg;myY manXøIelOg 6 nigXøIexov 5
x2  4x  4  x2  x2  2x 1 A x 1 C naM[ cMnYnXøITaMgGs;KW 6  5  11
x2  2x  3  0
krNIBiess ³ a  b  c  1   2   3  0 k> cab;)anXøIexovmYyy:agtic
naM[ x  1 minykeRBaH x  0
1
RBwtþikarN_cab;)anXøIexovmYyy:agtic bMeBjCa
c
x2    
3
3
mYyRBwtþikarN_ cab;)anXøeI lOgTaMgGs;
a 1
eK)an ³ P(exovmYyy:agtic) = 1-P(lll)
dUcenH kMNt;)antémø x 3 .
437
9

naM[ P(exovmYyy:agtic) = 1   116  105  94 x> bgðajfa O sßitenAelI AH 


4 29
 1   0.88
eday AH  Cakm<s;énRtIekaNsm)at ABC
33 33
naM[ AH  k¾CaknøHbnÞat;BuHén ABC Edr
dUcenH P(exovmYyy:agtic)  29 33
 0.88 . -tamRTwsþI knøHbnÞat;BuHTaMgbIénRtIekaNRbsBVKña
)anmYycMNuc EdlCap©itrgVg;carwkkñúgRtIekaNenaH
x> cab;)anXøIelOgBIr nigXøIexovmYy
naM[ p©itrgVg;carwkkñgú ABCRtUvsßitenAelI AH 
RBwtþikarN_ cab;)anXøIelOgBIr nigXøIexovmYyKW
¬l2.x1¦ GacCa ³ ¬llx¦ b¤ ¬lxl¦ b¤ ¬xll¦ dUcenH bgðaj)anfa O sßitenAelI AH  .
naM[RbU)abcab;)anXøIelOgBIr nigXøIexovmYyKW ³ - rYcKNna AO CaGnuKmn_én R ¬kaMrgVg; O ¦
P¬l2.x1¦ = P¬llx¦ + P ¬lxl¦ + P ¬xll¦ tag P nig Q CacMNucb:Hrvag ABC nigrgVg;
6 5 5 6 5 5
(   )(   )(   )
11 10 9 11 10 9
5 6 5
11 10 9
naM[ APOQ Cakaer BIeRBaHvaman ³
 5  5  5
    OP  OQ  R nig PAˆ Q  APˆ O  AQˆ O  90 o

 33   33   33 
5 5 naM[ OA CaGgát;RTUgkaerEdlmanRCug R
 3    0.45
33 11 tamRTwsþI CaGgát;RTUgkaer OA  R  2
dUcenH P¬l2.x1¦  115  0.45 . dUcenH KNna)an OA  R 2 .
VI. tambRmab;RbFaneyIgsg;rUb)an ³ K> KNna R CaGnuKn_én a
A
Q
eyIgman OA  OH  AH
P
a
O
 R eday OA  R 2 / OH  R nig AH 
a 2
2
B
H
C
eK)an R 2R
a 2

k> KNna AH CaGnuKmn_én a 2


R 2 1  
a 2
eday ABC CaRtIekaNEkgsm)atmankm<s; AH 2

naM[ mMu)at ABH  ACH  45 o


R
a 2

2 2 1 
-kñúgRtIekaNEkg ABH EkgRtg; H R
a 2 2 1 
¬eRBaH H CacMeNalEkgén A elI BC ¦ 
2 2 1 2 1  
eK)an sin ABˆH  AH  AH  AB  sin ABˆ H R
2  2 a  1  2
a
AB 22  1 
 2 

naM[ AH  a  sin 45 o  a 
2 a 2
  2
2 2 dUcenH KNna)an R  1 
2
a
 .
 
dUcenH KNna)an AH 
a 2
2
.
438
9

439
9

sm½yRbLg ³ 13 sIha 2001


viBaØasa ³ KNitviTüa ry³eBl ³ 120 naTI BinÞú ³ 100

I. cUrKUssBaØa  kñúgRbGb;enAxagmuxcemøIyEdlRtwmRtUvmanEtmYyKt; ³
1> cUrkMNt;témø m EdlnaM[bnÞat; y  2  mx RsbnwgbnÞat; y  3x  1 .
k> □ m  3 x> □ m  1
K> □ m  1 X> □ m  73 .
2> tamrUbenH eKman BH  2 , HC  8 , AH  x . cUrkMNt;témø x ³ A
k> □ x  10 x> □ x  4 x

K> □ x  4 X> □ x  16 B
2 H 8
C

KNna A   2 2 5 5  88 2020  .


2 2

II.

1
III. edaHRsayRbB½n§smIkartamedETmINg; ³ 2x x2 yy  89 2
.

IV. kñúgfg;mYymanXøIcMnYn 12 RKab; EdlkñúgenaHmanEtXøIBN’s nigXøIBN’exµA . rkcMnYnXøIBN’exµA


edIm,I[)anRbU)abénXøIBN’s esµInwg 32 .
V. b£sSI 9 edImmanRbEvg ¬KitCa m ¦ 4 , 12 , 10 , 8 , 2 , 4 , x , 8 , 4 .
k> cUrkMNt;témø x edIm,I[ x CamFüménRbEvgb£sSITaMgenaH .
x> Tajrkemdüan énRbEvgb£sSITaMgenaH .
VI. eK[bIcMNuc A 1 , 1 ; B3 , 2 nig C0 , 4 .
k> edAcMNuc A ; B nig C enAkñúgtRmúyGrtUNremEtmYy .
x> sresrsmIkarbnÞat; AB .
K> KUsemdüan CM  énRtIekaN ABC . rkkUGredaenéncMNuc M .
X> KUskm<s; CH  énRtIekaN ABC . rksmIkarbnÞat; CH  .

440
9

VII. rgVg;mYymanp©it O kaMRbEvg 4 cm nigmanGgátp; ©it AB . H CacMNuckNþalén OB  . bnÞat;


mYyEkgnwg AB Rtg; H ehIyCYbrgVg; O Rtg;cMNuc M nig N . bnÞat;  AM  nig NB
CYbKñaRtg;cMNuc I .
k> cUrKNnargVas; MA nig MB .
x> cUreRbóbeFob ABI nig NMI .

8
cemøIy
I. KUssBaØa  kñúgRbGb;enAmuxcemøIyEdlRtwmRtUv³ III. edaHRsayRbB½n§smIkartamedETmINg; ³
1> kMNt;témø m ³ K> ☑ m  1 . 1
eRBaH bnÞat; y  2  mx Rsbnwg y  3x  1 eyIgman 2x x2 yy  89 2
tamedETmINg; ³

D  1  4  5
kalNa a  a  2  m  3  m  1 . Dx  25
Dx  9  16  25  x  5
D 5
2> kMNt;témø x ³ x> ☑ x  4 . D y  10
D y  8  18  10  y  2
eRBaH x  2  8  x  16  4
2
D 5

EtRbEvgRCugminGacGviC¢man A
dUcenH RbB½n§smIkarmanKUcemøIy x  5, y  2 .
KW x  0 dUcenH x  4 x

B C IV. rkcMnYnXøIBN’exµA ³
II. KNna A ³ 2 H 8
tag x CacMnnY XøIBN’exµA naM[
A
 2  5    8  20 
2 2

cMnYnXøIBN’sKW 12  x eRBaHXøITaMgGs;12 RKab;


 2  5  8  20 
2  2 10  5  8  2 160  20 ebItag P(XøIBN’s) CaRbU)abcab;)anXøIBN’s


2  5 2 2 2 5   eK)an P(XøBI N’s) = 1212 x
35  2 10  2 16 10


2 2 5 2 5   tammRmab;RbFaneK)an ³ P(XøIBN’s)  32

35  2 10  8 10
naM[ 1212 x  32

2 2 5 2 5  
36  3 x  24
35  6 10
 3 x  36  24
22  5
3 x  12
35  6 10 35
  10  x4
6 6

dUcenH KNna)an A  10 
35
. dUcenH cMnYnXøIexµAKW 4 RKab; .
6

441
9

V. k> cUrkMNt;témø x edIm,I[ x CamFüm eK)an 1  a   1  b   a  b  1 1


eyIgmanTinñn½y 4 , 12 , 10 , 8 , 2 , 4 , x , 8 , 4 -eday  AB  : y  ax  b kat;tam B3 , 2
eK)an x  4  12  10  8 92  4  x  8  4 eK)an 2  a  3  b  3a  b  2 2
enaH x  529 x eRBaH x  x ¬ x CamFüm¦ yk 2  1 enaHeK)an ³
3a  b  2

9 x  52  x  8x  52  x  6.5  a  b  1 naM[ a
1
4a  1 4
dUcenH témøkMNt;)anKW x  6.5 m . tam 1 :  a  b  1  b  1  a  1  14  54
x> Tajrkemdüan énRbEvgb£sSITaMgenaH
cMeBaH x  6.5 m nigeyIgerobTinñn½ytamlMdab; dUcenH smIkarbnÞat;  AB : y  1 x  5
4 4
.
eyIg)an 2 , 4 , 4 , 4 , 6.5 , 8 , 8 , 10 , 12 K> KUsemdüan CM  énRtIekaN ABC
mancMnYntYénTinñn½yKW n  9 ¬CacMnYness¦ ¬ emIlrUb cMeBaHkarKUs emdüan CM ¦
naM[ Me CatYTI 9 2 1  5 -rkkUGredaenéncMNuc M
tamTinñn½yeRkayBIerobtamlMdab; tYTI 5 RtUvnwg eday M CacMNuckNþalénRCug AB eK)an ³
cMnYn 6.5 enaHnaM[ Me  6.5  x  xB y A  yB 
M A , 
 2 2 
dUcenH Taj)anemdüan Me  6.5 .  1 3 1 2 
M , 
 2 2 

k> edAcMNuc A ; B nig C enAkñúgtRmúyEtmYy  3


VI. M 1 , 
 2
eyIgman A 1 , 1 ; B3 , 2 nig C0 , 4  3
dUcenH kUGredaenrk)anKW M 1 , 
 2
.
C 0 , 4 X> KUskm<s; énRtIekaN ABC
¬ emIlrUb cMeBaHkarKUs km<s; CH ¦
B3 , 2
rksmIkarbnÞat; CH  : y  ax  b
A 1 , 1 HM
eday CH   AB   a  a  1
naM[ 14  a  1  a  4
x> sresrsmIkarbnÞat; AB ehIy CH  kat;tam C0 , 4
bnÞat; AB manrag y  ax  b enaHeK)an ³ 4   4 0  b  b  4
-eday  AB  : y  ax  b kat;tam A 1 , 1 dUcenH smIkarbnÞat; CH  : y  4x  4 .
442
9

VII. k> KNnargVas; MA nig MB


I
M

A  H
4 cm O
// // B

-RtIekaN AMB man M enAelIrgVg; nig AB


CaGgát;p©it enaHvaCaRtIekaNcarwkkñúgrgVg;
dUcenH RtIekaN AMB CaRtIekaNEkgRtg; M .
-eday AB  MN  Rtg; H naM[ MH Ca
km<s;énRtIekaN AMB
-tamTMnak;TMngkñúgRtIekaNEkg AMB
MA2  AH  AB  MA  AH  AB
MA  6  8  4 3 cm
MB2  BH  AB  MB  BH  AB
MB  2  8  4 cm

dUcenH RbEvg MA  4 3 cm , MB  4 cm .
x> eRbóbeFob ABI nig NMI
edayRtIekaN ABI nig NMI man ³
-mMu ABI  NMI ¬mMucarwkmanFñÚsáat;rYm MB ¦
-mMu AIB  NIM ¬CamMurYmEtmYy¦
dUcenH ABI NMI lkçxNÐdMNUc m>m .

443
9

sm½yRbLg ³ 19 sIha 2002


viBaØasa ³ KNitviTüa ry³eBl ³ 120 naTI BinÞú ³ 100


I. cUrKUssBaØa  kñúgRbGb;enAxagmuxcemøIyEdlRtwmRtUvmanEtmYyKt; ³
1> etI 2 Cab£srbs;smIkarNamYy ?
k> □ 2x  x  2  0
2
x> □ x  3x  3 2  2  0
2

K> □ x  2x  3  0
2
X> □  x  x  2  1  0 .
2

2> cMNuc A1 , 1 sßitenAkñúgtMbn;cemøIyénvismIkarNamYy ?


k> □ y  x 1 x> □ x  y  5
K> □ x  y  2 X> □ y  x  3 .
II. cUrpÁÚpÁgrvagpleFobRtIekaNmaRténmMu  nigtémørbs;va ³
pleFobRtIekaNmaRténmMu  témøpleFobRtIekaNmaRt cemøIy
1> cot g k> 0.8 1>  g
C
2> cos 10
x> 1.25 2> 
3> tan 6
 K> 0.6 3> 
4> sin A 8 B
X> 0.75 4> 
g> 34
3 18  2 27  45
III. cUrsRmYlkenSam ³ F
3 8  2 12  20
.
IV. hwbmYydak;Gavburs nigGav®sþI. GavbursmancMnYn 8 BN’Rkhm nig 2 eTotBN’s. Gav®sþIman
cMnYn 6 BN’Rkhm nig 4 eTotmanBN’s . eKhUtykGavmYyBIkñúghwb .
k> kMNt;RbU)abEdleKhUt)anGav®sþI .
x> kMNt;RbU)abEdleKhUt)anGavBN’Rkhm .
V. rkBIcMnYn x nig y Edl x  y  9 nig xy  14 .
VI. eK[bnÞat; D : y  x  1 enAkñúgtRmúyGrtUNrem xoy  .

444
9

1> sg;bnÞat; D  .
2> rksmIkarbnÞat; L Edlkat;tamcMNuc A0 , 3 ehIyEkgnwgbnÞat; D  .
sg;bnÞat; L enAkñúgtRmúyCamYy D  .
3> KNnakUGredaenéncMNucRbsBV M rvagbnÞat; L nig D  ehIyepÞógpÞat;lT§pltamRkaPic .
VII. M CacMNucmYyenAelIrgVg; C  Edlmanp©it O nigkaM R . rgVg; C  Edlmanp©it M nigkaM r

Edl R  r  CYbrgVg; C  Rtg; A nig B . . bnÞat; OM  CYbrgVg; C  Rtg; D ehIyCYbrgVg;


C  Rtg; E nig F Edl E  DM  . bnÞat;   Ekgnwg DM  Rtg; F ehIyCYb DA nig
DB  erogKñaRtg; I nig J .
1>KNnargVas;mMu DAˆ M nig DBˆM . TajbBa¢ak;fa DI  nig DJ  b:HnwgrgVg; C  Rtg; A nig B .
2>bgðajfactuekaN AMFI carwkkñúgrgVg;mYy .
3>eRbobeFob ADM nig FDI .
4>TajbBa¢ak;fa FDAD DM AM

DI

FI
ehIy AD  DI  2R2R  r  .

8
cemøIy
I. KUssBaØa  kñúgRbGb;enAmuxcemøIyEdlRtwmRtUv³ III. sRmYlkenSam ³ F

1> etI 2 Cab£srbs;smIkarNamYy ? F


3 18  2 27  45
3 8  2 12  20
x> ☑ x  3x  3 2  2  0
2
9 2 6 3 3 5

eRBaH 2  3 2  3 2  2  0  0  0 Bit
2
6 2 4 32 5

2> cMNuc A1 , 1 enAkñúgtMbn;cemøIyénvismIkar ³ 



33 2 2 3 5  3
23 2 2 3 5 2
K> ☑ x  y  2
eRBaH A1 , 1 enaH 1 1  2  0  2 Bit . dUcenH sRmÜlkenSam)an F
3
2
.
II. pÁÚpÁgrvagpleFobRtIekaNmaRténmMu  nigtémø³ IV. Gavbursman 8 BN’Rkhm nig 2 BN’s
sin  0.6
sin  
6
10
 0.6 tan   
cos 0.8
 0.75 Gav®sþIman 6 BN’Rkhm nig 4 BN’s
8
cos   0.8 cot 
cos 0.8
  1.33 naM[cMnYnGavTaMgGs; 8  2  6  4  20
10 sin  0.6
dUcenH 2>  k k> kMNt;RbU)abEdleKhUt)anGav®sþI
C
6  4 10
P(hUt)anGavRsþI)    0.5
3>  X 6
10
20 20

4>  K A 8 B dUcenH P( hUt)anGavRsþI)  0.5 .
445
9

x> kMNt;RbU)abEdleKhUt)anGavBN’Rkhm 2> rksmIkarbnÞat; L


8  6 14 7
P(hUt)anGavRkhm) 
20
 
20 10
 0.7 smIkarbnÞat;EdlRtUvrkmanrag L : y  ax  b
-eday L : y  ax  b kat;tamcMNuc A0 , 3
dUcenH P(hUt)anGavRkhm)  0.7 .
eK)an 3  a  0  b  b  3
V. rkBIrcMnYn x nig y -ehIy L : y  ax  b EkgnwgbnÞat; D 
eyIgman x  y  9 1 nig xy  14 2 eK)an a  a  1  a  a1
tam 1 : x  y  9  x  y  9 3 Et D : y  x  1 manemKuNR)ab;Tis a  1
yk 3 CMnYskñúg 2 eK)an ³
2 :  y  9y  14 naM[ a  11  1
y 2  9 y  14
y 2  9 y  14  0
dUcenH smIkarbnÞat; L : y   x  3 .
man   9  4 114  81 56  25
2
- sg;bnÞat; L enAkñúgtRmúyCamYy D 
naM[ y    921 25  9 2 5  42  2
1
edayeRbItaragtémøelx edIm,Isg;bnÞat; L
x 1 0
  9  25 9  5 14 L  : y   x  3 ³
y2    7 y 2 3
2 1 2 2
cMeBaH y  2 ³ 3 : x  y  9  2  9  7 enAkñúgtRmúyCamYy D  .
1

cMeBaH y  7 ³ 3 : x  y  9  7  9  2 3> KNnakUGredaenéncMNucRbsBV M


2
eyIgman D : y  x  1 nig L : y   x  3
dUcenH rk)ancemøIyBIrKU x  7 , y  2 edaypÞwmsmIkarGab;sIusrvagbnÞat; D  nig L
x  2 , y  7 . eK)an ³ x 1  x  3
VI.1> sg;bnÞat; D  2x  4  x  2 /
eyIgman D : y  x  1 x 1 2 naM[ y  x  1  2  1  1
y 0 1
eyIgsg;RkaPic)an ³ dUcenH kUGredaenéncMNucRbsBV M 2 , 1 .
- epÞógpÞat;lT§pltamRkaPic
D  : y  x  1 tamRkaPiceyIgeXIjfa ebIeyIgeFVIcMeNalEkg
BIcMNucRbsBV eTAelIG½kSTaMgBIrenaHeyIg)an
kUGredaenéncMNucRbsBVKW M x  2 , y  1
L  : y   x  3 dUcCamYykarKNnaxagelIBitEmn .
dUcenH karKNnaepÞógpÞat;edayRkaPic .
446
9

VII. tambRmab;RbFaneyIgsg;rbU )an ³ naM[ctuekaN AMFI manplbUkmMuQm ³


  J MFˆI  MAˆ I  90 o  90 o  180 o /
F dUcenH ctuekaN AMFI carwkkñúgrgVg;mYy .
M B
I 
3> eRbóbeFob ADM nig FDI
C  E
A O
 eday ADM nig FDI man ³
-mMu ADM  FDI ¬CamMurYm¦
C  D -mMu DAM  DFI  90 ¬mMuEkgdUcKña¦
o

1> KNnargVas;mMu DAˆ M nig DBˆM dUcenH ADM FDI tamlkçxNÐ m>m .
man A CacMNucenAelIrgVg; C  ehIy MD Ca 4> TajbBa¢ak;fa FD
AD DM AM
 
Ggát;p©iténrgVg;p©it C  naM[mMu DAˆ M CamMucarwk DI FI
tamry³sRmayxagelI
knøHrgVg; ADM AD DM AM
  
FDI
dUcenH mMu DAˆ M  90 . o FD DI FI

cMeBaHAD DM

vi)ak RtIekaN DAM CaRtIekaNEkgRtg; A FD DI

A  DI
enaHTaj)an AD DI  DM  FD
eday A  C   DI  b:HnwgrgVg; C  Rtg; A eday DM  2R nig FD  2R  r
DI  AM

dUcKñaEdr B enAelIrgVg; C  ehIy MD Ca dUcenH AD  DI  2 R  2 R  r  .


Ggát;p©iténrgVg;p©it C  naM[mMu DBˆM CamMucarwk
knøHrgVg;
dUcenH mMu DBˆ M  90 . o

vi)ak RtIekaN DBM CaRtIekaNEkgRtg; B


B  DJ
eday B  C   DJ  b:HnwgrgVg; C  Rtg; B
DJ  BM

2> bgðajfactuekaN AMFI carwkkñúgrgVg;mYy³


edayctuekaN AMFI man ³
-mMu MFˆI  90 eRBaH  Ekgnwg DM  Rtg; F
o

-mMu MAˆ I  90 eRBaH DI  b:HnwgrgVg; C  Rtg; A


o

447
9

sm½yRbLg ³ 09 kBaØa 2003


viBaØasa ³ KNitviTüa ry³eBl ³ 120 naTI BinÞú ³ 100

I. cUrKUssBaØa  kñúgRbGb;enAxagmuxcemøIyEdlRtwmRtUv ³
1> etIsmIkarNamYymanb£sBIrepSgKña ?
k> □ x  2x  3  0
2
x> □ x  4x  4  0
2

K> □ 2x  5x  3  0
2
X> □ x  12  x   0 .
2

2> EFG CaRtIekaNEkgRtg; E ehIyman FG  6 cm nig EFˆG  60 . KNnaRbEvg EF KW ³


o

k> □ EF  6 3 cm x> □ EF  6 33 cm
K> □ EF  3 3 cm X> □ EF  3 cm .
II. emGMe)AmYyehIrcuHTMelIpáamYyTgenAkñgú sYnc,armYy EdlmanpáaBN’s 1Tg páaBN’Rkhm 2 Tg
páaBN’sVay 3 Tg nigpáaBN’elOg 4 Tg . cUrpÁÚpÁgrvagRBwtþikarN_ nigRbU)abrbs;vaEdlRtUvKña ³
RBwtþikarN_ RbU)abénRBwtþikarN_ cemøIy
1> {emGMe)AcuHTMelIpáaBN’s } k> 0.4 1>  x
2> {emGMe)AcuHTMelIpáaBN’Rkhm } x> 0.1 2> 
3> {emGMe)AcuHTMelIpáaBN’sVay } K> 0.5 3> 
4> {emGMe)AcuHTMelIpáaBN’elOg } X> 0.2 4> 
g> 0.3
III. kMNt;témø m edIm,I[bnÞat; L  EdlmansmIkar y  m  2m  4x RsbnwgbnÞat; L  Edlman
1
2
2

smIkar y  m  2x  1.
IV. KNna A  2 98  2 54  200  3 16  32
3 3

 2  5  1  5 
B    
1  2 5  1  2 5 
.
  
V. taragxageRkamenHbgðajBIcMnYnkumar enAkñúgPUmiEdlekItCm¶WxVak;man;tamfñak;Gayu ³
fñak;énGayu ¬qñaM¦ 0-2 2-4 4-6 6-8
cMnYnkumar ¬nak;¦ 4 3 6 7

1> sg;tarageRbkg;ekIn énTinñn½yxagelI .


2> KNnam:Ut nigfñak;énGayuEdlCaemdüan énTinnñ ½yxagelI .
448
9

VI. ekan K  manmaD V  32cm nigmankm<s; h . ekan K  manmaD V  4cm nigmankm<s;


1 1
3
1 2 2
3

h  5cm . ekan K  nigekan K  CaekandUcKña . ¬ebkçCnmin)ac;KUsrUbeT¦ . KNnakm<s; h


2 1 2 1

énekan K  . 1

VII. enAkñúgtRmúyGrtUNrem xoy  mYy eKmancMNuc A1 , 2 ; B0 ,  2 nig C3 , 0 .


1> kMNt;smIkarbnÞat; BC  .
2> kMNt;kUGredaenrbs;cMNuckNþal I én AC . kMNt;smIkarbnÞat; BI  .
 2
y  x2
3> edaHRsayRbB½n§vismIkartamRkabPic 
3
3
.
y  x2
 2
VIII. eKman ABC CaRtIekaNEkgRtg;kMBUl A ehIyman AB  3cm , AC  4cm . rgVg; C  man 1

Ggát;p©it AB ehIyCYbRCug BC  Rtg; H .


1>bgðajfa AH  Cakm<s;énRtIekaN ABC . KNna BC nig AH .
2>rgVg; C  mYyeTotcarwkeRkARtIekaN AHC . M CacMNucmYyenAelIFñÚtUc CH . bnÞat;
2

CM  CYbbnÞat;  AH  Rtg; N . eRbobeFob MAN nig HCN . TajbBa¢ak;fa


AM  HN  MN  CH .

8
cemøIy
I. KUssBaØa  kñúgRbGb;enAmuxcemøIyEdlRtwmRtUv³ II. pÁÚpÁgrvagRBwtþikarN_ nigRbU)abrbs;vaEdlRtUvKña³
1> smIkarEdlmanb£sBIrepSgKñaKW ³ páaBN’s 1Tg BN’Rkhm 2 Tg BN’sVay 3 Tg
K> ☑ 2x  5x  3  0
2
nigBN’elOg 4 Tg enaHsrub 1 2  3  4  10
eRBaH tamkrNIBiess a  b  c  2  5  3  0 -RbU)abTMelIpáaBN’ s P(s)  101  0.1
eK)anb£s x  1 nig x   ac   52 .
1 2 -RbU)abTMelIpáaBN’ Rkhm P(Rkhm)  102  0.2
2> KNnaRbEvg EF KW ³ F -RbU)abTMelIpáaBN’ sVay P(sVay)  103  0.3
X> ☑ EF  3 cm eRBaH
o
60 6 cm

-RbU)abTMelIpáaBN’ elOg P(elOg)  104  0.4


tampleFobRtIekaNmaRt E G

cos EFˆG 
EF
 EF  EF  cos EFˆG
dUcenH eyIgpÁÚpÁg)an 2>  X
EF
 6  cos60o
3>  g
1
 6   3 cm
4>  k .
2
449
9

III.kMNt;témø m edIm,I[bnÞat; L //L  1 2 V. 1> sg;tarageRbkg;ekIn énTinñn½y


eyIgmanbnÞat;BIrKW L : y  m  2m  4x
1
2
Gayu ¬qñaM¦ cMnYnkumar f eRbkg;ekIn f
0-2 4 4
L2  : y  m  2x  1 2-4 3 7
bnÞat;BIrRsbKñaluHRtaEtemKuNR)ab;TisesµIKña 4-6
6-8
6
7
13
20
eK)an ³ m  2m  4  m  2
2
srub 20
m2  m  6  0 / 2> KNnam:Ut nigfñak;énGayuEdlCaemdüan
man   1  4 1  6  1  24  25
2
edaym:UtKWCa p©itfñak;énGayuEdlmaneRbkg;eRcIn
enaH m   1 2125   12 5  26  3
1 CageK ehIyfñak;Gayu 6-8 maneRbkg; 7 eRcInelIs
m2 
 1  25  1  5 4
  2 eK mann½yfam:UtKWCa p©itfñak;Gayu 6-8
2 1
eK)an Mo  6 2 8  7
2 2

dUcenH témøkMNt;)an m1  3 , m2  2 .
dUcenH m:UtKNna)anKW Mo  7 .
IV. KNna fñak;GayuEdlCaemdüan Cafñak;GayuEdlman
A  2 98  23 54  200  33 16  32
 2 2  49  23 27  2  2 100  33 8  2  16  2 eRbkg;témøkNþalénTinnñ ½y
 14 2  63 2  10 2  63  2  4 2 naM[ Me CatémøéntYTI 202  10
 14 2  63 2  10 2  63  2  4 2
0
tamtarageRbkg;ekIntYTI 10 enAkñúgfñak;Gayu 4-6
dUcenH KNna)anKW A  0 . dUcenH fñak;EdlCaemdüanKWfñak;Gayu 4-6 .
 2  5  1  5  VI. KNnakm<s; h énekan K  1 1
B    
 1  2 5  1  2 5 
  bRmab; ekan K  nigekan K  CaekandUcKña
1 2


2  5 1  5  naM[ pleFobFatuRtUvKñaesµIKña
1  2 5 1  2 5  3


22 5  5 5 eK)an  hh   VV  h  h  VV
1 1
1 2 3
1

 2  2 2
1  20
3 5 eday V1  32cm 3 / V2  4cm 3 nig h 2  5cm

 19
3 5
naM[ h1  5  3
32
4

19  5  3 8  5  2  10 cm /
3 5
dUcenH KNna)an B
19
. dUcenH RbEvgkm<sK; Nna)anKW h  10 cm 1 .

450
9

VII.eyIgmancMNuc A1 , 2 / B0 ,  2 / C3 , 0 3> edaHRsayRbB½n§vismIkartamRkabPic


1> kMNt;smIkarbnÞat; BC  
y 
2
x2
bnÞat;RtUvrkmanrag BC  : y  ax  b eyIgmanRbB½n§vismIkar 
3
3
y  x2
 2
-eday BC  : y  ax  b kat;tam B0 ,  2  2
y  3 x  2
naM[  2  a  0  b  b  2 1 eyIgsg;bnÞat;RBMEdnénsmIkar  3
y  x  2
-eday BC  : y  ax  b kat;tam C3 , 0  2

naM[ 0  a  3  b  a   b3 2 edayeRbItaragtémøelxedIm,Isg;bnÞat;RBMEdn


-yk 1 CMnYskñúg 2 eK)an ³
2
y  x2
3
ni g 3
y  x2
2
b   2 2 x 3 3 x 2 2
2 : a  a 
y 4 0 y 5 1
3 3 3

dUcenH smIkarbnÞat; BC : y  2 x  2 . eyIgsg;RkaPic)an ³ 3


3 y x2
2

2> kMNt;kUGredaencMNuckNþal I én AC  y


2
3
x2

kUGredaenéncMNuckNþalGgát; AC kMNt;eday
 x  xc y A  y c 
I A , 
 2 2 
1 3 2  0 
I , 
 2 2 
I 2 , 1

dUcenH kUGredaencMNuckNþal I 2 , 1 . EpñkminqUtCacemøIyénRbB½n§vismIkar .


VIII. tambRmab;RbFaneyIgsg;rb U )an ³
-kMNt;smIkarbnÞat; BI 
bnÞat;RtUvrkmanrag BI  : y  cx  d C
M
-eday BI  : y  cx  d kat;tam B0 ,  2 N
C 2  4cm
naM[  2  c  0  d  d  2 H

-eday BI  : y  cx  d kat;tam I 2 , 1 3cm


A B
naM[ 1  c  2  d  c  1 2d 2 C1 
-yk 1 CMnYskñúg 2 eK)an ³
2 : c  1  d 
1   2 3

¬edIm,IkMu[mankarlM)akkñgú kareFVIdMeNaHRsay
2 2 2
edaysarrUb nigdMeNaHRsayenATMB½repSg dUcenH
dUcenH smIkarbnÞat; BI  : y  3 x  2
2
. ´sUmykrUbeTATMB½rfµIedIm,IgayRsYl ¦
451
9

C 2> eRbobeFob MAN nig HCN


M mMu AMˆ C  AHˆ C  90 ¬mMumanFñsÚ áat;rYm AC ¦
o

C 2  4cm
H
N naM[ AMˆ N  90 ¬CamMubEnßmCamYymMu AMˆ C ¦
o

ehIy CHˆN  90 ¬CamMubEnßmCamYymMu AHˆ C ¦


o

A 3cm B eday MAN nig HCN man ³


C1  -mMu AMN  CHN  90 ¬mMuEkgdUcKña¦
o

1> bgðajfa AH  Cakm<s;énRtIekaN ABC -mMu MAˆ N  HCˆN ¬mMucarwkmanFñÚsáat;rYm MH ¦


kñúg ABH man AB CaGgát;p©itrgVg; C  nig 1 dUcenH MAN HCN tamlkçxNÐ m>m .
H sßitenAelIrgVg; C  enaH ABH CaRtIekaN
MAN
1

EkgRtg; H KW AH  BH vi)ak ³ HCN



AM MN

CH HN
ehIykñúgRtIekaN ABC man AH  KUsecjBI naM[ AM  HN  MN  CH

kMBUl A kat; BC Rtg; H dUcenH Taj)an AM  HN  MN  CH .


dUcenH AH  Cakm<s;énRtIekaN ABC .
vi)ak ³ AHC  90 . o

- KNna BC nig AH
kñúgRtIekaNEkg ABC EkgRtg;kMBUl A
ehIyman AB  3cm , AC  4cm tamBItaK½r
eK)an ³ BC  AB  AC
2 2 2

BC  32  4 2
BC  5 cm
-tamTMnak;TMngkñúgRtIekaNEkg ABC
eK)an ³ BC AH  AB AC
AB  AC
naM[ AH 
BC
Edlman AB  3cm , AC  4cm , BC  5 cm
eK)an AH  3 5 4  125  2.4 cm
dUcenH eyIgKNna)anRbEvg
BC  5 cm , AH  2.4 cm .
452
9

453
9

sm½yRbLg ³ 10 kkáda 2004


viBaØasa ³ KNitviTüa ry³eBl ³ 120 naTI BinÞú ³ 100

I. cUrKUssBaØa  kñúgRbGb;enAxagmuxcemøIyEdlRtwmRtUvmanEtmYyKt; ³
1> etIsmPaBNamYyEdlRtwmRtUv ³
k> □ 81  3
4
x> □ 16  4
3

K> □ 1.69  1.3 X> □  5  5 .


2

2> EFGH CactuekaNEkgmYyEdlman EF  1.5 cm nig EG  3 cm . KNna GEˆH ³


k> □ GEˆH  60 o
x> □ GEˆH  35 o

K> □ GEˆH  45 o
X> □ GEˆH  30 . o

II. x CamFümTinn ñ ½y. Mo Cam:UtTinñn½y. Me CaemdüanénTinñn½y. cUrpÁÚpÁgEpñk A nig B EdlRtUvKña ³


A ¬Tinñn½y¦ B ¬mFümsßiti¦ cemøIy
1> 2 , 3 , 4 , 1 , 6 k> Me  4 1>  X
2> 2 , 4 , 5 , 6 , 1 x> Mo  1 2> 
3> 4 , 6 , 3 , 7 , 5 K> x  3 3> 
4> 1 , 2 , 5 , 4 , 1 X> Me  3 4> 
g> x  5
III. enAkñúgfñak;eronmYyeKe)aHeqñateRCIserIssisS 3 nak; kñúgcMeNamsisSQreQµaH 7 nak; EdlkñúgenaH
mansisSRbus 5 nak; nigsisSRsI 2 nak;. rkRbU)abEdleKeRCIserIs)ansisSRsI 1 nak;y:agtic.
IV. eKykesovePA 200 k,aleTAEck[sisSkñúgfñak;myY . ebIeKEck 4 k,aldUcKña enaHenAsl;esovePA
20 k,al. ebIEck[sisSRbusmñak; 4 k,al nigsisSRsImñak; 5 k,al enaHxVHesovePAcMnYn 5 k,al.

rkcMnYnsisSenAkñúgfñak;eronenaH .
V. 1> edaHRsayRbB½n§vismIkartamRkabPic  yy  2x  2 .

2> ABC manRCug AB  x 1 , BC  x  4 nig AC  x  2 Edl x  1 .
kMNt;témøén x edIm,I[ ABC EkgRtg;kMBUl A .
VI. 1> enAkñúgtRmúyGrtUNrem xoy  sg;bnÞat; D : y   2x  2 .
2> bnÞat; L kat;tamcMNuc A 1 , 0 ehIyEkgnwg D  .
454
9

k> rkemKuNR)ab;TisénbnÞat; L .


x> sresrsmIkarbnÞat; L .
K> sg;bnÞat; L enAkñúgtRmúyCamYy D  .
VII. kñúgrgVg;p©it O mYymanGgátF; ñÚBIrminb:unKña AB nig CD EdlEkgKñaRtg;cMNuc I . M CacMNuc
kNþalén BD . bnÞat; MI  CYbGgát; AC  Rtg; N .
1> bgðajfa MBI nig MDI CaRtIekaNsm)at .
2> eRbóbeFob IBD nig NCI . TajbBa¢ak;fa MN    AC  .
3> eKyk IA  a nig IC  b . KNna AC nig IN [Cab;Tak;TgeTAnwg a nig b .

8
cemøIy
I. KUssBaØa  kñúgRbGb;muxcemøIyEdlRtwmRtUv ³ Et P(bbb) = P(b)  P(b¼b)  P(b¼bb)
1> etIsmPaBNamYyEdlRtwmRtUv ³ 
5 4 3 2
  
7 6 5 7
K> ☑ 1.69  1.3 eRBaH 1.69  1.3  1.3
2
eK)an P(s1y:agtic) = 1  72  75  0.71
2> KNna GEˆH ³ X> ☑ GEˆH  30 o

eRBaH sin GEˆH  13.5  0.5  GEˆH  30 o


dUcenH s y:agtic) = 75  0.71 .
P( 1
E H
1.5cm 3cm IV. rkcMnYnsisSenAkñúgfñak;eronenaH
F G tag x CacMnYnsisSRbus ¬KitCa nak;¦
II. pÁÚpÁgEpñk A nig B EdlRtUvKña ³ y CacMnYnsisSRsI ¬KitCa nak;¦
2>  k eRBaH 2 , 4 , 5 , 6 , 1 man Me  4 tambRmab;RbFan eyIg)anRbB½n§smIkar ³
3>  g eRBaH 4 , 6 , 3 , 7 , 5 man x  5 4 x  4 y  200  20 4 x  4 y  180 1
  
4 x  5 y  200  5 4 x  5 y  205 2
4>  x eRBaH 1 , 2 , 5 , 4 , 1 man Mo  1
edaHRsayedayyk 2  1 ³
III. rkRbU)abeRCIserIs)ansisSRsI 1 nak;y:agtic
4 x  5 y  205

sisSQreQµaH 7 nak; manRbus 5 nak; RsI 2 nak; 4 x  4 y  180
-RBwtþikarN_eRCIserIs)ansisSRsI 1 nak;y:agtic y  25

CaRBwtþikarN_bMeBjCamYyRBwtþikarN_eRCIserIsKµan cMeBaH y  25 ³ 1 : 4x  4 y  180


)ansisSRsI mann½yfaerIs)ansisSRbusTaMgGs; b¤ x  y  45 nak; CacMnnY sisSsrubenAkñúgfñak;
naM[ P(s1y:agtic bbb) dUcenH cMnYnsisSsrubenAkñúgfñak;man 45 nak; .
455
9

V. 1> edaHRsayRbB½n§vismIkartamRkabPic VI. 1> sg; D : y   2x  2 kñúgtRmúyGrtUNrem


y  x2

y  2
L  : y  2 x  2
y  x2
eyIgsg;bnÞat;RBMEdn 
y  2
edayeRbItaragtémøelx ³
y  x2 nig y2 D  : y   x  2
2
x 1 2 x 1 2
y 1 0 y 2 2

2> k> rkemKuNR)ab;TisénbnÞat; L


y2
eday L  D naM[ a  a  1
eK)an  12  a  1  a  2
dUcenH emKuNR)ab;Tisén L KW a  2 .
y  x2
x> sresrsmIkarbnÞat; L
bnÞat;RtUvrkmanrag L : y  ax  b
EpñkminqUtCatMbn;cemøIyénRbB½n§vismIkar .
EdlmanemKuNR)ab;Tis a  2 ¬rk)anxagelI¦
2> kMNt;témøén x edIm,I[ ABC EkgRtg; A
-eday L kat;tam A 1 , 0
eyIgman AB  x 1 , BC  x  4 / AC  x  2
eK)an 0  2 1  b  b  2
tamRTwsþIbTBItaK½r ³ ABC EkgRtg;kMBUl A
luHRtaEt BC  AB  AC 2 2 2 dUcenH bnÞat;rk)anKW L : y  2 x  2 .
x  42  x  12  x  22 K> sg;bnÞat; L enAkñúgtRmúyCamYy D 
x 2  8 x  16  x 2  2 x  1  x 2  4 x  4
x 2  8 x  16  2 x 2  2 x  5
¬sUmemIlrUbxagelI Edl)ansg;bnÞat; D  ¦
x 2  6 x  11  0 VI. tamRmab;RbFaneyIgsg;rUb)an ³

man    3  1  11  9  11  20
2
C

naM[ x    31 20  3  2 5  0 minyk A


N I
B

1


  3  20 O M
x2   3 2 5 
1
D
dUcenH témøkMNt;)anKW x  3 2 5 .
¬´sUmykrUbeTAdak;TMB½rfµIedIm,IgayRsYlbkRsay¦
456
9

C mann½yfa INC CaRtIekaNEkgRtg; N


A
N I
B ehIyman N enAelI AC


O

M dUcenH eyIgTaj)an MN    AC  .
3> KNna AC nig IN [Cab;Tak;Tg a nig b
D
k> bgðajfa MBI nig MDI Ca  sm)at eyIgman IA  a nig IC  b
eyIgman AB nig CD EdlEkgKñaRtg;cMNuc I -cMeBaH RtIekaNEkg AIC EkgRtg; I
nigman M CacMNuckNþal BD tamBItaK½r AC  IA  IC
2 2 2

AC  IA2  IC 2  a 2  b 2
enaH BDI CaRtIekaNEkgRtg; I Edlman
IM  Caemdüan RtUvnwgGIub:Uetnus BD
tamTMnak;TMngkñúgRtIekaNEkg AIC EkgRtg; I
eK)an MI  MB  MD eK)an ³ IN  AC  IA IC
 IC
-eday MBI man MI  MB naM[ IN  IAAC
ab ab a 2  b 2
dUcenH MBI CaRtIekaNsm)at .  
a2  b2 a2  b2

-eday MDI man MI  MD dUcenH eyIgKNna[Cab;Tak;Tg a nig b )an


dUcnH MDI CaRtIekaNsm)at . AC  a 2  b 2 ÉktaRbEvg
ab a 2  b 2
vi)ak ³ mMu)at MDI  MID IN 
a2  b2
ÉktaRbEvg .
2> eRbóbeFob IBD nig NCI
eday IBD nig NCI man ³
-mMu NCI  IBD ¬mMucarwkmanFñÚsáat;rYm AD ¦
-mMu NIC  IDB eRBaH
NIC  MID ¬CamMuTl;kMBUl¦

IDB  MDI  MID ¬vi)akxagelI¦

dUcenH IBD NCI tamlkçxNÐ m>m .


vi)ak ³ INC  DIB
Et BDI CaRtIekaNEkgRtg; I enaH DIB  90 o

naM[ INC  DIB  90 o

457
9

sm½yRbLg ³ 11 kkáda 2005


viBaØasa ³ KNitviTüa ry³eBl ³ 120 naTI BinÞú ³ 100

I. cUrKUssBaØa  kñúgRbGb;enAxagmuxcemøIyEdlRtwmRtUvmanEtmYyKt; ³
1> eKmanTinñn½y 1 , 2 , 3 , a , 8 , 7 , 3a  8 . kMNt;témø a edIm,I[mFümTinñn½yKW x  7 .
k> □ a  4 x> □ a  5
K> □ a  6 X> □ a  7 .
2> RtIekaN ABC CaRtIekaNsm)atman AB  AC  5cm nig BC  6cm nigkm<s; AH  enaH
eK)an cos ABˆ H KW ³
k> □ cos ABˆ H  56 x> □ cos ABˆ H  54
K> □ cos ABˆH  53 X> □ cos ABˆ H  12 .
II. fñak;eronmYymansisSGayu 12 qñaM 5 nak; sisSGayu 13 qñaM 8 nak; nigGayu 14 qñaM 7 nak;. RKU)an
ehAsisSmñak;edayécdnü . cUrpÁÚpgÁ Epñk A nig B [)ancemøIyRtwmRtUv ³
A B cemøIy
1> RbU)abEdlRKUehAsisSmanGayu 12 qñaM k> 0.65 1> 
2> RbU)abEdlRKUehAsisSmanGayu 13 qñaM x> 0.6 2> 
3> RbU)abEdlRKUehAsisSmanGayueRkam 14 qñaM K> 0.4 3> 
4> RbU)abEdlRKUehAsisSmanGayu 14 qñaM X> 0.35 4> 
g> 0.25
III. edaHRsaysmIkar  28  1x  12  x  2  3 .
IV. edaHRsayRbB½n§smIkartamRkabPic xy  2x  2 .

V. sYnc,armYymanragCactuekaNEkg manbeNþayelIsTTwg 9 m . eKsg;rbgB½T§CMuvijsYnenaH .
rkRbEvgrbgebIvamanépÞRkLa 112 m . 2

VI. kñúgtRmúyGrtUNrem xoy  eKmancMNuc A0 , 1 nig B4 ,  3 .


1> kMNt;smIkarbnÞat; AB .
2> kMNt;kUGredaenéncMNuckNþal M én AB. kMNt;smIkarbnÞat; D  Ekgnwg AB
nigkat;tamcMNuc M .
3> sg;bnÞat; AB nig D  enAkñúgtRmúyEtmYy .
458
9

VII. ABC CaRtIekaNcarwkkñgú rgVg;p©it O Edlman AB  AC  BC  2 cm . eKKUsGgát;p©it AD


Edlkat; BC  Rtg; H .
1> KNna AD nig BD ebI sin 30  12 , cos 30  23 .
o o

2> eRbóeFob BHD nig AHC . TajrkpleFobdMNUc .


3> yk K CacMNuckNþal BD . bnÞat; KH  kat;RCug  AC  Rtg; I . bgðajfa KI   AC  .

8
cemøIy
I. KUssBaØa  kñúgRbGb;enAxagmuxcemøIyRtwmRtUv III. edaHRsaysmIkar ³
1> kMNt;témø a ³ x> ☑ a  5  28  1x  12  x  2 3
2 7  1x  2 3  x  2 3
eRBaH X  1  2  3  a 78  7  3a  8 2 7x  x  x  2  3  2 3
4a  29 2 7x  2  3
 7  4a  29  49  a  5
7 2 3
x
2> cos ABˆ HKW ³ K>
☑ cos ABˆ H 
3
5
2 7
17  21
eRBaH   ABH man 5cm
A
5cm
x
14
cos ABˆ H 
3 IV. edaHRsayRbB½n§smIkartamRkaPic ³
5 B H C
6cm eyIgmanRbB½n§vismIkar xy  2x  2

II. pÁÚpÁgEpñk A nig B [)ancemøIyRtwmRtUv ³ sg;bnÞat;RBMEdnénsmIkar xy  2x  2
sisSGayu 12 qñaM 5 nak; sisSGayu 13 qñaM 8 nak; 
x 0 2
nigGayu 14qñaM 7nak; naM[krNIGac  5  8  7  20 taragtémøelxRtUvKña y x2
y 2 0

naM[ 1> P(12qñaM)  205  0.25 x2


x 2 2
y 2 0
2> P(13qña)M  208  0.4
3> P(eRkam14qñaM)  520 8  0.65
4> P(14qña)M  207  0.35
dUcenH eyIgpÁÚpÁg)an ³ 1>  g
2>  K
y  x2 x2
3>  k
4>  X . EpñkminqUtCacemøIyénRbB½n§vismIkar .
459
9

V. rkRbEvgrbgénsYnc,arenaH -kMNt;smIkarbnÞat; D 
tag x CaRbEvgTTwg ¬Edl x  0 KitCa m ¦ smIkarbnÞat;RtUvkMNt;manrag D : y  ax  b
naM[ RbEvgbeNþayKW x  9 eday D : y  ax  b kat;tam M 2 ,  1
tambRmab;RbFaneK)an ³ eK)an 1  a  2  b  b  2a 1
x x  9   112
eday D  AB   a  a  1
x 2  9 x  112  0
man   9  4 1 112   81  448  529
2 naM[ a  a1  11  1 ¬eRBaH a  1¦
naM[ x   9 2 1529   9 2 23  0 minyk
1
eK)an b  2a 1  2 11  3
 9  529  9  23
x2 
2 1

2
7 dUcenH smIkarbnÞat; D : y  x  3 .
cMeBaH x  7 enaH x  9  7  9  16
3> sg;bnÞat; AB nig D  kñúgtRmúyEtmYy
eK)an P  2( TTwg + beNþay )
taragtémøelxRtUvKña
 27  16   46 m /
 AB  : y   x  1 nig D  : y  x  3
edayRbEvgrbgCaRbEvgbrimaRtrbs;sYn x 0 1 x 1 2
y  2 1
dUcenH RbEvgrbgsYnc,arKW P  46m . y 1 0

VI. eKmancMNuc A0 , 1 nig B4 ,  3 D  : y  x  3


1> kMNt;smIkarbnÞat; AB
bnÞat;EdlRtUvrkmanrag  AB  : y  ax  b
-eday  AB  : y  ax  b kat;tam A0 , 1
 AB  : y   x  1
eK)an 1  a  0  b  b  1
-eday  AB  : y  ax  b kat;tam B4 ,  3
eK)an  3  a  4  b  4a  b  3 VII. KNna AD nig BD
A
cMeBaH b  1 enaH 4a  b  3  4a  3  b
eK)an 4a  3 1  a  1 2 cm
 O I
dUcenH smIkarbnÞat; AB  : y   x  1 . B //
//
H C
K
2> kMNt;kUGredaenéncMNuc M kNþal AB D

 0  4 1   3 
M ,   M 2 ,  1
¬ebIrUb nigsRmayenATMB½repSgBIKña enaHvaBi)akdl;
 2 2 
GñkGan dUcenH´sUmdak;rUb nigsRmayenATMB½rfµI
dUcenH kUGredaencMNuckNþalKW M 2 ,  1 . edIm,IkMu[GñkGanBi)ak ¦
460
9

A -TajrkpleFobdMNcU
2 3
2 cm
 O I
eday BHD
AHC

BH HD BD
 
AH HC AC
 3 
2 3
3

C
B
K
//
//
H
dUcenH pleFobdMNUcEdlTaj)anKW
D
-eday ABC man AB  AC  BC  2 cm
BH HD BD

AH HC AC
 
3
3
.
naM[ ABC CaRtIekaNsm½gS
3> bgðajfa KI   AC 
vi)ak ABC  ACB  BAC  60 o

-eyIgman BAD  30 enaH DAC  30 o o

-eday ABD man B enAelIrgVg; nig AD Ca


naM[ AH Cakm<sé; n ABC
Ggát;p©it enaH ABD CaRtIekaNEkg Rtg; B .
Taj)an AHˆ C  DHˆ B  90 o

  ABDman ADB  ACB  60 o

naM[ BHD CaRtIekaNEkgRtg; H


¬mMumanFñÚsáat;rYm AB ¦
-mMu DBC  DAC  30 ¬manFñÚsáat;rYm CD ¦ o

naM[   ABD CaRtIekaNEkgknøHsm½gS


-kñúgRtIekaNEkg BHD man K CacMNuckNþal
vi)ak BAD  30 o

BD naM[ HK  CaemdüanRtUvnwgGIub:Uetnus BD


-eK)an cosBAˆ D  AD AB
 AD 
AB
cos BAˆ D tamRTwsþI eK)an BK  KD  KH
eday AB  2cm , cos BAˆ D  cos 30  23 -RtIekaN BKH man BK  KH
o

enaH AD  2  2  2  4  4 33 cm naM[ BKH CaRtIekaNsm)at


3 3 3
2
vi)ak DBC  BHK  30 o

-ehIy sin BAˆ D  BD  BD  AD  sin BAˆ D -mMu BHK  IHC  30 ¬mMuTl;kBM Ul¦ o

AD
 kñúgRtIekaN IHC man ³
eday AD  4 33 cm , sin BAˆ D  sin 30  12 o

-mMu IHC  30 nigmMu ICH  ACB  60


o o

enaH BD  3  2  3 cm
4 3 1 2 3
enaHmMumYyeTot CIH  90 ehIy I enAelI AC o

dUcenH AD  4 33 cm , BD  2 33 cm . naM[ KI   AC 


2> eRbóeFob BHD nig AHC dUcenH eyIgRsay)anfa KI   AC  .
eday BHD nig AHC man ³
-mMu BHD  AHC ¬CamMuTl;kMBUl¦
-mMu ADB  ACB  60 ¬sRmayxagelI¦ o

dUcenH BHD AHC lkçxNÐdMNUc m>m .


461
9

sm½yRbLg ³ 10 kkáda 2006


viBaØasa ³ KNitviTüa ry³eBl ³ 120 naTI BinÞú ³ 100

I. cUrbMeBjcenøaHkñúgtaragxageRkam[)anRtwmRtUv edayKNna x 2 , 3 x , x  17 cMeBaH x  8 .
x 8 8

x2 8
3
x

x  17

II. taragxageRkamenH CakarbgðajBIcMnYnsisS énGnuviTüal½ymYy Edl)anTTYl)anC½ylaPI


sisSBUEkRbcaMqñaM ³
fñak;énGayu ¬qñaM¦ 10-12 12-14 14-16 16-18

cMnYnsisS ¬nak;¦ 3 4 6 3

1> sg;tarageRbkg;ekIn énTinñn½yxagelI .


2> KNnam:Ut nigrkfñak;emdüan énTinñn½yxagelI .
III. k> edaHRsaysmIkar x  2  8  x . 2

x> rk m nig p edaydwgfasmIkar x  mx  p manb£s x  2 nig x  5 .


2
1 2

IV. KNna A   5 5 2  5 2 2 


 
   
B  4 2 4 5  3 3  4 10 1  4 1000  4 2  3 3 .
 1
y  x 1
V. edaHRsayRbB½n§vismIkar  2 .
 y  x
VI. kñúgkabUbmYymanRkdasR)ak; 15 snøkw . enAkñúgenaHmanRkdasR)ak;BIrRbePTKW 1000` nig 5000`.
TwkR)ak;srubman 39000 ` . cUrrkcMnnY RkdasR)ak;RbePTnImYy ² .
VII. fg;mYymanGkSrCaBakü HAPPY . eKlUkykGkSrkñúgfg;mgþ mYyedaymindak;cUlvijeT .
rkRbU)abénRBwtþikarN_ ³
1> lUkyk)anGkSr P .
2> lUkyk)anGkSr AY tamlMdab;enH .
462
9

3> lUkyk)anGkSr HPP tamlMdab;enH .


VIII. rgVg;p©it O manGgát;p©it AB Edl AB  5 cm . bnÞat; L mYyb:HnwgrgVg; O Rtg;cMNuc C ehIy
AC  4 cm . bnÞat;  AD  CYbrgVg;mþgeTotRtg; D ehIyEkgeTAnwg L  Rtg; E . M CacMNuc

RbsBVrvag  AC  nig BD . N CacMNuckNþalGgát; AD .


1> kMNt;RbePT ABC nig ABD. KNnabrimaRtén ABC .
2> bgðajfactuekaN CENO CactuekaNEkg .
3> eRbóbeFob MAB nig MDC . Tajfa MA MC  MB MD .

8
cemøIy
I. bMeBjcenøaHkñúgtarag[)anRtwmRtUv 2> KNnam:Ut nigrkfñak;emdüan
x 8 8 tamtaragTinñn½yfñak;Gayu 14-16 maneRbkg;esµI 6
x2 8 8 eRcInCageK naM[fñak;Gayu 14-16 Cafñak;m:Ut
3
x 2 2
Etm:UtCatémøénp©itfñak; enaH Mo  14 2 16  15
x  17 3 5
dUcenH m:UtKNna)anKW Mo  15 .
-cMeBaH x  8 enaH
x2   82  8  8 -emdüanCatémøkNþal
3
x  3  8  2 edayTinñn½ymaneRbkg;srub 16 naM[témøkNþal
x  17   8  17  9  3 éneRbkg;KW 162  8 . tamtarageRbkg;ekIn ³ eRbkg;
-cMeBaH x  8 enaH
8 sßitenAkñúgfñak; 14-16
x2   82  8 8
3
x 3 82 dUcenH fñak;Gayu 14-16 Cafñak;énemdüan .
x  17  8  17  25  5
II. 1> sg;tarageRbkg;ekIn III. k> edaHRsaysmIkar
x  22  8  x
fñak;énGayu eRbkg; f eRbkg;ekIn f  x 2  4x  4  8  x
10-12 3 3 x 2  3x  4  0
12-14
14-16
4
6
7
13
tamkrNIBiess a  b  c  1   3  4  0
16-18 3 16 naM[ x  1 , x   ac   14  4
1 2
srub 16

dUcenH smIkarmanb£s x 1  1 , x2  4 .
463
9

x> rk m nig p V. edaHRsayRbB½n§vismIkar




1
y  x 1
2
eyIgmansmIkar x  mx  p 2  y  x

manb£s x  2 nig x  5
1 2 eyIgsg;bnÞat;RBMEdn 
1
y  x 1
2
 y  x
cMeBaHtémøbs£ TaMgBIr eK)an ³
2 2  m  2  p  0 taragtémøelxRtUvKña
 2
5  m  5  p  0 y
1
2
x 1 nig yx
4  2 m  p  0
  x y x y
25  5m  p  0 2 2 0 0
 p  2m  4 4 3 1 1
 
 p  5m  25
 5m  25  2m  4
-sg;RkaPic
3m  21 yx
m7
cMeBaH m  7 : p  2m  4  2  7  4  10 y
1
2
x 1

dUcenH témøEdlrk)anKW m  7 , p  10 .
IV. KNna
 5 2 
A    

 5  2 5  2 



5 5 2  2 5 2    EpñkminqUtCacemøIyénRbB½n§vismIkar .

5 2 5 2   VI. rkcMnYnRkdasR)ak;RbePTnImYy²
5  10  10  2

52 tag x CacMnYnRkdasR)ak; 1000`
y CacMnYnRkdasR)ak; 5000`
7

3
tambRmab;RbFaneK)an ³
dUcenH KNna)an A
7
.  x  y  15  x  y  15
 
3 
1000x  5000 y  39000  x  5 y  39

  
B  4 2 4 5  3 3  4 10 1  4 1000  4 2  3 3  tamedETmINg; D  5 1  4
Dx 36
 4 10  4 2  3 3  4 10  4 10000  4 2  3 3 Dx  75  39  36  x   9
D 4
 4 10000 D y 24
D y  39  15  24  y   6
 4 10 4  10 D 4

dUcenH KNna)an B  10 . dUcenH RkdasR)ak; 1000` man 9 snøwk


RkdasR)ak; 5000` man 6 snøkw .
464
9

VII.fg;manGkSr HAPPY man 5 GkSr -KNnabrimaRtén ABC


naM[ krNIGac  5 eday ABC CaRtIekaNmanGuIb:Uetnus AB
rkRbU)abénRBwtþikarN_ ³ tamRTwsþIbTBItaK½r AB  AC  BC
2 2 2

1> lUkyk)anGkSr P eday AB  5 cm nig AC  4 cm


edayGkSr P mancMnYn 2 enaHkrNIRsbesµI 2 naM[ BC  AB  AC 2 2

naM[ P(P) = 52  0.4  52  42  9  3 cm


eK)an PABC  AB  AC  BC
dUcenH RbU)ablUk)anGkSr P KW P(P) =
2
.
5  5  4  3  12 cm

2> lUkyk)anGkSr AY tamlMdab;enH dUcenH brimaRt ABC KW P  12 cm . ABC

P(AY)  P(A)  P(Y/A)


1 1 1
    0.05
2> bgðajfactuekaN CENO CactuekaNEkg
5 4 20
edayctuekaN CENO man ³
dUcenH RbU)ablUk)an AY KW P(AY) = 20 . -mMu OCˆE  90 ¬bnÞat;b:HEkgnwgkaMrgVg;¦
1
o

-mMu NEˆC  90 ¬eRBaH  AD  L Rtg; E ¦


o

3> lUkyk)anGkSr HPP tamlMdab;enH


P(HPP)  P(H)  P(P/H)  P(P/HP)
-mMu ONˆE  90 ¬kaMrgVg;EkgGgát;FñÚRtg;cMNuckNþal¦
o

1 2 1 1
     0.033 ctuekaNEdlmanmMuTaMgbICamMuEkg enaHvaCa
5 4 3 30
ctuekaNEkg ¬eRBaHmMuTI4 k¾CamMuEkgEdr¦
dUcenH RbU)ablUk)an HPP KW P(HPP) = 30 .
1

dUcenH ctuekaN CENO CactuekaNEkg .


VIII. tambRmab;RbFaneyIgKUsrUb)an ³
3> eRbóbeFob MAB nig MDC
E
eday MAB nig MDC man ³
C
D L  -mMu MAB  MDC ¬mMucarwkmanFñÚsáat;rYm BC ¦
N
A
M
B
-mMu AMB  DMC ¬mMuTl;kBM Ul¦
O
dUcenH MAB MDC tamlkçxNÐ m>m .
1> kMNt;RbePT ABC nig ABD ³ vi)ak MAB 
MA MB

eday ABCmankMBUl C enAelIrgVg;manGgát;p©it MDC MD MC
Taj)anBIsmamaRt MA MC  MB MD
AB  enaHvaCaRtIekaNcarwkknøHrgVg;
dUcenH Taj)an MA MC  MB MD .
dUcenH ABC CaRtIekaNEkgRtg; C .
465
9

sm½yRbLg ³ 09 kkáda 2007


viBaØasa ³ KNitviTüa ry³eBl ³ 120 naTI BinÞú ³ 100

I. cUrKUssBaØa  kñúgRbGb;enAxagmuxcemøIyEdlRtwmRtUvmanEtmYyKt; ³
1> smIkar 2x 1  2 manb£s ³
k> □ x  1 22 x> □ x  2  2

K> □ x  2  2 X> □ x  12 2 .


2> RtIekaN ABC CaRtIekaNEkgRtg;kMBUl A . AH Ekgnwg BC  Rtg; H Edl BH  4.5cm
nig HC  2 cm . rkRbEvg AH ³
k> □ AH  9 cm x> □ AH  6.5 cm
K> □ AH  3 cm X> □ AH  3 cm .
II. 1> edaHRsayRbB½n§smIkar 2x xy3y3 11 .

2> edaHRsayvismIkar 2 x  1  4 x  x rYcbkRsaycemøyI elIG½kSéncMnYnBit .
2 2

III. enAkñúgRKYsarmYy «BukmanGayueRcInCagkUnc,gcMnYn 25 qñaM ehIykUnc,gmanGayueRcInCagkUnb¥Ún


cMnYn 5 qñaM . rkGayumñak;² ebIdwgfaplbUkénGayuGñkTaMgbIesµIngw 65 qñaM .
IV. sisSRbus 3 nak; nigRsI 3 nak; )anQreQµaHe)aHeqñat edIm,IeRCIserIseFVICaRbFanfñak; nigbnÞab;mk
eRCIserIsGnuRbFanfñak;mñak;eTot . rkRbU)abénRBwtþki aN_ ³
1> eRCIserIs)anRbFanfñak;CasisSRbus .
2> eRCIserIs)anRbFanfñak;CasisSRsI .
3> eRCIserIs)anRbFanfñak;CasisSRsI nigGnuRbFanfñak;CasisSRbus .
V. enAkñúgtRmúyGrtUNrem xoy  eKmanbnÞat; L  : y  x  2 nigmancMNuc A1 , 2 .
1

1> rksmIkarbnÞat; L  Edlkat;tamcMNuc A1 , 2 ehIyEkgCamYybnÞat; L  .


2 1

2> rkkUGredaencMNucRbsBVrvagbnÞat; L  nig L  ehIyepÞógpÞat;lT§pltamRkabPic .


1 2

VI. rgVg;p©it O manGgát;p©it BC  Edl BC  4 cm . A CacMNucmYyenAelIrgVg; O Edl ABˆC  30 .o

bnÞat; AE  CYbCamYy BC  Rtg; D ehIyCYbrgVg; O mþgeTotRtg; E .


1> kMNt;RbePTRtIekaN ABC nigRtIekaN AOC .
466
9

2> KNna AB , AC , AEˆB .


3> eRbóbeFobRtIekaN ACD nigRtIekaN BED. rYcRsaybBa¢ak;fa DA DE  DB  DC .

8
cemøIy
I. KUssBaØa  kñúgRbGb;enAmuxcemøIyEdlRtwmRtUv -bkRsaycemøIyvismIkarelIG½kScMnYnBit ³
1> smIkar 2x 1  2 manb£s ³ x 
1 0 x
3
k> ☑ x  1 22 EpñkminqUtCacemøIyénvismIkar .
eRBaH 2 x  1  2  2 x  2  1 III. rkGayurbs;mñak;²

enaH x  1  2 2  x  12  1  x  1  22 tag x CaGayukUnc,g ¬Edl x  0 KitCaqñaM¦


2> rkRbEvg AH ³ X> ☑ AH  3 cm naM[ Gayu«BukKW x  25
tamTMnak;TMngkñúgRtIekaNEkg ABC km<s; AH ³ nig GayukUnb¥ÚnKW x  5
AH 2  HB  HC
C 2 cm
H
tambRmab;RbFan eK)ansmIkar ³
AH  HB  HC 4.5 cm x  25  x  x  5  65
3 x  20  65
 4.5  2  3 cm A B
3 x  45
II. 1> edaHRsayRbB½n§smIkar ³ x  15
eyIgmansmIkar 2x xy3y3 11 tamedETmINg; naM[ Gayu«BukKW x  25  15  25  35 qñaM

eK)an D  3  2  1 nig GayuknU b¥ÚnKW x  5  15  5  10 qñaM
Dx  2
Dx  9  11  2  x
D

1
 2 / dUcenH «BukmanGayu 35 qñaM /
D y  11  6  5  y
Dy 5
 5 / kUnc,gmanGayu 15 qñaM
D 1
kUnb¥ÚnmanGayu 10 qñaM .
dUcenH RbB½n§manKUcemøIy x  2 , y  5 .
IV. sisSRbus 3 nak; nigRsI 3 nak;
2> edaHRsayvismIkar ³ naM[ cMnYnkrNIGac  3  3  6 . rkRbU)ab ³
2 x  12  4 x 2  x 1> eRCIserIs)anRbFanfñak;CasisSRbus
4x 2  4x  1  4x 2  x
3x  1
RbusCaRbFan GacRbusCaGnu> b¤GacRsICaGnu>
1 P(b>CaRbFan) = P(b>b) + P(b>s)
x
3 = P(b)  P(b¼b) + P(b)  P(s¼b)

dUcenH vismIkarmancemøIy x   13 . 3 2 3 3 15 1
     
6 5 6 5 30 2
467
9

dUcenH P(b>CaRbFan 1
) =  0.5 . naM[ y  x2
5
2
2
1
2
2

2> eRCIserIs)anRbFanfñak;CasisSRsI dUcenH cMNucRbsBVrvag L  nig L  KW ³


1 2

 1
sisSRsICaRbFan GacRbusCaGnu> b¤GacRsICaGnu> x 

5
2
, y 
2
.
P(s>CaRbFan) = P(s>b) + P(s>s)
-epÞógpÞat;tamRkaPic
= P(s)  P(b¼s) + P(s)  P(s¼s)
3 3 3 2 15 1 L2  : y   x  3
     
6 5 6 5 30 2

dUcenH P(b>CaRbFan) = 12  0.5 .


3> eRCIserIs)anRbFanCaRsI nigGnu>CaRbus
P(RbFanRsI>GnuRbus) = P(s)  P(b) L1  : y  x  2
3 3 3
    0.3
6 5 10

dUcenH P(RbFanRsI>GnuRbus) = 0.3 . tamRkaPiceXIjfa cMNucRbsBVénbnÞat;TaMgBIr


V. 1> rksmIkarbnÞat; L  mankUGredaen  x  52 , y  12  nigEkgKñaBitEmn .
2

eyIgman L  : y  x  2 nigcMNuc A1 , 2 VI. 1> kMNt;RbePT ABC nigRtIekaN AOC


1

smIkarbnÞat;RtUvrkmanrag L  : y  ax  b
2
A

-eday L  kat;tam A1 , 2


2
30 o 
B C
eK)an 2  a 1  b  b  2  a O D

-eday L   L 
2 1
E
eK)an a 1  1  a  1 -edayRtIekaN ABC manGgát;p©it BC  nig A enA
naM[ b  2  a  2   1  3 elIrgVg; ehIy ABˆC  30 enaHvaCaRtIekaNcarwkknøHrgVg;
o

dUcenH smIkarbnÞat;RtUvrkKW L  : y   x  3 . dUcenH ABC CaRtIekaNEkgknøHsm½gS .


2

2> rkkUGredaencMNucRbsBVrvag L  nig L 


1
vi)ak ACB  60
2
o

eyIgman L  : y  x  2 nig L  : y   x  3
1 2
-eday AOC man OA  OC ¬kaMrgVg;EtmYy¦
pÞwmsmIkarGab;sIus énbnÞat;TaMgBIr L  nig L  enaHvaCaRtIekaNsm)at EdlmanmMu)at ACˆB  60
1 2
o

eK)an x  2   x  3  x  52 dUcenH AOC CaRtIekaNsm½gS .


468
9

KUsrUbEtmYy)anehIy ´RKan;Etdak;[RsYlemIl
A

30 o  C
B
O D

2> KNna AB , AC , AEˆB


kñúg ABCman ABˆC  30 nig BC  4 cm o

naM[ cos ABˆ C  BC


AB
 AB  BC  cos ABˆ C

eK)an AB  4  cos 30 o
 4
2
3
 2 3 cm

ehIy sin ABˆC  BCAC


 AC  BC  sin ABˆ C

eK)an AC  4  sin 30 o
 4
1
2
 2 cm

dUcenH RbEvg AB  2 3 cm nig AC  2 cm .


cMeBaH AEˆB  ACˆB  60 eRBaH AEˆB nigmMu
o

ACˆ B CamMucarwkelIrgVg;EtmYy nigmanFñÚsáat;rYm AB

dUcenH AEˆB  60 . o

3> eRbóbeFob ACD nig BED


eday ACD nig BED man ³
-mMu AEˆB  ACˆB  60 ¬manbBa¢ak;xagelI¦
o

-mMu EDˆ B  CDˆ A ¬mMuTl;kMBUl¦


dUcenH ACD BED tamlkçxNÐ m>m .
ACD
vi)ak BED

DA DC

DB DE

Taj)an DA DE  DB  DC .

469
9

sm½yRbLg ³ 19 kBaØa 2008


viBaØasa ³ KNitviTüa ry³eBl ³ 120 naTI BinÞú ³ 100


I. cUrKUssBaØa  kñúgRbGb;enAxagmuxcemøIyEdlRtwmRtUvmanEtmYyKt; ³
1> etIsmIkarmYyNa Edlmanb£sBIrepSgKña ?
 k> x  2 x  1  0  x> x  9  0  K> 5 x  7 x  1  0  X>  x  5x  8  0
2 2 2 2

2> ABC CaRtIekaNEkgRtg;kMBUl A EdlmanGIub:Uetnus BC  a nig ACˆB  60 . o

RCug ABmanRbEvg ³
 k> AB   x> AB   K> AB   X> AB 
a a 2 a 3 a 3
2 2 3 2
II. eKføwgGgár 7 fg;CabnþbnÞab;ehIy)anlT§pl ³ 3 kg , 5 kg , 4 kg , 9 kg , 10 kg , 4 kg nig 7 kg .
cUrpÁÚpÁgEpñk A nig Epñk B rYcsresrcemøIyenAkñúgEpñk C [)anRtwmRtUv ³
Epñk A Epñk B C ¬cemøIy¦
1> m:UténTinñn½yxagelI k> 5 kg 1> 
2> mFüménTinñny½ xagelI x> 7 kg 2> 
3> emdüanénTinnñ ½yxagelI K> 6 kg 3> 
X> 4 kg
III. 1> KNna A   2  8 nig B  12  3  1
2

2> KNna C  2 32 12   6  1 .


IV. kñúgRbGb;mYymanesovePAlMhat;FrNImaRtcMnYn 5 k,al nigesovePAlMhat;BICKNitcMnYn7 k,al.
suPaB cab;ykesovePA 2 k,alecjBIRbGb;enaHedayécdnü .
1> rkRbU)ab EdlsuPaB cab;)anesovePAlMhat;FrNImaRtTaMgBIrk,al .
2> rkRbU)ab EdlsuPaB cab;)anesovePAlMhat;BICKNity:agticmYyk,al.
V. 1> cUrsg;cMNuc A 1 , 2 nig B2 , 0 enAkñúgtRmúyGrtUNrem xOy .
2> rksmIkarbnÞat; AB .
3> sg;bnÞat; D  mansmIkar y  32 x enAkñúgtRmúyxagelI. bgðajfabnÞat; D  nigbnÞat; AB
EkgKña .
470
9

VI. bUNa manGayueRcInCag cinþa . plbUkGayuGñkTaMgBIresµI 33 qñaM . ehIypldkGayuGñkTaMgBIresµI


3 qñaM. rkGayurbs; bUNa nigGayurbs; cinþa .

VII. eK[RtIekaNsm½gS ABC EdlmanRCugRbEvg 4 cm . eKKUsknøHbnÞat; At  RsbnwgRCug BC  .


H CacMeNalEkgén C elI  At  .

1> KNna CH nig AH edaydwgfa cos60  12 nig sin 60  23 .


o o

2> KNna BH .
3> KNnaépÞRkLaénRtIekaN ABH .

8
cemøIy
I. KUssBaØa  kñúgRbGb;xagmuxcemøIyEdlRtwmRtUv³ III. 1> KNna ³
1> smIkarEdlmanb£sBIrepSgKñaKW ³ A  22  8
☑ K> 5x  7 x  1  0 eRBaHvaman   0
2  22  22  2  2  2 2

Edl   7  4  5 1  49  20  29  0 .
2
dUcenH KNna)an A 22 2

2> RCug ABmanRbEvg ³ ☑ X> AB  a 2 3 B  12  3  1


 2 3  3 1
eRBaH sin 60  BC
ABo
C
o  3 1
enaH AB  BC  sin 60 60 a o

AB  a 
3 A B dUcenH KNna)an B  3  1 .
2
II. pÁÚpÁgEpñk A nig Epñk B [)anRtwmRtUv ³ 2> KNna
  6  1
22 2
pÁÚpÁg)an ³ 1>  X / 2>  K / 3>  k C
3 1
edaysareyIgmanTinñn½ydUcxageRkam ³ 21  2  3  1
3 kg , 5 kg , 4 kg , 9 kg , 10 kg , 4 kg nig 7 kg

 3  1 3  1   6  1
21  2  3  1
enaHeyIgGacKNna)annUv ³    6  1
3 1
-m:Ut KW 4kg eRBaH 4kg maneRbkg;eRcInCageK  3 1 6  2  6 1

-mFüm  3  5  4  97 10  4  7  427  6  3 2

-emdüanCatYTI n 2 1  7 2 1  4 énTinñn½yerob dUcenH KNna)an A 3 2

tamlMdab;KW 3 , 4 , 4 , 5 , 7 , 9 , 10 KitCa kg .
471
9

IV. kñúgRbGb;manesovePAFrNImaRt 5 k,al nig -eday AB kat;tam A 1 , 2 eyIg)an


esovePA BICKNit 7 k,al ab  2 1
naM[ cMnYnkrNIGac  5  7  12 -eday AB kat;tam B2 , 0 eyIg)an
1>rkRbU)absuPaBcab;)anesovePAFrNImaRtTaMg 2 2a  b  0 2
eK)an P(F>F) = P(F)  P(F¼F) tam 1 nig 2 eyIg)anRbB½n§smIkar ³
5 4 5  a  b  2
    edayeRbIviFIedETmINg; ³
2a  b  0
12 11 33

dUcenH RbU)ab P(F>F)  335  0.15 . enaH D  1 2  3


2> rkRbU)abEdlsuPaB cab;)anesovePABICKNit Da  2  0  2
Da

D 3
2
a

2
3
y:agticmYyk,al D
Db  0  4  4  b  b 
4 4

D 3 3
RBwtþikarN_Edlcab;)anesovePABICKNit 1 k,al
y:agtic CaRBwtþikarN_bMeBjCamYyRBwtþikarN_cab; dUcenH smIkarbnÞat;EdlRtUvrkKW
Kµan)anesovePABICKNitmYyk,alesaH  AB : y   2 x  4
3 3
eK)an P(F>F) + P(B>1y:agtic ) = 1
naM[ P(B>1y:agtic ) = 1  P(F>F) 3> sg;bnÞat; D : y  32 x enAkñúgtRmúyxagelI
=1 
5 28
  0.85 eyIgsg;bnÞat; D  edayeRbItaragtémøelx
33 33 x 0 2
D  : y  3 x
dUcenH P(B>1y:agtic )=1 
5 28

33 33
 0.85 . 2 y 0 3
-bgðajfabnÞat; D  nigbnÞat; AB EkgKña
V. sg;cMNuc A 1 , 2 nig B2 , 0 enAkñúgtRmúy
edaybnÞat; AB : y   23 x  43 manemKuN
GrtUNrem xOy
R)ab;Tis a   23 nigbnÞat; D : y  32 x
D  : y  2 x
3 manemKuNR)ab;Tis a  32 ehIyplKuNemKuN
A R)ab;TisKW   23    32   1 enHbBa¢ak;fa
B bnÞat; D  nig bnÞat; AB EkgKñaBitEmn
2
y  x
3
4
3 dUcenH D    AB  RtUv)anbgðaj .

2> rksmIkarbnÞat; AB


smIkarbnÞat;EdlRtUvrkmanrag  AB  : y  ax  b
472
9

VI.rkGayurbs; bUNa nigGayurbs; cinþa ³ -kUsIunus ³ cosCAˆ H  AH


AC
tag x CaGayurbs; bUNa ¬KitCaqñaM¦ naM[ AH  AC  cos CAˆ H
y CaGayurbs; cinþa ¬KitCaqñaM¦ 1
 4  cos60o  4 
 2 cm
2
Edl x  y  0 eRBaH cos60 o 
1
bRmab; ³ plbUkGayuGkñ TaMgBIrKW 33 qñaM nigpl 2

dkGayuGñkTaMgBIrKW 3 qñaM -sIunus ³ sin CAˆ H 


CH
AC

tambRmab;enHeyIgsresr)anRbB½n§smIkar ³ naM[ CH  AC  sin CAˆ H


3
 x  y  33  4  cos60o  4   2 3 cm

x y 3
eyIgedaHRsayedaybUkbM)at; 2

 x  y  33
eRBaH sin 60 o 
2
3


eyIg)an ³ x  y  3
2 x  36  x  18
dUcenH AH  2 cm nig CH  2 3 cm .
cMeBaH x  y  33 nig x  18 2> KNna BH
naM[ y  33  x b¤ y  3318 enaH y  15 eday At //BC
At  CH 
 CH   BC Rtg; C

dUcenH bUNa manGayu 18 qñaM nig enaH BCH CaRtIekaNEkgmanGIub:Uetnus BH


cinþa manGayu 15 qñaM . tamRTwsþIbTBItaK½r BH  BC  CH 2 2 2

VII. tambRmab;RbFaneyIgsg;rbU )an ³  2


 4 2  2 3  28 >
A H t naM[ BH  28  2  7  2 7 cm 2

4 cm dUcenH KNna)an BH  2 7 cm .
3> KNnaépÞRkLaénRtIekaN ABH
B C tamrUb S  S  S
ABH ABCH BCH
1> KNna CH nig AH ³ Edl ABCH CactuekaNBñayEkg
eyIgman ABC CaRtIekaNsm½gS enaH ACˆB  60 o

naM[ S  AH  BC  CH
knøHbnÞat; At  RsbnwgRCug BC 
ABCH
2

2  4  2 3
 6 3 cm 2
naM[ CAˆ H  ACˆB  60 ¬mMuqøas;kñúgxñat; AC ¦
o 2

RtIekaN AHC CaRtIekaNEkgRtg; H eRBaH ehIy BCH CaRtIekaNEkgRtg; C


BC  CH 4  2 3
H CacMeNalEkgén C elI  At 
naM[ S BCH 
2

2
 4 3 cm 2

-tamTMnak;TMngRtIekaNmaRtkñúgRtIekaNEkg dUcenH S ABH  6 3  4 3  2 3 cm 2 .


AHC man GIub:Uetnus AC  4 cm eyIg)an ³

473
9

sm½yRbLg ³ 06 kkáda 2009


viBaØasa ³ KNitviTüa ry³eBl ³ 120 naTI BinÞú ³ 100


I. cUrKUssBaØa  kñúgRbGb;enAxagmuxcemøIyEdlRtwmRtUvmanEtmYyKt; ³
1> sisSmYyRkummankm<s;erogKña 14dm , 13dm , 15dm , 13dm , 14dm , 12dm , 11dm .
cUrrkmFüménTinñn½y ³
k> □ X  14 dm x> □ X  13dm
K> □ X  11dm X> □ X  12 dm .
2> AI  Cakm<s;énRtIekaNEkg ABC cMeBaHGIub:Uetnus BC  ehIy IB  4cm nig IC  3cm .
AI  manRbEvg ³
k> □ AI  2 2 cm x> □ AI  12 cm
K> □ AI  2 3 cm X> □ AI  3 2 cm .
II. kñúgRbGb;mYymanesovePAlMhat;FrNImaRtcMnYn 5 k,al nigesovePAlMhat;BICKNitcMnYn 7 k,al.
suPaB cab;ykesovePAmþgmYyk,alcMnnY 2 dg CabnþbnÞab;Kña ecjBIRbGb;enaHedayécdnü nigmin
dak;cUlvijeT.
1> rkRbU)ab EdlsuPaB cab;)anesovePAlMhat;FrNImaRtTaMgBIrk,al .
2> rkRbU)ab EdlsuPaB cab;)anesovePAlMhat;BICKNity:agticmYyk,al.
III. BUsuxepJIR)ak; 2 000 000 erol enAFnaKar A TTYl)anGRtakarR)ak; 5% kñúg 1 qñaM nigepJIR)ak;cMnYn
3 000 000 erol enAFnaKar B )anTTYlGRtakarR)ak; 6% kñúg 1 qñaM . etIBUsuxTTYl)ankarR)ak;

srubb:unµankñúg 1 qñaM BIFnaKarTaMgBIr A nig B .


IV. sYnc,armYymanragCactuekaNEkg EdlmanknøHbrimaRtesµI 14 m . eKdwgfa 3 dgénTTwg esµInwgBak;
kNþalénbeNþay.
1> rkRbEvgTTwg nigbeNþayrbs;sYn .
2> rkcMnYnedImpáakñúgsYn ebIeKdwgfaépÞrbs;sYn 1 m manedImpáacMnYn 4 edIm.
2

V. eK[bnÞat; L  : y  x  1 nig L  : y   x  3 enAkñúgtRmúyGrtUNrem xoy  mYy .


1 2

1> sg;bnÞat; L  nig L  enAkñúgtRmúy xoy  .


1 2

474
9

2> rkkUGredaenéncMNucRbsBVrvag L  nig L  tamkarKNna ehIyeFVIkarepÞógpÞat;lT§pltam


1 2

RkaPic .
3> TajrkcemøIyénvismIkartamRkabPic  yy  x x1 3 .

VI. rgVg;p©it O mYycarwkeRkARtIekaN ABC Edlman BAˆ C  60 , AB  6cm nigmMuBIreTotCamMuRsYc.
o

eKKUs BD Ekgnwg  AC  Rtg; I ehIyCYbrgVg;p©it O Rtg; D . eKedAcMNuc E enAelI BD


[)an BAˆ E  CAˆ D .
1> KNna AI nig BI .
2> bgðajfa ACD nig ABE dUcKña . TajbBa¢ak;fa AB CD  AC  BE .
3> eRbóbeFob ABC nig AED . TajbBa¢ak;fa BC  AD  AC  ED ehIy
AB  CD  BC  AD  AC  BD .

8
cemøIy
I. KUssBaØa  kñúgRbGb;xagmuxcemøIyEdlRtwmRtUv ³ 2> rkRbU)abEdlsuPaB cab;)anesovePABICKNit
1> rkmFüménTinñn½y ³ sisSTTYl)anBinÞú y:agticmYyk,al
¬eRBaH RbGb;TaMg 4 KµancemøIyRtwmRtUv¦ RBwtþikarN_Edlcab;)anesovePABICKNit 1 k,al
2> KNna AI manRbEvg ³ K> ☑ AI  2 3 cm y:agtic CaRBwtþikarN_bMeBjCamYyRBwtþikarN_cab;
eRBaH tamTMnak;TMngkñúg   ABC km<s; AI ³ Kµan)anesovePABICKNitmYyk,alesaH
AI 2  IB  IC C 3cm mann½yfa
AI   IB  IC I
4cm
cab;)anesovePAFrNImaRtTaMgBIrk,al
 3 4
 2 3 cm A B eK)an P(F>F) + P(B>1y:agtic ) = 1
II. kñúgRbGb;manesovePAFrNImaRt 5 k,al nig naM[ P(B>1y:agtic ) = 1  P(F>F)
esovePA BICKNit 7 k,al dUcenH P(B>1y:agtic ) = 1  335  33
28
 0.85 .

naM[ cMnYnkrNIGac  5  7  12 III. rkkarR)ak;EdlBUsxu TTYl)anBIFnaKarTaMgBIr


1>rkRbU)absuPaBcab;)anesovePAFrNImaRtTaMg 2 -karR)ak;EdlKat;TTYl)anBIFnaKar A KW ³
eK)an P(F>F) = P(F)  P(F¼F) 2 000 000  5%  100 000 erol
5 4 5
  
12 11 33 -karR)ak;EdlKat;TTYl)anBIFnaKar B KW ³
dUcenH RbU)ab P(F>F)  335  0.15 . 3 000 000  6%  180 000 erol

475
9

-karR)ak;EdlTTY)anBIFnaKarTaMgBIr A nig B KW
100 000  180 000  280 000 erol L2  : y   x  3 L1  : y  x  1

dUcenH BUsux TTYl)ankarR)ak; 280 000 erol.


IV. 1> rkRbEvgTTwg nigbeNþayrbs;sn Y c,ar
tag x CaRbEvgTTwgrbs;snY ¬KitCa m ¦
y CaRbEvgbeNþayrbs;sn Y ¬KitCa m ¦
tambRmab; RbFaneyIg)anRbB½n§smIkar ³ 2> rkkUGredaenéncMNcu RbsBVrvag L  nig L  1 2

 x  y  14
eyIgman L  : y  x  1 nig L  : y   x  3
1 2
  x  y  14  x  y  14

3x 
y  
6 x  y
 
6 x  y  0
eyIgpÞwmsmIkarGab;sIusénbnÞat;TaMgBIr
 2 x 1  x  3
 x  y  14 2x  2

bUkGgÁnigGgÁ 6 x  y  0 x 1
7 x  14  x  2 m
cMeBaH x  1 enaH y  x  1  1  1  2
naM[ y  6x  6  2  12 m
dUcenH cMNucRbsBVvvag L  nig L  KW 1 , 2 .
1 2

dUcenH RbEvgTTwg x  2 m nig


tamRkab cMNucRbsBVrvag L  nig L  eday
RbEvgbeNþay y  12 m . 1 2

kareFVIcMeNalEkgelIG½kSTaMgBIrKW x  1 nig y  2
2> rkcMnYnedImpáaEdlmanenAkñúgsYn ³ dUcKñaCamYycemøIytamEbbKNnaR)akdEmn .
épÞsYnTaMgGs;  TTwg  beNþay 3> TajrkcemøIyénvismIkartamRkabPic
S  x  y  2  12  24 m
2
> eyIgmanRbB½n§vismIkar  yy  x x1 3

edayépÞsYn 1 m manedImpáacMnYn 4 edIm enaH
2

tamRkab EpñkminqUtCacemøIyénRbB½n§vismIkar .
cMnYnedImpáakñúgsYn  4  S  4  24  96 edIm
VI. 1> KNna AI nig BI C
dUcenH cMnYnedImpáamanenAkñúgsYnKW 96 edIm . kñúgRtIekaNEkg ABI
D
I E
V. 1> sg;bnÞat; L  nig L  enAkñúgtRmúy  xoy 
1 2 EkgRtg; I man 
A 6 cm B
eyIgman L  : y  x  1 nig L  : y   x  3
1 2 BAˆ C  60 , AB  6cm
o

eyIgeRbItaragtémøelx edIm,Isg;bnÞat; naM[ cosBAˆ C  ABAI


 AI  AB  cos BAˆ C
L  : y  x  1 nig L  : y   x  3
1

x y
2

x y
eK)an AI  6   3cm dUcenH AI  3cm .
1
2
0 1 0 3 ehIy sin BAˆ C  AB
BI
 BI  AB  sin BAˆ C
1 2 1 2
-sg;Rkab L  nig L 
1 2
eK)an BI  6  23  3 3 cm enaH BI  3 3 cm
476
9

2> bgðajfa ACD nig ABE dUcKña


Binitü ACD nig ABE man ³
-mMu BAˆ E  CAˆ D ¬smµtki mµ¦
-mMu ACˆ D  ABˆ E ¬mMucarwkmanFñÚsáat;rYm AD ¦
dUcenH ACD ABE lkçxNÐdMNUc m>m .
ACD
vi)ak ABE

AB BE

AC CD

dUcenH eyIgTaj)an AB  CD  AC  BE 1


3> eRbóbeFob ABC nig AED
BinitüKUén ABC nig AED man ³
-mMu ADˆ E  ACˆB ¬mMucarwkmanFñÚsáat;rYm AB ¦
-mMu DAˆ E  CAˆ B eRBaH
CAˆ D  CAˆ E  DAˆ E
BAˆ E  CAˆ E  CAˆ B  DAˆ E  CAˆ B
BAˆ E  CAˆ D

dUcenH ABC AED lkçxNÐdMNUc m>m .


ABC
vi)ak AED

BC AC

ED AD

dUcenH eyIgTaj)an BC  AD  AC  ED 2


-edaybUkGgÁnigGgÁén 1 nig 2 eK)an ³
 AB  CD  AC  BE

 BC  AD  AC  ED
AB  CD  BC  AD  AC  BE  AC  ED
AB  CD  BC  AD  ACBE  ED

eday BE  ED  BD eRBaH E enAelI BD


naM[ AB CD  BC  AD  AC  BD .
dUcenH AB CD  BC  AD  AC  BD .

477
9

sm½yRbLg ³ 05 kkáda 2010


viBaØasa ³ KNitviTüa ry³eBl ³ 120 naTI BinÞú ³ 100

I. cUrKUssBaØa  kñúgRbGb;enAxagmuxcemøIyEdlRtwmRtUvmanEtmYyKt; ³
1> RtIekaNEkg EFG manGIub:Uetnus FG  5 nig EFˆG  60 . EF  manRbEvg ³
o

k> □ EF  5 33 x> □ EF  5 23
K> □ EF  52 X> □ EF  5 3 .
2> rfynþ 5 eRKÓg)andwgkgT½B 8 nak; / 11 nak; / 11 nak; / 8 nak; nig 12 nak;erogKña . mFümén
Tinñn½yxagelIenHKW ³
k> □ X  11 nak; x> □ X  12 nak;
K> □ X  8 nak; X> □ X  10 nak; .
II. KNna A
1
7 3

1
7 3
B3 2 
3
  
4  3  3 12  3 2  45 4  5 8 .
x y z
  
III. 1> edaHRsayRbB½n§smIkar 2 3 4
2 x  3 y  4 z  33
3x  2m  2010 x  m
2> edaHRsaysmIkar 4

2
manGBaØat x .
IV. fñak;eronmYymansisSRsIcMnYn 17 nak; nigsisSRbuscMnYn 23 nak;. RKU)anehAsisSBIrnak;Ca
bnþbnÞab;edayécdnü edIm,I[eLIgedaHRsaylMhat;RbU)ab RbLgRbNaMgKña .
1> rkRbU)abEdlRKUehA)ansisSRsITaMgBIrnak; .
2> rkRbU)abEdlRKUehA)ansisSRbus 1 nak;y:agtic .
3> rkRbU)abEdlRKUehA)ansisSTaMgBIrePT .
V. kñúgtRmúyGtUNrem xoy  eyIgmancMNuc Ax , 2 ; B9 , 2 nig C3 , y  .
1> rkGab;sIuséncMNuc A edaydwgfa AB  12 nigGredaenéncMNuc C edaydwgfa BC  10 .
2> rksmIkarbnÞat; d  : y  ax  b Edlkat;tamcMNuc B9 , 2 nig D3 , 10  .
478
9

VI. rgVg;mYymanp©it O nigGgát;p©it AB Edl AB  8cm . M CacMNucmYyénrgVg;p©it O ehIyEdl


BM  3cm . eKbnøay AB  xag B [)an BE  3cm . bnÞat; L  mYyEkgnwg  AE  Rtg; E

ehIy L CYb  AM  Rtg; N .


1>KNna AE nig AM .
2>bgðajfa MAB nig EAN dUcKña . Tajrk AN nig EN .
3>bgðajfactuekaN BENM carwkkñúgrgVg;mYy EdleKnwgbBa¢ak;TItaMgp©it I nigRbEvgkaM r rbs;va.

8
cemøIy
I. KUssBaØa  kñúgRbGb;enAxagmuxcemøIyRtwmRtUv ³ tamlkçN³smamaRteK)an ³
2x 3 y 4z 2x  3 y  4z
1> EF  manRbEvg ³ K> ☑ EF  52 eRBaH ³ 4
 
9 16

4  9  16

33
 21
F cos EFˆG 
EF
FG
b¤ x y z
  
33
2 3 4  21  7

11
60 o 5
EF  FG  cos60 o  x 11  22
2   7 x   7
E G 1 5  
 5   y 11 
2 2 naM[    y  
33
2> mFüménTinñn½yKW ³ X> ☑ X  10 nak; eRBaH 3  7  7
 z 11  44
8  11  11  8  12 50 4   7 z   7
X   10 nak;  
5 5
II. KNna
dUcenH KUcemøIyénRbB½n§smIkarKW ³
1 1
A
7 3

7 3
x
22
7
, y
33
7
, z
44
7
.
7 3 7 3 2 7
 
 7  3 7  3 73  2> edaHRsaysmIkar

7 eyIgman 3x  2m4 2010  x 2 m
2
eyIgTajecjBIsmamaRtenH)an ³
B3 2 
3
 
4  3  3 12  3 2  45 4  5 8 
 3 8  3 2  3  36  3 2  3  45 32 23x  2m  2010  4x  m 
 2 68 3x  2m  2010  2x  m 
 0 3x  2m  2010  2 x  2m
3x  2 x  2m  2m  2010
x y z
  
III. 1> edaHRsayRbB½n§smIkar 2 3 4 x  2010
2 x  3 y  4 z  33

eKman 2x  3y  4z b¤esµnI wg 2x 3 y 4z
 
dUcenH x  2010 Cab£sénsmIkar .
4 9 16

479
9

IV. kñúgfñak;mansisS ³ RsI 17 nak; nigRbus 23 nak; AB  9  x 2  2  22


naM[ krNIGac  17  23  40 AB  9  x 
2

12 2  9  x 
2
1> rkRbU)abEdlRKUehA)ansisSRsITaMgBIrnak;
eK)an P(s>s) = P(s)  P(s¼s) b¤ 9  x   144 enaH 9  x   144  12
2


17 16 34
   0.17
naM[ 99  xx  1212   xx  213
40 39 195  

dUcenH RbU)abRKUehAsisSRsITaMgBIrnak;KW ³ dUcenH Gab;sIuskMNt;)an x  3 , x  21 .


P(s>s)   0.17 .
34
195 -rkGredaen éncMNuc C ebI BC  10
2> RbU)abEdlRKUehA)ansisSRbusmñak;y:agtic BC  3  92   y  22
BC  36   y  2 
RBwtþikarN_EdlRKUehA)ansisSRbusmñak;y:agtic
2

10 2  36   y  2 
2

CaRBwtþikarN_bMeBjCamYyRBwtþikarN_KµanRbusesaH
b¤  y  2  64 enaH y  2  8
2

mann½yfa ehA)ansisSRsITaMgBIrnak;
eK)an P(s>s) + P(b>1y:agtic ) = 1 naM[  yy  22  88   yy  10
6
 
naM[ P(b>1y:agtic ) = 1  P(s>s)
dUcenH GredaenkMNt;)an y  6 , y  10 .
dUcenH P(b>1y:agtic ) = 1  195
34 161
 .
195 2> rksmIkarbnÞat; d  : y  ax  b
3> rkRbU)abEdlRKUehA)ansisSTaMgBIrePT ³ -eday d  : y  ax  b kat;tam B9 , 2
sisSTaMgBIrePTGac RbusnigRsI b¤ RsInigRbus eK)an 2  a  9  b  9a  b  2 1
P(BIrePT) = P(bs) + P(sb) -eday d  : y  ax  b kat;tam D3 , 10 
= P(b)  P(s¼b) + P(s)  P(b¼s)
23 17 17 23
eK)an 10  a  3  b  3a  b  10 2
   
40 39 40 39 -edayyk 1 - 2
391 391 391
   9a  b  2
1560 1560 780 
eK)an 3a  b  10
dUcenH P(BIrePT)  780
391
 0.50 . 6a  8  a
4
3
 4
V. eyIgmancMNuc Ax , 2 ; B9 , 2 nig C3 , y tam 2 ³ 3a  b  10  b  10  3    14
 3
1> -rkGab;sIuséncMNuc A ebI AB  12
dUcenH smIkarbnÞat; d  : y   4 x  14
3
.

480
9

VI. KNna AE nig AM 3> bgðajfactuekaN BENM carwkkñúgrgVg;mYy ³


N eday AMˆ B  90 enaH NMˆ B  90 ¬mMubEnßm¦
o o

M
I ctuekaN BENM manplbUkmMuQmKña
8cm 3cm NMˆ B  BEˆ N  90  90  180
o o o
/
A E
O B
L  dUcenH ctuekaN BENM carwkkñúgrgVg;mYy .
-bBa¢ak;TItaMgp©it I nigRbEvgkaM r rbs;va
-KNna AE  AB  BE  8  3  11cm
ctuekaN BENM carwkñúgrgVg;man NMˆ B  90 o

-KNna AM ³ ABM CaRtIekaNcarwkknøH mann½yfa BN  CaGgát;p©iténrgVg;enH


rgVg;manGgát;p©it AB enaH ABM CaRtIekaNEkg
tamBItaK½r AM  AB  BM
2 2 2 dUcenH p©it I CacMNuckNþalén BN  .
AM 2  8 2  32
AM 2  55  AM  55 cm
-rkRbEvgkaM r rbs;va
kñúg   BEN ³ tamRTwsþIbTBItaK½r
2> bgðajfa MAB nig EAN dUcKña BN 2  BE 2  EN 2
eday MAB nig EAN man BN  BE 2  EN 2
-mMu AMˆ B  AEˆN  90 ¬RtIekaNEkgTaMgBIr¦
o
 33 
 3  
2

2

-mMu MAˆ B  EAˆ N ¬mMumankMBUlrYm A dUcKña¦  55 


1089
 9
dUcenH MAB EAN tamlkçxNÐTI m>m . 55
99
 9
MAB
vi)ak ³ EAN

AB MA

5
AN EA 45  99

5
naM[ AN  EAMA AB enaH AN  1155 8  885555 cm 12 12 5
  cm
5 5
dUcenH AN 
88 55
55
cm
eday I kNþal BN 
EA  MB
ehIy MB MA
  EN 
12 5
EN EA MA naM[ r
BN
2
 5
2

6 5
5
cm

enaH EN  1155 3  335555 cm


dUcenH kaMrgVg;manRbEvg r  6 5 5 cm .
dUcenH EN 
33 55
55
cm

481
9

sm½yRbLg ³ 04 kkáda 2011


viBaØasa ³ KNitviTüa ry³eBl ³ 120 naTI BinÞú ³ 100

I. RtIekaN ABC mYymanrgVas;RCug AB  x  3 , AC  x  1 nig BC  x  3 Edl x  3 .
rktémø x edIm,I[RtIekaN ABC CaRtIekaNEkgRtg; A .
II. edaHRsayRbB½n§vismIkartamRkabPic  yy  22  x .

III. kñúges,agmYymanXøIBN’Rkhm 4 RKab; BN’exov 3 RKab; nigBN’exµA 5 RKab;. sisSBIrnak;)anykXøI
mñak;mYyRKab; CabnþbnÞab;ecjBIes,agedayécdnü ehIymindak;cUlvijeT .
1> rkRbU)abEdlsisSTI1 yk)anXøIBN’Rkhm ehIysisSTI2 yk)anXøIBN’exµA .
2> rkRbU)abEdl sisSTaMgBIrnak; yk)anXøIBN’exovmYyRKab;mñak; .
IV. BinÞúeFVIetsþsisSmYyRkum TTYl)anlT§pldUcxageRkam ³
BinÞú 0 1 2 3 4

cMnYnsisS 1 2 5 1 x

1> rktémø x ebIBinÞú 3 KWCaemdüanénTinñn½y .


2> rkcMnYnsisS)aneFVIetsþ .
3> rkcMnYnsisS Edl)anBinÞúticCag b¤esµI 3 .
V. eKmanRtIekaN EFG EkgRtg; E ehIyrgVas;mMu EGˆ F  30 . K CacMNucmYyenAelIRCug EG
o

EdlmanrgVas;mMu EKˆF  45 nigrgVas; EK  4cm .


o

1> KNnargVas;mMu GFˆK nigRbEvg EF nig FG .


2> KNnaépÞRkLaénRtIekaN EFG .
VI. kñúgtRmúyGrtUNrem xoy  mYyeKmancMNuc A1 , 0 nig B3 ,  2 .
1> rksmIkarbnÞat; AB .
2> rksmIkarénbnÞat; D  EdlEkgnwg AB Rtg;cMNuckNþal E rbs;Ggát;enH .

8
482
9

cemøIy
I. rktémø x edIm,I[ ABC CaRtIekaNEkgRtg; A III. kñúges,agmanXøI Rkhm 4 exov 3 nigexµA 5
eyIgmanRtIekaNEkg ABC B x  3 naM[cMnYnkrNIGac  4  3  5  12
x 3
EdlcMeBaH x  3 mandUcrUb ³ A x 1 C 1> rkRbU)abEdlsisSTI1yk)anXøIRkhm nig
tamRTwsþIbTBItaK½r ³ edIm,I[ ABC CaRtIekaN sisSTI2 yk)anXøIBN’exµA
EkgRtg; A luHRtaEt ³ AB  AC  BC 2 2 2
naM[ P¬Rkhm exµA¦  P¬Rk¦  P¬xµ¼Rk¦
4 5 5
  
eK)an x  3  x  1  x  3
2 2 2
12 11 33
x2  6x  9  x2  2x  1  x2  6x  9
dUcenH P¬Rkhm exµA¦  335  0.15 .
x  10 x  1  0
2

man    5  1  24
2
2> rkRbU)abEdlsisSyk)anXøIBN’exovdUcKña
naM[ x    51 24  5  2
1 6 3 minyk naM[ P¬exov exov¦  P¬exov¦  P¬exov¼exov¦
3 2 1
  5  24   
x2   5 2 6 12 11 22
1
dUcenH P¬exov exov ¦  221  0.045 .
dUcenH témørk)anKW x  5  2 6 ÉktaRbEvg .
IV. 1> rktémø x ebIBinÞú 3 KWCaemdüanénTinñn½y
II. edaHRsayRbB½n§vismIkartamRkabPic eyIgmantaragTinñn½y
eyIgman  yy  22  x eyIgsg;bnÞat;RBMEdn ³ BinÞú 0 1 2 3 4

D1  : y  2  x D2  : y  2 cMnYnsisS 1 2 5 1 x
x y x y
0 2 0 2 eyIgGacerobBRgayTinñn½ytamlMdab;
1 1 1 2 0 ,1,1, 2 , 2 , 2 , 2 , 2 , 3 , 4 , 4 , ...
 
8 tY x tY
4
eday 3 CaemdüanKWCatYEdlenAkNþaleK
2
naM[cMnYntYEdlenAsgxag 3 RtUvEtesµIKña
4 2 2 4
dUcenH x  8 CatémøEdlRtUvrk .
2 2> rkcMnYnsisSEdl)aneFVIetsþ
4 cMnYnsisS)aneFVIetsþ  1  2  5 1  x , x  8
 9  8  17 nak;
dUcenH tamRkaPicEpñkEdlminqUtCacemøIy
rbs;RbB½n§vismIkar . dUcenH sisS)aneFVIetsþmancMnYn 17 nak; .
483
9

3> rkcMnYnsisSEdl)anBinÞútci Cag b¤esµI 3 naM[ 1


S EFG   EF  EG
2
cMnYnsisS)anBinÞúticCag b¤esµIbI  1 2  5 1 1
  4  4 3  8 3 cm 2
2
dUcenH cM>sisS)anBinÞúticCag b¤esµIbIKW 9 nak; .
dUcenH KNna)an S  8 3 cm . EFG
2

V. 1> KNnargVas;mMu GFˆK nigRbEvg EF nig FG


F VI. 1> rksmIkarbnÞat;  AB 

-kñúgRtIekaN eKmancMNuc A1 , 0 nig B3 ,  2


FKG man ³ E K
4 cm
45 o
G smIkarbnÞat; EdlRtUvrkmanrag y  ax  b
30 o

KGˆ F  GFˆK  EKˆ F ¬ EKˆ F mMueRkARtIekaN ¦


-ebI y  ax  b kat;tam A1 , 0
Taj)an GFˆK  EKˆF  KGˆ F eK)an 0  a 1  b  a  b 1
eK)an GFˆK  45  30  15 o o o
-ebI y  ax  b kat;tam B3 ,  2
eK)an  2  a  3  b  3a  b  2 2
dUcenH KNna)anrgVas;mMu GFˆK  15 . o
-edayyk 1 CMnYskñúg 2
-RtIekaN FEK CaRtIekaNEkgsm)at eK)an 3 b  b  2
eRBaH FEK CaRtIekaNEkgRtg; E nigman  2b  2  b  1 /
mMu)at EKˆF  45 o -cMeBaH b  1 CMnYskñúg 1
vi)ak RCugCab;mMu)at EF  EK  4cm 1 : a  b   1  1

dUcenH KNna)an EF  4 cm . dUcenH smIkarbnÞat;EdlrkKW y   x  1 .


-kñúgRtIekaNEkg EFG 2> rksmIkarénbnÞat; D 
tamrUbmnþRtIekaNmaRt sin EGˆ F  FG EF kUGredaenéncMNuc E kNþalGgát; AB KW
1 3 0  2 
E ,   E 2 ,  1
Taj)an FG  sin EGˆ F
EF  2 2 
bnÞat;EdlRtUvrkmansmIkar D : y  ax  b
eday EF  4 cm nig sin EGˆ F  sin 30  0.5 o

-eday D : y  ax  b kat;tam E2 , 1


eK)an FG  0.5  8 cm
4
eK)an  1  2a  b 3
dUcenH KNna)an FG  8 cm . -ehIy D  AB b¤ D  AB 
naM[ a  a  1  a  a1  11  1
2> KNnaépÞRkLaénRtIekaN EFG
kñúgRtIekaNEkg EFG tamBItaK½r -enaH 3 : 1  2 1  b  b  3
EG  FG  EF  EG  FG  EF
2 2 2 2dUcenH smIkarbnÞat; D : y  x  3 .
2

enaH EG  8  4  48  4 3 cm
2 2

484
9

485
9

sm½yRbLg ³ 16 kkáda 2012


viBaØasa ³ KNitviTüa ry³eBl ³ 120 naTI BinÞú ³ 100

I. cUrKUssBaØa  kñúgRbGb;enAxagmuxcemøIyEdlRtwmRtUv manEtmYyKt; ³
RtIekaN ABC mYyman BAˆ C  90 , AB  3 , AC  a nig BC  4 . rktémø a .
o

 k> a  5  x> a  7  K> a  7  X> a  1

II. cmáarragctuekaNEkgmYymanvimaRt 2x 1 nig 20 KitCaEm:Rt . eKdaMeBatenAelIépÞdI 160 m 2

ehIyeKdaMl¶enAelIEpñkdIenAsl; EdlmanragCactuekaNEkgmanvimaRt x nig 5 KitCaEm:Rt .


rktémø x .
III. sBVéf¶enHsuxmanGayu 35 qñaM ehIyesAmanGayu 14 qñaM. eKdwgfa t qñaMeTAmuxeTot suxnwgman
GayutUcCag 3 dg EtFMCag 2 dgénGayurbs;esA . rkRKb;témøKt;viC¢manén t EdlGacman .
IV. kñúgfg;mYymanXøIs XøIexµA nigXøIRkhm TaMgGs;cMnYn 24 RKab; .
1> eKcab;ykXøI 1RKab;edayécdnü. eKdwgfaRbU)abEdleKcab;)anXøIsesµInwg 13
nigRbU)abEdlcab; )anXøRI khmesµI 14 . rkcMnYnXøIexµA .
2> eKcab;ykXøImþg 3 RKab;edayécdnü. rkRbU)abEdleKcab;yk)an XøITaMgbImanBN’dUcKña .
V. cMNuc M 1 , 1 nig N 3 , 1 enAkñúgtRmúyGrtUNrem xOy mYy.
1> rkRbEvg MN nigkUGredaenéncMNuckNþal P énGgát; MN .
2> rksmIkarbnÞat; d Edlkat;tamcMNuc P nig R2 ,  2 . sg;Ggát; MN nigbnÞat;
d kñúgtRmúy xOy EtmYy .

VI. rgVg;p©it O mYycarwkeRkARtIekaN ABC mYyEdlmankm<s; AH ninmMukñúgTaMgbICamMuRsYc . D Ca


cMNucqøúHén A cMeBaHp©it O .
k> bgðajfa ABˆC  ADˆ C nig ACˆB  ADˆ B
x> eRbóbeFob HAB nig CAD rYcehIy HAC nig BAD .
K> bgðajfa AB AC  AD AH ehIy BAˆ D  HAˆ C .

8
486
9

cemøIy
I. KUssBaØa  kñúgRbGb;xagmuxcemøIyEdlRtwmRtUv³ eyIgGacsresr)an 0  t  7 eRBaH t  0
rktémø a ³ ☑ K> a  7 eRBaH tamRTwsþIbT edaycMnYnKt;viC¢man enAcenøaH 0 nig 7 man ³
BItaK½r BC  AB  AC
2 2
B
2 1,2,3,4,5,6

naM[ AC  BC  AB
2 2
3
2
4
dUcenH témøcMnYnKt; t EdlGacmanKW ³
enaH AC  BC  AB 2 2 1 , 2 , 3 , 4 , 5 nig 6 .
A a? C

 42  32  7 IV. kñúgfg;manXøI s exµA Rkhm TaMgGs; 24 RKab;


II. rktémøén x ³ 20 1> rkcMnnY XøIexµA ¬rebobTI1¦
eyIgGactagcmáarenaH 160 m 2
edayplbUkRbU)abénRBwtþki arN_TaMgGs;kñúg
eBat l¶
)andUcrUbxagsþaM 5
x
viBaØasaEtmYyesµI 1
eXIjfa épÞdIsrub = épÞddI aMeBat + épÞdIdaMl¶ eyIg)an P(s) +P(Rkhm) + P(exµA) = 1
eyIg)an 202x  1  160  5x Taj)an P(exµA) = 1- [ P(s) +P(Rkhm) ]
40 x  20  160  5 x 1 1 7 5
 1     1 
40 x  5 x  160  20 3 4 12 12
35x  140 eRBaHsmµtikmµ s nig Rkhm
P( ) 
1
3
P( )
1
4
140
x 4
35 mü:ageTot tag n CacMnYnXøIBN’exµA
dUcenH témørk)anKW x4m . enaHeyIg)an P(exµA)  24n
III. rkRKb;témøKt;viC¢manén t EdlGacman ³ naM[eyIgpÞwm)an ³ 24n  125 b¤ n  5 1224  10
smµtikmµ ³ sBVéf¶suxGayu 35 qñaM nig esAGayu dUcenH cMnYnXøIBN’exµAmancMnYn 10 RKab; .
14 qñaM ehIy t qñaMeTot suxmanGayutc U Cag 2 dg
÷rkcMnYnXøIexµA ¬rebobTI2¦
EtFMCagBIrdgénGayuesA
-tag x CacMnYnXøIs kñgú cMeNamXøITaMg 24 RKab;
enaHeyIgGacsresr)an RbB½n§vismIkar ³
35  t  314  t  eyIg)an P(s)  24x Etsmµtikmµ P(s)  13

35  t  214  t 
Edl t CacMnYnKt;viC¢manRtUvrk

35  t  42  3t
enaHeyIg)an 24x  13 enaH x  243  8 RKab;

35  t  28  2t -tag y CacMnYnXIøRkhménXøITaMg 24 RKab;
35  42  3t  t

35  28  2t  t
eyIg)an P(Rkhm)  24y Et P(Rkhm)  14
 7  2t

enaHeyIg)an 24y  14 enaH y  244  6 RKab;
7  t

487
9

eday XøITaMgGs;mancMnYn 24 RKab; 2> rksmIkarbnÞat; d


naM[ cMnYnXøIexµA  24  ¬cMnYnXøIs+cMnYnXøIRkhm¦ smIkarbnÞat;EdlRtUvrkmanrag d : y  ax  b
 24  8  6  10 RKab; -eday d  kat;tamcMNuc P1 , 1
dUcenH XøIBN’exµAmancMnYn 10 RKab; . enaHeyIg)an a  b  1 1
-eday d  kat;tam R2 ,  2
¬GñkKYreFVItamrebobTI2 eRBaHvaTak;TgsMNYrbnþ¦ enaHeyIg)an 2a  b  2 2
2> rkRbU)abcab;)anXøITaMgbImanBN’dcU Kña -eyIgyksmIkar 2  1 eyIg)an ³
 2 a  b  2
eKcab;ykXøImþgbI cat;Tku CaviBaØasahUtehIymin 
 a  b 1
dak;vij enaHvaTak;Tgdl;karcab;bnþbnÞab;eTot. a  3
eyIgmanXøI s 8RKab; / Rkhm 6RKab; / exµA 10RKab; yk a  3 CMnYskñúg 1
-XøIbI BN’dUcKña GacCa sTaMgbI b¤ RkhmTaMgbI 1 :  3  b  1 naM[ b  4
b¤ exµATaMgbI enaHeyIg)an ³ dUcenH smIkarEdlRtUvrkKW d : y  3x  4 .
P(XøIbIBN’dUcKña) = P(sss) +P(RkRkRk)+P(xxx)
 8 7 6   6 5 4   10 9 8 
          - sg;Ggát; MN nigbnÞat; d kñúgtRmúy xOy
 24 23 22   24 23 22   24 23 22 
4 7 3 3 5 2 5 9 4
EtmYy
         
 12 23 11   12 23 11   12 23 11  eyIgman M 1 , 1 nig N 3 , 1
84 30 180
  
3036 3036 3036 ehIy d : y  3x  4 xy 04 11
294 49
   0.097
3036 506 eyIgsg;bnÞat; d nigGgát; MN dUcxageRkam ³
dUcenH P(XøIbIBN’dUcKña)  3036
294 49

506
 0.097

V. 1> rkRbEvg MN
eyIgman cMNuc M 1 , 1 nig N 3 , 1
d : y  3x  4
naM[ MN  3  1  1 1  4  4
2 2 2

M  1 , 1 N 3 , 1
dUcenH rk)an MN  4 ÉktaRbEvg .  
P1 , 1

-rkkUGredaencMNuckNþal P énGgát; MN ³
eyIg)an P  12 3 , 1 2 1  b¤ P1 , 1
dUcenH rk)ancMNuckNþal MN KW P1 , 1 .
488
9

VI. tambRmab;RbFaneyIgKUsrUb)an ³ K> bgðajfa AB AC  AD AH


A
eday HABCAD

AB AH

AD AC

O
 naM[eyIg)an AB AC  AD AH
H
B C dUcenH AB AC  AD AH )anbgðajrYc .
D
k> bgðajfa ABˆC  ADˆ C nig ACˆB  ADˆ B -bgðajfa ehIy BAˆ D  HAˆ C
-eday ABˆ C nig ADˆ C CamMucarwkEdlman eday HAC  HAˆ C  BAˆ D
BAD
FñÚsáat;rYm AC naM[ ABˆC  ADˆ C
b¤GacsresrCa BAˆ D  HAˆ C
dUcenH ABˆC  ADˆ C RtUv)anbgðajrYc .
dUcenH BAˆ D  HAˆ C RtUv)anRsaybBa¢ak; .
-eday ACˆ B nig ADˆ B CamMucarwkEdlman
FñÚsáat;rYm AB naM[ ACˆB  ADˆ B
dUcenH ACˆB  ADˆ B RtUv)anbgðajrYc .
x> ÷ eRbóbeFob HAB nig CAD
eday HAB nig CAD man ³
-mMu AHˆ B  ACˆD  90 CamMuEkgdUcKña eRBaH
o

AH Cakm<s;én ABC nig ACˆ D CamMucarwk

knøHrgVg;EdlmanGgát;p©it AD edaysar D
qøúHnwg A cMeBaHp©it O .
-mMu ABˆ H  ABˆ C  ADˆ C ¬bgðajxagelIrcY ¦
dUcenH HAB CAD tamlkçxNÐ m>m .
÷ eRbóbeFob HAC nig BAD
eday HAC nig BAD man ³
-mMu AHˆ C  ABˆ D  90 CamMuEkgdUcKña eRBaH
o

AH Cakm<s;én ABC nig ABˆ D CamMucarwkknøH

rgVg;EdlmanGgát;p©it AD .
-mMu ACˆH  ACˆB  ADˆ B ¬bgðajxagelIrYc¦
dUcenH HAC BAD tamlkçxNÐ m>m .
489
sYsþI¡ elakGñkmitþGñkGanCaTIRsLaj;rab;Gan enAkñúgEpñkenHelakGñknwg)aneXIj GMBIviBaØasaFøab;
ecjRbLgsisSBUEkfñak;TI 9 RKb;qñaM KWcab;BIqñaM 1986 rhUtdl;qñaM 2012 . ´sUmGP½yeTasdl;elakGñk
mitþGñkGan cMeBaHEpñkenHmanEtviBaØasab:ueNÑaHKµancemøIyeT eRBaHlMhat;FrNImaRtxøHenAkñúgsm½yGtItkal
´xVHÉksarBieRKaH nigGñksYreyabl;bEnßm edaysarsisSBUEkCMnan;enaH eRcInCaGñkeFVIkar EdlKat;KµaneBl
evlaRKb;RKan; edIm,I[´)anRbwkSaeyabl;CamYyKat; . EteTaHCay:agNak¾eday ´)andkRsg;lMhat;kñúg
viBaØasaTaMgenHmYycMnYn eTAbkRsaykñúgEpñk lMhat;l¥² nigcemøyI . eyIgKYrdwgfa viBaØasaénqñaMnImYy²man
BIr KWRbLgelIkTI 1 nigRbLgelIkTI 2 . ´nwgBüamerobcMcemøIyrbs;viBaØasaTaMgenH enAeBleRkay .
RbsinebIelakGñkmanbBaða b¤cm¶l;Rtg;cMNucNa EdlmanenAkñúgEpñkenH elakGñkGacTak;Tg;eTAkan;
RKU b¤mitþPkþirbs;GñkEdlmansmtßPaB b¤GñkeroberogesovePAenHkñúgeBlevlaTMng .

vi

: ០៤ ១៩៨៦

: ៧( ១ ២ ៣០

  

7

I. edaHRsaykñúg Q smIkar ³ 4  2 x .
x2 7
II. sYnc,arrbs;ksikrsux manragctuekaNEkg nigbrimaRtmanrgVas; 296m . edIm,IerobcMpøÚvPUmi[Rtg;
KN³kmµkarPUmiseRmckat;beNþaydIKat;Gs; 20m EtedIm,I[RkLaépÞénsYnenAdEdl KN³kmµkarPUmi
seRmcbEnßmTTwgdI[Kat; 12m . cUrrkvimaRtrbs;sYnmunerobcMpÚøvPUmi .
III. «BuknarImanGayu 48 qñaM / narImanGayu 13 qñaM nigb¥ÚnRbusnag Gayu 6 qñaM. etIry³eBlb:unµanqñaM eTIb
Gayu«BuknarIesµI ³
k> 73 énGayunarI . x> 15 dgénGayukUnRbusKat; .
K> esµIplbUkGayuknU Kat;TaMgBIr . X> esµIBak;kNþalénplbUkGayukUnKat;TaMgBIr .
IV. ABC CaRtIekaNsm)atkMBUl A . P CacMNucmYyenAelI BC  . I nig J CacMeNalEkgén P

elIbnÞat; AB nig  AC  .


k>  CabnÞat;EdlRsbeTAnwg AB kat;tam C . J  CacMeNalEkgén P elIbnÞat;  .
bgðajfa J  qøúHnwg J eFobnwgbnÞat; BC  .
x> bgðajfa plbUk PI  PJ efr kalNa P rt;elI BC  .
K> P CacMNucmYyenAelIbnÞat; BC  eRkA BC  . bgðajfa PI  PJ efr viC¢man b¤mYyefr GviC¢man.
V. C  CargVg;Ggát;p©it AB  nig M CacMNucenAelI C  xusBI A nig B . M  CacMNucqøúHén M
eFobnwg AB .
k> I , J , K , L CacMNuckNþalerogKñaén MB , BM  , M A , AM  .
Rsayfa IJKL CactuekaNEkg .
x> kMNt;TItaMgén M edIm,I[ctuekaNEkg IJKL Cakaer .
K> etImanb¤eTTItaMgén M EdleFVI[ctuekaNEkg IJKL CactuekaNesµI EtminEmnCakaer ?

8
 490 

: ០៤ ១៩៨៦

: ៧( ២ ២ ៣០

  

I. 1> rk x , y , z CaFaturbs; Q sMNMuéncMnnY sniTan Edl x  y  z  3 nig 2x  5y  8z .


2> edaHRsaykñúg Q smIkar 12  xx  0 , 12  xx  1 .
1> RsaybBa¢afa ebI a nig b CacMnYnsniTanBIrxusKña enaHeK)an a 2 b  ab .
2 2
II.

2> Gnuvtþn_rUbmnþxagelIbgðajfa 1 999 2  2 001 2


 3 999 999 .
2
III. yuvCnbInak;KW k , x nig K cUlrYmkILart;RbNaMgcm¶ay 100 m .
eBlEdl {K} dl;eKaledA eKeXIj {k} enAxVH 10 m eTot .
eBlEdl {k} dl;eKaledA eKeXIj {x} enAxVH 10 m eTot . eKsnµt;faGñkTaMgbIrt;kñgú el,Ónefr.
1> rkpleFobénel,Ónrbs; k nigel,Ónrbs; K .
2> eBl K eTAdl;eKaledA etIvaenAXøatBI x b:unµanEm:Rt .
IV. eKmanRtIekaN ABC , A CacMNuckNþalGgát; BC  nig D CacMNucqøúHén C eFobnwg A .
bnÞat; AB nig DA CYbKñaRtg; E .
1> RsaybBa¢ak;fa AB  3 AE .
2> O CacMNucminenAelIbnÞat; AB nigminenAelI  AC  . kMNt;cN M uc M elI AB nig
1

M elI  AC  Edl O CacMNuckNþalén M M  .


2 1 2

3> P CacMNucmYyelI BC  . eKKUs PP  Rsbnwg  AC  nig PP  Rsbnwg AB Edl P enAelI
1 2 1

 AB  nig P enAelI  AC  . rksMNMucMNuckNþalénGgát; P P  kalNa P rt;elI BC  .


2 1 2

4> H CaeCIgkm<s;KUsecjBI A . B CacMNucqøúHén B eFobnwg  AH  nig C  CacMNucqøúHén C


eFobnwg  AH  .
k> RsaybBa¢ak;facMNcu B , C , H , B , C rt;Rtg;Kña .
x> kMNt;enAelIrUbenH Ggát;BImanemdüaT½rrYmKña .
kMNt;smÁal; ³ sMNYrTI1 , TI2 , TI3 nigTI4 énlMhat;FrNImaRtminTak;TgKñaeT .

8  491 

: ០២ ១៩៨៧

: ៨( ១ ២ ៣០

  

I. eK[ a  b  c  1 , 1a  b1  1c  0 a  0 , b  0 , c  0 .
bMPøWfa a  b  c  1 .
2 2 2

II. eKmanbIcMnYnxusKña a , b , c .
KNnaplbUk S  a  baa  c  b  cbb  a   c  a c c  b .
2 x  y  3
III. edaHRsayRbB½n§smIkar 
 x  y  3
.
smÁal; ³ témødac;xatén a KW a
a 
ebI a  0 .
 a ebI a  0
IV. eKmanRtIFa A  x  4x  72 .
2

rktémøén x EdlnaM[RtIFa A mantémøGb,brma rYcR)ab;témø Gb,brmaenH .


V. eK[cMNuc B nig C enAelIrgVg; O kaM R ¬ O  BC  ¦ . A CacMNucERbRbÜlelIFñÚFM  BC .
cMNuc H CaGrtUsg;énRtIekaN ABC .
1> bMPøWfa k> BAˆ O  HAˆ C .
x> cMNuc H , G , O rt;Rtg;Kña ¬ G CaTIRbCMuTm¶n;énRtIekaN ABC ¦
2> rksMNMuéncMNuc H ¬bBa¢ak;lImIténcMNucenHpg¦ .

 492 

: ០២ ១៩៨៧

: ៨( ២ ២ ៣០

  

I. edaHRsaysmIkarxageRkamkñúgsMNMucMnYnKt;rWuLaTIb ³
4  x 3  x 2  x 1 x
    4 .
1986 1987 1988 1989
II. 1> sresrBhuFaxageRkamCaplKuNktþa ³
Ax   x 4  x 3  x  1
B x   x 4  x 3  2 x 2  x  1
2> sRmÜlRbPaKsniTan ³
Ax 
E x  
Bx 
.
3> bgðajfa Ex mantémøCacMnYnviC¢man b¤sUnü cMeBaHRKb;témøén x .
III. KNnatémøelxénkenSam ³ aa  bb ebIeKdwgfa 2a  2b  5ab b  a  0 .
2 2

IV. rgVg;p©it O nigrgVg;p©it O kat;KñaRtg; A nig B . xñat;ERbRbYlmYyKUsecjBI B kat;rgVg;p©it O


Rtg; E nigkat;rgVg;p©it O Rtg; F .
1> bnÞat;b:HrgVg;p©it O Rtg; E nigbnÞat;b:HrgVg;p©it O Rtg; F RbsBVKñaRtg;cMNuc P .
bMPøWfa EPF manrgVas;efr .
2> bnÞat; EO nigbnÞat; FO RbsBVKñaRtg; M .
bMPøWfa cMNuc E , M , A , F , P enAelIrgVg;mYy .
3> bMPøWfa emdüaT½rén EF  kat;tamcMNucnwgmYy .

 493 

: ០២ ១៩៨៨

: ៨( ១ ២ ៣០

  

I. edaHRsaysmIkar x  3x  3x  4  0 .
3 2

II. eKmanRbB½n§smIkar ³ 52xx  3yym1 ¬ m CacMnYnKt;sÁal;¦ .


2

kMNt;sMNMutémøKt;ngi GviC¢manén m EdlnaM[KUcemøIy x nig y énRbB½n§epÞógpÞat;lkçxNÐ x  0  y .
III. eKmanbIcMnYn a , b , c Edl a  b  c  0 nig abc  0 .
KNna E  b  a5  c  b  a5  c  a  b5  c .
2 2 2 2 2 2 2 2 2

IV. Ggát; PQ enAkñúgrgVg;p©it O ehIyEkgnwgkaM OS  Rtg;cMNuc I . bnÞat; SP nig SQ
kat;rgVg;erogKñaRtg;cMNcu M nig N .
k> bMPøWfa rgVg;carwkeRkARtIekaN PQN kat;tamcMNuc M .
x> K CacMNucqøHú éncMNuc S cMeBaHcMNuc O . bMPøWfa SMˆ I  PKˆ Q  SNˆ I .
K> knøHbnÞat;BuHkñúgénmMu PIK kat;Ggát; PK  Rtg;cMNuc E . H CacMeNalEkgéncMNuc E
elIGgát; IK  . bMPøWfa ³  IP IPIK IK HE  1 .

 494 

: ០២ ១៩៨៨

: ៨( ២ ២ ៣០

  

I. etIcMnYn A  2  5 ¬ n  IN ¦ bBa©b;edaycMnYnsUnücMnYnb:unµan ?
n 2 n10 

II. bMPøWfa cMeBaHRKb;témø a nig b eK)an ³


a b  5a  9b  6ab  30 a   45  .
2 2 2 2 2

rktémøGb,brmaénkenSamRbPaK ³ F   2xx 513 .


2
III. 2

IV. eKmanmMuRsÜc xOy . cMNuc C cl½tenAelIRCug Ox  nigcMNuc D cl½telIRCug Oy  edaybMeBj
lkçxNÐ ³ OC  OD  2a ¬ a CaRbEvgefr¦. rksMNMucMNuckNþal M én CD .
V. eKdwgfakñúgRtIekaNEkg plbUkkaerénrgVas;RCugmMuEkgTaMgBIr esµIngw kaerénrgVas;GIub:Uetnus .
cUrGñksg;Ggát;FñÚ AB énrgVg;p©it O kaMRbEvg 5 cm Edl AB  8 cm ehIy AB kat;tam
cMNuc P ¬ P enAeRkArgVg;p©it O ¦ .

 495 

: ០២ ១៩៨៩

: ៨( ១ ២ ៣០

  

I. sresrcMnYn N  3a  1  4a  6a  9 CaplKuNktþadWeRkTI1 .


2 2

II. KNna ³ E  x xbaa b  x  xabb a   a axbb x ¬ x ; a ; b CabIcMnYnxusKña ¦ .


III. eK[smIkar 2x  3m  1  m  2  5  3  2  3x  3 ¬ m CacMnYnsÁal; ¦ .
kMNt;sMNMutémøKt;GviCm¢ anén m edIm,I[smIkar manb¤sCacMnYnviC¢man .
IV. AM  CaemdüanénRtIekaN ABC . R)ab;eQµaH BAC tamkrNInImYy²dUcxageRkam ³
k> ebI AM   BC2 .
x> ebI AM   BC2 .
V. cMNuc A cl½telIknøHrgVg;Ggát;p©it BC  . enAeRkARtIekaN ABC eKsg;RtIekaNEkgsm)at
ABE Bˆ  90  nigRtIekaNEkgsm)at ACF Cˆ  90  . I nig K CacMeNalEkgerogKña
o o

éncMNuc E nig F elIbnÞat; BC  . bMPøWfa plbUk EI  FK manrgVas;efr .

 496 

: ០២ ១៩៨៩

: ៨( ២ ២ ៣០

  

bMPøWfa RbPaK F  2ba 51 mantémøelxCacMnYnGviC¢mancMeBaHRKb;témø a nig b .


2
I. 2

II. edaHRsaysmIkar x 1 5  x 2 x  x  21410  2 x  1 .


III. rkBIrcMnYnKt; a nig b a  b edaydwgfaplbUk énBIrcMnYnenaH CaBhuKuNén 15 ehIypldk
kaeréncMnYnTaMgBIresµI 45 .
IV. eK[rgVg;p©it O nigGgát;FñÚ AB . cMNuc I enAkñúgrgVg; cMNuc M enAelIrgVg; nigcMNuc E
enAeRkArgVg; . eRbóbeFob AMB , AIB nig AEB .
V. G CaTIRbCMuTm¶né; nRtIekaN ABC .   CabnÞat;mYykat;tamcMNuc G . H , K , K 

CacMeNalEkgerogKñaénkMBUl A , B , C elIbnÞat;  . bMPøWfa AH  BK  CK  .

 497 

: ០២ ១៩៩០

: ៨( ១ ២ ៣០

  

I. eK[BhuFa ³ E  4a x  20a  9x  45 12ax  60a .


2 2 2 2 2

1> sresr E CaplKuNktþa .


2> kMNt;témø a nig x EdlnaM[ E mantémøGb,brma .
II. rkmYycMnYnedaydwgfa bIdgéncMnYnenaH nigkaeréncMnYndEdlenaH CacMnYnpÞúyKña .
III. eK[RtIekaN ABC Edl AB  BC2 ehIy BC2  AC  BC .
rgVg;Ggát;p©it BC  nigrgVg;Ggátp; ©it AB kat;KñaRtg;cMNuc B nig E .
rgVg;Ggát;p©it BC  nigrgVg;Ggátp; ©it AC  kat;KñaRtg;cMNuc C nig F .
rgVg;Ggát;p©it AB nigrgVg;Ggátp; ©it AC  kat;KñaRtg;cMNuc A nig K .
1> bnÞat; BE  nig CF  RbsBVKñaRtg;cN M uc M . etIcMNuc A tagGVIcMeBaHRtIekaN MBC .
2> bnÞat; EF  nig BC  RbsBVKñaRtg;cN M uc P . bMPøWfa EPˆ B  FAˆ C  2 AMˆ F .
3> cMNuc N cl½telIknøHrgVg;Ggát;p©it BC  EdlKµancMNuc E nig F .
rksMNMucMNuckNþal I én MN  .

 498 

: ០២ ១៩៩០

: ៨( ២ ២ ៣០

  

3x
x
5x  3 2 x
I. edaHRsaysmmIkar 4

3

3
.
4 3

II. eK[ ³ a  b  1 . KNnatémøelxén P  2a  b  3a  b  1 .


3 3 2 2

III. eK[ctuekaNBñay ABCD )attUc AB. knøHbnÞat;BuHén A nig D RbsBVKñaRtg;cMNuc O .


H nig K CacMeNalEkgén O elI AB  nig DC  . bMPøWfa AD  AH  DK .
IV. eK[RtIekaN ABC . eKedAcMNuc P enAkñúgRtIekaN ABC Edl CBˆ P  CAˆ P . M nig N
CacMeNalEkgén P elIRCug BC  nig AC  . I CacMNuckNþalén AB . bMPøWfa IM  IN .

 499 

: ០២ ១៩៩១

: ៨( ១ ២ ៣០

  

sRmÜlRbPaKsniTan ³ E  aa  35ab  2b
2 2
I. 2
ab  6b 2
.
II. kMNt;sMNMutémøén x x   EdlepÞógpÞat;lkçxNÐ ³ x  12 x  3  3x 1  4  x  5 .
III. edaHRsaysmIkar x  3x  7 x  9x  30  0 .
4 3 2

IV. eK[ctuekaNBñay ABCD )attUc AB Edl DC  2 AB . M nig N CacMNuckNþalerogKña


én BD nig AC  . RsaybMPøWfa AB  2 MN .
V. eK[cMNuc E enAeRkArgVg;p©ti O . tam E eKKUsbnÞat;BIr kat;rgVg; O ³ bnÞatt;TI1 kat;rgVg;Rtg;
cMNuc A nig B ehIybnÞat;TI2 kat;rgVg;Rtg;cMNuc C nig D . BC  AD  I  .
RsaybMPøWfa 2AEˆC  AIˆC  BCˆD   3BOˆ D .

 500 

: ០២ ១៩៩១

: ៨( ២ ២ ៣០

  

I. eK[smIkar ³ mx  2x  m  x 1 ¬m CacMnYnKt;sÁal;¦ .


2

kMNt;témø m EdlnaM[smIkarminGacman .
II. R)ab;témøtUcbMputénRtIFa ³ x  43 x  409 .
2

III. RsaybMPøWfa ³ ebIRtIekaN ABC nig ABC b:unKña enaHrgVg;carwkeRkAénRtIekaNTaMgBIr RtUvb:unKña.


IV. eK[RtIekaN ABC . cMNuc M cl½tenAelIRCug AB . eKsg;RbelLÚRkam BMCN Edlman
Ggát;RTUg BC  . rksMNMukMBUl N .

 501 

: ០៤ ១៩៩២

: ៨( ១ ២ ៣០

  

I. pldkrvagRkLaépÞénkaerBIresµInwg 1152 m ehIypldkrvagRbEvgRCugénkaerTaMgBIresµI 16 m .


2

KNnaRbEvgRCugénkaernImYy² .
II. mYycMnYnKt;pSMedaybYnelx elxxÞg;rayKW b elxxÞg;db;KW 5 elxxÞg;ryKW a ehIyelxxÞg;Ban;KW 3 .
KNna a nig b edaydwgfa cMnYnenaHEckdac;nwg 5 pg nig 9 pg .
III. RtIekaN ABC nigRtIekaN ABC man BC  BC nigmMu Aˆ  Aˆ  . RsaybMPøWfa rgVg;carwkeRkA
énRtIekaNTaMgBIr CargVg;b:unKña .
IV. cMNuc B nig C enAelIrgVg;p©it O . cMNuc A cl½telIrgVg;enH . H CaGrtUsg;énRtIekaN ABC .
RsaybMPøWfa AH  b:un ehIyRsbnwgGgát;nwgmYy kalNa A ERbRbÜl .

smÁal; ³ sisSGaceFVIlMhat;NamYymunk¾)an .

 502 

: ០៤ ១៩៩២

: ៨( ២ ២ ៣០

  

I. k> sresrRtIFa A  841a  870ab  225b CakaeréneTVFa .


2 2

x> sresrBhuFa B  196a  841a b  870ab  225b CaplKuNbYnktþa .


4 2 2 3 4

II. eK[bIcMnYnKt; a , b , c xusBIsnU ü Edl a  b  c .


RsaybMPøWfa 1a  ab 3abc
bc  ca
.
III. eKedAcMNuc M enAkñúgRtIekaN ABC .
k> bgðajfa BMˆ C  BAˆ C .
x> eKKUskm<s; AH  nigknøHbnÞat;BuH AD énmMu BAC D  BC  .
KNna HAˆ D CaGnuKmn_én B̂ nig Ĉ .
IV. G CaTIRbCMuTm¶né; nRtIekaN ABC . eKKUs GD //AB  eday D  BC  nig E  BC  .

RsaybBa¢ak;fa BD  DE  EC .

smÁal; ³ sisSGaceFVIlMhat;NamYymunk¾)an .

 503 

: ០៣ ១៩៩៣

: ៨( ១ ២ ៣០

  

I. edaHRsaysmIkarkñúgsMNMucMnYnKt;rWuLaTIb ³ 6x  3x  8x  4  0 .
3 2

II. sRmÜlkenSam ³ E   22aa 11  22aa 11   1  1  1a  41a  .
    
2

III. eK[RtIekaNsm)at OAB OA  OB  . eKedAcMNuc C enAelIRCug OA rYcekbnøayRCug


OB  [)an BD Edl BD  AC . CD kat; AB  Rtg;cMNuc M .
RsaybBa¢ak;fa cMNuc C nig D qøúHKñaeFobnwgcMNuc M .
IV. eK[RtIekaN ABC Edl AB  AC . knøHbnÞat;BuHénmMu BAC kat;emdüaT½rén BC 
Rtg;cMNuc I . H CacMeNalEkgéncMNuc I elI AB .
RsaybBa¢ak;fa AB  AC  2 AH .

smÁal; ³ sisSeFVIlMhat;NamYymunk¾)an .

 504 

: ០៣ ១៩៩៣

: ៨( ២ ២ ៣០

  

I. RsaybBa¢ak;fa RbPaK x 1993


2
 4x  7
mann½yCanic© .
II. KNna A  5  3  8  60 .
III. RsaybBa¢ak;fa cMnYn N  44a  1  100 Eckdac;ngw 32 .
2

IV. eK[ctuekaNBñay ABCD )attUc AB. Ggát;RTUg AC  nig BD kat;KñaRtg;cMNcu O .
bnÞat;Rsbnwg DC  EdlKUsecjBI O kat;RCugeRTt AD nig BC  erogKñaRtg;cMNcu M nig N .
1> RsaybBa¢ak;fa O CacMNuckNþalén MN  .
2> ebI AB  a , CD  b KNna MN CaGnuKmn_én a nig b .
3> RsaybBa¢ak;fa MN 2

1
AB DC

1
.

smÁal; ³ sisSGaceFVIlMhat;NamYymunk¾)an .

 505 

: ០៣ ១៩៩៤

: ៨( ១ ២ ៣០

  

I. RbGb;katugmYyragRbelBIEb:tEkg EdlmanvimaRt 180 mm , 60 mm , 90 mm . rkcMnYnKUbtic


bMputEdlGacerobbMeBjkñúgRbGb;enH .
II. rkBIrcMnYnKt; a nig b a  b edaydwgfa plbUkénBIcMnYnenH CaBhuKuNén 9 ehIypldkkaer
éncMnYnTaMgBIresµI 63 .
III. etIcMnYn A  2  5 n  IN  bBa©b;edaysUnücMnYnb:unµan ?
2n 3n 5

IV. G CaTIRbCMuTm¶g;énRtIekaN ABC . eKKUs GD  //  AB  rYcKUs GE  //  AC  eday D nig E

CacMNucénGgát; BC  . RsaybBa¢ak;fa BD  DE  EC .


V. ABC CaRtIekaNsm)atkMBUl A . P CacMNucmYyenAelI BC . I nig J CacMeNalEkgerogKña

én P elIbnÞat; AB nig  AC  .


1>  CabnÞat;EdlRsbnwg AB kat;tam C . J  CacMeNalEkgén P elIbnÞat;  .
bgðajfa J  qøúHnwg J eFobnwgbnÞat; BC  .
2> bgðajfaplbUk PI  PJ efr kalNa P rt;elI BC  .
3> P CacMNucmYyelIbnÞat; BC  eRkA BC  . bgðajfa PI  PJ efr .

 506 

: ០៣ ១៩៩៤

: ៨( ២ ២

  

x  y 1
I. KNnatémøén F
x  y 1
eFobnwgtémø a  0
edaydwgfa ³ a  x  bx 1

b  1y  a  b b  1 .
II. edaHRsaysmIkarxageRkam kñúgsMNMucnM YnKt;rWuLaTIb ³
5  x 4  x 3  x 2  x 1 x
     5 .
1989 1990 1991 1992 1993
III. rkbIcMnYnKt;rLWu aTIb a , b , c edaydwgfacMeBaHRKb;témø x eK)an ³
x  a x  10   1  x  bx  c  .
IV. eK[Ggát; AB enAkñúgrgVg;p©it O ehIyEkgnwgkaM OC  Rtg;cMNcu D . bnÞat;  AC  nig BC 
kat;rgVg;erogKña Rtg;cMNuc E nig F .
k> bMPøWfargVgc; arwkeRkARtIekaN ABE kat;tamcMNuc F .
x> G CacMNucqøHú éncMNuc C cMeBaHcMNuc O . bMPøWfa CED  AGˆ E  CFˆD .
K> knøHbnÞat;BuHkñúgénmMu ADG kat;Ggát; AG Rtg;cMNuc H . I CacMeNalEkgéncMNuc H
elIGgát; DG  . bMPøWfa ³  DADA  DG  IH
 DG
1 .

 507 

: ០២ ១៩៩៥

: ៨( ១ ២

  

I. KNnakenSam ³ A  x  11x  2  2  x23  x  1  x3x  3 .


II. RsaybBa¢ak;fa cMnYn N  4n  3  25 Eckdac;ngw 8 cMeBaHRKb;témøén n .
2

III. edaHRsaysmIkarkñúgsMNMucMnYnKt;rWuLaTIb ³ 2x  5x  x  5x  3  0 .
4 3 2

IV. eKedAcMNuc B elIGgát; AC  . O Cap©itrgVg;ERbRbYl Edlkat;tamcMNuc B nig C .


1> BM  CaGgát;p©ti énrgVg;p©it O . rksMNMuéncMNuc M .
2> bnÞat;  AM  kat;rgVg;p©it O Rtg;cMNuc P . rksMNMuéncMNuc P .
3> AT  CabnÞat;b:HrgVg;p©it O Rtg;cMNuc T . rksMNMéu ncMNuc T .

smÁal; ³ sisSGaceFVIlMhat;NamYymunk¾)an .

 508 

: ០២ ១៩៩៥

: ៨( ២ ២

  

I. dak;kenSam A nig B CaplKuNktþa ³


A
3
x  53x  32  3 x  12 5  x 
2 2
B  x  2  x  7   x  2  x  7 
2 2 2

II. edaHRsayRbB½n§smIkar ³
2 x  2 y  3xy
 .
6 x  y  4 xy
III. KNna 7  2 12  4  12 .
IV. RtIekaN ABC carwkkñúgrgVg;p©it O kaM R . eKKUskm<s; AH  énRtIekaN ABC .
1> RsaybBa¢ak;fa ABAH AC  2R .
2> bnÞat;b:HrgVg; O Rtg;cMNuc A kat;bnÞat; BC  Rtg;cMNuc I .
2

RsaybBa¢ak;fa ICIB  AC AB
.2

smÁal; ³ sisSGaceFVIlMhat;NamYymunk¾)an .

 509 

: ៨( ១ ២

  

 510 

: ៨( ២ ២

  

 511 

: ០៧ ១៩៩៧

: ៩ ១ ៣

  

I. sresrkenSam A nig B CaplKuNktþa ³


  
A  a x2 1  x a2 1  70 cm
B  2 x  3  9x  5
2 2

II. BinitürUb ³ kñúgbnÞb;mYymankm<s; 2.20 m bursmñak; 2 .1 m

kMBugbBaÄrTU Edlmankm<s; 2.10 m nigmanCeRmA 2.2 m

70 cm . etIbusenHGacbBaÄrTUenH)anEdrb¤eT ?

III. BinitürUb ³ RtIekaN OAB man OA  13cm ; B


OB  5 cm nig AB  12 cm . 12
5
k> etIrUbenHRtwmRtUvEdrb¤eT ? O

13 A

x> cUrKUsrUbenHeLIgvij[)anRtwmRtUv .
IV. kñúgtRmúyGrtUNrem xOy eK[cMNuc A1 , 8 ; B 2 , 1 nig C  23 , 7  .
RsaybBa¢ak;fa ³ cMNcu A , B nig C rt;Rtg;Kña .
V. RtIekaN ABC carwkkñúgrgVg;p©it O . H CaGrtUsg;énRtIekaN ABC ehIy M CacMNuc
kNþalén BC  . RsaybBa¢ak;fa AH  2 OM .
RsaybBa¢ak;fa RbPaK 2xx 53 mantémøelxCacMnYnviC¢man b¤sUnücMeBaHRKb;témø x .
2

VI. 4

 512 

: ០៧ ១៩៩៧

: ៩ ២ ៣

  

I. KNna ³ E  11 6 2  11 6 2 .


II. edaHRsaysmIkar ³ x  3x 10  0 .
4 2

2 2 2 F
III. KNna ³ S  a  baa  c   b  cbb  a   c  acc  b .
IV. xügehonmYyenAcm¶ay 1.50 m BICBa¢aMgEdlman 2m
B
km<s; 2 m . xügehonenHvartampøvÚ EIF Edl 1 .5 m
I A

I  AB  ¬emIlrUb¦. kMNt;TItaMgén I elI AB 



E
edIm,I[RbEvgpøÚv EIF xøIbMput .
V. eK[RtIekaNsm)at OAB OA  OB  . eKedAcMNuc C enAelIRCug [OA] rYcbnøayRCug [OB]
[)an BD  AC . CD kat; AB Rtg;cMNuc M . RsaybBa¢afa cMNuc C nig D qøúHKñaeFobnwg
cMNuc M .
VI. eKmankaer ABCD . M nig N CacMNuckNþalerogKñaénRCug AB nig BC 
k> RsaybBa¢ak;fa MC  nig ND EkgKñaRtg; I .
x> eRbóbeFob AI nig BC .

 513 

: ១៥ ១៩៩៨

: ៩ ១ ៣

  

I. KNnaplbUk ³ S  3   5   7  2 3  .
2 2 2 2

II. bBa¢ak;dWeRkénsmIkar ³ x  2x  5  x  4 .


2

III. KNnaplEck ³ D  9x  27  2 4 x  12 ¬eday x  0 ¦


IV. dak;kenSam E  4 x  9  53  2 x CaplKuNktþa .
2

V. eK[RbPaK F  BA  2xx14 . kMNt;sMNMutémøén x EdlnaM[ A  0 nig B  0 .


VI. eK[Ggát; AB nigcMNuc C cl½tkñúgbøg;EdlxNÐedaybnÞat; AB . rgVg;Ggát;p©it AC  p©it O
kat;bnÞat; AB Rtg;cMNuc E ehIykat;rgVg;Ggát;p©it AB Rtg;cMNucmYyeTot D .
1> RsaybBa¢ak;fa cMNuc B , C nig D rt;Rtg;Kña .
2> M CacMNucqøúHén E eFobnwgcMNuc O . rksMNMucMNuc M .

smÁal; ³ ebkçCnGaceFVIlMhat;NamYymunk¾)an .

 514 

: ១៥ ១៩៩៨

: ៩ ២ ៣

  

x2  2 y2 x2  2 y2 x2
I. eK[ ³ 306

294
. KNna y2
.
x 2
1
II. edaHRsaysmIkar ³ x 2 
1
.
x 2 x 2 2

x 2 x 2
III. eKEcknMeTA[ekµgbInak; . GñkTImYyTTYl)anBak;kNþal éncMnYnnMsrub nignMmyY kMNat;eTot.
bnÞab;mkGñkTIBIrTTYl)an Bak;kNþaléncMnYnnMEdlenAsl; ¬eRkayeBlGñkTImYyTTYlykehIy¦
nignMmYykMNat;eTot . TIbBa©b;GñkTIbTI TYl)an Bak;kNþaléncMnYnnMEdlenAsl; ¬eRkayeBlGñkTIBIr
TTYlykehIy ¦ nignMmYykMNat;Tot ehIynMBuMmanenAsl;eToteT . cUrrkcMnYnnMEdleKman .
IV. ABC CaRtIekaNEkgRtg; C ehIy CH  Cakm<s;énRtIekaNenH . rgVg;Ggát;p©it  AH  nig BH 

kat; AC  nig BC  erogKñaRtg; P nig Q . bgðajfa PQ CabnÞat;b:HrYménrgVg;TaMgBIrenaH .


V. A CacMNucmYycl½tenAelIrgVg;p©it O ehIy IJ  CaGgát;p©itnwg . cMNuc P CacMeNalEkgBI A

mkelI IJ  . Q CacMNucmYyenAelI OA Edl OQ  OP . rksMNMucMNuc Q kalNa A


cl½tenAelIrgVg; .
VI. C  CargVg;mYyEdlmanp©it O ehIy P CacMNucmYysßitenAkñúgrgVg; . eKKUsbnÞat;mYykat;tam P
CYbrgVg;Rtg; A nig B . I CacMNuckNþalén AB .
k> rksMNMucMNuc I kalNa AB vilCMuvij P .
x> bBa¢ak;TItaMgén AB edIm,I[Ggát;FñÚ AB manrgVas;xøIbMput .

smÁal; ³ ebkçCneFVIlMhat;NamYymunk¾)an .

8
 515 

: ២១ ១៩៩៩

: ៩ ១ ៣

  

I. edaHRsaysmIkar 2  2 2  2   2 .
18 17 15 14 x

II. etImancMnYnKt;b:unµanEdlsßitenAcenøaH 99 nig 9999 edaydwgfacMnYnKt;TaMgenaHCakaerR)akdpg


ehIyEckdac;nwg 11 pg .
III. edaHRsayRbB½n§smIkar xxyyz 4 6z  13 . 2

IV. rkRkLaépÞénRtIekaN ABC tamrUbxageRkam ³
5 B

4 5

2 A C

V. KNnaplbUkmMutamrUbxageRkam ³
S  Aˆ  Bˆ  Cˆ  Dˆ  Eˆ  Fˆ

C F

A B

D E

 516 

: ២១ ១៩៩៩

: ៩ ២ ៣

  

I. cMeBaHRKb;cMnYn x bgðajfa ³ 0  x  61x  10  1 .


2

II. plbUkénBIrcMnYnesµInwg 1 . bgðajfa plKuNvatUcCag b¤esµInwg 14 .


III. ebI abc  10 KNnatémøénplKuN P Edl ³
  1 1 1   1 1 
    .
1 1 1
P    
 a  b  c  a b c  ab  bc  ac  ab bc ac 
IV. eK[ MNPQ CakaermYyEdlmanRkLaépÞesµInwg 9x ehIy ABCD CactuekaNEkgEdl
2

AM BM CP DP 1

AQ BN CN DQ 2
   . KNnaépÞRkLaén ABCD .
Q A M

B
D

P C N

V. P nig Q zitenAelIrgVg;mYyEdlmanp©it O . bnÞat;mYyb:HrgVg;Rtg; P ehIyeKedAcMNuc R eday


ély:agNa[rgVas; PR esµInwgRbEvgFñÚ PQ . bgðajfa EpñkqUtTaMgBIrmanépÞRkLaesµIKña .

O
Q
M

P R

 517 

: ១១ ២០០០

: ៩ ១ ៣

  

I. edaHRsaysmIkar x  6 x   2  3 x .
2000 1998 1999

II. a nig b CacMnYnKt;rWuLaTIb . cUrkMNt; a nig b edIm,I[smIkarxageRkamenHepÞógpÞat;Canic© ³

a 2  b  44  24 2 .
2

III. enAkñúgtRmúyGrtUNrem eK[cMNucEdlmankUGredaen ³ 0, 0 ; 0,1 ; 0, 2 ; 1, 0 ; 1,1 ; 2, 0 .


rkrgVas;RCugénRKb;RtIekaN minb:unKñaEdlmankMBUlCa 3 cMNuc kñúgcMeNamcMNucTaMg 6 Edl[ .
IV. ABCD nig DEFG CakaerEdlmanRCugerogKña a nig x . AOˆ B   .

KNna x CaGnuKmn_én a nig tg .


B C

a E F
x

A D G O

V. knøHbnÞat;BuHkñúgénmMu B énRtIekaN ABC mYykat;knøHbnÞat;BuHkñúg nigeRkA énmMu A Rtg; I nig E


nigkat;rgVg;carwkeRkARtIekaN ABC Rtg; D . RsaybMPøWfa ID  DE .

 518 

: ១១ ២០០០

: ៩ ២ ៣

  

I. eK[kenSam ³
A  100 2  99 2  98 2  97 2  96 2  ...  1
B  100  99  98  97  96  ...  1
k> cUreRbóbeFob A nig B .
x> etI 2  2 Eckdac;nwg 5 b¤eT ? cUrbgðaj .
20 18

1
 27  3 1
K> sresr A  3 54     3
 4  4
CasV½yKuNén 2 .
II. edaHRsayRbB½n§smIkar ³
 x  y  35
3 3


xy  6
.

III. RkLaépÞctuekaNEkgmYynAdEdl kalNaeKbEnßm 2.5 dm elIbeNþay nigdk 23 dm BITTwg
b¤kalNaeKdk 2.5 dm BIbeNþay nigEfm 43 dm elITTwg .
KNnaRkLaépÞctuekaNEkgKitCa dm . 2

IV. eragcRkmYymankmµkrelIsBI 600 nak; nigticCag 700 nak; . ebIkmµkrTaMgGs;eFVIkarCaRkumEdl


manKña 5 nak; b¤ 7 nak; b¤ 9 nak; enaHKµansl;kmµkreT . rkcMnYnkmµkrTaMgGs; .
V. rgVg;p©it O kaM R nigrgVg;p©it I kaM r R  r  b:HKñaRtg; A nigb:HbnÞat;EtmYyRtg; B nig C .
KNnaRkLaépÞénRtIekaN ABC CaGnuKmn_én R nig r .

 519 

: ០៣ ២០០១

: ៩ ១ ៣

  

I. cUrRsaybMPøWfa x  13  2  1 2  1 CacMnYnKt;viC¢man .


3 3 3
3

II. eK[ a , b , c CabIcMnYnEdlepÞógpÞat; TMnak;TMngTaMgbIxageRkamenH ³


a  2b  1  0 ; b  2c  1  0 ; c  2a  1  0 .
2 2 2

cUrKNna S  a  b  c .
2002 2002 2002

 x  y  z  xyz

III. cUredaHRsayRbB½n§smIkar  x  y  z  xyz .
 x  y  z  xyz

IV. tamcMNuc M enAkñúgRtIekaN ABC mYy eKsg;Ggát; EF  , GH  nig IJ  RsberogKñanwg
AB , AC  nig BC  edaycMNuc G nig I enAelI AB . E nig J enAelI AC  . F nig H
enAelI BC  . RtIekaN MEJ; MFH nig MGI manépÞRkLaerogKña S , S nig S . 1 2 3

RsaybMPøWfa RtIekaN ABC manépÞRkLaesµInwg  S  S  S  . 1 2 3


2

V. eK[RbelLÚRkam ABCD mYy . enAxageRkA ABCD eKsg;kaer ABMN , BCPQ , CDRS nig
ADXY Edlmanp©iterogKña I , I , I nig I . cUrRsaybMPøf
1 2 3 W a ctuekaN I I I I Cakaer .
4 1 2 3 4

 520 

: ០៣ ២០០១

: ៩ ២ ៣

  

I. 1> cUreRbóbeFob 1 2000  nig 2001  2  2000  .


2 2

2> edaymineRbI 2000  4 000 000 nig 2001  4 004 001


2 2

cUrKNna A  1  2000   2000 


 
2001 
2000
2001
. 2

II. eK[ a  b  c  1 ; a  b  c  1 nig ax  by  cz  m .


2 2 2

cUrKNna P  xy  yz  zx .
III. eKmansmIkardWeRkTI2 ³ ax  2bx  c  0 EdlmanDIsRKImINg; 
2
1

bx  2cx  a  0 EdlmanDIsRKImINg; 
2
2

cx  2ax  b  0 EdlmanDIsRKImINg; 
2
3

cUrKNna       rYcbgðajfa y:agticNas; k¾mansmIkarmYy kñúgcMeNamsmIkarTaMgbIenaH


1 2 3

CasmIkarmanb£sEdr .
IV. eK[ AA  d  ; BB  d  ; AA  a ; BB  b . A

A nig B sßitenAEtmçagénbnÞat; d  . M CacMNuc B 


a
RbsBVrvagbnÞat;  A B  nig  A B   . b
d 
cUrRsaybMPøWfa cm¶ayBI M eTAbnÞat; d  mantémøefr A B
kalNacMNuc A nig B cl½telI d  ¬cMNuc A epSgBI B ¦.
V. rgVg;p©it O kaM r nigrgVg;p©it O kaM r b:HKñaxageRkARtg; M . rgVg;p©it O kaM r mYyeTotb:H
1 1 2 2 3 3

xageRkAnwgrgVg;p©it O pg ehIyb:HxageRkAnwgrgVg;p©it O pg . bnÞat;bH: rYmRtg; M énrgVg; O 


1 2 1

nig O  CYbrgVg; O  Rtg; A nig B . I CacMNuckNþalénGgát; AB .


2 3

cUrRsaybMPøWfa AB  4rr rrr . 3 1 2

1 2

8
 521 

: ២៣ ២០០២

: ៩ ១ ៣

  

I. cUrRsaybMPøWfa ³ 2001 2003 2002  12002  12002  12002  1  2002 1 .


2 4 8 16 32

II. cUredaHRsaysmIkar ³ x  2x  4x  2 x  3  x  2 x  7 .


2 2 2

III. eK[ a ; b ; c CabIcMnYnEdlepÞógpÞat;TMnak;TMng abc  1 .


cUrRsaybMPøWfa ³ 1  a1 ab  1  b1 bc  1  c1 ca  1 .
IV. cUrRsaybMPøWfa ³ 49  20 6  49  20 6  2 3 .
4 4

V. eK[ctuekaNEkg ABCD mYy . I CacMNucmYyenAxagkñúgctuekaNEkg ABCD ehIyEdl


IA  a ; IB  b ; IC  c ; ID  x . cUrRsaybMPøWfa ³ x  a  b  c .
2 2 2 2

VI. ABC CaRtIekaNEkgRtg; A ehIyman ABˆ C  2 . rgVg;p©it M kaM R carwkkñúgRtIekaN ABC .

rgVg;p©it N kaM r b:HrgVg;p©it M kaM R ehIyrgVg;p©ti N kaM r enaHb:HnwgRCug AC  nig BC  rbs;
RtIekaN ABC eTotpg .
1> cUrRsaybMPøWfa ³ MN  sin R45 r   .
o

2> cUrKNnapleFob Rr .

 522 

: ២៣ ២០០២

: ៩ ២ ៣

  

I. ctuekaNe):ag ABCD carwkkñúgrgVg;p©it O mYy. bnÞat; AB CYb CD Rtg; I ehIybnÞat;  AD 
CYb BC  Rtg; J Edl IAˆ J  3 ; AJˆB  2 ; AIˆD   . cUrKNna  KitCadWeRk .
II. eK[ x  y  1 nig bx  ay Edl a  0 , b  0 .
2 2 2 2

cUrRsaybMPøWfa ³ xa  by  a  2b .
2002 2002

1001 1001 1001

III. cUredaHRsayRbB½n§smIkar ³
 x  y  y  z   187

 y  z  z  x   154 ¬Edl x; y; z CacMnYnviC¢man¦ .
 z  x  x  y   238

IV. eK[ 1 1 1
  2
a b c
nig .
a  b  c  abc

cUrRsaybMPøWfa ³ a
1
2
1 1
.
 2  2 2
b c
V. EFGH CasYnc,arragctuekaNBñay Edlman)atFM EF  a nig)attUc GH  b . eKEcksYnc,ar

enaHCaBIrEpñk EdlmanépÞRkLaesµIKña edayeFVIrbgtam MN  Rsb)atTaMgBIrehIy M  EH  /


N  FG  . cUrKNnaRbEvg MN [Cab;Tak;Tg a nig b .

VI. P CacMNucmYyenAelIrgVg;p©it O kaM R Ggát;p©it AB  .

1> cUrKNnaépÞRkLaGtibrmarbs;RtIekaN PAB .


2> RsaybMPøWfa PA PB  2R ehIy PA  PB  2R 2 .
2

 523 

: ២៨ ២០០៣

: ៩ ១ ៣

  

I. KNnaplbUkedaymineRbI]bkrN_Kitelx ³ S  13  151  351  ... 9999 1


.
II. cMnYnmYymanelxbIxÞg; Edlman 4 CaelxxÞg;ray. ebIeKelIkRtLb;elxxÞg;ray mkdak;xagmux
énelxBIrxÞg;eTot enaHeKnwg)ancMnYnTIBIrfµImYyeTot EdlmanelxbIxÞg; ehIyman 4 CaelxxÞg;ry.
ebIeKdwgfa {cMnYnTIBIr} elIs {cMnYn 400 dkcMnYnTImYy} 400 . rkcMnYnTImYyenaH .
III. rkRKb;cMnYnKt;sniTan EdlepÞógpÞat;smIkar 2x  2xy  y  25 .
2 2

2003
 
IV. bgðajfa  4  5 3  5 48  10 7  4 3  4 CacMnYnKt;sniTan .
 

V. rgVg;BIrmanp©ti O nig O ehIyenAeRkAKña . bnÞat;b:HrYmeRkAmYyb:HrgVg;TaMgBIrerogKñaRtg; M


1 2

nig N Edl MN  a . bnÞat;b:HrYmkñgú mYyb:HrgVg;TaMgBIrerogKñaRtg; P nig Q Edl PQ  b /


a  b  0 . KNnaplKuNrvagkaMénrgVg;TaMgBIrenaH[Cab;Tak;Tg a nig b .
VI. M CacMNucmYyenAxagkñúgRtIekaN ABC mYy. bnÞat; BM  CYbRCug AC  Rtg; N . bnÞat;
CM  CYbRCug AB  Rtg; L . BLM , CMN nig MBC manépÞRkLaerogKña 5 cm , 2

8 cm nig 10 cm . KNnaépÞRkLaénctuekaN ALMN .


2 2

 524 

: ២៨ ២០០៣

: ៩ ២ ៣

  

I. A
1

2

3
1 2 1 2  3 1 2  3  4
 ...
2002
1 2  3  ... 2003
.
bgðajfa A  1
1
1 2  3  ... 2003
.
II. eKdwgfa a  b  a  3b  4a  b  10a  3b  29  0 .
2 2

KNnatémøelxén 2a  4b .
III. kñúgkarRkal)atbnÞb;mYy EdlmanragCactuekaNEkg eK)aneRbI\dækRmalBN’Rkhm nigBN’s
EdlmanragCakaer ehIymanRCugRbEvg 5dm . eK)aneRbI\dækRmalBN’Rkhm nig\dækRmalBN’s
Gs;cMnYnesµIKañ . bnÞab;enaHmanTTwg 5x nigbeNþay 5 y ¬KitCa dm ¦. eK)anRkal\dæBN’Rkhm
TaMgGs;enAtamekonCBa¢aMg EtmYyCYrB½TC§ MuvijbnÞb; ehIyeRkABIenaH eK)anRkal\dækRmalBN’s
TaMgGs; . kMNt;cMnYnKt; x nig y TaMgGs;EdlGacman .
IV. a , b , c CabIcMnYnKt;EdlepÞógpÞat; a  b  c  1 ; a  b  c  1 nig a  b  c  1 .
2 2 2 3 3 3

KNnatémøelxén P  a  b  c . 2002 2003 2004

V. M CacMNucmYyenAkñúgkaer EFGH mYy ehIyEdl MEˆ G  MGˆ H   0    45  . o o

KNnargVgs;mMu EFˆM [Cab;Tak;Tgnwg  .


VI. rgVg;bImanp©iterogKña I , I nig I ehIymankaM R dUcKña . rgVg; I  CYbrgVg; I  Rtg;cMNuc O
1 2 3 1 2

nig A . rgVg; I  CYbrgVg; I  Rtg;cMNuc O nig B . rgVg; I  CYbrgVg; I  Rtg;cMNuc O nig C .


2 3 3 1

bgðajfacMNuc A , B nig C sßitenAelIrgVg;EtmYy EdlmankaM R Edr .

 525 

: ១១ ២០០៤

: ៩ ១ ៣

  

I. rk x edIm,I[ 3x  6x  9x  12x  1  12  13  14 .
II. KUb 7 b:un²Kña pÁúMCab;KñadUcrUbxagerkamenH ³

maDénsUlItEdlpÁúM)anesµI 189 cm . rképÞRkLaTaMgGs;énsUlItenH .


3

III. rkRKb;témø m m  0 edIm,I[smIkar mx  m 1 0 manb£sviC¢man .


IV. ksikrmñak;mandMLÚg 5 l¥IEdlmanTm¶n;erogKña 7kg , 10kg , 14kg , 18kg nig 19kg .
Kat;)anlk;dMLÚg 4 l¥I[mnusS 2 nak;. GñkTI1 Tij)andMLÚgEdlmanTm¶n;esµI 2 dgénTm¶n;
dMLÚgrbs;GñkTI2 . rkTm¶n;dMLÚgEdlenAsl; .
V. RtIekaN ABC mYymanrgVas;RCug x , x  a nig x  2a Edl a CacMnYnKt;viCm¢ an .
rkRKb;rgVas;RCugénRtIekaN ABC EdltUcCagb¤esµI 10 edIm,I[RtIekaN ABC CaRtIekaNEkg .
VI. RtIekaNmYymanrgVas;RCug 3 , 4 nig 5 . RtIekaNenHcarwkkñúgrgVg;mYy. tag A , B nig C CaépÞ
RkLaénEpñkbøg; EdlenAkñúgrgVg; nigenAeRkARtIekaN ehIy C CaépÞRkLaEdlFMCageK .
KNna A + B [Cab;Tak;Tgnwg C .

 526 

: ១១ ២០០៤

: ៩ ២ ៣

  

I. ebI 1
x2
5 etI x 1 3 esµIb:unµan ?
II. a nig b CacMnYnKt;viCm ¢ an . edaydwgfa a  b  13 cUrKNna a  b .
2 2

III. eK[kaer ABCD EdlRCugmanrgVas; 1 Ékta . enAelIRCug BC  eKdak;cMNuc E eday


BE  2  1 . RsaybMPøWfa AE  CaknøHbnÞat;BuHénmMu BAC .

IV. eK[cMnYn t  1 , t  3 , t  6 , t  10 , t  15 , t  21 EdlcMnYnnImYy²CaplbUkén


1 2 3 4 5 6

cMnYnKt;tKña . edaysegátKRmUEdl[ cUrsresrkenSam t . KNna t  t rYcTajrkrUbmnþn n n

sRmab;KNna t . KNna t .
n 100

V. mnusScas;mñak;eFVIkargarmYycb;kñúgry³eBl 15h . ekµgmñak;eFVkI argardEdlcb;kñúgry³eBl 20h .


k> etImnusScas;mñak; minekµgmñak;rYmKñaeFVIkargarenHcb;kñúgry³eBlb:unµanem:ag ?
x> etImnusScas;b:unµannak; nigekµgb:unµannak;rYmKñaeFVIkargarenHcb;kñgú ry³eBl 6h ?
VI. ABC CaRtIekaNsm½gS . enAelIbnøayénRCug AB  eKdak;cMNuc P ¬ A enAcenøaH P nig B ¦.

tag a CargVas;RCugénRtIekaNsm½gS ABC r CakaMrgVg;carwkkñúgRtIekaN PAC r CakaMrgVg;Edl


1 2

b:HRCugénmMu P ehIyb:HRCug BC  xageRkARtIekaN ABC . rgVg;kaM r b:H PA Rtg; D b:H CA
1

Rtg; E nigb:H PC  Rtg; H . rgVg;kaM r b:H BC  Rtg; F b:H PB  Rtg; G nigb:H PC  Rtg; I .
2

k> bgðajfa DG  HI  3a2 .


x> KNna r  r eday[Cab;Tak;TgEtnwg a .
1 2

 527 

: ០៥ ២០០៥

: ៩ ១ ៣

  

I. ebI A  1  12  14  18  161 nig B  1 12 A . etI B elIs A b:unµan ?


II. sresrcMnYn 93 CaplbUkénsV½yKuNeKal 3 .
III. karRbLgmYyman 25 sMNYr. cemøIyRtUvmYy)anBinÞú 4 ehIycemøIyxusmYy)anBinÞú 1 . sisSmñak;
eqøIy)anRKb;sMNYr ehIy)anBinÞúsrub 70 . etIsisSenaHeqøIyRtUv)anb:unµansMNYr ?
IV. enAkñúgrUbxageRkammanRtIekaNEdlmanqUt ¬ ¦ kaertUcBN’s ¬ ¦ nigkaerFMenAkNþal
BN’exµA ¬ ¦ . tag A CaépÞRkLasrubénRtIekaNqUtTaMgGs; B CaépÞRkLasrubénkaerBN’s
TaMgGs; nig C CaépÞRkLaénkaerexµAenAkNþal.
bNþacemøIyxageRkamenH etIcemøIyNaxøHCacemøIyRtwmRtUv ?
eRBaHGVI ?
k> A  B x> B  C
K> 2 A  3C X> 3B  2C .
V. RtIekaN ABC mYymanmMu A  90 nig AB  AC  1 . eKbnøay AC  eTAxag C [)an CD  CB .
o

rkRkLaépÞénRtIekaN BCD KitCaÉktaépÞRkLa .


VI. kILakrbInak;Bak;Gav Edlmanpøakelxxus²Kña . kILakrTI1 Bak;GavEdlmanpøakelx1 / kILakrTI2
Bak;GavEdlmanpøakelx2 nigkILakrTI3 Bak;GavEdlmanpøakelx3 . muneBlRbkYtkILakrTaMgbI
)anTukGavcUlKñaenAkñgú es,agEtmYy . eBlerobRbkYtkILakrTaMgbInak; lUkykGavBIkñúges,ag
mkBak; . Ca]TahrN_kILakrTI1 TI2 TI3 lUkyk)anerogKñaGavEdlmanelx 3 / 2 / 1 . ehIy
lT§plsresrCa 3 , 2 , 1 .
k> cUrsresrlT§plTaMgGs;EdlGacekItman .
x> rkRbU)abénRBwtþikarN_ A ³ {kILakrlUkyk)anGav EdlmanpøakelxminEmnCaelxrbs;xøÜn
TaMgbInak; } .

8
 528 

: ០៥ ២០០៥

: ៩ ២ ៣

  

I. rkcMnYnKt;tUcbMput C C  1 Edl C  a  b / a nig b CacMnYnKt; .


2 3

II. k> sresr 4  2 3 Carag a  b .


x> KNna 2  2 2  2 4  2 3 .
III. mFümPaKén n cMnYnKt;xusBI 0 a , a , a , ... , a  esµI 22 . ebIeKEfm 29 eTAelI n cMnYnKt;TaMg
1 2 3 n

enaH enaHmFümPaKfµIKW 23 . etIeKmancMnYnKt;TaMgGs;b:unµan ?


IV. mnusSmñak;rt;edayel,Ón 10 km/h enAelIcm¶aypøÚv 1 km rYcedIrbnþedayel,Ón 5 km/h enAelI
cm¶aypøÚv 3 km . rkel,ÓnmFümrbs;mnusSenaH elIcm¶aypøÚvTaMgGs; .
V. enAkñúgkaer ABCD EdlmanrgVas;RCug 4 cm eKKUsRtIekaNsm½gS AEF , E enAelIRCug BC  nig
F enAelIRCug CD .

k> rkrgVas;RCugRtIekaNsm½gS AEF [Cab;Tak;Tgnwg cos15 . o

x> edaydwgfa cos a  1  cos


2

2
2a
ni g cos30 
1
2
o
cUrKNna cos15 . o

K> TajrkrgVas;RCugénRtIekaNsm½gS AEF edayKittémøCacMnYnTsPaKykRtwm 101 én cm .

 529 

: ០៥ ២០០៦

: ៩ ១ ៣

  

I. smIkar x  bx  1  0 manb£smYyesµI 4 dgénm£smYyeTot . etI b esµIb:unµan ?


2

2 1

II. rktémø x EdlepÞógpÞat;smIkar x  x  2  0 .


3 3

III. KNna X  a 1b 1c 1 1 edaydwgfa a  b  4 , ab  3 nig ab  ac  bc  31 .


3

IV. KNna x  y  z edaydwgfa 2x  y  z  7 , x  2 y  z  8 nig x  y  2z  3 .


V. enAkñúgkaer ABCD eKsg;RtIekaNsm½gS AID. KNnargVas;mMu BIˆC KitCadWeRk .
VI. rkbIcMnYnKt;vCi ¢manxusKña x , y nig z edaydwgfa xy  yz  106 .

 530 

: ០៥ ២០០៦

: ៩ ២ ៣

  

KNna S  32 64 96 128 ......200


2 2 2 2 2
I. 2 2 2 2
300
.
2

II. KNnaépÞRkLaénrUbxageRkamedaydwgfa ABC nig PQR CaRtIekaNsm½gS EdlmanrgVas;RCug


3 cm ehIyRCugBIrénRtIekaNmYy kat;RCugmYyénRtIekaNmYyeTot)an 3 Ggát;b:un²Kña .
A
P Q

B C
R

III. kumar 3 nak; A , B , C manXøIerog²xøÜn . ebIsinCa A [XøI 1RKab;eTA B ehIy B [XøI 1 RKab;eTA C
enaHGñkTaMgbImancMnYnXøIesµIKña . rkcMnYnXøI Edlmñak;²man edaydwgfamñak;²manXøIticCag 10 RKab;
ehIycMnYnXøIrbs; A nig B CacMnYnbzm .
IV. enAkñúgrUbxageRkam ABCD CactuekaNBñayEkg . AD  DI , M CacMNucrt;enAelI CD .
kMNt;TItaMgén M enAelI CD edIm,I[ AM  MB manRbEvgtUcbMput .
B C

M
A D I

V. k> sresrcMnYnbzmEdltUcCag 100 .


x> 5 cMnYnbzmtUcCag 100 dUcCa 2 , 41 , 53 , 67 , 89 RtUv)ansresredayeRbIRKb;elxBI 1 dl; 9
ehIyelxnImYy² eRbIEtmþg. cUrrksMNMuén 5 cMnYnbzmdUcenHepSgeTot[)an 5 sMNMu .

8
 531 

: ១៩ ២០០៧

: ៩ ១ ៣

  

I. bursmñak;RtUvkarpwkTwkmþg 700 cm b:uEnþKat;mankaTwkEtBIrKW kamYymancMNuH 500 cm nigka


3 3

mYyeTotmancMNuH 300 cm . etIbursenaHeFVIdUcemþc eTIbGacpwkTwk[Gs; 700 cm .


3 3

II. cUrbMeBjcenøaHenAkñúgRbmaNviFIxageRkam edayeRbIelxxus²KñaBI 1 dl; 5 ³


2  .....  .....   .....   ......   6 .
III. ]bmafa a  a  1 . cUrKNnatémøénkenSam A  a  2a  4a  3a  2 .
2 4 3 2

IV. enAkñúgkareFVIEtsþKNitviTüaelIkTI2 sisSmñak;)anBinÞúelIsBinÞúEdl)ankalBIeFIVEtsþelIkTI1 cMnYn


16 BinÞú. sisSenaHsgÇwmfaxøÜnnwg)anBinÞú 88 kñúgkareFVIEtsþelIkTI3 ehIyBinÞúenHnwgdMeLIgmFümPaK

énBinÞúTaMgbIelIkdl; 80 BinÞú. etIsisSenaH)anBinÞúb:unµanxøH enAkñúgkareFVIEtsþelIkTI2 nigenAelIkTI2 ?


V. rkRKb;KUéncMnYnKt;rWuLaTIb x , y  EdlepÞógpÞat; 1x  1y  13 .
VI. kñúgrgg;p©it O kaM R eKKUsGgát;FñÚ AB Edl AB  R rYcKUsGgát;FñÚ AC  Edl BAˆ C  45 . o

k> KNnargVas;mMu ACˆ B nig ABˆ C .


x> KNna BC nig AC CaGnuKmn_én R . ¬eK[ cos 45  22 nig cos 30  23 ¦
o o

 532 

: ១៩ ២០០៧

: ៩( ២ ៣

  

I. eday[témøeTAcMnYnKt; n BI 0 , 1 , 2 , 3 , ...
cUrbBa¢ak;témø n EdlnaM[vismPaB 2  n Bit .
n 2

II. []TahrN_témø X mYy ¬ X CacMnYnBitEdlnaM[smPaB X 1  X 1 Bit¦ nig[témø X mYy


EdlsmPaBenHminBit . etItémø X NaxøH EdleFVI[smPaBenHBit ?
III. rkBIrcMnYnKt;viC¢man a nig b EdlmanplbUkesµInwg 92 nig a 1 CaBhuKuNén b .
IV. cUrrkcMnYnKt;minGviC¢mant²Kña EdlmanplbUkesµInwg 10 . ¬ GacCaplbUkén 2 cMnYnKt; 3 cMnYnKt;
4 cMnYnKt; >>> ¦.

V. ABC CaRtIekaNsamBaØ EdlmanmMu A CamMuTal . eKKUskm<s;  AH  nigemdüan AO

¬ H enAcenøaH B nig O ¦. eK[ HAˆ O  ACˆB   nig BC  2a . KNna AH nig AO


CaGnuKmn_én a nig  .

 533 

: ០៦ ២០០៨

: ៩ ១ ៣

  

I. KNnakenSam A eday[lT§plCabIktþa ³ A  12  2823  6 .


II. cUrbMEbkRbPaK 3031
CaplbUkénbIRbPaK EdlRbPaKnImyY ²manPaKEbg CacMnYnbzm .
III. eK[smIkar ax  bx  c  0 a  0 Edlmanb¤sBIrepSgKñaKW x nig x .
2
1 2

k> bgðajfa 2ax  b  0 nig 2ax  b  0 .


1 2

x> KNna y  2axx  b  2axx  b .


1 2

1 2

IV. yuvtInak;manR)ak; 2500 erol ykeTATijEtmbiTsMbuRt[)anBIrRbePT EdlRbePTTI1 mantémø


300 erol nigRbePTTI2 mantémø 500 erol. etIyuvtIenaHTijEtm)anb:unµansnøwk ? etIkñúgenaHman

RbePTEtm 300 erol b:unµansnøwk ? nigRbePTEtm 500 erol b:unµansnøkw ?


V. eK[kaer ABCD. eKbnøayRCug AB eTAxag B [)an BE  AB . M CacMNucenAkñúgkaereday
MAC  MCD  x .

k> KNnargVas;mMu AMˆ C KitCadWeRk .


x> bgðajfactuekaN AMCE CactuekaNcarwkñúgrgVg;mYy. bBa¢ak;p©iténrgVg;enH ?
K> bgðajfa MC  CaknøHbnÞat;BuHmMu EdlpÁúMedaybnÞat;  AM  nig ME  .

 534 

: ០៦ ២០០៨

: ៩ ២ ៣

  

I. eK[BIrcMnYnBit a nig b Edl 2  a  5 nig 3  b  7 . cUrkMNt;témøFMbpM uténRbPaK ba .


II. mnusSmYyRkummanKña 13 nak;. cUrBnül;famanmnusS 2 nak;y:agtic ekItenAkñúgExEtmYy .
III. cMeBaHcMnYnBitxusKña a , b , c cUrKNna ³ A  a  baa  c  b  abb  c  c  a c c  b .
IV. sresrcMnYnEdlmanelxbIxÞg; edayeRbInitþsBaØa abc ehIy abc  100 a  10b  c ¬]TahrN_
432  100 4 10  3  2 ¦ . cUrrk a nig b edaydwgfa 5ab  500  500  ab5 .

V. RtIekaNmYymanrgVas;RCug a , b , c EdlepÞógpÞat; 24a  18b  12c .


k> KNnargVas;RCugénRtIekaN edaydwgfa b  c  10 cm .
x> KNnargVas;km<s;énRtIekaNEdlRtUvnwgRCug a .
VI. cUrKNnaépÞRkLaénctuekaN ABCD EdlmanrUbdUcxageRkam ³
C

B 10cm
5cm
8cm
4cm
A D
H K

8
14cm

 535 

: ០១ ២០០៩

: ៩ ១ ៣

  

I. etImancMnYnKt; n b:unµanxøH EdlepÞógpÞat; 72  13n  54 .


24
E B
II. RtIekaN ABC RtUv)ankat;ecjBIRkdasragCactuekaNEkg
12
dUcrUbxagsþaM . etIRkdasEdlenAsl;manb:unµanPaK ?
A
III. 1> ebI x 1 2  2 etI x 1 3 esµIb:unµan ? C 10 D

2> ebI x 1 3  3 etI x 1 4 esµIb:unµan ?


3> Tajrk x  1n  1 ebIeKdwgfa x 1 n  n .
A
IV. r)arEdkRtg;mYymanRbEvg 25m RtUv)aneKdak;Ep¥kcugxagelI 25 m
C

eTAelICBa¢aMgQrmYy Edlcm¶ayBICBa¢aMeTAcugr)arxageRkam
manRbEvg 20 m . ebIeKbgçitcugr)arxageRkamecjBICBa¢aMg
O
Efm 4 m eTot etIcugr)ar)anFøak;cuHcMnYnb:nu µan m ?¬emIlrUb¦ D 4 m B 20 m

V. Rcvak;EdlmankgmUlb:un²Kña 2 manRbEvg 12 cm . Rcvak;EdlmankgmUlb:un²Kña 5 ¬b:nu nwgkgRcvak;


mun¦ manRbEvg 27 cm . etIRcvak;kgmUlb:un²Kña 40 ¬b:unnwgkgRcvak;mun¦ manRbEvgb:unµan ?

12 cm 27 cm

VI. RtIekaN ABC mYyEkgRtg; A ehIymanbrimaRt 60 cm nigépÞRkLa 120 cm . 2

cUrrkrgVas;RCugnImYy²rbs;RtIekaN ABC .

 536 

: ០១ ២០០៩

: ៩ ២ ៣

  

I. bgðajfaRtIekaNEdlmanrgVas;mMu  ; 2 ; 3 CaRtIekaNEkg .
II. cUrrktémø x nig y enAkñúgrUbxageRkamenH ³
40 o
o
40
o yo
x 3x o

80 o

III. eKdwgfa f x f x  1  9 nig f 3  81 . cUrrk f 9 .


IV. ABC CaRtIekaNEkgRtg; A . eKKUskm<s;  AH  énRtIekaN ABC rYcKUsemdüan  AM  én

RtIekaN AHC . rkRbEvg AM ebIeKdwgfa AB  5cm nig AH  4cm .


V. ExSBYrmYymanRbEvgbeNþay 15 m nigTTwg 5 cm .
eKykExSBYrenaHmkrMu[)anCafasmYy dUcrUbxagsþaM.
etIeKGac)anfasEdlmanGgát;p©itRbEhlb:unµan cm ?
VI. x; y nig z CabIcMnYnKt;viC¢mantUcCag 9 . rkRKb;cemøIyrbs;smIkar x  y  z  20 .

 537 

: ០១ ២០១០

: ៩ ១ ៣

  
I. eK)ansaksYrsisSmkRbLgsisSBUEkcMnYn 240 nak; GMBImeFüa)ayeFVIdMeNIrmkkan;raCFanIPñMeBj.
GMBImeFüa)ayeFVIdMeNIrTaMgenaHrYmman ³ A: rfynþRkug B: rfynþtak;sIu
B  135 o

C: rfynþpÞal;xøÜn nig D: eTacRkyanynþ . RkabpøitenHbgðajBIlT§pl D  45o


C  60 o
A  120 o

énkarsaksYr ¬dUcrUb¦. cUrrkcMnYnsisSEdleFVIdMeNIrtammeFüa)aynImYy² .


II. sisSmYyRkum)anRBmeRBógcUlluyKñaedIm,IeFVIBiFICb;elogmYy. luyEdlRbmUl)ansrub 770 000 erol
edaydwgfasisSRbusmñak;RtUvbg; 30 000 erol nigsisSRsImñak;RtUvbg; 20 000 erol.
cUrkcMnYnsisSRbus nigcMnYnsisSRsIénRkumenH .
III. cUrrkcMnYnKt; x Edl 700  x  800 edaydwgfaplbUkRKb;xÞg;én x esµI 17 nigebIeKbþÚrelxxÞg;ryeTA
xagsþaMéncMnYnenaH eK)ancMnYnfµImYyticCagcMnYnedImc 441 .
 1  1  1   1 
IV. cUrKNnakenSam ³ A  1  1  1    ...  1  
 2  3  4   n 1
B  0.001   0.02   0.0001  10 10 .
2 3 2 2 5

3x  7 y  m
V. 1> eK[RbB½n§smIkar  . cUrkMNt;témø m edIm,I[RbB½n§smIkarmanKUcemøIyviC¢man.
2 x  5 y  20
2> cUredaHRsaysmIkar a b x  b x  a x  a b  0 ¬ x CaGBaØat ehIy a , b  0 ¦ .
2 2 4 4 2 4 2 2 2

3> eK[smIkar 2x  1  5  ax . cUrrktémø a edIm,I[smIkarmanb£stUcCag b¤esµI 1 .


VI. TIFøamYymanépÞRkLaesµI 864 m eKEckTIFøaenHCabIcMENk. eKdwgfacMENkGñkTIbI esµInwgplbUk
2

cMENkGñkTImYy nigcMENkGñkTIBIr. pleFobcMENkGñkTImYy nigcMENkGñkTIBIresµInwg 11/ 5 .


rképÞRkLaéncMENknImYy² .
VII. eK[RtIekaNsm½gS ABC EdlRCugmanrgVas;esµI 2a . I CacMNuckNþal AB  .
enAxageRkARtIekaN ABC sg;knøHrgVg;BIrEdlman AI nig BI  CaGgát;p©itrYcsg; A B

FñÚ énrgVg;Edlmanp©it A kaM AB EdlFñÚénrgVg;p©it A kaM AB x½NÐedaycMNuc B nig


C nigsg;FñÚénrgVg;Edlmanp©t i B kaM AB Edlx½NÐedaycMNuc A nig C . eK)an C
rUbmYy ragebHdUg ¬dUcrUbxagsþaM ¦. KNnaépÞRkLaénrUbebHdUgenH .
VIII. eK[RtIekaNEkg ABC EkgRtg; A EdlRCugénmMuEkgmanrgVas;esµI b nig c . AI  CaknøHbnÞat;BuHénmMu
BAC . cUrKNnargVas; AD .

8
 538 

: ០១ ២០១០

: ៩ ២ ៣

  

I. 1> cUrKNnakenSam ³ A  40 2  57  40 2  57 .
2> eK[smIkar x  2ax  k  0 manb£smYyesµI a  b  0 . cUrkMNt;témø k .
2

2 2 2
3> cUredaHRsayRbB½n§smIkar EdlmansmIkarnImYy²KW ³ 12xx  y ; 12yy  z ; 12zz  x .
2 2 2

II. eK[RtIekaN ABC Edlman AB  c ; AC  b nig BC  a . D ; M nig N CaeCIgénknøHbnÞat;BuH


mMukñúgTaMgbIrbs;RtIekaN edaydwgfa AD  x ; BM  y nig CN  z .
cUrRsaybMPøWfa 1x  1y  1z  1a  b1  1c .
III. f x  CaGnuKmn_kMNt;cMeBaHRKb;cMnYnBit x nigepÞógpÞat; xf  x  2  x  9 f  x  .
2

cUrRsaybBa¢ak;fasmIkar f x  0 manb£sbIy:agtic .


IV. cUrrkcMnYnEdlRtUvdak;bnþenAral;esrILÚsIuk kñúgcMeNamcemøIyEdl)anesñIeLIgTaMgR)aM .
]TahrN_ ³ 3 6 12 24 ? k> 72 x> 18 K> 37 X> 48 g> 11
cemøIyRtwmRtUvKW X> 48 ¬eRBaHcMnYnEdl)anRtUvKuNnwg 2 ral;elIk¦ .
1> 1 5 9 13 ? k> 15 x> 16 K> 17 X> 18 g> 19 .
2> 5 2 4 1 3 ? k> 0 x> 1 K> 2 X> 3 g> 4 .
3> 6 9 13 18 ? k> 23 x> 24 K> 25 X> 26 g> 27 .
4> 18 54 45 135 126 ? k> 136 x> 257 K> 378 X> 499 g> 620 .
V. P ; Q ; R nig S CacMNuckNþalerogKñaénRCug AB  ; BC  ; CD nig DA rbs;RbelLÚRkam

ABCD . cUrKNnaépÞRkLaénctuekaNEdlpÁúMedaybnÞat;  AQ  ; BR  ; CS  nig DP  edaydwgfa

épÞRkLarbs;RbelLÚRkam ABCD esµI a . 2

VI. eK[ctuekaNBñaysm)at ABCD mankm<s; h carwkkñúgrgVg;p©it O . mMup©iténrgVg;sáat;edayRCug


eRTténctuekaNBñaymanrgVas;esµI  . cUrKNnaépÞRkLaénctuekaNBñay CaGnuKmn_én h nig  .

8
 539 

: ០១ ២០១១

: ៩ ១ ៣

  

I. 1> eKmanbnÞHelah³mYysnøwkEdlmankRmas; 3mm ehIymanm:as 264kg . cUrrképÞRkLaénbnÞH


elah³enHKitCa m edaydwgfam:asmaDénelah³enH 8kg/dm .
2 3

2> eKmancMnnY Kt;tKña 91 , 92 , 93 , 94 , 95 , 96 , 97 , 98 nig 99 .


cUrdak;cMnYnKt;xagelIkñúgkaertUc²énrUbxagsþaM eFVIy:agNa[)an ³
plbUkCYredknImYy² esµInwgplbUkCYrQrnImYy² esµInwgplbUkCYr
Ggát;RTUgnImYy² esµnI wg 285 .
¬smÁal; ³ cMnnY nImYy²eRbI)anEtmþg¦
II. 1> cUrKNnakenSam D  243   81  4  16  .
0. 2 0.25 0. 5 0.75

2> x nig x Cab£sénsmIkar ³ x  2m  1x  m  2m  0 . cUrbgðajfa x  x CacMnYnefr.


1 2
2 2
1 2

3> cUrrktémø m edIm,I[RbB½n§smIkar ³ 32xx  75 yy  m20 manKUcemøIyviC¢man .



III. RkumsamKÁImYymandIERsBIrkEnøgmanragCakaerdUcKña EdlmanépÞRkLaxusKña 75m ehIyplbUk 2

brimaRtdIERsTaMgBIrkEnøg manrgVas;esµI 100 m . cUrKNnaplRsUvtamkEnøgnImYy² edaydwgfa


ERsTaMgBIrkEnøg TTYlpl)an 0.35kg/m . 2

IV. cUrrkKUéncMnYnKt;rWuLaTIb x , y  EdlepÞógpÞat; x  249 xy  250 y  6033 .


2 2

V. eK[kaer ABCD EdlmanrgVas;RCugesµI a . enAkñúgkaerenH A B

sg;mYyPaKbYnénrgVg; EdlmankaMman rgVas;esµI a ehIyp©it Q


M
N
rbs;vaCakMBUlTaMgbYnrbs;kaer ABCD . P

cUrKNnabrimaRtrUbpáakUlab AMBNCPDQA Edlsg;)anenH . D C

VI. Ggát;RTUgénctuekaNBñay ABCD EckctuekaNBñayCaRtIekaN 4 . s nig s CaépÞRkLaén 1 2

RtIekaNEdlmanRCugmYyCa)at rbs;ctuekaNBñay .
cUrrképÞRkLa S énctuekaNBñay CaGnuKmn_eTAnwg s nig s . 1 2

8
 540 

: ០១ ២០១១

: ៩ ២ ៣

  

I. 1> cUrRsaybBa¢ak;fa 3 2
2 3
.
x  4 x  3 x  2 x 1
2> cUredaHRsaysmIkar   
2008 2009 2010 2011
0 .
 x y 5
  
3> edaHRsayRbB½nsmIkar  y x 6 cMeBaH
x y0 .
x  y  5

II. cUrrkcMnYnmYyEdlmanelx 4 xÞg; abca edaydwgfa abca  5c  1 . 2

III. eK[kenSam M  x  y  2 z  t cMeBaH x , y , z nig t CacMnYnKt;minGviC¢man . cUrrktémø


2 2 2 2

tUcbMputén M nigbNþatémøRtUvKñaén x , y , z nig t edaydwgfa x  y  t  21 nig


2 2 2

x  3 y  4 z  101 .
2 2 2

IV. a , b nig c CargVas;RCugrbs;RtIekaN . cUrRsaybBa¢ak;fa ³

3ab  bc  ca   a  b  c   4ab  bc  ca  .
2

V. kaermYyRCugmanrgVas;esµInwg 1 ÉktaRbEvg. tamcMNucRbsBVénGgát;RTUgkaer eKKUsbnÞat;cl½tmYy.


cUrKNnaplbUkkaeréncm¶ay BIkMBUlTaMgbYnrbs;kaer eTAbnÞat;cl½t .
VI. eK[RtIekaNsm)at ABC kMBUl A manépÞRkLaesµI S . MN Ca)atmFümRtUvnwg)at BC ehIy
O CacMNucRbsBVén MN nigkm<s;KUsecjBIkMBUl A . eKbnøay CO  kat; AB Rtg; D nigbnøay

BO  kat; AC Rtg; E . cUrrképÞRkLaénctuekaN ADOE CaGnuKmn_eTAnwg S .

 541 

: ០២ ២០១២

: ៩ ១ ៣

  

I. ¬10BinÞú¦ KNnaplbUk ³
1> A  a  a1 1   a  11 a  2   ...   a  20111 a  2012  , a0

2> B  2011 2012  2012  ...  2012  2012  2012  1  1 .


9 8 3 2

II. ¬10BinÞú¦ 1> cUrrktémøénb£skaer ³ 2012  2011 2012  2011 2012  2011 ... .
2> M nig N CaBIrcMnYnénkaerR)akdtUcCag 100 .
ebI M  N  27 cUrKNnatémøén M  N .
III. ¬10BinÞú¦ cUreRbóbeFobBIrcMnYnxageRkam ³
1> A  2013 nig B  2010 2012 2014 2016 .
4

2> P  2  3  4 nig Q  3 24 .
30 30 30 10

IV. ¬10BinÞú¦ haglk;smÖar³kILamYy )aneFVIkmµviFIlk;BiessdUcteTA ³ ral;GtifiCnTI 25 RtUvTTYl)an


rgVan;)al;mYy nigral;GtifCi nTI 35 RtUvTTYl)anrgVan;GavyWtmYy . vIr³ CaGtifiCnTImYy EdlTTYl
)anrgVan;TaMg)al; nigGavyWt.
1> etImanGtifiCncMnYnb:nu µannak; Edl)ancUlTijhagenHmunvIr³ ?
2> dar:a CaGtifiCnEdlTTYlrgVan;mun vIr³ ehIybnÞab;mkKWvIr³ .
etIdar:aCaGtifiCnTIb:unµan ? ehIyTTYl)anrgVan;GVI ?
V. ¬10BinÞú¦ kñúgkarsikSaRsavRCavsisS 8000 nak;)an[dwgfa sisSEdlcUrcitþeronKNitviTüa mancMnYn
esµInwgbIdgéncMnYnsisSEdlcUlcitþeronrUbviTüa nigmancMnYn 48 nak; cUlcitþeronTaMgBIrmuxviC¢a .
cUrrkcMnYnsisSEdlcitþeronrUbviTüa nigcMnYnsisScUlcitþeronKNitviTüa .
VI. ¬10 BinÞú¦ xYbkMeNItTI n rbs; davId nwgRbRBwtþeTAenAqñaM n énstvtSTI 21 enH. etIxYbkMeNItelIkTI
2

32 rbs;edvIdeFVIenAqñaMNa ?

VII. ¬20 BinÞú¦ ABCD CactuekaNEkgmYy Edl AB  a nig AD  b , E CacMNucmYysßitenAelIRCug CD .


RtIekaN AED manRkLaépÞesµInwg 15 énépÞRkLactuekaNe):ag ABCE . cUrKNnaRbEvgGgát; DE
[Cab;Tak;TgeTAnwg a .
 542 

VIII. ¬20 BinÞú¦ PQRS CactuekaNe):agEdlmancMNuc A , B , C nig D CacMNuckNþalerogKñaénGgát;


PQ , QR , RS nig SP ehIy M CacMNuckNþalénGgát; CD nig H CacMNucmYyenAelIGgát; AM Edl

HC = BC . bgðajfa BHM  90 .o

 543 

: ០៤ ២០១២

: ៩ ២ ៣

  
x y
I. ¬15 BinÞú¦ 1> eK[ x  3 5 nig y  3 5 . KNna x y
.
2> eRbóbeFobcMnYn n  1  n nig n  n  1 .
II. ¬15 BinÞú¦ 1> rksMNMuéncMnYnKt;viC¢manTaMgGs; (x , y , z) EdlepÞógpÞat;lkçxNÐEdl[dUcxageRkam³
  1 , x  y  z .
1 1 1
x y z
2> rkRKb;sN M MucemøIyéncMnYnKt;viCm¢ an (x , y , z) EdlbMeBjlkçxNÐ[RbB½n§smIkar ³
3x  8 y  z  27

6 x  3 y  z  33
.

¬15 BinÞú¦ 1> eK[ x  12  3  


. KNnakenSam  x   .
1 1 n
III. n
3 n  1  x2
 
1 1
x 2  x 2
2> eKman x 2
x

2
3 . KNnakenSam P 3

3
.
x x2 2

IV. ¬15 BinÞú¦ edaHRsaysmIkar nigvismIkarxageRkam ³


1> 1 x  1 x  5 .
3 3 3

2> x x 8  x  2 .
3

V. ¬10 BinÞú¦ kMNt;témø m edIm,I[bnÞat; y  mx RbsBVnwgGnuKmn_ y  x 1  x  2 Rtg;bIcMNuc .


B
A
VI. ¬10 BinÞú¦ tamrUbxagsþaM KNnaplbUkmMu ³ C
A  B  C  D  E  F  G . G
D
F
E
VII. ¬20 BinÞú¦ eK[ctuekaN ABCD carwkkñúgrgVg;p©it O .
bgðajfa AC  BD  AB  CD  AD  BC .

8
 544 

  

I. bgðajfa ³ 6  2 5  13  48  3  1 .
3  x 2  x x 1
II. edaHRsaysmIkar ³ 1997  
1998 1999
 3 .

III. bIcMnYn x , y , z smamaRtnwg 5 , 7 , 11 .


1> KNnacMnYnTaMgbIenH kalNa ³
k> x  2 y  3z  48
x> x  my  z  72 ¬ m CacMnYnKt;EdleK[¦
rktémø m minRtUvman .
2> kñúglT§plénsMNYr 1> ¬x¦ eKcg;[ x , y , z CacMnYnKt;viC¢man. etIcMnYn 16  7m RtUvbMeBj
lkçxNÐy:agNa ? etImantémø m b:unµanEdlbMeBjlkçxNÐenH .
3> eKcg;[ m CacMnYnKt; . KNna m ¬mancemøIyeRcIn¦ nigtémørbs; x , y , z .
IV. eK[kaer ABCD manRCugRbEvg a . eKcg;sg;RtIekaNsm½gS EdlmanGgát;RTUg AC CaRCug ehIy
mankMBUl E enAxag B . O CacMNucenAkNþalén AC  .
k> bgðajfacMNuc E , B , O nig D rt;Rtg;Kña .
x> KNna BE CaGnuKmn_én a .
K> bgðajfa EB ED  AB . 2

X> BI E eKKUsbnÞat; EP  b:HrgVg;carwkeRkAkaer ABCD Rtg; P . bgðajfa EP  a .

GñkeFVIviBaØasa
eday has; sm,tþi

 545 

  

I. KNnakenSamxageRkam ³
k> Y  169   14  52    2 1 16    5  4 19
3


27 

x> Z  11 2  2 1 3  3 1 4    49 1 50  50 1 51 .
II. RsaybBa¢ak;faebI a , b , c CargVas;RCugénRtIekaNmYy enaHeK)an ³
a  b  c  2ab  bc  ca  .
2 2 2

edaHRsayRbB½n§smIkar xxy  6y  35 .
3 3
III.

IV. sYnc,arragctuekaNEkgmYy manépÞRkLa 720 m . ebIeKbEnßmbeNþay 6 m nigbnßyTTwg 4 m
2

enaHépÞRkLasYnc,arminpøas;bþÚreT . KNnavimaRténctuekaNEkgenH .
V. eK[RtIekaN ABC mYy carwkkñgú rgVg;p©it O kaM R . bnÞat;KUsBI B RsbnwgbnÞat;b:HrgVg;Rtg; A
kat;bnÞat; AC Rtg; D .
1> bgðajfa ABC~ ADB rYcbBa¢ak;fa AB  AC  AD . 2

2> bnÞat;mYyKUsBI B RsbnwgbnÞat;b:HrgVg;Rtg; C kat; AC Rtg; E nigrgVg;Rtg; F . cUreRbobeFob


ABC nig BEC .

3> bgðajfa BED nig BFC CaRtIekaNsm)at .


4> eKsnµt;fa BAC  60 . R)ab;eQµaHRtIekaN BFC rYcKNna BF CaGnuKmn_én R .
o

PñMeBj> éf¶TI 28 Ex kumÖ³ qñaM 2003


GñkeFVIviBaØasa
eday Tk; yUGUn

3
 546 

  

bM)at;r:aDIkal;BIPaKEbg .
3
11
I. 3
2m  3 12n
II. edaHRsaysmIkar ³ 32 x 2 1
 x 0
3
2
3
.
1 2
III. begáItsmIkaredaysÁal;bs£ 4
nig  1 4 2 .
  x4
IV. 1> edaHRsayRbB½n§smIkar 
2 y
.
 y  2x  7
2> sg;bnÞat;EdlmansmIkar TaMgBIrelIRkabPicEtmYy . bBa¢ak;cemøIyelIRkaPic ebI M CacMNuc
RbsBVénbnÞat;TaMgBIr .
3> sresrsmIkarénbnÞat;Edlkat;tamcMNuc P 2 , 5 nig Q4 , 2 .
bgðajfabnÞat;enHkat;tam M .
4> bnÞat;TI1 kat;G½kS xx Rtg; A ehIybnÞat;TI2 kat;G½kS yy Rtg; B . sresrsmIkarbnÞat; AB .
5> bgðajfa AM  BM rYcKNnaRkLaépÞRtIekaN AMB .
V. eK[rgVg; C O , R  nigrgVg; C O , R b:HKñaxageRkARtg; A eday OA  4 cm nig OA  2 cm .
tam A eKTaj ExSFñÚEkgKña eday AB C O , R  nig AC  C O , R .
k> bgðajfa OB // OC .
x> BC  OO  I  . bgðajfa I CacMNucnwg ehIybBa¢ak;TItaMgva edayKNna IO nig IO .
K> eKKUs OH Ekgnwg BC Rtg; H . rksMNMucMNuc H kalNa B rt;elI C O , R  .

eFVIenAERBklab> éf¶TI 10 Ex mIna qñaM 2003


GñkeFVIviBaØasa
eday hIug sm,tþi

3
 547 

  

I. dak; E CaplKuNktþa E  ab  c   ba  c   ca  b  .


2 2 2 2 2 2

II. eK[cMnYnviC¢man a , b , c , m , n , p Edl a  b  c  1 . eKdwgfacMnYn a , b nig c RcassmamaRt


erogKñaeTAnwgcMnYn m , n nig p . bgðajfaeK)an am  bn  cp  1 11 1 .
 
m n p

III. eK[kenSam A  xx  13 . kMNt;cMnYnKt; x viC¢man edIm,I[kenSam A CacMnYnKt;viCm¢ an .


IV. RsaybBa¢ak;fa smIkarxagsþaMenHmanb£sCanic© ³ x  ax  b  x  bx  c  x  cx  a  0
cMeBaH x CaGBaØat nig a , b , c CacMnYnBit .
V. RsaybMPøWfa 49  20 6  49  20 6  2 3 .
4 4

VI. eK[RtIekaN ABC EdlRCugnigkm<s;manrgVas;erogKña a , b , c nig h , h , h . 1 2 3

RsaybBa¢ak;fa a  b  c 1a  b1  1c   h  h  h  h1  h1  h1  .


1 2 3
 1 2 3 
VII. eK[RtIekaNsm)at OAB Edlman)at AB nig AOB  30 . bnÞat;Ekgnwg OA Rtg; O kat; o

bnøayén)at ABRtg; D . bnÞat;Ekgnwg OB  Rtg; B kat; OD  Rtg; E .


RsaybMPøWfa RtIekaN BED CaRtIekaNsm)at .

GUrEbkk¥m> éf¶TI 18 Ex mIna qñaM 2004


GñkeFVIviBaØasa
eday Kwm b‘unFI

 548 

  
I. cUrKUssBaØa kñúgRbGb; cMeBaHcemøIyEdlRtwmRtUv ³
eK[bIcMNuc A , B , C mankUGredaenerogKñaKW 3 , 1 ;  4 , 4 nig  2 , 5 .
k> kUGredaenéncMNuc I kNþal  AC KW ³ I  12 , 23   I  3 , 92   . 5 
I  , 3
2 

x> kUGredaencMNuc D qøúHénc B eFobnwg I KW ³ D 3 ,  2  D 1 , 2  I  2 , 1 .


K> tamrUbxageRkamenH eK[  d  //  d  . KNnatémø x ³
1 2

10

12
x 6 cemøIy ³ x  66  x  36  x  11 .
x

II. k> KNna A  16 16 Edl x CacMnYnkt;viC¢man .


4 x 1 x x

x> edaHRsaysmIkar x  x  2 1  A .
K> rktémø a EdlnaM[bnÞat; y   a  2 x  3 EkgnwgbnÞat; y  12 x  1 . rYcsg;bnÞat;TaMgBIrenH
kñúgtRmúyGrtUNremEtmYy .
X> rktémø a nig b EdlnaM[RbB½n§smIkarxageRkam manKUcemøIyeRcInrab;minGs; ³
6 x  y  30
ax  by  10 
1>  2>  1
 b  1 x  y  b  a
.
4 x  3 y  5 
 3
III. eK[BIrcMnYn tUc b¤FMCagKñabIÉkta ehIyplbUkkaeréncMnYnTaMgBIrenH 89 . rkcMnYnTaMgBIrenH .
IV. eK[RtIekaN ABC carwkkñúgrgVg; . knøHbnÞat;BuHén BAC kat;RCug  BC  Rtg; I nigFñÚ BC Rtg; J .
1> RsaybMPøWfa ABI AJC rYcbBa¢ak;fa AI  AJ  AB  AC .
2> Bgðajfa BAJ  CBJ .
3> RsaybMPøWfa JB  JA  IJ . 2

PñMeBj> éf¶TI 28 Ex kumÖ³ qñaM 2003


GñkeFVIviBaØasa ³ R)ak; kaNaDI

3  549 

  
I. RsaybBa¢ak;fa ³
2

k> 11a aa  a   11 aa  1 .


  
x> 2 3  2 3  6 .
 
 
  2  x  x  1 
2

II. KNnatémøénkenSam ³ A
1
2

1
2
 
  
 .
  2 x 1   
2 x 1    15 x 1 
 1    1   
  3   3  

III. KNnatémø x ; y Edl y  0 BIsmIkar ³ x2  4x  y  6 y  13  0 .


 x2  2 y  1  0
 2
IV. eK[ x ; y ; z Edl  y  2z 1  0 . KNnakenSam ³ A  x 2000  y 2000  z 2000 .
z2  2x 1  0

V. eK[RtIekaN ABC mYyman  AM  Caemdüan nigkm<s;  AH  .
2
RsaybBa¢ak;fa AB  AC  2 AM  BC2 .
2 2 2

VI. RsaybBa¢ak;fa TIRbCMuTm¶n; p©itrgVg;carwkeRkARtIekaN nigGrtUsg;


énRtIekaNmYyCabIcMNucrt;Rtg;Kña.

 550 

  
I. BICKNit
1> edaHRsaysmIkar ³ 4x  3x  x .
4 3 2

2> eK[ M 1 ,  2 CacMNuckNþalénGgát; AB Edl A 2 , 2 .


KNnakUGredaenéncMNuc B .
3> KNna A  x 1 x  x 1 x  x2x 1 .
2 2 2

xa xa
2
4> sRmÜlkenSam ³ F xa
xa
xa .
1
xa
5> eTscrN_mñak;ecjdMeNIrenAem:ag 7 BIeCIgPñMeTAkMBUlPñMkñúgel,ÓnmFüm 300 m/h . luHcuHBI
elIPñMmkvijedayel,ÓnmFüm 450 m/h mkdl;eCIgPñMvijenAkEnøgecjdMeNIr. enAevlaem:ag
18h 45mn . kñúgry³eBleFVIdMeNIr eTscrN_enaHsRmakGs; 4h 15mn . rkcm¶aypøÚvBI

eCIgPñMeTAkMBUlPñM .
 2 x  y 2 x  y  x 24
    1
6> edaHRsayRbB½n§smIkar ³  2 x  y 2 x  y  2 x  y 11 .
3 y  2 x  1
  2
II. FrNImaRt
1> CD Cakm<s;énRtIekaNEkg ABC RtUvnwgGIbu :Uetnus. ebI AC  6 , AD  3 , BD  x .
KNna x .
2> eK[rgVg;p©it O BIcMNuc A mYyelIrgVg; eKKUsGgát;FñÚBrI  AB ,  AC enAmçagénGgát;p©itKUs
ecjBI A . bnÞat;b:HRtg; A nig C énrgVg;Rtg; S . Ggát;RTUgmYyEkgnwg  AB kat;  BC 
Rtg; T RsaybMPøWfa ³
k> SAT nig TAB sm)at ehIymMu)atrbs;vamanrgVas;esµIKña .
x> R)aMcMNuc A , S , C , T nig O sßitenAelIrgVg;EtmYy .
K> ST  //  AB .


3551 

  
I. KNna A  19781979 1979 1979  ... 1980 1 .
9 8 7

II. eK[ x  3  2  3  2 KNna f  x  x  3x .


3 3 3

III. eK[smIkar  x  a  b x  b  c x  a  ca  b  c  abcx .


edaHRsaysmIkarebI a  2 , b  3 , c  4 .
IV. rktémøKt;én x , y nig z EdlepÞógpÞat;smIkar  x  4 y 1 z  25  80xyz .
2 2 2

edaHRsayRbB½n§smIkar ³  xy  73xx  37 yy  00 .


3

V.

3

VI. rktémøKt;én x , y nig z EdlepÞógpÞat;smIkar x  y  z  4  2 x  2  4 y  3  6 z  5 .


VII. eK[RtIekaN ABC manemdüan AM , BN , CP .

1> RsaybMPøWfa AM  BN  CP  AB  BC  AC .
2> RsaybMPøWfa AM  BN  CP  AB  BC2  AC .
VIII. eK[mMuEkg xoy enAelI ox  eKedAcMNuc B nigenAelI oy  eKedAcMNucbIKW C , D nig E Edl

OC  DE  CD  OB . RsaybMPøWfa ODB  OEB  45 . o

¬viTüal½ysn§rm:uk¦ PñMeBj> 03 mIna 2004


GñkeFVIviBaØasa
eday Ect lI

 552 

  
I. cUrKUssBaØa kñúgRbGb;xagmuxcemøIyEdlRtwmRtUv ³
1> kenSam A  53 95  13 45  5 15 mantémø ³
k>  A  33 x>  A  5 K>  A  3 5 X>  A  55 .
2> ABC CaRtIekaNEkgRtg; A Edl BC  5 3 cm nig AC  5 2 cm . AB manRbEvg ³
B
k>  AB  2 5 cm x>  AB  4 cm 5 3 cm

K>  AB  5 cm X>  AB  5 cm A C
5 2 cm
II. eRbóbeFobcMnYn 3  3 nig 6  2  3  .
III. plbUkBIrcMnYnesµI 84 ÉplEckvaesµInwg 3 . rkcMnYnTaMgBIrenaH .
 8  3x x  3
 3  4
IV. edaHRsayRbB½n§vismIkar  .
 x 1  x  2
 5
V. KNna ¬KitCa cm¦ RbEvgRCugTaMgbI x , x  2 , x  4 énRtIekaNEkgmYy .
VI. edaHRsaysmIkar ³ k>  2 x  2 35  4x  43   12  x   0
  
x> 4x  2x 1  19x  3 .
2

VII. ABC CaRtIekaNEkgRtg; A Edl BC  2a nig Bˆ  60 . o

KNnargVas;RCug AB nig AC CaGnuKmn_én a .


VIII. kñúgRtIekaN ABC eKKUskm<s;  BD nig CE . kñúgRtIekaN ADE eKKUskm<s;  DF  nig  EG  .

k> eRbóbeFob AC
AE
ni g AD
AF
rY c AB
AD
ni g AE
AG
.
x> bgðajfa AC  AF  AB  AG .
K> bgðajfa  BC // GF  . ¬GnuvTi üal½y h‘un Esn burI 100 xñg ¦

3
 553 

  
I. KNna P
1

1

1
 b  c   a  ac  b  bc   c  a  b  ab  c  ac   a  b   c  bc  a 2  ab 
2 2 2 2 2

4a 3 8a 7
E
1

1
 2
2a
 
a  b a  b a  b 2 a 4  b 4 a 8  b8
.
II. eK[smIkar 3a  3b  10ab nig b  a  0 . rktémøénRbPaK F  aa  bb .
2 2

III. cMBYyTwkbI EdlkñúgenaHmanBIrbBa©ÚlTwkdak;kñúgGag nigmYyeTotbeBa©jTwkBIGag. edIm,IbMeBjGag


cMBYyTI1 RtUvkarry³eBl 1h 30mn . cMENkÉcMByY TI2 vijvaGacbMeBjGagkñúgry³eBlminsÁal;.
edIm,IbeBa©jTwk[Gs;BIGag cMBYyTI3 RtUveRbIry³eBl 2h . ebIeKebIkcMBYyTaMgbI[hUrTwkRBmKña eK
segáteXIjfa GagenaHeBjkñúgry³eBl 40mn . etIcMBYyTI2 RtUveRbIry³eBlb:unµanedIm,IbMeBjGag?

a  b  c  1 1
 2 2 2
IV. eK[ a  b  c  1  2 . RsaybBa¢ak;fa xy  yz  xz  0 .
x y z
    3
a b c
V. eK[RtIekaN ABC carwkkñúgrgVg;p©it O sg;km<s;  AH  nigkaM  AO . Bgðajfa BAH  OAC .
VI. RsaybMPøWfa cMeNalEkgcMNuckMBUlTaMgbYn énRbelLÚRkamelIGgát;RTUgrbs;va CakMBUlRbelLÚRkam
mYyeTot .

¬TYlsVayéRB ¦

 554 

  
I. RsaybMPøWfa 4  2 3  4  2 3  2 3 .
II. KNnatémøelxénkenSam A  1 1 2  2 1 3

1
3 4
 ... 
1
99  100
.
 x 2  y 2  130
III. edaHRsayRbB½n§smIkar  .
 xy   x  y   47

IV. eKman a  b  c  0 nig ab  bc  ca   4010


2
. cUrKNna a  b  c  0 .
4 4 4

V. eK[ctuekaNBñaysm)at ABCD man  BC  B C


Ca)attUc Edl BC  16cm . Ggát;RTUg  BD
nigRCugeRTt  AB EkgKñaRtg; B .
eK[ BD  40 cm . KNna AD  x . A H O D

VI. eK[RtIekaNsm½gS ABC Edl AB  AB  BC  a ehIyman  AH  Cakm<s; . I CacMNucmYyenA


elI  AH  Edl AI  AH3 . bnÞat;Rsbnwg  AH  KUsecjBI C CYbbnÞat; BI Rtg; M .
1> KNna CM nig BM CaGnuKmn_én a .
2> KNnaRkLaépÞctuekaN AHCM CaGnuKmn_én a .

¬vi> vtþekaH ¦

 555 

  
x 1 x  2 x  3
I. KNna x Edl 1999 x
 
2000 2001 2002
 4 .

II. eRbóbeFobcMnYn 14 48 nig 7  4 3  7  4 3


4 4

III. edaHRsayRbB½n§smIkar nigBiPakSatémø m xageRkam ³


 x  my  1
 .
mx  3my  2m  3
IV. fñalsMNabmYymanragCactuekaNEkgmanRkLaépÞ 400 m nigbeNþaymanRbEvgelIsTTwg 23 m .
2

cUrrkvimaRtfñalsMNabenaH .
V. cUrsg;bnÞat; D : y  2x  5 nigeK[bnÞat;  :  m 1 x  my  m  2 , m Ca):ar:aEm:t .
k> KNnakUGredaencMNucRbsBV I rvag  D nigbnÞat;    cMeBaH m  13 .
x> KNna m edIm,I[bnÞat;  D    .
K> KNnaépÞRkLaénRtIekaN EdlpÁúMedaybnÞat;  D nigG½kSkUGredaen .
VI. cUrsresrCaRbB½n§bIvismIkar ¬kñúgrUb¦ ³ D  2
4
 D3  tbMnc;eMly
I
1 4
2 o  D1 

VII. eK[rgVg;BIr (O) nig (O’) manp©it O nig O’ mankaMRbEvg R < R’ . rgVg;TaMgBIrb:HKñaxageRkARtg; A .
eKKUsbnÞat;b:HrYmkñúg nigbnÞat;rYmeRkA Edlb:HrgVg; (O) Rtg; B nigb:HrgVg; (O’) Rtg; C . bnÞat;b:HTaMg
BIrkat;KñaRtg; M .
1> bgðajfa MA  MB  MC ehIyfaRtIekaN ABC EkgRtg; A .
2> bnÞat;  BA nigbnÞat; CA kat;rgVg; (O’) nig (O) Rtg; D nig E . bgðajfa  BE CaGgát;
p©itrgVg; (O) ehIy CD CaGgát;p©iténrgVg; (O’) . R)ab;eQµaHctuekaN BCDE .
3> bgðajfa BEA  ABC eRbóbeFobRtIekaN BCD nig BCE rYcbgðajfa BC  BE  CD . 2

4> eK[ R  4.5cm nig R  2cm . KNna BC nigRkLaépÞctuekaN BCDE .

3  556 

  
I. eK[ 2a  2b  5ab Edl b  a  0 . cUrKNnatémøelxén F  aa  bb .
2 2

2 2
II. eKman ax  by  0 . bgðajfa a a b  x x y  1 .
2 2 2 2

 x  y
2
x2  y 2
III. cUreRbóbeFobpleFob nig ebI x  y  0 .
x2  y 2  x  y
2

IV. RsaybBa¢ak;fa 3  3  2  3  4  24 Eckdac;nwgcMnYn 27 .


2005 2006 2005 2006 2005 2005

V. eK[bIcMnYnKt;rWuLaTIbtKña a , b , c Edl a  b  c .

k> KNnaplbUk S  a  b  c CaGnuKmn_én b .


x> Tajrktémø a , b , c edaydwgfa S  333 .
VI. edaHRsaysmIkar  y  2 x  6   2 y  x  4   0 .
2 2

VII. dImYykEnøgmanragCactuekaNEkgmanépÞRkLa S Edl 2hm < S < 3hm eKEckdIenaHCaLÚt_² man 2 2

épÞRkLa 150m , 180m nig 210m .


2 2 2

k> KNnaRkLaépÞéndIragCactuekaNEkgenaH .
x> KNnaRbEvgbeNþay nigTTwgctuekaNEkgenaH ebIdwgfaTTwgesµInwg 74 énbeNþay .
VIII. eK[RtIekaNEkg ABC EkgRtg; A Edl Bˆ  2Cˆ nig BC  a . enAeRkARtIekaN ABC sg;RtIekaN

sm½gS ABF nig ACM .


k> eRbóbeFobRkLaépÞénRtIekaN ABF nigRkLaépÞénRtIekaN AFM .
x> KNnaRkLaépÞctuekaN BCMF CaGnuKmn_én a .
IX. eK[RtIekaNsm)at ABC man)at  BC  . D CacMeNalEkgéncMNuc B elI  AC  .

bgðajfa BC  2 AC  CD .
2

¬vi> )ak;TUk ¦

3
 557 

  
3
I. edaHRsaysmIkar nigepÞógpÞat;cemøIy ³  x  x  4  8 .
2 4

II. RsaybBa¢ak;fa 49  20 6  49  20 6  2 3 .
4 4

1
a b 
III. KNnakenSam A ab Edl a  b  0 .
1
1
a 2  b2

eK[cMNuc A  1 , 0 ; B 1 , 0 ; C  11  xx 2x 
enAkñúgtRmúyGrtUNremEtmYy. bgðajfa
2
IV. , 

2
1  x2 

ABC CaRtIekaNEkgRtg; C .
V. ABC CaRtIekaNEkgRtg; A kMNt;edayrgVas;RCug AB  c , AC  b . knøHbnÞat;BuHkñúgénmMu A kat;

GIub:Uetnus  BC  Rtg; D . bnÞat;Rsbnwg (AD) EdlKUsecjBI B CYbbnøayénRCug CA Rtg; E .


k> RsaybMPøWfa AE  AB rYcKNna BE .
x> KNna AD .
K> KNnaépÞctuekaN ADBE .
¬Gnu> TYlGMBil ¦

 558 

  

KNnakenSam A  3273 , B  2  16  16 .
3
4 x 1 x x
I. 6

 x 18  x 21  x 15  x 12  x 555
II. edaHRsaysmIkar ³ k> 241979    
1985 1982 1988 1991 111
 0 .
x 1
x> 84  16 . x 1
x

1 1 1 1
 a  b  c   12

1 1 1 7
III. edaHRsayRbB½n§smIkar ³     .
 c b a 12
1 1 1 5
 a  c  b  12

IV. eK[bnÞat;BIrKW  D  : y   2m  5m  4 x  2m  5m  5 nig  
m
2 2
m  : y   m2  m  1 x  m2  m 1
kMNt;témø m edIm,I[bnÞat;  D  nig   RsbKña .
m m

x  0
y  0

V. edaHRsayRbB½n§vismIkar ³  tamRkaPic .
x  y  0
3 x  4 y  36

VI. rktémø x edIm,I[cm¶ayrvagBIrcMNuc A 2 , 9 nig B  x , 9 esµInwg 2004 Ékta .


VII. eK[ A 2 , 2 ; B  3 ,  3 nig C  6 , 6 . bgðajfa A , B , C sßitenAelIbnÞat;EtmYy .

VIII. ABC CaRtIekaNEkgRtg; C ehIy CH  Cakm<s;énRtIekaNenH . rgVg;Ggá;p©it  AH  nig  BH  kat;

 AC  nig  BC  Rtg; P nig M . bgðajfa (PM) CabnÞat;b:HrYménrgVg;TaMgBIrenaH .


IX. eK[kaer ABCD EdlmanRCug 12cm. tamcMNuc M enAelIGgát;  AB eKKUsrgVg;p©it M kat;tam A.

KNnargVas; AM edIm,I[rgVg;enH b:HnwgrgVg;Ggát;p©it  BC  .


X. knøHrgVg;manGgát;p©it AB = 20cm . tamcMNuc M elIknøHrgVg;enH eKKUs  MH    AB Rtg; H .

KNnargVas; AH edIm,I[ AH esµInwgBIrdg MH .


vi> b£sSIEkv> éf¶TI 09 mIna 2005
eday cab e):as‘ag

3
 559 

  
I. KNnakenSam S
1

1

1 2 2  3 3  4
1
 ... 
1
1999  2000
P
1
2x 1
.
 2 n  4 x 2  4 x  1
n

II. eK[ a , b , c CargVas;RCugTaMgbIénRtIekaNmYy. cUrbgðajfasmIkar b x  b 2 2 2


 c2  a2  x  c2  0

Kµanb£s .
3  5  13  48
III. bgðajfa X  2 6 2
CacMnYnKt;viC¢man .
 x  xy  y  1

IV. edaHRsayRbB½n§smIkar ³  y  yz  z  4 .
 z  zx  x  9

V. sisS 6 nak; A , B , C , D , E nig F RbLgsisSBUEkKNitviTüaRbcaMsala. KNemRbeyaKRbkasfa
mansisSBIrnak;)anRbLgCab;edayKat;niyayfa ³
1> A nig C Cab; 2> B nig E Cab; 3> F nig A Cab; 4> B nig F Cab; 5> D nig A Cab; .
eKdwgfa XøaTaMgR)aMmanXøamYyxussuT§saF ehIybYnXøaeTotmanmYyXøa²RtUvBak;kNþal. etIsisS
BIrnak;NaEdl)anRbLgCab; .
VI. cUrRsaybBa¢ak;fa A  11...11  22...22 CacMnYnmYymankaerR)akd .
2n tYén 1 n tYén 2
VII. manRtIekaN ABC EkgRtg; A nigkm<s;  AH  . R , R , R CargVas;kaMrgVg;carwkkñúgRtIekaN
1 2 3

ABC , AHB nig ACH . bgðajfa R + R + R = AH . 1 2 3

vi> c,arGMeBA> éf¶TI 13 mIna 2005


eday nk kn

 560 

  
I. eRbóbeFobcMnYn edaymin)ac;eRbIm:asIunKitelx ³  2000 nig 3000 . 3000 2000

II. KNnatémøelxénkenSam A   x  x  x  2x 1 cMeBaH x  1 2


4 3 2 2005

1
.

2  1 1 2 1 1
III. rktémøGb,brmaénBhuFa ³ W  a b  5a  9b  6ab  30a  45 .
2 2 2 2 2

IV. RsaybMPøWfa a  b  c  d 1  a  b  c  d RKb;témøén a , b , c , d .


2 2 2 2

eK[kenSam P  x x2x2  5 . rkRKb;témøKt;viC¢manén x edIm,I[ P mantémøelxCacMnYnKt; .


4 3
V.

 xy  64

VI. 1> edaHRsayRbB½n§smIkar ³ 1 1 1 .
x  y  4

2> KNna N  4  15  4  15  2 3  5 .
3> RsaybBa¢ak;fa RKb;cMnYnBit a , b , c enaHsmIkar x 1 a  x 1 b  c1 manb£sCanic© . 2

VII. eK[RtIekaN ABC . km<s;  AM  ,  BN  nig CP kat;KñaRtg; H .

bgðajfacMNuc H Cap©itrgVg;carwkkñúgRtIekaN MNP .


VIII. eK[RtIekaN ABC ehIy M , N , P CacMNuckNþalerogKñaénRCug  AB ,  BC  ,  AC  .

bgðajfa S  4S .
ABC MNP

vi> bwgRtEbk> éf¶TI 10 mIna 2005


eday kuy EkvLúg

 561 

  

I. 1. KNna A  12 19  6 10   3 2  2 5 , B  3  5  3  5
10 5
.
2 2  3 5 2 2  3 5
2. rkcMnYnEdlmanelxBIrxÞg; AB ^epÞógpÞat; ³ AB  BA  1980 .
2 2

II. 1. edaHRsaysmIkar 2 3  3  x 3  y 3 Edl x , y CacMnYnsniTan .


2 x  12 x  17 x  2
2. KNnatémøelxénkenSam ³ A 
3 2

x2
27
ebI a  b  a  c ,
x 13 169

 c  b  2a  b  c 
.
a  c
2

III. 1. eKman x  0 , y  0 , z  0 ehIyEdl x  y  z  xyz , x  xy . RsaybBa¢ak;fa x  3 .2 2

2. smIkar a 1  x   2 2bx  c 1  x   0 Edl a , b , c CaRCugénRtIekaNEkg ABC Edl Cˆ  90 ³


2 2 o

k> RsaybBa¢ak;fa smIkarmanb£sBIr x , x . 1 2

x> ebI x  x  12 cUrkMNt;témøelxén M  a b c .


2
1
2
2

3x  2 xy  y  0
2 2

IV. 1. edaHRsayRbB½n§smIkar  2 .
 x  5 y  6
2. enAkñúgRtIekaN ABC man Bˆ  120 , BA  8 cm , BC  12 cm . cMNuc P rt;elI  AB edayepþIm
o

BIcMNcu A eTA B nigcMNuc Q rt;elI  BC  ecjBI C eTA B . ebI P rt;kñúgel,Ón 1cm/s ehIy Q rt;
kñgú el,Ón 2cm/s ecjdMeNIrkñúgeBlEtmYy.
k> etIb:unµanvinaTIeRkay eTIbépÞRkLaRtIekaN PBQ esµInwgBak;kNþalépÞRkLaRtIekaN ABC ?
x> kñúgeBlCamYyKñaenaH etI  PQ manRbEvgb:unµan ?
V. 1. ctuekaNesµI ABCD EdlmanRCugesµI a . R nig r CakaMénrgVg;carwkeRkARtIekaN ABD nigRtIekaN

ABC . RsaybBa¢ak;fa .
1 1 4
 
2 2 2
R r a
2. kñúgRtIekaN ABC man BAC  60 .  AD  CaknøHbnÞat;BuHmMu BAC  60 kat;  BC  Rtg; D .
o o

ebI SS  85 ehIyrgVg;p©it I carwkkñúgRtIekaN ABC manépÞesµI 12 .  AB ,  BC  nig  AC 


ABC

ADC

b:HrgVg;p©it I Rtg;cMNuc E , N , M . KNnaRbEvg AD ?

3
 562 

  
I. rktémø x nig y edaydwgfa x  y  120 nig PGCD  x , y   30 .
II. eK[ ba  dc bgðajfa 44ba 33dc  xbxa  ydyc rYcrktémø x nig y .
III. eKmanbIcMnYn a  0 , b  0 , c  0 Edl a  b  c  1 nig 1a  b1  1c  0 .
RsaybMPøWfa a  b  c  1 .
2 2 2

IV. edaHRsay nigBiPakSa ³ ax  2x  1  0 , a Cab)a:r:aEm:t .


2

V. ebkçCnRbLgmñak;cab;eqñat 3 snøwkkñúgsMNYr 22 EdlerobcMeLIgedayKN³emRbeyaK. kñúgsMNYr 22


enHman ³ 10 sMNYrBICKNit , 7 sMNYrRtIekaNmaRt , 5 sMNYrnBVnþsaRsþ .
k> KNnaRb)ab edIm,I[sMNYrTaMgbICasMNYrBICKNit .
x> KNnaRbU)ab edIm,I[sMnYrTaMgbI CaBIKNit RtIekaNmaRt nignBVnþ .
 xy  64

VI. 1> edaHRsayRbB½n§smIkar ³ 1 1 1 .
x  y  4

2> KNna N  4  15  4  15  2 3  5 .
3> RsaybBa¢ak;fa RKb;cMnYnBit a , b , c enaHsmIkar x 1 a  x 1 b  c1 manb£sCanic© .
2

VII. eK[RtIekaN ABC EkgRtg; A nig  AH  Cakm<s; Edl  AC  a , ACB  30 . o

k> KNna CH , BH , ABH .


x> R)ab;eQµaHRtIekaN IAC ( I CacMNuckNþal  BC  ) .
K> rgVg;Ggát;p©it  AB nigrgVg;Ggát;p©it  AC kat;KñaRtg;cMNcu mYyeTot M .
RsaybMPøWfa M RtYtelI H enAelI  BC  .
Gnu> BgTwk> éf¶TI 13 mIna 2005
eday RkUc suvNÑara

3
 563 

  
I. rktémøKt; x , y , z EdlepÞógpÞat;smIkar  x  4 y 1 z  25  80xyz .
2 2 2

II. eK[ a , b , c CaRCugénRtIekaN ABC . KNnaRkLaépÞénRtIekaN ABC edaydwgfa ³


.
2 2 2
2a 2b 2c
b , c ,a
1 a 2
1 b 2
1 c 2

III. edaHRsaysmIkar xx 11  xx  44  xx  22  xx  33 .


IV. eK[ x  y  x  z  y  z . RsaybBa¢ak;fa 1x  1y  1z  0 .
 x 2  y 2  z 2  xy  yz  xz
V. edaHRsayRbB½n§smIkar  2005 .
 x  y  z  3
2005 2005 2006

VI. KNnatémøtUcbMputénBhuFa 2x  4xy  5 y 12 y  13 .


2 2

VII. edaHRsaysmIkar 2 1 x 3 y
3 2 .2 x  4 y 1

VIII. eK[RtIekaN ABC EkgRtg; A nigman  AD Cakm<s; . RsaybBa¢ak;fa AD  BC  AB  AC .

IX. eK[RtIekaN ABC man Aˆ  2Bˆ  4Cˆ . RsaybBa¢ak;fa .


1 1 1
 
AB BC AC
X. eK[RtIekaN ABC sm)atkMBUl A Edl Aˆ  20 . RsaybBa¢ak;fa AB  BC  3 AB  BC .
o 3 3 2

vi> sn§rm:uk> éf¶TI 08 mIna 2005


eday Ect lI

 564 

  
I. eK[ a  b  c  1 , a  b  c  1 , ax  by  cz  m . cUrKNna
2 2 2
P  xy  yz  zx .
II. KNna A  1  12 1  13 1  14   ... 1  991 
B  6  4 x 1 81x  2 81
x

C  4 74 3  2 3
S   x  1   x  2   x  3  ...   x  100 rYcbBa¢ak;BItémøelx S ebI x  1 , 2 , 3 .
III. edaHRsaysmIkar ³ k> 2 x  5  3x  5  2 .
x> 2x  x 2005
2  2 x 2004  2
2 x2 3 x
 x  1  x  x2003  x2002 
1
K> 3 9  x 2003
 .
 x
x y z t
 x  xy  y  2  3 2  2  3  4  6
IV. edaHRsayRbB½n§smIkar ³ k>  2 x>  zyt xzt xyt xyz .
 x  y  6
2
     14625
 x y z t

V. eK[smIkarbnÞat; y  32m mx
1
.
k> sresrsmIkarbnÞat;Edlkat;tamcMNuc A0 , 1 .
x> K CacMNucEdlsßitelIbnÞat;tagRkaPicRKb; m . KNnakUGredaenén K’ qøúHnwg K eFobnwg A .
VI. ctuekaNEkg ABCD EdlTTwg AB  a tamcMNuckNþalén  AB eKKUsmMu KOH  90 EdlcMNuc o

H enAelI  AD nigcMNuc K enAelI  BC  .

k> eRbóbeFobRtIekaN AOH nigRtIekaN OBK rYcKNna AH × BK .


x> eK[ AH = x nig BK = y KNnaépÞGb,brmaénRtIekaN KOH .

Gnu> Twkl¥k;> éf¶TI 01 mIna 2005


eday hYt sIN u n

3
 565 

  

I. KNnakenSam ³ A  2  2  3 7  2  3 6  7  2  3 6  7  2 .
2 
5 3 x 1
II. edaHRsaysmIkar ³ k> 5  6 2 4
5 5
x
8
x> 3
8 x 1
 4x .
4
 1
III. KNnakenSam 4 1
x  4 ebI  x    1296 nig x
4 .
y  y y
IV. RsaybBa¢ak;fa 52003  52004  52005 Eckdac;nwg 31 .
1 1 1
x  y  z  4  xy  18

V. edaHRsayRbB½n§smIkar ³ k>  x>  2 .
 x  y  24
2
 2  1 8
 xy z 2

VI. KNnacm¶ayrvagcMNuc A6 , 6 eTAbnÞat; D : y  x  4 kñúgtRmúyGrtUNrem .


VII. eKmansmIkar x  mx  4  0 KNnatémø m nigb£smYyeTot edaysÁal;b£sesµI 2 .
2

VIII. tamrUbxagsþaM  AB //  EF  //  DC . B


E z
RsaybBa¢ak;fa 1x  1y  1z . y
x
A F C

IX. eK[rgVg; C(O , R) nigGgát;p©it  AB nigcMNuc M rbs;rgVg; C ehIybnÞat; D b:HrgVg; C Rtg;cMNcu A .
P CacMeNalEkgén M eTAelI (AB) ehIy Q CacMeNalEkgén M eTAelI (PQ) .

k> bgðajfa AIO CaRtIekaNEkg .


x> bnÞat;Edlb:HrgVg;Rtg; M kat;bnÞat; D Rtg; T . bgðajfa AM CaknøHbnÞat;BuHénmMu QMO nig TMP .
K> bgðajfa AIO ATM ehIy AIP AOM .
X> KNna AQ , AI , AP ebI AT = x , OA = R .
vi> b‘unr:anI h‘un Esn vtþPñM> éf¶TI 03 mIna 2005
eday Eg:t can;sux

3  566 

  
I. rktémø x edIm,I[ x x x
   
x
5 10 15 20
1 1 1
 1   
2 4 5
.
sRmYl xx 11 .
2
II.

III. rkRKb;témø m Edl m  0 edIm,I[smIkar  m  2 x  m  5  0 manb£sviC¢man .


IV. RsaybMPøWsmPaB 2  3  2  3  2 .
2  2 3 2  2 3
V. GtþBlkrBIrnak;rt;edayclnaesµIenAelIpøvÚ mYymanragCargVg; Edlmanrg; 480 m. ebIGtþBlkrTaMgBIr
rt;tamTisedApÞúyKña enaHGñkTaMgBIrCYbKña 45 vinaTImþg. ebIGtþBlkrTaMgBIrrt;tamTisedAEtmYy enaH
Gñkrt;elOnECgGñkrt;yWt ral;bInaTI.
k> KNnael,ÓnénGtþBlkrTaMgBIr .
x> ]bmafa A CacMNucecjdMeNIr etIGtþBlkrmñak;² rt;b:unµanCMu)anCYbKñaRtg; A .
VI. eK[RtIekaN ABC mYycarwkkñúgrgVg; (C) Edl Bˆ  Cˆ  90 .
o

k> bgðajfa Ggát;p©it AA’= 2R énrgVg; (C) Rsbnwg  BC  .


x> bgðajfa AB  AC  4R .
2 2 2

K> eK[  AH  Cakm<s;KUsecjBIkMBUl A . RsaybMPøWfa AH  HB  HC .


2

b£sSIEkv> éf¶TI 28 kumÖ³ 2005


eday pn sar:at

 567 

  
1
2000
3 9
I. 1> KNna A 
4
 
 16 
1
. 1
1
2

2> cUrkMNt;TtI aMgvg;Rkck edIm,I[)ansmPaB ³ B  8  4  2  2  9  3  3  3  4  0 .


II. 1> cUrdak;kenSam A CaplKuNktþa Edl A  a b  c   b  a  c   c  a  b  .
2 2 2 2 2 2

2> bgðajfa ³ ebi a  b  c enaH a 2bab c  b 2cbc a  0 Edl a  0 , b  0 , c  0 .


2 2 2 2 2 2

x 1 x  2 x  3
III. 1> edaHRsaysmIkar ³ 2004 x
  
2005 2006 2007
4 .

 x  3  2 y 1  2 1
2> edaHRsaysmIkar  .
2 x  3  y  1  4  2
IV. 1> cUrrkbIcMnnY Kt;esstKña edaydwgfa plbUkcMnYnTaMgbIesµInwg 909 .
2> cUrrkbYncMnYnKt;KU edaydwgfaplbUkcMnYnTaMgbYnesµInwg 1028 .
V. eKmanrgVg;p©ti O mYycarwkeRkARtIekaNsm)at ABC kMBUl B nigkm<s;  AP Edlbnøayvakat;rgVg;
Rtg; K . eKbnøaykaM  AO kat;rgVg;Rtg; F rYcKUs OM    BC Edlbnøayvakat;rgVg;Rtg; E . H
CaGrtUsg;énRtIekaN .
k> cUrRsaybBa¢ak;fa BCK  CBF  BCH nig FAE  EAK .
x> bgðajfa FK  2MP .
K> cUrRsaybMPøWfabIcMNuc F , M , H sßitenAelIbnÞat;EtmYy .
X> cUreRbobeFob OM nig AH .

GUEbkk¥m> éf¶TI 13 mIna 2005


eday Xun Bisidæ

3
 568 

  

I. bgðajfa ³ 6  2 5  13  48  3 1 .
3  x 2  x x 1
II. edaHRsaysmIkar 2002  
2003 2004
 3 .

III. eK[bIcMnYn a , b , x epÞógpÞat;tMnak;TMngTaMgbIxageRkam ³


a  2b  1  0 , b  2 x  1  0 , x  2a  1  0 .
2 2 2

cUrKNnaplbUk S  a  b  x .
2005 2005 2005

IV. eK[smIkar  x 1 x  4 x  5 x  6  m .


k> kMNt;témø m edIm,I[smIkarKµanb£s .
x> kMNt;témø m edIm,I[smIkarmanb£sDub .
K> edaHRsaysmIkarebI m = 8 .
V. kñúgtRmúyGrtUNrem mancMNuc A 3 , 2 ; B 9 , 2 nig C 3 , 10 . KNnabrimaRt nigRkLaépÞén
RtIekaN ABC .
VI. eK[bnÞat; D : 2  3x  y  0 nig D : 4  m  1 m x  my  0 . rktémø m EdlnaM[ ³
1 2

k> bnÞat; D nig D kat;KñaelIGk½ SGab;sIus .


1 2

x> bnÞat; D nig D EkgKña .


1 2

VII. RCugénkaer ABCD manrgVas;esµI 1 . enAelIRCug  AB nigRCug  AD edAcMNuc E nig F EdlbrimaRt
RtIekaN AEF esµI 2 . RsaybBa¢ak;fa ECF  45 . o

Gnu> TYlR)asaTEsnsux> éf¶TI 09 mIna 2005


eday BuT§ v½nþ

 569 

  
I. 1> dak;CaplKuNktþadWeRkTI 1 nUvBhuFa ³ P  x  2x  5x  6 . 3 2

2> dak;CaplKuNktþadWeRkTI 2 nUveTVFa ³ A  x  64 . 4

3> bgðajfa plbUk S  39  51 Eckdac;nwg 45 .


51 39

ax  by  cz  dt  0
bx  ay  dz  ct  0

II. eKman  . bgðajfa x y  z t 0 b¤ a bc d 0 .
cx  dy  az  bt  0
dx  cy  bz  at  0

III. edaHRsayRbB½n§smIkar ³
 yz 10
1 1 5 x  3
x  y  6 
k> xx 1y y 651  18
2 2
  zx 15
x>  K>   .
  1  1  13 y 2
 x 2 y 2 36  xy 6
 
z 5
IV. 1> brimaRténRtIekaN ABC manrgVas;esµInwg 80 cm . eyIgsnµt;fa BC  a , AC  b , AB  c .
eK[ a , b , c smamaRtnwgcMnYn 5 , 7 , 4 . KNna a , b , c .
2> tamcMNuc M enAelI  BC  eKKUs ME //  AB Edl E  AC . eK[ MB  x ehIy y Ca
brimaRténctuekaN ABME . KNna y CaGnuKmn_én x .
3> sg;bnÞat; D tagGnuKmn_én y EdlrkeXIjkñúgsMNYrTI 2 .
4> KNnatémø x ebI y = 56 .
V. ABC CaRtIekaNcarwkkñúgrgVg;p©it O kaM R ehIy [AH] Cakm<s;. eKsnµt; AH  ha , AC  b ,

AB  c , BC  a . S CaRkLaépÞénRtIekaN ABC . RsaybBa¢ak;fa abc  4RS .

vi> vtþekaH ³ eday )an fn

3
 570 

  
I. sRmYlkenSam ³ A   xx  11  22xx 11    2xx11  2xx11  .
 m  1 1  m 4m2    1 1  m 
B   2   2  3  2  1  .
 1  m 1  m m 1    m  m 
2
m

II. 1> edaHRsayRbB½n§smIkar Edlman x CaGBaØat nig m Ca):ar:aEm:t ³


5x  m 2x  m m 5 5  x 
6
1 
5
 
5 20
.
2> rktémø m Kt;viC¢man edIm,I[ 0  x  10 .
III. KNnakenSam ³ A   2  3    2  3  .
2004 2005

5 2 6 52 6
  2  25 .
2
B 
52 6 52 6
IV. eK[rgVg;p©it O kaM R Ggát;p©it AB . sg;kaM OI Ekgnwg AB . J CacMNucmYyelIFñÚtUc AI . BJ kat;
OI Rtg; K ehIy AJ kat; OI Rtg; L nig AK kat;rgVg;Rtg; P .

1> RsaybMPøWfa B , P , L enAelIbnÞat;EtmYy .


2> bgðajfactuekaN AOKJ nig JKPL carwkkñúgrgVg; .
3> RbdUc BOK nig ALO rYcbgðajfa OK  OL  R . 2

4> RsaybBa¢ak;fa OJ b:HrgVg;carwkeRkActuekaN JKPL .

PñMeBj ³ éf¶TI 14 mIna 2005


eday Qwm esg

 571 

  
I. RsaybBa¢ak;fa 5  5  5 Eckdac;nwg 31 .
2001 2002 2003

II. KNnatémøénkenSam A  1 1 2  2 1 3  3 1 4  ...  99 1 100 .


III. eK[ a  0 , b  0 nig a  b  c  0 .
k> RsaybBa¢ak;fa a  b  c  3abc .
3 3 3

x> eKtag c  2n . RsaybBa¢ak;fa aa bb  cc  3n .


3 3 3

2 2 2

IV. cabmYyhVÚgehIreTATMelIpáaQUkkñúgRsHmYy. ebIcabmYyTMelIpáaQUkmYy enaHmancabmYyKµanpáaQUkTM


EtebIcabBIrTMpáaQUkmYy enaHenAsl;páaQUkmYyTMenr. rkcMnYnpáaQUk nigcMnYncab .
edaHRsayRbB½n§smIkar  xy  33xy 66  22yx 21 .
2

V.

2
 
VI. Ggát;RTUgénctuekaNBñaysm)at EckctuekaNBñayenH)anRtIekaNcMnnY 4 EdlkñúgenaHRkLaépÞ
RtIekaNBIr Cab;nwg)atmanrgVas; S nig S . KNnaRkLaépÞénctuekaNBñayxagelI .
1 2

VII. eK[RtIekaN ABC . M CacMNuckNþalén [AB] , D CacMNucmYyenAelI [MB] Edl 2MD = DB ,


MCD  BCD . cUrbgðajfa ACD  90 . o

vi> h‘un Esn b‘unr:anIvtþPñM ³ éf¶TI 09 mIna 2005


eday erol rdæa

 572 

  
I. RsaybBa¢ak;fa plbUk 385a 1001b  Eckdac;nwg 77 .
3 5

 x2  2 y  1  0
 2
II. eK[ x , y , z Edl  y  2z 1  0 . KNna A  x 2000
 y 2000  z 2000 .
z2  2x 1  0

III. eK[smIkar  4  m x  4x  m  0 . manb£sBIrKW x , x . kMNt;témø m edIm,I[smIkarman
2
1 2

plbUkkaerénb£sesµI 10 .
IV. kukmYyehIrCYbRksarmYyhVÚgk¾ERsksYrfa {sYsþI¡ mitþTaMg 100 { eBlenaHemxül;énhVÚgRksareqøIyfa
{eT¡ cMnYnBYkeyIgminRKb; 100 eT} cMnYnBYkeyIgbUknwgcMnBYkeyIg bEnßmBak;kNþalcMnYnBYkeyIg ehIy
Efm 14 éncMnYnBYkeyIg RBmTaMgmitþÉgeToteTIbRKb;cMnnY 100 . KNnacMnnY RksarenAkñúghVÚg .
V. eK[bIcMNuc A , B , C sßitenAelI (xy) . KUsRtIekaNsm½gS ABD nigRtIekaNsm½gS BCE enAEt
mçagénbnÞat;enH .
1> RsaybMPøWfa AE = CD .
2> eK[ M nig N kNþalRCug [AE] nig [CD] . bgðajfaRCug BM = BN .
3> RsaybMPøWfa RtIekaN BMN CaRtIekaNsm½gS .

vi> h‘un Esn b‘unr:anIvtþPñM ³ éf¶TI 07 mIna 2005


eday em:A TUc

 573 

  
I. KNna S
a

b

 a  b  a  c   b  c  b  a   c  a  c  b 
c
.
II. eK[bIcMnYn x , y , z xusBIsUnü Edl x  y  z  1 nig 1x  1y  1z  0 . bgðajfa x 2
 y2  z2  1 .
III. dak;CaplKuNktþanUvkenSam A  ab  x  y   xy  a  b  .
2 2 2 2

RsaybBa¢ak;smPaB 1 2axaxa ax x 1x  1  a 1 2axax  x    1 1a .


2 2
IV.
 
2 2 2 2
 

V. 1> kñúgtRmúyGrtUNrem rksmIkarénbnÞat;kat;tamcMNuc A6 , 2 nig B  2 ,  2 rYcrksmIkarExS


emdüaT½rrbs;Ggát; [AB] .
2> eK[cMNuc C  2 , 6 . bgðajfaRtIekaN CAB CaRtIekaNsm)at .
VI. rgVg;mYykaMmanrgVas; R . eKKUsGgát;FñÚBrI EkgKña [BC] nig [DE] ehIyRbsBVKñaRtg; A ehIy [BB’]
CaGgát;p©iténrgVg;enaH . RsaybMPøWfa AB  AC  AD  AE  4R .
2 2 2 2 2

VII. RtIekaNsm)at BAC nig ADC EkgRtg; A nig D manRCugrYm [AC] ehIytaMgenAsgxagRCugrYmenH.
eK[ AB  AC  a .
1> bgðajfactuekaN ABCD CactuekaNBñayEkg .
2> KNna BC , AD , BD .
3> Ggát;RTUgTaMgBIrrbs;ctuekaNkat;KñaRtg; O . bgðajfa OAD COB rYcrkpleFob
dMNUc RBmTaMgKNna OA , OB , OC , OD .

PñMeBj ³ éf¶TI 23 kumÖ³ 2005


eday R)ak; kaNaDI

 574 

  
2 3 2 3
I. 1> bgðajfa   2 .
2  2 3 2  2 3
5 3 2 27  2 3 3 4
2> KNnatémøelxénkenSam A
3
.
9 16
II. eK[ a nig b CaBIrcMnYnmansBaØadUcKña . bgðajfa k> a 2
 b2  2ab x> ba  ba  2 .
III. 1> dak;CaplKuNktþa E  12  x  3  27  x  5 .
2 2

2> edaHRsaysmIkar 4x  5x 1  0 .
4 2

3> edaHRsayvismIkar 1x 21  1x 12  x  4 .


 x y 1
  
4> edaHRsayRbB½n§vismIkar a  2 a  2 a  2
 .
 x  y  1
a  2 a  2 a  2

IV. 1> KNnabIcnM Yn x , y , z edaydwgfaplbUkvaesµInwg 292 ehIy x nig y smamaRtnwgcMnYn 150 nig 90
ehIy y nig z smamaRtnwgcMnYn 24 nig 21 .
2> eK[knSam E   a 1 x  2 b  4 x  3c 15 . cUrkMNt;témø a , b nig c edim,I[BhuFa E = 0
2 2

cMeBaHRKb;témøén x .
9 x  6 y  10 z  1
V. edaHRsayRbB½n§smIkar 
 6 x  4 y  7 z  0 .
 x2  y 2  z 2  9

VI. eK[rgVg; C (O , R) Ggát;p©it [AB] EdlbnÞat; AB EkgnwgbnÞat; D Rtg; H . (B enAcenøaH O nig H).
M  C (O , R) bnÞat; AM , BM nigbnÞat;b:HrgVg;Rtg; M kat;bnÞat; T er[gKñaRtg; D , C nig I .

bnÞat; AC kat;rgVg;Rtg; E .
1> bgðajfa I CacMNuckNþalén [DC] rYcTajbBa¢ak;fa (IE) CabnÞat;b:HénrgVg; C (O , R) .
2> K CacMNucRbsBVrvag (OI) nig (ME) , J CacMNucRbsBVrvag (ME) nig (AB) . RsaybBa¢ak;fa
OJ  OH  OI  OK  R rYcbgðajfabnÞat;(ME) kat;tamcMNucnwgmYykalNa M rt;elIrgVg; (C).
2

3
 575 

  
I. RbsinebI a bc  2 nig ab  2 . KNnaplKuN P  abc .
2004 2005 2004 2004

II. rkBIrtYbnÞab;énsVIútcMnYn 1002 , 2005 , 4011 , ....... , ......... .


 1 2 3  x 2  y 2  2 xy  8 2
 
III. edaHRsayRbB½n§smIkar k> 
 x 1 y  2 z  3 x>  .
 x  2 y  3z  56  x  y  4

IV. KNnaplbUk S edayRsaybBa¢ak;rUbmnþTUeTACamunsinKW ³ 1
 ... rYceTIbKNna ³
 n  1 n  n n 1

S
1

1
2 1 1 2 3 2  2 3
 ... 
1
100 99  99 100
.
V. eK[ [AM] CaemdüanénRtIekaN ABC EkgRtg; A . ehIyman AB = 4cm , AC = 5cm . KNna
épÞRkLaénRtIekaN AMB nigRtIekaN AMC .
VI. man [Sx) , [Sy) nig [Sz) tamlMdab;enH Edl xSy  60 , ySz  90 . cMNuc A , B nig C enAelIknøH
o o

bnÞat;TaMgbIerogKña eday SA = SB = SC = a .
1> KNna AB , BC nig AC CaGnuKmn_én a . rYcR)ab;RbePTRtIekaN ABC .
2> H CacMNuckNþalén [AC] . bgðajfa [SH] EkgnwgépÞénRtIekaN ABC .
bEnßm ³ xSz  120 .o

vi> b‘unr:anI hs vtþPñM> éf¶TI 14 kumÖ³ 2005


eday hYt riT§I

 576 

  
125  4 2 4 5
sRmÜlr:aDIkal; ³ .
4
I. 4
625  10
x y
II. eK[ x  y  0 nig 2 x 2  2 y 2  5 xy . KNna E
x y
.
 x 2  y 2  13
III. edaHRsayRbB½n§smIkar  .
 xy  6
IV. eK[smIkar x   4a  34 x  4a  a  2 x  4  a 1  0 .
3 2 2

k> edaHRsaysmIkarcMeBaH a   12 .
x> edaHRsaysmIkarxagelI[Cab;GnuKmn_én a .
V. cMkarmYyragCactuekaNEkg. RkLaépÞvaesµIBIrdgénplbUkbeNþay nigTTwg. KNnavimaRtcMkar .
VI. bgðajfa 39  51 Eckdac;ngw 45 .
51 39

VII. KNna B  10110001100000001 ...1 00...00 1 .


énelx
2n 1 0

VIII. eKmanRbGb;bYndUcKña nigb:unKña. kñúgRbGb;TI1manXøIs 1 Rkhm 1. kñúgRbGb;TI2manXøIs 2 Rkhm 3


kñúgRbGb;TI3manXøIs 3 Rkhm 5 . kñúgRbGb;TI4manXøIs 4 Rkhm 7 . eKeRCIserIsRbGb;mYy rYceK
ykXøImYyedayécdnü ecjBIRbGb;enaH . RbU)abedIm,I[eKerIsykRbGb;TI1 esµInwg 101 . KNna
RbU)abedIm,I[eKerIsyk)anXøIBN’s .
IX. eK[ctuekaN ABCD mYycarwkkúñgknøHrgVg;Ggát;p©it [AD] nigp©it O ehIy
AB  2 5 cm , BC  2 5 cm nig CD  6cm . KNnakaMrgVg;carwkeRkActuekaNenH .

PñMeBj> éf¶TI 05 kumÖ³ 2004


eday taMg RTI

3
 577 

  
I. KNnatémøelxénkenSam ³ A   2005  02005 12005  2 ... 2005  3005
15 2 27  8 3 4 9 16  5 3 4
.
3 3 4 3 3
B 
4
9 3 16 33 4
2
 1 1 x 1  1  x  x2 1
II. 1> eK[1  x 2
0 cUrsRmÜlkenSam A  x       1 .
 1 x  1 x 1 x  1 x  2

 4  4  4   
2> eK[ a  1 . bgðajfa f  a   1  1  1    ...  1 
4
2
CacMnYnGviC¢man .
 1  9  25    2a  1 

III. 1> edaHRsaysmIkar 12  x  14  x  2 .


3 3

2> edaHRsayRbB½n§smIkar  xy 11  22yx 12 .


3


3
 
IV. 1> RtIekaN ABC sm)atkMBUl A , [AH] Cakm<s; . RtIekaNsm)at ABCmanbrimaRt 32 cm nig
manRkLaépÞesµInwg 48 cm . KNnargVas;RCug nigkm<s; [AH] énRtIekaN ABC .
2

bBa¢ak; rgVas;RCug nigkm<s;ykEtcMnYnKt; .


2> H CaGetUsg;énRtIekaN ABC .
k> bgðajfa H’ CacMNucqøúHén H eFobnwg (BC) sßitenAelIrgVg;carwkeRkAénRtIekaN ABC .
x> I , M nig Q Cap©itrgVg;carwkeRkAerogKñaénRtIekaN AHB , AHC nig BHC .
bgðajfa (IM)//(BC) nig (MQ)//(AB) .

Gnu hs bUrI100xñg> éf¶TI 25 kumÖ³ 2005


eday C½y sMGun

 578 

  
I. eK[smPaB  x  y    y  z    z  x    x  y  2z    y  z  2x    x  z  2 y  .
2 2 2 2 2 2

bgðajfa x  y  z .
1 1 1
x  y  z  2

II. edaHRsayRbB½n§smIkar  .
 2  1 4
 xy z 2

III. edaHRsaysmIkar 2 x  2 x  1  4 x  1 .
2

IV. eK[kenSam A   x  y  x  2 y  x  3y  x  4y   y . bgðajfakenSam A CakaerR)akd .


4

V. KNnakenSam ³ B  1 1 2  1  21  3  1  2 1 3  4  ...  1  2  3 1 ...  100 .


VI. RsaybBa¢ak;fa TIRbCMuTm¶n; p©itrgVg;carwkeRkA nigGrtUsg; énRtIekaNmYy CabIcMNucrt;Rtg;Kña .

Gnu hs bUrI100xñg> éf¶TI 01 mIna 2005


eday Tk; yUGUn

 579 

  

I. bgðajfa ³ 6  2 5  13  48  3 1 .
3  x 2  x x 1
II. edaHRsaysmIkar 1997  
1998 1999
 3 .

III. eK[bIcMnYn x , y , z smamaRtnwgcMnYn 5 , 7 , 11 .


1> KNnacMnYnTaMgbIkalNa ³
k> x  2 y  3z  48 .
x> x  my  z  72 , m CacMnYnKt;EdleK[ . rktémøén m minRtUvman .
2> kñúglT§plénsMNYr 1>¬x¦ eKcg;[ x , y , z CacMnYnKt;viC¢man . etIcMnYn 16-7m RtUvbMeBj
lkçxNÐy:agNa ? etImantémø m b:unµanEdlbMeBjlkçxNÐenH ?
3> eKcg;[ m CacMcMnYnKt; . KNna m ¬mancemøIyeRcIn¦ nigtémørbs; x , y , z .
IV. eK[kaer ABCD manRCugRbEvg a . eKsg;RtIekaNsm½gS EdlmanGgát;RTUg [AC] CaRCug ehIyman
kMBUl E enAxag B . O CacMNuckNþalénRCug [AC] .
k> bgðajfacMNuc E , B , O nig D rt;Rtg;Kña .
x> KNna BE CaGnuKmn_én a .
K> bgðajfa EB  ED  AB . 2

X> BI E eKKUsbnÞat; (EP) b:HrgVg;carwkeRkAkaerRtg; P . bgðajfa EP = a .

eday hak; sm,tþi

 580 

  
I. rkcMnYnEdlmanelxBIrxÞg; ab EdlepÞógpÞat; ab  ba  1980 .
2 2

x  5 x  3 x  2000 x  2002
II. edaHRsaysmIkar    .
2000 2002 5 3
2
III. bM)at;r:aDIkal;BIPaKEbg ³ A  .
8
4  16  64  256
8 8 8

IV. KNnabIcMnYnviC¢man a , b , c edaydwgfa ab  30 , ac  40 , bc  75 .

V. eKmanR)ak;my Y cMnYnEck[mnusSbInak; A , B , C . R)ak;cMENk A nig B elIscMENk C cMnYn 500 `


R)ak;cMENk A nig C elIscMENk B cMnYn 330 ` nig R)ak;cMENk B nig C elIscMENk A cMnYn 60 `.
1> rkcMnYnR)ak;EdlRtUvEck[mnusSTaMgbI .
2> KNnaR)ak;EdlmnusSmñak;RtUvTTYl)an .
VI. edaHRsaysmIkar ³ 0.17  2.3  x  0.3 .

VII. eK[kaer ABCD nig M mYyenAelI BC . (AM) kat; (DC) Rtg; N .

RsaybBa¢ak;fa AB1  AM1  AN1 .


2 2 2

VIII. kMNt;RbePTRtIekaN ABC edaydwgfakm<s; AH nigemdüan AM Eck BAC CabIcMENkEdlman

rgVas;esµIKña .

Gnu> pSaredImfáÚv> 01 mIna 2005


eday eqg Kwmetg

 581 

  
I. sRmÜlkenSam E  625  2 80 .
4 3 3

II. eKmanBIcMnYn a  b  0 bgðajfa RKb;cMnYn n  eK)an *


a n  bn .
III. edaHRsayRbB½n§smIkar a a b 8b  3 .
 3 3

IV. hIugmYyenAkñúgGNþÚgmYymanCeRmA 7m . stVhIugenHxMetageLIgmkelI Edlkñgú mYyéf¶etageLIg)an


3m EtFøak;cuHvij 1m . etIb:unµanéf¶eTIbhuIgenH eLIgputBIGNþÚg ?

V. eK[RtIekaNsm)at COB Edlman)at [OB] nigkm<s; [OA] .


RsaybBa¢ak;fa AC  OB  AB  OC .
2 2 2 2

vi hs Cm<Úv½n > 28 kumÖ³ 2005


eday Rs‘un saerOn

 582 

  
I. 1> etIcMnYn A  2  5 ,  n   bBa©b;edayelxsunücMnYnb:unµan ?
n 2 n 1 *

2> eRbóbeFobcMnYn 200 nig 300 .


300 200

x  4 x  3 x  2 x 1
II. edaHRsaysmIkar 2001   
2002 2003 2004
4 .

III. edaHRsayvismIkar 1x 21  1x 12  x  4 .


IV. enAkñúgtRmúyGrtUNrem edAcMNuc A 6 , 0 ; B 6 , 0 nig C 3 , 9 .
1> sresrsmIkarRCugénRtIekaN ABC .
2> sresrsmIkaremdüaT½rrbs;RCugRtIekaN ABC .
V. eK[RtIekaNEkgsm)at PMN EkgRtg; M . kñúgmMu MNP eKKUsknøHbnÞat; [Nx) Edlkat; [MP]
Rtg; D . tamcMNuc P eKKUsbnÞat;Ekgnwg [Nx) Rtg; E ehIyknøHbnÞat;enHkat; (MN) Rtg; F .
1> bgðajfabnÞat; (FD) Ekgnwg [NP] rYcKNnargVas;mMu NFD .
2> bgðajfa M , D , E , F sßitenAelIrgVg;EtmYy rYcbBa¢ak;p©iténrgVg;enH .
3> bgðajfa MD  MF .
4> kñúgkrNI MNx  30 nig NP  a . KNna MN nig DM CaGnuKmn_én a .
o

vi samKÁI ¬PñMeBj¦ > 20 kumÖ³ 2005


eday vn vNÑa

 583 

  
2
x xy y  x  y 
I. KNnakenSam E 
 x y
 xy 
 x  y 
.
  
II. eK[RtIekaN ABC EkgRtg; A manbrimaRt 12cm nigépÞRkLa 6cm . KNnargVas;RCugénRtIekaN
2

ABC enH .
x y
 xy  1

yz
III. edaHRsayRbB½n§smIkar  2 .
 yz
x z
 5
 xz

eK[kenSam A  xx 3xx 24 .


8 4
IV. 4 2

1> sRmÜlRbPaK A .
2> rktémø x EdlnaM[ A = 4 .
V. eK[rgVg;nwg C(O , R) Edlmanp©it O nigkaM R nigGgát;p©it [AB]. H CacMNucmYyEdlEckGgát; [AB]
tampleFob 13 eday H enAcenøaH A nig B ehIy HA < HB . eKKUsGgát;FñÚ [CD] Ekgnwg [AB]
Rtg; H .
1> KNna HA nig HB CaGnuKmn_én R ehIyTajbBa¢ak;fa H CacMNuckNþalén [OA] .
2> KNna CH , CA nig CB CaGnuKmn_én R .
3> P CacMNucGefrenAelI [CD]. bnÞat; (AP) kat; C(O , R) Rtg; M’ CacMeNalEkgén M elI [AB]
k> rksMNMucMNucénp©itrgVg; carwkeRkActuekaN HPMB kalNa P rt;elI [CD] .
x> eRbóbeFobRtIekaN AHP ; AMB nig BMM’ ehIyTajrktémøénplKuN AP  AM
CaGnuKmn_én R .
4> eKsnµt;fa PAH  30 . kñúgkrNIenHKNna AP , AM nig BM CaGnuKmn_én R .
o

Gnuvi> TYlsVayéRB

3
 584 

  
I. eK[ a  b  c  1 , a  b  c  1 nig ax  by  cz . RsaybBa¢ak;fa xy  yz  zx  0 .
2 2 2

II. edaHRsaysmIkar 3x  11 3x  2  3x  21 3x  3   3x  31 3x  4  3x2 4 .


III. KNnatémøelxénknSam A  x 12x 12x 12x 12x  ... 12x 12x 1 cMeBaH x  11 .
17 16 15 14 13 2

IV. rkcMnYnKt;EdlenAcenøaH 40 nig 50 edaydwgfa ebIbrþÚ lMdab;elx eK)anCYbcMnYnfµImYyeTot esµInwg 3223


éncMnYnenaH .
V. eKmanRtIekaNsm½gS ABC nigrgVg;carwkeRkARtIekaNenH mankaM R . eKP¢ab;cMNuckNþal D énFñÚ AC
eTAcMNuckNþal H énRCug [BC] nigbnøay [DH] [RbsBVrgVg;Rtg;cMNuc M .
1> R)ab;RbePTRtIekaN DCH .
2> KNna DH CaGnuKmn_én R .
3> RsaybBa¢ak;fa HD  HM  HC  HB .
4> rktémø HM nig DH CaGnuKmn_én R .
vi> samKÁI ¬PñMeBj¦

 585 

  
I. KNnatémøelxénknSam A  1 1 2  2 1 3  3 1 4
 ... 
1
99  100
.
II. cUrRsaybBa¢ak;fa 4  2 3  4  2 3  2 3 .
x  4 x  3 x  2 x 1
III. edaHRsaysmIkar 2001   
2002 2003 2004
4 .

2 x  y  z  18

IV. edaHRsayRbB½n§smIkar x  2 y  z  1 .
x  y  2z  2

V. tamcMNuc O enAkñúgRtIekaNsm½gS ABC eKKUsGgát;EkgeTAnwgRCugTaMgbIénRtIekaNenaH .
bgðajfa plbUkcm¶ayBIcMNuc O eTARCugTaMgbI esµInwgrgVas;km<s;énRtIekaNenaH .
VI. eKmanRtIekaNsm½gS ABC Edl AB  AC  BC  a nigman [AH] Cakm<s;. I CacMNucmYyenAelI
[AH] Edl AI  . bnÞat;Rsbnwg [AH] KUsecjBI C CYbbnÞat; (BI) Rtg; M .
AH
3
1> KNna CM nig MB CaGnuKmn_én a .
2> KNnaRkLaépÞctuekaN AHCM CaGnuKmn_én a .
vi> samKÁI éf¶TI 25 kumÖ³ 2005
eday haM sarun

 586 

  
I. edaHRsaysmIkar ³ k> 5x  3 2
x>  2x  3  5x 2 2

II. eK[kenSam A  5x  7 x  2x nig B  2x  2   x  5x  6x  .


3 2 3 2

k> dak;kenSam A nig B CaplKuNénktþa .


x> sRmÜlRbPaK E  BA .
K> KNnatémøelxén E cMeBaH x  2 .
III. kMNt;témø m nig n edIm,I[smIkarCaÉlkçN³PaB ³ x13  x m 4   x  3n x  4 .
 D
IV. eK[RbB½n§smIkar 2y y2xx  74  D '
.

1> k> sg;bnÞat;TaMgBIrenAkñúgtRmúyGrtUNremEtmYy .
x> KNnakUGredaenéncMNuc M CacMNucRbsBVénbnÞat;TaMgBIr .
2> sresrsmIkarbnÞat;Edlkat;tamcMNuc P  2 , 5 nig Q  4 , 2 . rYcbgðajfabnÞat;enHkat;tam
cMNuc M Edlrk)anxagelI .
3> bnÞat;TI1kat;G½kS x’x Rtg;cMNuc A nigbnÞat;TI2kat;G½kS y’y Rtg; B . sresrsmIkar bnÞat; AB .
4> bgðajfa AM Ekgnwg BM rYcKNnaRkLaépÞénRtIekaN AMB .
V. eKmanRtIekaNEkg ABC Edl AB = 4cm ; AC = 3cm . eKP¢ab;cMNuckNþalerogKña A’ , C’ , B’
énRCug BC , AB , AC .
1> eRbógeFobRtIekaN ABC nigRtIekaN A’B’C’ .
2> eRbóbeFobrgVas;km<s; A’P énRtIekaN A’B’C’ nigkm<s; AQ énRtIekaN AB’C’ .
eK[RtIekaN ABC mYy Edl AB = c , AC = b nig mMu A = 45 . BH Cakm<sé; nRtIekaN ABC
o

Edl  BH    AC . KNna BC CaGnuKmn_én b nig c .


eday S.B

3
 587 

  
I. bM)at;r:aDIkal;BIPaKEbg ³ A
20
.
3 5  2 2 5
 x  y  3

II. edaHRsayRbB½n§smIkar  x  z  2 .
 xy  yz  zx  2

III. eK[ a  b  1 . cUreRbóbeFob ba 11 , ba nig ba 11 .
IV. eK[RtIekaN ABC manrgVas;RCug a , b , c nigRtIekaN A’B’C’ manrgVas;RCug a’ , b’ , c’ . ]bmafa
ABC ABC cUrbgðajfa aa  bb  cc   a  b  c  a  b  c  .

V. knøHbnÞat;BuHénmMu A rbs;RtIekaN ABC kat;rgVg;carwkeRkARtIekaNenHRtg; D .


RsaybMPøWvismIkar AD  AB 2 AC .
vi> RBHsIusuvtßi> éf¶TI 05 kumÖ³ 2005
eday Gwug Kwmh‘ag

 588 

  
I. eRbóbeFobBIrcMnYn x  5 2  7  5 2  7 nig
3 3
y  51  10 2  51  10 2 .
II. eK[ M  2 40 12  2 75  3 5 48 nig N  2

1

3
.
8  2 15 52 6 7  2 10
bgðajfa M = N .
III. bgðajfa k> 1 1 2  2 1 3  3 1 4  ...  9999 1 10000  99 .
x> 11  12  13  ...  10000 1
 100 .

IV. edaHRsaysmIkar ³ k>  x  2004  x  0 .


2004 2004

x> 1 x x  2 x x  3 x x  ...  2004x  x  2005x  x .


V. k> sresrCaplKuNktþanUvsmPaB ³ 1a  b1  1c  a  1b  c .
x> bgðajfa ebIbIcMnYnminsUnü a , b , c ehIymanBIrcMnYnpÞúyKñay:agtic enaHeK)anTMnak;TMng ³
1 1 1
  
a b c abc
1
.

x  y
K> edaHRsayRbB½n§smIkar x , y ³ 
 x  y  20 .
 2
 x  y  15
 3
VI. enAem:ag 6h30mn narImYyRkumcab;epþImsÞÚgRsUvBIPøWmçageTAPømW çageTotEdlmancm¶ay 100m eday
el,Ón 30m/h ehIyyuvCnmYyRkumeTotcab;epþImsÞgÚ BIPøWmçagedayel,Ón 20m/h .
k> etIRkumTaMgBIrsÞÚgCYbKñaenAem:agb:unµan ? x> etIRkumnImYy²sÞÚg)anRbEvgb:unµanEm:Rt ?
VII. eK[kaer OACB p©it I . sg;rgVg;p©it P (P enAelI AB) kat;tam O kat;bnÞat; (OA) Rtg; M nig (OB)
Rtg; N . k> RbdUcRtIekaN CMA , CNB nig CPI . KNna AM PI
rYcbgðajfa CP   MN  .
x> sg; C’EdlCacugTI2énGgát; [CC’] énrgVg; (P) bnÞat;TajBI C’Ekgnwg [OC’] kat;
[MN] Rtg; Q . bgðajfa [CC’) BuHmMu OCQ .

3 vi> RBHsIusuvtßi> éf¶TI 13 mIna 2005


 589 

  
I. edaHRsaysmIkar 3  3  30 .
2 x 2 x

II. RsaybBa¢ak;fa ebI abc  1 nig a  36 enaHeK)an


3 a2
3
 b 2  c 2  ab  bc  ca .
III. sRmÜlRbPaK A  x3x14xx 1x eday x  0 .
2

x y 1
 xyz  2

yz 5
IV. edaHRsayRbB½n§smIkar ³   .
 xyz 6
x z 2
 
 xyz 3
V. RCugBIrénctuekaNEkg ABCD manrgVas; 20cm nig 30cm . kMNt;TItaMgrbs;kBM UlRbelLÚRkam
MNPQ EdlcMNuc M , N , P , Q enAelIRCugerogKña [BC] , [AB] , [AD] , [DC] nig

MB  BN  QD  DP edIm,I[épÞRbelLÚRkamFMbMput rYcKNnaépÞFMbMputenaH .

VI. kñúgRtIekaN ABC emdüan [AA’] , [BB’] nig [CC’] kat;KñaRtg; G . F CacMNuckNþal [BG] . BI F
eKTajbnÞat;Rsbnwg (AB) ehIyBI G eKTajbnÞat;Rsbnwg (AC) ehIybnÞat;TaMgBIrkat;KñaRtg; E .
bgðajfa cMNuc B’, E , C’ rt;Rtg;CYr .

vi> TYlTMBUg> éf¶TI 09 mIna 2005


eday G‘uk sam:aNa

 590 

  
I. n CacMnYnKt;FmµCati  n  . rkcMnYnKt;FmµCati n edIm,I[BIcnM Yn n 12 nig n  77 CakaerR)akd.
II. k> eRbóbeFob 1  2012 nig 2013  2  2012 .
2 2

x> edaymineRbI 2012  4048144 nig 2013  4052169 cUrrKNna ³


2 2

2
 2012  2012
A  1  2012  
2
 
 2013  2013
.
III. bgðajfa N  2013 5  3  29 12 5 CacMnYnKt; .
IV. eK[bIcMnYn a , b , c EdlepÞógpÞat;lkçxNÐ a  2b  1  0 , b  2c  1  0 , c  2a  1  0
2 2 2

cUrKNnakenSam F  a  b  c  2012 .
14 2 2013

V. kñúgrgVg;p©it O Ggát;p©it [BC] Edl BC = 5cm . eKKUsGgát;FñÚ [BA] Edl BA = 3cm .


k> KNnargVas;RCug AC .
x> RsaybMPøWfa cos ABC  sin ABC  1
2 2

K> eKKUsemdüan [BM] énRtIekaN ABC Edlbnøayrbs;va kat;rgVg;p©it Rtg;cMNuc D .


Bgðajfa MB  MD  MA . 2

Gnu> s<W¬x¦> éf¶TI 24 mkra 2013


eday Gan suxKn§a

 591 

  
I. eRbóbeFobcMnYn 2012  2014 nig 2 2013 .
edaHRsayRbB½n§smIkar  xx  y y z zxy  yz2013
 zx
2 2 2

II. .

2013 2013 2013 2013

III. KNnaplbUk S  2 1 1 1 
1

1
 ... 
1
.
2 3 2 2 3 4 3 3 4  n  1 n  n  n  1
2 3 2 3
IV. bgðajfa   2 .
2  2 3 2  2 3
V. eK[ ax  by  cz  0 nig abc 
1
2013
.
ax 2  by 2  cz 2
cUrKNnakenSam A  .
bc  y  z   ac  x  z   ab  x  z 
2 2 2

VI. ABC CaRtIekaNcarwkkñúgrgVg;p©it O kaM R . ehIy [AH] Cakm<s;. eKsnµt; AH  h a , AC  b ,

AB  c , BC  a . S CaépÞRkLaénRtIekaN ABC . RsaybBa¢ak;fa abc  4RS .

Gnu> s<W¬x¦> éf¶TI 24 mkra 2013


eday Gan suxKn§a

 592 

  
I. A B
A  a  x 2  1  x  a 2  1

B   2 x  3  9  x  5 
2 2

2 
II. A 1 , 8  , B  2 ,  1 , C  , 7  ។
3 
1 1 1
III. a,b,c a  b  c 1   0 a 2  b2  c 2  1
a b c
3x 2  14
IV. K
x2  4
V. O E [AC] [BD]
ME
[AD] [BC] M N
EN
VI. ABC [AM] E [AB] BE  2EA CE [AM]
O
O [AM]
CE  4OE

> ២ ៤

 593 

  

I. E  2 x2  y 2  2x x  y  x  y
II. A  a2b2 b  a   b2c2  c  b   a2c2  c  a 
a c ad  bc 2cd
III.  
b d 2ab ad  bc
IV. 110 cm 7cm
4cm

V. ABC H O D [BC] ,
E [AC] G
ABH DOE
AH  2DO
២ AHG DOG
H ,G,O
GH GO

> ២ ៤

 594 

  

I. A  11  6 2  11  6 2
II. x4  3x2  10  0
1
  
3
III. y 3
2 1 3
2 1
3
IV. a,b,c
a 2  2b  1  0 , b 2  2c  1  0 , c 2  2a  1  0 S  a 2004  b2004  c 2004
V. O 30o

> ២ ៤

 595 

  

x  4 x  3 x  2 x 1
I.    4
2001 2002 2003 2004
4
8 2 1 1
II. 
4
8 2 1  4
8 2 1 2

3 9 15 6n  3 75
III. n    ...  
2 2 2 2 2
 6  12  18  ...  96 
2

២ A
12  24  36  ...  192 
2

1 1 1 1
IV. S    ... 
2 1 1 2 3 2  2 3 4 3  3 4 100 99  99 100
V. ABC Aˆ  20o ˆ
ABC [Bx) ˆ  20o
CBx
AB3  BC 3  3 AB2 BC
VI. ABCD O ˆ  OBA
OAB ˆ  15o ˆ
OCD

 596 

  

I.  24x 112x 18x 16x 1  330


II. x, y A   x  y  x  2 y  x  3y  x  4 y   y 4
III. 72002  72003  72004 57

IV. 220 1  5  1  20 6  2 5
V.  D1  : 2  3x  y  0  D2  : 4  m  1 m x  my  0 m

 D1   D2 
 D1   D2 
VI. ABCD 1 [AB] [AD] E F AEF
2 ˆ  45o
ECF

២ ៤

 597 

  

I.
6 5 21

22004 3  2 1  2004 11  72
ax3  by 3  cz 3

II. 1 1 1 3
xa 2  by 2  cz 2  3 a  3 b  3 c
x  y  z 1

1 1 1 1
n 1    ...  1
1 2 2  3 3  4 n  n  1
III. x, y 2 xy  y  4 x 2  2 x  1

A  2004 xy  2003 y3
f  x   x17 13x16  13x15 13x4  ... 13x2 13x 1 x  12
15x  9 y  z  300
IV.  (x , y , z
 x  y  z  100
V. ABCD 48 cm 2 5,7
ABCD
VI. ABCD [AB] E D [AC] F
H [AE] [BC] DFH

២ ៤

 598 

  
1
I.  D1  : y  2x  4  D2  : y   x4
2
 D1   D2  T
 D1   D2  A B TO2  OA  OB
TAB TAB
II. x y 110 m
5m 2m 150 m 2
III. ABCD A D [AB]
[AC] [BC]
IV. O [AB] O’ AB O’ A B M N
O’ (MA) , (MB) (O) C D (NA) , (NB) (O) E F
MAD NBE

២ ២ ៤

 599 

  

I. A  1002  992  982  972  ...  32  22  1


B  100  99  98  97  ..  3  2  1 A B
1
 27  3 1
២ E  54     3 
3
2
 4  4
3x3  x 2  8 x  4  0
x 1 x  2 x  3
  3
2000 1999 1998
៤ 3951  5139 45
II. 18 cm , 24 cm 36 cm

III. A(-3 , 2) ; B(9 , 2) C(3 , 10)


ABC
IV. ABC O R
R
V. ABCD O B [AD]
[AC] E A [BC] [BD] F
OA OB  OC  OF  OD  OE
២  EF  || CD

២ ២ ៤

 600 

  

I. 1999  2001 2 2000


1 1 1 1
២ S   ...  
2 1 1 2 3 2  2 3 1999 1998  1998 1999 2000 1999  1999 2000

II. A  3 9  80  3 9  80

២ B  A  3 26  15 3  2  3  
III. a,b x 4  2 x3  ax 2  bx  1 ២

២ x,y x 2  y 2  24
a,b,c
ab  bc  cd  a2  b2  c2  2  ab  bc  cd 
 x  xy  y  1

IV.  y  yz  z  3
 z  zx  x  7

២ a,b,c a  b  c  abc
a  b2  1 c2  1  b  a 2  1 c2  1  c  a 2  1b2  1  4abc
x2   2m  3 x  m2  3m  0
m 1  x1  x2  6
V. ABC A [AH] , R1 , R2 , R3
ABC , AHB AHC R1  R2  R3  AH
២ (O) (O’) A B [AC] [AD]
(O) (O’) (CA) (O’) E
B EBD
AC  CE  2BE 2

២ ៤

3  601 

  

I. S  1 3 3 
P  2 3 3 
A 2x  4
II. F  x A0 B0
B x 1
 4x 4 x  1  x  1  4 
2

2
 1
III. A 2 A x
 x  6 x  9  x  1  x  x  1
2
2

2 x  3 y  z  4

IV. 4 x  2 y  3z  1 ។
 6 x  y  2 z  7

2
V. 2.5 dm dm
3
4
2.5 dm dm dm 2
3
VI. A B
AAB
២ A B
A
VII. C(O , R) [CD] [AB ] H M BC M
(DC) T [CD] [AH] I
[MA) CMD
២ TIM
ACM IMD
MA MI  MC  MD
៤ HIMB
AMB ˆ "  30o
MAB AH MB R

២ ៤

3
 602 

  

I. 385a3  1001b5 77
3x 2  14
II. K
x2  4
III. a,b,c

ab  bc  ac  a2  b2  c2  2  ab  bc  ac  .
 x2  2 y  1  0
 2
IV. x,y,z  y  2z 1  0 A  x 2000  y 2000  z 2000
z2  2x 1  0

V.  4  m x 2  4 x  m  0 x1 , x2
m 10
VI. G ABC H , K , K’ A,B,C 
G AH  BK  CK '
[KK’] , [AG] [AG] 
R
VII. (C) C R I OI  [MN] I
3
[MN]
[MN]
J [MN] J [MN] I

២ ២ ៤

 603 

  

3
4 x 1 3 3
I. A 16 x  16 x B 6
27
2
C ឃ D  4  2 3  1 3
6 3

1 3  3 9
3

4
1  1 x
II. x  4
4
 x    1296 4
y  y y
x  12 x  11 x  74 x  73
III.   
77 78 15 16
IV. A  4  15  4  15  2 3  5
V. ABC O OD   BC D OE   AC
C OF    AB F BD2  CE 2  AF 2  DC 2  AE 2  FB2
VI. ABC A [AH] H [AC] E  AB D

AB 3 BD

AC EC

 604 

  

I. 
A  1 3 3  26  2 26  2 28  6 3  1 
B  3 2 27  48  3 
A B
1 1 1 1
II. A    ... 
1 2 2 3 3 4 99  100
III. a,b x 2  px  1 c,d x 2  qx  1
 a  c b  c  a  d  c  d   q2  p2
IV. x3  3ax2  3  a 2  bc  x  a3  b3  c3  3abc  0
V. R [BC] [DE] A ឃ

AB  AC  AD  AE  4R2
2 2 2 2

VI. O a [BC]  AB a

AC a ABC
D  AB CD  AO E E  AB   AC 
1
F F  AC  2
AF CF a

2
BFC ABC
3
OF   BC  OF   BC  G (G A
 AG   BF  H H ABC

 605 

  
I. a,b,c ab  bc  ac  a2  b2  c2  2  ab  bc  ac  .
1 1 1 1
II. A    ... 
1 2 2 3 3 4 99  100
1
III. x2  x  1  0 x14 
x14
1 1 1
x2  x  1  0 x1930  1930
 x1945  1945
 x1987  1987
x x x
x 1 x6 x2 x5
IV.  2  2  2
x  2 x x  12 x  35 x  4 x  3 x  10 x  24
2

 x  2 y  3 z  4v  31
3 x  3 y  4 z  v  8 x y z v
    
V.   1 3 4 5
2 x  y  5 x  3v  13  xyz  xyv  yzv  xzv  20
2 x  2 y  z  v  17

VI.  AB  CD O OB M OC  N


ˆ  OBD
OCM ˆ , OBN ˆ
ˆ  OCA  AN  ||  DM 


 606 

  
 3 x  3 y  4
I. 
 xy  27

II. E  12  6 3  3 729  4 9
III.
x2  2 x  1  0

 t  2 0
3

IV. 8
37.50%
V.

230 150 75 300


1 cm 50
 y  x  2 1
VI. 
 x  2 y  2  2 
 1
I 0 ,   PQ P Q 1
 2
 2  yy '
VII. ABCD AB  AD A  BD H
I  BH  , J CD ˆ
AIJ

 607 

  

I. A  4  15  4  15  2 3  5
35  278  2  911 
II.
15  816  12  319 
x y
III. x y0 2 x 2  2 y 2  5 xy E
x y
1 1 1 1
IV. A   
42 56 72 90
1 1 1 1
B   
15 35 63 99
x y 3
  
V. 5 3 4
 xy  yz  zx  21
1 1
VI. ២ x
10  72 10  6 2
VII. ABC  AM  ;  BN  CP  MA ;  NB  PC 
MNP H

 608 

  

1 1 1 1
I.    (x ab  0)
a b x a b x
1 1 5
x  y  6

II.  ,  x  0 , y  0
 1  1  13
 x 2 y 2 36
x xy y 2 y xy
III.   1 , x  0 , y  0 , x  y
 x  y x y  x y x y

3a 2  14
IV. K K
a2  4
n  n  1
V. 1  2  3  ...  n  , n
2
VI. 9A 40 9B 36

VII. A,B,C AB = 5cm, BC = 12 cm


(AC) [Ax) [Cy) A C [Ax) [Cy) P M
AP = 4cm CM = 15cm [BM] [BP]
AP BC
BAP MBC
AB CM
២ PBM
H B [MP] PABH BCMH

៤ AHC

២ ៤

3
 609 

  

I. ab  1 a5  b5   a3  b3  a 2  b2    a  b 

 x2  2 y  1  0
 2
II. x, y,z  y  2z 1  0 A  x 2000  y 2000  z 2000
z2  2x 1  0

BC 2
III. ABC  AM   AH  AB 2  AC 2  2 AM 2 
2
IV. 0  a 1 x 1  a   2 a

2 2  a 1 
  1  a  
1 1  a 
1 1
1  x2 1  x2
 x  1  y  2002  2003
V. 
 x  2002  x  1  2003
VI.

២ ២ ៤

 610 

  

x y
I. x y x 2  y 2  2 xy  2
y x
II. x 2003  3x 2002  2 x 2001
III. a,b,x
a 2  2b  1  0 , b 2  2 x  1  0 , x 2  2a  1  0 S  a 2003  b2003  c 2003
1
 a b  a b   a 2  b2 
IV. a,b A     2     2   
 b a  b a   2ab 
V. a b a b 1
 a  b   a2  b2  a4  b4    a8  b8   0
 x 1 y
 a  2  a  2   a  2
VI. 
 x  1  y
 a  2 a  2 a  2
VII. P ABC  AA ,  BB CC
2  AA  BB  CC  3P
VIII. xoy 120o , [oz ) xoy M [oz ) M
[ox) [oy ) A B M [oy ) [ox)
M B [oz ) [ox) B
AM  AM MM  OA AM OM
   
OA AB OB AB AB BB
1 1 1
២  
OM OA OB

៤ ២

3
 611 

  

I. ២

3x 2  2 x  5  0 ២ x  2 x  4
2

x2  0.4 x  0.16  0 ៤ x  7 x  15


2

II.
2  x 3  x  4  1 3x  7 3  5  3x  x  5 x  3 x 1 x  2
    ២   
3 2 4 6 2 2 4 3 6
 5
3 14 x  3  3 x  2 6 x  7  4 x  7
 ៤ 
 4  x  4   3 x  14  8 x  3  2 x  25
 2
III. 720 m 2 6m 4m

IV. ABC (3 , 0) ; (10 , 3) (3 , 6)

V. x
B
\

N x?
9 3 \ 6

M 8 P
A C
24

៤ ២

 612 

  

1 1 1 1
I. A    ... 
1 2 2  3 3  4 99 100
1 1 1 1
B    ... 
1 2 2 3 3 4 899  900
II.  x  3 x  4 x  4 x  6  m
m
m
m8

III. x, y,z x yz 0 x2  y 2  z 2  2

x3  y 3  z 3  3xyz
S  xy  yz  xz
IV. [Ox) [ Ny) MON MNO MON M
[Ox) [ Ny) A B  AB   NO
V. ABCD AB  4 dm AD  3.5 dm  BC 
 BD BCD S ABD  6 dm 2 ABCD

 613 

  

4
125  4 2 4 5
I. 4
625  10
x y
II. x y0 2 x 2  2 y 2  5 xy E
x y
 x 2  y 2  13
III. 
 xy  6
IV. x3   4a  34 x2  4a  a  2 x  4  a 2  1  0
1
a
2
a
V.
VI. 3951  5139 45
VII. B  101100011000001 ... 100...001
2n 1 elx 0
VIII. ឃ 1 ឃ 1 ឃ 2
ឃ 3 ឃ 3 ឃ 5 ឃ 4 ឃ 7

1

10
IX. ABCD  AD O
AB  2 5 cm , BC  2 5 cm CD  6 cm

២ ២ ៤

 614 

  
4
53  5 54
I. A
2
511

II. B 2  3 74 3 


x  7 x   2  x 2  7 x   48
2 2
III.
1 1 1 2
  
 3x  1 3x  2   3x  2  3x  3  3x  3 3x  4  3x  4

2 19 2 3
IV. 2  
5 15 3 5
2 x  2 y  3xy
V. 
 y  6 x  4 xy
3x 2  14
VI. K 2
x 4
VII. ABC A Aˆ  2 Bˆ
VIII. ABC R D AC
H [BC] [DH] M
DCH
DH R
HD  HM  HC  HB

> ២ ២ ៤

 615 

  

1 1
 2 3  2 3
I. A  2    3  
 6 2   4 3 
2 3 2 3
B 
2  2  2 3  2  2  2 3 
II. 4 x 163 x1  8
III. mx2   m  n 1 x  2m  n  8  0 m n S 4
P2
IV. a,b,c a b 50 70 b c
7 9 a,b,c a  2b  3c  90
V. ABC Bˆ  20o [AB] M BM  AC
ˆ
AMC
VI. O [AB] R d  [OB] I M
(AM) (BM) d  N P
3 2
ANI PBI IN  IP  IA  IB  R
4
A,I,P,M
 BN    AP 

>

 616 

  

I. 20022003  20021979 6
II. a , 36 a < 36 a PPCM  a , 36  72 PGCD  a , 36  12
1
III. a  b  c 1 a 2  b2  c 2 
3
IV. x 2
 x  3 x 2  x   4

 x  y  2
2

 2 , (x  0 ; y  0 x  y  1)
 x  y  2
V. [AB] A  2 , 3 ; B  4 , 4
VI. ឃ 2 4 6 ឃ

VII. 4
2 4
VIII. R ABC BC 20 cm
ABC R
IX. ABC O B (OA) M (AC) N
ABM ABC ABN
X. ABC [AK] [BD] H O ABC
[AC] E [BC] F
AH  2OF
G ABC H,G,O

 617 

  
I. a b a 2  b2  2ab

1999   2001
2 2

 3999999
2
II. a b
1
x2  x  1  0 x14   1
x14
a b
III.  a  b  c  d  a  b  c  d    a  b  c  d  a  b  c  d  
c d
C  6 2  32  32

A   a  b  c    a  b  c    a  b  c    a  b  c 
2 2 2 2
IV.
1 1 1
a bc  0  2  2 0
b  c  a c  a  b a  b2  c 2
2 2 2 2 2

 x 2  y 2  xy  37 1
 2
 x  z  xz  28  2 
2
V.
 2
 y  z  yz  19  3
2

VI. 1  2  x 2
 
 2 1 2 x 1 3 2  0
x x x
   1  abc  x  a  b  c 
ab bc ac
x x x
  2
 a  b  a  c   b  a  b  c   c  a  c  b 
VII. O a [BC] [AB] a
AC a ABC
D [AB] [CD] [AO] E E (AB)
1
(AC) F F [AC] AF CF a
2
2
BFC ABC
3
(OF) [BC] (OF) [BC] G(G A
(AG) (BF) H H ABC ៕

3  618 

  
     3  
2
I. A 2 32 5  2 32 5 3 5 3 3 5

 2a  2b    2a  2b   3a  b 3a  b  3  3 2 1 
2 2

F
9b2  6ab  a 2
E3  3
2  1  
 3 
II. a,b,c ab  bc  ca  0  a  bb  c  c  a   abc  0
III. a,b,c a bc  0 abc  0
5 5 5
E  2  2 2 2
b a c b a c a b c
2 2 2 2 2

IV. a,b,c,d a n , bn , cn , d n
(n

a,b,c a , b , c

a 2
 b2  c 2  a2  b2  c2    aa  bb  cc 
2

1 1 1
V. a,b,c a  b  c 1   0 a 2  b2  c 2  1
a b c
a 2  b2  c2
a,b,c a  b  c 1 ab  bc  ca  0 K
2003
VI. A 6 ,  3 ; B  2 , 5 ; C  4 ,  8

A
ABC
a2 x2
VII. ax  by  0  1
a 2  b2 x2  y 2
x xy y 2 y xy
  x, y x0
 x  y x y  x y x y

VIII. ABC A [AH] E D H [AB]


[AC] M [BC]
 ED   AM 
K [AM] [HD]  BK  ||  ED BEDK
M [EH] [AB] L  ML   AH  [BK]

3  619 

  
3
I. M
2 3 5
1
a b 
II. ab
1
1
a  b2
2

III. 218
 217  215  214   2x
 x  y  4 1
IV. 
 xy  z  6 z  13  2 
2

 1  1 1 1  1  1 1 1 
V. P         abc  10
 a  b  c  a b c  ab  bc  ac  ab ac bc 
VI. MNPQ 9x 2 ABCD
AM BM CP DP 1
    ABCD ?
AQ BN CN DQ 2

Q A M

D
២ ៤

P N
C

 620 

  
I. A  ab  a  b   bc b  c   ca  c  a 
II. A 2 , 2 ; B  3 ,  3 ; C  6 , 6
x y z
  
III.  5 3 4

 xy  yz  zx  21
IV. A, B,C AB  1 BC  x
5 1 BC AC
x 
2 AB BC
V. 10 m M
3m
5
VI. AB  9cm C D AB
2
VII. O [AB] [CD] (AB > CD) P H K
[AB] [CD]
I [OP] O,K,P,H
ˆ
HPO ˆ
KPO
PH PK

 621 

  

I. 220 1  5  1  20 6  2 5 3
3 3 3  3 3 3 3  23 3
1 1 1
x  y  z  2  x  xy  y  2  3 2

II.   2
 2  1 4  x  y  6
2

 xy z 2
III. x 2  px  1  0 a,b x 2  qx  2  0 b,c
b  a b  c   pq  6
IV. 10 20 1.70 m
1.61 m
V. ABCD  AC   BD , AC  20a , BD  12a M
AM 1 BN 2 AP 3
[AB] , N  BC  , P  AD  ,  , 
AB 2 BC 3 AD 4
MBNP S ABD  2S BCD

 622 

  

1
  
3
I. A 3 3
2 1 3
2 1
3

3  2 2  2 1
1
a b 
ab ,
1
 a  b  0
1
a 2  b2

ឃ B  6  6  6  ...
II. ២ ax 2  2bx  c  0 1
bx 2  2cx  a  0 2
cx 2  2ax  b  0 3  1  2  3 

III. a,b,c abc  1 S


1

1

1
1  a  ab 1  b  bc 1  c  ca
.
IV.
V. O [AB]  xy  A C
[AB] D E C [AB]  xy  M [DE]
AMO
M C
O C  xy  F
[CA) OCE
ឃ AME AFC

២ ២ ៤

3
 623 

  

I. M  a 2  b2  2ab  1 N  a 2  b2  1  2ab

P  4a 2b 2   a 2  b 2  1
2 P
M
N
4  3x 6 12 x  1
II. 3  
3x  2 3x  2 3  x  5
x x2  4
x 4  4 x 2  32  0
1 1 1
III. a,b,c x  y  z 1   0 x2  y 2  z 2  1
x y z
IV. MNP M’N’P’ MI M’I’
MI
M I 
V. O R  D A  D (AB)
(AC) O B C OE  D E BC OE
OA I H
OIH OEA OE  OI  OH  OA
OB2  OA  OH

 624 

២០១២-២០១៣

: ១២០ ១០០

EE o DD

I. 52013  52014  52015 31 ។



 , 2 3 ។
II. n 1  n n  n 1 ។

A  2012  2013  2013  2013  2014  4026 2013 ។


6  4 3  2 5  15
N ។
23 3  5
a  b  c  0
III. a,b c  ។
ab  bc  ca  0
 a 1  b2   c  1  2011 ។
14 2013

IV. 1  20122 20132  2  2012 ។


20122  4048144 20132  4052169
2
 2012  2012
A  1  2012     ។
2

 2013  2013
V. ABCD O ។ AC  BD  AB  CD  AD  BC ។
VI. O R AB AB  R AC ˆ  45o ។
BAC
ˆ
ACB ˆ ។
ABC
BC AC R។
2 3
cos 45o  , cos 30o  )
2 2

 625 

២០១២-២០១៣

: ១២០ ១០០

EE o DD

5 3 2 27  2 3 3 4
I. M  22  288 , N ។
4
9 3 16
 1 1
x y  x y  2 x y z
   
II.   5 3 4 ។
 3  4 7 
 xy  yz  zx  21
 x  y x  y

III. A  2012 5  3  29  12 5 ។
2a  3b 3b  4a a 2
IV.  ។ ។
3 2 b2
P   x 1 x  2 x  3 x  6 x P

V. ABCD A D (  AB ។

AC  AC   BC ។
ABC ។ B  AC  P។
PBA PBC ។
PB 2  PA  PC ។
BA2 PA
 ។
BC 2 PC

 626 

២០១២-២០១៣

១: ១ ០ ១០០

EE o DD

I. 1) A  20133  2013  2014  2012


2) B  1  2  2  4  5  6  7  8  9  ...  2014  2015  2016
x x  20132013
2014
3) x
2n
44  ។ n ។
4 128
4) 22
II. 1) 92009
9 2010
 92011  92012  92013 11 ។
2) a,b c abc a  b  c  1 ; ab  bc  ac  4 abc  2 ។
20131111  1 20132222  1
III. A B ។
20132222  1 20133333  1
13 2000 2013
 a  b  c
IV. a  b  c  ab  ac  bc ។
2 2 2
S  8   8   8   ។
 b  c  a
V. x 2  3x  1  0 x1 x2 ។

P
x 2
1  2x

x 2
2  2 x2 

 x1  1  x2  1
2 2

VI. a,b,c a 2  2b  1  0 ; b 2  2c  1  0 ; c 2  2a  1  0 ។
S  a 2013  b2013  c 2013 ។
VII. ABCD 5cm ។ O។ OA
OB x2   2m 1 x  4  m 1  0 ។
m ។
ABCD ។
VIII. O1 O2 P។ AB O2  O1 
C ។ PC O2 D។
PD APB ។
PC  AC  BC  PA  PB ។
2

 627 

២០១២-២០១៣ (១៤/០២/២០១៣)

២: ១ ០ ១០០

EE o DD

I.  x x ។ 2.63  2 ។
x  20132  2014  x ។
II. N  2010  2011 2012  2013 1 ។ N

III. 2ax  y  1 x4 ay  2 ។ a ។
x6 A B
IV. A B   ។ B A ។
x  2x x x  2
2

V. f f  x   2 f 5  x   x x ។ f 1 ។
VI. ( A  3 2 5  3 2 5 ។
VII. ( ABCD 42 cm 2 ។ E F AC ។
3 AE  FC   4EF ។ ។

B C
F
E

A D

 628 

(០៣/០៣/២០០ )

១: ១ ០ ១០០

EE o DD

I. x  x  4  4m  0 ។
2 2 6 2 2 6
II.  2 ។
2  2 3 2  2 3
3
III.  x 1 ។
1
x
x 1
x
3 x
IV. a  3 2  17 ; b  4  19 ; c  5  32 ។  cba ។

V. N ។

X X
X 34 ។ X

X+1 15 ។ X N។
VI. ABC A [AD] ។ O1 O2
ABC ACD ។  O1O2   AB  AC  K L។ S1 S2
ABC AKL ។ S1  2S2 ។

 629 

(១៧/០២/២០១០)

១: ១២០ ១០០

EE o DD
2010
I. ។
x  4x  7
2

1
II. A x  x
2

x 1
x xx x
x  14
III. x 5  3 ។
3 x 5
a 2  b2
IV. a b ។  ab ។
2
9992  1 0012  1 999 998 ។
V. O R  AB CD I។
IA2  IB2  IC 2  ID2  2R2 ។
VI. G ABC ។ G AB
AC BC P Q។ BP , PQ QC ។

 630 

(១ /០២/២០០៩)

២: ១ ០ ១០០

EE o DD
a c
I. a,b,c d ។ 
b d
 ab  cd    a 2  c 2  b2  d 2  ab  cd   a  c  b  d  ។
2

II. a b ។ a 2  b2  41 ។ a b ។
III. x y A  2 x 2  9 y 2  6 xy  6 x  12 y  30
។ ។

m2  x  2  m2  x  2 
2 2

IV. ( ។  2  2 x  m  1   m  1  1 ។
2
m
8 8
V. ABC BC  a , CA  b , AB  c ។ A BC
D។I ។

IA
DB , DC a,b c ។
ID
I BC AB E AC F។ EF ។
VI. O O AB = 2R OC AB ។ O
I  I OA J  J OB ។  OC

    O C I  M J  N។
C  x ។
OI R x ។ x
MN ។
S OI O ។ S
O   I  ។ S ។

 631 

(១៣/០៣/២០០ )

១: ១ ០ ១០០

EE o DD

I.
1 2  3
12 
6
2 35
12  2 2 
6
3 4  7
12  2 2  32 
6
...............................

13  1 23  1 33  1 20083  1
S    ...  ។
11 2 1 3 1 2008  1
II. p ។ p ។

III. 2008o ។ ។
1 1 1 1
IV. ( n    ...   43 ។
4 5 5 6 6 7 n  n 1
V. n 2
 
: 4 x  4 3  4 x  3n  24  0

។ n ។
VI. ABC ។

ABC ។
VII. 1 n។
2008 ។ ។

 632 

(១៣/០៣/២០០ )

២: ១ ០ ១០០

EE o DD
b c bc
I. a0 , b0   2008 ។ ។
a b ab
II.  x , y x 2  20082  y 2  2007 2 ។
III. ABC P ABC
PAB , PAC , PBA ។ P

3
IV. ( A B A។
8
100 km/h A។ A
A។ B B។
V. A, B, C
។ A
B C ៣។

16 $ ។ ។

VI. 12
។ 5
។ 9
10
។ 3

 633 

(១ /០២/២០១០)

១: ១ ០ ១០០

EE o DD
246912
I. A ។
123457  123456 123458
2

II. 2a3  326  5b9 2a3 5b9 5b9 9


a b ។
1666...66
III. F 2010 ។
6666...64
IV. ( * a * b  ab a b ។

a *b  b * a a *b * c    a *b * c

a * bn   a * n  * c  a *b  a *  bn  ( a, b, c, n ។
n

V. 9។ x 19 96
1 d
។ x ។ a b c

VI. 10 ។ 10
3 1 ។

VII. ABCD 9 ។ 9 a, b,
c, d, e, f, g, h i 1។
ABCD ។

7 634 

(១ /០២/២០១០)

២: ១ ០ ១០០

EE o DD

I. a 2  a  1 ។ A  a 4  2a3  4a 2  3a  3 ។
1 1 1
II.  x , y   ។
x y 3
III. n 0, 1, 2, 3, … n
2n  n 2 ។

IV. ( a b 92 a 1 b ។
V. ABC A ។ [AH] [AO] ( H
B O ។ HAO  ACB   BC = 2a ។ AH AO
 a។
VI. ABCD ។ AB B BE = AB ។ M
MAC  MCD   ។

AMC ។

AMCE ។ ។

 635 

(១ /០២/២០០៩)

១: ១២០ ១០០

EE o DD

I. ។ 8 4 ។

9 ។ ។

1 3 2 2 3
II. A  ។
2 3 3 2 2 3
 xy  100

III. 1 1 1 ។
x  y  5

IV. ( O M ។ M
M ។

V. ។

12 2 cm ។

 636 

(១ /០២/២០០៩)

២: ១២០ ១០០

EE o DD

A   x  1   3x  2     x  1   3x  2   ។
2 2 2
I.
 
A x ។
A
B ។
2x  2
x B 2 ។
x 2 x 2

II. x 2 x 2  2 ។
x 2
1
x 2
 2 1
 y  1  2x 1  1 1

III.  ។
 4  2 3  2
 2 x  1 y  1
5x  4 x6
IV. (  6x  2  4  ។
5 3
V. BAC ADC A D AC
។ AB  AC  a ។
ABCD ។ BC, AD BD a ។
O។ AOD COB
។ OA, OB, OC OD a ។
[BC] [BD] K។
[CK] BCD ។ DK OK ។

 637 

(២០០៩-២០១០)

១: ១២០ ១០០

EE o DD
4 40
I. f  x   x2  x  ។
3 9
a  b 1 ។ P  2  a3  b3   3  a 2  b2   1 ។
II. 2  x  y   16  3xy ។

III. A  3 7 5 2  3 7 5 2 ។
333
IV. ( a b  a  b 9
63 ។
V. ABCD [AB] ។ A D O H
K O [AB] [CD] ។ AD = AH + DK ។
VI. a, b, c ABC O R [AH] S
ABC ។ abc  4RS ។
VII. O [AB] ។ C ។M N
A B C។D C AB ។
CD2  AM  BN ។

 638 

(២០០៩-២០១០)

១: ១២០ ១០០

EE o DD

I. a b ។

A a  b B  a  b  2 ab ។
ab a 2  b2
A B ។
2 2
II. m, n, p q
m 2  2n  1  0 , n 2  2 p  1  0
។ I  m 2010  n 2010  p 2010  q 2010 ។
p  2q  1  0 , q  2 m  1  0
2 2

x 9 x 8 x 7 x 6 x 5
III.     5 ។
2009 2008 2007 2006 2005
IV. ( x1 , x2 a 2  pa  1  0 y1 , y2
b 2  qb  1  0  x1  y1  x2  y1  x1  y2  x2  y2   q2  p2 ។

V. a, b, c
a2  b2  c2  2  ab  bc  ca  ។
VI. 218  220 5 ។

VII. 1  2000  2
 2001 2
 2  2000  ។
VIII. 100 ។

1 20 100 ។ ។

IX. ABC [AD) BAC ។ BC D។


AB DB
 ។
AC DC
X. ។ A B ។

។ PmUi A 

 PmUi B
7  639 

(១ /០២/២០១០)

១: ១ ០ ១០០

EE o DD
1 1
I. 2
x 5 x2
II. 4
49  5 24  4 49  5 24  2 3 ។
III. A  2008  2010 B  2 2009 ។
1 2 3 2009
IV. ( A    ...  ។
1 2 1  2  3 1  2  3  4 1 2  3  ...  2010
1
A  1 ។
1 2  3  ...  2010
V. x2  y 2  1 bx 2  ay 2 a0 , b0 ។
x 2010 y 2010 2
 1005  ។
 a  b
1005 1005
a b

VI. ( a,b c
a  b  c  1 , a 2  b 2  c 2  1 , a 3  b3  c 3  1 ។
A  a 2008  b2009  c2010 ។
VII. ( O1 r1 O2 r2 M។ O3 r3
O1 O2 ។ M O1
O2 O3 A B។I [AB] ។
4r3 r1r2
AB  ។
r1  r2

 640 

(១ /០២/២០១០)

១: ១ ០ ១០០

EE o DD

2 2
I. A A ។
12  8  3  6
II. n x2  x  6 n2 x ។

III. 7h30mn A B 5 km/h ។ B


1h A 20 km/h ។ -

17 km ។ A 13h6mn ។
A B ។
IV. ( 1  a  b  c  a 1 bc ។ ba ។
V. ABC A  90o AB  AC  1 ។ AC C
CD  BC ។ BCD ។

VI. ( ABC A [BN] ។ AC D


CD  CA ។ BD  2BN ។

 641 

(២២/០៥/២០០៩)

១: ១ ០ ១៥០

EE o DD

1
  
3
I. 2  2 x1 4x  4x 3 3
2 1 3
2 1 ។
3
II. a, b, c a bc ; b  ac c  a b a 2  b2  c 2  2 ។
a 2  b2  c2  2abc  2 ។
III.  x 1 x  2 x  3 x  4  3 ។
IV. ( 72016 ។
V. a, b c
a 2  2b  1  0 ; b 2  2c  1  0 ; c 2  2a  1  0 ។ S  a 2009  b2009  c 2009 ។
VI. ( 10 m M
3m ។ M។
VII. ( ABC A។ A AB [BC] [AC] D
E។ BD = 20 , DC = 16 ។ AC2 ។
VIII. ABCD ។ D DA [BD] X។
B BC [BD] Y។ XY = 2
AXC AYC ។
IX. X, Y, Z
Z X Z X។
X Z Z X។
X Z។
1
Y ។
4

 642 

(២៧/០២/២០០៩)

១: ១២០ ១០០

EE o DD

1
I. 13 x 3  2 x 3 x  20 ។
x
II. 12 cm 8 cm។
x cm
។ x
64 cm 2 ។
III. a, b, c a3  b3  c3  3abc  0 ។
IV. ( B  a 2  b2  ab a  b 1 ។
V. a, b, c ។ x, y, z ។
1 1 1 1 1 1
     ។
x y z a b c
VI. ( A។ M [BC] ។
MB2  MC 2  2MA2 ។

 643 

(២៧/០២/២០០៩)

២: ១២០ ១០០

EE o DD

 1 3
2 x  y  x

I.  ។
2 y  1  3
 x y
II. f  n   32n  7 ,  n   ។ f  n  1  f  n  8។
n n 2 n3
III. n E   ។
3 2 6
1
IV. ( ។
8
20%
។ ។

V. 13 cm 17 cm ។

VI. ( ABC ។ AB , BC CA BB '  AB , CC '  BC


AA '  CA ។ S A ' B 'C '  7 S ABC ។

 644 
sYsþI¡ elakGñkmitþGñkGanCaTIRsLaj;rab;Gan enAkñúgEpñkenHelakGñknwg)aneXIj BIlMhat;l¥² nig
cemøIyrbs;vaCaeRcIn Edlbgáb;eTAedayviFIsaRsþedaHRsayl¥² RbkbedayKMnitEbøk² . ´Føab;erobcMEpñkenH
rYcehIykalBImun EtmanlMhat;cRmúHfñak;rhUtdl;fñak;TI 12

critlkçN³enAkñúgEpñkenH KWEckecjCaBIrRkumrYmmman RkumRbFanlMhat;l¥² nigRkumcemøIyén


lMhat;l¥² EdlpÁÚpgÁ KñaeTAedayelxerogBIxagmuxcab;BI 1 rhUtdl; 400 . sUmkMuBüayamedaHRsaylMhat;
TaMgenHtamlMdab;lMeday ebIcg;eFVIlMhat;Na cUreFVlI Mhat;enaHEtmþgeTA .
RbsinebIelakGñkmanbBaða b¤cm¶l;Rtg;cMNucNa EdlmanenAkñúgEpñkenH elakGñkGacTak;Tg;eTAkan;
RKU b¤mitþPkþirbs;GñkEdlmansmtßPaB b¤GñkeroberogesovePAenHkñúgeBlevlaGñkKitfaKYr .

vii
lMhat;l¥²
1> rkRKb;KUéncMnYnKt;viCm¢ anén a , b EdlepÞógpÞat; TMnak;TMng ³ a  b  a  b  8 . 2 2

2> RsaybBa¢ak;fa E CakaerR)akd Edl E  x  1000 x  2000 x  3000 x  4000   1000 . 4

3> rkelxenAxÞgr; ayéncMnYn ³ A  3  7  3  7 . 2011 2012 2013 2014

4> KNnakenSam ³ P  3  2 2  3  2 2 .
17  12 2 17  12 2
5> KNnatémø x edIm,I[ F mantémøGb,brma ¬tUcbMput¦ Edl ³ F  x 1x  2x  3x  6 .
6> KNna x  y edaydwgfa x  y  8100 nig x  y  30 .
2 2 3 3

7> RsaybMPøWfa ³ 11 2  21 3  31 4  ...  n  n1  1  1 Edl n CacMnYnKt;viCm¢ an ehIyFMCag 1 .
8> eK[ n CacMnYnKt;viC¢man Edl n Ecknwg 7 [sMNl; 5 ehIy n Ecknwg 8 [sMNl; 3 .
k> etI n Ecknwg 56 [sMNl;b:unµan ? x> rkcMnYn n enaHedaydwgfa 5616  n  5626 .
2 2

9> eK[sVIútcMnYnBit a  1  1  1n  


n 1  1   Edl n  1
 1
   n

bgðajfa A  a1  a1  a1  ...  a1 CacMnYnKt; .


1 2 3 20

10> KNnatémøelxénkenSamelx A  182  33125  182  33125 .


3 3

11> eK[BIrcMnYnKt;viC¢man x , y Edlman 2 xy  y  4 x  2 x  1 . KNna A  x  y . 2 2011 2011

12> eK[ f x  x 13 x  13 x 13 x  ... 13 x  13 x 1 . KNna f 12  .


2011 2010 2009 2008 2

13> brimaRténRbelLÚRkam ABCDmanrgVas;esµInwg 48cm . rgVas;km<s;énRbelLÚRkamenHman


smamaRt 5 : 7 . KNnargVas;RCugénRbelLÚRkam ABCDenaH .
14> rksmIkaremdüaT½rénGgát; AB edaydwgfa A2 , 3 nig B4 , 4 .
15> fg;mYymanXøIBN’Rkhm 2 XøIBN’elOg 4 nigXøIBN’exov 6. eKcab;ykXøI 2 edayécdnü. rkRbU)ab
cab;)anXøI 2 minEmnBN’exov .
KNnakenSam A  126 12  18  ... 96
2
16> 
24  36  ... 192 4
3
. 2

17> eK[ x  y  0 nig 2 x 2


 2 y 2  5 xy KNna E  xx  yy .
18> sRmYlkenSam E  2x  y  2x x  y  x  y  x  y x  y  Edl x  y  0 .
2 2

19> edaydwgfa sin a  cos a  2 sin 45  a  nig 3 cos a  sin a  2 cos30  a  .
o o

cUrKNnaplKuN ³ k> A  1  cot1 1  cot 2 1  cot 3  1  cot 44 


o o o o

x> B   3  tan1  3  tan 2  3  tan 3   3  tan 29  .


o o o o

 645  -
20> ebIeK[bIcMnYnBitviC¢man a , b , c . cUrbgðajfa ³ a  bb  cc  a  8abc .
21> eK[bIcMnYnBitviC¢man a , b , c . bgðajfa ³ a  b  c  ab  bc  ca .2 2 2

22> rkb¤sKt;viCm¢ anénsmIkar ³ 2x  y   16  3xy .


23> RsaybBa¢ak;fasmIkar ³ x  ax  b  x  bx  c  x  cx  a  0 manb¤sCanic©
EdlenAkñúgsmIkarenHman x CaGBaØat nig a , b , c CacMnYnBit.
x
24> eK[TMnak;TMng ³ f x   4 4 2 . x

k> KNna f x  f  y  edaydwgfa x  y  1 .


x> TajrkplbUk S  f  2011 1 
 f 

 2 
 2011 
 f 
 3   2010 
  ...  f 
 2011 
 .
 2011 
25> KNnalImIt ³ lim x .
x0
x

26> KNna ³ A  2010 2011  2011  2011  ...  2011  2012  1 .


2010 2009 2008 2

27> eK[BIrcMnYn a nig b EdlepÞógpÞat; a b  1 . bgðajfa a  ba  b a  b  a  b   0 . 2 2 4 4 8 8

28> KNna A  1  12   1  13   1  14   ...  1  2011
1 
 .

29> edaHRsaysmIkar ³ 4x  5x  1  0 .
4 2

30> ebI abc  1 sUmbgðaj[eCOfa ab 1a  1  bc 1b  1  ca 1c  1  1 .


31> KNna S  2 1 1 1 2  3 2 1 2 3  4 3 1 3 4  ... 100 99 1 99 100 .
32> sUmbgðaj[eKeCOfa ³ 2002  2002 Eckdac;nwg 6 .
2003 1979

33> RKb;cMnYnBit a , b , c EdlepÞógpÞat; a  b  c  1 . bgðaj[eKeCOfa a  b  c  13 . 2 2 2

34> RKb;cMnYnBit a , b , c ,..., x , y viC¢man nigxusBI 1. sUmbgðaj[eKeCOfa ³


log b  log c  log d .... log y  log a  1 .
a b c x y

35> eyIgman f x  log 11  xx  . bgðajfa f x  f  y  f  1xxyy  .
 
36> edaHRsaysmIkar  7  48    7  48   14 .
x x

37> RKb; a nig b CacMnYnBitviC¢man nigxusBI 1. sUmbgðajfa ³


log a b  log b a  2log a b  log ab b   log b a  1  log a b .
38> sUmbgðajfa ³ 11  cos
cosa 1  cosa 4 cot a
 
a 1  cosa sin a
.
39> edaHRsaysmIkar ³ 5  50  x . log x log 5

 646  -
40> eKe)aHRKab;LúkLak;mYyEtmþgKt; . KNnaRbU)abEdle)aH)anelxess b¤ elxFMCag 5 .
41> edaHRsaysmIkar x  4x  16x  ... 4n x  3  x  1 .
42> cMeBaHRKb;cMnYnKt;viC¢man n eK[ f n  1 . KNnaplbUk
3
n  2n  1  n 2  1  3 n 2  2n  1
2 3

f 1  f 3  f 5  ...  f 999997   f 999999  .


 x y
5log y x  log x y   26
1
4  5  400
43> edaHRsayRbB½n§smIkar ³ a/  b/  .
 xy  64 5 x  6 y 
1
 900
44> eK[ a nig n CacMnYnviC¢man ehIy n Kt; EdlplKuN a  a 1 2  a3  ...  an  1 . sUmbgðajfa
1  a1 1  a2 1  a3 ...  1  an   2 n .
 7 x 3  3x 2 y  21xy 2  26 y 3  342 4 x  3 y 1  27 y  171
45> edaHRsayRbB½n§smIkar a/  3 b/  x 1 2 y
.
9 x  21x y  33xy  28 y  344 8  2  3  172
2 2 3 x

46> cMnYnmYymanelxR)aMxgÞ ;EdlelxxÞg;vaerobtamlMdab;KW x , x  1 , x  2 , 3x , x  3 ehIyeKdwgfa


cMnYnenaHCakaerR)akd . cUrrkcMnYnenaH .
47> KNnaplbUk S  1 1 2  2 1 3  3 1 4  ...  2024 1 2025 .
48> RsaybBa¢ak;fa ³ f n  3  7 Eckdac;nwg 8 Edl n CacMnYnKt; .
2n

49> man f x  2x  1 nig g f x  x  3x  1 . sUmKNna g 3 .


2

2 3
50> RKb;cMnYnKt;viC¢man n RsaybMPøWfa n3  n2  n6 k¾CacMnYnKt;viC¢manEdr .
51> KNnaplbUk S  11 2  21 3  31 4  ... 20101 2011 .
52> eK]bmafa x  x 1  0 . cUrKNna x  x 1 .
2 2011
2011

53> cMnYnmYyCakaerR)akdmanrag abcd . edaydwgfa ab  cd  1 cUrrkcMnYn abcd enaH .


54> sUmbgðajfa n! 3 cMeBaHRKb;cMnYnBitviC¢man n  7 .
n

55> GnuKmn_ f kMNt;eday f x  a  12  f x  f x cMeBaHRKb; x  0 nig a  0 .


2

sUmbgðaj[eXIjfa f CaGnuKmn_xYbelIcenøaH 0 ,   .
56> cMeBaH n! n  n 1 n  2 ...  3 2 1 ¬ n! Ganfa n hVak;tUErül¦. bgðajfa 0 ! 1 .
21n  4
57> bgðajfa F  14 n3
CaRbPaKsRmYlmin)an cMeBaHRKb;cMnYnKt;FmµCati n .
58> rkcMnYnKt;FmµCatitUcbMput Edlman 6 CaelxenAxÞg;Ékta ¬xÞg;ray¦ ehIyebIbþÚrelx 6 eTAxagmuxeK
bMputvij enaHeKnwg)ancMnYnfµI EdlesµInwg 4 dgéncMnYnedIm .

 647  -
59> eK[lkçxNÐbI EdlbursbInak; eTAsþIdNþwgnarIbInak; 1, 2 , 3 CaKUGnaKt dUcxageRkam ³
A, B , C

k> ebIburs A CaKUnwgnarI 1 enaH burs B CaKUnwgnarI 2


x> ebIburs A CaKUnwgnarI 3 enaH burs C CaKUnwgnarI 1
K> ebIburs B minCaKUnwgnarI 3 enaH burs C CaKUnwgnarI 1 .
sMNYrsYrfa etIbursNa CaKUnwgnarINa ?
60> rk 11 cMnYnminGviC¢man EdlcMnYnnImYy² esµInwgkaerénplbUkén 10 cMnYnepSgeTot .
abc  0
61> eK[ a , b , c CacMnYnBit epÞógpÞat; ab  bc  ca  0
. KNna A  a  1  b  1  a  1 . 2010 2011 2012

62> KNnaplbUk S  1
1
12
1 1 1
 2  1  2  2  ...  1 
2 2 3
1
2010 2

1
2011 2
.
63> edaHRsaysmIkarman x CacMnYnKt;³ k> x 2010
 x 2011  x 2012  x 2013  4

x> x  x  x  x  0 . 2010 2011 2012 2013

64> edaHRsaysmIkar ³ 2 x9 y  2  9 . ¬eKsresr 2 x9 y bB¢ak;faCaelx4xÞg;mann½yfa 0  x  9 , 0  y  9 ¦ .


x y

65> rkRKb;cMnYnKt;FmµCati N  a a a ...a EdlnaM[ 2a a a ...a 1  12


1 2 3 n
21
. 1 2 3 n

1a a a ...a 2 1 2 3 n

66> KNnaplbUk ³ S
1

1

1
1 2  3 2  3  4 3  4  5
 ... 
1
nn  1n  2
.
67> sRmYlkenSam ³ 2  3   2  3   3 .
2011 2012

68> sUmbgðaj[eXIjfa x 11  x 1 1  x 2 1  x 4 1  x 81 .


2 4 8

69> cMeBaH n! n  n 1 n  2 ...  3  2 1 . KNnaplbUk S  21!  32!  43!  ...  n n 1! .
70> kMNt;elxxagcugénplbUk ³   1  2  3  ... 2010  2011 .
2 2 2 2 2

71> kMNt;elxxagcugénplbUk ³   1!2!3!... 2010!2011! .


72> eK[ a  a  1b  b  1  1 . KNnatémø A  a  ab .
2 2 2012 2011

73> edaHRsaysmIkar ³ 2011  x log 2012 x


 4022 .log 2012 2011

74> eK[ ab  1 . RsaybBa¢ak;[eXIjfa ³ a  b  a  b a  b   a  b .


5 5 3 3 2 2

x 2  2 y  1  0
 2
75> eK[ x , y nig z EdlepÞógpÞat;  y  2z  1  0 . KNnakenSam ³
A  x 2010  y 2011  x 2012 .
 z 2  2x  1  0

ad  bc
76> BIsmamaRt a c

b d
. bgðajfa 2ab

2cd
ad  bc
.
1
 a  a  a 2  b 2 
77> a nig b CaBIrcMnYnKt;viC¢man. cUrsRmYlkenSam ³ A   
b
 2  
b
 2   .
 b a  b a  2011 ab 

 648  -
78> eK[ a  b  c  1 , a  b  c  1 nig ax  by  cz  m . cUrKNna
2 2 2
P  xy  yz  zx .
79> eK[RtIekaNsm)aTmYy EdlmanmMumYymanrgVas;esµI  
nigRCugBIrEdlGmnwgmMuenH manrgVas;esµIKña ehIyesµI a . a

KNnaRkLaépÞRtIekaNenH CaGnuKmn_én  nig a .


80> eRbóbeFobcMnYn k> 6  5 nig 21 x> 1  11  72 nig 2 3  2 . 2011 2011

81> rktémøKt;én x , y nig z EdlepÞógpÞat;smIkar ³ x  y  z  4  2 x  2  4 y  3  6 z  5 .


ax 3  by 3  cz 3

82> eK[  1  1  1 1 . RsaybMPøWfa 3
ax 2  by 2  cz 2  3 a  3 b  3 c .
 x y z

83> RsaybBa¢ak;fa TIRbCMuTm¶n; G p©itrgVg;carwkeRkARtIekaN O nigGrtUsg;énRtIekaN H énRtIekaN


mYy CabIcN
M ucrt;Rtg;Kña .
84> eK[ P  a b  5a  9b  6ab  30a  45 . cMeBaHRKb;témøéncMnYnBit a nig b bgðajfa P  0 .
2 2 2 2 2

85> sUmbgðaj[eXIjfa 39  51 Eckdac;nwg 45 .


51 39

86> KNna A  10110001100000001 ...1000...001 .


man 2 n
 1 dgénelx 0
6n  3 300
87> KNnatémø n EdlnaM[ 3

9

2011 2011 2011
15
 ...  
2011 2011
. 1 1 1 1
x2  2x  y  a  b  c   12
 2 
1 1 1 7
88> edaHRsayRbB½n§smIkar k>
y  2y  z x>     .
 z 2  2z  x  c b a 12
  111 5
x  4 x  3 x  2 x 1 
 a c b 12
89> edaHRsaysmIkar³   
2007 2008 2009 2010
4 .
8 2 1
90> RsaybB¢aak;fa .
4
1

4
8 2 1  4
8 2 1 2

91> sRmYlkenSam B   2 xx



1  
  1 
x 2 
 .
 x x 1 x  1   x  x  1 

92> eK[bIcMnYnrYmman ³ A  888...88 B  222...222 nig C  444...444


man n tYénelx * man n  1 tYénelx @ man 2n tYénelx $
RsaybB¢ak;[eXIjfa A  B  C  7 CakaerR)akd .
93> kalNa)aténRtIekaNmYyekIneLIg 10% etIkm<s;énRtIekaNenaHRtUvfycuHb:unµanPaKry edIm,I[va
enAEtrkSaRkLaépÞ[enAdEdl .
94> eK[ a , b nig c CargVas;RCugénRtIekaNmYy.Rsayfa ³ ab  bc  ac  a  b  c  2ab  bc  ac . 2 2 2

 649  -
95> eK[bIcMnYn a , b nig c Edl a  b  c  0 nig abc  0 . KNna E Edl
E
2011
b a c
2 2 2
 2
2011
b a c
2 2
 2
2011
a  b2  c2
.
96> bgðajfa 2010  2012 2011  12011  12011  12011  1  2011  1 .
2 4 8 16 32

97> eK[ ABC CaRtIekaNmYy Edlman AM  Caemdüan . bMPøWfa 2 AM  AB  AC .


98> RtIekaNmYymanrgVas;RCug x , x  a nig x  2a Edl x  0 nig a CacMnYnKt;viC¢man . rkRKb;rgVas;
RCugénRtIekaNenH EdltUcCag b¤esµI 10 edIm,I[vaCaRtIekaNEkg .
99> RtIekaNmYymanrgVas;RCug 3 , 4 , 5 . RtIekaNenHCaRtIekaNcarwkkñúgrgVgm; Yy . tag A , B , C
CaRkLaépÞEpñkbøg; EdlenAkñúgrgVg; nigeRkARtIekaN ehIy C CaEpñkRkLaépÞEdlFMCageK . KNna
A  B CaGnuKmn_nwg C .

100> eK[ctuekaN ABCD CactuekaNBñay nigman MN Ca)atmFümén A 6 cm B


ctuekaNBñayenH. KNnaRbEvg)atmFümenAcenøaHGgát;RTUg x dUcrUb? M x
N
C
ebIeKdwgfa )attUc AB  6 cm nig)atFM CD  10cm . D
10 cm
2 2 2 2
101> cUrKNnaplbUk S  11 3  32 5  53 7  ...  1999
1000
 2001
.
102> RsaybBa¢ak;[eCOfa 6  6  6  ... 6  30  30  30  ... 30  9
man n r:aDIkal; man n r:aDIkal;
103> k> cUrKNnaplbUk S  1 2  3  2  3  4  3  4  5  ...  nn  1n  2
x> RsaybBa¢ak;fa 4S 1 CakaerR)akd cMeBaHRKb;cMnYnKt;viC¢man n .
104> eK[ 0  x  x  x  ...  x . RsaybBa¢ak;fa x  xx  xx  ...x  x  x  xx  xx  ...x  x
1 2 3 12
1 2 3 9 1 2 3 12
7 .
3 6 9 4 8 12

105> ebI x , y CacMnYnBitEdlepÞógpÞat; x  3xy  y  60 . rktémøFMbMputénplKuN xy .


2 2

106> rkplbUkrgVas;mMu ¬KitCadWeRk¦énmMu A, B , C , D , E nig F C F

enAkñúgrUbxagsþaM . A B
E
D
107> RsaybBa¢ak;facMnYn A  111...11222...22 EdlmancMnYntYénelx! nigelx@ esµIKñaKW 2011tY.
RsaybBa¢ak;fa A CaplKuNénBIrcMnYnKt;tKña .
108> ]bmafa x , x Cab¤sénsmIkar ³ 2011 x  t  2011 x  2011  0 . rktémøtcU bMputénkenSam ³
1 2
2

2
x x 1 1
H  x  x   4    .
2 1 2
2 1
2 x x  1 2 

109> KNnatémøelxén A  3 5 3 5 3... .


 650  -
110> eKmanBhuFa . eKdwgfasMNl;énviFIEckrvag px nig x esµI 1 ehIysMNl;énviFIEckrvag px
p x 

nig x 1esµI 2 . rksMNl;énviFIEckrvag px nig xx  1 . ¬GñkGaceRbobeFoCamYylMhat;TI8 TMBr½ TI!¦


111> RsaybBa¢ak;facMnYn 5  5  5 Eckdac;nwg 31 .
2011 2012 2013

112> rkelxéntYcugeRkaybg¥s;éncMnYn 2 . 34

113> rkelxéntYcugeRkaybg¥s;éncMnYn 123456789 2011

114> RsaybBa¢ak;facMnYn A  13  2  1 2  1 CacMnYnKt;viC¢man .


3 3 3
3

115> bBa¢ak;témøéncMnYnBit x edayeRbIsBaØavismPaB. edaydwgfa x CacMnYnEdlmanEpñkKt;manelx


BIrxÞg; kalNaeKsresrvaCacMnYnTsPaK.
116> k> eRbóbeFobcMnYn 1 2000  nig 2001  2  2000 
2 2

x> edaymineRbI 2000  4000000nig 2001  4004001KNna A  1  2000   2000


2 2

2001 

 
2000
2001
. 2

2 2 2

117> eK[TMnak;TMng x  x y  y  x y  a . RsaybBa¢ak;fa a  x


2 3 4 2 2 3 2 4 3 3
y 3
.
118> eRbóbeFobcMnYn 2001  2002 nig 2 2002 .
119> RsaybBa¢ak;fa ³ A  111...111 444...44 1 CakaerR)akdéncMnYnKt; .
2n énelx! n énelx$
 1 1  1 1 
120> sRmYlkenSam ³ E  1
     
2  Edl  p , q  0 .
 p  q 2
 p q  3 
p q  p 
q 

121> KNnatémøelxénkenSam ³ A  3  3 9
125 3
27
 3 9
125
27
.
122> RsaybBa¢ak;fa ³ 3 3
2 1  3
1 3 2 3 4
9

9

9
.
123> KNnatémøelxénkenSam S  6  6  6  6  ... .
bc
124> ]bmafa a  0 , b  0 nig ba  bc  2011 . KNnatémøén ab
.
x  1 x  5 x  7 x  11
125> edaHRsaysmIkar ³ 1991   
1987 1999 1981
4 .

126> edaHRsaysmIkar ³ x  6 x   2  3 x .
2000 1998 1999

127> edaHRsaysmIkar ³ log log log x  2 .


2 3 4

128> KNnaplbUk A  111111 ...111...11 .


n énelx!

129> edaHRsaysmIkar ³k> x x


 xx x> x  x x
 x x 
x
.

 651  -
a  b 2  2ac  29

130> eK[ a , b , c CacMnYnBitEdlepÞógpÞat;RbB½n§smIkar  b  c  2ab  18
2
. cUrKNna a  b  c .
 c  a 2  2bc  25

131> KNnaplbUk S  1 2  3  ... 2011 . ¬edaHRsayBMutamrUbmnþsVúIt¦.
132> fñak;TI 9B mansisSsrub 28 nak;. sisSRbusmankm<sm; Füm 1.68 m nigsisSRsImankm<s;mFümKW
1.60 m . ebIdwgfakm<s;mFümrbs;sisSenAkñúgfñak;TaMgGs;KW 1.66 m . rkcMnYnsisSRsIenAkñúgfñak;.
133> KNnakenSam A  a  b  c    a  b  c   a  b  c   a  b  c  .
2 2 2 2

134> eK[ a  b  c  0 . bgðajfa b 2011


c a
 2
2011
c  a b 2

2011
a b c
2
0 . 2 2 2 2 2 2

135> kMNt;témø a nig b edIm,I[BhuFa x  2x  ax  bx  1 CakaerénRtIFadWeRkTI@ x  px  q .


4 3 2 2

136> cUrRsabMPWøfa 49  20 6  49  20 6  2 3 .
4 4

137> KNna E   3  5  3  5    3  5  3  5  .


2 2

138> kMNt;témø m nig n edIm,I[smIkar x 1 3  x m 4  x  3n x  4 manb¤seRcInrab;minGs; .


139> kMNt;témø a edIm,I[RbB½n§smIkar 2xxyya3 manb¤sepÞógpÞat; x  y .

140> eK[ a , b , c epÞógpÞat; a  b  c  0 nig ab  bc  ca  0 . KNna D  a  1  b  1  c  1 . 2010 2011 2012

141> KNna S  1 3  2  4  3  5  ...  2010  2012   1  2  3  ...  2010  . 2 2 2 2

142> bgðajfa A  9  8  7  6  5  1  2  3  4 Eckdac;ngw 5 .


2011 2011 2011 2011 2011 2011 2011 2011 2011

143> eK[GnuKmn_BIr f nig g kMNt;eday ³ f : x  f x  px  2 Edl p Ca):ar:aEm:RtBit nig


g : x  g x   4 x  3.
k> KNna f  g nigKNna g  f . x> kMNt;témø p edIm,I[eK)an f  g  g  f .
144> eKman P  cos 2x cos 2x cos 2x  ... cos 2x . bgðajfa P  21  sin xx .
n 2 3 n n n
sin
2n
145> edaHRsayRbB½n§smIkarkñúg  énRbB½n§smIkar  x  y  5440
2 2
.
PGCDx, y   8
146> edaHRsaysmIkarkñúgsMNMucMnYKt;FmµCati énsmIkar xy  3x  2 y  37 .
147> bMPøWfacMnYn N  4n  3  25 Eckdac;nwg * .
2

148> RsaybMPøWfa ebIeKmansmPaB a  b c  d   ad  bc enaHeKRtUv)an a , b , c nig d CatYén


2 2 2 2 2

smamaRtmYy.
a2 x2
149> eK[ ax  by  0 . bgðajfa 
a2  b2 x2  y 2
1 .

 652  -
150> eK[ a  b  c  0 nig Edl a  0 , b  0 , c  0 . KNnatémøén K  a 2011b c
.
2 2 2
1 1 1
  0
a b c
151> bgðajfa A  2011 5  3  29  12 5 CacMnYnKt; .
152> edaHRsaysmIkar x  4x  4  x  2x 1 .
2 2

153> KNnabIcMnYnKt;tKña ebIeKdwgfa plEckénplKuNcMnYnTaMgbI nwgBak;kNþalkaerénplbUkcMnYnTaMgbI


esµI 130
21
.
154> ebI x yz  7 nig xy  7 . cUrKNnatémøelxén xyz .
2 3 3 2 9

155> rktémøelxén 22  22 .
2012 2011

2012 2011

156> cMeBaH a  b  c KNnakenSam H  a  baa  c   b  cbb  a   c  acc  b .


2 2 2

157> eKdaMkUneQI 40edIm B½T§CMuvijcmáarmYymanragCactuekaNEkg. etIcmáarenaHGacmanRkLaépÞFMbMput


b:unµan ? ebIeKdwgfa KmøatBIedImeQImYyeTAedImeQImYyeTotmanRbEvg 2 m .
158> bgðajfacMnYn 3 BMuEmnCacMnYnsniTan. ¬cMnYnsniTan CacMnYnmanTRmg;CaRbPaK ba Edl >>> ¦ .
159> ebI x nig y CacMnnY Bit cUredaHRsaysmIkar x  y  0 . 2 2

160> KNna A  2 34 B  2
3
C  2
3 3
D  2  E  2
3
.
4 4 4 4
161> KNna A  2 22
B  2  C2 2 2
D2 22
E  2 2 . 2 2 2

162> sresrCaTRmg;RbPaKén A  1.2 B  0.12 C  0.12 D  1.20 .


163> eKmankaermYymanRCug a . KNnaGgát;RTUg nigcm¶ayxøIbMputBIcMNucRbsBVrvagGgát;RTUgeTARCugkaer.
164> eKmanRtIekaN ABC EkgRtg;kMBUl B ehIymanRbEvgRCug AB  1. A
1 2
D CacMNucmYyenAelIRCug BC  Edl AD  CD  2 . KNna AC . B D
2 C

165> edaHRsaysmIkar  2  3  2  2  3  2  2  3  x  2011  0 .


 
166> edaHRsaysmIkar x  2 x  4x  2 x  3  x  2 x  7 .
2 2 2

167> ]bmafasmIkar ax  bx  c  0 manb¤s x nig x . tag S  x  x Edl n CacMnYnKt;FmµCati.


2
1 2 n
n
1
n
2

RsaybBa¢ak;fa aS  bS  cS  0 .
n 1 n n 1
2
 2y 2
x  2y 2 2 2
168> eK[ x 306 
294
. KNna x
y
. 2

169> KNna A  x  2  2x  2x  8  x  8 cMeBaH x  2011 .


2 2 2012

170> eK[ a , b nig c CabIcMnYnxusBIsUnü Edl a  b  c  1 nig 1a  b1  1c  0 .


RsaybMPøWfa a  b  c  1 .
2 2 2

 653  -
171> rkBIrcMnYnBitEdlmanplbUkesµI 13 ehIyplbUkcRmasrbs;vaesµInwg 13
40
.
172> eK[smIkar x  1x  2x  3x  4  k .
k> edaHRsaysmIkar cMeBaH k  3 . x> kMNt;témø k edIm,I[smIkarmanb¤s .
173> eRbóbeFobBIrcMnYn ³ 200 nig 300 . 300 200

174> eRbóbeFobcMnYn 31 nig 17 .


11 14

175> ebI a Cab¤sénsmIkar x  1 ehIy a  1 . cUrKNna 1  a  a  a    a .


2011 2 3 2010

176> KNnaplbUk S  3  3  3  ... 3 . ¬BMuedaHRsaytamrUbmnþsVIútFrNImaRt¦.


2 3 n

177> KNna A  1  21 1  31 1  41   ...  1  n1  .


     
2 2 2 2

178> kMNt;témø m énRbB½n§smIkar 23xx  4yym7 edIm,I[vamanb¤s nig y  0 . x0



179> KNna E 1  2  3  ...  2009  2010  2011  2010  2009  ...  3  2  1 .
a  b  c  3
b  a  d  4

180> edaHRsayRbB½n§smIkar  .
a  c  d  5
b  c  d  6

181> bgðajfa 7 3 7 4 7 7 ehIy 4  4  4  4 . 3 4

182> eK[BhuFa f x   x  4 x  1 Edl n CacMnYnKt;FmµCati ¬ n  ¦. kMNt;témø n edIm,I[viFIEckén


n 2

BhuFaxaelI nigBhuFa x  3 mansMNl;esµI 46 .


183> edaHRsaysmIkar ³ x
x
1 .
2
x
2
x
2
x
2
1 1 x
184> kMNt;RKb;cMnYnKt; n EdleFVI[ B  4  4  4 CakaerR)akdénmYycMnYnKt; .
27 1016 n

185> edaHRsaysmIkar x xx  1  x xx  1 .


4 2 2

1 1 1
 x  y  z 3
x  y  z  6 

186> edaHRsayRbB½n§smIkar ³ k>  xy  yz  zx  12 x> 1
 
1

1
3 .
2 2 2  xy yz zx
x  y  z 3  1 1
  xyz

187> eK[ 4  4  23 . KNna 2  2 .
a a a a

188> rkcMnYnlT§PaBTaMgGs; kñúgkarbegáItelxTUrs½BÞ Edlmank,alRbB½n§ ³ k> 011 x> 097 .


 654  -
189> edayeRbIelxTaMgbYnman ³ 1 , 2 , 4 nig 9 . etIeKGacbegáIt)anelxbYnxÞg; b:unµanrebobxusKña?
190> ]bmafasmIkar x  px  1  0 manb¤s a , b nigsmIkar x  qx  2  0 manb¤s b , c .
2 2

RsaybBa¢ak;fa b  ab  c  pq  6 .
191> RsaybMPøWfa ebIeKman a  b  c  abc nig 1a  b1  1c  2 enaHeyIg)an a1  b1  c1  2 . 2 2 2

192> rkBIrcMnYnedaydwgfa plbUk plKuN nigplEckrvagcMnYnTaMgBIr esµIKña .


193> KNnaplKuN P  1 x1 x 1 x ... 1 x  .
2 4 2n

194> KNnaplbUk S  1 1 2  1  21  3  1  2 1 3  4  ... 1  2  3 1... 2011 .


195> KNnaplbUk S  x 1 x  x  13x  2  x  15x  6  x  71x  12  x  91x  20 cMeBaH x  95 .
2 2 2 2 2

196> ebI abc  1 bgðajfa ab aa  1  bc bb  1  ca cc  1  1  a1 ab  1  b1 bc  1  c1 ca .
197> rktémø x ebI ³ xxxx  1  x  1 Edl x CacMnYnKt;FmµCati .
x2

198> sRmYlkenSam A  log 2  log 3  log 4  log 5  log 6  log 7 .


3 4 5 6 7 8

199> KNnaplbUk S  log1 x  log1 x  log1 x  ... log1 x Edl k CacMnnY Kt;FmµCati .
a a2 a3 ak

 x1  x2  x3    x2011  0
 2
 x1  x22  x32    x2011 2
0
200> edaHRsayRbB½n§smIkar ³  3 .
 x1  x2  x3    x2011  0
3 3 3

.........................................................


 x1  x2  x3    x2011  0
2011 2011 2011 2011


x  3 y  2
201> edaHRsayRbB½n§smIkartamedETmINg; ³  3  
.
2  3 x  3 y  1
202> eK[BIcMnYnBit x nig y mansBaØadUcKña. bgðajfa ³ k> x  y  2 xy x> xy  xy  2 . 2 2

203> sRmYlkenSam ³ E   a  b   4 ab nig F  a b  b a Edl a  0 , b  0 .


2

a b ab
rYcKNna ³ E  F nig E F .
204> cMeBaH x  100 KNnatémøénkenSam ³ A  x 100x  x 100x  x 100x . n n1 n 2 n3 n 4 n5

205> KNnatémøénkenSam A  22  30  20  27  8 .
3 3

206> eK[ a nig b CaBIrcMnYnKt;tKña . edaydwgfa a  b  321 cUrKNnatémøén a nig b . 2 2

207> dUcemþcEdlehAfamMuTl;kBM Ul ? bgðajfamMuTl;kMBUlCamMub:unKña .


208> rkcMnYnKt;viCm¢ an n edIm,I[ x  x  6 esµInwg n Edl x CacMnYnKt; .
2 2

209> eRbóbeFobkenSam ³ aa bb nig a  b .


2 2 2

2
a  b
2 2

 655  -
210> eKmansmIkar m  1x 2
.
 m  3x  3  m  0

k> kMNt;témø m edIm,I[ x  1 Cab¤sénsmIkarxagelI rYcKNnab¤smYyeTot .


x> kMNt;témø m edIm,I[smIkarmanb¤sDub rYcKNnab¤sDubTaMgenaH .
211> rkBIrcMnYn x nig y edaysÁal;plbUk nigplKuN ³ x  y  25 .
2 2

xy  12

x  y  x  y  8
2 2
212> edaHRsayRbB½n§smIkar  2
 x  y  xy  7
2

213> dUcemþcEdlehAfa)atmFüménRtIekaN ? bgðajfa)atmFüménRtIekaN esµInwgBak;kNþal)atmYyén


RtIekaNenaH .
214> etImancMnYnKt; n b:unµanxøHEdlepÞógpÞat; 72  12n  54 ? 24
E B
215> RtIekaN ABC RtUv)ankat;ecjBIRkdasragCactuekaNEkg
12
dUcrUbxagsþaM . etIRkdasEdlenAsl;manb:unµanPaK ?
A
216> Tajrk x  2012  ? ebIdwgfa x  2011  2011 .
1 1 C 10 D
A

217> r)arEdkmYymanRbEvg 25m RtUv)anEp¥kcugxagelI eTAelI 25 m


C

CBa¢aMgQrmYy Edlcm¶ayBICBa¢aMeTAcugr)arxageRkamman
RbEvg 20 m . ebIeKbgçitr)aecjBICBa¢aMgEfm 4 m eTot
O
etIcugr)ar)anFøak;cuHcMnYnb:unµan m ?¬sUmemIlrUbxagsþaM ¦ D 4m B 20 m

218> RtIekaN ABC mYyEkgRtg; A ehIymanbrimaRt 60 cm nigmanépÞRkLa 120 cm . cUeKNna 2

rgVas;RCugnImYy²énRtIekaN ABC .
219> bgðajfaRtIekaNEdlmanrgVas;mMuTaMgbI  , 2 nig 3 CaRtIekaNEkg .
220> ebIeKdwgfa f x f x  1  9 nig f 3  81 . cUrrk f 9 .
221> suxeFVIdMeNIredaymeFüa)aybIRbePTKW rfynþ)an 83 ehIym:UtU)an 53 éncm¶aypøÚvTaMgGs;.
k> rkRbPaKtagcm¶aypøÚvEdlsuxRtUvedIr . x> rkcm¶aypøÚvTaMgGs;ebIsuxedIr)an 2 km .
222> k> sresr 45m CaPaKryén 1km . x> sresr 1kg CaPaKryén 800 g .
223> Ggát; MN manrgVas; 18 cm . I CacMNucmYyenAelI MN ehIy MI  NI . A nig B CacMNuckNþal
erogKñaén MI nig NI . KNna AB .
224> RsaybBa¢ak;fa plbUkmMukñúgBIrénRtIekaN esµImMueRkAmYyénRtIekaN EdlminCab;mMuTaMgBIrenaH .
2011 2012
 
225> KNnatémøénkenSam A   4  5 3  5 48  10 7  4 3  4 .
 

 656  -
2 x  2 y  2 z  7
  x
226> edaHRsayRbB½n§smIkar ³ y
2  2  2 
z 7
4
.

 x  y  z  3
2011

227> a nig b CaBIrcMnYnKt;viC¢manEdl a  b  15  216 . cUrKNna  2ba  .


228> eKdwgfa log 2  0.3010.... cUrrkcMnYnxÞg;énEpñkKt;viC¢manén A  2 . 2011

229> edaHRsaysmIkar 3 x2  5  2 x3  7  x  1 .
230> eRbóbeFobcMnYn ab nig PGCD a, b PPCM a, b cMeBaH a  90 , b  280 .
231> epÞógpÞat;ÉklkçN³PaB ³ 1  sin x  cos x   21  sin x 1  cos x  .
2

232> edaHRsayRbB½n§smIkar ³ xyx yz 4 6z  13 .


2

233> eKmannimitþsBaØaelakarIt ³ Logx , log x , lg x , ln x , log x , log x . etInimitþsBaØaNaxøHCa
10 e

elakarItTsSPaK ¬elakarIteKal 10¦ ehIyelakarItNaxøHCaelakarItenEB ¬elakarIteKal e ¦ ?


234> RtIekaNmYymanrgVas;RCug 7 , 8 nig 11 . rkRkLaépÞrgVg;carwkkñúgRtIekaNenH .
235> eKmanbIcMnYn a , b , c epÞógpÞat; a  b  c  1 , a  b  c  1 nig a  b  c  1 .
2 2 2 3 3 3

KNnatémøelxénkenSam P  a  b  c . 2010 2011 2012

236> bgðajfa ³ a  b  c  d  1  a  b  c  d .
2 2 2 2

237> Gayu«BukticCagplbUkGayukUnTaMgbI 3qñaM. ebIdwgfa ³ Gayu«Buk nigkUnTaMgbIsmamaRtnwgcMnYn ³


15 , 7 , 5 , 4 . rkGayumñak;² .
60 50

238> eRbóbeFobBIrcMnYn ³  259  nig  625


225 
 .
   
ab a  b 
2
239> eK[smamaRt a c

b d
RsaybMPøWfaeK)an ³  .
cd c  d 2
240> rkBIrcMnYnKt;viC¢mantKña edaydwgfapldkkaeréncMnYnTaMgBIresµInwg 321 .
241> ekµgelgXøIbInak;KW A , B nig C manXøIsmamaRtnwg 3 , 4 , 5 . eRkayeBlQb;elgXøIGkñ TaMgbIman
smamaRt 15 , 16 , 17 . etIekµgNaQñH ekµgNacaj; ?
242> edaHRsaysmIkar 3x  6x  9x  12x  1  12  13  14 .
 x  xy  y  1

243> edaHRsayRbB½n§smIkar ³  y  yz  z  4
 z  zx  x  9

244> eRbóbeFobcMnYn 2 manelx 2 cMnYn 1001dg nig manelx 3 cMnYn 1000dg .
2 3
3
22
33

 657  -
245> edaHRsaysmIkar ³ x  3  x  4  x  4  x  5  x  5  x  3  x .
246> bursmñak;eFVIdMeNIrBIraCFanIPñMeBjeTAkan;extþsVayerog. eRkayqøgsaLagGñkelOgKat;eFVIdMeNIrbnþ
dl;Rtg;cMNucmYy Kat;)anecalEPñkeXIjbegÁalR)ab;cm¶aydak;fa {sVayRCM 17 km } nigbegÁalbnÞab;
dak;fa {sVayerog 25 km }. sMNYrsYfa etIcm¶ayBI sVayRCM eTAsVayerogmanRbEvgb:unµan km ?
247> cabmYyhVÚgehIrTMelIpáaQUkénRsHmYy. ebIcabmYyTMelIpáaQUkmYy enaHmancabmYyKµanpáaQUkTM .
EtebIcabBIrTMpáaQUkmYy enaHmanpáaQUkmYyKµancabTM. cUrrkcMnYncab nigcMnYnpáaQUk .
248> stVExVkTMelIEmkxVav )ak;EmkR)avgab;bIrs;BIr cuHebIstV 120 gab;b:unµan ? rs;b:unµan ?
249> davI manGayutageday x ebIKitKU[xÞic x CaBhuKuNén8 edaydwgfa x FMCag10 nigtUcCag 20 .
etIsBVéf¶ davIRsImanGayub:unµan ?
250> ksikmñak;mancMNIsRmab;pÁt;pÁg;eKarbs;Kat; 40 k,al)an 35 éf¶. ebIKat;TijeKa 10 k,albEnßmeTot
etIkat;GacpÁt;pÁg;cMNIdEdl[eKa)anb:unµanéf¶ ?
251> eK[bIcMnYnBit a , a nig a Edl a  sin a , a  cos a  sin a nig a  cos a  cos a .
1 2 3 1 1 2 1 2 3 1 2

bgðajfa a  a  a  1 .
2
1
2
2
2
3

252> KNnaplKuN P  1  x  x 1  x  x 1  x  x  1  x  x  .


n
2 2 4 4 8 2n 2n 1

253> ebI a  0 , b  0 nig c  0 bgðajfa a  3b  c  a 1 b  b 1 c  c 1 a .


254> KNnaplbUk S  11!2  2!3  3!  n  n! Edl n! nn 1n  2 3 2 1 .
n

255> KNnaplbUk   


 n  n  1 n  n  1 .
9999
1

 
4 4
n 1

a  b  c  3abc
3 3 3
256> sRmYlkenSam ³ A
a  b 2  c 2  ab  bc  ca
2
.
 a b  a4  b4
4

257> RsaybBa¢ak;fa ebIeKman a c


 enaHeK)an ³    4 .
b d cd  c d4
258> eKman 6 2  52  11 , 56 2  45 2  1111 , 556 2  445 2  111111 , 5556 2  4445 2  11111111 , ... .
BIkarbgðaj]TahrN_xagelIcUrrkrUbmnþTUeTA nigRsaybBa¢ak;rUbmnþenaHpg.
259> KNnaplbUk ³ S  cos 1  cos 2  cos 3    cos 89 .
2 o 2 o 2 o 2 o

260> KNnaplbUk ³ S  sin 0  sin 1  sin 2    sin 90 .


2 o 2 o 2 o 2 o

261> cUrkMNt;témø a nig b edIm,I[cMnYn abba CaKUbR)akdénmYycMnYnKt; .


262> k> kMNt;témøelxénGBaØat a , b , c nig d éncMnYn abcd edaydwgfa abcd 9  dcba .
x> bBa¢ak;fa abcd nig dcba suT§EtCakaerR)akdénmYycMnYnKt;.
263> BIrcMnYnEdlCakaerR)akd edaydwgfaplKuNva elIsplbUkvacMnYn 4844 .
 658  -
264> RtIekaNEkgmYymanépÞRkLa 24 cm nigmanbrimaRt 24 cm . KNnargVas;RCugnImYy²rbs;RtIekaN.
2

265> manBIrcMnYnKt;tKña EdlcMnYnTImYyCacMnYnbzm nigcMnYnFMbnÞab;CakaerR)akd . cUrrkcMnYnTaMgBIenaH .


266> eKmanrgVg;b:un²KñaehIyRtUv)an
eKerobdUcrUbxagsþaM Edlman 2m
km<s;esµInwg 2m . rkkaMrgVg;.
267> Rsay[eXIjfa RtIekaNcarwkknøHrgVg;CaRtIekaNEkgCanic© .
268> BhuekaNmYymanplbUkmMukñúgminelIsBI 2011 . etIBhuekaNenHGacmancMnYnRCugeRcInbMputb:unµan ?
o

269> cUrRsayfa rgVas;RCugnImYy²rbs;RtIekaNRtUvEttUcCagknøHbrimaRtrbs;vaCanic© .


270> bursmñak;man)arImUledayéd 10 edIm. Kat;Ck;)arIedayrkSaknÞúy)arI ebICk;Gs;bIedIm enaHKat;Gac
ykbnÞúy)arIEdlsl;mUl)anmYyedImfµvI ij . etIKat;Ck;b:unµandgeTIbGs;)arITaMg 10 edImKµansl; ?
271> edaHRsaysmIkar k> 3  4  5
x x x
x> 3  4  5  6 . x x x x

272> RsaybBa¢ak; BIsmPaB  y  z   z  x   x  y    y  z  2 x   z  x  2 y   x  y  2 z 


2 2 2 2 2 2

enaHeK)an x  y  z .
273> cUrKNna S  ab  cd edaydwgfa a  b  c  d  2011 nig ac  bd  0 .
2 2 2 2

274> k> RsaybBa¢ak;fa sin12a  cot a  cot 2a .


x> cUrKNnaplbUk S  sin1 a  1 a  1 a  1 a    1 a .
n
sin
sin 2 sin 3 sin n
2 2 2 2
x
cot
275> k> RsaybBa¢ak;fa 1
1

cos x cot x
2 .
    
    
 
x> cUrKNnaplKuN Pn  1 
1
 1
1
1 
 cos x  cos x  cos x 
1
    1 
1
 cos x 
 .
    
 2  22   2n 
276> eK[bYncMnYn x, y, z nig viC¢man edaydwgfa
t
x y z t
  
2 3 4 6
nig yzt ztx txy xyz
x

y

z

t
 14625 .
KNna x , y , z nig t .
277> eKmanBhuFa P  x  x  1 nig Q  2  x  x .
2 2

k> kMNt;témø x edIm,I[ P mantémøGb,brma. x> kMNt;témø x edIm,I[ Q mantémøGtibrma.


278> rkcMnYnKt;viCm¢ an n tUcbMput Edl n mansMNl; 1 , 2 , 3 , 4 , 5 eBlEcknwg 2 , 3, 4 , 5 , 6 erogKña .
279> rkcMnYnKt; n tUcbMput Edl n Eckdac;nwg 7 EtebIEcknwg 2 , 3 , 4 , 5 , 6 [sMMNl;esµI 1 Canic© .
280> RsaybBa¢ak;fakenSam E  cos x  sin x  3sin x cos x mantémøefr RKb;témørbs; x .
6 6 2 2

 659  -
281> edaHRsayRbB½n§smIkar ³ x  y3  9
3
.
 xy  2
282> edaHRsaysmIkar 3  3 3  3   4  3 .
2011 2010 2009 2008 x

283> sresr N CaplKuNktþadWeRkTI1 Edl N  3a  1  4a  6a  9 . 2 2

284> RsaybBa¢ak;fa cMnnY N  44a  1  100 Eckdac;nwg 32 .


2

285> eK[ a  b  1. cUrKNnatémøelxénkenSam P  2a  b  3a  b  1 . 3 3 2 2

286> rkBIrcMnYnKt;viC¢man a nig b edaydwgfa a  b  24 .2 2

287> kaerBIrmanpldkRkLaépÞesµI 1152 m nigmanpldkRbEvgRCugesµI 16 m . KNnaRCugkaernImYy² .


2

288> KNnakenSam A  x  11x  2  2  x23  x   1  x 3x  3 .


289> rkmYycMnYn edaydwgfa bIdgéncMnYnenH nigkaeréncMnYndEdlenH CacMnYnpÞúyKña .
290> eKmancMnYn N  12345678910111213...998999EdlcMnYnenHsresrBIelx 1 dl;elx 999 .
etIelxb:unµanenARtg;TItaMgtYTI 2011 rab;BIxageqVgéd ?
291> eK[GnuKmn_ f kMNt; x     1 , 0 eday x2 x  1 f x   f  1x   x  1 . cUrKNna plbUk
 
S  f 1  f 2  f 3    f 2011  .

292> RbGb;mYymanragCaRbelBIEb:tEkg manvimaRt 180mm , 600mm , 90 mm . rkcMnYnKUbticbMputEdl


GacerobbMeBjkñgú RbGb;enH.
293> edaHRsayRbB½n§smIkar 26xx2yy43xyxy .

294> rkBIrcMnYnKt; a nig b Edl a  b . ebIeKdwgfa plbUkrbs;vaCaBhuKuNén 15 ehIypldkkaerrbs;
vaesµInwg 45 .
295> x, y , z CacMnYnsniTanxusBIsUnü. ebI A  yz  zy , B  xz  xz , C  xy  xy . bgðajfa témøén
A  B  C  ABC minGaRs½ynwg x , y , z .
2 2 2

296> yuvCnbInak;KW A , B , C eFVIkarrt;RbNaMgKñacm¶ay 100 m . eBlrt;RbNaMgeKsegáteXIjfa ³


xN³eBl A rt;dl;TI B enAxVH 10 m eTot nigxN³eBl B rt;dl;TI C enAxVH 10 m eTot . eKsnµt;
faGñkTaMgbIrt;kñúgel,Ónefr. sYrfa xN³eBlEdl A rt;dl;TI etI C enAcm¶ayb:unµanEm:Rt BI A ?
 x  y  z  xyz

297> edaHRsayRbB½n§smIkar ³  x  y  z  xyz .
 x  y  z  xyz

298> eKman a , b , c CabIcnM YnxusKña. KNnaplbUk S  a  baa  c   b  c bb  a   c  a c c  b .

 660  -
299> rkRKb;bNþaKUéncMnYnKt;viC¢man x , y  epÞógpÞat;smIkar x  x  13  y .
2 2

300> eKmancMnYn A  2  5 Edl n CacMnYnKt;viC¢man. etIcMnYn A bBa©b;edayelxsUnü ¬0¦b:unµan ?


n 2 n 1

301> eK[bIcMnYnviCm¢ an a , b , c . edaHRsaysmIkar xbc a  xac b  xab c  2 1a  b1  1c  .


 
 
302> rkb¤sKUbén Z  8 ,  .
 4
303> eKmanKUbcMnnY 7 manTMhMb:un²Kña² ehIypÁMúP¢ab;Kña)andUcrUbsUlItxagsþaM ³
edaydwgfasUlItenHmanmaD 448cm cUrrképÞRkLaTaMgGs;rbs;slU ItenH.
3

304> eKmanRtIekaN ABC mYy nigman AM  Caemdüan . bgðaj[eXIjfa 2 AM  AB  AC .


305> GñkebIkbrmñak;ecjBITIRkug A eTATIRkug B edayel,ÓnefrCaragral;éf¶. Kat;cab;GarmµN_eXIjfa ³
ebIKat;bEnßmel,Ón 3 km/ h enaHKat;eTAdl;eKaledA muneBlkMNt; 1em:ag. EtebIKat;bnßyel,Ón
2 km/ h enaHKat;eTAdl;eKaledA eRkayeBlkMNt; 1 em:agEdr. KNna ry³eBlFmµta el,ÓnFmµta

nigcm¶ayBITIRkug A eTATIRkug B .
306> mYycMnYnmanplKuNxøÜnÉgesµInwgplbUkxøÜnÉg. KNnamYycMnYnenaH .
307> kñúgkic©RbCMumYymanmnusS 10 nak; )ancUlrYm ehIyGñkTaMgenaH)ancab;édsVaKmn_KñaeTAvijeTAmk.
ebIdwgfaGñkTaMgGs;)ancab;édKña RKb;²Kña. cUrrkcMnYnénkarcab;édKñaTaMgGs;.
308> RsaybBa¢ak;facMnYn 2  3 Eckdac;nwg 35 .
9 9

309> sRmYlRbPaKsniTan ³ E  aa  35ab  2b


2 2

2
ab  6b 2
.
310> RtIekaNmYymanbrimaRt 24 cm . ebIRbEvgRCugTaMgbIsmamaRterogKñaKW 3 : 4 : 5 cUrKNnargVas;
RCugnImYy²rbs;va.
311> cMnYnKt;mYymanelxBIrxÞg; EdlmanplbUkelxtamxÞg;esµInwg 9 . ebIeKdUrxÞg;rayeTACaxÞg;db;vji
enaHeK)ancMnnY fµIeRcInCagcMnYncas; 63 . KNnacMnnY Kt;enaH.
312> brimaRténRbelLÚRkam ABCD manrgVas;esµI 48 cm . rgVas;km<s;énRbelLÚRkamsmamaRt 5 : 7 .
KNnargVas;RCugénRbelLÚRkam ABCD .
313> rktémø x EdleFVI[kenSam P  x 1x  2x  3x  6 mantémøtUcbMput ? rkémøtUcbMputenaH.
314> mnusSmYyRkumeLIgCiHLan. ebImñak;GgÁúyekAGImYy enaHenAsl;mnusS 4 nak;KµankEnøgGgÁúy. EtebI
mnusSBIrnak; GgÁúyekAGImYy enaHenAsl;ekAGI 4 KµanmnusSGgÁúy. rkcMnYnmnusS nigcMnYnekAGI.
315> plbUkénBIrcMnYnesµInwg 1. bgðajfaplKuNvatUcCag b¤esµInwg 14 .
316> kaermYymanRkLaépÞ 100 cm carwkkñúgknøHrgVg;mYy. rkRkLaépÞénkaerEdlcarwkkñúgrgVg;TaMgmUl.
2

 661  -
317> kMNt;témøéncMnYnBit a nig b edaydwgfa a 2
 b2  0 .
 x y z
  
318> edaHRsayRbB½n§smIkar ³  2 3 4 .
 xy  yz  zx  26

319> KNnatémøKt;kenSamelx ³ E  2  5  2  5 . 3 3

320> BIrcMnYnmanplbUkesµI12 nigmanplKuNesµI 4 . KNnaplbUkcRmascMnYnTaMgBIrtamBIrrebobepSgKña.


321> ebI x  3x  8 CaktþamYyénkenSam x  rx  s cUrKNnark r nig s .
2 4 2

322> KNna A  2  4  6 ... 2n .


323> eKman S  1  21  2 1  2 1  2  ...  1  2  . cUrKNna S  1 .
2 4 8 1024 1024

324> edaymineRbIm:asIunKitelx cUrKNnarktémøén A  1234568  1234567 2012


12345679
. 2

325> rkcMnYnKt;viCm¢ an n edIm,I[ 4  n CacMnYnbzm . 4

326> rkelxxagcugénplKuN 7  2013  7  2013 . 2012 2010 2000 2012

327> KNnarkelxxagcugén A Edl A  2012 . 2013

328> rkRKb;cMnYnKt;viC¢man n EdleFVI[ 2  1 Eckdac;nwg 7 . n

329> bgðajfa 12  2 Eckdac;ngw 10 .


2012 2008

330> eKmancMnYn A Edl A  n  n . bgðajfa cMeBaHRKb;cMnnY Kt;viC¢man n enaH A Eckdac;nwg 30 .


5

331> eK[ f CaGnuKmn_BhuFakMNt;eday f x   x  3x . cUrKNna f x cMeBaH 3

x3 3 2 3 .
3 2

332> rkBIrcMnYnKt;viC¢man a nig b EdlmanplbUkesµI 92 nig a 1 CaBhuKuNén b .


333> cMeBaHcMnYnKt;viC¢man n / eK[ P  n  1n  2n  3 ...  n  n nig P  1 3  5  ...  2n  1 .
bgðajfa P Eckdac;ngw P nigrkplEckrbs;vapg .
334> eK[ 2a3  326  5b9 Edl 2a3 nig 5b9 CacMnYnmanelxbIxÞg;. ebI 5b9 Eckdac;nwg 9 KNna a  b .
335> ]bmafa a  a  1 cUrKNnatémøelxénkenSam A  a  2a  4a  3a  3 .
2 4 3 2

2 2
336> KNnakenSam A eday[lT§plCaplKuNbIktþa Edl A  .
12  8  3  6

337> KNnatémøelxén S  23  64  96  ......  300


2 2 2 2

2 2
200
. 2 2

338> RtIekaNEkgmYymanGIub:UetnusesµI 13 cm nigplbUkRCugBIreTotesµI 17 cm . rkrgVas;RCugmMuEkgTaMgBIr.


339> RsaybBa¢ak;fa ³ 3  2 . 2 3

340> bgðajfa A  6 Edl A  6  6  6  ...  6  24  24  24  ... 24 . 3 3 3 3

 662  -
x  4 x  3 x  2 x 1
341> edaHRsaysmIkar ³   
2008 2009 2010 2011
0 .
342> rkbIcMnYnKt;vCi ¢manxusKña x , y nig z EdlepÞógpÞat;RbB½n§smIkar  yx zy106 .

343> edaHRsaysmIkar ³ x  1  x  1 . 30 o d 
344> eKmanbnÞat;BrI RsbKñaKW d  l  . x
l 
KNnamMu ³ x énrUbxagsþaM . 110o
A
345> eKmanRtIekaNsm½gSBIr KW ABC nig DEF EdlmanrgVas;RCug D F

esµI 3 cm dUcKña. RtIekaNTaMgBIrpÁúMKñadUcrUbxagsþaM )anRtIekaNsm½gS B C

tUc²cMnYn 6 EdlmanrgVas;RCugesµI 1 cm dUcKña. KNnaRkLaépÞénrUbxagsþaM . E

346> eKmancMnYn 1 , 2 , 3 , 4 , 5 , 6 , 7 , 8 , 9 . cUrykelxTaMgenHeTAbMeBjkñúgRbGb;


kaerxagsþaM edIm,I[plbUkCYredk esµIplbUkCYrQr esµIplbUkGgÁt;RTUg esµIngw 15 .
347> RtIekaNmYymanrgVas;RCug a , b nig c EdlepÞógpÞat; 24a  18b  12c .
k> KNnargVas;RCugénRtIekaN ebIeKdwgfa b  c  10 cm .
x> KNnargVas;km<s;énRtIekaN EdlRtUvnwgRCug a .
348> kñúgfñak;eronmYyeKerobsisSCaRkum. ebIerobsisS 8 nak;kñúgmYyRkum enAsl;sisS 4 nak;eTot. EtebI
erobsisS 9 nak;kñúgmYyRkum enaHenAxVHsisS 2 nak;eTot. rkcMnYnsisSkñúgfñak;eronenaH .
349> rkmYycMnYnebIdwgfa cMnYnenaHminFMCagBIr ehIyk¾mintUcCagBIrEdr.
350> rkmYycMnYn EdlmanplbUkesµIplKuNénxønÜ Ég.
351> mFüménBIrcMnYnesµI 2012 nigmFüménbIcMnYnk¾esµInwg 2012 Edr. cUrrkcMnYnTI 3 .
352> x CacMnYnKt;viCm¢ an manelxBIrxÞg;. cUrKNna x ebIdwgfa 44 Eckdac;nwg x sl;sMNl; 10 .
353> edaHRsaysmIkar HE  SHE .
2

354> kMNt;témø a nig b edIm,I[kenSam ax  bx  54x  27 GacsresrCaKUbéneTVFamYy)an .


3 2

rYckMNt;eTVFaenaH .
355> RtIekaN ABC EkgRtg; A manGIub:Uetnus BC  17 cm nig AB  AC  23cm . KNna AB nig AC .
356> eK[RbB½n§smIkar 23xx  57yy  20
m
. kMNt;témø m edIm,I[KUcemøIyénRbB½n§smIkarviC¢man .

1 1

357> KNnakaMrgVg;énrUbxagsþaM ³ 1 1

1 1

358> KNnacMnYnKt;FmµCati n EdleFVI[cMnYn n 13 nigcMnYn n  76 CakaerR)akd .


 663  -
5x

359> KNnatémømMu x KitCadWeRk énrUbxagsþaM ³ 3x


4x
7x
6x
2
2y2 2z 2
360> edaHRsayRbB½n§smIkar EdlmansmIkarnImYy²KW 2x
1  x2
 y 1 ,
1  y2
 z 2  ,
1 z2
 x 3 .
361> RsaybBa¢ak;[eXIjfa plbUkmMukñúgTaMgbIénRtIekaNesµInwg 180 . o

362> cUrRsaybBa¢ak;BIRTwsþIbTBItaK½r EdlfakñúgRtIekaNEkg a  b  c man c CaGIub:Uetnus. 2 2 2

363> cUrrkcMnYnmYyEdlmanelxbYnxÞg;KW abca edaydwgfa abca  5c  1 . 2

364> eKmanBIrcMnYn a  b  0 . bgðajfa RKb;cMnYnKt;FmµCati n enaHeK)an a  b . n n

365> KNnaplKuN P  22  36  12 4
 ... 
9900
100
.
366> dak;kenSam P  x  y    y  z   z  x  CaplKuNktþa .
3 3 3

367> eK[ a b  0 EdlepÞógpÞat;lkçxNÐ 3a  3b  10ab . KNnatémøénkenSam P  aa  bb .


2 2

368> eK[kenSam A  xx  13 . kMNt;témø x viC¢man edIm,I[ A CacMnYnKt;viC¢man .


369> bgðajfa ebI a  b  c  0 ehIy a  0 , b  0 , c  0 enaHeK)an a  b  c  3abc . 3 3 3

370> edaHRsaysmIkar x  2012   x  0 . 2012 2012

371> KNnatémøénkenSam A  x  x  x  2x 1 cMeBaH x  1 2 1


4 3 2
. 2012


2 1 1 2 1 1
372> sYnc,armYymanragCactuekaNEkg EdlmanépÞRkLa 720 m . ebIeKEnßmbeNþay 6m nigbnßyTTwg 2

4m enaHépÞRkLarbs;sYn minmankarpøas;bþÚreT . KNnavimaRténsYnc,arenaH.


373> eK[BIrcMnYnEdltUc b¤FMCagKña 3 Ékta. ebIdwgfaplbUkkaeréncMnYnTaMgBIresµI 89 cUrrkcMnYnTaMgBIr.
374> KNnatémø x , y Edl y  0 BIsmIkar ³ x  4x  y  6 y 13  0 . 2

375> k> eRbóbeFob 3  3 nig 62  3  .


x> eRbóbeFob 14 48 nig 7  4 3  7  4 3 .
4 4

376> edaHRsaysmIkar x  2  x  3  1 .
4 4

377> rkcMnYnEdlmanelxBIrxÞg; AB EdlepÞógpÞat; AB  BA  1980 . 2 2

378> ebI ab  4 , ac  5 , bc  20 . KNna a , b , c .


379> KNna A  2025  1 2025  2 2025  3  ...  2025  50  .
2 2 2 2

380> etIcMnYn P  1 2  3...100 bBa©b;edayelxsUnücMnYnb:unµanxÞg; ?

 664  -
381> etIcMnYn 2012! manelxsUnüenAxagcugb:unµan ?
382> KNna N  4  15  4  15  2 3  5 .
383> kñúgfg;mYymanXøIBN’s nigXøIBN’exµAcMnYn 12 RKab; . rkcMnYnXøI s edIm,I[RbU)ab énXøIexµAesµI 13 .
384> eK[ ABC Edlmanemdüan AM , BN nig CP .
k> RsaybMPøWfa AM  BN  CP  AB  BC  AC .
x> RsaybMPøWfa AM  BN  CP  AB  BC2  AC .
385> eK[bIcMnYnKt;ruWLaTIbtKña a , b , c Edl a  b  c .
k> KNnaplbUk S  a  b  c CaGnuKmn_én b .
x> Tajrktémøén a , b , c edaydwgfa S  333 .
386> kukmYyehIrCYbRksamYyhVÚgk¾ERsksYrfa {sYsþImitþTaMg 100 }. emxül;énhVÚgRksa)aneqøIytbvijfa
{eTcMnYnBYkeyIgminRKb; 100 eT} . cMnYnBYkeyIgbUkcMnnY BYkeyIg bEnßmBak;kNþaléncMnYnBYkeyIg
ehIyEfm 14 énBYkeyIg RBmTaMgmitþÉeToteTIbRKb; 100 . cUrrkcMnYnRksakñúghVÚgTaMgGs; .

 x  y  z  xy  yz  xz
2 2 2

387> edaHRsayRbB½n§smIkar  2011



.
x  y  z  32012
2011 2011

388> brimaRténRtIekaN ABCmYymanrgVas;esµI 80 cm . ebIRCugTaMgbImansmamaRterogKña 5 , 7 , 4 .


cUrKNnargVas;RCugnImYy² .
389> eK[ 1a  b1  1c  0 . KNnatémøénkenSam P  abc
bc ac
 
a 2
b
.2 2

 x  3  2 y 1  2
390> edaHRsayRbB½n§smIkar  .
2 x  3  y  1  4

391> eKman a  1 , b  1 nig a  b . cUreRbóbeFob ba  11 , ba nig ba 11 .


392> eRbobeFob A  5 2  7  5 2  7 nig B  5110 2  5110
3 3
2 .
393> edaHRsaysmIkar 1 x x  2 x x  3 x x  ... 2013x  x  0 .
394> k> dak;CaplKuNktþanUvsmPaB 1 1 1
  
1
a b c abc
.
x> bgðajfabIcMnYnBitminsUnü a , b , c ehIymanBIrcMnYnpÞúyKñamYyy:agtickñgú cMeNam a , b , c enaH
eK)anTMnak;TMng 1a  b1  1c  a  1b  c .
395> edaHRsaysmIkar 3  3  30 .
2 x 2 x

396> sRmYlRbPaK A  x3x 14xx 1x eday x  0 .


2

 665  -
x  y 1
 xyz  2

yz 5
397> edaHRsayRbB½n§smIkar   .
 xyz 6
x  z 2
 
 xyz 3
398> RtIekaNEkgmYymanrgVas;RCug CabIcnM YnKUtKña . KNnabrimaRt nigRkLaépÞrbs;RtIekaNenH.
399> sRmYlkenSam F  xx 11 .
2

400> KNnacm¶ayBIcMNuc A6 , 6 eTAbnÞat; D : y   x  4 kñúgtRmúyGrtUNrem.


401>
402> dak;kenSam x  1x  3x  5x  7  24 CaplKuNktþa .
403> eKmanbIcMnYn a , b , c epÞógpÞat;TMnak;TMng a  b  c  1 nig a  b  c  1 .
2 2 2 3 3 3

RsaybBa¢ak;[eXIjfa ³ a  b  c  1 . 2 3

. KNna A  bc y  z  ax acxby z cz abx  y .


2 2 2
404> eK[ ax  by  cz  0 nig a  b  c  2005
1
2 2 2

405> cUrkMNt;témø x nig y edIm,I[cMnYn N  3x82 y Eckdac;nwg 3pg nigEckdac;nwg 11pg .


406> eKmancMnYn A  2 . cUrrkelx ³ k> mYyxÞg;xagcug x> BIrxÞg;xagcug K> bIxÞg;xagcug .
2004

407> KNnaplbUk S  101  102  103  ... 10n .


n 2 3 n

408> edaHRsayRbB½n§smIkar ³  3x  y 


x y
9
x y
324  18x 2  12 xy  2 y 2
.

409> eK[ a CacMnYnKt;FmµCati . bgðajfa n  , a  1  an  1  1 CaBhuKuNén a . n 1 2

410> dak;kenSamxagsþaMCaplKuNktþadWeRkTI! ³  a  4b   2  a  2b  x  a  b  0 .
2 2 3 3 4 4

411> RsaybBa¢ak;fa 4a  4a  5a  4a  1 0 , a CacMnYnBit .


4 3 2

412> eK[smIkar x  a  bx  b  cx  c  aa  b  c  abcx . edaHRsaysmIkarebI a  2, b  3, c  4 .


413> eK[ x , y , z CabIcMnYnviC¢manminsUnü . cUrbgðajfa x1  y1  z1  x  y9  z . 2 2 2 2 2 2

414> enAkñúgtRmuyGrtUNemeK[bIcMNuc . bnÞat; D  nig L kat;tam C .


A1, 2, B2 , 3, C 3, 4

k> sresrsmIkabnÞat; AB x> rksmIkarbnÞat; D Rsbnwg AB K> rkbnÞat; L Ekgnwg AB .
415> edaHRsaysmIkar x x  21x  x x12x6 35  x x 4x2 3  x x10x5 24 .
2 2 2 2

416> edaHRsaysmIkar 24 x  112 x  18x  16x  1  330 .


417> edaHRsaysmIkar x  3x  4  2x  5x  3  3x  2x 1 .
2 3 2 3 2 3

 666  -
418> bgðajfa 151  12  34  56  ... 100
99

1
10
.
419> dak;CaplKuNktþanUvsmIkar 16x  20x  5x  1  0 ebIeKdwgfa  1 Cab¤sénsmIkarenH .
5 3

420> ebI a  0 , b  0 , c  0 Edl a  b  c . cUrRsaybMPøWfa 1a  ab 3abc


bc  ca
.
421> rkbIcMnYnKt;rLWu aTIb a , b , c edaydwgfa cMeBaHRKb;témø x eK)an ³ x  ax 10   1  x  bx  c .
 x1  x2  x3  ... x2000  1
 x  x  x  ... x
 1 3 4 2000  2

422> edaHRsayRbB½n§smIkar  x1  x2  x4  ... x2000  3 EdlmancMnYn 2000 smIkar .


.......................................

 x1  x2  x3  ... x1999  2000

423> edaHRsaysmIkar x  1x  2x  3x  4  3 .


424> eKmanctuekaNBñay ABCDmYyman)at AB  80 cm , CD  40 cm . KNnaépÞRkLaénctuekaN
BñayenH ebIdgw faRCug AD  68 cm nig AC  84 cm .
425> fñak;TI 9C mansisSbIRbePTKW BUEk mFüm exSay. enAedImqñaMcMnYnsisSsmamaRtnwg 3 , 4 , 7 luHdl;
cugqñaMcMnYnsisSsmamaRtnwg 2 , 5 , 7. ebIdwgfacugqñaMsisSexSayfycuH 10 nak; cUrrkcMnYnsisSsrub.
426> eK[ ax  by  cz  k Edl k CacMnYnKt;FmµCati. bgðajfa ax  by  cz  a  b  cx  y  z .
2

427> k> rkbIcMnYnKt;esstKña ebIplbUkcMnYnTaMgbIesµInwg 909 .


x> rkbYncMnYnKt;KUtKña ebIdwgfaplbUkcMnYnTaMgbYnesµnI wg 1028 .
428> edaHRsaysmIkar 0.17  2. 3  x  0. 3 .
429> xügmYyenAkñúgGNþÚglU manCeRmA 7m.eBlyb;xügenHvareLIgelI)an 3m EtenAeBléf¶eRkamGMNac
énkemþAvaFøak;cuH 1m vij. ]bmafa xügBüayamvareLIgrhUt etIb:unµanéf¶eTIbxügecjrYcBIGNþgÚ .
430> enAem:ag 6:30 mn narImYyRkumcab;epþImsÞgÚ RsUvBIPøWmçag eTAPøWmçageTot Edlmancm¶ay 100m eday
el,Ón 30m/h ehIybursmYyRkumeTotcab;epþImsÞÚgBIPøWmçagedayel,Ón 20m/h .
k>etIRkumTaMgBIrsÞÚgCYbKñaenAem:agb:unµan ?
x> etImYyRkum²sÞÚg)anRbEvgb:unµan m ?
431> KNnakenSam F  x  2  3 2x  5  x  2  3 2x  5 .
432> eKmankaer ABCD nigcMNuc K enAelIGgát;RTUg AC . bgðajfa KA  KC  2KD . 2 2 2

433> KNnakenSam x  y1 ebIdwgfa  x  1y   1296 nig xy  4 .


4
4
 
ab  8
434> edaHRsayRbB½n§smIkar 3 .
 a  b 3
3

 667  -
435> eK[RtIekaNsm)at COB Edlman)at OB nigkm<s; OA . Rsayfa AC  OB  AB  OC .
2 2 2 2

436> ABC CaRtIekaNEkgRtg;kMBUl A manbrimaRtesµI 12 cm nigmanRkLaépÞesµI 6 cm . KNnargVas;


2

RCugnImYy² énRtIekaNEkg ABC enH .


437> rkcMnYnKt;EdlenAcenøaH 40 nig 50 edaydwgfa ebIeKbþÚrlMdab;elx enaHeKnwg)anCYbcMnYnfµImYyeTot
EdlesµInwg 3223
éncMnYnenaH .
438> tamcMNuc O kñúgRtIekaNsm½gS ABC eKKUsGgát;EkgeTAnwgRCugTaMgbIénRtIekaNenaH.
bgðajfaplbUkcm¶ayBIcMNuc O eTARCugTaMgbIesµInwgrgVas;km<sé; nRtIekaNenaH .
439> bM)at;r:aDIkal;BIPaKEbg A  20
.
3 5  2 2 5
 x  y  3

440> edaHRsayRbB½n§smIkar  x  z  2 .
 xy  yz  zx  2

441> eK[RtIekaN ABC manrgVas;RCug a , b , c nigRtIekaN ABC manrgVas;RCug a , b , c . ]bmafa
ABC dUcnwg ABC bgðajfa aa  bb  cc  a  b  ca  b  c .

442> ABC CaRtIekaNEkgRtg; A Edlman BC  2a nig B  60 . kMNt;témø a edIm,I[témøbrimaRt


o

esµInwgtémøépÞRkLa .
443> enAmanlMhat;bnþCaeRcInTot>>>.

 668  -
1> rkRKb;KUéncMnYnKt;viC¢manén a , b -ebI A  2 enaHelxcugén 3 4 k A
KW ³ 9
eyIgman ³ a 2  b2  a  b  8 -ebI A  3 enaHelxcugén 3 4 k A
KW ³ 7
eyIg)an ³ 4a 2  4b 2  4a  4b  32 naM[ 3  3 2011 45023
manelxcug 7
4a 2  4a  1  4b 2  4b  1  34 32013  345031 manelxcug 3
2a  12  2b  12  34  BinitüelxcugsV½yKuNén 7 ³
eday 34  36  6 ehIy 2a 1 nig 2b 1CacMnYness
2
71  7 , 7 2  49 , 7 3  343 , 7 4  2401
enaH 34 CaplbUkkaercMnYnesstUcCag 6 . 7 5  16807 , 7 6  ...9 , 7 7  ...3 , 78  ...1 , ...
cMnYnesstUcCag 6man 1, 3, 5 eyIgTaj)anTUeTA sV½yKuNén 7 ³ eyIgman 7 , k  N 4k  A

manEt 3  5  34 b¤ 5  3  34 eyIg)an ³
2 2 2 2
-ebI A  0 enaHelxxagcugén 3 KW ³ 1 4 k A

 2a 1  3 nig 2b 1  5 enaH a  2 , b  3 b¤ -ebI A 1enaHelxxagcugén 3 KW ³ 7 4 k A

 2a  1  5 nig 2b  1  3 enaH a  3 , b  2 -ebI A  2 enaHelxxagcugén 3 KW ³ 9 4 k A

dUcenH KUcMnYnepÞógpÞat;KW ³ aa  32 ,, bb  23 . -ebI A  3 enaHelxxagcugén 3 KW ³ 3 4 k A

 naM[ 7  7 manelxcug 1
2012 4503 0

2> RsaybBa¢ak;fa E CakaerR)akd eyIgman ³ 7 7 manelxcug 9


2014 4503 2

E  x  1000 x  2000 x  3000 x  4000   1000 4


eyIg)an ³
tag y  1000 eyIg)an ³ elxcug A =elxcug 7  elxcug1  elxcug 3  elxcug 9
E   x  y  x  2 y  x  3 y  x  4 y   y 4 = elxcug 7  elxcug 7
  x  y  x  4 y  x  2 y  x  3 y   y 4 = elxcug 4
 
 x 2  4 xy  xy  4 y 2 x 2  3 xy  2 xy  6 y 2  y 4  dUcenH cMnYn A manelx 4 enAxagcug .
 x 2
 5 xy  4 y 2
x 2
 5 xy  6 y  y 2
 4

 x 2

 5 xy  5 y 2  y 2 x 2  5 xy  5 y 2  y 2  y 4  4> KNnakenSam P ³ eyIgmankenSam
 x   32 2 3 2 2
2
2
 5 xy  5 y 2  y4  y4 P 
 x 2
 5 xy  5 y 
2 2 17  12 2 17  12 2

dUcenH 
E  x 2  5xy  5 y 2  CakaerR)akd .
2

2  2 2 1

2  2 2 1
9  12 2  8 9  12 2  8

3> rkelxenAxÞg;rayéncMnYn A ³ 
2 2  2 2  12

2 2  2 2  12

eyIgman A  3  7  3  7 2011 2012 2013 2014 3 2  12 2  2 2  2



3 2  12 2  2 2 
2

 BinitüelxcugsV½yKuNén 3 ³ 
 2  1 2


 2  1 2

3 3
1
, 3 9
2
, 3  273
, 3  81
4 3  2 2  2
3  2 2  2

2 1 2 1
35  243 , 36  729 , 37  2187 , 3  6561 , ...  
32 2 3 2 2
eyIgTaj)anTUeTA sV½yKuNén 3 ³ eyIgman 3 4k  A
,k  N

 
2 1 3  2 2    2  13  2 2 
-ebI A  0 enaHelxcugén 3 KW ³ 1  2 2 
2
4 k A 32
 3 2  432 2 3 2  43 2 2  2
-ebI A 1enaHelxcugén 3 KW ³ 3 4 k A

dUcenH kenSam P  2 .
 669 
5> KNnatémø x edIm,I[ F mantémøGb,brma tamlMnaMxagelIeKGacbMEbk)an dUcxageRkam ³
eyIgman ³ 1 1 1
 
F   x  1 x  2  x  3 x  6  1 2 1 2
1 1 1
  x  1 x  6  x  2  x  3  
23 2 3

 x 2  6 x  x  6 x 2  3x  2 x  6  
 x 2
 5x  6 x  5x  6  2
 bUkGgÁnwgGgÁ 1 1 1
 
3 4 3 4
 x  >>>>>>>>>>>>>>>>>>>>>>>
2
2
 5 x  36
eday x  5x  0 enaH x  5x
2 2 2 2
 36  36 1
 
1 1
n  n  1 n n  1
naM[ F mantémøGb,brmaKW F  36 1 1 1 1 1 1
   ...   
eyIg)an x  5x  36  36
2 2
1 2 2  3 3  4 n  n  1 1 n  1
x  5x  0 2 2
b¤ 1

1

1
 ... 
1

n
1 2 2  3 3  4 n  n  1 n  1
x  5x  0
2

xx  5  0 cMeBaH n CacMnYnKt;viC¢manehIyFMCag!


naM[ x  0 , x  5 eK)an n  n 1 b¤ n n 1  1
dUcenH témøKNna)anKW x  0 , x  5 . naM[ 1

1

1
 ... 
1
1
1 2 2  3 3  4 n  n  1
6> KNnatémøén x2  y2 ³ dUcenH 1

1

1
 ... 
1
1 .
1 2 2  3 3  4 n  n  1
eyIgman ³ x  y  30
x  y 3  303
8> k> rksMNl;ebI n Ecknwg 56

x 3  3x 2 y  3xy 2  y 3  27000 smµtikmµ ³ n Ecknwg 7 [sMNl; 5


x 3  y 3  3xyx  y   27000 ebItag q  CaplEck enaHeK)an n  7q  5
1 1

eday x  y  8100 nig x  y  30


3 3
n Ecknwg 8 [sMNl; 3
eyIg)an ³ 8100  3xy  30  27000 ebItag q  CaplEck enaHeK)an n  8q  3
2 2
90 xy  27000  8100
18900 eyIg)anRbB½n§smIkar nn  87qq  53  87
1

xy   210  2
90
Et x 3  y 3   x  y x 2  xy  y 2  dkGgÁnwgGgÁ 
8n  56q1  40
8100  30 x  210  y  2 2
 7n  56q2  21
n  56q1  q2   19
x 2
 210  y 2
  270
x 2  y 2  270  210
ebItag q CacMnYnKt;FmµCati Edl q  q  q 1 2

x 2  y 2  480 eyIg)an n  56q  19


dUcenH témøKNna)anKW ³ x 2  y 2  480 . dUcenH témøsMNl;Edl n Ecknwg 56 KW 19 .
x> rkcMnYn n ³ eday 5616  n  5626
7> RsaybMPøWfa ³ 11 2  21 3  31 4  ...  n  n1  1  1 naM[ 5616  56q  19  5626
Binitü ³ 1n  n 1 1  nnn11n  nn1 1 5597  56q  5607
99.946  q  100.125

 670 
eday q CacMnYnKt;FmµCati enaH q  100 10> KNnatémøelxénkenSamelx A
eyIg)an n  56100 19  5619 eyIgman ³ A  182  33125  182  33125
3 3

dUcenH cMnYn n Edlrk)anKW ³ n  5619 . tamrUbmnþ a  b  a  3a b  3ab  b


3 3 2 2 3

naM[ ³  a  b  3aba  b 
3 3

9> bgðajfa A  a1  a1  a1  ...  a1 CacMnYnKt; 3


1 2 3 20 A3   3 182  33125  3 182  33125 
2 2
 
eyIgman ³ a n
 1  1
 1  1    1  1  
 n  n
naM[ A3  182  33125  182  33125 
3
1

1  3 3 182  33125  3 182  33125   A
 
an  1
2
 1
2

1  1    1  1  
 n   n A3  364  33 1822  331252  A

 1
2
 1
2 A3  364  33 33124  33125  A
1  1    1  1  
 n   n A3  364  3 A
 2 2
 1  1 A3  3 A  364  0
1   1    1  1  
 n   n edayemIleXIjb¤sgay A  7 eyIg)an ³
 1 1 
2
 1 1 
2

1  12  2  1       1  12  2  1      A3  3 A  364  0


 n  n    n  n  
 
 A3  7 A 2  7 A 2  49 A  52 A  364  0
1 1  1 1 
2 2

12  2  1      12  2  1      A 2  A  7   7 A A  7   52 A  7   0
n  n   n  n  
 A  7A2  7 A  52  0
2n  2n  1
2
2n  2n  1 2
   A  7  0

n2 n2 Taj)an  A 2
 7 A  52  0
4 
n
naM[  A  7  0  A7
n  2n  2n  1
2
2n 2  2n  1 
   eRBaHKµanb¤s edaysar   0
 A2  7 A  52  0
4  n2 n2 


1
 2n 2  2n  1  2n 2  2n  1  dUcenH témøelxénkenSam A  7 .
4
eyIg)an ³ 11> KNna A  x  2011
y 2011
1 1

a1 4
5 1   eyIgman ³ 2 xy  y  4 x 2
 2x 1
y 2 x  1  4 x  2 x  1
 
2
1 1
 13  5
a2 4 4x 2  2x 1
y
1 1
 
  25  13 2x 1
a3 4 2 x2 x  1  1
>>>>>>>>>>>>>>>>>>>>>>>>>>>>>>>>>> y
2x 1
1

a20 4
1
841  761  y  2x 
1
2x 1
1 1 1
   ... 
a1 a2 a3
1

a 20 4
1
841  1   smµtikmµ x , y CacMnYnKt; enHmann½yfa
 29  1  7
1 y  2x 
1
2x 1
CacM n Y n Kt; l u H RtaEt 1
2x 1
CacMnYnKt;
4
dUcenH KNna)an A  7 CacMnYnKt; .
 671 
ehIy 2 x1 1 CacMnYnKt;luHRtaEt 2x 1  1 smµtikmµ smamaRtkm<s; 5 : 7 mann½yfa hh
5
7
1

2

naM[ x  1Taj)an y  2 x  2 x1 1 naM[ AD  smmUl


5 AD AB AD  AB
 
AB 7 5 7 12
1
 2 1 
2 1  1
 2 1  1 EtRbelLÚRkammanbrimaRt 48cm
naM[ 2 AB  AD   48 enaH AB  AD  24
eyIg)an ³ A  x  y 2011 2011
 12011  12011  2
AB AD  AB 24
dUcenH KNna)an A  x 2011
 y 2011  2 . eyIg)an AD 5

7

12

12
2

TMnak;TMngsmamaRt AD  2  AD  10cm
12> KNna f 12  ³ eyIgman ³ 5
AB
 2  AB  14cm
f x   x 2011
 13x 2010
 13x 2009
 13x 2008
 7

 ...  13x  13x  12 dUcenH rgVas;RCugRbelLÚRkamKW 10cm nig 14cm .


 x 2011  12 x 2010  x 2010  12 x 2009 
 x 2009  12 x 2008  x 2008 
14> rksmIkaremdüaT½rénGgát; AB 
 ...  12 x 2  x 2  12 x  x  1
smIkaremdüaT½rénGgát; AB CasmIkarbnÞat;Edl
 x 2010  x  12  x 2009  x  12  kat;tamcMNuckNþalGgát; ABehIyEkgnwgGgát; AB.
 x 2008  x  12  x 2007  x  12  eyIgman ³ A2 , 3 nig B4 , 4
 ...  x x  12   x  1 naM[kUGredaencMNuckNþalénGgát; ABKW ³
 x  12( x 2010  x 2009  x 2008  x 2007   x  xB y A  y B   2  4 3  4   7 
I A ,   I ,   I  3, 
 ...  x)  x  1  2 2   2 2   2

cMeBaH x  12 CMnYscUl eyIg)an ³ -bnÞat;  AB kat;tam A2 , 3 enaH A  AB 


f 12  12  12(122010  122009  122008  122007  naM[kUGredaencMNuc A epÞógpÞat;smIkar³
 ... 12)  12  1 3  2a  b GacsresrCa 2a  b  3 1
f 12   0  11  11 -bnÞat;  AB kat;tam B4 , 4 enaH B   AB 
dUcenH eRkayBIKNna f 12   11 . naM[kUGredaencMNuc B epÞógpÞat;smIkar³
4  4a  b GacsresrCa 4a  b  4 2
13> KNnargVas;RCugénRbelLÚRkam ABCD
yksmIkar 2  1 eyIg)an ³
2
A B
4a  b  4

h1
h2
 2a  b  3 1
D C
2a  1 naM[ a  12
eyIgmanrUbmnþRkLaépÞRbelLÚRkam -smIkaremdüaT½rRtUvrkmanrag y  ax  b
S ABCD  AB  h b¤ S  AD  h
1 ABCD 2
eday y  ax  b   AB  enaH a  a  1
Taj)an AB  h  AD  h
1 2 b¤ 12  a  1  a  2
)ansmamaRt hh  AD AB
1
ehIy emdüaT½rkat;cMNuckNþal I enaH I   y  ax  b
2

 672 
eyIg)an ³ 72   2  3  b  b 
7
2
6
19
2
17> KNna E  xx  yy
naM[smIkaremdüaT½rEdlRtUvrkGacsresr³ y  2x  192 cMeBaH E
x y
smmUl E 2

 x  y
2

x y x  y 2
dUcenH smIkaremdüaT½rénGgát; ABKW³ y  2x  192 . eyIgman ³ 2 x  2 y  5xy 2 2

15> rkRbU)abcab;)anXøI@ minEmnBN’exov naM[ 2 x  2 y  5xy 2 2


2 x 2  y 2  5 xy
RBwtiþkarN_cab;min)anXøIBN’exov mann½yfa ³ 5
x 2  y 2  xy 
)anXøIRkhmTaMg@ b¤ elOgTaMg@ b¤ Rkhm!elOg! 2

-RbU)abcab;)anXøIRkhmTaMg@KW P ¬k>k¦= 122  111  661 -Efm 2 xy elIGgÁTaMgBIrénsmIkar 


-RbU)abcab;)anXøIelOgTaMg@KW P ¬l>l¦= 4 3 1
 
eyIg)an x  2xy  y  52 xy  2xy
2 2

12 11 11
-RbU)abcab;)anXøIRkhm!elOg!KW ³ smmUl x  y   92 xy 2

2 4 4 2 4
P ¬k!>l!¦= P ¬k>l¦÷ P ¬l>k¦=        
 12 11   12 11  33
-Efm  2 xy elIGgÁTaMgBIrénsmIkar 
eyIg)an x  2xy  y  52 xy  2xy
2 2

naM[ RbU)abcab;XøI@minEmnBN’exovKW ³
smmUl x  y   12 xy 2

P ¬minEmnexov¦= P ¬k>k¦÷ P ¬l>l¦÷ P ¬k!>l!¦


9

1 1 4
 
eyIg)an ³ E2 
x  y 2 xy
 2 9
66 11 33
1  6  8 15 5
x  y 2 1
xy
   2
66 66 22
Taj)an E  9  3 eRBaH x  y  0 enaH E  0
dUcenH RbU)abcab;min)anXøIBN’exovKW P  225 .
dUcenH eRkayBIKNna E  3 .
16> KNnakenSam A ³ eyIgman 18> sRmYlkenSam E ³ eyIgman
A
6  12  18  ... 962  3
12  24  36  ... 1922 4 E  2x 2  y 2  2x x  y  x  y  x  y x  y 

6  12  18  ...  96
2

3
 x  y   2 x x  y   x  x  y 
2 2 2 2 2 2 2

2  6  2 12  2 18  ... 2  96 4


2


6  12  18  ... 962  3  x  y   2 x x  y   x  x  y 
2 2 2 2 2 2 2 2

26  12  18  ... 962 4


 2
6  12  18  ...  96
2

3   x  y   x  x  y 
2 2
2
2 2

2 6  12  18  ...  96 4
2
 x  y   x  x  y   x
2 2 2 2

1 3
  1
4 4 dUcenH eRkayBIsRmYlrYc E  x .
dUcenH eRkayBIKNna A  1 .
 673 
19> KNnaplKuN  c  a   2 ca iii 
k> A  1  cot1 1  cot 2 1  cot 3  1  cot 44  edayKuNGgÁnwgGgÁén i  ii iii eyIg)an ³
o o o o

tamrUbmnþ cos a
 cot a eyIg)an a  b  b  c  c  a   2 ab  2 bc  2 ca
sin a
a  b b  c c  a   8 ab  bc  ca
 cos1o  cos 2 o  cos3o   cos 44o 
A  1  
o 
1  
o 
1  
o 
 sin 1  sin 2  sin 3 
   1  
o 
 sin 44 
a  b b  c c  a   8abc
 sin 1o  cos1o
A  
 sin 1o
 sin 2 o  cos 2 o

 sin 2 o
 sin 3o  cos3o

 sin 3o
  sin 44 o  cos 44 o
    
  sin 44 o



dUcenH eyIgeXIjfa a  bb  cc  a  8abc Emn .
edaydwgfa sin a  cos a  
2 sin 45 o  a 
 2 sin 44 o
 2 sin 43 o
 2 sin 42 o
  2 sin 1  o
21> bgðajfa a  b  c  ab  bc  ca ³
2 2 2

A        
 sin 1
o  sin 2o

 sin 3o



 sin 44o 
  eday a , b , c CacMnYnBitviC¢man eyIg)an ³
 sin 44 o  sin 43o  sin 42 o   sin 1o 
 a  b   0  a 2  2ab  b 2  0  a 2  b 2  2ab i 
44
A  2          2
o  o  o 
 o 
 sin 1  sin 2  sin 3   sin 44 
 b  c   0  b 2  2ab  c 2  0  b 2  c 2  2bc ii 
eRkayBIsRmYl eyIg)an A  2  4 194 304
2
22

dUcenH lT§plénplKuNKW A  2  4 194 304 .  c  a  0  c  2ab  a  0  c  a  2ca iii


2 2 2 2 2
22

x> B   3  tan1  3  tan 2  3  tan 3   3  tan 29  edaybUkGgÁngw GgÁén i   ii  iii eyIg)an ³
o o o o

tamrUbmnþ cos sin a


 tan a eyIg)an a  b  b  c  c  a  2ab  2bc  2ca 2 2 2 2 2 2

 sin 1o
a
 sin 2 o   sin 29 o 
 
2 a 2  b 2  c 2   2ab  bc  ca 
B   3   3     3  
 cos1o  cos 2 o   cos 29 o 
a 2  b 2  c 2  ab  bc  ca
 3 cos1o  sin 1o  3 cos 2 o  sin 2 o 
B   o

 o
 3 cos 29 o  sin 29 o 
  
 


dUcenH eXIjfa a 2
 b 2  c 2  ab  bc  ca Emn .
 cos1  cos 2   cos 29 o 

edaydwgfa 3 cos a  sin a  2 cos 30 o  a   22> rkb¤sKt;rbs;smIkar


 2 cos 29 o  2 cos 28 o  2 cos 27 o 
B   o
 o
 cos1  cos 2  cos 3 
 o
 2 cos1o 
    
 cos 29 

o 
eyIgman 2x  y   16  3xy
 cos 29 o  cos 28 o  cos 27 o   cos1o  2 x  2 y  16  3 xy
B  2 29  o 
 o 
 o 
     
 cos1  cos 2  cos 3   cos 29
o
 3 xy  2 x  2 y  16
eRkayBIsRmYl eyIg)an B  2  536 870 912 29 4
3xy  2 x  2 y   16 
4
¬Efm 43 ¦
3 3
dUcenH lT§plénplKuNKW B  2  536 870 912 . 29

x3 y  2  
2
3 y  2  48  4
3 3
20> cUrbgðajfa ³ a  bb  cc  a  8abc  2
 x  3 y  2  
52
 3
eday a , b , c CacMnYnBitviC¢man eyIg)an ³  2
3

3 x  3 y  2   52
  a  b  0
2
 3
a2  2  a  b  b2  0 3x  23 y  2  52
a  b   2 ab i  edaycg;rkEtb¤sKt;én x nig y EdlepÞógpÞat;
dUcKñaenHEdr eyIg)an ³ ehIy
1 52

52  2  26 b¤
 52  1

52  26  2
 b  c   2 bc ii   4  13

13  4

 674 
-cMeBaH 33yx22521  3x  3
   
 x 1 24> k> KNna f x  f  y 
 3 y  54  y  18
eyIgmanTMnak;TMng f x   4 4 2
x

 3x  2  2  3x  4
-cMeBaH    ¬
minykeRBaHminKt;¦ x

3 y  2  26 3 y  28 naM[ f  y   4 4 2
y

 3x  2  4  3x  6 x  2 y

-cMeBaH     x y

3 y  2  13 3 y  15 y  5 eyIg)an f x  f  y   4 4 2  4 4 2 x y

dUcenH KUb£sKt;EdlrkeXIjrYmman³ 
4x 4 y  2  4 y 4x  2

  
 x  1 x  2
b¤  x  18  x  5
. 
4x  2 4 y  2  
 ,  ,
4 x y  2  4 x  4 x y  2  4 y
y  18 y  5
 y 1
 y2    x y
4  2  4 x  2  4 y  22
23> RsaybBa¢ak;fasmIkarEdl[manb¤Canic© edaydwgfa x  y  1 enaHeyIg)an
eyIgman x  ax  b  x  bx  c  x  cx  a  0 f x   f  y   4  2  4  4  2  4 x y

4  24  24  2 x y 2
edayBnøatsmIkarxagelI eyIg)an ³ 8 24  24 x y
 1
x 2  bx  ax  ab  x 2  cx  bx  bc  x 2  ax  cx  ac  0 8  2  4x  2  4y

3x 2  2ax  2bx  2cx  ab  bc  ac  0 dUcenH f x   f  y   1 tamlkçxNÐEdl[ .


3x 2  2a  b  c x  ab  bc  ac   0
eXIjfaeRkayBIbRgYmrYcvaCasmIkardWeRkTI2 x> TajrkplbUk S ³ eyIgman
 1   2   3   2010 
edIm,I[smIkardWeRkTI2manb¤sCanic© luHRtaEt   0 S  f  2011   f  2011   f  2011   ...  f  2011 
eyIg)an   b  ac Edl b  b2
2
edayyl;fa f x  f  y   1 kalNa x  y  1
   a  c  c   3ab  bc  ac 
2 ehIyeXIjfa
 1   2010 
 1 enaH f 
1 2010
 a  b  c  2ab  bc  ac   3ab  bc  ac 
2 2 2
  f  1
2011 2011  2011   2011 
 a 2  b 2  c 2  ab  bc  ac 
 2   2009 
1

 2a 2  2b 2  2c 2  2ab  2bc  2ac 
2

2009
2011 2011
1 enaH f 
 2011 
f
 2011
 1

2
>>>>>>>>>>>>>>>>>>>>>>>>>>>>>>>>>>>>>>>>>>>>>>>>>>>>>>>>>>>>>>>>>>>>>>>>>>>>>>>
1

 a 2  2ab  b 2  b 2  2bc  c 2  c 2  2ac  a 2   1005   1006 
2 1005 1006
 1 enaH f  f   1
1

 a  b   b  c   c  a 
2 2 2
 2011 2011  2011   2011 

eday
2
a,b,c CacMnYnBit enaHeyIg)an ³ edaybUkGgÁngw GgÁ
 1   2   2010 
 a  b   0 f  f   ...  f    1  1  ...  1
2
 2011   2011   2011 
 b  c   0 man !00% tY
2

 c  a   0
2
 1   2   2010 
a  b   b  c 2  c  a 2  0
2 f  f   ...  f    1  1005
 2011   2011   2011 
naM[ 12 a  b  b  c  c  a  0 Emn
2 2 2

dUcenH eyIgTaj)anplbUk S  1005 .


dUcenH smIkarmanb¤sCanic© .
 675 
25> KNnalImIt lim x x0
27> bgðajfa
x

tag y  x naM[ ln y  ln x smmUl ln y  x ln x a  ba  b a  b  a  b   0


x x 2 2 4 4 8 8

eyIgGacbMElg)an ln y  ln1x ¬lkçxNÐ x  0 ¦ Binitü a  b  a  b a  b  8 8 4 4 4 4

x

 a2  b2 a2  b2 a4  b4  
eyIg)an lim ln y  lim
ln x
¬ragminkMNt;¦  a  b a  b  a  b a  b 4  2 2
 4

x 0 x 0 1
EtbRmab; ³ ; a b  1
eyIg)an a  b  a  ba
  naM[
x 8 8 2
 b2 a4  b4
-eyIgGacedaHRsaytamrebobTI! ¬edaHedaytag¦³
a  b a  b a  b   a  b   0 2 2 4 4 8 8

tag u  1x naM[ x  u1 ebI x  0 enaH u   a  ba  b a  b   a  ba  b a  b   0



2 2 4 4 2 2 4 4

¬)anCah‘andak; x  0 eRBaH x  0 xitCitBIxagsþa¦M


 00

eyIg)an ³ dUcenH a  ba  b a  b  a  b   0 . 2 2 4 4 8 8

1
lim
ln x
 lim
ln
u  lim ln u   lim ln u  0 28> KNna A  1  12   1  13   1  14   ...  1  2011
1 1 


x 0 1 u  u u  u u  u
x  1  1  1  1 
A  1    1    1    ... 1  
¬eRBaHtamrUbmnþ 1a  a / log b  x log b / lim lnuu  0 ¦
1
a
x
a
 2  3  4  2011
u 
 2  1  3  1  4  1  2011  1 
naM[ lim ln y  0  ln lim y  0
x0 x0
  
 2   3   4 
  ... 
 2011 

1 2 3 2010 1
lim y  e  lim x  1 .
0 x     ...
2 3 4

2011 2011
x0 x 0

- eyIgGacedaHRsaytamrebobTI@ ¬LÚBItal;¦³ dUcenH eRkayBIKNna A


1
2011
.
1
cMeBaH lim ln y  lim
x 0
ln x
x 0 1
 lim x  lim  x   0
x 0 1 x 0
29> edaHRsaysmIkar ³ 4x  5x 4 2
1  0
 2
x x eyIgman ³ 4 x  5x  1  0 4 2

¬eFVIedrIevTaMgPaKyk nigPaKEbg¦ 4x 4  4x 2  x 2 1  0
naM[ lim ln y  0
x0
 ln lim y  0  
x0
  
4x 2 x 2 1  x 2 1  0 
lim y  e 0
x0
 lim x  1
x 0
x
. x  14 x
2 2
 1  0
x  1x  12 x  12 x  1  0
dUcenH eRkayBIKNnaeXIjfa lim x x 1 .
x0
plKuNktþaesµI 0 luHRtaktþanImYy²esµI 0 ³
26> KNna A ³ eyIgman eyIg)an ³  x 1  0  x  1
 x 1  0  x  1


A  2010 2011 2010  2011 2009  2011 2008  ...  2011 2  2012  1   2x 1  0  x
1
2
A  2011  1(20112010  20112009  20112008 
1
 ...  20112  2011  1)  1  2x 1  0  x
2

 
A  2011 2011  12011  1  2011 2011  1  1  2011 2011
dUcenH smIkarmanb¤sbYnKW x   1,  12 , 12 ,1 .
dUcenH KNna)an A  20112011 .
 676 
30> bgðajfa ab 1a  1  bc 1b  1  ca 1c  1  1 eyIg)an ³ 1
2 1 1 2

1
1

1
2
BinitüGgÁTImyY ³ 1

1

1
3 22 3 2 3

1 1
  1 1 1
ab  a  1 ab  a  1  
1 1 a a 4 3 3 4 3 4
    ……………………….
bc  b  1 bc  b  1 a abc  ab  a
1 1 1

a  
eRBaH abc  1 100 99  99 100 99 100
1  ab  a
1 1 ab ab 1 1 1 1 1
      ...   
ca  c  1 ca  c  1 ab a  abc  abc  ab 2 1 1 2 3 22 3 100 99  99 100 1 100
ab 1 1 1 1 9
 eRBaH abc  1   ...   1 
a  1  ab 2 1 1 2 3 2  2 3 100 99  99 100 10 10

eyIg)an ³
1

1

1
ab  a  1 bc  b  1 ca  c  1
dUcenH S  2 1 1 1 2  3 2 1 2 3  ...  100 99 1 99 100  109 .
1 a ab
  
1  a  ab 1  a  ab 1  a  ab

1  a  ab
1
32> bgðajfa 2002 2003
 20021979 Eckdac;nwg 6
1  a  ab
eyIgman ³
20022003  20021979
dUcenH 1

1

1
1 BitEmn .
ab  a  1 bc  b  1 ca  c  1 
 20021979 200224  1 

 20021979 2002  1 200223  200222  ... 2002  1 
31> KNna  2  2002 1978

 2001 2002  2002  ... 2002  1
23 22

S
1

1

1
 ... 
1  2  2002 1978

 3  667 2002  2002  ... 2002  1
23 22

2 1 1 2 3 2  2 3 4 3  3 4 100 99  99 100
 6  2002 1978

 667 2002  2002  ... 2002  1
23 22

tamlT§pl cMnYn 2002  2002 CaBhuKuNén 6 2003 1979

Binitü ³ dUcenH cMnYn 2002  2002 Eckdac;nwg 6 .


2003 1979

1
n  1 n  n n  1 33> bgðajfa a  b  c  13 2 2 2


1

n  1n n  1  n  eyIgman a  b  c  1 elIkCakaer eyIg)an
a 2  b 2  c 2  2ab  2bc  2ca  1

 n 1  n  i 
n  1n    a 2  b 2  c 2  1  2ab  2bc  2ca
n 1  n n 1  n
n 1  n n 1  n
RKb;cMnYnBit a , b , c enaHeyIg)anvismPaB ³
 
n  1n n  1  n  n  1n a 2  b 2  2ab

n 1 n 1 1   b 2  c 2  2bc
     c 2  a 2  2ca
n  1n n  1n n n  1 
2a 2  2b 2  c 2  2ab  2bc  2ca ii 

 677 
edayyk i   ii enaHeyIg)an ³ 
1
x y 

1  xy   x y 
 a 2  b 2  c 2  1  2ab  2bc  2ca i   log   f  
 x y   1  xy 
 2 1 
2a  2b 2  2c 2  2ab  2bc  2ca ii   1  xy 
3a 2  3b 2  3c 2  1  x y 
dUcenH eXIjfa f x   f  y   f   Emn .
Taj)an 3a  b  c   1  a  b  c 
2 2 2 1 2 2 2  1  xy 
3
36> edaHRsaysmIkar x
 7  48    7  48   14
   
x

dUcenH eXIjfa a 2
 b2  c2 
1
R)akdEmn .
3 eXIjfa ³  7  48  7  48   49  48  1
34> bgðajfa log b  log c  log d .... log y  log
a b c x y a 1 Taj)an  7  48    1    7  48 
1

 7  48 
tamrUbmnþbþÚreKal log b  log
log b
a
a
eyIg)an ³  

log a b  log b c  log c d .... log x y  log y a  1 eyIg)ansmIkarfµI ¬CMnYs 7  48 eday 1


¦
7  48
log b log c log d log y log a  
x
  ...  1  1 x
   7  48   14
log a log b log c log x log y  7  48   
11  
tag t   7  48 
x

dUcenH log a b  log b c  log c d .... log x y  log y a 1 .

 x y 
smIkareTACa 1t  t  14 smmUl t 14t  1  0 2

35> bgðajfa f x   f  y   f  
 1  xy  man    7  1  48 2

1 x 
eyIgman ³ f  x   log 
1 x 

naM[ t    71 48  7  48   7  48 
2

1 y 
naM[)an ³ f  y   log 
t2 
  7   48
 7  48 
1
1 y  1 7  48
1 y 
eyIg)an ³ 1 x 
f x   f  y   log   log  
 7  48 
1

1 x  1 y 
2
 1  x  1  y 
 log      7  48 
 
 1  x  1  y 
cMeBaH
2

 1  x  y  xy  t1   7  48 
 log    
 1  x  y  xy 
enaH 
x 2
7  48    7  48 
 1  xy   x  y     
 log  
 1  xy    x  y   Taj)an x2
edayEckTaMgPaKyk nigPaKEbgnwg 1  xy  cMeBaH t 2   7  48 
2

 1  xy   x  y    
 
1  xy 
2
enaH  7  48    7  48 
x

naM[)an ³ f x   f  y   log  
 1  xy   x  y  
 1  xy  
 Taj)an x  2
dUcenH smIkarmanb¤s x  2  x  2 .
 678 
37> bgðajfa ³ 39> edaHRsaysmIkar ³ 5  50  x log x log 5

log a b  log b a  2log a b  log ab b  log b a  1  log a b Binitü 5 bMBak;elakarItenEBr eK)an


log x

Binitü ³ ln 5 log x  log x  ln 5


log a b  log b a  2log a b  log ab b   log b a  1 ehIy 5log x bMBak;elakarItenEBr eK)an
 log b log a  log b log b  log a
    2     1 ln x log 5  log 5  ln x
 log a log b  log a log ab  log b
 log 2 a  2 log a log b  log 2 b 
  1 
log a 
  1
eXIjfa log x  ln 5  log 5  ln x
 log a log b  log ab  naM[ 5  x
log x log 5

 log a  log b 2  
  1 

log a

  1 CMnYs x eday 5
log 5 log x

 log a log b   log a log b 


 log a  log b 2  log a  log b  log a  eyIg)ansmIkar 5  50  5 log x log x

  
   1
   2  5log x  50  5log x  25
 log a log b  log a log b
 log a  log b 2   5log x  25  5log x  5 2
log b
     1

 log a log b  log a  log b  Taj)an log x  2  x  10  100 . 2

 log a  log b 
    1  1  log a b  1  log a b dUcenH smIkarmanb¤sKW x  100 .
 log a 
dUcenH eRkayBIbgðajrYceXIjfa 40> KNnaRbU)abEdle)aH)anelxess b¤ elxFMCag 5
-RKab;LúkLak;manmux 6 naM[cMnYnkrNIGacesµI 6
log a b  log b a  2log a b  log ab b  log b a  1  log a b
-elxessrYmman ³ 1 , 3 , 5 man 3 krNI
naM[RbU)abe)aH)anelxessKW P ¬ess¦  63
38> bgðajfa 11  cos
cosa 1  cosa 4 cot a
 
a 1  cosa sin a -elxFMCag 5 KWmanEtelx 6 man 1 krNI
Binitü RBmTaMgKNnaGgÁTImYy eyIg)an ³ naM[RbU)abe)aH)anelxFMCag 5 KW P ¬ elxFMCag 5 ¦  16
1  cosa 1  cosa

1  cosa 1  cosa eyIg)an ³ RbU)abEdle)aH)anelxess b¤ elxFMCag 5

1  cosa   1  cosa 
2 2

KW P ¬ess¦ b¤ P ¬ elxFMCag 5 ¦  63  16  23
1  cosa 1  cosa 

1  cosa  1  cosa 1  cosa  1  cosa 
1  cosa 1  cosa  dUcenH RbU)abe)aH)anelxess b¤ elxFMCag5esµInwg 23 .
2 cosa  2 4 cosa 4 cosa 4 cot a
    
1  cos a sin a sin a sin a sin a
2 2
41> edaHRsaysmIkar
dUcenH 1  cosa 1  cosa 4 cot a
  BitEmn . eyIgman x  4x  16x  ... 4 x  3  x 1 n

1  cosa 1  cosa sin a


elIkGgÁTaMgBIrCakaer nigKNnaCabnþbnÞab; eyIg)an ³

 679 
x  4 x  16 x  ...  4 n x  3  x  1

 f 1  2 2  0
1 3
 3


x  4 x  16 x  ...  4 n x  3  x  2 x  1  2

 f 3  1 3 4  3 2 

4 x  16 x  ...  4 n x  3  2 x  1


  f 5  3 6  3 4
1
2


4 x  16 x  ...  4 n x  3  4 x  4 x  1 ............................

16 x  ...  4 n x  3  4 x  1  2

 f 999999  1 3 1000000  3 999998 


16 x  ...  4 n x  3  16 x  8 x  1
...  4 n x  3  8 x  1 f 1  f 3  f 5  ...  f 999997  f 999999 
1 3 6
2

10  0 
...................................................... 1
  100  50
2
4n x  3  2n x  1
4n x  3  4n x  2  2n x  1 dUcenH f 1  f 3  f 5  ...  f 999997   f 999999   50 .
2  2  2n x
2n x  1 43> edaHRsayRbB½n§smIkar
2 2n x  1  5log y x  log x y   26 i 
a/ 
1  xy  64 ii 
x
4n ¬GñkKYcaMrUbmnþbþÚreKal log x  log1 y ¦ y
x

dUcenH smIkarmanb¤s 1
x n
4
. -tam i  ³ 5log x  log y   26
y x

1 26
 log x y 
42> KNnaplbUk log x y 5

f 1  f 3  f 5  ...  f 999997   f 999999  5 log 2x y  26 log x y  5  0

f n  
1 tag log x y  t eyIg)ansmIkarfµIKW ³
n 2  2n  1  3 n 2  1  3 n 2  2n  1  26t  5  0 ¬tamDIsRKImINg;  ¦
3
5t 2
1

3
n  1 2
 n 1 
3 2 3
n  1 2 man    13  25  169  25  144
2

 n  1  n  1 3 3 naM[ t   135 12  15 / t    135  12  5


1 1

 n  1  n  1 n  1  n  1 
3 3 3 2 3 2 3
n  12 
 cMeBaH t
1
naM[ 1
log x y   y  x 5
1

5 5

1
n 1 n 1
 n  1  n  1 3 3
cMeBaH t  5 naM[ log y  5  y  xx
5


1
2

3
n 1  3 n 1  -tam ii : xy  64
1 1 6

krNI y  x enaH x  x  64  x  2  x  2
5 5 5 6 5

naM[eyIg)an ³
naM[ y  x  y  2   y  2
1 1
5 5 5

krNI y  x enaH x  x  64  x  2  x  2
5 5 6 6

 680 
naM[ y  x  y  2  y  32
5 5 
 

Dx  2 ln 4 ln 6  ln 2 5
 2

naM[


x
D 
ln 4 ln 6  ln 2 5 
dUcenH RbB½n§smIkarmanKUcemøIyBIrKW ³ 
 
 y  D y   2 ln 4 ln 6  ln 5  2
2

x  32 , y  2 b¤ x  2 , y  32  . 
 D 
ln 4 ln 6  ln 2 5 
 x y
 4  5  400
1
 dUcenH RbB½n§smIkarmanKUcemøIyKW ³
b/  x  2 , y  2 .
5 x  6 y 
1
 
 900
tam ³
 4 x  5 y  1
400
¬bMBak; elIGgÁTaMgBIr¦ 44> bgðajfa
ln

ln 4 x  5 y   ln 
 1  1  a1 1  a2 1  a3 ...  1  an   2 n

 400 
ln 4 x  ln 5 y  ln 1  ln 400 eday a CacMnYnviC¢man enaHeyIg)an ³
x ln 4  y ln 5  0  ln 20 2 1  a1  2 a1

x ln 4  y ln 5  2ln 4  5 1  a2  2 a2
x ln 4  y ln 5  2 ln 4  2 ln 5 i  
 1  a3  2 a3
tam ³
  5x  6 y  1
900
¬bMBak; elIGgÁTaMgBIr¦
ln ...................

1  a  2 a
   1 
ln 5 x  6 y  ln    n n

 900  1  a1 1  a2 1  a3 ... 1  an   2 n a1a2 a3 ...an


ln 5 x  ln 6 y  ln 1  ln 900
x ln 5  y ln 6  0  ln 30 2
smµtikmµ plKuN a  a  a  ...  a  1
1 2 3 n

x ln 5  y ln 6  2ln 5  6 enaHeyIg)an 1  a 1  a 1  a ... 1  a   2


1 2 3 n
n

x ln 5  y ln 6  2 ln 5  2 ln 6 ii  dUcenH 1  a 1  a 1  a ... 1  a   2 . n


1 2 3 n

tam i  nig ii eyIg)anRbB½n§smIkar ³


 x ln 4  y ln 5  2 ln 4  2 ln 5

45> edaHRsayRbB½n§smIkar
 x ln 5  y ln 6  2 ln 5  2 ln 6  7 x 3  3x 2 y  21xy 2  26 y 3  342
a/  3
edaHRsaytamedETmINg; ³ 9 x  21x y  33xy  28 y  344
2 2 3

D  ln 4 ln 6  ln 2 5 edaybUkGgÁngw GgÁ eyIg)an ³


Dx   ln 4  2 ln 5ln 6   2 ln 5  2 ln 6ln 5  7 x 3  3x 2 y  21xy 2  26 y 3  342
 3
 2 ln 4 ln 6  2 ln 5 ln 6  2 ln 2 5  2 ln 5 ln 6 9 x  21x y  33xy  28 y  344
2 2 3

 2 ln 4 ln 6  2 ln 2 5 16 x 3  24 x 2 y  12 xy 2  2 y 3  686

 2 ln 4 ln 6  ln 2 5  8 x 3  12 x 2 y  6 xy 2  y 3  343
D y  ln 4 2 ln 5  2 ln 6  ln 5 2 ln 4  2 ln 5
2 x 3  3  2 x 2  y  3  2 x   y 2  y 3  7 3
 2 ln 4 ln 5  2 ln 4 ln 6  2 ln 5 ln 4  2 ln 2 5
2 x  y 3  7 3
 2 ln 4 ln 6  2 ln 2 5
2x  y  7 i 

 2 ln 4 ln 6  ln 2 5 
edaydkGgÁnwgGgÁ eyIg)an ³
 681 
 7 x 3  3 x 2 y  21xy 2  26 y 3  342
 3 46> rkcMnYnmanelxR)aMxÞg;enaH
9 x  21x y  33xy  28 y  344
2 2 3

 2 x 3  18x 2 y  54 xy 2  54 y 3  2 -cMnYntamxÞg;nImYy²RtUvEt FMCag0 ¬eRBaHxÞg;edImKW x ebI


x 3  9 x 2 y  27 xy 2  27 y 3  1
x  0 vaBuMEmnCaelxR)aMxÞg;eT vaKWCaelxbYnxÞg; ¦ nigtUcCag b¤
x 3  3  x 2  3 y   3  x  3 y   3 y   1
2 3

x  3 y 3  13 esµI 9 ¬eRBaH ebIFMCag9 vaCaelxeRcInxÞg; naM[cMnYnxÞg;elIsBI5¦ .


x  3y  1 ii 
tamry³ i  nig ii  eyIg)anRbB½n§smIkarfµIKW³ eyIgmancMnYn ³ x , x 1 , x  2 , 3x , x  3 naM[
2 x  y  7  2x  y  7 0  x  9 0  x  9
 smmUl 
  2 x  6 y  2
0  x 1  9 0  x  8
 x  3y  1  
naM[ x  1 3y  1 3  4
5y  5  y  1 0  x  2  9 smmUl 0  x  7
 0  3x  9 0  x  3
 
dUcenH RbB½n§smIkarmanKUcemøIy x  4  y  1 . 0  x  3  9 0  x  6

4 x  3 y 1  27 y  171
b/  x eyIgGacsresrCa tamry³lkçxNÐTaMg5xagelI lkçxNÐrYmKW 0 x3
1 2 y
8  2  3  172
x

   
 2 x  3  3 y  3 y  171

2 3 mann½yfa ³ x  1 , 2 , 3
 x 3 erob[tamlMdab;
 
 2  2  3  3  172
x
 
y 2
cMeBaH x  1 cMnYnenaHKW 12334 minyk ¬eRBaHminCakaerR)akd¦
edaybUkGgÁngw GgÁ ³ cMeBaH x  2 cMnYnenaHKW 23465 minyk ¬eRBaHminCakaerR)akd¦
   2
 
3
3  2 x  3 y  3 y  171
 x 3

2    
2
 3  2 x  3 y  172
cMeBaH x  3 cMnYnenaHKW 34596  186 ¬CakaerR)akd¦ 2

2 
x 3
 2 2
   
3
 3  2 x  3 y  3  2 x  3 y  3 y  343 dUcenH cMnnY manelxR)aMxÞg;enaHKW 34596 .
2  3  x y 3
7 3

2 3  7
x y
i  47> KNnaplbUk S ³ eyIgman
edaydkGgÁnwgGgÁ ³ S
1

1

1
 ... 
1
  
3  2
 x 3
x 2
 
3y  3  171 y 3 1 2 2 3 3 4 2024  2025

2    
 3 2  3
x y 2
 172 Binitü rYcbUkGgÁnigGgÁ énkenSamxageRkam
3 
y 3
 3 2  3
x
 
y 2
 
 3 2 x 2
 
y 3
 3  2 x  1
 2 1  2 1  1  
1
 2 1
3  2    1 y x 3 3 2 1

3 y  2 x  1 ii   3 2  3  2 1  
1
3 2
 3 2

edayyk i   ii eyIg)an ³  4 3  4 3 1 


4
1
3
 4 3

 2 3  7
>>>>>>>>>>>>>>>>>>>>>>>>>>>>>>>>>>>>>>>>>>>>>>>>>>>>>>>>>>>>>>>>>>>>>>>>>>
x y
 y
3  2  1
x

2  3y  2  3  2025  2024  2025  


2024  1 
1
2025  2024
 2025  2024
3 3y
 y 1
1 1 1
S   ...   2025  1
cMeBaH y  1 enaH 2 3  7
x y
1 2 2 3 2024  2025
2x  7  3  45  1  44
2 x  22  x2 dUcenH plbUkKNna)an S  44 .
dUcenH RbB½n§smIkarmanKUcemøIy x  2 nig y  1 .
 682 
48> RsaybBa¢ak;fa ³ f n  3  7 Eckdac;nwg 8 2n
50> RsaybMPøWfa n3  n2  n6 CacMnYnKt;viC¢man
2 3

eyIgman f n  3  7 2n
tag A  n3  n2  n6 2 3

cMeBaH n  0 : f 0  3  7  1  7  8 Eckdac;nwg 8


0

n  1 : f 1  3  7  9  7  16 Eckdac;nwg 8
2

eyIg)an A
2n  3n 2  n 3
>>>>>>>>>>>>>>>>>>>>>>>>>>>>>>>>>>>>>>>>>>>>>>>>>>>>>>>>>>>>>>>>>> 
6
n 2  3n  n 2 

]bmafa vaBitdl; n  k : f k   3  7 Eckdac;nwg 8 2k
6
nn  1n  2 
eyIgnwgRsaybBa¢ak;fa n  k  1 k¾Eckdac;nwg 8 Edr 
6

cMeBaH n  k  1 : f k  1  3    7 ebI A CacMnYnKt; luHRtaEt nn  1n  2 Eckdac;nwg 6


2 k 1

3 7 mann½yfa nn  1n  2 Eckdac;nwg 2 pg nig 3 pg


2k 2

 93  7 2k

 8  3  3  7 
-ebI n CacMnYnKt; enaH n , n 1, n  2 CabIcMnYnKt;tKña
2k 2k

eday 8 3 Eckdac;nwg 8 ¬eRBaH 8 3 CaBhuKuNén 8 ¦ -bIcMnYnKt;tKñay:agehas; k¾mancMnYnKUmYyEdr


2k 2k

niig 3  7 Eckdac;nwg 8 ¬]bmafaBit xagelI ¦


2k
naM[ nn  1n  2 Eckdac;nwg 2
naM[ f k  1  8  3  3  7 Eckdac;nwg 8
2k 2k
-bIcMnYnKt;tKñay:agehas; k¾mancMnYnmYyCaBhuKuN
dUcenH f n  3  7 Eckdac;nwg 8 RKb; n CacMnYnKt; . én 3 Edr naM[ nn  1n  2 Eckdac;nwg 2
2n

eyIgGacbkRsaytamviFImüa:geTot enaHeyIg)an nn  1n  2 Eckdac;nwg 6


9  1 8 ¬mann½yfa 9 Eckdac;nwg 8 [sMNl; 1 ¦ 2 3
dUcenH RKb;cMnYnKt;viC¢man n enaH n3  n2  n6
9  1 8 ¬GacelIkCasVy
n n
½ KuN)an RKb; n CacMnYnKt;¦
3  1 8 ¬eRBaH 9  3 ehIy 1  1 ¦
2n 2 n k¾CacMnYnKt;viC¢manEdr .
3  7  1  7 8 smmUl 3  7  8 8
2n 2n

51> KNna S  11 2  21 3  31 4  ... 20101 2011


Et 8  0 8 naM[ 3  7  0 8 Edr
2n

n 1 n
dUcenH f n  3  7 Eckdac;nwg 8 RKb; n CacMnYnKt; . Binitü ³ n  n  1  nn  1  nn  1
1 1 1
2n

tamlMnaMxagelIeKGacbMEbk)an dUcxageRkam ³
49> KNna g 3 ³ 1 1 1
 
eyIgman f x  2x  1 1 2 1 2
1 1 1
 
[ f x  3 smmUl 2x  1  3  x  1 23 2 3
ehIyman g f x  x  3x  1 2
bUkGgÁnwgGgÁ 31 4  13  14
naM[ g 3  1  3 1  1 ¬eRBaH f x   3 nig x  1 ¦
2
>>>>>>>>>>>>>>>>>>>>>>>
g 3  5 1 1 1
 
dUcenH eRkayBIKNna g 3  5 . 1 1
2010  2011 2010 2011
1 1 1 1
   ...  
1 2 2  3 3  4 2010  2011 1 2011

 683 
b¤ 1

1

1
 ...
1

2010 -yl;fa ³ cd CacMnYnKt; enaHRtUvEtmanktþamYy kñúg
1 2 2  3 3  4 2010  2011 2011
cMeNamktþa n 10  nig n  10  EdlRtUvEtEckdac;nwg
dUcenH eRkayBIKNnaplbUk S  2010 . 101 eRBaH 101CacMnYnbzm . enaHeyIgTaj)an ³
2011
n  10  101b¤ n  10  101
52> KNna x  x 1 2011
2011 naM[ n  111 b¤ n  91
eyIgman x  x 1  0 Taj)an
2
x
1
 1 enaH -cMeBaH n  111 minyk eRBaH n Caelx4xÞg; enaH n  100
2

x
 1  1 
  x   x     1 1  x 2  2  2  1  x 2  2  1
1 1 -cMeBaH n  91 enaH n  8281 yk 2

 x 
 1 
x
1
x x
dUcenH cMnYnenaHKW abcd  n  8281 . 2

  x   x 2  2    1 1  x 3  3  x   1  x 3  3  2
1 1 1
 x  x  x x x
 1  3 1 
  x   x  3    1  2  x  4  x  2  2  x  4  1
4 1 2 1 4 1 54> bgðajfa n! 3 cMeBaH n  7 n

 x  x  x x x
 1  1
  x   x 4  4    1 1  x 5  5  x 3  3  1  x 5  5  1
1 1 1 eyIgman n! 3 n

 x  x 
 1  5 1 
x x
  x   x  5    1 1  x  6  x  4  1  x  6  2
6 1 4 1 6
x
1 -cMeBaH n  7 enaH 7! 5040  3  2187 ³ Bit 7

 x  x  x x x
-cMeBaH n  8 enaH 8! 40320  3  6561 ³ Bit 8

>>>>>>>>>>>>>>>>>>>>>>>>>>>>>>>>>>>>>>>>>>>>>>>>>>>>>>>>>>>>>>>>>>>>>>>>>>>>>>>> >>>>>>>>>>>>>>>>>>>>>>>>>>>>>>>>>>>>>>>>>>>>>>>>>>>>>>>>>>>>>>>>>>>>>>>>>>>>>>>>>>>>>>
tamlMnaMenH eyIgGacTaj)antYTUeTAKW ³ -]bmafavaBitrhUtdl; n  k enaH k! 3 ³ Bit k

1  1 ebI n  3k
x  n
 Edl k CacMnYnKt;FmµCati -eyIgnwgRsaybB¢aak;fa vaenAEtBitcMeBaH n  k 1
 2 ebI n  3k
n
x
eday 2011  3  670  1 manTRmg;xusBI 3k eyIg)an k  1 ! 3 k 1

1
naM[ x 2011  2011
 1 k  1  k !  3  3k
x
dUcenH eRkayBIKNna eXIjfa x 2011

1
 1 . eday k! 3k ehIy k  1  7  3
enaHeyIg)an k  1  k !  3  3 KWCakarBit
2011
x k

53> rkcMnYn abcd enaH dUcenH eXIjfa n! 3 cMeBaHRKb;cMnYnBitviC¢man n  7 .


n

tambRmab;RbFan cMnYn abcd CakaerR)akd -eRBaHebI k! 3 nig k  1 3 naM[ k  1k! 3  3
k k

eyIg)an abcd  n Edl n CacMnYnKt;


2
b¤ k  1 ! 3 . k 1

eyIgsresr)an ab00  cd  n 2
55> bgðaj[eXIjfa f CaGnuKmn_xYb
enaHTaj)an 100 ab  cd  n 1 2
GnuKmn_ f manEdnkMNt; D  0 ,  
bRmab;bEnßm ab  cd  1 b¤ ab  1  cd 2 cMeBaH RKb; x  D enaH x  2a  D eRBaH a  0
yk 2 CMnYskñúg 1 ³ eyIgman ³ f x  a  12  f x  f x 2

 
100 1  cd  cd  n 2
naM[ f x  2a  f x  a  a
100  100cd  cd  n 2
f x  a   f x  a 
1
 
2
101cd  n 2  100 2

cd 
n  10n  10 1 2  1 2 
2

f x   f x      f x   f x  
1
   
101 2 2  2 

 684 
1 1 2  1 2
    f x   f x      f x   f x   f x   f x  
2
cgCaRbB½n§  2
21n  4  ad

2 2   4   3
14n  3  bd
   f x   f x    f x   f x   f x   f x  42n  8  2ad i 
1 1 2 1 2 2

2 2 4 
42n  9  3bd ii 
   f x   f x 
1 1 2

2 4
2
edayyk ii  i  eyIg)an ³
1  1
   f x     42n  9  3bd
2  2 
42n  8  2ad
  f x  
1 1
2 2 1  3b  2a d
 f x  eday a nig b CacMnYnKt; enaH 3b  2a k¾CacMnYnKt;Edr
eRBaH f x   cMeBaH x  0
1
2 ehIy d CatYEckrYmFMbMput RtUvEtCacMnYnKt;
eyIg)an f x  2a  f x cMeBaH RKb; x  0 ,   -yl;fa ³ plKuNBIrcMnnY Kt;esµI ! manEtmYykrNIKt;
dUcenH GnuKmn_ f CaGnuKmn_xYbenAelI 0 ,   . KWcMnYnKt;nImyY ²esµIKña ehIyesµInwg ! .
Taj)an d  1
56> bgðaj[eXIjfa 0 ! 1 21n  4
dUcenH F  14 CaRbPaKsRmYlmin)an .
eyIgman ³ n! n  n 1 n  2 ...  3 2 1 n3

vaBitcMeBaH n CacMnYnKt;viCm¢ an 58> rkcMnYnKt;FmµCatitUcbMput


mann½yfa n  1 , 2 , 3 , ...  -tambRmab; ³ eyIgBMudwgcMnYnxÞg; éncMnnY enaHeT
eyIgGacsrsr)andUcTRmg;xageRkam ³ Etvamanelx 6 enAxÞg;Ékta ¬xÞg;ray¦
n !  n  n  1 ! -eyIgGactagcMnYnenaHeday ³ a a a ...a 6 EdlTIenH 1 2 3 n

eRBaHedaysar n  1 ! n  1 n  2 ...  3  2 1 0  a , a , a ,... a  9 nig a a a ...a CacMnYnKt;


1 2 3 n 1 2 3 n

eyIgTaj)an n  1 ! nn! -eyIg)ancMnnY fµI 6a a a ...a 1 2 3 n

KNnatémøCaelx cMeBaH n  1 -tambRmab;eyIg)anTMnak;TMng ³


4  a a a ...a 6  6a a a ...a
naM[ 1  1 ! 11!  0 ! 11  1 1 2 3 n 1 2 3 n

4a a a ...a 0  6  6000...0  a a a ...a


1 2 3 n 1 2 3 n
dUcenH eXIjfa 0 ! 1 BitR)akdEmn . 4a a a ...a  10  6  6  10  a a a ...a
1 2 3 n
n
1 2 3 n

eRBaH 1 ! 1 edIm,I[gayRsYlkñúgkarsresr tag a a a ...a  A 1 2 3 n

21n  4 eyIg)an ³ 410 A  6  6 10  A n

57> bgðajfa F  14 n3


CaRbPaKsRmY l mi n )an 40 A  24  6 10  A n

39 A  6 10  24
tag d CatYEckrYmFMbMputén 21n  4 nig 14n  3
n

2 10  8 210  4 n n
A 
eyIg)an 21n  4  ad Edl a CacMnYnKt;viC¢man 13 13

14n  3  bd Edl b CacMnYnKt;viC¢man -eday A CacMnYnKt; ehIy 2 Eckmindac;nwg 13 enaH


manEt 10  4 Eckdac;nwg 13
n

 685 
-eyIgnwg[témø n bnþbnÞab;tUcbMputedIm,I[1310  4 n
60> rk 11 cMnYnenaH EdlminGviC¢man
¬mann½yfa 13 CatYEckén 10  4 ¦ n
tag11cMnYnenaHeday ³ x , x , x , ... , x 1 2 3 11 0

-cMeBaH n  1 : 10  4  6 Eckmindac;nwg 13 tambRmab;RbFan eyIg)an ³


n  2 : 100  4  96 Eckmindac;nwg 13 x1  x2  x3  x4  ...  x11 
2
1
n  3 : 1000  4  996 Eckmindac;nwg 13 x2  x1  x3  x4  ...  x11 
2
2
n  4 : 10000  4  9996 Eckmindac;ngw 13 x3  x1  x2  x4  ..  x11 
2
3
n  5 : 100000  4  99996 Eckdac;nwg 13 >>>>>>>>>>>>>>>>>>>>>>>>>>>>>>>>>>>>>>>>>>>>>> >>>
>>>>>>>>>>>>>>>>>>>>>>>>>>>>>>>>>>>>>>>>>>>>>>>>>>>>>>>>>>>>>>>>>>>>>>>>>>>>>>>>> x11  x1  x2  x3  ..  x10 
2
11
-cMeBaH n  5 eyIg)an A  21013 4  15384
5
-edaydkGgÁnigGgÁén 1 - 2 eyIg)an ³
dUcenH cMnYnenaHKW 153846 . x1  x2  x2  x3  x4  ...  x11   x1  x3  x4  ...  x11 
2 2

tamrUbmnþ a 2  b 2  a  b a  b  enaH


59> etIbursNa CaKUnwgnarINa ? x1  x2  x2  x3  x4  ...  x11   x1  x3  x4  ...  x11 

-eXIjfamanbursbInak;clU sþIdNþwgnarIbInak;. burs x2  x3  x4  ...  x11   x1  x3  x4  ...  x11 


 x2  x1 x1  x2  2x3  2x4  ...  2x11 
EdlcUlsþIdNþwgdMbUg manCeRmIsbI burscUlsþIbnÞab;
2

 x1  x2 x1  x2  2x3  2x4  ...  2x11 


manCeRmIsBIr nigburscugeRkaymanCeRmIsEtmYyKt;
2

-tameKalkarN_plKuN ³ cMnYnCeRmIsTaMgGs;KW x1  x2   x1  x2 x1  x2  2x3  2x4  ...  2x11 2  0


x1  x2 1  x1  x2  2x3  2x4  ...  2x11 2  0
3  2 1  3! 6 CeRmIs EdlCeRmIsTaMgenaHrYmman³
1 A CaKUnwg1 / B CaKUnwg 2 / C CaKUnwg 3 ¬xuslkçxNÐ K¦ naM[ x  x   0  x  x
1 2 1 2

2  A CaKUnwg 1 / B CaKUnwg 3 / C CaKUnwg 2 ¬xuslkçxNÐ k¦


ehIy 1 x  x  2x  2x  ...  2x   0
1 2 3 4 11
2

3 A CaKUnwg 2 / B CaKUnwg 1 / C CaKUnwg 3 ¬xuslkçxNÐ K¦ edayyk 1 - 3 / 1 - 4 / …/ 1 - 11 tam
4  A CaKUnwg 2 / B CaKUnwg 3 / C CaKUnwg 1 ¬xuslkçxNÐ K¦
lMnaMxaelIenaHeyIg)an ³ x  x  x  ...  x 1 2 3 11

5 A CaKUnwg 3 / B CaKUnwg 1 / C CaKUnwg 2 ¬xuslkçxNÐ x¦


naM[ x  x  x  x  ...  x 
1 1 1 1 1
2

x1  10 x1 
2

6  A CaKUnwg 3 / B CaKUnwg 2 / C CaKUnwg 1


x1  100 x12
-eXIjfaedIm,IeKarBtamlkçxNÐmanEtmYyCeRmIsmYyKt;KW³ x1 1  100 x1   0

A CaKUnwg 3 / B CaKUnwg 2 / C CaKUnwg1 Taj)an x  0 , 1  100x  0  x  100


1
1
1 1

dUcenH KUsVamIPriyafµI EdleKarBtamlkçxNÐKW ³ dUcenH cMnYnTaMg 11 EdlrkeXIjKW


x  x  x  ...  x  0 b¤
1 2 3 11
burs A CaKUnwgnarI 3 burs B CaKUnwgnarI 2 burs C CaKUnwgnarI1 .
x  x  x  ...  x 
1 2
1
100
. 3 11

 686 
61> KNna A  a  1  b  1  a  1 2010 2011 2012

2010
  
n2 n2  n 1  n n2  n 1  n2  n 1   
n 2 n  1
2
abc  0
eK[ a , b , c epÞógpÞat; ab
n 1

  bc  ca  0

2010
n 2
 n 1 2

cMeBaH a  b  c  0 n n  1
2 2
n 1

a  b  c 2  0 n 2  n  1 2010 nn  1  1 2010 


2010
1 
    1 
a 2  b 2  c 2  2ab  bc  ca   0 n 1 nn  1 n 1 nn  1 n 1  nn  1
1 1 
eday enaHeyIg)an
2010 2010 2010
1
ab  bc  ca  0  1    2010     
n 1 n 1 nn  1 n 1  n n 1
a 2  b2  c 2  0
-yl;fa ³ plbUkkaerénmYycMnYnBitesµI 0 mann½yfa eyI g )an S  2010    
1 1 
 n n  1

2010

n 1

1 1 
cMnYnBitTaMgenaHmantémøesµI²Kña ehIyesµInwg 0  2010   
 1 2011 

-naM[ a  b  c  0  2011 
1

2011  1 2 2

2011 2011
eK)an ³ A  a  1  b  1  a  1 2010

2011 12011 1  2010  2012
2011 2012

2011 2011
 0  1  0  1  0  1
2010 2011 2012

dUcenH eRkayBIKNnaplbUk S  2010 2012


2011
.
 1   1  1  1
 mUlehtu   n  n  1    1  2011  emIllMhat;TI51 .
1 1  1 1 
2010

dUcenH eRkayBIKNnaeXIjfa A  1 . n 1

62> KNna 63> edaHRsaysmIkar ³


S  1
1
2
1 1 1
 2  1  2  2  ...  1 
1
2

1
2011 2
k> x  x  x  x
2010 2011 2012 2013
4
x  1  x  1  x  1  x 2013  1  0
1 2 2 3 2010
2010 2011 2012
eXIjfa S CaplbUkmantYTUeTAKW 1
1 2 
1
n  12
tamrUbmnþ³  
n
a n  1  a  1 a n1  a n2  ...  a  1
EdlmancMnYntYsrub 2010 tY
 x 2010  1  x  1x 2009
 x 2008  ...  x  1
eKGacsresrCa S   1  n1  2010
1
n  12  1  x  1x  ...  x  1
2
n 1  x 2011 2010
 x 2009

 1  x  1x  ...  x  1
2010
1 1
S 
n 1
1
n 2

n  12  x 2012 2011
 x 2010
2010
n 2 n  1  n  1  n 2
2 2
 x 2013  1  x  1x 2012
 x 2011  ...  x  1
  n 2 n  1
n 1
2
edIm,I[gaysRmYlkñgú karsresr eyIgtag
2010 2
 2
 
n n  2n  1  n  2n  1  n 2
 2
  X  x  x  ...  x  1 enaHeyIg)an
2009 2008
n n  1
2 2
n 1

2010
n 4  2n 3  n 2  n 2  2n  1  n 2 x 2010  1  x  1X
  n 2 n  1
 
2

x 2011  1  x  1 x 2010  X
n 1

2010
n  2n  3n  2n  1
4 3 2

 
n 1 n 2 n  1
2 x 2012  1  x  1x 2011
 x 2010  X 
n4  n3  n2  n3  n2  n  n2  n  1  1  x  1x  x 2011  x 2010  X 
2010
x 2013 2012
 
n 1 n 2 n  1
2

 687 
x 2010
 1  x 2011  1  x 2012  1  x 2013  1  manEt cMnYnKt; x  5 EtmYyKt;EdlnaM[ 4  x
x  1x 2012
 2x 2011
 3x 2010
 4X 
Eckdac;nwg( ¬ eRBaH 0  x  9 ¦
naM[ x  1x  2 x 2012 2011
 3x 2010  4 X  0  -epÞógpÞat; ³ 2 x9 y  2  9 x y

Taj)an ³  x  1  0  x 1
2592  25  9 2
x 2012
 2x 2011
 3x 2010
 4X  0
2592  32  81
eRBaH x CacMnYnKt; 2592  2592
dUcenH smIkarmanb¤s x  1 .
dUcenH smIkarmancemøIy x  5 , y  2 .
x> x 2010  x 2011  x 2012  x 2013  0 ¬kMueXIjEvgfaBi)ak eKRKan;EtsRmay[eyIgyl;BI

x 2010 1  x  x 2  x 3  0  mUlehturbs;va)anc,as;las;b:ueNÑaH ¦
Taj)an ³  x 2010  0  x  0 65> rkRKb;cMnYnKt;FmµCati N  a a a ...a 1 2 3 n

 1  x  x  x  0 ¬eRBaH x CacMnYnKt;¦
2 3

eyIgman 12aa aa aa ......aa 21  12


21
Edl 1 2 3 n

dUcenH smIkarmanb¤s x  0 . 1 2 3 n

N  a a a ...a mancMnYn n tY naM[eyIg)an ³ 1 2 3 n

64> edaHRsaysmIkar ³ 2 x9 y  2  9  2a a a ...a 1  2 10  10a a a ...a   1


x y
1 2 3 n
n 1
1 2 3 n

-lkçxNÐ 0  x  9 , 0  y  9 nig x , y CacMnYnKt;  2 10  10 N  1 n 1

1a a a ...a 2  110  10a a a ...a   2 n 1

-Binitü 2 CacMnYnKU enaH 2  9 CacMnYnKUEdr


x x y
 10  10 N  2
1 2 3 n
n 1
1 2 3 n

naM[ 2 x9 y CacMnYnKU mann½yfa y CacMnYnKU b¤ 0 1 eyIg)an 2 10  10 N  1  21 naM[ n 1

n 1
10  10 N  2 12
-Binitü ebI y  4 enaH 9  6561  2 x9 y naM[
4

2110  10 N  2  122 10  10 N  1 n 1 n 1

y CacMnYnKt;tUcCag4 mann½yfa y  0 ,1, 2 , 3  2 2110  210N  42  24 10  120N  12 n 1 n 1

-tam 1 nig 2 Taj)an y  0 b¤ y  2 210N  120N  24 10  2110  12  42 n 1 n 1

n 1
-cMeBaH y  0 enaH 2 x90  2  1  2 x90  2 x 90 N  3 10  30
x

30 10  30 n

Et 2 CacMnYnminmanelx 0 enAxagcugCadac;xat
x N
90
naM[ 2x90  2 CakrNIminBit RKb; x CacMnYnKt;
x
N
10  1 n

-cMeBaH y  2 enaH 2 x92  2  9  2 x92  81  2 x 2 3 x

Binitü 10  100...000 naM[ 10 1  99...999 n n

edaycMnYn 81 2 CaBhuKuNén( enaH 2x92 CaBhuKuN


x

n dgelx 0 n dgelx (
én( Edr mann½yfa 2x92 Eckdac;nwg( . 10  1 99 ... 999 n

-ehIy 2x92 Eckdac;nwg( luHRtaplbUkelxtamxÞg; eyI g )an N


3

3
 33 ... 333
n dgelx #
KW 2  x  9  2  13  x  1 3  x  4  x Eckdac;nwg(
enHmann½yfa N   3 , 33 , 333 , 3333 , ... 
 688 
epÞógpÞat; ³ ebI N  3 , 132
231 21  11 21 Bit
 
12  11 12
68> bgðajfa ³ x 1 1  x 1 1  x 2 1  x 4 1  x 8 1
2 4 8

dUcenH RKb;cMnYnKt;rk)anKW N  33...333 . Binitü ³  x 11  x 1 1   x 2 1  x 4 1


2 4

n dgelx #
  x  1   x  1  2 4
    2  4
  x  1x  1  x  1 x  1
66> KNnaplbUk ³  2  2
2 2
 4
4
1 1 1 1 x 1 x 1 x 1
S    ...     
1 2  3 2  3  4 3  4  5 nn  1n  2 
2 x2 1  2 x2 1
 4
4

x 1 x 1
2
 
2
x 1

Binitü ³ nn1 1  n  11n  2  nnn12nn2 


2x2  2  2x2  2
x 1
4
 4
4
x 1


2 4
 4  4
4


4 x4 1  4 x4 1   
nn  1n  2 x 1 x 1 x4 1 x4 1   
4x  4  4x  4
4 4
8
1 1 1 1    8
Taj)an   
nn  1n  2 2  nn  1 n  1n  2
x 1
8
x 1

tamlMnaMenH eyIgGacsresr)andUcxageRkam ³ dUcenH 1



1
 2
2
 4
4
 8
8
.
x 1 x 1 x 1 x 1 x 1
1 1 1 1 
  
1  2  3 2 1  2 2  3  69> KNnaplbUk S  21!  32!  43!  ...  n n 1!
1 1 1 1 
Binitü ³ n n 1!  nn11!1
  
2  3  4 2  2  3 3  4 
 1 1 1 1 
  
3  4  5 2  3  4 4  5  n  1  1

>>>>>>>>>>>>>>>>>>>>>>>>>>>>>>>>>>>>>>>>>>>>>>>>>>>>>>>>>>>>>>>>>>>>>>>>>>>>>>
1 1 1 1 
n  1! n  1!
nn  1n  2 2  nn  1 n  1n  2
  

n  1  1  1  1
1 1 1  n  1 n! n  1! n! n  1!
S 
2 1  2 n  1n  2

1  n 2  3n  2  2 
¬eRBaH n  1! n  1 n! ]Ta> 5! 5 4! ¦
S 

2  2 n 2  3n  2 

 eXIjfa eKGacbMEbk n n 1!  n1!  n 1 1!
n  3n2
S

4 n 2  3n  2  eyIgGacbMEbktYnImYy² rYcbUkGgÁnigGgÁ dUcxageRkam³
n 2  3n
dUcenH eRkayBIKNnaeXIjfa S

4 n 2  3n  2  .
1 1 1
 
2! 1! 2!
2 1 1
 
67> sRmYlkenSam ³ 2  3  2  3 2011 2012
 3 3! 2! 3!
3 1 1
tag E  2  3   2  3   3
2011 2012
  
4! 3! 4!
 3   2  3   2  3  3
 2
2011 2011

>>>>>>>>>>>>>>>>>>>>>>>>>
 2  3  2  3   2  3   3
2011

 2  3   3
n 1 1
 4  3
2011
 
n  1! n! n  1!
 2 3 3
1 2 3 n 1 1
2    ...  
2! 3! 4! n  1! 1! n  1!
dUcenH sRmYl)an 2  3  2  3 2011 2011
 32 . S  1
1
n  1!
 689 
dUcenH 1 n  1!1 .
S  1  1!  1 2!  2 3!  6 4!  4 5!  0
n  1! n  1!
6!  0 7!  0 >>>>>>>>8!  0 9!  0
70> kMNt;elxcugén   1  2  3  ... 2010  2011 eyIgBinitüeXIjfa elxxagcugéncMnYnTaMgLay Edl
2 2 2 2 2

¬yl;dwg³ eKsresr 13   9 mann½y elxxagcug FMCag $ suT§EtCaelxsUnüdUc²Kña TaMGs; naM[ ³


2

én 13  169 manelx 9 enAxagcug .¦


2
  1!2!3!... 2010!2011!
Binitü   1!  2!  3!  ... 2010!  2011!
 1  2  6  4  0  0  ... 0  0  0
1   11   21   ...  2001   1
2 2 2 2
3
2   12   22   ...  2002   4
2 2 2 2
dUcenH elxxagcugén  KW   3 .
3   13   23   ...  2003   9
2 2 2 2

4   14   24   ...  2004   6


2 2 2 2 72> KNnatémø A  a  ab 2012 2011

5   15   25   ...  2005   5


2 2 2 2 eyIgman a  a 1b  b 1 1 
2 2

6   16   26   ...  2006   6


2 2 2 2 -KuN  nwg a  a 1 enaHeyIg)an ³
2

7   17   27   ...  2007   9


2 2 2 2
a   
a 2 1 a  a 2 1 b  b2 1  a  a 2 1  
8   18   28   ...  2008   4
2 2 2 2
a 2

 a2 1 b  b2 1  a  a2 1   
9   19   29   ...  2009   1
2 2 2 2
 b  b2 1  a  a2 1 1
10   20   30   ...  2010   0
2 2 2 2 -KuN  nwg b   vijmþg enaHeyIg)an ³
b2 1

edayplbUkelxcug² éndb;²tYCabnþbnÞab;esµI²Kña KW a  a 1b  b 1b  b 1 b  b 1


2 2 2 2

1  4  9  6  5  6  9  4 1  0  45 ehIy a  a  1b  b  1  b  b  1
2 2 2 2

 a  a2 1  b  b2 1 2
  1  2  3  ...  2010  2011
2 2 2 2 2

man 201 éndb;²tY -tam 1 nig 2 cgCaRbB½n§ bUkGgÁ nigsRmYl
naM[   12  22  32  ... 20102  20112  eyIg)an ³   b  b  1  a  a  1 2 2

 a  a 2  1  b  b 2  1
 201 45  20112 
a b  a b
 5  1  6
 2a  2b
dUcenH elxxagcugén  KW   6 . a  b
-naM[ A  a 2012  ab 2011
71> kMNt;elxcugén   1!2!3!... 2010!2011! a  a a   a  a a 
2012 2011 2012 2011

¬GñkKYKitfa mYycMnYnEdlKuNnwg !0 b¤ )anBIplKuN a  a   1  a a   1  a


2012 2011 2011 2012 2012

én @ nig% suT§swgEtpþl; cMnYnEdlmanelx0 xagcug¦ a a 0 2012 2012

rUbmnþ hVak;tEU rül n! n  n 1 n  2...  3 2 1 dUcenH eRkayBIKNna )anplmk A  0 .


eyIg)an ³
 690 
73> edaHRsaysmIkar ³ b¤ x 2
   
 2x 1  y 2  2 y 1  z 2  2z 1  0 
eyIgman 2011  x log 2012 x log 2012 2011
 4022 x  12   y  12  z  12  0
-tag A  2011 log 2012 x eday x  12  0 ,  y  12  0 , z  12  0
naM[ log A  log 2011
2012 2012
log 2012 x manEtmYykrNIKt; EdlepÞógpÞat;smIkarxagelIKW
log 2012 A  log 2012 x  log 2012 2011 i  témøénktþanImYy²esµI 0
-tag B  x log2012 2011 x  12  0  x  1  0  x  1

naM[ log 2012 B  log 2012 x log 2012 2011
eyIg)an³  y  1  0  y  1  0  y  1
2


z  1  0  z  1  0  z  1
2

log 2012 B  log 2012 2011  log 2012 x ii 

edaypÞwm i  nig ii naM[ A  x 2010  y 2011  x 2012


  1   1   1
2010 2011 2012

eyIg)an log A  log B smmUl A  B


2012 2012  1   1  1  1
-smIkareTACa 2011  2011  4022 log 2012 x log 2012 x
dUcenH eRkayBIKNnaeXIjfa A 1 .
2  2011 log 2012 x
 2  2011
 bc
76> bgðajfa ad2ab 2cd
2011log 2012 x  20111

ad  bc
log x 1
eyIgman b  d naM[ c  d eyIg)an ³
2012
a c a b
x  2012
dUcenH eRkayBIedaHRsaysmIkarmanb¤s x  2012 . a  ad  ab 1 ehIy b  bc  ab 2
c cd cb d dc ad
bc ad  bc ad  bc
74> Rsayfa a  b  a  b a  b   a  b naM[ ad
5 5
 3

cd dc cd  dc
3
 2
2cd
i  2

Binitü a  b a  b   a  b
3 3 2 2 ab ab ab  ab
  
2ab
ii

 a 5  a 3b 2  a 2 b 3  b 5  a  b  cb ad cb  ad cb  ad

 a 5  b 5  a 2 b 2 a  b  a  b
tam 1 enaH i   ii eyIg)an ³
 a 5  b 5  a  b a 2b 2  1 ad  bc

2ab
naM [ ad  bc

2cd
2cd cb  ad 2ab cb  ad
edaysmµtikmµ eK[ ab  1 enaH a b 2 2
1 ad  bc
dUcenH eXIjfa 2ab  ad  bc . 2cd
eyIg)an a  b a  b   a  b
3 3 2 2

 a 5  b 5  a  b1  1
77> sRmYlkenSam
 a 5  b5  0  a 5  b5 1
 a  a  a 2  b 2 
dUcenH eXIjfa a 5
 
 b 5  a 3  b 3 a 2  b 2  a  b   . A   
b
 2  
b
 2  
 b a  b a  2011 ab 
 a  2 2 1
75> KNnakenSam ³ A  x
2
2010
 y 2011  x 2012 b 2 a b 
     2  
x 2  2 y  1  0  b a   2011ab 
 
 2
eyIman  y  2z 1  0 ¬bUkGgÁnigGgÁ¦  a  2
a b  b
2
 a 2  b 2  1
 z 2  2x 1  0     2     2  
  b  b a  a   2011ab 
 
x 2  2 y 1 y 2  2z 1 z 2  2x 1  0

 691 
1
a
   2
ab b  a 2  b 2 
  2   kñúgRtIekaNEkgFMman sin   ah  h  a sin a
b ba a  2011ab 
a b  2011ab 
RkLaépÞénRtIekaN=¬)at x km<s;¦¼@
   2   2  2 2 
b a  a  b  naM[ S  12 ah  12 a  a sin   12 a sin  2

 a b  2011ab 
    2 2 
 b a  a  b 
 a 2  b 2  2011ab 
dUcenH épÞRtIekaNsm)atenHKW S  12 a 2
sin  .
   2 2 
 2011
 ab  a  b 
dUcenH eRkayBIsRmYl A  2011 . 80> eRbóbeFobcMnYn
k> 6  5 nig 21
78> KNna P  xy  yz  zx ]bmafa ³ 6  5  21
eyIgman ax  by  cz  m 6  2 30  5  21 ¬elIkCakaer¦
Taj)an x  am , y  bm , z  cm 2 30  10
naM[ P  xy  yz  zx ¬elIkCakaermþgeTot¦
4  30  100
 ambm  bmcm   cm am 120  100 BitR)akdEmn
 abm2  bcm 2  cam 2
 ab  bc  ca m 2
naM[kar]bmaKWRtwmRtUv mann½yfa 6  5  21
eyIgman a b  c 1
dUcenH eRkayBIeRbóbeFobKW 6  5  21 .
a 2  b 2  c 2  2ab  2bc  2ca  1
a 2  b 2  c 2  2ab  bc  ca   1
EtsmµtikmµbEnßmKW a 2  b2  c 2  1 x> 1  2011
11  72 nig 2 2011
3 2
naM[ 1  2ab  bc  ca  1 Binitü ³ 1  2011 11  72
2ab  bc  ca   0
 1  2011 9  2  4  18
ab  bc  ca  0
2 2
 1  2011 9  2 9  2  2
eyIg)an P  ab  bc  ca m 2
 0  m2  0 
 1  2011 3  2  2

eday 1 
2
2011
3  2   0
dUcenH eXIjfa P  xy  yz  zx  0 . 
1  22011 3  2  2011 3  2   2
0

79> KNnaRkLaépÞRtIekaNenH CaGnuKmn_én  nig a 1 3  2   2 3  2


2011
2
2011

eyIgman RtIekaNsm)aT b¤Gacsresr 1  11  72  2 3  2 2011 2011


edayeFVIcMeNalEkgBImMuBMuEmn  a
dUcenH lT§pl 1  11  72  2 3  2 . 2011 2011

enaHeyIg)anrUbdUcxagsþaM h

 692 
81> rktémøKt;én x , y nig z 3 1 1 1
a  3 b  3 c       3 ax 2  by 2  cz 2
x y z
eyIgman smIkar
 3 ax 2  by 2  cz 2
x  y  z  4  2 x 2  4 y 3 6 z 5
 x 2 x 2  y 4 y 3  z 6 z 5  4  0
dUcenH eXIjfa 3
ax 2  by 2  cz 2  3 a  3 b  3 c .
 x  2  2 x  2  1   y  3  4 y  3  4 
83> RsaybBa¢ak;fa H / G nig O CabIcMNucrt;Rtg;Kña
  z  5  6 z  5  9  0
A
   
x  2 1 
2
y 3 2  
2
z 5 3  02

D
 

x  2 1  0 2

eday  
 y  3  2   0 edIm,I[epÞógpÞat;smIkar
2

C H
G
O

 

z  5  3  0
2 

B A M C
xagelIKwmanEtmYykrNIKt;KW ³
 x  2  1  0
2
 x  2 1  0 ¬emIleTArUbdUcsµúKsµaj EtebIRKb;RKg)an KµanGVIBi)ak¡¡¡¦
 
 y  3  2  0   y  3  2  0 -]bmafa O Cap©itrgVg;carwkeRkARtIkekaN ABC nig H
 2 

 z  5  3  0  z  5  3  0

2
CaGrtUsg;énRtIekaN ¬cMNucRbsBVkm<s;TaMgbI¦
 x  2 1 x  2  1 x  3
ehIy BD CaGgÁt;p©iténrgVg;p©ti O
  
 y  3  2  y  3  4  y  7 eyIg)an BAˆ D  BCˆD  90 ¬mMucarwkknøHrgVg;¦o

 z  5  9  z  14
 z  5  3   Taj)an AB  AD nig BC   CD
dUcenH témøKt;EdlrkeXIj x , y , z   3 , 7 ,14  . ehIy AB  CH  nig BC   AH 
naM[ AD CH  nig CD AH
82> RsaybMPøWfa ax  by 3 2 2
 cz 2  3 a  3 b  3 c eyIg)an AHCD CaRbelLÚRkam enaH AH   CD 1
eyIgman 1x  1y  1z  1 -yk AM  CaemdüanénRtIekaN ABC naM[
ax 2 by 2 cz 2 M CacMNuckNþalénRCug BC 
naM[   1
ax 3 by 3 cz 3 eyIg)an OM  Ca)atmFümén BCD
eday ax 3  by 3  cz 3 eyIg)an naM[ OM  Rsbnwg CD nig OM   12 CD 2
ax 2  by 2  cz 2 ax 2  by 2  cz 2
¬!¦
3

ax 3
1 Taj)an 3
a
x tam 1 & 2 ³ OM   12 AH  b¤ MOAH
 2 3
ax 2  by 2  cz 2 ax  by 2  cz 2
2
-RtIekaN AHG nig MOC man ³
¬@¦
3

by 3
1 Taj)an 3
b
y
ax 2  by 2  cz 2 ax 2  by 2  cz 2
mMu AHˆ G  MOˆ G mMuqøas;kñúg ¬eRBaH OM  Rsb AH ¦
¬#¦
3

cz 3
1 Taj)an 3
c
z mMu AGˆ H  MGˆ O mMuTl;kMBlU
edayyk 1  2  3 eyIg)an naM[ RtIekaN AHG nig MOC CaRtIekaN dUcKña
tamkrNI m>m ¬mMuBIrb:unerogKña¦
 693 
vi)ak ³ ~ MOG
AHG

AH

AG
4 naM[ cMnYn 39 Eckdac;ngw 9 51

MO MG
 51 39  17  3  17 39  3 39  17 39  3 37  3 2  17 39  3 37  9
39

tam 3 & 4 ³ naM[ MG


AG
 2 or AG  2MG
naM[ cMnYn 51 Eckdac;ngw 9
39

enHbBa¢ak[eXIjfa G BiCaTIRbCMuTMgn;rbs; ABC eyIg)anplbUk 39  51 Ecknwg 9   51 39

dUcenH H / G nig O CabIcMNucrt;Rtg;KñaR)akdEmn . -tamry³  nig   ³ 39  51 Eckdac;nwg 9 5 51 39

84> bgðajfa P  0 eyIgman ³ dUcenH 3951  5139 Eckdac;nwg 45 .


P  a b  5a  9b  6ab  30a  45
2 2 2 2 2

 a 2 b 2  6ab 2  9b 2  5a 2  30a  45 86> KNna A ³ eyIgman


  
 b 2 a 2  6a  9  5 a 2  6a  9 
 a  6a  9 b  5
2 2 A  101 10001 100000001 ...1000...001
 a  3 b  5
2 2 man 2 n
 1 dgénelx 0

 100  110000  1100000000  1  ...  1000 ... 00  1


eday a  3 RKb; a CacMnYnBit
2
0
man 2 dgénelx 0
n

b  5  0 RKb; b CacMnYnBit
2

naM[plKuN a  3 b  5  0 Canic© 2 2  
 102  1 104  1 108  1  ... 102  1    n


10  1 10
1 2
  
 1 10 2  1 10 4  1 10 8  1  ...  10 2  1   n

dUcenH eRkayBIbgðajeXIjfa P  0 Canic© . 2


10
1
2
 1
10 4
 
 1 10 4  1 108  1  ... 10 2  1   n

85> bgðajfa 39  51 Eckdac;nwg 45 51 39


10
1
 10
8
 
 1 108  1  ... 10 2  1  n

 Binitü 39 elxcugKW 9 39 elxcugKW 1 1
1 2 2

 ...........................................................
39 elxcugKW 9 39 elxcugKW 1
3 4

>>>>>>>>>>>>>>>>>>>>>>>>>>>>>>>>>>>>>>>>>>>>>>>>>>>>>>>>>>>>>>>>>>>>>>>>>>>>>>>>>>>>>>>>>>>>>>>>>>>>>>>>>>>>>>>>>>>>>>>>>>>>>>>>>>>>>>>  
1
10  1 10
2
2n

 1 10 2  1
n

...9 ebI n CacMnYnKt;ess
manTRmg; 9 n

ebI n CacMnYnKt;KU 
1

n 1
10 2  1 
...1 99
naM[ cMnYn 39 CacMnYnEdlmanelx 9 enAxagcug .
51
 1000...00  1
1

 Binitü elxcugKW 1
511 51 elxcugKW 1 man 2 dgénelx 0
2 99
n 1

51 elxcugKW 1 3
51 elxcugKW 1 4
 999...99
1

>>>>>>>>>>>>>>>>>>>>>>>>>>>>>>>>>>>>>>>>>>>>>>>>>>>>>>>>> 99
man 2 dgénelx ( n 1

eXIjfa sV½yKuNén%! CacMnYnEdlmanelx! cugCanic©


 10101...01 ¬ man 2  1 xÞg;¦ n 1

naM[ cMnYn 51 CacMnYnEdlmanelx 1enAxagcug .


39

-eyIg)anplbUk 39  51 CacMnYnmanelx 0 cuug dUcenH A  10101...01 ¬ man 2  1xÞg;¦ .


51 39 n 1

naM[ plbUk 39  51 CacMnYnEckdac;nwg%  51 39

]TahrN_³ 10110001  1010101 Bit .


-mü:ageTot
 39 51  13  3  13 51  351  13 51  3 49  3 2  13 51  3 49  9
51

 694 
87> KNnatémø n ³ eyIgman -cMeBaH A  0 eyIg)an³
3 9 15 6n  3 300 C  A  0  0 ehIy B  A
2 2 4
 04  0
   ... 
2011 2011 2011 2011 2011  A  1 x 0  1  x x  1
3  9  15  ... 6n  3 300

naM[  
 B  1  y  0  1  y 

y 1
2011 2011 C  1  z 0  1  z z 1
  
3  9  15  ... 6n  3  300
31  3  5  ... 2n  1  3 100
-cMeBaH A 1 eyIg)an³
1  3  5  ... 2n  1  100 C  A  1  1 ehIy B  A
2 2 4
 14  1

n 2  10 2 A  1 x 1  1  x x  0
  
n  10 naM[  B  1  y  1  1  y   y  0
C  1  z 1  1  z z  0
  
GñkRtUvcaMfa 1  3  5  ...  2n  1  n 2
-ykKUcemøIyéntémø A, B, C CMnYsrk
x, y, z
dUcenH témøKNna)anKW n  10 . x  0

x  1

dUcenH KUcemøIyénRbB½n§KW y  0 b¤ y 1 .
z  0 z 1
88> edaHRsayRbB½n§smIkar ³  
1 1 1
x2  2x  y 1
 a  b  c   12 1
 2
k> y  2y  z eyIg)an³ x> 
1 1 1 7
2 edaHRsayedaybUkGgÁ
 z 2  2z  x    
  c b a 12
x2  2x   y x2  2x 1  1 y 1  1  1  5
 a c b 12
3
 2  
 y  2 y  z   y2  2 y 1  1 z
-edaybUkGgÁnwgGgÁénsmIkar 1 & 2 eyIg)an ³
 z 2  2z  x  z 2  2z 1  1 x
  1 1 1 7 2 6 2 12
     b 4
x  2x 1  1 y
2
 x  1  1  y 2
b b 12 12 b 12 6
 2 
 y  2 y  1  1  z   y  1  1  z
2
-edaybUkGgÁnwgGgÁénsmIkar
1 & 3 eyIg)an ³
 z 2  2z 1  1 x   z  12  1  x 1 1 1 5 2 4 2 12
        a 6
1  x 2  1  y  a a 12 12 a 12 4

1  y   1  z 
2 -edaybUkGgÁnwgGgÁénsmIkar
1 & 3 eyIg)an ³
 1  z 2  1  x  1 1 7 5 2 12 2 12
       c 2
c c 12 12 c 12 12
edaytag A  1  x , B  1 y , C  1 z
 A2  C 1 dUcenH RbB½n§smIkarmanKUcemøIy a, b, c  6,4,2 .
 2
eyIg)an³ C  B 2
B2  A 3
 89> edaHRsaysmIkar eyIgmansmIkar
-tam 1 : A  C 2 x  4 x  3 x  2 x 1
 
2007 2008 2009 2010
 4

-tam 2 : C  B 2
 A4  B x4
2007
1 
x 3
2008
1 
x2
2009
1 
x 1
2010
1  0

-tam 3 : B  A 2
 A8  A  x4

  x3
 1  
  x2
 1  
  x 1
 1  

 1  0
       
naM[ A  A 8
A8  A  0
2007 2008 2009 2010
 x  4  2007   x  3  2008   x  2  2009   x  1  2010 
    0
 2007   2008   2009   2010 

A A7  1  0  x  2011   x  2011   x  2011   x  2011 
    0
enaH A0 b¤ A 7
 1  0  A7  1  A  1  2007   2008   2009   2010 

 695 
x  2011  1

1

1

1 
0 8 2 1
dUcenH .
4
  1
2007 2008 2009 2010 
naM[ x  2011  0  x  2011 4
8 2 1  4
8 2 1 2

ehIy  1
 
1

1

1 
0
 2007 2008 2009 2010  91> sRmYlkenSam
2 x x 1   x 2 
dUcenH smIkarmanb¤sEtmYyKt;KW x  2011 . B      1 
x  1   x  x  1 
 

 x x 1
2 x x 1   x 2 
     1 
  x  x 1

8 2 1 
90> RsaybB¢aak;fa 
4
   
1 3
 x 1 x 1
8 2 1  8 2 1 2
 1   x 2 
4 4
2 xx
     1  
tag n  8 2 1  8 2 1 , n  0 enaH  
 x 1 x  x 1  x  1   x  x  1 
 

2 x  x x  x  1  x    
4 4

x 1  x 2


n2   4
8 2 1  4
8

2 1 
2
 x  1x  x  1 x  x 1
  2 x  x  x  x 1 x  x 1 x  2
 
  4 8 

2  1    4 8 
 
2  1 

 
x 1 x  x 1  x  x 1
x 1 x 1
 2  4 8  2  1  4 8  2  1   
    
x 1 x  x 1  x  x 1

 8 x 1 x  x 1 1
2
  2  1 
2
   ,0  x  1
 24 8  2
  
4
  x 1 x  x 1 x 1 x 1

 8 dUcenH eRkayBIsRmYl B  x 1 1 .
2
  2  1 
2
n 2  24 8  2 4
 
 24 8  2 8  2 1 
 24 8  2 2 2  2  1
92> RsaybB¢aak;[eXIjfa A  B  C  7 CakaerR)akd
 24 8  2 2 1
-tag x  111...11 naM[ 999...99  9x
man n tYénelx ! man n tYénelx !
 2 4 8  2  1 

Tajrk vij eyIg)an³
n

Taj)an ³ 10 n
1  9x  10 n  9 x  1 ¬sUmKit¦
n 2  2 4 8  2  1 
  -eyIg)an A  888 ...88  8  111 ...11  8x
n  2 4 8 

2  1 

eRBaH n  0 BMuEmn  eT man n tYénelx ! man n tYénelx !

 2 4
8 2 1 ehIy B  222...222  222...220  2
edaypÞwmtémørbs; n eyIg)an ³ man n  1 tYénelx @ man n tYénelx @
4
8 2 1  4
8 2 1  2  4
8 2 1  222...22 10  2  2 x 10  2  20x  2
8 2 1 man n tYénelx @
naM[
4

4
8 2 1  4
8 2 1 nig C  444 ...444  444 ..44  10 n  444 ..44
man 2n tYénelx $ man n tYénelx $ man n tYénelx $
4
8 2 1 1
 
2 4
8 2 1 2  4 x9 x  1  4 x  36 x 2  4 x  4 x  36 x 2  8 x

 696 
naM[ A B C 7  b  a  c  b 2  ab  bc 5

 8 x  20 x  2  36 x 2  8 x  7   c  a  b  c 2  ac  bc 6
 36 x 2  36 x  9 edayyk 5  6  7 : eyIg)an
 6 x   2  6 x  3  3
2 2
a 2  b 2  c 2  2ab  bc  ac   
 6 x  3  666...66  3  666...69
2 2 2

man n tYénelx ^ man n  1 tYénelx ^


-tamry³  &   eyIg)an ³
ab  bc  ac  a 2  b 2  c 2  2ab  bc  ac 
dUcenH A  B  C  7 CakaerR)akd . dUcenH ab  bc  ac  a 2
 b 2  c 2  2ab  bc  ac .
93> rkPaKryénkm<s;EdlRtUvfycuH
95> KNna E ³ eyIgman
h
RkLaépÞRtIekaN 1
S  bh
E
2011

2011

2011
b 2 b a c b a c
2 2
a b c
2 2 2 2 2 2 2

-tag x CaPaKryénkm<s;EdlRtUfycuH -tamsmµtikmµ a  b  c  0 eyIgTaj)an³


-)atRtIekaNfµIbnÞab;BIekIn 10% KW b  10%b  abc  0  b  c  a

-km<s;RtIekaNfµIeRkayBIfycuH x KW h  xh b  c   a   b  2bc  c  a


2 2 2 2 2

naM[RkLaépÞénRtIekaNfµKI W S   12  b  100


10 
b h  xh 
naM[ b  a  c  2bc 1 2 2 2


 abc  0  a  c  b
-tambRmab;RbFanRkLaépÞénRtIekaNTaMgBIesµIKña a  c   b  a  2ac  c  b
2 2 2 2 2

eyIg)an 12 bh  12 b  0.1bh  xh naM[ b  a  c  2ac 2 2 2 2

1  1.11  x   1  1.1  1.1x


 abc  0  a  b  c
1.1  1 0.1
x
1.1

1.1
 0.09091  9.09 % a  b 2
  c 
2
 a 2  2ab  b 2  c 2
naM[ a  b  c  2ab
2 2 2
3
dUcenH km<s;RtUvfycuH 1
11
or 9.09% . -tam 1 , 2& 3 eyIg)an
2011 2011 2011
E  2  2
94> Rsayfa ab  bc  ac  a  b  c 2 2 2
 2ab  bc  ac  b a c
2 2 2
b a c
2 2
a  b2  c2
-RKb; a , b nig c CacMnYnBiteyIg)an³ E
2011 2011 2011
 
 2bc 2ac  2ab
 a  b   0  a 2  b 2  2ab 1
2
2011 2011 2011
  
 b  c   0  b 2  c 2  2bc 2 
2
2bc 2ac 2ab
 a  c   0
2
 a 2  c 2  2ac 3  2011a  2011b  2011c

edayyk 1  2  3 : eyIg)an 2abc
 abc
2a 2  b 2  c 2   2ab  bc  ac  2011 0
 2abc 
a 2  b 2  c 2  ab  bc  ac 
-müa:geTot a , b nig c CargVas;RCugénRtIekaN eRBaH a  b  c  0 nig abc  0
eyIg)an ³ dUcenH eRkayBIKNnaeXIjfa E  0
 a  b  c  a 2  ab  ac 4

 697 
96> bgðajfa man    a    3a   a  3a  4a 2 2 2 2 2

 naM[ x    a   4a  a  2a  3a yk
2
     
2010  2012 2011 2  1 2011 4  1 2011 8  1 2011 16  1  2011 32  1

BinitüGgÁTI! ³ ¬edayeRbIrUbmnþ a  ba  b  a  b ¦ 1


  a   4a
2 2
2
x  a  2a   a minyk
2010  2012 2011  12011  12011  12011  1
2 4 8
1 16

 2011 12011 12011  12011  12011  12011  1


2 4-eyIg)anrgVas;RCugénRtIekaNKW 3a , 4a , 5a
8 16

 2011  12011  12011  12011  12011  1


2 2 4
rgVas;RCugEvgCageK KWtUcCag b¤esµI !0 mann½yfa
8 16

   
 20114  1 20114  1 20118  1 201116  1 
 2011  12011  12011  1
8 8 16 5a  10  a  2
 2011  12011  1
16 16
ehIy a CacMnYnKt;viCm¢ an enaH a  1 nig a  2
 2011  1
32
 cMeBaH a  1 enaHRtIekaNmanrgVas;RCug 3, 4 , 5

dUcenH eRkayBIkarbgðaj eXIjfa  cMeBaH a  2 enaHRtIekaNmanrgVas;RCug 6, 8 , 10

   
2010  2012 2011 2  1 2011 4  1 2011 8  1 2011 16  1  2011 32  1    dUcenH rgVas;RCugénRtIekaNEkgtUcCag b¤esµI!0
KW³ 3, 4, 5 nig 6, 8, 10 .
97> bMPøWfa 2AM  AB  AC
-yk N kNþal AC  A 99> KNna A  B CaGnuKmn_nwg C
naM[ AN  AC 1
N BinitülkçN³RCugrbs;RtIekaN
2 C -tamBItaK½r eXIjfa³
-bRmab; AM  Caemdüan B M 3  4  5 mann½yfavaCaRtIekaNEkg
2 2 2

naM[ M kNþal BC  -ehIyvaCaRtIekaNCarwkkñúgrgVg;enaHeyIgTajfa


Taj)an MN Ca)atmFüménRtIekaN ABC RbEvgRCugEvgCageKmanrgVas;esµI 5 CaGgát;[p©it
ehIy MN  AB 2
2 -eyIgKUsrUb)abdUcxageRkam³
-tamc,ab;vismPaBkñúgRtIekaN AMN rgVg;enHman kaM R  52
eK)an ³ AM  MN  AN 3 C

-edayyk 1 & 2 CMnYykñúg 3 A


4 5

eyIg)an AM  AB 2

AC
2
 2 AM  AB  AC
3
B

dUcenH eXIjfa 2AM  AB  AC R)akdEmn . -RkLaépÞRtIekaN S  1  3  4  6 ÉktaépÞ 


2
-RkLaépÞrgVg; S  R     52   254 ÉktaépÞ
2

98> rkRKb;rgVas;RCugénRtIekaNEkg  
2

-eday a CacMnYnKt;viC¢man -RkLaépÞEpñk C KW C  R2  S2  258 ÉktaépÞ


2

enaHeyIg)anrgVas;RCug x  x  a  x  2a
-edayvaCaRtIekaNEkg tamBItaK½r naM[ A  B  S  S  C  

25  25  25


  6   6 C 6
eyIg)an x  x  a   x  2a 
2 2
4 
2
8  8
x  x  2ax  a  x  4ax  4a
2 2 2
dUcenH A  B  C  6 .
2 2

x 2  2ax  3a 2  0
 698 
100 KNnaRbEvg)atmFümenAcenøaHGgát;RTUg x 4S   1  1  2  2  3  ... 1000  1000 
1 3 3 5 5 1999 2001 
-tag P nig Q CacMNuc A 6 cm B 1 2  2 3  999 1000  1000
RbsBVrvagGgát;RTUg nig M P x Q N 4S  1          ...  
3 3  5 5
 
 1999 1999  2001
)atmFüm eyIg)an ³ D 10 cm C
man 999 tYénKUplbUk
- MP Ca)atmFümén DAB eRBaH MP kat;tamcMNuc 4S  1  1  1  1  ... 1  1000
cMNuckNþal AD ehIyRsbnwg AB ¬)atmFümctu>Bñay¦ manelx! cMnYn 999 tY
2001

naM[ MP  AB 
6 cm
 3 cm 1000 2001000  1000 2002000
2 2 4S  1000   
- MQ Ca)atmFümén ACD eRBaH MQ kat;tamcMNuc 2001
4  500500 500500
2001 2001

cMNuckNþal AD ehIyRsbnwg CD ¬)atmFümctu>Bñay¦  S  4  2001  2001


naM[ MQ  CD 2

10 cm
2
 5 cm
dUcenH plbUkKNna)anKW S  500500 .
2001
-eyIg)an x  PQ  MQ  MP  5cm  3cm  2 cm
dUcenH RbEvgcenøaHGgát;RTUgKW x  2 cm . 102 RsaybBa¢ak;fa ³
101 KNnaplbUk S  11 3  32 5  53 7  ...  19991000 2001 6  6  6  ... 6  30  30  30  ... 30  9
2 2 2 2

-Binitü ragTUeTAéntYnImYy²rbs; S eyIgBinitüemIlGgÁTI! dUcbgðajxageRkam ³


n 2

4n 2
-cMeBaH 6  6  6  ... 6  6  6  6  ... 9
2n  12n  1 42n  12n  1
2n  n  2n  n
2 2 Et 6  6  6  ... 9  3 ¬beBa©jBIr:aDIkal;¦

42n  12n  1 naM[)an ³ 6  6  6  ... 6  3 (i)
  
n 2n  1  n 2n  1 

42n  12n  1 -cMeBaH 30  30  30  ...  30  30  30  30  ...  36

1  n2n  1 n2n  1  Et 30  30  30  ...  36  6 ¬beBa©jBIr:aDIkal;¦


  
4  2n  12n  1 2n  12n  1 
1 n n 
naM[)an ³ 30  30  30  ...  30  6 (ii)
  
4  2n  1 2n  1  -edaybUkGgÁnigGgÁ ³ (i)  (ii) eyIg)an ³
eyIg[témø n  1, 2, 3,...,1000 eyIg)an ³ 
 6  6  6  ...  6  3 (i)

ebI n  1³ 1 2

1  1

1  3 4 1 3 
1 
 30  30  30  ...  30  6 (ii)

ebI n  2 ³ 2 2
1 2 2
   
35 4  3 5 
6  6  6  ...  6  30  30  30  ...  30  9

ebI n  3 ³  53 7  14  15  17 


2
dUcenH eyIgGacRsaybBa¢ak;)anfa ³
>>>>>>>>>>>>>>>>>>>>> >>>>>>>>>>>>>>>>>>>>>>>>>>>>>>>>>>>>>>>>>>>>> 6  6  6  ...  6  30  30  30  ...  30  9
ebI n  1000³ 1000 2 1 1000 1000 
  
1999  2001 4 1999 2001  bBa¢ak; ³ karedaHRsayxagelIKWman n r:aDIkal;dUcKña.
1 1 1 2 2 3 1000 1000 
S        ... 
4 1 3 3 5 5
 
1999 2001  ¬GñkGacsegçbkaredaHRsay[xøICagenHk¾)an¦

 699 
4 S  1  t t  2   1
103 k> cUrKNnaplbUk
 t 2  2t  1
S  1 2  3  2  3  4  3  4  5  ...  nn  1n  2
 t  1
2

eyIgBinitütYnImYy²én S manragCa k k  1k  2 b¤Gacsresr)an³ 4S 1  n  3n1 2 2

Et k k  1k  2 dUcenH 4S 1 CakaerR)akd .


 k k  1k  2    4
1
4
104 RsaybBa¢ak;fa ³
x1  x2  x3  ...  x9 x1  x2  x3  ...  x12
 7
 k k  1k  2 4  k  k 
1
x3  x 6  x9 x4  x8  x12
4
 k k  1k  2 k  3  k  1
1 eyIgman ³ 0  x  x  x  ...  x
1 2 3 12
4
naM[ x  x  x  x  x  x  3x
1 2 3 3 3 3 3 1
 k  3k k  1k  2   k  1k k  1k  2 
1
4 x4  x5  x6  x6  x6  x6  3x6 2
 k k  1k  2 k  3  k  1k k  1k  2 
1
4
x7  x8  x9  x9  x9  x9  3x9 3
edayyk 1 + 2 + 3 eyIg)an ³
edaytémø k  1, 2 , 3, ... , n eyIg)an ³  x1  x 2  x3  3 x3

   x 4  x5  x 6  3 x 6
1  2  3  4 1  2  3  4  0 1  2  3
1
 x  x  x  3x
  7 8 9 9
2  3  4  1 2  3  4  5  1  2  3  4 x1  x 2  x3  ...  x9  3x3  x6  x9 
 4


b¤sresr)an ³ x  xx  xx  ...x  x
1 2 3 9
3 i 
 3  4  5  3  4  5  6  2  3  4  5
1
3 6 9

 4
...................................................
ehIy x  x  x  x  x  x  x
1 2 3 4 4 4 4  x4  4x4 4 
 x5  x6  x7  x8  x8  x8  x8  x8  4x8 5
nn  1n  2   nn  1n  2 n  3 
1
 4 x9  x10  x11  x12  x12  x12  x12  x12  4x12 6 
 n  1nn  1n  2
1  2  3  2  3  4  3  4  5  ... nn  1n  2  
edayyk 4 + 5 + 6 eyIg)an ³
 x1  x 2  x3  x 4  4 x 4

nn  1n  2 n  3
1
  x 5  x 6  x 7  x8  4 x8
4 x  x  x  x  4x
 9 10 11 12 12

dUcenH eRkayBIKNna S  14 nn  1n  2n  3 x1  x 2  x3  ...  x12  4x 4  x8  x12 


x1  x2  x3  ...  x12
b¤sresr)an ³ ii 
x 4  x8  x12
4
x> RsaybBa¢ak;fa 4S 1 CakaerR)akd CalT§plRKan;Etyk i   ii CakareRsc
eyIgman S  14 nn  1n  2n  3  x1  x 2  x3  ...  x9
3
 x3  x 6  x9

naM[ 4S  1  4  14 nn  1n  2n  3  1 
 x1  x 2  x3  ...  x12  4
 nn  3n  1n  2  1 
 x 4  x8  x12

 
 n 2  3n n 2  3n  2  1  dUcenH
ebItag t  n 2  3n eyIggayRsYlKuN enaH x1  x2  x3  ...  x9 x1  x2  x3  ...  x12
 7
x3  x 6  x9 x4  x8  x12

 700 
105 rktémøFMbMputénplKuN xy A  111...11222...22
eyIgman x  3 xy  y  60
2 2
 111...1110 2011  2111...11
GacEfmfy x  2xy  y  5xy  60
2 2

 111...11 10 2011  2 
x  y 2  5 xy  60 
1
9

999...99 10 2011  2 
60   x  y 
2
xy 
5 
 999...99 10 2011  1  3
1

xy  12 
x  y 2 9
 999...99999...99  3
1
5
eday x  y  2
0 
 x  y
2
0
9
 999...99  999...99  3
1 1
5
3 3
naM[ xy  12  0  12 CatémøFMbMput cMeBaH x  y  333...33333...33  1

dUcenH témøplKuNFMbMputénplKuN xy  12 . Edlman elx#cMnYn @0!! tY


106 rkplbUkrgVas;mMu A, B , C , D , E nig F dUcenH
A  111 ... 11222 ... 22  333 ... 33 333 ... 33  1
EfmcMNuc M nig N Rtg;RbsBVdUcrUb ³
C
F EdlmancMnYntYénelx! /@nigelx# esµIKñaKW 2011tY .
A
M 2
1 1
2 N B 108 rktémøtUcbMputénkenSam H ³
D
E
ebI x , x Cab¤smIkar 2011 x  t  2011 x  2011  0
1 2
2

BinitüplbUkmMukñúgén ³ tamrUbmnþ plbUkplKuNb¤s eyIg)an ³


-RtIekaN ANF ³ A  N  F  180 1 ¬!¦ x x  
o
b t  2011
-RtIekaN BMC ³ B  M  C  180 ¬@¦
1 2
oa 2011
1
c  2011
-ctuekaN DMNE ³ D  M  N  E  360 ¬#¦2 2
xx  
a 2011
o  1 1 2

edaybUk (1)+(2)+(3) eyIg)anplbUkmMu ³


2

eyIgman ³ H  x  x   4 x 2 x 2 1
2 1 2 1
 
1 

 x1 x2 
A  B  C  D  E  F  M 1  M 2  N1  N 2  720 o

EtmMu M  M  180 , N  N  180


o o
H   x 2  x1 
2  x  x 2 x 2  x1 
 4 1  
2

x1 x 2 
1 2 1 2

naM[ A  B  C  D  E  F  180  180  720 o o o  2


2
2  1 
b¤ A  B  C  D  E  F  360 o
  x 2  x1   4 x 2  x1  
2

1
 2 x1 x 2


dUcenH plbUkmMu A  B  C  D  E  F  360 . o
  1 1  
2

  x 2  x1  1  4   
2

  2 x1 x 2  
107 RsaybBa¢ak;fa A CaplKuNénBIrcMnYnKt;tKña
  1 1  
2

eyIgman A  111...11222...22 man @0!!tYdUcKña 


 x  2 x1 x 2  x 1  4
2
2
2
1   
  2 x1 x 2  
¬BinitüedayykcitþTukdak;BImYycMNuceTAmYycMNuc¦
 1  
 
2
 1
  x1  x 2   4 x1 x 2 1  4   
2

eyIgGacbMEbk A CaTRmg;gaydUcxageRkam ³   2 x1 x 2  

 701 
edayCMnYstémøénplbUknig plKuN eyIg)an ³ eday xx  1 CaBhuFadWeRkTI@ enaH sMNl;CaBhuFa
 t  2011  2   1 1  
2 dWeRkTI! manragCa rx  ax  b .
H    
  4  1  1  4   
 2011     2  1   -]bmafa qx  CaplEckén px nwg xx  1
 t  2011  2
 

  4 10
eyIg)an ³ px  xx  1 qx   ax  b
 2011   -cMeBaH p0  1 nig p1  2 eyIg)anRbB½n§smIkar³
 t  2011   p0  1 00  1qx   a  0  b  1
2

 10   40  
 2011   p1  2  11  1qx   a 1  b  2
 b 1 b  1

eday  t 2011  t  2011
2 2
2011 
 0 naM[ 10
 2011 
  40  40 a  b  2 a  1
 
enaHsMNl;KW r x   ax  b  x  1
dUcenH témøtUcbMputén H  40 cMeBaH t  2011 .
dUcenH sMNl;énviFIEckKW rx  x  1 .
109 KNnatémøelxén A  3 5 3 5 3...
111 RsaybBa¢ak; 5 2011
 52012  52013 Eckdac;nwg 31
eyIgman ³ A  3 5 3 5 3... eyIgman 5  5 2011 2012
 52013
A 2  3 5 3 5 3... 
 5 2011 1  5  5 2 
 31 5 2011
A  9  5 3 5 3 5 3...
4

A  45  3 5 3 5 3...
4
dUcenH 52011  52012  52013 Eckdac;nwg 31 .
A 4  45 A
112 rkelxéntYcugeRkaybg¥s;éncMnYn 2 34

Gacsresr A  45 A  0  AA  45   0
4 3 BinitüsV½yKuNén @ dUcxageRkam ³
naM[)an  A  0 minykeRBaH A  0 2   2 2   2
1 5
2   2 9

 A  45  0  A  45 yk
2   4 2   4
2 6
2   4 10

2   8 >>>
3 3

2   8 2   8
3 7 11

dUcenH eRkayBIKNna A  3
45 . 2   6 2   6
4 8
2   6 12

eXIjfa elxcugsV½yKuNén @ manxYbesµI$vileTAmk


110 rksMNl;énviFIEckrvag p x  nig xx  1 ehIy 3  81
4

¬GñkRtUvcaMfa ³ sMNl;énviFIEck f x nig x  a  KW r  f a naM[ 2   2  2  2  2  2  2  6  2  2


81 80 80 420

Edl f x CaBhuFa nig r  f a CasMNl; .¦


bRmab; Ecknwg x )ansMNl; 1
p x  dUcenH cMnYn 2 manelx 2 enAxagcugeKbg¥s; .
34

mann½yfa p0  1
ehIy px Ecknwg x 1)ansMNl; 2 smÁal; ³ eKkMNt;sresr 2   8 mann½yfa
11

mann½yfa p1  2 elxxagcugén 2  KWelx 8 . 11

-dWeRkénsMNl;Eck RtUvtUcCagdWeRkéntYEck
 702 
113 rkelxéntYcugeRkaybg¥s;éncMnYn123456789 -ebItémø x GviCm¢ aneyIg)an 10  x  100
2011

elxxagcugéncMnYn 123456789 KWGaRs½yEt 2011

elIsV½yKuNelxxagcugéncMnYnenHb:ueNÑaH mann½yfa dUcenH témøéncMnnY BitbBa¢ak;)anKW ³


10  x  100 b¤ 10  x  100 .
elxxagcug123456789 =elxxagcug 9 2011 2011

-BinitüelxxagcugsV½yKuNénelx ( ³ 116 k>eRbóbeFobcMnYn 1 2000 nig 2001  2  2000  2 2

9   9 , 9   1 , 9   9 , 9   1 , ...
1 2 3
Binitü ³ 2001  2  2000
4
2

naM[ 9    1 ebI n KU
9 ebI n ess
n
 1  2000  2  2000
2


 1  2 1  2000  2000  2  2000
eyIg)an 9   9 eRBaH @0!! CacMnYness
2 2
2011

 1  2000 2

dUcenH elxxagcugbg¥s;én 123456789 KW 9 . 2011


dUcenH 1  2000   2001  2  2000  .
2 2

x> KNna A  1  2000   2000


2

114 RsaybBa¢ak;facMnYn A CacMnYnKt;viC¢man  
 2001 
2000
2001
2

eyIgman A  13  2  1 2  1
3 3 3
3
eyIgman ³ A  1  2000   2000 
 
2000 2
2

 2001  2001

1
  
3 eday 1  2000   2001  2  2000  eyIg)an ³
2 2

A3 3
2 1 3
2 1
3  2000  2000
2

A  20012  2  2000    
3
1
3

3


2  1  3 2  3  3 2  3  3 2  1
3 2


 2001  2001
2
 2000  2000
  2001   
3
1
3

3


2  1  2  3  3 2  3  3 2  1
2


 2001 
2000 2000
2001

 2001  
3 
3

2  1 1  3 2  3 2 

2

  2001
2001 2001

3 2 2

3 3
2  3 2  3 2 1 3 2  3 2 dUcenH eRkayBIKNnaeXIjfa A  2001 .
2 2 2
 3 2 1  1 117 RsaybBa¢ak;fa a  x  y 3 3 3

dUcenH A 1 CacMnYnKt;viC¢man . eyIgman ³ x  x y  y  x y  a


2 3 4 2 2 3 2 4

a x2  3 x4 y2  y2  3 x2 y4
115 bBa¢ak;témøéncMnYnBit x edayeRbIsBaØavismPaB
 3
x6  3 x4 y2  3
y6  3 x2 y4
-bRmab; ³ x CacMnYnBit
mann½yfa x GacCacMnYnviC¢man b¤GviC¢man  3
x4 x 3 2
 3 y2  y  3
y4 3 2
 3 x2 
-smµtikmµ ³ x CacMnYnEdlmanEpñkKt;manelxBIrxÞg;  3 x2 x
3 2
 y 3
y  y 2
 3 2 3 2
 3 x2 
mann½yfa 10  x  100 ¬TRmg;enHeFVI[ x CacMnYn  
3
x2  3 y   x  y 
2 3 2 3 2

EdlmanEpñkKt;manelxBIrxÞg;Canic©¦  y 
3
 3
x2  3 2

 703 
cMeBaH a   x  y 
3 2 3 2
3

dUcenH cMnYn A CakaerR)akdéncMnYnKt; .


eyIgGacTaj)anCa ³ a   x 2 3 2
 y
3 2

3

120 sRmYlkenSam E ³
b¤ a  x 3 2 3 2
 3 y2
eyIgman ³
m

tamrUbmnþ ³ a  a
n m n
E
1  1 1
    
2

 1
 
1 
2

eyIg)an ³ a  x  y
3
2
3
2
3
 p q  2
 p q  3 
p q  p  q 
1  pq 2  p q
      
dUcenH eyIgRsay)anfa a
2
3
x y
2
3
2
3
.  p q 
2
 pq   p q 


pq 
3

1 p  2 pq  q  2 pq  2
  
118 eRbóbeFobcMnYn 2001  2002 nig 2 2002  
p q  
2
pq 
  p q 
2
pq

eyIgdwgfa 2001  2002 


1
2

 
p  q  2 pq 

 2

EfmGgÁTaMgBIrénvismPaBnwg 2002 eyIg)an ³ 


p q
2

pq 


 p  q  2
pq

2001  2002  2002  2002 


1 p q
2



 
2 pq 

 2

smmUl 2001  2002  2 2002



p q
2
pq  

pq 
 p  q 
2
pq
1 2 2
  
    pq p q
2
p q
2

dUcenH eRbóbeFo)an 2001  2002  2 2002 . 1


pq pq


119 RsaybBa¢ak;fa A CakaerR)akdéncMnYnKt; pq

eyIgman A  111...111 444...44 1 dUcenH sRmYl)an E  pq1 .


2n énelx! n énelx$

A  999...999  999...99  1
1 4
9 9
121 sRmYlkenSam E ³
2n énelx( n énelx(
eyIgman ³ A  3  9  125
27
 3 3 3 9
125
27
A
1 2n
9
 4
10  1  10 n  1  1
9
   ¬RsedoglMhat;elx 10 TMB½r!¦
1 1 4 4
 10 2 n   10 n   1
tamrUbmnþ ³ a  b 3  a 3  3a 2b  3ab 2  b 3
 a 3  b 3  3aba  b 
9 9 9 9
1 4 4
 10 2 n  10 n 
9 9 9 eyIg)an ³
1

 10 2 n  4 10 n  4  
A3   3  9 
125  
  3  9 
 
125 

9  27   27 
2
1
 2 1
 
 2 10 n  2   10 n  1  3    
 33  3  9 
125 
 3  9 
125 
A
3 3   27  27 
2
 10 n  1   999...99 
2
2
   1    1  125 
 6  33 32   9   A
 3   3 
 27 
 333...33  1  333...34 2
2
125
 6  33  A
n énelx# n  1 énelx# 27

 704 
naM[)an ³ A3  6  5 A
A  5A  6  0
3 dUcenH Rsay)anfa 3 3
2 1  3
1 3 2 3 4
9

9

9
.
A3  A  6 A  6  0
 
A A 2  1  6 A  1  0 123 KNnatémøelxénkenSam S ³
 A  1A 2  A  6  0
eyIgman ³ S  6  6  6  6  ... , S 0
naM[ A 1  0  A  1 naM[ S  6  6  6  6  6  ...
2

ehIy A  A  6  0
2

S 2  6  6  6  6  6  ...
eRBaH a 11240  0 S2  6 S

dUcenH eRkayBIsRmYlEbbbec©keTs A 1 . S2 S 6  0
man   b 2  4ac   1  41 6   25  5 2
2

122 RsaybBa¢ak;fa 3 3 1 2
2 1  3  3  3
4

S 
  1  5
 2
9 9 9  1
21
tag A  2 1 3 3 eyIg)an    1  5
 S2  3
 21
A3  3 2  1
 3

2  1 1  3 2  3 22  cMeBaH S  2 minyk eRBaH S  0
1

1  3
2  3 22  dUcenH témøelxKNna)anKW S  3 .
3
2  13
124 KNnatémøén ab  bc ³
3
1
A3  

1 3 2  3 4 1 3 2  3 4 
eyIgman ba  bc  2011 Edl a  0 , b  0
ehIy B3
1 3 2 3 4
9

9

9 Taj)an ba  2011  b  2011a
1 2 4
B  3 3 3
9 9 9
ehIy bc  2011  c  2011b
1

 3 1 3 2  3 4  naM[ ab  bc  2011aa  b2011b  2011a ab b  2011
9

   dUcenH témøKNna)anKW ab  bc  2011 .


1
3 1 3 2 1 3 2  3 4

9 1 23

3
1
9 1 2
3
13  3 2 3
  125 edaHRsaysmIkar
x  1 x  5 x  7 x  11
3 eyIgman 1991    4
B 3 1987 1999 1981

9 1 3 2  x 1
1
x5
1
x7
1
x  11
1  0
27 3 1991 1987 1999 1981
B3  
 
x  1  1991 x  5  1987 x  7  1999 x  11  1981
9 1 3 2
3
1  3 1  3 2  3 1  3 2 2  3 2 3
2    0
1991 1987 1999 1981
3 1 x  1992 x  1992 x  1992 x  1992
     0
3  33 2  33 4 1  3 2  3 4 1991 1987 1999 1981
x  1992  1  1  1  1   0
eXIjfa A 3
 B3  A B  1991 1987 1999 1981 

 705 
naM[ x 1992  0  x  1992 90 A  100  1000  10000  ... 10 n1  10n

 9 A  10  100  1000  ... 10  10
n

dUcenH smIkarmanb¤s x  1992 . 81A  10 n1  9n  10


10 n1  9n  10
126 edaHRsaysmIkar b¤Gacsresr ³ A
81
eyIgmansmIkar x  6 x   2  3 x
2000 1998 1999

Taj)an x   2  3 x  6 x  0
n 1
 9n  10
2000 1999 1998
dUcenH plbUkKW A  10 .
x   2  3 x  6 x  0
2 1998
81

naM[ x  0  x  0
1998
129 edaHRsaysmIkar k> x  x x x

x   2  3 x  6  0
2
GñkRtUvcaM rUbmnþ ³  f x      f x   
u x v x

smIkarenHman S  2  3 , P  2  3  6
eyIg)an 2 nig 3 Cab¤énsmIkar naM[)an ff xx 10ux  vx  0

dUcenH smIkarmanb¤sbIKW x   0 , 2 , 3 . eyIgman ³ x x


 xx
127 edaHRsaysmIkar log log log x  2
x

2 3 4 Gacsresr x x
x 2

eyIgman log log log x  2 x  0



naM[)an
2 3 4

eyIg)an log log log x  2 log 2 x  1 x  x   0


2 3 4 2   2 

log 2 log 3 log 4 x  log 2 2 2 eyIg)an  x  1  0  x 1
log 3 log 4 x  4 x x x2
log 3 log 4 x  4 log 3 3  x 0  x   x
2 2 4
log 4 x  34 b¤ 4 x  x  4 x  x  0  x4  x   0
2 2

log 4 x  81log 4 4 naM[ x  0 , x  4


x  481
cMeBaH x  0 minyk eRBaH x  0
dUcenH smIkarmanb¤s x  481 .
¬rUbmnþelakarIt log a x  k  x  a k ¦ dUcenH smIkarmanb¤s x  1  x  4 .
128 KNnaplbUk A  111111 ...111...11 x> x   x  ¬lMnaMdUcsMNYr k>¦x x xx

eyIgman ³ A  111111 ...111...11 tYcugmanelx eyIg)an ³ x  x   x  x  xx xx x2 xx

! cMnYn n dg eyIgGacsresr ³ x  0
naM [ )an
 x  1x  x   0
9 A  9  99  999  ... 999...99 ¬tYcugman n dgelx(¦  2 x

9 A  10  1  100  1  1000  1  ... 10  1 eyIg)an  x  1  0  x  1


n

9 A  10  100  1000  ... 10n  n 


 x2  x x  0  x2  x x  x  2
edayKuN ¬  ¦nwg !0 eyIg)an ³
90 A  100  1000  10000  ...  10 n1  10 n   dUcenH smIkarmanb¤s x  1 b¤ x2 .
eyIgyk ¬   ¦-¬  ¦ eyIg)an ³

 706 
130 KNna a  b  c bRmab; ³ km<s; mFümsisSRbus 1.68m
a  b 2  2ac  29

km<s; mFümsisSRsI 1.60 m
eyIgman  b  c  2ab  18
2
km<s; mFümsisSenAkñúgfñak; 1.66 m
 c  a 2  2bc  25

edaybUkGgÁngi GgÁ eyIg)an ³ eyIg)ansmIkar ³ 1.68x 1.60 y  1.66  28
a 2  b 2  c 2  2ab  2bc  2ac  a  b  c  72
1.68 x  1.60 y  46 .48 1
a  b  c 2  a  b  c   72 sisSsrubkñúgfñak; ³ x  y  28
a  b  c 2  a  b  c   72  0 b¤ 1.68x  1.68y  28 1.68
tag t  a  b  c eyIg)an ³ 1.68 x  1.68 y  47 .04 2
smIkarfµIgaysresr niggayKit t 2
 t  72  0 edayyk (2)-(1) eyIg)an ³
man   1 288  289  17 2 1.68x  1.68 y  47.04

  1  17 1.68x  1.60 y  46.48 0.56
t  2  9 a  b  c  9 0.08 y  0.56  y  7
naM[   0.08
t   1  17  8 a  b  c  8
 2 dUcenH cMnYnsisSRsIenAkñgú fñak;KW 7 nak; .
dUcenH KNna)an a  b  c   9 , 8 . 133 KNnakenSam A
131 KNnaplbUk S  1 2  3  ... 2011 eyIgman ³
A  a  b  c    a  b  c   a  b  c   a  b  c 
eyIgman S  1 2  3  ... 2011 ¬!¦
2 2 2 2

b¤srsesr S  2011 2010  2009  ...1 ¬@¦ edayeRbIrUbmnþ a 2


 b 2  a  b a  b  eyIg)an ³
A  a  b  c    a  b  c a  b  c    a  b  c  
edayyk (1)+(2) eyIg)an ³  a  b  c   a  b  c a  b  c   a  b  c 
 S  1  2  3  ... 2011 A  a  b  c  a  b  c a  b  c  a  b  c 
  a  b  c  a  b  ca  b  c  a  b  c 
S  2011 2010  2009  ... 1
A  2a 2b  2c   2b  2c 2a 
2S  2012  2012  2012  ... 2012
A  2a2b  2c  2b  2c   2a  4b  8ab
Edlmanelx 2012 cMnYn
2011 dg
eyIg)an ³ dUcenH kenSamEdlKNna)anKW A  8ab .
2S  2012  2011 134 bgðajfa b 2011 
2011
2

2
2011
c a c  a b a b c
2 2 2 2 2 2 2
0
2012  2011
S
2
 1006  2011  2023066 eyIgman a  b  c  0 naM[ ³
 b  c  a  b  c    a 
2 2

dUcenH plbUkKNna)anKW S  2023066 .  b 2  2bc  c 2  a 2


 b 2  c 2  a 2  2bc 1
132 rkcMnYnsisSRsIenAkñúgfñak;
tag x CacMMnYnsisSRbus  c  a  b  c  a    b 
2 2

y CacMMnYnsisSRsI Edl x  0 , y  0  c 2  2ca  a 2  b 2


 c 2  a 2  b 2  2ca 2

 707 
 a  b  c  a  b    c 
2 2
136 cUrRsabMPWøfa 4
49  20 6  4 49  20 6  2 3
 a 2  2ab  b 2  c 2 Binitü ³
 a 2  b 2  c 2  2ab 3 4
49  20 6  4 49  20 6
tamry³ (1) , (2) & (3) eyIg)an ³
2011 2011 2011
 4 25  20 6  24  4 25  20 6  24
 2  2
b c a
2 2 2
c  a b
2 2
a  b2  c2  4 5 2  20 6  2 6  5 2
4 2
 
 20 6  2 6
2

2011 2011 2011


  
 2bc  2ca  2ab 
 4 5 2 6   5  2 6 
2
4
2

2011a  2011b  2011c


  5 2 6  52 6
 2abc
2011a  b  c  
2
3 2 6 2 
2 2
3 2 6  2
2

 2abc
   
2
0  2  2
  3 2   3 2
 
dUcenH 2
2011
2 2
2011 2011
 2 2 2  2 2 2 0
b c a c  a b a b c .  3 2 3 2 2 3

135 kMNt;témø a nig b dUecñH eyIgbMPøW)anfa 4


49  20 6  4 49  20 6  2 3 .
eyIgman ³ x  px  q
2 2

 x 4  p 2 x 2  q 2  2 px 3  2qx 2  2 pqx 137 KNna E ³


 x 4  2 px 3   p 2  2q x 2  2 pqx  q 2 2 2
E   3  5  3  5    3  5  3  5 
bRmab; ³ x 4

 2x 3  ax2  bx  1  x 2  px  q 
2
   

eyIg)an   3  5  3  5    3  5  3  5  


   
x 4  2 x 3  ax 2  bx  1  3  5  3  5    3  5  3  5 
 
 x 4  2 px 3  p 2  2q x 2  2 pqx  q 2    
2 p  2  p 1   3  5  3  5  3  5  3  5  
 p 2  2q  a  p 2  2q  a  
 
Taj)an ³  smmUl  3  5  3  5  3  5  3  5 
 2 pq  b  2 pq  b  
 q  1
2  q  1
   2 3  5  2 3  5 
 p  2q  a
2   
cMeBaH

 4 3  5 3  5 
 2 pq  b
-krNI p 1 & q 1  4 9  5  4 4  8
a  3
naM[ 121211ba
2
  dUcenH KNna)an E  8 .
 b  2
-krNI p  1 & q  1
138 kMNt;témø m nig n
naM[ 1 212 11 ba a  1
2


 
b  2
eyIgman x 1 3  x m 4  x  3n x  4
 a  3 , b  2
RbPaKmann½ykalNa x  3 , x  4
dUcenH témøkMNt;)anKW  . tRmUvPaKEbg rYclubPaKEbgecal eyIg)an ³
a  1 , b  2

 708 
x  4  m  x  3  n eyIg)andwgehIyfa ebI a , b , c enaH
x  4  mx  3m  n
x  mx  n  3m  4 a2  0 , b2  0 & c2  0

x
n  3m  4 -manEtmYykrNIKt;EdleFVI[ a  b  c 2 2 2
0
1 m
KWmann½yfa a  0 , b  0 & c  0
2 2 2

edIm,I[RbPaK x mantémøeRcInrab;minGs; luHRtaEt naM[ a  0 , b  0 & c  0


PaKykesµIsUnü nigPaKEbgesµIsUnü dUcKña eyIg)an ³ D  a  1  b  1  c  1 2010 2011 2012

-eyIg)an ³  0  1
2010
 0  1
2011
 0  1
2012

n  3m  4  0 n  3m  4 n  1
     1   1   1  111  1
2010 2011 2012

1  m  0 m  1 m  1
dUcenH KNna)an D 1 .
dUcenH témøkMNt;)anKW m  n  1 .
141 KNna S ³
139 kMNt;témø a eyIgman ³  S  1  3  2  4  3  5  ...  2010  2012 
  12  2 2  32  ...  20102 
1
eyIgmanRbB½n§smIkar ³ 2xxyya3 2 Binitü

1 3  2  4  3  5  ... 2010  2012
edaybUkGgÁnigGgÁ ³  2  12  1  3  13  1  4  14  1  ... 2011 12011 1
 x y a      
 2 2  1  3 2  1  4 2  1  ... 20112  1  
  2  3  4  ... 2011  2010
2 x  y  3
2 2 2 2

a3
3x  a  3  x 
3
naM[
S  2 2  3 2  4 2  ...  20112  2010 
tamsmIkar 1 : y  a  x  a  a  3  2a  3  12  2 2  3 2  ...  20102 
3 3
 2 2  32  4 2  ...  20112  2010  12  2 2  32  ...  20102
edIm,I[epÞógpÞat; x  y  20112  2010  12
 20112  2011
eyIg)an ³ a 3 3  2a3 3  20112011  1
 2011 2010  4042110
a  3  2a  3
a6 dUcenH KNna)anKW S  2011 2010  4042110 .
dUcenH témøkMNt;)anKW a6 .
142 bgðajfa A Eckdac;nwg 5 ³
140 KNna D  a  1  b  1  c  1
2010 2011 2012
eyIgman ³
smµtikmµ a  b  c  0 nig ab  bc  ca  0 A  92011  82011  7 2011  62011  52011  12011  22011  32011  42011
A  9 2011  4 2011   8 2011  3 2011   7 2011  2 2011   6 2011  12011   5 2011
naM[)an a  b  c   0
2

tamrUbmnþ ³
a 2  b 2  c 2  2ab  2ac  2bc  0

a n  b n  a  b  a n1b  a n2 b 2  ...  a 2b n2  ab n1 
a  b  c  2ab  ac  bc   0
2 2 2

a2  b2  c2  2  0  0
eyIgeXIjfa ³

 9 2011  4 2011  9  4 9 2010  4  9 2009  4 2  ... 9  4 2010 
a2  b2  c2  0
 5k1 , k1  9  2010
49 2009
 4  ... 9  4
2 2010

 709 
 8 2011  3 2011  8  38 2010  3  8 2009  3 2  ...  8  3 2010 
 5k 2 , k 2  8 2010  3  8 2009  3 2  ...  8  3 2010 
144 bgðajfa Pn 
1 sin x

2 n sin x
 7 2011  2 2011  7  27 2010  2  7 2009  2 2  ...  7  2 2010  2n
 5k 3 , k 3  7 2010  2  7 2009  2 2  ...  7  2 2010 
 6 2011  12011  6  16 2010  1  6 2009  12  ...  6  12010 
eyIgman x x x
Pn  cos cos 2 cos 3  ... cos n
2 2 2 2
x

 5k 4 , k 4  6 2010  1  6 2009  12  ...  6  12010 


5 2011
 5  5 2010
tamrUbmnþ ³ sin a  2 sin cos
a
2
a
2
 5k 5 , k 5  5 2010
eyIg[témø x x x
a  x , , , , ..., n1
x
eyIg)an ³ A  5k  5k 1 2  5k 3  5k 4  5k 5 2 4 8 2
 x x
 5k1  k 2  k3  k 4  k5  sin x  2 sin 2 cos 2

eXIjfa A CaBhuKuNén 5 mann½yfa A Eckdac;nwg 5 sin x  2 sin x cos x
 2 4 4
 x
dUcenH bgðaj)anfa A Eckdac;nwg 5 . 
 sin  2 sin cos
x x
 4 8 8
................................
143 k> KNna fog nigKNna gof 
sin x  2 sin x cos x
eyIgman f : x  f x  px  2  2 n 1

2n 2n

g : x  g x   4 x  3
x x x x x
sin x  2 n sin n cos cos cos  ... cos n
naM[ fog  f g x  2 2 4 8 2
 f 4 x  3
Pn
 p4 x  3  2 x
sin x  2 sin n Pn
n

 4 px  3 p  2 2
ehIy gof  g  f x  Taj)an 1 sin x
Pn  n 
2 sin x
 g  px  2 2n
 4 px  2  3
 4 px  8  3
dUcenH bgðaj)anfa Pn 
1 sin x

2 n sin x
.
 4 px  5 2n

dUcenH KNna)an fog  4 px  3 p  2 145 edaHRsayRbB½n§smIkarkñúg 


ehIy gof  4 px  5 . x  y  5440 1
eyIgman ³ PGCD
2 2

x, y   8 2

x> KNna p edIm,I[ fog  gof
-tam 2 ³ PGCD x, y   8 mann½yfa
eyIgman fog  4 px  3 p  2 nig gof  4 px  5
x  8a & y  8b Edl a , b CacMnyY bzmrvagKña
naM[ fog  gof
ehIy a  b eRBaH x  y  5440  0 2 2

enaH 4 px  3 p  2  4 px  5
3 p  5  2 -tam 1 ³ x  y  5440
2 2

p  1 x  y x  y   5440
8a  8b 8a  8b   5440
dUcenH témøKNna)anKW p  1 . 8 2 a  b a  b   5440

 710 
 y  3  43  y  46
a  ba  b  85   
 x  2 1  x3
eday a , b CacMnYnKt;FmµCati nigedaHRsaykñúg   y 3 1  y4
1 85 b¤   
eyIg)anplKuNktþaén 85  5 17 eday a  b  x  2  43  x  45

   a  b  5
dUcenH smIkarmanb¤s x  3, y  46 
enaHeyIg)an ³ a  b  85 i or a  b  17 ii
a b 1
b¤ x  35 , y  4 .
 
a  b  1

i  ³ a  b  85 147 bMPøWfacMnYn N  4n  3 2
 25 Eckdac;nwg 8
2a  86  a  43
eyIgman ³ N  4n  3  25 2

ehIy b  85  a  85  43  42  4n  3  5 2
2

a  b  5  4n  3  54n  3  5

a  b  17  4n  24n  8
ii  ³ 2a  22  a  11  22n  1 4n  2
ehIy b  17  a  17 11  6  82n  1n  2

-cMeBaH a  43 , b  42 eXIjfa N CaBhuKuNén 8 mann½yfa N Eckdac;nwg 8


naM[ xy  88ab  88  43  344 dUcenH eXIjfa N  4n  3  25 Eckdac;nwg 8 . 2

 42  336
-cMeBaH a  11 , b  6 148 bMPøWfacMnYn a , b , c nig d CatYén smamaRt
naM[ xy  88ab  88 11  88 smµtikmµ a  b c  d   ad  bc
2 2 2 2 2

 6  48
a 2 c 2  a 2 d 2  b 2 c 2  b 2 d 2  a 2 d 2  2adbc  b 2 c 2
dUcenH RbB½n§smIkarmanKUcemøIyBIrKW a 2 c 2  2adbc  b 2 d 2  0
x , y   344 , 336  b¤ x , y   88 , 48  . ac  bd 2  0
ac  bd  0
146 edaHRsaysmIkarkñúgsMNMucMnYKt;FmµCati ac  bd
a c
eyIgman ³ xy  3x  2 y  37 
b d
xy  3x  2 y  37
xy  3x  2 y  6  37  6 dUcenH eyIg)antYénsmmaRt a c

b d
.
x y  3  2 y  3  43
 y  3x  2  43 a2 x2
149 bgðajfa   1
edayedaHRsaykñúgsMNcMu MnYnKt;FmµCati a2  b2 x2  y 2

mann½yfa  y  3 nig x  2 CacMnYnKt;FmµCati smµtikmµ ³ ax  by  0


2 2

ehIyplKuNcMnYnFmµCatiesµI 43 manEtBIrkrNIKW naM[ x   bya  x  bay 2


2

431
eyIg)anktþanImYy²KW ³
2 2
43  
1 43
yktémø x  bay CMnYscUl  eyIg)an ³
2
2

 711 
a 2
b2 y 2 151 bgðajfa A CacMnYnKt;
 a2 1
a2  b2 b2 y 2
y 2
eyIgman ³
a2
b2 y 2 A  2011 5  3  29  12 5
2
a 2
 2 2a 2 2 1  2011 5  3 20 2  12 5  32
a b
2 2
b y a y

a2 b2 y 2
a2
a2
 2011 5  3 2 5 3 
2

  1
a2  b2 a2 b2 y 2  a2 y 2  2011 5  32 5 3
2 2 2
a b y
 2 1
a b
2 2
b  a2 y2   2011 5 5 2  2 5  12
a2

b2
1  2011 5  5 1  2

a2  b2 a2  b2
a2  b2  2011 5  5  1  2011 1  2011
1
a2  b2
1 1 Bit dUcenH eXIjfa A  2011 CacMnYnKt; .
a2 x2
dUcenH eXIjfa 
a2  b2 x2  y 2
1 . 152 edaHRsaysmIkar
a2  b2  c2
eyIgman ³ x  4x  4  x  2x 1
2 2

150 KNnatémøén K
2011 elIkGgÁTaMgBIrCakaer enaHeyIg)an ³
smµtikmµ ³ 1a  b1  1c  0 x2  4x  4  x2  2x  1
2 x  3
bc  ac  ab
0 3
abc x
2
naM[ ab  bc  ac  0 Edl a  0 , b  0 , c  0 epÞógpÞat;
smµtikmµbEnßm ³ a  b  c  0  3  3
2
 3
2
 3
    4    4      2    1
naM[ a  b  c   0  2 2
  2  2   2
a 2  b 2  c 2  2ab  2bc  2ac  0 9
64 
9
 3 1
a 2  b 2  c 2  2ab  bc  ac   0
4 4
9 9
a2  b2  c2  2  0  0 2  2
4 4
a2  b2  c2  0 1 1

eRBaH ab  bc  ac  0 4 4
1 1

eyIg)an ³ K  a 2011
b c
2 2 2
0
 0 2 2
2011
dUenH smIkarmanb¤s x
3
.
dUcenH témøKNna)anKW K 0 . 2
¬ebImin[lkçxNÐ RtUvepÞógpÞat;[eK¦

 712 
153 KNnabIcMnYnKt;tKña 155 rktémøelxén 22
2012
 2 2011
2012
 2 2011
tag n CacnYnTI2 enaHcMnYnTI1KW n 1 nigTI3KW n  1 2  1  1
eyIgman 22  22  22
2012 2011 2011

Edl n CacMnYnKt;viC¢man 2012 2011 2011


2  1 3
-tambRmab;RbFaneyIgsresr)an ³ dUcenH témøén 22  22
2012 2011

2012 2011

1
3
.
n  1nn  1 
130
 n  1  n  n  1 
2


 2


21
156 KNnakenSam H ³
n  1nn  1  130 eyIgman H  a  baa  c  b  cbb  a   c  acc  b
2 2 2

2
 3n  21
 
 2 kenSamenHmanPaKEbgrYm a  ba  cb  c enaH
130  9n 2  a2 b2 c2
nn  1n  1    H  
21  4  a  b a  c  b  c b  a  c  a c  b 
 n n2 1   19514
n 2

a2

b2

c2
a  b a  c  b  c a  b  a  c b  c 
14n 2
 1  195n a 2 b  c   b 2 a  c   c 2 a  b 

14n 2  195n  14  0 a  b a  c b  c 
man   195 2
 4  14   14  
a 2 b  a 2 c  ab 2  b 2 c  ac 2  bc 2
 
a 2  ac  ab  bc b  c 
 38025  784  38809  197 2
a 2 b  a 2 c  ab 2  b 2 c  ac 2  bc 2
  195  197 
naM[  n1 
2 14

1
14
minyk a 2 b  a 2 c  abc  ac 2  ab 2  abc  b 2 c  bc 2
a 2 b  a 2 c  ab 2  b 2 c  ac 2  bc 2
  195  197  2 1
 n2   14 a b  a 2 c  ab 2  b 2 c  ac 2  bc 2
2 14
-cMeBaH n  14 ducenH eRkayBIKNna témø H  1 .
eyIg)an cMnYnTI1KW n 1  14 1  13 157 rkRkLaépÞFMbMputéncmáar
cMnYnTI3KW n 1  14 1  15 tag a CaRbEvgTTwg nig b CaRbEvgbeNþay
dUcenH cMnYnKt;TaMgbItKñaKW 13 , 14 , 15 . bRmab; ³ kUneQIman 40 edIm b

154 KNnatémøelxén xyz KmøaténkUneQI 2 m


smµtikmµ ³ x yz  7 i  nig xy
2 3 3 2
 79 ii 
naM[ cmáarmanbrimaRt P  2 40m
edayyk 1 2 eyIg)an ³ Et P  2a  b
 x 2 yz 3  7 3 eyIg)an 2a  b  2  40 m  a  b  40 m
 2
 xy  7
9
12
-cMeBaHctuekaNEkgEdlmanbrimaRtdUcKña KWctuekaN
x 3 y 3 z 3  712  xyz  7 7  74 EkgmanTTwgesµIbeNþaymanépÞFMbMput .
3 12 3

dUcenH témøén xyz  7 4 . enaH a  b  a  a  40  a  20m


 713 
naM[ RkLaépÞFMbMputKW ³ 160 KNna A , B , C , D nig E ³
3 2  4  3 8  3
S Max  ab  aa  a 2  20m  400 m2
11
2
A2   
4 4 4 4
dUcenH RkLaépÞFMbMputéncmáarKW S Max  400 m 2 . B  2
3 23 6
  
3
4 4 4 2
158 bgðajfacMnYn 3 BMuEmnCacMnYnsniTan C  2
3 2 3 23 6
    
3
4 1 4 1 4 4
¬cMnYnsniTanCacMnYnmanTRmg;CaRbPaK a ehIy a nig b Ca 2
b
3
D  2   2 
3  3  3
cMnYnbzmrvagKña ¬bzmrvagKñaKWtYEckrYmFMbMputén a nig b esµI!¦¦ 4 22 2 2
-]bmafa 3 CacMnYnsniTan 3 2 4 3 8 3 8  3 11
E  2      
naM[ 3 manTRmg;CaRbPaKsRmYlmin)an 4 4 4 4 4 4 4

-tag 3  ba  3  ba  a  3b
2

2
161 KNna A , B , C , D nig E ³
2 2

A  2  2  2  2  2  2  16 22 4

-eXIjfa a CaBhuKuNén 3 enaH a k¾CaBhuKuN


2

B  2   4  4  4  16 2 2 2

én 3 Edr mann½yfa a  3k , k CacMnYnKt; 22


C  2  2  16 4

-naM[ a  3b  3k   3b  b  3k
2 2 2 2 2 2

2 2

-eXIjfa b CaBhuKuNén 3 enaH b k¾CaBhuKuN D  2  2  16


4
2

E  22  24  8 2

én 3 Edr mann½yfa b  3 p , p CacMnYnKt;


-naM[ 3  ba  33kp  kp GacsRmYl)an mann½yfa 162 sresrCaTRmg;RbPaKén A , B , C ³
A  1.2 mann½yfa A  1.2222222222222...
vaBMu)aneKarBtamlkçxNÐcMnYnsniTan enaHvaBMuEmnCa
naM[ 10A  12.2222222222222...
cMnYnsniTaneT ¬GBa©wgmanEt CacMnYnGsniTan¦
sikSapldk
dUcenH cMnYn 3 BMuEmnCacMnYnsniTan . 
 10 A  12.2222222222...
 A  1.22222222222...
¬xagelIenHbkRsaytamsMeNIpÞúy tkáviTüa ¦ 9 A  11  A
11
9
159 edaHRsaysmIkar x  y  0 2 2

smµtikmµ³ x nig y CacMnYnBit enaH x  0 nig y  0


2 2 mann½yfa B  0.121212121212...
B  0.12

manEtmYykrNIKt;EdlepÞógpÞat;smIkar x  y  0 2 2 naM[ 100B 12.121212121212...


KW x  0 & y  0 sikSapldk
 100B  12.1212121212...
dUcenH smIkarmanb¤ x  0 & y  0 . 
 B  0.121212121212...
12 4
¬lMhat;xagelIRtUvkarcMeNHdwg BMuEmnbec©keTseT¦ 99B  12 B 
99

33

 714 
C  0.12mann½yfa C  0.1222222222222... 164 KNna AC
naM[ 10C  0.1222222222222... ABC nig ABD CaRtIekaNEkg
A
1 2
sikSapldk tamRTwsþIbTBItaK½r ³ B D
2 C
 10C  1.2222222222222...
 -cMeBaHRtIekaNEkg ABD
 C  0.12222222222222...
 C
1.1 11
 AD2  AB2  BD2
9C  1.1
9 90  BD2  AD2  AB2
D  1.20 mann½yfa  BD  AD2  AB2
D  1.2000000...  1.2 
12
10

6
5
.  BD  2 2  12  3
-cMeBaHRtIekaNEkg ABC
163 KNnaGgát;RTUg nigcm¶ayxøIbMputBIcMNuc AC 2  AB2  BC 2
RbsBVrvagGgát;RTUgeTARCugkaer AC  AB2  BC 2
a  AB2  BD  DC 
2
Ggát;RTUg
a   12   32 
2
 1 3  4 3  4
x cm¶ayxøIbMputBIcMNuc  8 4 3  2 2 3
a RbsBVeTARCugkaer
2 dUcenH RbEvg AC  2 2  3 .
-ebI d CaRbEvgGgát;RTUgkaerEdlmanRCug a ³
tamRTwsþIbTBItaK½r ³ d  a  a  2a 2 2 2 2
165 edaHRsaysmIkar
naM[ d  2a  d  a 2 eRBaH d  0
2 eyIgmansmIkar ³
 
 2  3  2  2  3  2  2  3  x  2011  0
dUcenH Ggát;RTUgkaer d a 2 ÉktaRbEvg . 



 2  3   2  2  3  2  2  3   x  2011  0
    
-cm¶ayxøIbMputBIcMNucRbsBVrvagGgát;RTUgeTARCugkaer  2 
 2  3  2 2   2  3   x  2011  0
KWCaRbEvgGgát;EdlP¢ab;BIcMNucRbsBVenaHeTAcMNuc 
   

kNþalénRCugkaerenaH .  2 
 2  3  2 2   2  3   x  2011  0
   

ebI x CaRbEvgxøIbMputenaH ¬tamRTwsþIbTBItaK½r¦  2  3  4  2  3  x  2011  0
 
eyIg)an ³ x   a2    d2 
2 2
2
 2  3  2  3  x  2011  0
     
  
2
d 
2
a
2
a 2 2
 2  3 2  3  x  2011
   
a
b¤ x      
2 2

 2 
  2  
 4  3 2  x  2011

a2 a2
 
a2

a  
 
2 4 4 2
x  2011

dUcenH cm¶ayxøIbMputKW x
a
ÉktaRbEvg . dUcenH b¤sénsmIkar x  2011 .
2

 715 
166 edaHRsaysmIkar 168 KNna x2
y2
eyIgmansmIkar ³ x2  2 y2 x2  2y2
x  2 x  4x  2 x  3  x  2 x  7
2 2 2 eK[ 306

294
x  2 x  4x  2 x  3  x  2 x  3  4
2 2 2
 2 2
 
294 x  2 y  306 x 2  2 y 2 
x  2 x  4  x  2 x  4  4
2 2
6  49x 2
 2y2   6  51x 2
 2y2 
x  2 x  3 x  2 x  4  1  4
2 2
49 x 2  92 y 2  51x 2  102 y 2
x  2 x  3  2
2 2 2
49 x 2  51x 2  92 y 2  102 y 2
x  2 x  3  2 2
 2 x 2  200 y 2

-cMeBaH x 2  2x  3  2 x 2  200

y2 2
x 2  2 x  1  0  x  1  0
2

x2
 x 1  0  100
y2
 x  1
x2
-cMeBaH x 2  2 x  3  2
dUcenH KNna)an .
y2
 100

x 2  2 x  1  4  x  1  2
2
169 KNna A ³
eday x  1  0 2
eK[ A  x  2  2x  2x  8  x  8
2 2

naM[ x  1  2 CasmIkarKµanb¤s
2
tamrUbmnþ a  b  a  2ab  b
2 2 2

dUcenH smIkarmanb¤s x  1 . eyIg)an A  x  2  x  8 2

 x  2   x  8
2

167 RsaybBa¢ak;fa aS  bS  cS  0 n 1 n n 1  x  2  x  8
2

eday x nig x Cab¤sénsmIkar ax  bx  c  0 2  10 2  100


cMeBaH x  2011 KµanTak;Tgnwgtémørbs; A
1 2
2012

enaHvaepÞógpÞat; ³ axax  bxbx  cc  00


2
1 1


2
2
2 dUcenH eyIgKNna)an A  100 .
 ax1  bx1  c  0
2
x1n1
eyIgGacKuNEfm  2
x2n1 170 RsaybMPøW;fa a  b  c  1
ax2  bx2  c  0
2 2 2

eyIg)an ax12 n1  bx11 n1  cx1n1  0


 2 n1 1 n 1 n 1
eK[ a  b  c  1
 ax2  bx2  cx2  0 naM[ a  b  c   1 2 2

ax n 1  bx1n  cx1n 1  0


  1n 1 n 1
a 2  b 2  c 2  2ab  2ac  2bc  1
 ax1  bx2  cx2  0
n

a 2  b 2  c 2  2ab  ac  bc  1 1


    
a x1n 1  x 2n 1  b x1n  x 2n  c x1n 1  x 2n 1  0 
eday S  x  x enaH S  x
n n n 1
x n 1 smµtikmµbEnßm 1a  b1  1c  0
n 1 2 n 1 1 2
bc  ac  ab
ehIy S  x  x
n 1
n 1
1
n 1
2 abc
0

dUcenH eyIg)an aS  bS  cS n 1 n n 1 0 .  bc  ac  ab  0 , abc  0

naM[smIkar 1 eTACa ³


 716 
a2  b2  c2  2  0  1 naM[ktþanImYy²esµIsUnü eyIg)an ³
a2  b2  c2  1
 x 2  5x  3  0
dUcenH eyIgeXIjfa a 2
 b2  c2  1 BitEmn . man   25 12  13
171 rkBIrcMnYnBit naM[ x   b2a    5 2 13
tag a nig b CaBIrcMnYnBitEdlRtUvrkenaH  x 2  5x  7  0

tambRmab; ³ a  b  13 1 man a  25


0
 28  3  0

ehIyplbUkcRmasKW 1a  b1  1340 2 naM[ smIkarKµanb¤s
tam 2 : 1a  b1  1340  aab b  13 40 dUcenH cMeBaH k  3 smIkarmanb¤s x   5 2 13 .
Taj)anplKuN ab  40 3
tam 1 & 3 eyIg)anRbB½n§smIkar³ aab b4013 x> kMNt;témø k edIm,I[smIkarmanb¤s
 BIsRmayxagelIBinitüRtg;kenSam ³
edaysÁal;plbUk nigplKuNeyIg)ansmIkardWeRkTI2 x 2
 5 x  5  12  k 
2

Edlman a nig b Cab¤s KWsmIkar ³ x 13x  40  0 2


x 2
 5x  5 
2
 k 1
man    13   4 1 40  169  160  9  3
2 3
eday x  5x  5  0
2 2

  13  3 10

a   5 naM[ k  1  0  k  1
naM[ 
2 1 2
  13  3 16
b 
 2 1

2
8 dUcenH témøkMNt;)anKW k  1 .
   13  3 10
b   5
b¤ 
2 1 2
  13  3 16
173 eRbóbeFobBIrcMnYn 200 nig 300 300 200

a   8
 2 1 2 Binitü ³ 200  200   8000000
300 3 100 100

a  5 a  8 ehIy 300  300   90000


200 2 100 100

dUcenH BIrcMnYnBitrk)an  b¤  .
b  8 b  5 tamry³lT§plbgðajfa 8000000   90000  100 100

172 k> edaHRsaysmIkar cMeBaH k  3 dUcenH Taj)anfa 200  300 . 300 200

eK[ x  1x  2x  3x  4  k 174 eRbóbeFobBIrcMnYn 31 nig 17 11 14

x  1x  4x  2x  3  3


x 2


 5x  4 x 2  5x  6  3
Binitü ³ 31  32  2  2 enaH 31  2
11 11 511 55 11 55

x  5x  5  1x  5x  5  1  3
2 2
ehIy 17  16  2  2 enaH 17  2
14 14 414 56 14 56

x  5x  5  1  3
2 2 2
eday 31  2  2  1711 55 56 14

x  5 x  5  2  0
2 2 2
dUcenH eRbóbeFob)an 31  17 . 11 14

x  5 x  5  2x  5x  5  2  0
2 2

x  5 x  3x  5 x  7  0
2 2

 717 
175 KNna 1  a  a  a    a2 3 2010  1  1  1  
A  1  2 1  2 1  2   ... 1  2 
1 
 2  3  4   n 
bRmab; ³ a Cab¤sénsmIkar x  1 2011
 2  1 2  1  3  1 3  1  4  1 4  1 
      
mann½yfa vaepÞógpÞat; a  1 2011
 2 2  3 3  4 4 
 n 1 n 1
eyIg)an ³ a 2011
1  0  ... 
 n
 
n 
a  1a 2011  a 2011  a 2011  ...  a  1  0  1 3  2 4  3 5   n 1 n 1
         ...   
b¤Gacsresr ³ a  11  a  a 2  a 3  ...  a 2010   0  2 2  3 3  4 4   n n 
1 1 1 1
smµtikmµ ³ a  1 enaH a 1 0 1
 1 1 1  ...
n 1 n 1

dUecñHmanEtktþa 1  a  a  a  ...  a   0
2 3 2 2010 n 2n
n 1
EdlnaM[plKuN a  11  a  a  a  ...  a   0 dUcenH eRkayBIKNna A  2n .
2 3 2010

dUcenH KNna)an 1  a  a  a   a  0 . 178 kMNt;témø m ³


2 3 2010

176 KNna S  3  3  3  ... 3


2 3 eyIgman ³ 23xx  4yym7
n


eyIgman ³ S  3  3  3  ...  3 1
2 3
edayedaHRsaytamedETmINg; eyIg)an ³
n

yk 1 3 ³ 3S  3  3  ...  3 2


2 3 n 1

D  ab  ab  2  12  14
edayyk 2  1 eyIg)an ³ D  cb  cb  7  4m x

3S  32  33  34  ... 3n1 D x 7  4m


 x 
D 14
 S  3  3  3  ... 3
2 3 n

3n1  3 D y  ac  ac  2m  21


2S  3n1  3  S
2 Dy 2m  21
n 1 x 
3
dUcenH KNna)an S
3
. D 14
2 edIm,I[vamanb¤s x0 nig y  0 luHRtaEt ³
 7  4m 
177 KNna A ³  14  0  7  4 m  0 m  
7
4
 2m  21   
BinitütYnImYy²manrag  0 2 m  21  0 m 
21
 14  2
  n 1
2
1
1  2  
b¤ Gacsresrsruby:agxøIKW 7
 m
21
 n  n2
4 2

n  1n  1
n2
n 1 n 1
dUcenH témøEdlkMNt;)anKW 
7
4
m
21
2
.
 
n n
eyIgman ³
 1  1  1   1 
A  1  2 1  2 1  2   ...  1  2 
 2  3  4   n 
edaybMEbkktþanImYy²eyIg)an ³
 718 
179 KNna E ³ 181 bgðajfa ³ 7  7  7  7 3 4

eyIgdwgehIyfa ³ 7  9  3 enaH 7  3 1


eyIgman
E  1  2  3  ...  2009  2010  2011  2010  2009  ...  3  2  1
7  8  2 enaH 7  2 2 
3 3 3

tag S  1 2  3  ... 2010 7  16  2 enaH 7  2 3


4 4 4

eyIg)an E  S  2011  S  2S  2011 edayyk ³ (1)+(2)+(3) : eyIg)an ³


S  1  2  3  ... 2010  7 3
 
 S  2010  2009  2008  ... 1  3 7  2
eday 2S  2011  2011  2011  ... 2011
4 7  2

7  7 4 7 7
3

man 2011 cMnYn 2010 dg


naM[ 2S  2011 2010 dUcenH eXIjfa 7 3 7 4 7 7 .
eyIg)an E  2S  2011
 2011 2010  2011
cMENkÉ karbgðajfa 4  4  4  4 3 4

 20112010  1 eyIgman ³ 4  2 1


 2011 2011  20112  2011 4  1  1 enaH 4  1 2
3 3 3

dUcenH eRkayBIKNna E  2011 . 4  1  1 enaH 4  1 3


4 4 4

edayyk ³ (1)+(2)+(3) : eyIg)an ³


180 edaHRsayRbB½n§smIkar ³  4 2

edaybUksmIkarTaMgbYnbBa©ÚlKña eyIg)an ³  3 4  1
4 4 1
a  b  c  3 1 
b  a  d  4 2 4 44 4 4
3


a  c  d  5 3
b  c  d  6 4 dUcenH eXIjfa 4  4  4  4 . 3 4

3a  3b  3c  3d  18  a  b  c  d  6 
182 kMNt;témø n ³
edayyksmIkar  dksmIkarnImYy²bnþbnÞab; tag g x CaplEckén f x  x  4 x  1 CamYy n 2

 a  b  c  d  6   a  b  c  d  6

1  a  b  c  3 
2  b  a  d  4 x  3 EdlmansMNl;esµI 46
d 3 c2
eyIg)an ³ x  4 x  1  x  3 g x  46
n 2

 a  b  c  d  6   a  b  c  d  6 ebIeyIg[témø x  3 enaHsmIkareTACa ³


3

 acd 5

4  b  c  d  6 3 n

 4  32  1  3  3  g 3  46
b 1 a0
3  35  0  46
n

3n  46  35
dUcenH RbB½n§smIkarmancemøIyKW 3n  81  3n  34 n4
a , b , c , d   0 , 1 , 2 , 3 . dUcenH témøkMNt;)anKW n4 .
 719 
183 edaHRsaysmIkar ³ 184 kMNt;RKb;cMnYnKt; n
eyIgmansmIkar ³ B  4  4  4 CakaerR)akdéncMnYnKt;
x
1
27 1016 n

x
2
2
x -krNI ³ 4 CatYcug eyIg)an ³
n

x B  4 27  41016  4 n
2
 
x
2  4 27 1  4 989  4 n27
1 1 x
 2  1
27 2
 
2
 21978  2 n27
smIkarmann½yluHRtaEt 1  x  0  x  1
2

 2  1
27 2
 
2
2
 2 1  21977  2 n27
Binitü RTg;RTayknÞúycugeRkayeK ³
x 
2 1 1 x  x  edIm,I[bMeBjrag a  b 2
 a 2  2ab  b 2 luHRtaEt
2  2 n27  21977
1 1 x 1 1 x
2  2 1 x  x  n  27  1977
 n  2004
1 1 x


1 2 1 x  x 1 -krNI ³ 4 CatYkNþal eyIg)an ³
n

1 1 x B  4 27  4 n  41016


12  2 1  1  x  1  x 2 
 4 27 1  4 n27  4989 
1 1 x
 2  1
27 2 2
2  2  
2 n 54 989 2


1 1 x 
2

 2  1
27 2 2
 2 1  2  2  
2 n 55 989 2

1 1 x
 1 1 x edIm,I[bMeBjrag a  b 2
 a 2  2ab  b 2 luHRtaEt
2 2 n 55  2 989
ebIeyIgyklT§plenHeTACMnYs vanwgkøayeTACaTRmg;  2n  55  989
dEdl²vij. ebICMnYsbnþbnÞab; cugeRkayeyIg)an ³ n
1044
x 2
1
1 1 x n  522
x  1 1 x
x 1  1 x
dUenH témøEdlkMNt;)anKW n  2004 , n  522 .
x  12  1  x 2
185 edaHRsaysmIkar ³
x 2  2x  1  1  x
eyIgman ³ x xx  1  x  x 1
4 2 2

x  3x  0
2
2
, x0
x
x x  3  0 1 1
x2 1 2  x 1
x  0 x x

x  3 1
x2  2  x 
1

-cMeBaH x  0 minyk eRBaHeBlepÞógpÞat; 0  1 1


x x
1
x2  2  2  2  x 
-cMeBaH x  3 yk eRBaHeBlepÞógpÞat; 1  1 x x
2
 1  1
x  2 x 
dUcenH smIkarmanb¤s x 3 .  x  x

 720 
ebItag t  x  1x eyIg)ansmIkardWeRkTI@ ³ eday 2 : xy  yz  zx  12
t2  2  t naM[ x  y  z  2 12  36
2 2 2

t2 t  2  0 x 2  y 2  z 2  12
man a  b  c  1  1  2  0 CakrNIBiess 2 x 2  2 y 2  2 z 2  24 
naM[ t  1 , t   ac   12  2
1 2
ehIy 2 : 2xy  2 yz  2zx  24  

-cMeBaH t  1 eyIg)an ³ eyIgyk (*)-(**) enaHeyIg)an ³


1
2 x 2  2 y 2  2 z 2  24
1 
x   1
x  2 xy  2 yz  2 zx  24
x  x 1  0
2 x 2  2 xy  y 2  y 2  2 yz  z 2  z 2  2 zx  x 2  0
1 1 3 x  y 2   y  z 2  z  x 2  0
x2  2  x     0
2 4 4
2
eday x  y 2  0 ,  y  z 2  0 , z  x 2  0
 1 3
x    0
 2 4
edIm,I[smIkarepÞótpÞat;manEtmYykrNIKt; KWktþa
eday  1 3
2

x   0 ,  0
nImYy²esµIsUnü mann½yfa ³
 2 4 x  y  0

y  z  0  x  y  z
2

enaH  x  12   34  0 naM[smIkarKµanb¤s z  x  0



-cMeBaH t  2 eyIg)an ³
2 tam 3 : 2  2  2  3 eday
x yz
x y z
1
x 2
x eyIg)an 2 2 2
  3
x x x
x 2  2x  1  0
222 6
x  12  0 3  x x  2
x 3
 x 1  0
x 1 dUcenH RbB½n§smIkarmanb¤s x  y  z  2 .
dUcenH eRkayBIedaHRsay smIkarmanb¤s x  1 .  1 1 1
 x  y  z 3 1

186 edaHRsayRbB½n§smIkar ³ x> 1
 
1

1
3 2
  xy yz zx
x  y  z  6 1  1 1
 xyz
3

k>  xy  yz  zx  12 2 
2 2 2
x  y  z 3
3 smIkarmann½yluHRtaEt ³ x  0 , y  0 , z  0

-tam 1 : 1x  1y  1z  3 smmUl xy xyz
yz  zx
3
smIkarmann½yluHRtaEt ³ x  0 , y  0 , z  0
tam 1 ³ x yz 6 naM[ xy  yz  zx  3 eRBaH 3 : xyz1  1
x  y  z  2  6 2 -tam 2 : xy1  yz1  zx1  3
x 2  y 2  z 2  2xy  yz  zx   36

 721 
x yz
xyz
3  x yz 3 1 k> 011
naM[  x  y  z 2  32 edayRbB½n§ 011 manelx 6xÞg;P¢ab;CamYyk,alRbB½n§
x 2  y 2  z 2  2 xy  yz  zx   9 ehIymanTRmg;Ca ³ 011 xxx xxx
x2  y2  z2  2 3  9 -CeRmIsenAtamxÞg;nImYy²man 10elxdUcKñaKW ³
x2  y2  z2  3 2
0 , 1 , 2 , 3 , 4 , 5 , 6 , 7 , 8 , 9
edayyk (2)-2×(1) eyIg)an ³
x 2  y 2  z 2  3
¬tameKalkarN_r)ab; ¦

2 x  2 y  2 z  6 naM[ cMnYnelxTaMgGs;KW ³
x  2 x  y  2 y  z 2  2 z  3
2 2

x 2
 
 2x  1  y 2  2 y  1  z 2  2z  1  0    10  10  10  10  10  10  10 6  1 000 000
x  12   y  12  z  12  0
eday x  1  0 ,  y  1  0 , z  1  0
2 2 2
dUcenH cMnYnlT§PaBénkarbegáItelxTUrs½BÞrbs;
edIm,I[smIkarepÞótpÞat;manEtmYykrNIKt; KWktþa RbB½n§ 011 KW 1 000 000 elx .
nImYy²esµIsUnü mann½yfa ³ x> 097
x  1  0 x  1
dUcKñanwgRbB½n§ 011 EdrRKan;EtRbB½n§ 097 man7xÞg;
 
y 1  0
z  1  0
 y  1
z  1
manTRmg; 097 xxx xxxx
 
naM[ cMnYnelxTaMGs;KW ³
dUcenH RbB½n§smIkarmanb¤s x  y  z  1 . 10  10  10  10  10  10  10  10 7  10 000 000

187 KNna 2  2 ³ a a
dUcenH cMnYnlT§PaBénkarbegáItelxTUrs½BÞrbs;
eyIgman 4  4  23 a a
RbB½n§ 097 KW 10 000 000 elx .
2 a 2
   23  2
 2  2 a
2

2  2   25
a a 2 189 rkcMnYnrebobénkarbegáItelx4xÞg;
2  2  25 a a
eyIgmanelxTaMgbYnKW 1 , 2 , 4 nig 9
a
2 2 5
eyIgnwgbegáIelx4xÞg;Edlman4elxxagelI edayKit
a

smÁal; ³ xN³bMBak;r:aDIkal;BMuEmn  25 eT eRBaH tamviFIdUcxageRkam ³


2  2  0 Canic© ¬CaGnuKmn_Gics,:ÚNg;Esül¦
a a

-xÞg;dMbUgman4CeRmIs xÞg;TIBIrman3CeRmIs
dUcenH eRkayBIKNna 2  2  5 . a axÞg;TIbImanBIrCeRmIs nigxÞg;cugeRkayman1CeRmIs
-tameKalkarN_r)ab; cMnYnrebob= 4 3 21  4! 12
188 rkcMnYnlT§PaBTaMgGs;énkarbegáItelxTUrs½BÞ
eyIg)andwgehIyfa enARbeTskm<úCaeyIgmanRbB½n§ TUrs½BÞCaeRcIn. dUcenH cMnYnrebobkñúgkarbegáItelx4xÞg;
RbB½n§xøHmanelx 6xÞg; xøHeTotman elx 7xÞg; ehIyP¢ab;CamYyk,al xusKñaman 12 rebob .
RbB½n§. etIcMnYnén karbegáItlxTUrs½BÞ y:agNaEdr ?
 722 
190 RsaybBa¢ak;fa ³ b  ab  c  pq  6 192 rkBIrcMnYnenaH ³
-smµtikmµ ³ a , b Cab¤sénsmIkar x  px  1  0 2
tag x , y CaBIrcMnYnEdlRtUvrkenaH
naM[ a abb  1 p 12 tambRmab;RbFan ³ plbUk=plKuN=plEck

-smµtikmµ ³ b , c Cab¤sénsmIkar x  qx  2  0 2 eyIg)an ³ x  y  xy  xy
naM[ b bcc  2q 34 -BinitüsmPaB ³ xy  xy  y  1y

-Taj)an a  bb  c   p q  pq naM[ y  1  y   1  y  1
2

ehIy ab  bc  1 2  3 -BinitüsmPaB ³ x  y  xy
b¤ 2ab  bc  6 cMeBaH y  1 enaH x 1  x Kµantémø x epÞógpÞat;
-Binitü pq  6  a  bb  c  6 cMeBaH y  1 enaH x  1   x  2x  1  x  12
 a  b b  c   2ab  bc 
 ab  ac  b 2  bc  2ba  2bc
dUcenH BIrcMnYnenaHKW 12 nig 1 .
 b  ba  bc  ac
2

 bb  a   cb  a 
 b  a b  c  193 KNnaplKuN P ³
eyIgman ³ P  1 x1 x 1 x ... 1 x 
2 4 2n

dUcenH eXIjfa b  a b  c   pq  6 . -Binitü ³ a  b  a  b a  b 


2 2

191 RsaybMPøWfaeyIg)an a1  b1  c1  2 naM[ a  b  aa  bb


2 2

2 2 2

smµtikmµ ³ a  b  c  abc naM[ a abc


bc
1 tamrUbmnþxagelIeyIg)an ³
P  1  x 1  x 1  x  ... 1  x 
2 4 2n
eyIgman 1a  b1  1c  2
1 x2 
n 1
 1  x 2  1  x 4  1  x 8 
        
tamrUbmnþ a  b  c  a  b  b  2ab  bc  ac
2 2 2 2 2 
 1  x  1  x  1  x 
4 
...
 1  x 2n



2 n 1
 1 1 1 1 x2
naM[     4
a b c

1 x
Edl x  1
2 2 2
1 1 1  1 1 1 
         2     4 1 x2
n 1

a b b  ab bc ac  dUcenH plKuNKNna)anKW P


1 x
.
1 1 1 cab
 2  2  2 4
 abc 
2
a b c
1 1 1 194 KNnaplbUk S ³
 2  2  2 1  4
a 2
b
1
c
1 1
eyIgman ³
 2  2 2 1 1 1 1
a 2
b c S    ...
1 2 1 2  3 1 2  3  4 1  2  3  ... 2011

dUcenH eXIjfa a1  b1  c1  2 .
2 2 2
Binitü ³ S k  1  2  3  ...  k

 723 
naM[ x 2  9 x  20  x  4x  5
S  1  2  3  ... k
 k enaH S   xx1 1  x  11x  2   x  21x  3 
S k  k  k  1  ... 2  1  
2S k  k  1  k  1  k  1  ... k  1  1 1 
   
 x  3x  4 x  4x  5 
man k dgén k  1 x2 x 1
S  
xx  1x  2 x  2x  3
Taj)an ³ 2S  k k  1  S  k k2 1
k k
x5 x3

eyIg)an ³ S  S1  S1  S1  ...  S 1 x  3x  4x  5
2 1 2
S  
2 3 4 2011

k k  1 xx  2 x  2x  3 x  3x  5


eday S k 
2 2x  6  x 2
 
S
1

1

1
 ...
1
xx  2x  3 x  3x  5
22  1 33  1 44  1 20112011  1
3 2
2 2 2 2  
2 2 2 2 xx  3 x  3x  5
S    ... 
22  1 33  1 44  1 20112011  1 3x  15  2 x 5
 
 1 1 1 1  xx  3x  5 xx  5
S  2    ...  
 23 3 4 45 2011 2012 
cMeBaH x  95 eyIg)an ³
ehIyBinitütYnImYy²én S manTRmg; ³ 5 5 5 1
S   
1

n 1 n

n 1

n 1
 
1 xx  5 95 95  5 5  19  100 1900
nn  1 nn  1 nn  1 nn  1 n n  1
naM[ dUcenH plbUkKNna)anKW S  1900
1
.
1 1 1 1 1 1 1 1 
S  2       ...   
2 3 3 4 4 5
1 1  1
2011 2012 
1005
196 bgðaj[eXIjBIsmPaB
 2    1  a

b

c

1

1

1
 2 2012  1006 1006 ab  a  1 bc  b  1 ca  c  1 1  a  ab 1  b  bc 1  c  ca

dUcenH plbUkKNna)anKW S  1006


1005
. -cMeBaH abc  1 Binitü ³
1 1 1
 
1  a  ab 1  b  bc 1  c  ca
195 KNnaplbUk S cMeBaH x  95 ³  1 c  1 a  1 b
      
eyIgman S  x 1 x  x  13x  2  x  15x  6 
2 2 2
 1  a  ab c   1  b  bc a   1  c  ca b 
c a b
1 1   
  2 c  ac  abc a  ab  abc b  bc  cab
x  7 x  12 x  9 x  20
2
c a b
  
eday x 2  x  xx  1 c  ac  1 a  ab  1 b  bc  1
a b c
  
x  3x  2  x  1x  2
2
ab  a  1 bc  b  1 ca  c  1

x 2  5 x  6  x  2x  3
dUcenH eyIg)ansmPaBR)akdEmn .
x  7 x  12  x  3x  4
2

 724 
197 rktémø x ³ 199 KNnaplbUk S ³
eyIgman xxxx  1  x  1 x2
¬rMlkw rUbmnþ log b  1 log b , 1  2  3  ... n  nn  1
an a
n 2
elxénxÞg;nImYy² CacMnYnKt;RtUvEtFMCag0 nigtUcCag10 eyIgman ³
eyIg)an 00  xx 110 10  1  x  10 S
1

1

1
 ... 
1
 log x log x log x a a2
log x a3 ak

elx x  2 CasV½yKuNRtUvmantémøFMCag0 
1

1
1

1
1
 ... 
1
1
log x
enaH x  2  0  x  2 2
log x a
3
log x a
k
log x a a

srublkçxNÐKW 3  x  9 
1

2
log x log x log x

a
3
 ... 
a
k
log x a a

mann½yfa x GacmantémøesµI 3 , 4 , 5, 6 , 7 , 8 , 9 
1
1  2  3  ... k 
log x
b:uEnþ xxxx  1 CaelxbYnxÞg; ehIymanEt 6 EtmYy 5
1
a

k k  1 k k  1
  
Kt;EdlmanelxbYnxÞg; naM[eyIgTaj)an ³ log x 2 a 2 log x a

 x 1  6 k k  1
  x7 dU c enH plbU k KNna)anKW S .
x  2  5 2 log x a

enaNeyIg)an ³ xxxx  1  x  1 x2

777 7  1  7  1
72 200 edaHRsayRbB½n§smIkar
7776  6 5 eyIgmanRbB½n§smIkar ³
7776  7776  x1  x2  x3    x2011  0 1
 2
 x1  x22  x32    x2011 2
 0 2
dUcenH témøEdlrk)anKW x  7 .  3
 x1  x2  x3    x2011  0 3
3 3 3

 .........................................................

198 sRmYlkenSam A ³  x12011  x22011  x32011    x2011

2011
 0 2011
¬rMlkw rUbmnþelakarIt log a b  log b c  log a c tamsmIkar 2 ³ x  x  x    x
2 2 2 2
0
1 2 3 2011
1
log a n b  log a b , log a a  1 >>> ¦ eday x  0 , x  0 , x  0 ,  , x
2 2 2 2
0
n 1 2 3 2011

eyIgman ³ naM[smIkar 2 epÞógpÞat;luHRtaEt ³


A  log 3 2  log 4 3  log 5 4  log 6 5  log 7 6  log 8 7 x12  x22  x32    x2011
2
0
 log 3 2  log 4 3  log 5 4  log 6 5  log 7 6  log 8 7 
 log 3 2  log 5 3  log 6 5  log 8 6
cMeBaHsmIkarepSgeTot k¾epÞógpÞat;EdrsRmab;témøenH
 log 3 2  log 5 3  log 6 5  log 8 6
 log 5 2  log 8 5
dUcenH RbB½n§smIkarmantémøb¤sesµI²KñaKW ³
1 1
 log 8 2  log 23 2  log 2 2  1 
1 x12  x22  x32    x2011
2
0 .
3 3 3
201 edaHRsayRbB½n§smIkartamedETmINg;
dUcenH eRkayBIsRmYleXIjfa A
1
.  
3 eyIgmansmIkar ³ 23 3x x 3 3 y y21  

 725 
edaHRsaytamedETmINg; eyIg)an ³ 203 sRmYlkenSam E & F ³
eyIgman E   a  ab b4 ab
D  ab  a b 2

  
 3  3  2  3  3 
 a  b   4 ab
   
 3  23 2

 1  2  3 
Dx  cb  cb a b

   
2 2
 2  3  1  3 
a  2 ab  b  4 ab
 2  3  3 a b
 3  3

2
a  2 ab  b

2
 a b 
2

D y  cb  cb a b a b
 3 1  2  3  2  a b

 3  43 
a b b a
ehIy F
 3  3 ab

naM[ x
D x  3  3 
  3 
ab a  b 
 a b

D 3 ab
D y  3  3
y
D

3
 3 dUcenH E a b , F a b .
KNna EFnig E F
dUcenH RbB½n§smIkarmanKUcemøIy E  F   a  b   a  b 
x  3  , y  3 .  a b a b 2 a
EF   a b   a b 
202 bgðajfa k> x  y  2 xy 2 2
2
 a  b  a b
2

eday x nig y CacMnYnBitenaHeyIg)an ³


x  y 2  0 dUcenH EF 2 a & EF  a b .
x 2  2 xy  y 2  0
x 2  y 2  2 xy 204 KNnatémøénkenSam A ³
dUcenH eXIjfa x  y  2 xy . 2 2 eyIgman ³
A  x n  100x n1  x n2  100x n3  x n4  100x n5
bgðajfa x> xy  xy  2  x n  100x n1   x n2  100x n3   x n4  100x n5 
 x n  100 x n 1  x n  2  100 x n 3  x n  4  100 x n 5
enAkñúgsMNYr k> eXIjfa x  y  2 xy 2 2
 x n 1  x  100   x n 3  x  100   x n 5  x  100 
smµtikmµ x nig y mansBaØadUcKña enaHmann½yfa 
  x  100  x n 1  x n 3  x n 5 
xy  0 Canic© eBlEcknwg xy vismIkar minbþÚrTisedA
cMeBaH x  100
naM[ A  100  100 100 n 1

 100 n 3  100 n 5  0
eyIg)an xxy  yxy  2xyxy  xy  xy  2
2 2

dUcenH témøKNna)anKW A  0 .
dUcenH eXIjfa x y
 2
y x
.
 726 
205 KNnatémøénkenSam A ³     180o

    180
o

eyIgman A  22  30  20 
3
27  3 8   0   

 22  30  20  27  3 8
3 dUcenH mMuTl;kMBUl    .
 3 22  30  20  27  2
208 rkcMnYnKt;viC¢man n ³
 3 22  30  20  5
eyIgman ³ x  x  6  n
2 2

 3 22  30  5 KuNnwg4 ³ 4x  4x  24  4n
2 2

 3 22  5 4 x 2

 4 x  1  23  4n 2
3 2n   2 x  12  23
2

dUcenH KNna)an A3 . 2n  2 x  12n  2 x  1  1 23


bRmab; ³ x nig n CacMnYnKt;
206 KNnatémøén a nig b ehIy n  x enaHeyIg)an ³
bRmab; a nig b CacMnYnKt;  2n  2 x  1  1

eday a  b  321  0 mann½yfa a CacMnYnFMCag
2 2 2n  2 x  1  23
4n  24  n6
b ehIysnñidæanfa b  a 1 eRBaH vaCacMnYnKt;tKña
-cMeBaH n6 eyIg)ansmIkar ³
eyIg)an ³ a  a  1  321
2 2
x 2  x  6  62
a 2  a  1  321
2
x 2  x  30  0

a 2  a 2  2a  1  321  eday 5   6  1 & 5 6  30
a  a  2a  1  321
2 2

322
tamRTwsþIbTEvüt enaH 5 &  6 Cab¤sKt;énsmIkar
a  161
2 dUcenH témøEdlkMNt;)anKW n  6 .
naM[ b  a 1  1611  160
a  b  2
a2  b2
dUcenH témøKNna)anKW a  161 , b  160 . 209 eRbóbeFobkenSam a  b nig 2 2
a  b2
207 mMuTl;kMBUlCaGVI? Binitü ³ aa bb  a abba b
2 2

2 2

mMuTl;kMBlU CamMumankMBUlrYmKña ehIyRCugénmMunImYy² a  b 



pÁúMKñabegáIt)anbnÞat;BIrkat;Kña nigrgVas;mMuTaMgBIrb:unKña . a  b 

a  b  a  b  a 2  b 2
 
 a  b  a  b  a  b 2
eyIgnwgRsayfa mMuTl;kMBUl   cMeBaH RKb;cMnYnBit a nig b Edl a  b enaH ³
 nig  CamMub:unKña a b a b
 a  b  a  b  2b  0 Bit
2 2 2 2
 2 2 2 2 2

eday     180 nig     180 eRBaHCamMurab


o oa  b a  b
2 2

sikSapldk ³
 727 
eday aabb  aa bb a2  b2 -cMeBaH m  1: dUcmanenAsMNYr k> bgðajrYc
2 2 2
 1
2
a  b2
enaHsmIkarmanb¤DubKW x  x  1 1 2

dUcenH eRbóbeFob)an aabb  aa bb . -cMeBaH m  53 smIkar  eTACa ³
2 2 2
2
2

 3  2  3   3
210 k> kMNt;témø m edIm,I[ x  1 Cab¤s    1 x     3  x  3      0
 5   5   5
eyIgman ³ m  1x  m  3x  3  m  0 
2
2 2 12
x  x 0
18

ebI x  1 Cab¤sénsmIkareyIg)an ³ 5 5 5

m  112  m  31  3  m  0
2 2
5

x  6x  9  0 
m 1 m  3  3  m  0 
x 2  6x  9  0 
1 m  0 x  32  0
m 1
naM[ smIkarmanb¤sdubKW x  x  3
cMeBaH m  1 smIkarka  eTACa ³ 1 2

1  1x 2  1  3x  3  1  0 dUcenH témøkMNt;)an m  1 , m  ac  53


1 2

2x 2  4x  2  0 nigb¤sDub x  x  1 b¤ x  x  3 .
1 2 1 2

x 2  2x  1  0
man a  b  c  1   2  1  0 ¬CakrNIBiess¦ 211 rkBIrcMnYn x nig y
eyIgsÁal; ³ xxy  12
y  25
2 2
naM[ x  1 , x  ac  11  1
1 2 eyIg)an

dUcenH témø m  1 nigb¤smYyeTotKW x  1 .  x 2  y 2  25

 2 xy  24
x> kMNt;témø m edIm,I[smIkarmanb¤sDub x  2 xy  y 2  1
2

x  y 2  1
smIkar m  1x  m  3x  3  m  0 
2
x  y  1
man   b  4ac
2
-cMeBaH x  y  1  x  y  1
  m  3  4m  13  m 
2

 m  6m  9  12m  4m  12  4m
2 2
naM[  y  1y  12  y  y  12  0 2

man y  1  1 2 414  1 2 49  28  4


2
 5m 2  2m  3 1
a  0
edIm,I[smIkarmanb¤sDub luHRtaEt
  1  12  4 14  1  49 6
  0 y2    3
 m 1  0  m  1 2 2 2
naM[  2  2
ebI y  4 : x  124  3 ehIy y  3: x  123  4
5m  2m  3  0 5m  2m  3  0 1 2

sRmab; ³ 5m  2m  3  0 man a  b  c  0
2
-cMeBaH x  y  1  x  y 1
naM[smIkarmanb¤s ³ m  1 , m  ac  53
1 2 naM[  y  1y  12  y  y  12  0 2

eyIgnwgrk b¤sDubcMeBaHtémø m nImYy²Edlrk)an man y    1  12  4 14  1 2 7  3


2

 728 
  1  12  4 14 1  7 x  1 , y  3 b¤ x  3 , y  1
y2   4
-cMeBaH S  3 & P  2 eyIg)an xxy y23
2 2
ebI y 3  3: x 
12
3
 4 ehIy y 4  4: x 
12
4
3 
x & y Cab¤sénsmIkar X  3 X  2  0
2

dUcenH BIrcMnYnEdlkMNt;)anKW ³ man a  b  c  0 enaHsmIkarmanb¤s ³


 x  3, y  4  x  4, y  3

x  3 , y  4
b¤ 
x  4 , y  3
. x  1 , y  2 b¤ x  2 , y  1
 

212 edaHRsayRbB½n§smIkar dUcenH RbB½n§smIkarmanKUcemøIybYnKU ³


x  1 , y  3 b¤ x  3 , y  1
1
eyIgman xx  yy  xyx y7 8
2 2


2 2
2 x  1 , y  2 b¤ x  2 , y  1 .
tag S  x  y nig P  xy
213 )atmFüménRtIekaNKWCaGVI?
tam 1 ³ x  2 xy  y  2 xy  x  y  8
2 2

x  y 2  2 xy  x  y   8 )atmFüménRtIekaN KWCaGgát;EdlKUsP¢ab;BIcMNcu
S 2  2P  S  8 3 kNþalénRCugBIrrbs;RtIekaN ehIyvaRsbnwgRCugTI#
tam 2 ³ x 2  y 2  xy  7 énRtIekaNenH nigmantémøesµIBak;kNþalRCugTI#.
A
x 2  2 xy  y 2  2 xy  xy  7
eyIgnwgbgðajfa MN  BC
x  y 2  xy  7 2 M N

S2  P  7 tamniymn½y B C

P S2 7 4 AM 
AB
/ AN 
AC
nigman MN Rsbnwg BC
2 2
yk 4 CMnYskñúg 3 eyIg)an ³ -RtIekaN ³ ABC & AMN man ³
S 2  2P  S  8

S2 2 S2 7  S 8  mMu AMN  ABC / mMu ANM  ACB ¬mMuRtUvKña¦
S 2  2S 2  14  S  8 naM[ AMN
ABC

MN AM
BC

AB
S2  S 6  0
 AM BC  AM BC
S2 S 6  0 naM[ MN  BCAB 
2 AM

2
  1  1  24  2
naM[ S 1   dUcenH )atmFüménRtIekaNekaNesµIBak;
2 3
S  2 : P   22  7  3 kNþalén)atTI# EdlRsbnwgva .
tam 4 ebI 
 S  3 : P  3  7  2
2

x  y  2 214 rkcMnYnKt; n EdlepÞógpÞat; 72  12n  54


-cMeBaH S  2 & P  3 eyIg)an 
 xy  3 eyIgman 72  12n  54 b¤Gacsresr 247  n  485
x & y Cab¤sénsmIkar X  2 X  3  0 2

man a  b  c  0 enaHsmIkarmanb¤s ³ smmUl 3  73  n  9  53


naM[ cMnYnKt;epÞógpÞat;KW n  4 , 5, 6 , 7 ,8, 9
 729 
dUcenH cMnYnKt;EdlepÞógpÞat;KW AO2  AB2 OB 2
 AO  AB2 OB 2
n  4 , 5, 6 , 7 ,8, 9 .
 252  20 2  625  400  225
215 rkPaKEdlenAsl;  15 m

-RtIekaNEkg AEB ³ -cMeBaHRtIekaNEkg COD man ³


24
1 E B CO 2  CD 2 OD 2
S AEB  12  24  144
2 12  CO  CD 2 OD 2
- RtIekaNEkg BDC ³ A
C 10 D
 252  24 2  625  576  49
1  7m
S BDC  10 12  60
2
eyIg)an AC  AO  CO  15 m  7 m  8 m
PaKenAsl;  S  S  144  60  204 PaK
AEB BDC
dUcenH RbEvgcugr)aFøak;cHu KW AC  8 m .
dUcenH PaKEdlenAsl;KW 204 PaK .
218 KNnargVas;RCugnImYy²énRtIekaN
216 Tajrk 1
x  2012
?
tagrUbRtIekaNenaHdUcrUbxageRkam ³
eyIgman ³ x  12011  2011 tambRmab;RbFan
B
a
c
smmUl x  2011  20111
b¤ x  2011
1
 2011 -brimaRt a  b  c  60 cm A b
C

naM[ x  12012  1 1 -RkLaépÞ 2 bc  120  bc  240cm


1 2

 2011  2012
2011 -tamBItaK½r a  b  c
2 2 2

a  b  c  60 1
1 1 2011
  
1  2011 2012 
1
2011
1
2011
srubmkeyIg)anRbB½n§smIkar  bc  240 2
 a 2  b 2  c 2 3

dUcenH Taj)an 1

2011
x  2012 2012
. tam 1 : a  b  60  c
b  c 2  60  a 2
217 rkRbEvgEdlcugr)aFøak;cuH b 2  c 2  2bc  3600  120a  a 2
A
C b 2  c 2  3600  120a  a 2  2bc 4
25 m
edayyk (2) nig (3) CMnYskñúg (4)
a 2  3600  120a  a 2  2  240
D 4 m B O
20 m
120a  3600  480
RbEvgcugr)aFøak;cuHKW AC  AO  CO 3120
a  26
tamRTwsþIbTBItaK½r 120
b  c  60  26  34
-cMeBaHRtIekaNEkg AOB man ³ naM[ 
 bc  240
tamEvüt b nig c Cab¤sénsmIkar ³
 730 
x 2  34 x  240  0 221 k> rkRbPaKtagcm¶aypøÚvEdlsuxRtUvedIr
man   b  ac   17   240  289  240  7
2 2 2
-bRmab; ³ suxeFVIdMeNIredayrfynþ)an 83  15 40
naM[ b   171   7  10 , c   171   7  24 suxeFVIdMeNIredaym:UtU)an 53  24 40
b¤Gac c   171   7  10 , b   171   7  24 naM[ kareFVIdMeNIrtamrfynþ nigm:UtKU W³ 40  40  40
15 24 39

dUcenH rgVas;RCugnImYy²rbs;RtIekaNKW enaH kareFVIdMeNIredayefµIeCIgKW 40 


39 1

40 40 40
10cm , 24cm , 26cm .
dUcenH RbPaKtagkareFVIdeM NIredayefµIeCIgKW 401 .
219 bgðajfaRtIekaNmanmMuTaMgbICaRtIekaNEkg
x> rkcm¶aypøÚvTaMgGs;ebIsuxedIr)an 2 km
bRmab;mMuTaMgbIKW  , 2 nig 3
suxedIr)an 2 km RtUvnwgRbPaK 401
edayeyIgdwgfaplbUkmMukgñú énRtIekaNesµI 180 o

eyIg)an   2  3  180 o mann½ y fa 1


40
 2 km cm¶aypøÚvTaMGs;KW
40
6  180 o    30 o  40  2 km  80 km
40
mMuTaMgbIénRtIekaNenHKW dUcenH cm¶aypøÚvTaMgGs;KW 80 km .
  30 o , 2  60 o , 3  90 o
RtIekaNenHmanmMumYymanrgVas; 90 vaCaRtIekaNEkg 222 k> sresr 45m CaPaKryén 1km
o

eday 1 km  1000 m
dUcenH RtIekaNmanmMuTaMgbICaRtIekaNEkg .
eyIg)an 45 1000
1

45

4.5
1000 100
 4.5%

220 rk f 9 dUcenH eyIgsresr)an 4.5% .


eyIgman f x f x  1  9 nig f 3  81 naM[
f 3  f 4  9  f 4 
9
 
9 1 x> sresr 1kg CaPaKryén 800 g
f 3 81 9 eday 1kg  1000g
f 4  f 5  9  f 5 
9 9
  81
f 4 1 / 9 eyIg)an 1000 800
1

1000 1 125
8
 
100 100
 125%

f 5  f 6  9 f 6 


9 9 1
  
f 5 81 9 dUcenH eyIgsresr)an 125% .
f 6  f 7   9 f 7  
9 9
   81
f 6 1 / 9 223 KNnaRbEvg AB ³
f 7   f 8  9 f 8 
9 9 1
  
f 7  81 9 bRmab; ³ MN  18cm
M A
f 8  f 9  9 f 9 
9 9  I
   81 
f 8 1 / 9 B

N
dUcenH rk)an f 9  81 .
eyIg)an ³ AB  AI 18BIcm
 731 
eday A nig B CacMNuckNþal erogKñaén MI nig NI  4  5 3  5 48  10 2  3  4  
naM[ AI  MI2 nig BI  NI2  4  5 3  5 28  10 3  4
enaH AB  MI2  NI2  4  5 3  5 5 2  10 3   3 2
4
MI  NI MN 18 cm
    9 cm
2 2 2  4 5 3 5 5 3  
2
4
dUcenH RbEvgKNna)anKW AB  9 cm .  4 5 3 5 5 3 4  
224 RsaybBa¢ak;faplbUkmMukñúgBIresµImMueRkAmYy  4  5 3  25  5 3  4

]bmafa eyIgmanRtIekaN nig  4  25  4


  45 4
mMu dUcrUbxagsþaM  
x
  1
eyIgnwgbgðajfa      naM[ A  1 20112012

-plbUkmMukñúgénRtIekaNesµI 180o
eday sV½yKuNén 2011CacMnYnmanelx 1 cugCanic©
enaH     x  180o
naM[ A  2011  ...1 CacMnYness
2012

-ehIymMu x    180 eRBaHvaCamMurab


o
eyIg)an A  1  120112012

sikSapldk ³ dUcenH témøKNna)anKW A  1 .


    x  180o

 x    180o 226 edaHRsayRbB½n§smIkar
     0      
rMlwkragsmIkardWeRkTI3 ³ tamRTwsþIbTEvüt smIkar manrag
dUcenH pkbUkmMukñúgBIrénRtIekaNesµImMueRkAmYy X 3  SX 2  RX  P  0 Edl a ,b,c Cab¤s ³
EdlminCab;ngw va . S  a  b  c

225 KNnatémøénkenSam A ³  R  ab  ac  bc . GñkGacepÞógpÞat;ragxagelI
 P  abc

eyIgman ³
2011 2012
edaykarBnøatkenSam x  ax  bx  c  0 .
 
A   4  5 3  5 48  10 7  4 3  4
 
2 x  2 y  2 z  7
  x
Binitü ³ 4  5 3  5 48  10 7  4 3  4 eyIgman y
2  2  2 
z 7
4

2  x  y  z  3
 4  5 3  5 48  10 2 2  4 3  3  4
2 x  2 y  2 z  7  2x  2 y  2z  7
 1 1 1 7  2 y 2 z  2 x 2 z  2 x 2 y 7

 4  5 3  5 48  10 2  3 
2
4  x y z 
4
 
2x2 y2z

4
 2 x  y  z2 32 
 2  2  2 x y z
2 2  8

 732 
 2x  2y  2z  7
 y z 7
 2x  2y  2z  7
 y z
cMeBaH a3 &b6 eyIg)an ³
2 2  2 2  2 2   8  2 2  2 2  2 2  14
x z x y x z x y
 23
2011 2011 2011
 4  2x2y2z  8  2a  6
       12011  1
 2x2y2z  8   b   6  6
edIm,IgayRsYl eyIgtag a  2 x
, b  2 y , c  2z dUcenH KNna)an  2ba   1 . 2011

abc  7  
eyIg)anRbB½n§fµI bc  ac  ab  14 tamRTwsþIbTEvüt 228 rkcMnYnxÞg;EpñkKt;éncMnYn A  2 2011

 abc  8

eyIgman ³ A  2 edaybMBak;elakarIteKal10
2011

a ,b,c Cab¤sénsmIkar X 3
 7 X 2  14 X  8  0
eyIg)an ³ log A  log 2 2011

¬manviFIedaHRsaysmIkardWeRkTI3tam Cadan b:uEnþBuMmankñúgkmµviFIsikSarbs;


log A  2011 log 2
RksYgGb;rM dUcenHeyIgenAEtedaHRsayedayrebobTajb¤sgay. sUmcgcaMfa
vaBMuEmnhYssmtßPaBrbs;eyIgeT eRBaHeKerobcMlMhat;)anl¥Nas; ¦ bRmab; ³ log 2  0.301
smIkarxagelImanb¤sgay ³ X  1 naM[ log A  2011 0.301
log A  605.311
eRkayBIEckBhuFa X  7 X 14X  8  0 nig X  1
3 2

eyIg)an ³ X  1X  6 X  8  0
2
edaycMnYn 605  605.311  606
naM[ 605  log A  606
 X  1 X  2 X  4  0
10605  A  10606
 X 1  0  X  a 1
naM[  
X  2  0  X  b  2
eday CacMnYnmanelx 606xÞg; ehIycMnYn
10605
X  4  0 X  c  4
  10 CacMnYnmanelx 607xÞg;
606

a  1 2 x  2 0 x  0
eyIg)an   y  Taj)an A CacMnYnEdlmanelx 606xÞg;
b  2   2  2   y  1
1

c  4 2 z  2 2 z  2
   dUcenH A  2 CacMnYnEdlmanEpñkKt; 606xÞg; .
2011

dUcenH RbB½n§smIkarmanb¤s x , y , z  0 ,1, 2 . 229 edaHRsaysmIkar 3 x  5  2 x  7  x 1


2 3
3 x 5 2 x 7
2011
227 KNna  2ba  eyI g man   x 1 , x  0
  2 3
eK[ a  b  15  216 33 x  5 22 x  7  6 x  1
 
6 6 6
33 x  5  22 x  7   6 x  1
a  b  9  36  6  6
2
 32  6 6  6 9 x  15  4 x  14  6 x  6
 3  6  2
5 x 1  6 x  6
 3 6 x 5
x  25
kenSaménGgÁTaMgBIrmanTRmg;dUcKña nig a nig b CaBIr
cMnYnKt;viC¢man naM[eyIgTaj)ankrNIEtmYYyKt;KW ³ cMeBaH x  25 eRkayBIepÞógpÞat; 4=4 Bit
a3 &b6
dUcenH smIkarmanb¤s x  25 .

 733 
230 eRbóbeFob ab nig PGCD a,b PPCM a,b 232 edaHRsayRbB½n§smIkar
eyIgman a  90 , b  280 edaybMEbkCaktþabzm ³ eyIgman  x  y  4  
x y 4
 xy  z  6 z  13  xy  z  6 z  13
2 2

a  90 2 b  280 2
45 3 140 2
edaysÁal;plbUk nigplKuN eyIgbegáIt)ansmIkar
15 3 70 2 X  4 X  z  6 z  13   0 Edlman x & y Cab¤s
2 2

5 5 35 5
1 7 7 man   b  ac 2

1

  2  z 2  6 z  13
2

naM[ a  90  2  3  5 nig b  280  2  5  7
2 3
 4  z 2  6 z  13
eyIg)an ³  ab  90  280  25200  z 2  6z  9

  z 2  6z  9 
 PGCD a , b   2  5  10
 z  3
2

 PPCM a , b   23  32  5  7  2520
eday  z  3  0 manEtmYykrNIKt; EdlnaM[
2

enaH PGCD a , b PPCM a , b  10  2520  25200 smIkarmanb¤sKW  z  3  0 enaH z  3 2

eXIjfa ab  PGCD a , b PPCM a , b  25200 -cMeBaH z  3 eyIg)ansmIkarfµI ³


dUcenH eyIgeRbobeFob)an 
X 2  4 X  32  6  3  13  0 
ab  PGCD a , b  PPCM a , b  . X 2  4X  4  0
 X  22  0
231 epÞógpÞat;ÉklkçN³PaB smIkarmanb¤sDub X  X  2 1 2
1  sin x  cos x   21  sin x 1  cos x 
2

Binitü ³ 1  sin x  cos x 2 eyIg)anb¤sénsmIkarKW ³  XX  xy  22 1

 2

tamrUbmnþ a  b  c  a  b  c  2ab  2bc  2ac dUcenH RbB½n§smIkarmancemøIy x  2 , y  2 , z  3 .


2 2 2 2

edayBnøattamrUbmnþxagelI eyIg)an ³
1  sin x  cos x  2 233 EbgEcknimitþsBaØaelakarIt
 1  sin x  cos x  2 sin x  2 cos x  2 sin x cos x
2 2 2
eyIgmannimitsþ BaØaelakarItdUcxageRkam ³
 12  1  2 sin x  2 cos x  2 sin x cos x Logx , log x , lg x , ln x , log 10 x , log e x
 21  sin x  cos x  sin x cos x 
 21  sin x   cos x  sin x cos x 
tamkareRbIR)as; rbs;GñkKNitviTüanimitþsBaØa ³
 21  sin x   cos x1  sin x  - elakarItTsSPaK b¤ elakarIteKal 10rYmman ³
 21  sin x 1  cos x 
log x , lg x , log 10 x
eRBaH edaysar sin x  cos x  1
2 2

naM[ 1  sin x  cos x   21  sin x 1  cos x 


2 - elakarItenEB b¤ elakarIteKal e rYmman ³
Logx , ln x , log x .
dUcenH 1  sin x  cos x  .
e
 21  sin x 1  cos x 
2

¬lMhat;xagelI RKan;Etcg;bgðajBInimitþsBaØaelakarIt¦
 734 
3a  b b  c a  c   0
234 rkRkLaépÞrgVg;carwkkñúgRtIekaNenH
a  b b  c a  c   0
-tamrUbmnþehrug épÞénRtIekaNKW a  b  0

S  p p  a p  b p  c 7

8 Taj)an b  c  0
a  c  0
Edlman a , b , c CargVas;RCug 11
r 
abc
ehIy p  2 CaknøHbrimaRt -cMeBaH a  b  0 tambRmab; a  b  c  1 enaH c  1
-smµtikmµ ³ RtIekaNmanrgVas;RCug 7 , 8 nig 11 ehIy a  b  c  1enaH a  b  0 Taj)an
2 2 2 2 2

a  0 , b  0 eyIgKNnatémøénkenSam ³
enaH p  7  82 11  13
P  a 2010  b 2011  c 2012  0 2010  0 2011  12012  1
eyIg)an ³ S  1313  713  813 11

-cMeBaH b  c  0 tambRmab; a  b  c  1 enaH a  1
 13  6  5  2
 2 13 15
ehIy a  b  c  1enaH b  c  0 Taj)an 2 2 2 2 2

-épÞRtIekaNFM )anBIplbUképÞRtIekaNtUc²bIEdlman b  0 , c  0 eyIgKNnatémøénkenSam ³


rgVas;RCugnImYy²Ca)at nigkaMrgVg; r énrgVg;carwkkñúg Pa b c 1  0  0 1
2010 2011 2012 2010 2011 2012

RtIekaN Cakm<s; eyIg)an ³ -cMeBaH a  c  0 tambRmab; a  b  c  1 enaH b  1


1  1  1
S    7  r     8  r     11  r 

 ehIy a  b  c  1enaH a  c  0 Taj)an 2 2 2 2 2

2  2  2 
a  0 , c  0 eyIgKNnatémøénkenSam ³
7 r 8r 11r
    13 r
2 2 2 P  a 2010  b 2011  c 2012  02010  12011  02012  1
2 13 15
naM[ 13r  2 13 15  r
13
srumk eTaHCakrNINak¾eday P 1 Canic©
-épÞrgVg;carwkkñúgRtIekaNKW S   r 2 dUcenH témøKNna)anKW P 1 .
2
 2 13 15 
S     
 236 bgðajfa a  b  c  d 1  a  b  c  d
2 2 2 2

 13 
 
4 13 15 60

eyIgman a  b  c  d 1  a  b  c  d
2 2 2 2

132 13 a2  a  b2  b  c2  c  d 2  d 1  0
60
dUcenH épÞRkkLargVg;carwkkñúgRtIekaNKW S  
13
.  2

1  2
4  
1  2
4  
1  2
4  
1
 a  a    b  b    c  c     d  d    0
4
2 2 2 2
 1  1  1  1
 a    b    c     d    0
235 KNnatémøénkenSam P  a 2010
 b 2011  c 2012  2  2  2  2

tamrUbmnþÉklkçN³PaB eday  a  12   0 , b  12   0 ,  c  12   0 ,  d  12   0


2 2 2 2

       
a  b  c 3  a 3  b 3  c 3  3a  b b  c a  c  naM[  a  12    b  12    c  12    d  12   0 BitCanic©
2 2 2 2

       
smµtikmµ ³ a  b  c  1 nig a  b  c 3 3 3
1

naM[ 1  1  3a  bb  ca  c


3 dUcenH a 2  b2  c2  d 2 1  a  b  c  d .

 735 
237 rkGayumñak;² ab a  b 
2
239 RsaybMPøWfaeK)an 
cd c  d 2
tag a CaGayu«Buk nig b , c , d CaGayukUnTaMgbI
eyIgmansmamaRt ³ smmUl a c

a b
 (1)
bRmab; ³ GayuGñkTaMgbYnmansmamaRt 15 , 7 , 5 , 4 b d c d

ebItag x CapleFobsmamaRtGayuGñkTaMgbYn -KuN (1) nwg db eyIg)an ³ ab b 2



cd d 2
(2)

eyIg)an ³ 15a  b7  5c  d4  x -KuN (1) nwg ac eyIg)an ³ a 2 ab


 (3)
c 2 cd
Taj)an ³ 15a  x  a  15x b7  x  b  7 x tam (2) nig (3) eyIg)anpleFobesµItKñaKW ³
2 2 2 2
c
5
 x  c  5x
d
4
 x  d  4x
a
c

ab b
cd d
 b¤ G acsresr 2
a
c

2ab b

2cd d2 2 2

smµtikmµ³ Gayu«BukticCagplbUkGayukUnTaMgbI 3qñaM eyIg)an a  2ab  b  a  2ab  b 2 2 2 2

c 2cd d c  2cd  d2 2 2 2

eyIg)an a  3  b  c  d a  b  2

15 x  3  7 x  5 x  4 x c  d 2
x3 2ab a  b
2
ab a  b
2
naM[   
naM[ ³ 15a  3  a  45 b
7
 3  b  21 2cd c  d 2 cd c  d 2

ab a  b 
2
c
5
 3  c  15
d
4
 3  d  12 dUcenH bgðaj)anfa 
cd c  d 2
.
epÞógpÞat; ³ 45  3  2115 12  48  48 Bit 240 rkBIrcMnYnKt;viC¢mantKña
dUcenH «BukmanGayu45qñaM nigkUnTaMgbImanGayu tag x CacMnYnTI1 enaHcMnYnTI2 bnÞab;KW x 1
erogKñaKW ³ 21qñaM , 15qñaM , 12qñaM . tambRmab;RbFaneyIg)an ³
60 50 x  12  x 2  321
238 eRbóbeFobBIrcMnYn  259  nig  625
225 


x 2  2 x  1  x 2  321
60 2 x  321  1
 9
60
  3 2   3
260
 3
120

Binitü ³  25 

            x
320
 160
 5   5 5 2

ehIy  225 

50
  15  2 

        3
50
250
 3
 
100
eday x  160 CacMnYnTI1 enaHcMnYnTI2 KW x  1  161
 625    25   5 5
120 100
dUcenH cMnYnKt;viC¢mantKñaenaHKW 160 nig 161 .
 3  3
eday  
5
 
5
eRBaHvamaneKal 53  1
60 120 100 50
241 etIekµgNaQñH ehIyekµgNacaj;
 9   3 3  225 
naM[    
 25  5
 
5
 
 625  tag N CacMnYnXøIsrub mann½yfa A  B  C  N
 9
60
 225 
50 -muneBlelg eyIgmansmamaRt ³
dUcenH eRkayBIeRbóbeFob   
 25 

 625 
. A B C
 
3 4 5
¬lMhat;xagelImaneRcInnak;Nas; EdlRclMePøcKitfa eKaltUcCag!¦

 736 
tamlkçN³smamaRteyIgGacsresr)an ³ 243 edaHRsayRbB½n§smIkar ³
A B C A BC N  x  xy  y  1
   

3 4 5 3 45 12 eyIgman ³  y  yz  z  4 eyIg)an
Taj)an ³  z  zx  x  9

A N 3N 12 N  x  xy  y  1  x  xy  y  1  1  1
   A 
3 12 12 48  
B N 4 N 16 N  y  yz  z  4   y  yz  z  1  4  1
   B   z  zx  x  9  z  zx  x  1  9  1
4 12 12 48  

C N
  C
5 N 20 N
  x y  1   y  1  2   y  1 x  1  2 i 
 
5 12 12 48  y  z  1   z  1  5    z  1 y  1  5 ii 
-eRkayeBlelg eyIgmansmamaRt ³  z  x  1   x  1  10

 x  1 z  1  10 iii 

A B C
 
15 16 17
edayKuNsmIkar ³ i  ii iii bBa¢ÚlKña eyIg)an
x  12  y  12 z  12  100
tamlkçN³smamaRteyIgGacsresr)an ³ x  1 y  1z  1  10
A B C A B C N
   
15 16 17 15  16  17 48 -cMeBaH x  1 y  1z  1  10 
Taj)an ³ EcksmIkar  nigsmIkar i , ii , iii eyIg)an ³

A N
  A
15N  x  1 y  1z  1 10
15 48 48   y  1x  1  2
  z 1  5 z  4

B N
  B
16 N  x  1 y  1z  1 10  
    x `1  2   x  1
 z  1 y  1
16 48 48 5  y 1  1  y  0

C N
  C
17 N   x  1 y  1 z  1 
10  
17 48 48  x  1z  1 10
-eRbobeFobmuneBlelg nigeRkayeBlelg ³ -cMeBaH x  1 y  1z  1  10  
ekµg A ³ 1248N  1548N mann½yfa A QñH EcksmIkar   nigsmIkar i , ii , iii eyIg)an ³
ekµg B ³ 1648N  1648N mann½yfa B rYcxøÜn  x  1 y  1z  1  10
  y  1x  1  2
  z  1  5  z  6
 x  1 y  1z  1  10  
ekµg C ³ 2048N  1748N mann½yfa C caj;     x `1  2   x  3
 z  1 y  1 5  y  1  1  y  2
 x  1 y  1z  1   10  
dUcenH ekµg ³ A QñH / B rYcxøÜn nig C caj; .  x  1z  1 10

242 edaHRsaysmIkar dUcenH RbB½n§smIkarmancemøIyBIrKW ³


x x x x
    1   
1 1 1 x , y, z   1, 0 , 4 b¤ x , y , z    3,  2 ,  6 .
3 6 9 12 2 3 4
x 1 1 1  1 1 1
  1       1    
3 2 3 4  2 3 4
244 eRbóbeFobcMnYn ³
x
  1  x  3 eyIgman 2 manelx 2 cMnYn 1001dg
2
22

3
dUcenH smIkarmanb¤s x  3 . ehIy 3 manelx 3 cMnYn 1000dg
33
3

Binitü ³ 2  2  16 ehIy 3  27
22 4 3

 737 
eXIjfa 16  27  2  3 22 225  240t  64t  64t  64
3
2 2

240t  289
eyIgdwgfa 2 tUcCag 3 Edlman 998 dg
2
22 33
3

289
t
CasV½yKuNbnþbnÞab;dUcKña 240
289 960  289 671
enaHeyIg)an 16  27 Edlmanelx2 nigelx3
2
22
3
33 naM [ x  4t  4
240

240

240
3

cMnYn 998 dgCasV½yKuNbnþbnÞab;dcU Kña epÞógpÞat; cMBaH x  240


671

eday 16 ¬man dgelx ¦  2 ¬man dgelx ¦


2
22
998 2
2
3 x  4 x  4 x  5 x  5 x  3 x
22
1001 2

3
3
3
3
49 289 289 529 529 49
273 ¬man 998 dgelx 3¦  33 ¬man 1000 dgelx 3¦  
240 240
 
240 240
 
240 240
7 2 17 2 17 2  232 232  7 2
dUcenH 2 22
2

¬man 1001 dgelx 2¦  3 33


3

¬man 1000 dgelx 3¦ 


2402

2402

2402
7 17 17  23 23  7
245 edaHRsaysmIkar 
240

240

240
x  3 x  4 x  4 x  5 x  5 x  3 x 119  391  161 671
 
¬vaCasmIkarGsniTan caM)ac;RtUvykcemøIEdlrkeXIjmkepÞógpÞat; ¦ 240 240

smIkarmann½ykalNa ³ eday x
671
240
enaH 671
240

671
240
Bit
3  x  0 x  3
 
4  x  0   x  4  x  3 dUcenH smIkarmanb¤s x  240
671
.
5  x  0 x  5
 
-edIm,IkMu[EvgeBk nigBi)akkñúgkarsresr ³ 246 rkcm¶ayBIsVayRCMeTAsVayerog
 x  4t

-smµtikmµ ³ begÁalcm¶ayTI1dak;fa {sVayRCM 17 km }
eyIgtag 4  x  t  3  x  t 1 eyIg)an ³ mann½yfaeFVIdMeNIr 17 km eTot dl;sVayRCM
5  x  t  1

4  t  t 1  t  t  t  1  t  1  t 1
ehIy begÁalcm¶ayTI2bnÞab;dak;fa {sVayerog 25 km }
4t  t  1t  t t  1  t  1t  1 mann½yfaeFVIdMeNIr 25 km eTot dl;sVayerog .
4  t  t 2  t  t 2  t  t 2 1 eyIgGacKUsrUbgayemIl
4  t   t 2 1  t 2  t  t 2  t PñMeBj 1km sVayRCM sVayerog
elIkGgÁTaMgBICakaer eyIg)an ³  
25km
 

?
4  t   t 2  1  t 2  t  t 2  t 17km

4  t 2  24  t  t 2  1  t 2  1  2t 2  2t t 2  1 tamrUbeyIgGacbkRsay)any:aggayfa
4  t 2  t 2  1  2t 2  2t t 2  1  24  t  t 2  1
cm¶ayBIsVayRCM eTAsVayerogKW ³
15  8t  8 t 2  1
25  1  17  26  17  9 km
elIkGgÁTaMgBIrCakaermþgeToteyIg)an ³
dUcenH cm¶ayBIsVayRCMeTAsVayerogKW 9 km .
 738 
247 rkcMnYncab nigcMnYnpáaQUk 249 rkGayurbs;davIRsIsBVé;f¶
tag x CacMnYncab nig y CacMnYnpáaQUk -smµtikmµ ³ Gayurbs;davItageday x ehIy x Ca
-tamsmµtikmµ ³ cabmYyTMpáaQUkmYy enaHsl;cab BhuKuNén mann½yfa x Gacmantémø CaeRcIndUcCa ³
mYyKµanpáaQUkTM eyIg)ansmIkar ³ 0 , 8 , 16 , 24 , 32 , 40 , 48 , ...

x 1  y i  kñúgcMeNamelxTaMgGs;xagelI manEtelx 16 eTEdl


-ehIy ebIcabBIrTMpáaQUkmYy enaHenAsl;páaQUkmYy eKarBtamlkçx½NÐ FMCag10 nigtUcCag 20
KµancabTM eyIg)ansmIkar ³ na[témø x manEtmYyKt;KW 16
x
 y 1 ii
2 dUcenH sBVéf¶davIRsImanGayu 16 qñaM .
-tamsmIkar i & ii eyIgcgCaRbB½n§smIkarKW ³
x 1  y
  x  y 1  x  y 1
250 rkcMnYnéf¶EdlKat;pÁt;pÁg;cMNIdEdl[eKa
   
x
  y 1 x  2 y  2  x  2 y  2 -eyIgsegáteXIjfa kalNacMnYneKakan;EteRcIn
2
edaybUkGgÁ nigGgÁénsmIkarTaMgBIr eyIg)an ³ enaHry³eBlsIukan;EtfycuH vaCasmamaRtRcas
 x  y 1 naM[ cMnYnéf¶EdlRtUvpÁt;pÁg;KW ³

35  40 1400
 x  2 y  2 40
40  10 
 35  
40  10  50
 28 éf¶
y 3  x  y 1  3 1  4

dUcenH cabmancMnYn 4 k,al / páaQUkmancMnYn 3 Tg . ¬BMuEmn 40  10 35  43.75 éf¶enaHeT ¦


40

248 rkcMnYnstVExVkEdlgab; nig rs; dUcenH cMnYnéf¶EdlRtUvpÁt;pÁg;KW 28 éf¶ .


-eyIgKYKb,IKitfa eRkABIstVExVkgab; nigrs;vaKµanGVI 251 bgðajfa a  a  a  1 2
1
2
2
2
3

eToteT ehIysmµtikmµniyayfa gab;bIrs;BIr enaH eyIgman ³


eyIgGacsnñidæan)anfa stVTaMgGs;mancMnYn 5k,al . a  sin a1 1  a  sin a 2
1
2
1

¬eRkABIgab; nigrs;KµanGVIeToteLIy ¦ a2  cos a1  sin a2  a  cos a1  sin 2 a2


2
2 2

-BicarNafa ³ ebIstV5 gab;3 enaHstV120gab;b:unµan? a  cos a  cos a  a  cos a  cos a


2 2 2
3 1 2 3 1 2

naM[ cMnYnstVgab;  1205 3  72 k,al naM[ a  a  a 2


1
2
2
2
3

 sin a  cos a  sin a  cos a  cos a


2
1
2
1
2
2
2
1
2
2

ebIstV5 rs;;2 enaHstV120rs;b:unµan?  sin a  cos a sin a  cos a 


2
1
2
1
2
2
2
2

naM[ cMnYnstVrs;  1205 2  48 k,al  sin a  cos a  1


2
1
2
1

eRBaH sin   cos   1


2 2

dUcenH cMnYnstVExVkEdlgab;man 72 k,al


ehIycMnYnstVExVkEdlrs;man 48 k,al . dUcenH eXIjfa a  a  a  1 R)akdEmn . 2
1
2
2
2
3

 739 
252 KNnaplKuN P n abc  ca 
1

1
abc ca
3
Binitü ³ 1  a  a  1  2a
2 4 2
 a4  a2  edaybUkGgÁngi GgÁénvismPaB ³ 1  2  3

 1  2a  a 2 4
 a 2

eyIg)an ³
 1  a   a 2 2 2

 1
 1  a  a 1  a  a 
1
a  b  c  a  b
2 2

 1  a  a 1  a  a  2 2 
 1 1
 
naM[  1 a  a 
1 a2  a4
 2 a  b  c b  c
1 a  a2  1 1
a  b  c  c  a
ebIbþÚ a Ca x eyIg)an ³
2k

3

1 1 1
  
1  x  x   11xx  xx
2k 1 2k  2
2k 2k 1 abc ab bc ca
2k 2k 1

eyIg[témø k  0 , 1 , 2 , 3, ... , n dUcenH eXIjfa a  3b  c  a 1 b  b 1 c  c 1 a .


ebI k  0 ³ 1  x  x   11xx xx
2 4
2
2
254 KNnaplbUk S  11!2  2!3  3!  n  n!
n

ebI k  1 ³ 1  x  x   11  xx  xx
4 8
2 4
2 4 Binitü ³ k  1! k  1  k!
k  1! k  k!k!
ebI k  2 ³  1  x  x   11 xx  xx
8 16
4 8
4 8
naM[ k  k!  k  1!k!
>>>>>>>>>>>>>>>> >>>>>>>>>>>>>>>>>>>>>>>>>>>>>>>>>>>>>>>>>>> ebI[témø k  1 , 2 , 3 , 4 , ... , n eyIg)an ³
1 x  x   1 x  x
2n 1 2n  2
ebI k  n ³ 2 n
2 n 1

n 1 1 1! 2!1!
1 x2  x2
n
2  2! 3!2!

1  x  x 1  x
2 2

 x 4 1  x 4  x8  1  x 2  x 2   n n 1
 3  3! 4!3!

4  4! 5!4!
n 1 n2
1 x2  x2

1 x  x2 .................

b¤Gacsresr n  n! n  1!n!
n 1 n2
1 x2  x2
Pn 
1 x  x2
1 1!2  2!3  3!... n  n! n  1!1!

dUcenH KNna)an P  1 1x x xx n


2 n 1 2n  2

2
. dUcenH eRkayBIKNnaeXIjfa S n  n  1!1 .
253 bgðajfa a  3b  c  a 1 b  b 1 c  c 1 a 255 KNnaplbUk   n  9999
1

n 1 4 n  4 n 1 



n 1  
tambRmab; a  0 , b  0 nig c  0 Binitü ³  n  1  n  n  1  n    n  1  n 
4 4 4 4

naM[ Et  n  1  n  n  1  n   1
abc  ab 
1

1
abc ab
1 naM[  n  1  n    n  11  n 
abc bc 
1

1
2 eyIg)an  n  1  n  n  1  n    n  11  n 
4 4 4 4
abc bc

 740 
Taj)an  n  1  n    n  1  n 1 n  1  n  257 RsaybBa¢ak;faeK)an  a  b   a  b
4 4
4 4
4 4 4

cd  c d 4 4

eyIg)an ³ eyIgman
   n  n  1 n  n  1    n  1  n  ba  dc  ac  db  ac  db  ca  db
9999 1  9999 4 4 4 4
4
1 4

 
4 4 4 4 4 4
n 1 n 1

ebI[témø n  1 , 2 , 3 , ... , 9999 eyIg)an ³ mü:ageTot ac  db  ca  db


4 2  4 1
 a   b   a b 
4 4 4

4 3  4 2
smmUl     
 c  d  cd 
 2
 4 4
 4 4  4 3 a4 b4  a   b 
............
tam 1& 2     
c4 d 4  c   d 

a4  b4  a  b 
4
4 10000  4 9999
 naM[  
c4  d 4  c  d 
 
9999
4
n  1  4 n  4 10000  4 1
 a b  a4  b4
n 1 4

dUcenH Rsay)anfa    .
 
9999
4
n  1  4 n  10  1  9 cd  c4  d 4
n 1

dUcenH 9999  1 
. 258 rkrUbmnþTUeTA
   
9

n  n  1 
n 1  n  n 1 4 4
eyIgman]TahrN_ ³
6 2  5 2  11
a 3  b 3  c 3  3abc
256 sRmYlkenSam A 2 2 2
a  b  c  ab  bc  ca 56 2  452  1111

eday a  b  c  3abc
3 3 3 5562  4452  111111
55562  44452  11111111
 a 3  3a 2 b  3ab 2  b 3  c 3  3a 2 b  3ab 2  3abc
..................................................
 a  b   c 3  3aba  b  c 
3


 a  b  c a  b   a  b c  c 2  3aba  b  c 
2
 eyIgeXIjfa elx1 ekIneLIgeTVdgCanic©

 a  b  c  a 2  2ab  b 2  ac  bc  c 2  3ab  dUcenH tamlMnaMKMrUxagelIenHeyIgTaj)anrUbmnþTUeTA
 a  b  c a 2
 b 2  ac  bc  c 2  ab  555 ... 56 2  444 ... 45 2  111 ... 111
 a  b  c a 2
 b2  c2  ab  bc  ac  man n xÞg; man n xÞg; man 2n xÞg;
naM[ RsaybBa¢ak;rUbmnþxagelI
a  b  c  3abc
tag
3 3 3
A An  555 ... 56 2  444 ... 45 2
a  b 2  c 2  ab  bc  ca
2

    555...55  1   444...44  1
2 2


a  b  c  a 2  b 2  c 2  ab  bc  ca

a 2  b 2  c 2  ab  bc  ca  5  4
2

   999...99  1    999...99  1
2

 abc 9  9 
Edl a  b  c  ab  bc  ca  0
2 2
5  4 
  10n  1  1   10n  1  1
2 2 2

9  9 
dUcenH eRkayBIsRmYl A  a  b  c .
 741 
2 2
cos2 1o  sin 2 1o  1
5
 
 4
An   10n  1  1   10 n  1  1

   2 o
9  9  cos2 1o  cos2 89o cos 2  sin 2  1
2 o

 2 o cos2 3o  sin 2 3o  1
5
 
4 
  10n  1  1  10 n  1  1    cos 2  cos 88
2 o


9 9  cos2 3o  cos2 87 o  ........................


5
 4
 
  10n  1  1  10n  1  1   ........................
cos2 44o  sin 2 44o  1

9 9  cos2 44o  cos2 46o  2
 2
 cos 45   
1
 
   2 o
  10n  1  10n  1  2    2 
9 
cos2 45o  

S 1
1

 10n  1 10n  1
9
  44 
2
1 88  1 89
1

 102 n  1  enaH S  44 
2

2

2
9
 999...999
1
9
Edlmanelx cMnYn 2n
9 dUcenH KNna)an S
89
.
2
 111...11 Edlmanelx 1 cMnYn 2n

dUcenH CakarBit rUbbmnþenHBitCaRtwmRtUvKW 260 KNna


S  sin 2 0o  sin 2 1o  sin 2 2o    sin 2 90o
555 ... 56 2  444 ... 45 2  111 ... 111
man n xÞg; man n xÞg; man 2n xÞg; ¬RsedoglMhat;TI259 ¦
259 KNna S  cos 1  cos 2  cos 3  cos 89
2 o 2 o 2 o 2 o
tamrUbmnþ sin 2     cos 
tamrUbmnþ cos 2     sin  eyIg)an ³
sin 2 0 o  cos2 0 o  1
b¤ cos90     sin 
o
sin 2 0 o  sin 2 90 o
 2 o
sin 1  cos 1  1
2 o

naM[ cos1  cos90  89   sin 89


o o o o  2 o
sin 1  sin 89
2 o
sin 2 2 o  cos2 2 o  1

sin 2 2 o  sin 2 88o 
 ........................

cos 2 o  cos 90 o  88 o  sin 88 o  
 sin 2 44 o  cos2 46 o  1
>>>>>>>>>>>>>>>>>>>>>>>>>>>>>>>>>>>>>>>>>>>>>>>>> ........................ 
sin 2 44 o  sin 2 46 o   2
2

  sin 45   
 
2 o
cos 44 o  cos 90 o  46 o  sin 46 o   
sin 2 45o  2 

enaHeyIg)an ³ S
45 
1
2
cos2 1o  sin 2 89o /
1 90  1 91
cos2 2o  sin 2 88o enaH S  45   
>>>>>>>>>>>>>>>>>>>>>>>>>>>>> 2 2 2

cos2 44o  sin 2 46o dUcenH KNna)an S


91
2
.
enaHeyIg)an ³

 742 
261 kMNt;témø a & b ¬eRBaH b  9 minRtUvmanRtaTukeT ¦
eyIgmancMnYn abba Edl 1  a  9 , 0  b  9 -cMeBaH b  0 eyIg)an 10 c9  9  9c01 4
eyIg)an abba  1000 a  100 b  10b  a tamTMnak;TMng 4 Taj)an c  8 eRBaH 8  9  72
 1001 a  110 b
ehIyEfmRtaTuk 8 KW 72  8  80 manelx0enAcug
 1191a  10 b 
smµtikmµ abba CaKUbR)akdéncMnYnKt; enaHryIg)an -cMeBaH c  8 eyIg)an 1089 8  9801
abba  k 3
dUcenH témøkMNt;)anKW a  1 , b  0 , c  8 , d  9
1191a  10b   k 3

91a  10b 
k3
11
x> bBa¢ak;fa abcd nig dcba suT§EtCakaerR)akd
eday a & b CacMnYnKt; enaH 91a 10b CacMnYnKt;Edr eday abcd  1089  33
2

naM[ k Eckdac;nwg 11 enaH k  11n / n Kt;viC¢man ehIy dcba  9801  99


2
3

eyIg)an k  11n Edl n  1 , 2 , 3 , ...


3 3
dUcenH cMnYn abcd nig dcba suT§EtCakarR)akd .
EtebI n  2 enaH k  22   10648  abba
3 3

naM[ cMnYnKt;viC¢man n mantémøEtmYyKt;KW n  1 263 rkBIrcMnYnCakaerR)akd


eyIg)an abba  k  11 1  11  1331
3 3 3 tag a  k CacMnYnTI 1 nig b  n CacMnYnTI 2 Edl
2 2

k & n CacMnYnKt; ehIy k  n b¤ k  n


Taj)an témø a  1 , b  3
tambRmab;RbFan ab  a  b  4844
dUcenH témøkMNt;)anKW a  1 , b  3 . eyIg)an ³ ab  a  b  4844
ab  a  b  1  4844  1
262 k> kMNt;témøelxénGBaØat a , b , c nig d ab  1  b  1  4845
eyIgmancMnYn abcd nig dcba tamxÞg;nImYy²enaH a  1b  1  4845
1 a  9 , 0  b  9 , 0  c  9 , 1 d  9
eday a  k nig b  n enaHeyIg)an ³ 48453
2 2

a  1b  1  4845 1615 5


eK[ abcd  9  dcba 1 k  1n  1  4845
2 2
323 17
19 19
tamTMnak;TMng 1 naM[kMNt;)antémø a  1 EtmYyKt; k  1k 1n  1n 1  4845 1
¬eRBaHebI a  1 enaHlT§plCaelx R)aMxÞg; ¦ Et 4845  3 51719 CaplKuNénbYncMnYnKt;
-cMeBaH a  1 eyIg)an 1bcd  9  dcb1 2 EdlmanEtmYyEbbKt;
tamTMnak;TMng 2 naM[kMNt;)antémø d  9 EtmYyKt; eday 3 5  4 14  1 nig 17 19  18 118 1
¬eRBaHmanEtkrNIenHeTIb 9  9  81 manelx!cug¦ enaH k 1k 1n 1n 1  4 14 118 118 1
-cMeBaH d  9 eyIg)an 1bc9  9  9cb1 3 -cMeBaH k  n Taj)an k  18 , n  4
tamTMnak;TMng 3 Taj)an b  0 -cMeBaH k  n Taj)an k  4 , n  18
 743 
naM[ 
a  k  18  324

2 2
265 rkcMnYnTaMgBIrenaH

b  n  4  16
2 2


tag x CacMnYnTI ! Edl x CacMnYnbzm enaHcMnYnTI@KW
a  k  4  16
2 2

b¤ 
 x  1 ehIy x  1 CakaerR)akd enaH x  1  n 2

b  n  18  324
2 2

epÞógpÞat; ³ 324 16  324  16   4844 Bit eyIg)an n 1  x smmUl n  1n  1  x


2

ehIy 16  324  16  324   4844 Bit Et x CacMnYnbzm naM[plKuNrbs; x KW x  x 1


eyIg)an n 1n  1  1 x
dUcenH BIrcMnYnEdlCakaerR)akdKW 324 & 16 . Taj)an ³ nn  11  1x  nn 12 x  3n  x2
  
264 KNnargVas;RCugnImYy²rbs;RtIekaN
tag a , b , c CargVas;RCugén dUcenH cMnnY TI!KW 3 nigcMnYnTI@KW 4 .
a c
RtIekaNEkg Edl c CaGIubU:etnus 266 rkkaMrgVg;
-tamBItaK½r ³ a  b  c 1
2 2 2 b
tag r CakaMrgVg;EdlRtUvrk 2m
-bRmab;RkLaépÞesµI 24 cm naM[ 12 ab  24 2
2
edayP¢ab;p©itrgVg;TaMgbIdUcrUb
-bRmab;brimaRtesµI 24 cm naM[ a  b  c  24 3 enaHeyIg)anRtIekaNEkg EdlmanTMnak;TMngCamYy r
tam 3 ³ a  b  24 c elIkCakaer tamBItaK½reyIg)an ³
a 2  2ab  b 2  242  48c  c 2 2  2r 2  2r 2  4r 2
a 2

 b 2  2ab  576  48c  c 2 4 4  8r  4r 2  4r 2  16r 2
edayyk 1 & 2 CMnYnskñúg 4 eyIg)an ³ 8r 2  8r  4  0
c 2  2  48  576  48c  c 2 2r 2  2r  1  0
96  576  48c tam   b  ac  1 2  3
2

480
48c  576  96 c
48
 10 cm naM[ r   1 2 3  0 minyk
1

cMeBaH c  10 enaH a  b  14 nig ab  48 r1 


1 3
ÉktaRbEvg
tamEvüteyIg)ansmIkar ³ x  14x  48  0 Edl
2 2

man a nig b Cab¤sénsmIkar dUcenH kaMrgVg;EdlRtUvrkKW ³ r   12 3


.
  b 2  ac  49  48  1
enaH a
7 1
1
6 , b
7 1
1
b¤ a  8 , b  6 267 RsayfaRtIekaNcarwkknøHrgVg;CaRtIekaNEkg
8
¬munbkRsay GñkRtUvsÁal; mMucarwk C
dUcenH rgVas;RCugnImYy²énRtIekaNEkgKW ³ nig mMup©ti Camunsin ³ mMucarwk CamMmu ankMBUl 
o
6 cm , 8 cm , 10 cm . elI r gV g ; ni gRCu g TaM g BI r kat; r gV g ; ÉmM u p © i t CamM u
EdlmankMBUlelIp©itrgVg; . A B

 744 
-TMnak;TMngrvagmMup©it nigmMucarwkKW 269 RsayfaRCugnImYy²énRtIekaNtUcCagknøH
ebI  CamMup©it nig  CamMucarwkEdlmanFñÚsáat;rYm brimaRtrbs;vaCanic©
enaHeyIg)anTMnak;TMng ³   2 . ]bmafaeyIgmanRtIekaN a c

-RtIekaNcarwkknøHrgVg; CaRtIekaNEdlmanRCugmYy EdlmanrgVas;RCug a , b , c b

CaGgát;p©iténrgVg; EckrgVg;CaBIrEpñkb:unKña nigmanknøHbrimaRt a  2b  c


-¬emIlrUb¦ mMu nig A manFñsÚ áat;rYm BC -tamvismPaBénRtIekaN eyIg)an ³
naM[ A  2 eday  CamMurab enaH   180 o
 a bc  2a  a  b  c  a 
abc
2
o abc
eyIg)an ³ A  1802  90 CamMuEkg . o
 b ac  2b  a  b  c  b 
2
abc
naM[ RtIekaNcarwkknøHrgVg;manmMuEkgCanigc©  c  ab  2c  a  b  c  c 
2

dUcenH RtIekaNcarwkknøHrgVg;CaRtIekaNEkgCanic© . dUcenH rgVas;RCugnImYy²énRtIekaNtUcCagknøH


brimaRtrbs;vaCanic© .
268 rkcMnYnRCugeRcInbMputénBhuekaN
-TMnak;TMngrvagcMnYnRCug nigmMuénBhuekaNKW ³ 270 rkcMnYndgénkarCk;)arIrbs;bursenaH
ebIBhuekaNmYyman n RCugenaHplbUkmMukñúgTaMgGs; -)arI 10 edIm  Ck;)an 10 dg
énBhuekaNKW ³ 180 n  2 ;
o enAsl;knÞúy)arIcMnYn 10 knÞúy  3 edIm nig 1 knÞúy
tambRmab;eyIg)an ³ -)arI 3 edIm nig 1 knÞúy  Ck;)an 3 dg
180 n  2   2011
o o enAsl;knÞúy)arIcMnYn 4 knÞúy  1 edIm nig 1 knÞúy
n2
2011 o

o
-)arI 1 edIm nig 1 knÞúy Ck;)an 1 dg
180
2011 enAsl; knÞúy)arIcMnYn 2 knÞúy x©IknÞúy)arIeK 1
n 2
180 naM[)an 3 knÞúy  1 edIm  Ck;)an 1 dg
1980  31
n
180
2 enAsl;knÞúy)arI 1 yksgeKvij enaH)arIGs;Kµansl;
n  11 
31
2 naM[cMnYndgénkarCk;)arIKW  10  3 11  15 dg
180
n  13 
31 dUcenH bursenaHRtUvCk;cMnYn 15 dgeTIbGs;)arIKat; .
180
Et n CacMnYnRCug EdlCacMnnY Kt; enaH n  13 271 edaHRsaysmIkar ³ k> 3  4  5 x x x

epÞógpÞat; 180 13  2  2011  1980  2011 Bit eyIgsegáteXIjfa x  2 Cab¤sénsmIkar


o o o o

eRBaH 3  4  5  9  16  25  25  25 Bit
2 2 2

dUcenH BhuekaNenHGacmanRCugeRcInbMputcMnYn 13 .
eyIgnwgbgðajfasmIkarmanb¤sEtmYyKt;KW x  2
 745 
x x x x x 3 3 3
 3  4 3  4 5 3  4 5
smIkarxagelIGacsresr ³      1
5  5
           
6 6 6 6 6 6
x 2 x x x
 3  3  3  4 5
-krNI x  2 eyIg)an    
5 5
smmUl         1
6 6 6

ehIy
x
4 4
   
2
naM[ x  3 minEmnCab¤sénsmIkar .
5 5 x 3
x
 3  4  3  4
x 2 2 -krNI x  3 eyIg)an  63    63 
edaybUkGgÁ nigGgÁ        
5  5 5  5 x 3
4 4
x x ehIy    
smmUl  53    54   1 6 6
x 3
5 5
naM[ x  2 minEmnCab¤sénsmIkar . ehIynig    
6 6
x

-krNI x  2 eyIg)an  53    53 


2
edaybUkGgÁ nigGgÁ enaHeyIg)an ³
x x x 3 3 3
 3  4 5  3  4 5
4
x
4
2
            
ehIy    
5 5
6 6 6 6 6 6
x x x
 3  4 5
x
 3  4  3  4
x 2 2 smmUl       1
edaybUkGgÁ nigGgÁ        
5  5 5  5
6 6 6
x
 3  4
x naM[ x  3 minEmnCab¤sénsmIkar .
smmUl     1
5  5 dUcenH smIkarmanb¤sEtmYyKt;KW x  3 .
naM[ x  2 minEmnCab¤sénsmIkar .
272 bgðajfa eKTaj)an x  y  z
dUcenH smIkarmanb¤sEtmYyKt;KW x  2 .
eyIgmansmPaB
edaHRsaysmIkar ³ x> 3  4  5  6             yz  zx  x y  y  z  2x  z  x  2 y  x  y  2z
2 2 2 2 2 2
x x x x

eyIgsegáteXIjfa x  3 Cab¤sénsmIkar eRBaH Binitü  y  z  2x  z  x  2 y   x  y  2 z 


2 2 2

3  4  5  6  216  216 Bit


3 3 3 3
tamrUbmnþ³ a  b  c  a  b  c  2ab  2ac  2bc 2 2 2 2

eyIgnwgbgðajfasmIkarmanb¤sEtmYyKt;KW x  3 enaHeyI g)anktþanImYy²KW ³


 y  z  2 x   y  z  4 x  2 yz  4 xy  4 xz
2 2 2 2

smIkarxagelIGacsresr ³  63    64    56   1  z  x  2 y   z  x  4 y  2 xz  4 yz  4 xy


x x x
2 2 2 2


 x  y  2 z   x  y  4 z  2 xy  4 xz  4 yz
2 2 2 2
x 3 
 3  3
-krNI x  3 eyIg)an    
6 6
 y  z  2 x 2  z  x  2 y 2  x  y  2 z 2 
x 3
6 x 2  6 y 2  6 z 2  6 xy  6 yz  6 xz 1
4 4
ehIy    
6 6
Binitü ³
5 5
x 3  y  z 2  z  x 2  x  y 2
ehIynig    
6 6  y 2  2 yz  z 2  z 2  2 xz  x 2  x 2  2 xy  y 2
 2 x 2  2 y 2  2 z 2  2 xy  2 xz  2 yz 2
edaybUkGgÁ nigGgÁ enaHeyIg)an ³
edayeFVIkarpÞwm 1 & 2 enaHeyIg)an
 746 
6 x 2  6 y 2  6 z 2  6 xy  6 yz  6 xz  274 k> RsaybBa¢ak;fa sin12a  cota  cot2a
2 x 2  2 y 2  2 z 2  2 xy  2 yz  2 xz
eRkayBIeFVIkarTUTat; eyIg)anliT§plKW ³ Binitü ³ cota  cot2a
cos a cos 2a
4 x 2  4 y 2  4 z 2  4 xy  4 yz  4 xz  0  
sin a sin 2a
2 x 2  2 y 2  2 z 2  2 xy  2 yz  2 xz  0
¬rUbmnþ cos 2a  2 cos a 1 , sin 2a  2 sin a cos a ¦
2

x  2 xy  y  y  2 yz  z  z  2 xz  x  0
2 2 2 2 2 2
cosa 2 cos2 a  1
x  y    y  z   z  x 
2 2 2
0  
sin a 2 sin a cosa
plbUkxagelIepÞógpÞat;manEtmYykrNIKt;KW ³ 
2 cos2 a  2 cos2 a  1
x  y 2  0 x  y  0 x  y
2 sin a cosa
   1

 y  z   0   y  z  0   y  z
2
sin 2a
 z  x  0 z  x
z  x   0  
2

dUcenH bgðaj)anfa sin12a  cota  cot2a .


naM[eyIg)an ³ x  y  z
dUcenH eKTaj)an x  y  z . x> KNnaplbUk
1 1 1 1 1
Sn     
273 KNna S  ab  cd sin a sin a a
sin 2 sin 3
a
sin n
a
2 2 2 2
eyIgman ³ tamsRmaybBa¢ak;sMNYr k> eyIg)an ³
2011ab  cd 
S  ab  cd   1 a
 sin a  cot 2  cot a
2011
2011ab  2011cd 

2011  1  cot a  cot a
 a

  
c  d 2 ab  a 2  b 2 cd
2
  sin
4 2
2011  2
 1 a a
abc  abd 2  a 2 cd  b 2 cd
2
  cot  cot
  sin a 8 4
2011
 4

 2 2
 
abc  a cd  abd 2  b 2 cd  ...................................
2011 
 1  cot a  cot a
acbc  ad   bd ad  bc  
 a 2 n1 2n
2011  sin n
 2

bc  ad  0  0 a
2011 S n  cot n1  cot a
2
eRBaH smµtikmµ ac  bd  0 dUcenH eRkayBIKNna a
S n  cot n1  cot a .
2
dUcenH KNna)an S  0 . x
cot
275 k> RsaybBa¢ak;fa 1
1

cos x cot x
2

cos x  1
Binitü ³ 1
1
cos x

cos x

 747 
¬rUbmnþ cos 2a  2 cos 2
a  1  cos 2a  1  2 cos 2 a ¦ 276 KNna x , y , z nig t
1 cos x  1
2 cos2
x
2 
x
2 cos2 sin
2
x
2
smµtikmµ ebIeyIg[pleFob 2x  3y  4z  6t  a
1  
x
cos x cos x cos x
cos x sin
2 Edl a  0 eRBaH bYncMnYn x , y , z nig t viC¢man
x
cos sin x
2 sin x
cos
x
2
Taj)an ³ x  2a , y  3a , z  4a , t  6a
  
cos x sin
x cos x
2
sin
x
2
naM[ yztx  ztxy  txyz  xyzt  14625 )anCa
x 72a 3 48a 3 36a 3 24a 3
cot
x 2     14625
 tan x cot  2a 3a 4a 6a
2 cot x
36a 2  16a 2  9a 2  4a 2  14625
x 65a 2  14625
cot
dUcenH Rsay)anfa 1
1
 2
cos x cot x
. a 2  225  a  15
eyIg)an ³
x> KNnaplKuN  x  2a  x  2  15  x  30
 y  3a  y  3  15  y  45
       
 1  1  1   1      
Pn  1  1  1 
 cos x  cos x  cos x
    1 
  cos x

  z  4a  z  4  15  z  60



2  22



 2n

 t  6a t  6  15 t  90

tamsRmaybBa¢ak;sMNYr k> eyIg)an ³


 x
dUcenH KNna)an x  30 , y  45 , z  60 , t  90 .
 cot
1 2
1  
 cos x cot x 276 k> kMNt;témø x edIm,I[ P mantémøGb,brma
 x

1 
1

cot
4 eyIgman ³ P  x  x  1 ¬edayeFVIkarEfmfytY¦
2

 cos x x P  x2  x 1
cot
 2 2 2 2
 1 1 1
 cot
x  x2  2  x        1
 1 8 2 2 2
 1  
 cos x x 2 2
cot  1 1 4  1 3
 4 4 x    x  
................................  2 4 4  2 4

 cot n1
x edIm,I[ P mantémøGb,brma luHRtaEtkenSam
 1 2
1  
2
 1 1 1
 cos n
x
cot n
x x  0  x 0  x  
 2 2  2 2 2



dUcenH témø
x
1
2
PeFVI[ mantémøGb,brma .
3
4
cot n1
x
2  tan x cot x
x> kMNt;témø x edIm,I[ Q mantémøGtibrma
Pn 
cot x 2 n1 eyIgman Q  2  x  x Gacsresr ³
2

dUcenH eRkayBIKNna Pn  tan x cot


x
n 1
. 
Q  2  x2  x  2  x2  x 
2

 748 
 2 1  1   1  
2 2
n  2a  1 n  1  2 a

Q  2  x  2 x      n  3b  1 n  1  3b
 2  2   2  
  
 1 1
2 n  4c  1  n  1  4c
 2x    n  5d  1 n  1  5d
 2 4  
2 n  6e  1 n  1  6e
9  1
 x 
2  2 naM[ n 1 RtUvEtEckdac;CamYy 2, 3, 4, 5 , 6
edIm,I[ Q mantémøGtibrma luHRtaEtkenSam mann½yfa n 1 CaBhuKuNén 2, 3, 4, 5 , 6
2
 1
x  0  x 0  x 
1 1
eday PPCM 2 , 3, 4 , 5 , 6  2  3  5  60 2

 2 2 2
dUcenH témø x  12 eFVI[ Q  94 mantémøGtibrma . naM[BhuKuNbnþbnÞab;én 2, 3, 4, 5 , 6 KW ³
 60 ,120 ,180 , 240 , 300 , 360 , 420 , ... enaHnaM[
n  1   60 , 120 , 180 , 240 , 300 , 360 , 420 , ...
278 rkcMnYnKt;tUcbMput n n   61 , 121 , 181 , 241 , 301 , 361 , 421 , ...
ebI a , b , c , d , e CacMnYnKt; Etsmµtikmµ n Eckdac;nwg 7
tambRmab;eyIg)an ³ ehIy kñúgcMeNamtémø n nImYy² tamlMdab;BItcU eTAFM
 n  2 a  1  n  2 a  1
n  3b  2 n  3b  2 manEttémø 301 eT EdltUcCageK nig Eckdac;nwg 7
 
n  4c  3  n  4c  3  dUcenH cMnYnKt;tUcbMputEdlRtUvrkKW n  301 .
 n  5d  4 
n  5d  4
 
n  6e  5 n  6e  5
280 RsaybBa¢ak;fakenSam E mantémøefr
n  1  2 a  2 n  1  2a  1
n  1  3b  3 n  1  3b  1 rMlwk cos x  sin x  1
2 2

 
 n  1  4c  4  n  1  4c  1 
a 3  b 3  a  b  a 2  ab  b 2 
n  1  5d  5 n  1  5d  1

n  1  6e  6

n  1  6e  1
eyIgman
naM[ n 1 RtUvEtEckdac;CamYy 2, 3, 4, 5 , 6 E  cos6 x  sin 6 x  3sin 2 x cos2 x
    3 3
 cos2 x  sin 2 x  3 sin 2 x cos2 x
mann½yfa n 1 CaBhuKuNén 2, 3, 4, 5 , 6  cos x  sin x cos
2 2 4

x  cos2 x sin 2 x  sin 4 x 
smµtikmµ rkEttémø n EdltUcbMput  3 sin 2 x cos2 x

naM[ n  1  PPCM (2 , 3, 4 , 5, 6)  2  3 5  60
2  cos4 x  cos2 x sin 2 x  sin 4 x  3 sin 2 x cos2 x
 cos4 x  2 cos2 x sin 2 x  sin 4 x
Taj)an n  60 1  59 
 cos2 x  sin 2 x  2

dUcenH cMnYnKt;tUcbMputEdlRtUvrkKW n  59 .  12
1
279 rkcMnYnKt;tUcbMput n
dUcenH
ebI a , b , c , d , e CacMnYnKt;
tambRmab;eyIg)an ³ RKb;témørbs; x eyIg)antémø E  1efrCanic© .
 749 
281 edaHRsayRbB½n§smIkar 283 sresr N CaplKuNktþadWeRkTI1
1 eyIgman N  3a  1  4a  6a  9
eyIgmanRbB½n§smIkar x xy y 2 9
3 3 2 2

 2
 3a  1  2 2 a  3
2 2

tamsmIkar 2 Taj)an x  2y 3 CMnYskñúg 1  3a  1  2a  33a  1  2a  3
3  3a  1  2a  63a  1  2a  6
2
eyIg)an    y 3  9  a  7 5a  5
 y
8
3
 y3  9 dUcenH sresr)an N  a  75a  5 .
y
8  y6  9 y3
y6  9 y3  8  0 284 Rsayfa N Eckdac;nwg 32 ³
tag y  t  y  t
3 6 2 eyIgman N  44a  1  100 2

 2 2 4a  1  10 2
eyIg)an t  9t  8  0
2
2

 24a  1  1024a  1  10


smIkarman a  b  c  1   9  8  0  8a  2  108a  2  10
naM[ t  1 , t  8  8a  88a  12
 8a  1  42a  3
cMeBaH t  1  y  1  y  1 3

 32a  12a  3
enaHtam 3 : x  2y  12  2 eday N  32a  12a  3 CaBhuKuNén 32
cMeBaH t  8  y  8  y  2 3

dUcenH N CacMnYnRtUvEtEckdac;nwg 32 .
enaHtam 3 : x  2y  22  1

dUcenH RbB½n§smIkarmanKUcemøIy 285 KNnatémøelxénkenSam P ³


x  2 , y  1 , x  1, y  2 . eK[ a  b  1 nigkenSam
   
P  2 a 3  b3  3 a 2  b 2  1
 2a  b a  ab  b   3a  b   1
2 2 2 2

282 edaHRsaysmIkar ³  2a  ab  b   3a  b   1


2 2 2 2

eyIgman 3  3 3  3   4  3
2011 2010 2009 2008 x  2a 2  2ab  2b 2  3a 2  3b 2  1
 a 2  2ab  b 2  1
32010 3  1  32008 3  1  4  3 x
2  2  32010 2008  4  3 x

  a 2  2ab  b 2  1 
 a  b   1
2
34018  3 x
 4018  x  12  1
 1  1
0
dUcenH smIkarmanb£s x  4018 .
dUcenH témøelxénkenSamKNna)an P0 .
 750 
286 rkBIrcMnYnKt;viC¢man a nig b ³ 288 KNnakenSam A

eyIgman a  b  24
2 2
A
1

2

3
x  1x  2 2  x 3  x  1  x x  3
eyIg)an a  ba  b  24 
1

2

3
ebI a nig b CacMnnY Kt;enaH a  b nig a  b k¾Ca x  1x  2 x  2x  3 x  1x  3
x  3  2x  1  3x  2
cMnYnKt;Edr EdleyIgnwgrkplKuNBIrcMnYnKt;esµI 24 
x  1x  2x  3
1  24 x  3  2 x  2  3x  6
2  12 
eday 24  

nig a  b   a  b x  1x  2x  3
 3 8 
1
 4  6 x  1x  2x  3
a  b  1
naM[  minykeRBaH a nig b minCacMnYnKt; dUcenH kenSamKNna)anKW A  1
.
a  b  24 x  1x  2x  3
a  b  2
b¤  edaHRsay)anKUcemøIy a  7 , b  5
 a  b  12
b¤ a  b  3
 minykeRBaH a nig b minCacMnYnKt; 289 rkmYycMnYnenaH
a  b  8
a  b  4
tag x CamYycMnYnEdlRtUvrkenaH
b¤  edaHRsay)anKUcemøIy a  5 , b  1 tambRmab;RbFan eyIg)antémø 3x pÞúyBI x
a  b  6
2

mann½yfa 3x   x 2

dUcenH smIikarmanKUcemøIy ba  57 b ba  15 .


  edaHRsay 3x   x 2

3x  x 2  0
x3  x   0
287 KNnaRCugrbs;kaernImYy²
 x0
tag a CargVas;RCugkaerFM ¬KitCa m ¦ 
3  x  0
b CargVas;RCugkaertUc ¬KitCa m ¦ x  0

tambRmab;RbFan eyIg)anRbB½n§smIkar  x  3

 a 2  b 2  1152 a  b a  b   1152 cMeBaH x  0 minykeRBaH 0 KµancMnYnpÞúyeT


  
 a  b  16  a  b  16
16a  b   1152 a  b  72
dUcenH cMnYnEdlRtUvrkenaHKW 3 .
  
 a  b  16  a  b  16
edaHRsaytamviFIbUkbM)at; eyIg)an ³ 290 rkelxEdlenARtg;TItaMgtYTI 2011
a  b  72
 eyIgmanelxsresrBI 1 dl;elx 999 KW
 a  b  16 enaH b  44 16  28 N  12345678910111213...998999
2a  88  a  44
edIm,IrkelxenARtg;TItaMgtYTI 2011
dUcenH RbEvgRCugkaernImYy²KW 44 m , 28 m . eyIgsikSatamEpñk²dUcbgðajxageRkam ³
 751 
-BIelx 1  9 mancMnYntY ³ 91  9 tY x2 2  x 1   x 2  1  x 
f    f x     
2  x  x 2  x   2  x  x 
-BIelx 10  99 mancMnYntY ³ 90  2  180 tY 1 x2 x  x2
f  f x   3
-BIelx 100 199 mancMnYntY ³ 100 3  300 tY  x 2 x 2 x
-BIelx 200  299 mancMnYntY ³ 100 3  300 tY edayyksmIkar 1  3 enaHeyIg)an ³
-BIelx 300  399 mancMnYntY ³ 100 3  300 tY  1
-BIelx 400  499 mancMnYntY ³ 100 3  300 tY  x2 x  1 f  x   f  x   x  1
  

-BIelx 500  599 mancMnYntY ³ 100 3  300 tY  f  1   x f  x   x  x
2 2

  x  2  x 2 x
-BIelx 600  699 mancMnYntY ³ 100 3  300 tY  x 
2
 x  x2 
 x2 x  1   f  x    x  1   
ebIeyIgsrubtYBIelx 1 699 vamancMnYntYKW  2  x   2  x 
2 x 
 x 2  x   x 2
2
f x  
 
x  12  x   x  x 2 
9  180  6  300  1989 tY 2 x 2 x
eyIgRtUvkar 22 tYeTot edIm,IbEnßm[RKb;tYTI 2011  2 3 2 2

4 x  2 x  2 x  x  x f x   x  3x  2  x  x 2
2

ehIytYbnþbnÞab;eTot bnÞab;BI 699 enaHKW ³ 4 x  2 x  2 x  f x   2 x  2


2 3

2 xx  2 x  1 f  x   2 x  1
2

700701702703704705706707708709800...999 > f x  
2 x  1
2 x x  1
2

tYTI 1990 tYTI 2000 tYTI 2011


f x  
1
x x  1
dUcenH elxEdlenARtg;TItaMgtYTI 2011 KWelx 7 . f x   
1 1
x x 1
eday[témø x  1 , 2 , 3 , ... , 2011 eyIg)an ³
291 KNnaplbUk S ³ 
 f 1  1  2
1 1

S  f 1  f 2  f 3    f 2011  


 f 2  1  1
1
eyIgman x2 x  1 f  x   f    x  1
 x
1 

2 3

  f 3  
1 1
ebIeyIg CMnYs eday
x
1
x
enaHeyIg)an ³  3 4
.....................
1 1  1  1  1 
 2   1 f    f    1  f 2011  1  1
x x   x  1/ x  x  2011 2012
12 x 1 1 x 
  f    f x  
x x   x x 1 1
S 
2 x 1 1 x 1 2012
f    f x   2 2012  1 2011
x2  x x S 
2 2012 2012
edayKuNGgÁTaMgBIrénsmIkar 2 nwg 2x x edIm,I
[emKuNén f  1x  esµInwg 1 eyIg)an ³ dUcenH plbUkKNna)an S
2011
2012
.
 752 
292 rkcMnYnKUbticbMputEdlRtUverobkñúgRbGb; 294 rkBIrcMnYnKt; a nig b
edIm,I[)ancMnYnKUbticbMptu erobenAkñúgRbGb; enaHmaD tambRmab; ³ eyIg)anRbB½n§smIkar
rbs;KUbRtUvFMbMput. enaHeyIgRtUvrkRCugKUbFMbMput  a  b  15n
 Edl n  1 , 2 , 3 , ... eRBaH a  b
a  b  45
2 2

EdlvimaRtTaMgbIrbs;RbGb;Eckdac; nwgRCugrbs;KUb.  a  b  15n  a  b  15n


  
-mann½yfa RCugrbs;KUbCatYEckrYmFMbMptu énvimaRt a  b a  b   45 15na  b   45
rbs;RbGb; EdleyIgnwgkMNt; a  b  15n
 
a  b  15n

 a b  3
-ebItag a CaRCugrbs;KUb ¬KitCa mm ¦  na  b   3  n

naM[ a  PGCD 180 , 60 , 90   2  3  5  30 ebI a nig b CacMnnY Kt;enaH a  b k¾CacMnYnKt;Edr


eRBaH 180  2  3  5
2 2
enaH n3 CacMnYnKt; mann½yfa 3 RtUvEtEckdac;nwg n
600  2 3  3  5 2
eyIgTaj)antémø n KW n  1 b¤ n  3
90  2  32  5
naM[ maDKUbesµI V  30  27 000 mm
3 3 -cMeBaH n  1 ³ aabb153 edaHRsaytambUkbM)at;

maDRbGb;esµI V  180  600  90  9720 000 mm 3 a  b  15

 a b  3 ehIy b  6
eyIg)an cMnYnKUbEdlerob)anKW 9720000
27000
 360 KUb 2a  18  a  9
a  b  45
dUcenH cMnYnKUbticbMputEdlGacerob)anKW 360 KUb . -cMeBaH ³
n3 
 a b 1
edaHRsaytambUkbM)at;
a  b  45

 a b 1 ehIy b  22
293 edaHRsayRbB½n§smIkar 2a  46  a  23

eyIgmanRbB½n§smIkar 26xx2yy43xyxy 12


 dUcenH RbB½n§smIkarmanKUcemøIyBIrKUKW
edaysikSapldksmIkar ³ 2  2  1 a  9 , b  6 b¤ a  23 , b  22  .
12 x  2 y  8 xy

 2 x  2 y  3xy
10 x  5 xy  10  5 y 295 bgðajfa A 2
 B 2  C 2  ABC minGaRs½y
 y2 nwg x, y , z
cMeBaH y  2 tamsmIkar 1 eyIg)an ³ eyIgman
2 x  2  2  3x  2 2 2
 y z  y  z
2
2x  4  6x y z
A    A 2            2
4x  4
z y  z y  z  y
2 2 2
x 1 z x z x  z  x
B   B2         2
x z x z  x  z
dUcenH RbB½n§smIkarmanKUcemøIy x  1 , y  2 . x y  x y  x  y
2

C    C 2            2
2 2

y x  y x  y  x
 753 
VB  t  100
naM[ A2  B 2  C 2 
VC  t  90
V
 B 
VC
10
9
 VB 
10VC
9
ii 
2 2 2 2 2 2
y z z x x y
  2 2 2  2 2 2  2 2
z 2
y x z y x edaypÞwm i  & ii enaHeyIg)an ³
y2 z2 z2 x2 x2 y2 9V A 10VC 100VC
 6        VA 
z2 y2 x2 z2 y2 x2 10 9 81
ehIy ebI t Cary³eBlEdl A rt;dl;TI enaH
y z  zx  x y  100
ABC         V A  t  100  t 
z y  x
z  y x  VA
y
  
xy z 2 x  x y 
     ebI d Cacm¶ayEdl C rt;)ankñúgry³eBl t enaH
x z 2 xy y  y x  100 100
d  VC  t  VC   VC   81 m
 y x 2
y z 2
z 2
x 
2 2 2
VA 100VC
 1  2  2  2  2  2  2  1
 x z z y x y  81
y2 x2 y2 z2 z2 x2
 2 2  2  2  2  2  2
mann½yfa xN³eBl t Edl A rt;dl;TIcm¶ay100 m
x z z y x y
eXIjfa C rt;)anEtcm¶ay 81 m b:ueNÑaH
eyIg)an A2  B 2  C 2  ABC
naM[ KmøatBI C eTA A KW 100 m  81 m  19 m
y2 z2 z2 x2 x2 y2
 6      
z2 y2 x2 z2 y2 x2

dUcenH cm¶ayKmøatBI C eTA A KW 19 m .
y2 x2 y2 z2 z2 x2 
 2  2  2  2  2  2  2 
 x z z y x y 
4
297 edaHRsayRbB½n§smIkar
 x  y  z  xyz 1

dUcenH A 2
 B 2  C 2  ABC  4 minGaRs½y x, y , z . eyIgman  x  y  z  xyz
 x  y  z  xyz
2
 3
296 rkcm¶ayKmøatBI C eTA A xN³eBl A rt;dl;TI pÞwm 1 & 2 eyIg)an ³
tag V , V , V Cael,ÓnerogKñaén A , B , C x yz  x yz  x y i 
A B C

t Cary³eBlEdl A rt;dl;TI
pÞwm 2 & 3 eyIg)an ³
t  Cary³eBlEdl B rt;dl;TI x yz  x yz  y  z ii 

bRmab; ³ xN³eBl A rt;dl;TI B enAxVH 10 m pÞwm 1 & 3 eyIg)an ³


x yz  x yz  z  x iii
naM[ V  t  100 1 nig V t  90 2
A B
tam i  , ii nig iii  eyIg)an x  y  z
eyIgeFVIpleFob 1 nig 2 eyIg)an ³ edayyk x  y  z CMnYskñúgsmIkar 1 eyIg)an ³
V A  t 100

V
 A 
10 9V
 VB  A i   x  x  x  xxx
VB  t 90 VB 9 10
x  x3
bRmab;bnþ ³ xN³eBl B rt;dl;TI C enAxVH 10 m x3  x  0
naM[ V t   100 3 nig V  t   90 4
B C  
x x2 1  0
xx  1x  1  0
eyIgeFVIpleFob 3 nig 4 eyIg)an ³
 754 
 x0
x 1  0
x  0 -cMeBaH 1 51  51 eyIgpÞwmktþaRtUvKña ³
naM[    x  1
 x  1  0  x  1 eyIg)an 22 yy  22xx 11  151

dUcenH RbB½n§smIkarmanRkumcemøIybIKW 2 y  2 x  2

x  y  z  0 b£ x  y  z  1 b£ x  y  z  1 2 y  2 x  50
y  x 1

 y  x  25
298 KNnaplbUk S ³ 2 y  26  y  13

eK[ a  b  c enaH x  y 1  13 1  12
S
a

b

c -cMeBaH 317  51 eyIgpÞwmktþaRtUvKña ³
a  b a  c  b  c b  a  c  a c  b 

a

b

c eyIg)an 22 yy  22xx 11  17
3

a  b a  c  b  c a  b  a  c b  c  


2 y  2 x  4
ab  c   ba  c   ca  b  

a  b b  c a  c  2 y  2 x  16
ab  ac  ab  bc  ac  bc y  x  2
 
a  b b  c a  c  y  x  8
0 2 y  10  y5

a  b b  c a  c  enaH x  y 2  52  3
0

dUcenH témøplbUkKNna)anKW S 0 . dUcenH KUcemøIyéncMnnY Kt;viC¢man manBIrKUKW ³


x  12 , y  13  b¤ x  3 , y  5 .
299 rkRKb;bNþaKUéncMnYnKt;viC¢man x , y 
eyIgman x  x  13  y
2 2 300 etIcMnYn A bBa©b;edayelxsUnü ¬0 ¦ b:unµan ?
naM[ y  x  x  13 ¬KuNGgÁTaMgBIrnwg 4 ¦
2 2 eyIgman A  2  5 n 2 n 1

4 y 2  4 x 2  4 x  52  2 n  5 n  n 1
4 y 2  4 x 2  4 x  1  52  1  2 n  5 n  5 n 1

4 y 2  4 x 2  4 x  1  51   10 n  5 n 1
2 y 2  2 x  12  51 eday -sV½yKuNén 5 CacMnYnmanelx 5 xagcugCanic©
2 y  2 x  12 y  2 x  1  51 naM[ 5 CacMnYnEdlmanelx 5 xagcugCanic©
n 1

eday x , y  CacMnYnKt;viC¢man -ehIy 10 manelx 0 cMnYn n dg


n

naM[ 2 y  2x  1  2 y  2x  1 naM[plKuN 10  5 manelx 0 cMnYn n dg


n n1

ehIyplKuNBIrktþaéncMnYnKt;esµInwg 51 KW ³
1 51
51  
dUcenH cMnYn A  2 n
 52 n1 bBa©b;edayelx 0
3 17 cMnYn n dg .
 755 
301 edaHRsaysmIkar 301 rkRkLaépÞTaMgGs;rbs;sUlIt ³
xa xb xc
 
1 1 1
 2    eyIgmanKUb 7 pÁúMP¢ab;Kña)anCa
bc ac ab a b c
ax  a   bx  b   cx  c   bc  ac  ab  sUlItmanmaD 448 cm 3

 2  3
abc  abc  naM[KUbnImYy²manmaD V  4487cm  64 cm 3

ax  a   bx  b   cx  c   2ab  bc  ac 


ax  a 2  bx  b 2  cx  c 2  2ab  2bc  2ac Taj)anRTnugrbs;KUb a  V  64 cm  4 cm
3 3 3

a  b  c x  a 2  b 2  c 2  2ab  2bc  2ac KUbnImYy²manépÞ 6a 2

a  b  c x  a  b  c 2
-Kitfa ³ KUb7 manKUbmYyenAkNþaleK )at;épÞGs;
x
a  b  c 2
a  b  c  nigKUb6 eTot)at;épÞGs;1dUcKña naM[épÞsUlIt ³
x  a  b  c    
S  7  6a 2  6a 2  6  a 2 
 42a 2  12a 2
dUcenH smIkarman x  a bc Cab£s .  30a 2 , a  4 cm
 30  4 cm  30  16 cm 2  480 cm 2
2

302 rkb£sKUbén Z ³
dUcenH épÞRkLaTaMgGs;rbs;sUlItKW S  480 cm . 2

eyIgman Z   8 , 4  manTRmg; Z  r ,  


Edl r Cam:UDul nig  CaGaKuym:g; 304 bgðajfa 2AM  AB  AC
b£sKUbén Z KW -eyIgman AM  Caemdüan A

   2k 
Z  r ,
k
3
 Edl k  0 , 1 , 2 naM[ cMNuc M kNþal BC N
 3 
-cMeBaH k  0 ³ Z 3
0  8 ,
 / 4  2  0   

-ykcMNuc N kNþal AB B M
C

 3 
   naM[ AN  AB 2
1
 2 , 
 12 
 / 4  2 1   
-kñúgRtIekaN ABC man MN Ca)atmFüm

-cMeBaH k  1 ³ Z1   3 8 ,
 3

 naM[ MN  AC 2
2
  2   3 
 2 ,
 12 3  
   2 ,
4 
 -edaybUk 1  2 eyIg)an ³
-cMeBaH k  2 ³ 
Z2  3 8 ,
 / 4  2  2    AN  MN 
AB AC

 2 2
 3 

 2 ,
 4   17 
   2 , 
-Et kñúgRtIekaN AMN tamvismPaBénRtIekaN
 12 3   12 
eyIg)an AM  AN  MN enaHeyIgTaj)an ³
dUcenH b£sKUbén Z KNna)anKW ³ AB AC
AM    2 AM  AB  AC
2 2
    3   17 
Z 0   2 ,  , Z1   2 ,
 12  
 , Z1   2 ,
4  

12 
. dUcenH eyIgeXIjfa 2 AM  AB  AC .
 756 
305 KNnael,Ón ry³eBl nigcm¶aycr 307 rkcMnYnénkarcab;édKñaTaMgGs;
rMlwkrUbmnþ cm¶aycrKW ³ d  v  t karcab;édBMumanRcMEdleLIy enaHeyIgKiteXIjfa ³
tambRmab;RbFan ³ GñkTI1 cab;)an 9 dg / GñkTI2 cab;)an 8 dg
-ebIbEnßmel,Ón 3 km/ h eTAdl;muneBlkMNt; 1 h GñkTI3 cab;)an 7 dg / GñkTI4 cab;)an 6 dg >>>
naM[)an ³ v  3t  1  d 1 rhUtdl;GñkTI10 )an 0 dg
-ebIbnßyel,Ón 2 km/ h eTAdl;eRkayeBlkMNt; 1 h naM[ cMnYndgénkarcab;édKñaKW ³
naM[)an ³ v  2t  1  d 2 9  8  7  6  5  4  3  2  1  0  45 dg

tam 1 & 2 eyIg)anRbB½n§smIkar ³ -GñkGaceFVItamrebobmü:ageTotKW ³


v  3t  1  d  vt  v  3t  3  d mnusSman 10nak; ehIycab;édKñaBIr²nak; mann½yfa³
  
v  2t  1  d vt  v  2t  2  d
 v  3t  3  d  vt  v  3t  3  0
vaCabnSM én10Fatu eRCIserIsyk2Fatu


 v  2t  2  d  vt

 v  2t  2  0 naM[ C 10 , 2  10 10 2!!2!  108!92!8 !  45 dg
eRBaH d  vt
  v  3t  3  0

dUcenH cMnYndgénkarcab;édTaMgGs;KW 45 .
bUkGgÁnigGgÁ ³  v  2t  2  0
t 5  0  t  5 h
cMeBaH v  2t  2  0 308 RsaybBa¢ak;fa 2  3 Eckdab;nwg 9 9
35

v  2  5  2  0 eyIgman 2  3  2   3 
9 9 3 3 3 3

v  12  0  v  12 km / h   
 2 3  33 2 6  2 3  33  3 6
naM[ d  v  t  12  5  60 km  8  27 2  2  3  3 
6 3 3 6

 35 2  2  3  3 
6 3 3 6

dUcenH eyIgKNna)an el,Ón v  12km/ h eXIjfalT§plén 2  3 CaBhuKuNén 35 9 9

ry³eBl t  5h cm¶aycr d  60km . enaH 2  3 RtUvEtEckdac;nwg 35 9 9

306 KNnamYycMnYnenaH dUcenH cMnYn 2 9


 39 Eckdac;nwg 35 .
tag x CamYycMnYnEdlRtUvrkenaH
tambRmab;RbFan eyIg)an ³ x  x  x  x 309 sRmYlRbPaKsniTan E ³
eyIgman E  aa  35ab  2b
2 2

b£ x  2x naM[ x  2x  0
2 2
2
ab  6b 2

x  0 a  b a  2b   a  b
xx  2  0 Taj)an  
x2  a  2b a  3b  a  3b

dUcenH cMnYnEdlRtUvKNnaKW 0 b£ 2 . dUcenH eyIgsRmYl)an E  aa3bb .

 757 
310 KNnargVas;RCugnImYy² 312 KNnargVas;RCugénRbelLÚRkam ABCD ³
tag x , y , z CargVas;RCugTaMgbIénRtIekaN KitCa cm tag h  5 RtUvnwg)at AB
AB
D C
h AB  5
hAD  7
tambRmab;RbFan eyIg)an 3x  4y  5z h  7 RtUvnwg)at AD A
AD B

tamlkçN³smamaRt eyIg)anpleFob eyIg)an épÞRkLaRbelLÚRlam ABCD KW


x y z x  y  z 24
    2
S  AB  h
ABCD b¤ S  AD  h
AB ABCD AD

3 4 5 3  4  5 12
eRBaH brimaRt x  y  z  24 cm naM [ AB  h  AD  h Taj)an
AB
h
h

AD
AD
AB
AB

AD

eyIg)an x
2  x6 cm tambRmab; smamaRtkm<s; 5 : 7 KW h AB 5

h AD 7
3
y
2  y 8 cm naM[ AD
AB 7

5
 5 AB  7 AD
4
z
2  z  10 cm  5 AB  7 AD  0 1
5
bRmab;brimaRt 2 AB  AD   48 cm
dUcenH rgVas;RCugénRtIekaNKW 6cm , 8cm , 10cm . AB  AD  24 2
tam 1 & 2 eyIg)anRbB½n§smIkar ³
311 KNnacMnYnKt;enaH 5 AB  7 AD  0
 bUkbM)at;edayyksmIkar 2 7
 AB  AD  24
tag ab CacMnYnKK;EdlmanelxBIrxÞg;enaH  5 AB  7 AD  0

naM[eyIg)an ³ 0  a  9 , 0  b  9 eyIg)an 7 AB  7 AD  168
12 AB  168  AB  14
tambRmab;RbFan eyIgcg)anRbB½n§smIkar ³ 5 AB 5  14
 ab 9 eday 5 AB  7 AD  AD    10
  7 7
ba  ab  63 dUcenH rgVas;RCugRbelLÚRkameRkayBIKNna)an
Binitü ba  ab  10 b  a   10 a  b 
KW ³ AB  14 cm , AD  10 cm .
 10b  a  10a  b
 9a  9b
 9 a  b  313 rktémø x edIm,I[ P mantémøtUcbMput
enaHeyIg)an RbB½n§smIkar  eTACa ³ eyIgman P  x 1x  2x  3x  6
 ab 9  ab 9
    x  1x  6 x  2 x  3
9 a  b   63  a  b  7
  
 x 2  5x  6 x 2  5x  6
 ab 9
  x  5 x   6
2 2 2

tamviFIbUkbM)at;  a  b  7
 x  5 x   36
2
 b8
2b  16
2

Taj)an a  9  b  a  9  8  a  1 eday x  5x  0 naM[ x  5x  36  36


2 2 2 2

dUcenH cMnYnKt;enaHKW ab  18 . enaHtémø P  x  5x  36  P  36


2 2

 758 
mann½yfa P mantémøtUcbMputesµI  36 315 bgðajfaplKuNénBIrcMnYnenaHtUcCag b¤esµI 14
cMeBaH x  5x  0  xx  5  0
2

naM[ x  0 , x  5 tag a nig b CaBIrcMnYnenaH


tambRmab; a  b  1 ¬elIkGgÁTaMgBIrCakaer¦
dUcenH x  0 , x  5CatémøEdleFV[I eyIg)an a  b  1 2 2

P  36 CatémøtUcbMput . a 2  2ab  b 2  1


2ab  1  a 2  b 2   1
314 rkcMnYnmnusS nigcMnYnekAGI mü:ageTot ebI a nig b CaBIrcMnYnenaHeyIg)an ³
-rebobTI1 ³ ¬tamRbB½n§smIkar ¦ a  b2  0
tag x CacMnYnmnusS nig y CacNYnekAGI a 2  2ab  b 2  0

Edl x nig y CacMnYnKt;  


2ab  a 2  b 2  2
 x  y  4
edaybUkGgÁngi GgÁén 1 nig 2 eyIg)an ³
tambRmab;RbFan eyIg)anRbB½n§smIkar y  4  x 2ab  1  a 2  b 2

 
 2
 2ab  a  b
2

2

 x y 4  x  4  y 1 1
Taj[gay    4ab  1  ab 
 x  2 y  8 2
4
 x  2 y  8
edaypÞwm 1 & 2 enaHeyIg)an ³ dUcenH plKuNénBIrcMnYnenaHtUcCag b¤esµI 14 .
2y 8  4  y
2y  y  4  8
y  12 316 rképÞkaerEdlcarwkkñúgrgVg;TaMgmUl r
tam 1 : x  4  y  x  4  12  x  16 tag s nig a CaépÞ nigRCug énkaertUc 
a

dUcenH mnusSmancMnYn 16 nak; /ekAGImancMnYn 12 . S nig b CaépÞ nigRCug énkaerFM


2

tambRmab; ³ s  100 cm r 2

-rebobTI2 ³ ¬tamsmIkar¦  b

naM[ a  s  100 enaH a  10 cm


2

tag x CacMnYnmnusS enaHcMnYnekAGIKW x  4


tamRTwsþIbTBIrtaK½r r CaRbEvgGIub:Uetnus ¬kaMrgVg;¦
Edl x CacMnYnKt; 2 2
a  10 
tambRmab;RbFaneyIg)ansmIkar ³ enaH r     a     10  125 cm
2
2

2
2 2 2

x
2
 x  4  4 nig 2r   b  b  4r  2b  b  2r
2 2 2 2 2 2 2

x  2 x  16 eyIg)an épÞRkLakaerFMKW S  b 2

x  16
naM[ S  2r  S  2 125  S  225 cm
2 2

naM[ cMnYnekAGIKW x  4  16  4  12
dUcenH épÞkaercarwkkñúgrgVg;TaMgmUlKW S  225 cm . 2

dUcenH mnusSmancMnYn 16 nak; /ekAGImancMnYn 12 .


 759 
317 kMNt;témøéncMnYnBit a nig b ³ x 2  y 2  z 2  52 x 2  y 2  z 2

eyIgman a  b  0 2 2
81 29
RKb;témøéncMnYnBit a nig b enaHeyIg)an ³ 
2 2 2
 2

29 x  y  z  52  81 x  y 2  z 2 
29x  y  z   29  52  81x  y  z 
2 2 2 2 2 2

a 0 , b 0 81x  y  z   29x  y  z   29  52
2 2
2 2 2 2 2 2

naM[ a  b  0
2 2
52x  y  z   29  52
2 2 2

cMeBaH a  b  0 mankrNIEtmYyKt;EdlepÞógpÞat;
2 2 x  y  z  29 2

2
2 2

2 2

KW ba  00 ¬KµankrNINaepSgeToteLIy¦ enaHeyIg)an ³ x4  y9  16z  29


29
1

naM[eyIgTaj)an ³
dUcenH cMnYnBitkMNt;)anKW a  0 nig b  0 . x
1  x  4  x  2
2
2

4
y2
1  y2  9  y  3
318 edaHRsayRbB½n§smIkar 9
 x y z z2
   1 1  z 2  16  z  4
eyIgman  2 3 4
2
16
 xy  yz  zx  26
dUcenH RbB½n§smIkarmanKUcemøIyRtUvKñaKW ³
tamsmamaRtenaHeyIg)an
x y z x yz x yz
x , y , z  2 , 3,  4 b¤ x , y , z  2 , 3, 4 .
1 :    
2 3 4 23 4 9
x y z x yz
   319 KNnatémøKt; énkenSamelx E ³
2 3 4 9
x 2
y 2
z 2
 x  y  z
2 eyIgman E  2  5  2  5 3 3

  
4 9 16 81 naM[ E   2  5  2  5 
3 3 3
3

x 2
y 2
z 2
x  y 2  z 2  2xy  yz  zx 
2
  
 
2
4 9 16 81  2  5  3 3 2  5   3 2  5  
   
x 2
y 2
z 2
x  y  z  2  26
2 2 2
    
2
 3 3 2  5  3 2  5   2  5
4 9 16 81   
2 2 2
x  y  z 2  52
2 2
 4  33 2  5  3 2  5  3 2  5  3 2  5 
x

y

z
 i   
4 9 16 81
eRBaHtam 2 : xy  yz  zx  26  4  33 2 2  5 2 E
E 3  4  3E
EttamsmamaRt eyIg)an E 3
 3E  4  0
x2
y z2
x y z2 2 2 2

4

9 16
 
4  9  16 E  1E 2  E  4  0
naM[ EE 1E 0 4  0 2

2 2 2
x2  y2  z2
x

y

z
 ii  eRkayBIedaHRsaymanEttémøKt; E  1 b:ueNÑaH
4 9 16 29
tam i  nig ii enaHeyIg)an ³
dUcenH KNnatémøKt;)anKW E  1 .
 760 
320 KNnaplbUkcRmaséncMnYnTaMgBIr 322 KNna A ³
tag a nig b CaBIrcMnYnenaH eyIgman A  2  4  6 ... 2n
 2  1  2  2  2  3  ... 2  n 
naM[plbUkcRmascMnYnTaMgBIrKW 1a  b1  2  2  2  ... 21 2  3  ... n 
-rebobTI1 ³ eyIgKNna tamtRmUvPaKEbgrYmKW ab  2 n  n!

naM[ 1a  b1  aab b eRBaH 2


22  2  2 n
...
n dgénelx 2
edaybRmab; plbUk a  b  12 nigplKuN ab  4 nig n! 1 2  3  ... n
eK)an 1a  b1  aab b  124  3 enaH 1a  b1  3
dUcenH témøEdlKNna)an A  2 n
 n! .
-rebobTI2 ³ edayEckGgÁ nigGgÁ
eyIgman a  b  12
323 KNna 1024
S 1 ³
edayEckGgÁTaMgBIrnwg ab eyIg)an ³
a  b 12
eyIgman
 3 eRBaH ab  4
1 1 12
ab

ab
  
a b 4      
S  1  2 1  2 2 1  2 4 1  28  ... 1  21024
 2  11  21  2 1  2  ... 1  2 
2 4 1024

dUcenH plbUkcRmascMnYnTaMgBIrKW 1 1
 3
a b
.  2  11  2 1  2 1  2  ... 1  2 
2 2 4 8 1024

 2  11  2 1  2  ... 1  2 


4 4 8 1024

 2  11  2  ... 1  2 
8 8 1024

321 KNnark r nig s ³  ...


ebI x  3x  8 CaktþamYyénkenSam x
2 4
 rx 2  s enaH  
 21024  1 1  21024 
 
x 4  rx 2  s  x 2  3x  8 x 2  ax  b   2 2048  1
 x 4  ax 3  bx 2  3x 3  3ax 2  3bx  naM[ 1024
S  1  1024 22048  1  1
 8 x 2  8ax  8b  1024 2 2048
 x 4  a  3x 3  b  3a  8x 2 
 1024 2 2   1024
 22  4
 3b  8a x  8b
edaypÞwmelxemKuNRtUvKñaén x enaHeK)an ³ dUcenH KNna)an 1024
S 1  4 .
a  3  0  a  3 a  3
r  b  3a  8

r  b98

 r7
 324 KNna A ³
    
 3b  8a  0 3b  24  0  b8 man A  1234568 1234567
2012
12345679
 s  8b  s  8b  s  64
2

2012
eK)an  
x  7 x  64  x  3x  8 x  3x  8
4 2 2 2
 
1234568  1234568  11234568  1
2

2012 2012
   2012
dUcenH témøKNna)anKW r 7 nig s  64 . 1234568  1234568  1
2 2
1 
dUcenH KNna)an A  2012 .
 761 
325 rkcMnYnKt;viC¢man n ³ 327 KNnarkelxxagcugén A ³
eyIgman 4  n  n  4n  4  4n
4 4 2
man A  2012  2010  2
2 2013 2013

 n  2  2n  eday 2010 manelxxagcug 0 enaHelxxagcugén


2 2 2

 n  2  2n n  2  2n 
2010  2 GaRs½yelIelxxagcugén 2
2 2
2013 2013

 n  2n  2n  2n  2
2 2

eday n  enaHRtUvEt n  2n  2  n  2n  2 -Binitü lkçN³TUeTAcMeBaHsV½yKuNén 2 KW ³


2 2

ebI 4  n CacMnYnbzm enaHvaCaplKuNBIrktþa Edl cMeBaH n  1 , 2 , 3, 4 , 5 , 6 , 7 , 8 , ... enaHelxcug


4

ktþaTI1esµInwg 1 nigktþaTI2esµInwgxøÜnÉg eyIg)an ³ én 2 KW 2 , 4 , 8, 6 , 2 , 4 , 8, 6 , ... manTRmg;TUeTA


n

n  2n  2  1
2
n  2n  1  0 KW ³ 2 manelxxagcug 2 ebI n  4k  1 , k 
2
n

  
n  2n  2  4  n n  n  2n  2  0 2 manelxxagcug 4 ebI n  4k  2 , k 
2 4 4 2 n

n  1  0 n  1  0 2 manelxxagcug 8 ebI n  4k  3 , k 


2
n
  
n n  1  2n  1  0
2 2
 n  1n  n  2   0 3 2

2 manelxxagcug 6 ebI n  4k n
, k 
n  1 n  1
 

n  1 n  2n  2  0
2 2
n  2 n  2  0 eday 2013  4  503 1 manTRmg; n  4k 1
2

cMeBaH n  2n  2  0 man ³
2
naM[ 2  2 manelxxagcugKW 2 2013 45031

  1  2  1  0 naM[smIkarKµanb£s
dUcenH elxxagcugén A KW 2 .
dUcenH cMnYnKt;viC¢manEdlrk)anKW n  1 .
328 rkRKb;cMnYnKt;viC¢man n ³
326 rkelxxagcugénplKuN ³ cMeBaH n  1 , 2 , 3, 4 , 5 , 6 , 7 , 8 , ... enaHeyIg)an
2 KW 2 , 4 , 8 , 16 , 32 , 64 , 128, 256 , ... ehIy
n

eyIgman 7  2013  7  2013


2012 2010 2000 2012

 7  7  2013  2013
2012 2000 sMNl;Eckén 2 nwg 7 KW 2 , 4 , 1, 2 , 4 , 1 , 2, 4 , 1 ,...
2010 2002
n

7 2012 2000
 2013 EdlsMNl;énplEckenHmanTRmg;TUeTAKW ³
2010 2002

 7  20134012 4012
-ebI n  3k  1 , k  enaH 2  7 mansMNl; 2 n

 7  2013
4012

 14091 4012
-ebI n  3k  2 , k  enaH 2  7 mansMNl; 4 n

 ...1 -ebI n  3k , k  enaH 2  7 mansMNl; 1 n

sV½yKuNénmYycMnYnEdlmanelx1xagcug esµInwgmYy cMeBaHkrNI 2  7 mansMNl; 1 Edl n  3k n

cMnYnEdlmanelx1 enAxagcugCanic©. naM[ 2 1  7 mansMNl; 0 Edl n

mann½yfa 2 1 Eckdac;nwg 7 kñúgkrNI n  3k n

dUcenH elxxagcugénplKuNKW 1 .
dUcenH 2 1 Eckdac;nwg 7 kalNa n

n CaBhuKuNén 3 .
 762 
329 bgðajfa 12  2 Eckdac;nwg 10 ³ 2012 2008
ehIy man n 1nn  1 CabIcMnYn
5n  1nn  1

edIm,I[ 12  2 Eckdac;nwg 10 eyIgRKan;Etbgðaj


2012 2008
Kt;tKñaenaH 5n 1nn  1 RtUvEtEckdac;nwg
faplkdén 12  2 manelxsUnüenAxagcug .
2012 2008
2 , 3 nig 5 b¤cMnYnenHRtUvEtEckdac;nwg 6 nig 5

elxcugén12  2 GaRs½yelIelxcugén 2  2
2012 2008 2012 2008
eday PGCD 6 , 5  1 enaHcMnYnEdlEckdac;nwg
-sikSasV½yKuNén 2 KW ³ 6 nig 5 CaCMnYnEdlEckdac;nwg 30

cMeBaH n  1 , 2 , 3, 4 , 5 , 6 , 7 , 8 , ... enaHelxcug


dUcenH cMnYn A  n 5
n Eckdac;nwg 30 .
én 2 KW 2 , 4 , 8, 6 , 2 , 4 , 8, 6 , ... manTRmg;TUeTA
n

KW ³ 2 manelxxagcug 2 ebI n  4k  1 , k 
n

331 KNna f x ³


2 manelxxagcug 4 ebI n  4k  2 , k 
n

eyIgman f x  x  3x 3

2 manelxxagcug 8 ebI n  4k  3 , k 
n

cMeBaH x  3  2 
3 3
3 2 naM[
2 manelxxagcug 6 ebI n  4k
n
, k  3
x 3   3 3 2 3 3  2 
eday 2  2 manTRmg; 2
2012 4503 4k  
  3 2    3  2  33   3 2  
3 2 x
naM[ 2 CacMnYnmanelx 6 enAxagcug
2012

 2 3  33 32  2 2  x
nig 2  2 manTRmg; 2
2008 4502 4k
 2 3  3x
naM[ 2 CacMnYnmanelx 6 enAxagcugEdr
2008
Taj)an x 3  3x  2 3 enaH f x   x 3  3x  2 3

eyIg)anpldkén 12  2 manelx 0 enAxagcug 2012 2008

dUcenH KNna)an f x   2 3 .
dUcenH cMnYn 12 2012
 2 2008 Eckdac;nwg 10 .
332 rkBIrcMnYnKt; a nig b ³
330 bgðajfa A Eckdac;nwg 30 ³
bRmab; a  b  92 naM[ a 1 b  93
cMeBaH RKb;cMnYnKt;viC¢man n eyIg)an ³
A  n5  n
eday a 1 CaBhuKuNén b enaH a  1  kb , k 

 n n4 1  eyIg)an kb  b  93  k  1b  93
 nn 2
 1n 2
1  naM[ b  k93 1 eday b CacMnYnKt;viC¢man

 nn  1n  1 n 2  4  5  
 nn  1n  1n 2
 4   5nn  1n  1 enaH 93 RtUvEtEckdac;nwg k 1
 nn  1n  1n  2 n  2   5nn  1n  1 1 0
93 92
 n  2 n  1nn  1n  2   5n  1nn  1 eday 93  1393 enaH k 1  
3
 k 
2

31  
eday n  2n 1nn  1n  2 CaR)aMcMnYntKña 31 30
enaHcMnYnenH RtUvEckdak;nwg 2 , 3 nig 5 -cMeBaH k 0 enaH b  93 minyk eRBaH a  b  92
b¤cMnYnenHRtUvEtEckdac;nwg 6 nig 5 -cMeBaH k  92 enaH b  1

 763 
-cMeBaH k 2 enaH b  31 335 KNnakenSam A ³
-cMeBaH k  30 enaH b  3 eyIgman a  a  1 nig
2

1  92  1  91 A  a 4  2a 3  4a 2  3a  3
 
eyIg)an b  31 naM[ a  92  b  92  31  61  a 4  2a 3  a 2  3a 2  3a  3
3

 92  3  89
 
 a 4  2a 3  a 2  3a 2  3a  3   
 a   
2
2
 a  3 a2  a  3
dUcenH cMnYnKt;viC¢manEdlrkeXIj a  91 , b  1   1  3 1  3
2

a  61 , b  31 nig a  89 , b  3 .  1 3  3
1
333 bgðajfa P Eckdac;nwg P ³ dUcenH kenSam A  1 RtUv)anKNna .
man P  n  1n  2n  3 ...  n  n
ehIy P  1 3  5  ...  2n  1 enaHeyIg)an ³ 336 KNna A [lT§plCaplKuNbIktþa ³
n!P  n!n  1n  2n  3  ... n  n 
 1  2  ... nn  1n  2n  3  ... n  n 
eyIgman A  12  2823  6
 1  2  ... nn  1n  2n  3  ... 2n  2 2

 1  3  5  ... 2n  12  4  6  ... 2n   12  8   3  6 
 P   2  12  22  3  ... 2  n  2 2

 P   1  2  ... n 2  2  2  ... 2 
4 3 2  3 3 2   
n!P  P   n!2 n 2 2

P  2 n  P  3 2  4 3 
tamlT§plbgðajfa P Eckdac;nwg P )anplEck 2 
 2 2  3 2  4 3 
    
n
3 2 3 2 4 3 4 3
 2 2  3 2 4 3  
dUcenH P Eckdac;nwg )anplEck 2
P n
. 
3  24  3
  2 2  3 2  4 3 
334 KNna a  b ³
eyIgman 2a3  326  5b9 Edl 0  a  9 ,0  b  9 dUcenH KNna)an A   2 2  3  2 2 3 .
bRmab;RbFan ³ 5b9 Eckdac;ngw 9
naM[ 5  b  9 Eckdac;nwg 9 enaHmanEt b  4 337 KNnatémøelxénkenSam E ³
eyIgman S  23  64  96  ......  300
2 2 2 2
200
eRBaH 5  4  9  18 Eckdac;nwg 9 2 2 2 2

eyIg)an 2a3  326  549 



2 2 1  2 2  32  ... 1002 

32 1  2 2  32  ... 1002 
2a3  223 Taj)an a  2 22 4
 
naM[ a  b  2  4  6 32 9

dUcenH eyIgKNna)an a  b  6 . dUcenH témøelxKNna)an E  94 .


 764 
338 rkrgVas;RCugmMuEkgTaMgBIr ³ 341 edaHRsaysmIkar ³
tag x nig y CargVas;RCugmMuEkg x 13 cm
x  4 x  3 x  2 x 1
 
2008 2009 2010 2011
 0

x  2012  2008 x  2012  2009 x  2012  2010 x  2012  2011


Edl x  0 , y  0 KitCa cm 2008

2009

2010

2011
0
y x  2012 x  2012 x  2012 x  2012
tambRmab;eyIg)an ³ 2008
1
2009
1
2010
1
2011
1  0

x  2012 x  2012 x  2012 x  2012


 x 2  y 2  132  x 2  y 2  169    0
  
2008 2009 2010 2011
 x  y  17  x  y  17 x  2012 1  1  1  1   0
 2008 2009 2010 2011 
cMeBaH x  y  17 elIkGgÁTaMgTaMgBIrCakaer naM[ x  2012  0  x  2012
x 2  2 xy  y 2  289

2 xy  289  x 2  y 2  ehIy 2008
1

1

1

1
2009 2010 2011
0

289  169
xy 
2
 60 dUcenH x  2012 Cab£sénsmIkar .
 x  y  17  5  12
eday  342 rkbIcMnYnKt;viC¢manxusKña x , y nig z ³
 xy  60  5 12
enaH x  5, y  12 b¤ x  12, y  5 eyIgman  yx zy106

dUcenH RCugmMuEkgTaMgBIr 5cm nig 12cm . eyIg[témøén x edIm,IKNnatémørbs; y nig z ³
eday x  y  6 naM[ 0  x  6 enaH ³
339 RsaybBa¢ak;fa 3  2 ³ 2 3 ebI x  1 eyIg)an y  5 nig z  5 ¬minyk¦
Binitü 9  8  3  2  3   2 
2 3 2
2
3
3
 x  2 eyIg)an y  4 nig z  6 ¬yk¦

nig 2 2 9
 2   2 
6 3
3
3
2
  x  3 eyIg)an y  3 nig z  7 ¬minyk¦

tamTMnak;TMng  nig   eyIg)an ³ x  4 eyIg)an y  2 nig z  8 ¬yk¦

3   2  Taj)an 3  2
2
2
3
2
2 3 x  5 eyIg)an y  1 nig z  9 ¬yk¦

dUcenH 3  2 RtUv)anRsaybBa¢ak; .
2 3 dUcenH bIcMnYnKt;viC¢manxusKñaEdlepÞógpÞat;KW ³
 x  2 , y  4 , z  6  ,  x  4 , y  2 , z  8
 x  5 , y  1, z  9 
.
340 bgðajfa A6 ³
A  6  6  6  ...  6  3 24  3 24  3 24  ...  3 24
343 edaHRsaysmIkar ³
 x  1  x  1 elIkGgÁTaMgBIrCakaer
 6  6  6  ...  6  6  6  6  ...  9  3
 x  1  x 2  2x  1
eday  3 24  3 24  ...  3 24  3 24  3 24  ...  3 27  3

x 2  3x  0
6  6  6  ...  6  24  24  ...  3 24  6
minyk eRBaHminepÞógpÞat;
3 3

A6
xx  3  0  x  0

x  3 yk
dUcenH A6 RtUv)anRsaybBa¢ak; . dUcenH smIkarman x  3 Cab£s .
 765 
344 KNnamMu x ³ 347 k> KNnargVas;RCugénRtIekaN
30 o d  eyIgman ³ 24a  18b  12c nig b  c  10 cm
o
x 30

o
l  tamlkçN³smamaRteyIg)an ³
110 b c bc 10 1
   
18 12 18  12 30 3
bnøayRCugénmMu x [kat;bnÞat; l 
naM[ a b
 
c 1

enaHeyIg)anRtIekaNmYy enAkñúgbnÞat;Rsb Edl;man 24 18 12 3
a 1
mMu x CamMueRkAénRtIekaNenH manrgVas;esµpI lbUkmMu  24  3
3a  24 a  8

kñúgBIrEdlminCab;nwgva . Taj)an b 1
 
 
 3b  18  b  6
18 3 3c  12 c  4
eyIg)an ³ x  30  180  110   100 .
o o o o
c 1

 
12 3

dUcenH mMu x  100 RtUv)anKNnabgðaj .
o

dUcenH rgVas;RCugénRtIekaNEdlKNna)anKW ³
a  8 cm , b  6 cm , c  4am
345 KNnaépÞRkLaénrUb ³
rMlkw RtIekaNsm½gSEdlmanRCug a enaHkm<s;rbs;vaKW ³ h  3
a
2 x> KNnargVas;km<s;EdlRtUvnwgRCug a ³
RkLaépÞrUbenH )anBIépÞRtIekaNFMmYynig D
A
F eyIgKUsrUb)antamlT§pl ³
RtIekaNtUcbI EdlsuT§EtRtIekaNsm½gS. B tamRTwsþIbTBItaK½r
C

ebI S CaépÞRtIekaNFM ¬ Big =FM¦


B
E
eyIg)an ³ c  4cm
b  6cm
h
nig S CaépÞRtIekaNtUc ¬ Small =tUc¦
S  x 2  h2  42 x
 a  8cm
naM[ S  S  3S B S 8  x   h  6
2 2 2

1 3  1 3   x 2  h 2  16 1
    3  3   3  1 1 
2 2  2 2  64  16 x  x  h  36
2 2
2

9 3 3 3

yk 1 CMnYskñúg 2 eyIg)an ³
4 4 64  16 x  16  36
 3 3 cm 2
 16 x  80  36
 44
dUcenH RkLaépÞénrUbKW S  3 3 cm 2 . x
 16
 x
11
4

346 bMeBjelxkñúgRbGb; ³ cMeBaH x  114 enaH h 2  16  x 2

eyIgGacbMeBj)antambRmab;RbFan dUcxageRkam ³ naM[  11 


h  16    
2
256  121

135 3 15

15 4 16 4 4
2 7 6  15
1  15
9
4
5
3 8  15 dUcenH km<s;énRtIekaNKW h
3 15
4
cm .
15 15 15 15

 766 
abc
348 rkcMnYnsisSenAkñúgfñak;eronenaH ehIy 3
 2012
tag x CacMnYnsisS enAkñgú fñak;eronenaH ¬KitCa nak;¦ enaH a  b  c  3 2012
tambRmab;RbFan eyIgsresr)ansmIkar ³ 4024  c  6036
x4 x2 c  6036  4024

8 9 c  2012
9 x  36  8 x  16
9 x  8 x  16  36 dUcenH cMnYnTI3 enaHKW 2012 .
x  52

dUcenH cMnYnsisSenAkñgú fñak;KW 52 nak; . 352 KNnacMnYnBIrxÞg; x ³


bRmab; ³ 44 Eckdac;nwg x sl;sMNl; 10
349 rkmYycMnYnenaH ebI k CaplEckrvag 44 nig x Edl k 
tag A CacMnYnenaH eyIg)an ³ 44  kx  10  kx  34
tambRmab;eyIg)an ³  AA  44  A4 eday 34  2 17 enaH kx  217

Et x CaelxBIrxÞg; enaHRtUvEt x  17 , k  2
dUcenH cMnYnEdlRtUvrkenaHKW 4 .
dUcenH cMnYnBIrxÞg;én x KW x  17 .
350 rkmYycMnYnenaH
353 edaHRsaysmIkar HE  SHE ³ 2

tag x CacMnYnEdlRtUvrkenaH
BinitüsmIkar ³ HE  SHE 2

tambRmab;RbFan eyIg)an ³
x x  x  x kaerénmYycMnYnenArkSaelxxagcugenAEt E dEdl
x 2  2x naM[ E  0 ,1, 5 , 6
x 2  2x  0
x x  2  0
-ebI E  0 enaH HE RtUvmanelxsUnüBIrenAxagcug
2

naM[  x0
enaH x  0 Et HE  SHE manelxxagcugmindUcKña enaH E  0
2

 
x  2  0 x  2 -eyIgepÞógpÞat;cMeBaH E  1, 5, 6 tamtémø H Edl
dUcenH cMnYnEdlRtUvrkenaHKW x0 , x2 . lT§plrbs; HE RtwmEtCaelxbIxÞg;b:ueNÑaH eyIg)an³
2

21  441 minepÞógpÞat; / 31  961 minepÞógpÞat;


2 2

351 rkcMnYnTIbIenaH 15  225 minepÞógpÞat; / 25  625 epÞógpÞat;


2 2

tag a , b , c CacMnYnTI1 / TI2 / TI3 erogKña 16  256 minepÞógpÞat; / 26  676 minepÞógpÞat;


2 2

tambRmab;RbFan ³ eXIjfa HE  SHE epÞógpÞat;eday 25  625


2 2

ab
2
 2012  a  b  4024 dUcenH E  5 , H  2 , S  6 .
 767 
354 kMNt;témø a nig b nigkMNt;eTVFaenaH ³ 356 kMNt;témø m ³
eyIgman ax  bx  54x  27 CaKUbéneTVFa
3 2
eyIgman 23xx  57yy  20m ¬edaHRsaytamedETmINg;¦

tag x    CaeTVFaEdlRtUvkMNt; 3 7
D  15  14  1
naM[ x      x  3 x  3 x  
3 3 3 2 2 2 3 2 5
m 7
pÞwmelxemKuN ³  axx  3bxx  354
 x  
3 3 2 2 2 3
Dx   5m  140
20 5

3
x  27
2

5m  140
  a
3
 3  a 8  a  x  5m  140
 2  2 36  b 1
eyIg)an ³ 3   b 3   b 
     3 m
Dy   60  2m
3   54   2   2
2
2 20
 3  27   3   3
  60  2m
 y  60  2m
1
epÞógpÞat; cMeBaH a  8 , b  36 nigeTVFa 2x  3 edIm,I[RbB½ns§ mIkarmanKUcemøIyviC¢manluHRtaEt ³
x  0 5m  140  0 m  28
eyIg)an 2 x  33  8 x 3  36 x 2  54 x  27 Bit 
y  0

 60  2m  0

m  30
 28  m  30

dUcenH kMNt;)an a  8 , b  36 nigeTVFa 2x  3 . dUcenH kMNt;)anKW 28  m  30 b¤ m  28 , 30  .

355 KNnargVas;RCug AB nig AC ³ 357 KNnakaMrgVg;énrUb ³ x


1 1
eyIgman AB  AC  23 cm C
BC  17 cm
tamrUb eyIg)an ³
x  2r  1 nig
1  r 1 1
¬elIkGgÁTaMgBIrCakaer¦ A B
AB 2  AC 2  2 AB  AC  529 1 tamRTwsþIbTBItaK½r 1 1
tamRTwsþIbTBItaK½r AB  AC  BC  17  289
2 2 2 2
 2  1  r  x  r 
2 2 2

naM[ 1 : 289  2AB  AC  529 2  2 2  1  r 2  x 2  2 xr  r 2


3  2 2  x 2  2 xr
naM[ AB AC  120
 AC  23 3  2 2  x  x  2r  , x  2r  1
eyIg)an  AB
AB  AC  120
¬tamRTwsþIbTEvüt¦ x  3 2 2

eday  AB  AC  8  15
 enaH AB  8 , AC  15 cMeBaH x  3  2 2
 AB  AC  8  15
 AB  AC  15  8
naM[ x  2r  1  3  2 2  2r  1
b¤  enaH AB  15 , AC  8  2r  2 2  2
 AB  AC  15  8
 r  2 1
dUcenH rgVas;RCugénRtIekaNEdlKNna)anKW ³
AB  8cm , AC  15cm b¤ dUcenH kaMrgVg;énrUbKW r  2 1 ÉktaRbEvg .
AB  15cm , AC  8cm .
 768 
358 KNnacMnYnKt;FmµCati n ³ 360 edaHRsayRbB½n§smIkar ³
bRmab; n 13 nig n  76 CakaerR)akd man 12xx  y 1 , 12yy  z 2 , 12zz
2

2
2

2
2

2
 x 3

naM[ nn  13 0
76  0
 n  76
eyIgdwgfa 1  x   0 2

tag n  13  a 1 nig n  76  b 2
2 2
enaH 1  2x  x  0 2

Edl a nig b CacMnYnKt; ehIy a  b  1  x 2  2x


2x 2
 x 4
2x
edayyk 1  2 eyIg)an ³ 
1 x2
1 
1 x2
 n  13  a 2
 tamlMnaMdUcKñaenH eyIgk¾)an ³
n  76  b
2
2 2
 y 5 nig  z 6
2y 2z
89  a 2  b 2
1 y 2
1 z 2

Gacsresr ³ a  ba  b  89 eyIgCMnYs³ 1 kñúg 4 / 2 kñúg 5 nig 3 kñúg 6
mann½yfa 89 CaplKuNénBIrcMnYnKt; ³ y  x

edaydwgfa 1 89  89 ¬eRBaH 89 CacMnnY bzm¦ eyIg)an z  y  x  y  z
x  z

eyIgpÞwm)an aa  bb  189 ¬edaybUkGgÁnigGgÁ¦
tam 1 ³ 12xx
2
 2x 2
y  x
a  b  1
2
1 x2
 2x 2  x  x 3
eyIg)an ³ a  b  89
2a  90  a  45 x  2x  x  0 3 2

xx  2 x  1  0
naM[ n  13  45 enaH n 13  2025 b¤ n  2012
2
2

xx  1  0
2

¬min)ac;sikSa b eRBaHeKalbMNgcg;rktémø n ¦
naM[ x  0 , x  1
dUcenH cMnYnKt;FmµCatiKNna)anKW n  2012 . eyIg)an ³ x  y  z  0 , x  y  z  1
359 KNnatémøénmMu x ³ 5x
dUcenH x  y  z  0 , x  y  z  1 .
tamc,ab;plbUkmMukñúg 3x
a b
7x

énRtIekaN eyIg)an ³ 4x
6x 361 RsaybBa¢ak;plbUkmMukñúgénRtIekaNesµI 180 o



3 x  4 x  a  180
o
KUsbnÞat;kat;kMBUl A ehIyRsbnwgRCug BC

6 x  7 x  b  o
180
20 x  a  b  180o
1 naM[ B  A ¬mMuqøas;kñúg¦ 1 1
A
2

ehIy 5x  a  b  180  a  b  180  5x 2


o o
C  A ¬mMuqøas;kñúg¦ 3
B C
eyIgyk 2 CMnYskñúg 1 enaHeyIgnwg)an ³ Et A  A  A  180 ¬mMurab¦ 1 3
o

20x  180  5x  360 b¤ 15x  180  x  12


o o
enaHeyIg)an A  B  C  180 ¬CaplbUkmMukñúg  ¦
o o o

dUcenH témømMuKNna)anKW x  12 . o
dUcenH plbUkmMukñúgRtIekaNTaMgGs;esµI 180 . o

 769 
362 RsaybBa¢ak;BIRTwsþIbTBItaK½r a  b  c ³
2 2 2
cMeBaH c  9 naM[ 5c  1  2116 minyk
2

-eyIgmanRtIekaNEkgEdlmanrgVas; a c ¬)anCayk 5c  1  1681 eRBaHelx abca


2

RCug a , b , c ehIy c CaGIub:Uetnus b


manTRmg;dUcelx 1681 Edl a  1 , b  6 , c  8
-eyIgpÁúMRtIekaNenH)andUcrUbxageRkam ³ dUcenH cMnYnmanelxbYnxÞg;enaHKW 1681 .
-ctuekaNxageRkA Cakaerman a A b a
rgVas;RCug a  b naM[ ³ b c
c
D 364 bgðajfa n  IN enaHeK)an a  b ³ n n

épÞkaerFM  a  b
2 B
c
c
b Binitü ³ a  b  a  ba  a b  ...  b 
n n n 1 n2 n

a
 a  2ab  b .
2 2
b C a
smµtikmµ ³ a  b  0 naM[ a  b  0 nig
a n1  a n2b  ... b n  0
- ABCD Cakaer eRBaH plbUkmMuRsYcBIrEdlenACab;mMu
naM[plKuN a  ba  a b  ...  b   0
n 1 n2 n

énctuekaNenH CamMubMeBjKñanaM[ ³ épÞkaertUc  c 2

eyIg)an a  b  0 Taj)an a  b
n n n n

-RtIekaNEkgnImYy²man RkLaépÞ  12 ab
dUcenH bgðaj)anfa a  b . n n

-tamrUbEdlpÁúM eyIg)anTMnak;TMngrvagRkLaépÞ ³
épÞkaerFM  épÞkaertUc  épÞRtIekaNtUcTaMgbYn 365 KNnaplKuN P  22  36  124  ...  9900
100
1 
a 2  2ab  b 2  c 2  4 ab 
2 
eyIgKNnaplKuN P )andUcxageRkam ³
a 2  2ab  b 2  c 2  2ab 2 6 12 9900
P    ...
2 3 4 100
a 2  b 2  c 2  2ab  2ab
1 2 23 3 4 99  100
a2  b2  c2     ...
2 3 4 100
dUcenH RTwsþIbTBItaK½rRtUv)anRsaybBa¢ak; . 
1 2

2 3

3 4
 ...
99  100
2 3 4 100
1  2 2  3 2  4 2  ... 99 2  100
363 rkmYycMnYnEdlmanelxbYnxÞg; abca ³ 
2  3  4  ... 100
eyIgman abca  5c  1 enaH a , b , c CacMnYnKt;
2

1  2  3  4  ... 99  10
2  3  4  ... 100
naM[ 0  a  9 , 0  b  9 , 0  c  9 1 10 1
 
ehIy 10000  abca  961 100 10

1002  5c  1  312


2
dUcenH KNna)anplKuN P  101 .
100  5c  1  31
cMeBaH 5c  1  31  5c  30  c  6 2
tam 1 nig 2 eyIg)an 6  c  9  c  7 , 8 , 9 kargarlM)ak KWCasRtUvénkarBüayam >>>
cMeBaH c  7 naM[ 5c  1  1296 minyk
2

cMeBaH c  8 naM[ 5c  1  1681 yk2

 770 
366 dak;kenSam P CaplKuNktþa ³ 368 kMNt; x viC¢man edIm,I[ A CacMnYnKt;viC¢man ³
eyIgman P  x  y    y  z   z  x 
3 3 3
eyIgman A  xx 13
tag a  x  y , b  y  z , c  z  x A
x  3  2  1  2
x 3 x 3
eyIg)an P  a  b  c 3 3 3

-edIm,I[ A CacMnYnKt;viCm¢ an luHRtaEt x 2 3 CacMnYn


eday a bc  x  y y  z z  x
abc  0 Kt;viC¢manEdr naM[ 2 RtUvEckdac;nwg x  3
a  b  c
-edaytYEckviC¢manén 2 KW 1 b¤ 2 enaHeyIg)an ³
a  b 3  c 3 x  3  1 x  1  3 x  4
a 2  3a 2 b  3ab 2  b 3  c 3     
x  3  2 x  2  3 x  5
a 2  b 3  c 3  3a 2 b  3ab 2
a 2  b 3  c 3  3aba  b  dUcenH kMNt;)antémø x  4 , x  5 .
eyIg)an ³ 369 bgðajfaeK)an a  b  c  3abc ³
3 3 3

P  3x  y  y  z x  y  y  z 
P  3x  y  y  z x  z  smµtikmµ ³ a  b  c  0 enaH a  b  c
P  3x  y  y  z z  x  eyIg)an ³
a  b 3  c 3
dUcenH dak;)an P  3x  y  y  z z  x  . a 2  3a 2 b  3ab 2  b 3  c 3
a 2  b 3  c 3  3a 2 b  3ab 2
367 KNnatémøénkenSam P  aa  bb ³ a 2  b 3  c 3  3aba  b 
smµtikmµ 3a  3b  10ab
2 2
a 2  b 3  c 3  3ab c 

eyIgman P  aa  bb Edl a b  0 enaH P  0 a 2  b 3  c 3  3abc

naM[ P 2

a  b
2
a 2  b 2  2ab
 2 dUcenH bgðaj)anfa a 2  b 3  c 3  3abc .
a  b2 a  b 2  2ab

P  223a 2  3b 2  6ab 370 edaHRsaysmIkar x  2012  2012


 x 2012  0
3a  3b 2  6ab
eyIg)an P2 
10ab  6ab 4ab 1
 
eyIg)an ³
10ab  6ab 16ab 4 x  2012 2012  x 2012  0
Taj)an P
1 1
 ¬ P  0¦ eRBaH x  2012 2012  x 2012
4 2
x  2012   x

dUcenH KNna)an P
1
. -cMeBaH x  2012  x smIkarKµanb£s
2
-cMeBaH x  2012   x naM[ x  1006

dUcenH smIkarmanb£s x  1006 .

 771 
371 KNnatémø  
A  x 4  x 3  x 2  2 x 1
2012
cMeBaH b  24 enaH a
720 720
  30
b 24
eyIgman ³ x  1
2
1 dUcenH KNna)anvimaRtrbs;sYnKW 30m nig 24m .

2 1 1 2 1 1

tag X
1

1 373 rkcMnYnTaMgBIr ³
2  1 1 2 1 1
tag x nig x  3 CacMnYnTaMgBIrenaH
2  1  1   2  1  1
   tambRmab;RbFan eyIg)an ³
 2  1  1 2  1  1
x 2  x  3  89
2
  

2

2
 2 x 2  x 2  6 x  9  89
 2 11  2 2 x 2  6 x  80  0
naM[ x
2
X

2
2
 2 x 2  3 x  40  0
x  8x  5  0
eyIg)an
2012
A   2  2  2  2 2  1
4 3 2

  naM[ xx  85  00  xx  58



 4  2 2  2  2 2 1 
2012
 
 12012  1 -cMeBaH x  8 naM[ x  3  8  3  5
dUcenH KNna)an A 1 . -cMeBaH x  5 naM[ x  3  5  3  8
372 KNnavimaRténsYnc,arenaH ³ dUcenH cMnYnTaMgBIrenaHKW 8 , 5 b¤ 5,8 .
tag a CabeNþay nig b CaTTwg énsYn 374 KNnatémø x, y BIsmIkar ³
tamsmµtikmµ ³ sYnmanépÞRkLa 720 m 2
eyIgmansmIkar x 2
 4x  y  6 y  13  0
eyIg)an ab  720 naM[ a  720 b
x 2
 

 4x  4  y  6 y  9  0

müa:geTot a  6b  4  ab x  2   y  3  0 2 2

ab  4a  6b  24  ab plbUkénBIrkaeresµI 0 luHRtakaerTaMgBIresµI 0 ³
 4a  6b  24 x  22  0  x  2  0 x  2
2a  3b  12 eyIg)an     
 2
 y  3  0   y  3  0 y  9
eyIg)an 2  720
b
 3b  12

2
240
 b  4 dUcenH témøKNna)anKW x2, y 9 .
b
480  b 2  4b
b 2  4b  480  0 cMNaM ³ ebIman A  B  0 enaHeyIg)an ³
2 2

man     2   480   484  22 2


2
A  0 nig B  0
  2  22
enaH b
1
 20  0 minyk eRBaHfa A  0 , B  0 .
2 2

  2  22
b  24
1
 772 
375 k> eRbóbeFob 3  3 nig 62  3  ³ -cMeBaH t 1 : 1 x  2  x  3

Binitü 62  3   12  6 3
dUcenH smIkarmanb£sBIrKW x2, x3 .
 96 33

 32  6 3  3
2
377 rkcMnYnEdlmanelxBIrxÞg; AB ³
 3  3  2
 3 3 eyIgman 2
AB  BA  1980
2

10 A  B 2  10B  A2  1980


dUcenH eRbóbeFob)an 3 3  6 2 3  . 10 A  B  10B  A10 A  B  10B  A  1980
9 A  9 B 11A  11B   1980
x> eRbóbeFob 14 48 nig 7  4 4
3 4 74 3 99 A  B  A  B   99  20
 A  B  A  B   20
Binitü 14 48  14 16  3  3 A 2  B 2  20
ehIy 7  4 3  7  4 3
4 4
eXIjfa kaerén A nigkaerén B manelxcugdUcKña
 7  4 3 7  4 3 
4
eday 0  A  9 , 0  B  9 eyIg)an ³
 4 49  48  4 1  1 -ebI A  81  B  61 minyk 61 minCakaerR)akd
2 2

edaysar 3 1
-ebI A  64  B  44 minyk 44 minCakaerR)akd
2 2

naM[ 14 48  4 7  4 3  4 7  4 3
-ebI A  49  B  29 minyk 29 minCakaerR)akd
2 2

dUcenH 1
48  4 7  4 3  4 7  4 3 . -ebI A  36  B  16 yk ¬CakaerR)akddUcKña¦
2 2

4 eyIg)an A  6 , B  4
376 edaHRsaysmIkar x  2  x  3 4 4
1 ³ dUcenH cMnYnmanelxBIrxÞg;enaHKW AB  64 .
eyIgmansmIkar x  2  x  3  1 4 4

tag t  x  2 naM[ x  3  t 1 378 KNna a,b,c ³


ab  4 i 
eyIg)ansmIkareTACa t  t  1  1 4 4
eyIgman 
ac  5 ii  naM[ abc2  400
t 4  t  1 t  1  1 bc  20 iii 
2 2

 
t 4  t 2  2t  1 t 2  2t  1  1  cMeBaH abc  400 Taj)an abc  20 
2

t 4  t 4  2t 3  t 2  2t 3  4t 2  2t  t 2  2t  1  1
2t 4  4t 3  6t 2  4t  0 -yk i  : abc
ab

 20
4
 c  5

 
2t t 3  2t 2  3t  2  0
-yk ii : abc 
 20
 b  4
2t t  1t  t  2  0
2 ac 5

naM[ tt10 0  tt 10 -yk iii : abc


bc

 20
20
 a  1
 
ehIy t  t  2 man   1 8  7  0 Kµanb£s
2

dUcenH KNna)an a   1 , b   4 , c  5
.
a 1 , b  4 , c  5
-cMeBaH t  0 : 0  x  2  x  2
 773 
379 KNna A ³ eyIgman 381 rkcMnYnelxsUnüenAxagcugén 2012!
A  2025  1 2025  2 2025  3  ...  2025  50  eday 2012! 1 2  3  ... 2012
2 2 2 2

eday 2025  45 enaHeyIgsresr)an ³


2
cMnYnelxsUnüenAxagcugéncMnYn 2012! bgáBIelxEdl
A  45  1 45  2 45  3  ...  45  50  CaBhuKuNén 5 , 5 , 5 , 5
2 2 2 2 2 2 2 2 2 3 4

kñúgcMeNam ktþaBI 45  1  rhUtdl; 45  50  ¬minyk 5 eRBaH 5  3125  2012 ¦


2 2 2 2 5

R)akdCamanktþamYyKW 45  45   0 2 2
-tag n CacMnYnBhuKuNén 5 1

naM[ A CaplKuN Edlman 0 Caktþa enaH A  0 enaH 5n  2012 KNna)an n  402 1 1

dUcenH KNna)an A  0 . -tag n CacMnYnBhuKuNén 5 = 25 2


2

enaH 25n  2012 KNna)an n  80 2 2

380 rkcMnYnelxsUnüenAxagcugrbs; P : -tag n CacMnYnBhuKuNén 5 = 125 3


3

eyIgman P  1 2  3...100 enaH 125 n  2012 KNna)an n  16 3 3

cMnYnelxsUnüxagcugrbs; P ekItBIcMnYnCaBhuKuNén 5 -tag n CacMnYnBhuKuNén 5 = 625 4


4

enaHeyIgRKan;EtrkcMnYnEdlCaBhuKuNén5 kñúgcMeNam enaH 625 n  2012 KNna)an n  3 4 4

elxBI 1 rhUtdl; 100 mann½yfaeyIgkMBugrkcMnYn eyIg)an cMnYnelxsUnüTaMgGs;éncMnYn 2012! KW


elxenAxagcugTaMgGs;rbs; P n  n  n  n  402  80  16  3  501
1 2 3 4

elxEdlCaBhuKuN 5 man ³ dUcenH cMnUn 2012! bBa©b;edayelxsUnücMnYn 501 .


5 , 10 ,15 , … , 100 man 20 elx

EtcMeBaHelx 25 , 50 , 75 , 100 ekItBIBhuKuNén 5 382 KNna N : eyIgman


N  4  15  4  15  2 3  5
dl;eTA 2 dg xusBIcMnYnepSgeTot enaHmanelx 5
8  2 15 8  2 15 62 5
cMnYn 4 eTot 
2

2
2
2
srubBhuKuNén 5 KW 20 + 4 = 24 énelx 5  5  3    5  3   2  5  1 2 2 2


dUcenH P manelxsUnüenAxagcMnYn 24 . 2 2 2


 5  3    5  3   2 5  1
***smÁal; ³ GñkGaceFVItamrebobdUcxageRkam ³
2 2 2
5 3 5  32 5 2

-ebI n CacMnYnBhuKuNén 5
1 2
enaH 5n  100 KNna)an n  20
1 1 
2
 2
2
-ebI n CacMnYnBhuKuNén 5 = 25
2
2

enaH 25n  100 KNna)an n  4


2 2
dUcenH KNna)an N  2 .
eyIg)an cMnYnelxsUnüTaMgGs; n  n  20  4  24 1 2

 774 
383 rkcMnYnXøIBN’ s ³ eyIgbUkGgÁ nigGgÁénvismIkar 1 , 2 nig 3 ³
tag x CacMnYnXøIBN’ s enaHcMnYnXøIBN’exµAKW 12  x  AB AC
 AM  2  2

RbU)abcab;)anXøIBN’exµAKW P(x)  1212 x (1) 
  BN 
BC AB

 2 2
Etsmµtikmµ RbU)abXøIBN’exµAKW P(x)  13 (2)  BC AC
CP  2  2

tam (1) nig (2) eyIg)an ³ AM  BN  CP  AB  BC  AC
12  x 1 12
  12  x 
12 3 3 dUcenH AM  BN  CP  AB  BC  AC .
 x  12  4
x 8 x> Rsayfa AM  BN  CP  AB  BC2  AC
dUcenH XøIBN’ s mancMnYn 8 RKab; . -tamc,ab;vismPaBkñúg ABM ³ AM  AB  MB
384 k> Rsayfa AM  BN  CP  AB  BC  AC tam  Gacsresr AM  AB  2 i 
BC

tambRmab;RbFaneyIgKUsrUb)an ³ -tamc,ab;vismPaBkñúg BNC ³ BN  BC  NC


A tam  Gacsresr BN  BC  AC 2
ii
o
P N -tamc,ab;vismPaBkñúg ACP ³ CP  AC  PA
o
// //
tam  Gacsresr CP  AC  AB 2
iii
B M C

eday AM , BN nig CP Caemdüanén ABC enaH eyIgbUkGgÁ nigGgÁ énvismIkar BC nig iii ³
i  , ii   
 AM  AB  2
M , N , P CacMNuckNþalerogKñaén BC , AC , AB 
 AC
  BN  BC 
PA 
AB
, MB 
BC
, NC 
AC
  2
2 2 2  AB
mü:ageTot eyIg)an )atmFüm cMnYnbIKW ³ CP  AC  2

AB BC AC
AM  BN  CP   
PM 
AC
, MN 
AB
, NP 
BC
  2 2 2
2 2 2
-tamc,ab;vismPaBkñúg AMP ³ AM  PA PM Gacsresr AM  BN  CP  AB  BC2  AC
tam  nig   eyIg)an AM  AB 
AC
1
2 2 dUcenH AM  BN  CP  AB  BC2  AC .
-tamc,ab;vismPaBkñúg BMN ³ BN  MB  MN
tam  nig   eyIg)an BN  BC 
AB
2 ***smÁal; ³ vismPaBkñúgRtIekaNmYyKW ³
2 2
-tamc,ab;vismPaBkñúg CPN ³ CP  NP  NC -plbUkRCugBIrRtUv > RCugmYyeTot
tam  nig   eyIg)an CP  BC 
AC
3 - pldkRCugBIrRtUv < RCugmYyeTot .
2 2

 775 
385 k> KNna S  a  b  c CaGnuKmn_én b ³ 387 edaHRsayRbB½n§smIkar
eyIgman a , b , c CabIcMnYnKt;ruWLaTIbtKña 1
eyIgman xx  y y z zxy yz3 xz
2 2 2


2011 2011 2011 2012
2
naM[ eyIg)an a  b 1 nig c  b 1
eyIg)an S  a  b  c tam 1 : x 2  y 2  z 2  xy  yz  xz
2 x 2  2 y 2  2 z 2  2 xy  2 yz  2 xz
 b  1  b  b  1
 3b x 2  2 xy  y 2  y 2  2 yz  z 2  z 2  2 zx  x 2  0
x  y 2   y  z 2  z  x 2  0
dUcenH KNna)an S  3b vaCaGnuKmn_én b . plbUkéneRcInkaeresµIsUnü luHRta kaernImYy²esµIsnU ü
x  y 2  0 x  y  0
x> Tajrktémøén a , b , c edaydwgfa S  333 eyIg)an  
 y  z   0   y  z  0  x  y  z
2

eyIgman S  3b EteyIgdwgfa S  333 


z  x   0
2 z  x  0

naM[ 3b  333 Taj)an b  111
tam 2 : x 2011
 y 2011  z 2011  32012
enaHeyIg)an a  b 1 b¤ a  1111  112 eyIg)an x 2011
 x 2011  x 2011  32012
nig c  b  1 b¤ c  111 1  110 3x 2011  32012
x 2011  32011
dUcenH KNna)an a  112 , b  111 , c  110 x3

386 rkcMnYnRksakñúghVÚgTaMgGs; ³ dUcenH RbB½n§smIkarmanb£s x  y  z  3 .


tag x CacMnYnRksaTaMgGs;enAkñúghVÚg
388 KNnargVas;RCugnImYy² ³
tambRmab; EdlCaBaküsmþIrbs;emxül;énhVÚgRksa
smµtikmµ ³ RtIekaN ABC manbrimaRt 80 cm
eyIgsresr)ansmIkardUcxageRkam ³
1 1
tag a , b , c CargVas;RCugTaMgbIénRtIekaN ABC
x  x  x  x  1  100
2 4 eyIg)an a  b  c  80
4 x  4 x  2 x  x  4  400
11x  396
edayRCugTaMgbImansmamaRterogKña 5 , 7 , 4
x  36 tamlkçN³smamaRt eyIgsresr)an ³
epÞógpÞat; ³ 36  36 18  9 1  100 a b c a  b  c 80
    5
5 7 4 5  7  4 16
100  100 Bit a
5  5
 a  25
dUcenH cMnYnRksaenAkñgú hVÚgTaMgGs;mancMnnY 36 . eyIg)an b 
  5  b  35
7 c  20
c 
4  5

dUcenH RCugTaMgbImanrgVas; 25cm , 35cm , 20cm .


 776 
389 KNnatémøénkenSam P  ab
c
bc ac
 
a b
: 2 2 2
391 eRbóbeFob ba  11 , ba nig ba 11
eyIgman 1a  b1  1c  0 eyIgman a  1 , b  1 nig a  b
1 1
 
1 eyIg)an ab  a  ab  b
a b c a b  1  ba  1
3 3
1 1  1 a a 1
       1
a b  c b b 1
1 1 3 1 1
 3     3
1 ehIy a  b enaH  a  b
a 3
b ab  a b  c ab  a  ab  b
1 1 1 3 1 1
 3  3     ab  1  ba  1
a 3
b c ab  a b 
a a 1
1 1 1 3  1  2
 3  3     b b 1
3
ab  c  a 1 a a 1
a
1
b
1 1
c
3
tam 1 nig 2 eyIg)an  
b 1 b b 1
3
 3 3 
a b c abc
a 1 a a 1
 1 1 1
abc 3  3  3   3 dUcenH eyIgeRbóbeFob)an  
b 1 b b 1
.
a b c 
bc ac ab
 
a2 b2 c2
3 392 eRbóbeFob A nig B :
P3 A 3 5 2 7 3 5 2 7

dUcenH KNna)antémøénkenSam P3 .  3 2 2  6  3 2 1  3 2 2  6  3 2 1

3 2 3  3 2 2  3 2  13 
390 edaHRsayRbB½n§smIkar ³ 3
2 3  3 2 2  3 2  13
 y  1  2 1
eyIgmanRbB½n§smIkar  x  3  2 3    2  1
2 1  3
3 3

2 x  3  y  1  4 2
 2  1   2  1  2
eyIgyk 1  2  2 enaHeyIg)an ³
 x  3  2 y  1  2 B  51  10 2  51  10 2

4 x  3  2 y  1  8  50  10 2  1  50  10 2  1

 3 x  3  6  5 2   2  5 2  1  5 2   2  5 2  1
2 2

 x 3  4  x 1
 5 2  1  5 2  1
x3  2 2 2

yktémø x  1 CMnYskñúgsmIkar 1 eyIg)an  5 2  1  5 2  1  2


1 : 1 3  2 y 1  2  2  2 y 1  2
eRkayBIKNna eXIjfa A  2 nig B  2 Edr
  2 y 1  0  y 1  0
 y 1  0  y  1
naM[ eyIg)an A  B  2
dUcenH RbB½n§smIkarmanKUcemøIy x  1 , y  1 . dUcenH eRbóbeFob)an A B .
 777 
393 edaHRsaysmIkar ³  1 1 1 
   
1 
  a  b b  c a  c 
a b c abc
eyIgman 1 x x  2 x x  3 x x  ... 2013x  x  0 ebI a , b , c mancMnYnpÞúyKñamYyy:agticKW ³
1  2  3 
  1    1    1  ...  
 2013 
 1  0  a  b  a  b   0
x  x  x   x   b  c  b  c   0
1 2 3
   ... 
2013
 2013  0  c  a  a  c   0
x x x x
1  2  3  ...  2013 1  2013
ebImankrNImYyy:agtic kñúgcMeNamkrNIxagelIenaH
x 1 1 1  1 
    0
1  2  3  ...  2013 a b c abc
x
 2013 1 1 1 1
  
Et 1  2  3  ...  2013 
2013
2
1  2013 a b c abc

 20131007 > dUcenH ebImanBIrcMnYnpÞúyKñamYyy:agtickñúgcMeNam


eyIg)an x
2013  1007
 1007 a , b , c enaHeyIg)anTMnak;TMng ³
 2013
1 1 1
  
1
a b c abc
.
dUcenH smIkarmanb£s x  1007 .
395 edaHRsaysmIkar ³
394 k> dak;CaplKuNktþa ³ eyIgmansmIkar 3  3 2 x 2 x
 30
eyIgman 1a  b1  1c  a  1b  c eyIg)an 32  3 x  32  3 x  30
bc  ac  ab

1  
9 3 x  3 x  30
abc abc 10
bc  ac  aba  b  c   abc 3 x  3 x 
3
a  b bc  ac  ab  cbc  ac  ab  abc  0 1 10
3x  x 
a  b bc  ac  ab  bc 2  ac 2  abc  abc  0 3 3
a  b bc  ac  ab  bc 2  ac 2   0 tag t  3 Edl t  0 ¬eRBaH 3  0 ¦
x x

a  b bc  ac  ab  a  b c 2  0 eyIg)an t  1t  103  3t  10t  3  0 2

a  b bc  ac  ab  c 2   0
   5  4 1 1
a  b bc  ab  ac  c 2   0 t1   3
a  b bc  a   a  c c  0 man   25  9  16 enaH 
3 3
t    5  4  3
a  b a  c b  c   0  2 3

dUcenH eyIgdak;CaplKuNktþa)an -cMeBaH t  3


1
1
 3 x  31  x  1

a  ba  c b  c   0 . -cMeBaH t  3 2  3x  3  x 1

dUcenH smIkarmanb£sBIrKW x  1 b¤ x  1 .
x> mancMnYnpÞúyKñamYyy:agtickñúgcMeNam a , b , c ³
tamsRmayenAkñúgsMNYr k> eyIg)an ³
 778 
396 sRmÜlRbPaK ³ 398 KNnabrimaRt nigRkLaépÞénRtIekaNenH ³
eyIgman A  x3x 14xx 1x eday x  0
2
x
x4

naM[ x 1  x 1 ehIy x  x


eyIg)an A  3xx 14xx1 x  3x 3xx1x  1
2 2
x2
tag x , x  2 , x  4 CaRCugénRtIekaNEkgenH
  x  1   x  1
 
3 x x  1   x  1  x  13 x  1 tamRTwsþIbTBItaK½r eyIg)an ³
x 2  x  2  x  4
2 2
1 1
 
3x  1 1  3x x 2  x 2  4 x  4  x 2  8 x  16
x 2  4 x  12  0
dUcenH sRmÜlRbPaK)an A
1
1  3x
.
x 2  6 x  2 x  12  0
x  6x  2x  6  0
397 edaHRsaysmIkar ³ eyIgman x  6x  2  0
x  y 1 x  6  0 x  6
 xyz  2 1 naM[   
 x  2  0  x  2 minyk
y z 5
  2 cMeBaH x6 enaH x  2  6  2  8
 xyz 6 
x  z 2  x  4  6  4  10
  3 eXIjfa RtIekaNenHmanRbEvgRCug 6 , 8 , 10
 xyz 3
2 x  y  z  1 5 2
xyz
  
2 6 3
naM[ brimaRt P = 6 + 8 + 10 = 24 ÉktþaRbEvg
2x  y  z  3  5  4

RkLaépÞ S  12  6  8  24 ÉktþaépÞRkLa
xyz 6
x yz
1 4 dUcenH KNna)an brimaRtesµI 24 ÉktþaRbEvg
xyz

RkLaépÞesµI 24 ÉktþaépÞRkLa .
4  1 : z

1
 xy  2

xyz 2 
399 sRmYlkenSam F  xx 11
2

4  2 : x  1  yz  6
  xyz   36
2

xyz 6 

4  3 : y  1  xz  3

-krNI x  1 naM[ x 1  x 1
eyIg)an F  xx 11  x x1x1 1  x  1
xyz 3  2

xyz  6
xyz  6 ,   z  3
xy 2
xyz  6 xyz  6
yz

6
 x  1 ,
xz

3
 y  2 -krNI x  1 naM[ F Kµann½y .
dUcenH RbB½n§smIkarmanKUcemøIy ³ -krNI x  1 naM[ x 1  x 1
eyIg)an F  xx 11  x1xx11 
2
x 1 , y  2 , z  3 b¤   x  1 .
x  1 , y  2 , z  3 .
 779 
400 KNnacm¶ayBIcMNuc A eTAbnÞat; (D) ³ 401 >>>>>>>>>>>>>>>¬enAmanbnþ¦
eyIgmancMNuc A6 , 6 nigbnÞat; D: y   x  4
A6 , 6 

D : y  x

 B2 , 2

D : y   x  4

-rkbnÞat;Edlkat;tam A6 , 6 nigEkgnwg D 


bnÞat;RtUvrkmanrag D : y  ax  b
eday D  D  a  a1   11  1
ehIy D kat;tam A6 , 6 enaH x  6 , y  6
eyIg)an D : 6  1 6  b  b  0
dUcenH bnÞat; D : y  x
-rkcMNucEdl D  RbsBVnwg D tageday B ³
pÞwmsmIkarGab;sIus eyIg)an ³
 x  4  x  2x  4  x  2 >

cMeBaH x  2  y  2 naM[cMNuc B2 , 2


-rkcm¶atBIcMNuc A6 , 6 eTAbnÞat; D: y   x  4
mann½yfa rkcm¶ayBI A6 , 6 eTAcMNuc B2 , 2
d  AB  xB  x A 2   y B  y A 2
 2  62  2  62
 16  16  32  4 2

dUcenH cm¶ayrk)anKW d 4 2 ÉktþaRbEvg .


 780 
sYsþI¡ elakGñkmitþGñkGanCaTIRsLaj;rab;Gan enAkñúgEpñkenHelakGñknwg)aneXIj BIkarbEnßmCUn
Biess² nigl¥² dUcCakarkmSanþKNitviTüa taragsV½yKuN >>>. ral;GVI² EdlsMxan; ehIy
Tak;TgnwgcMeNHdwgKNitviTüafñak;TI 9 RtUv)an´erobcMCUnmitþGñkGanenAkñúgEpñkenH (...) .
critlkçN³sMxan;enARtg;EpñkenH KWkarKNnaelx[)anrh½s dwgBIcMNucl¥² EdlmanTMnak;TMng
CamYyemeronKNitviTüafñak;TI 9 . karkmSanþsb,ayCamYyKNitviTüa eFVI[eyIgmancMNg;cMNUlcitþ edIm,I
sikSaemeronKNitviTüa . elIsBIenHeTAeTot EpñkenHk¾manbgðajGMBIkarrIkceRmInrbs;KNitviTüapgEdr.
RbsinebIelakGñkmanbBaða b¤cm¶l;Rtg;cMNucNa EdlmanenAkñúgEpñkenH elakGñkGacTak;Tg;eTAkan;
RKU b¤mitþPkþirbs;GñkEdlmansmtßPaB b¤GñkeroberogesovePAenHkñúgeBlevlasmRsb .

viii
 : 001 kñúgcMeNamrUbTaMgbYnxageRkam etIrUbNamYyCarUbEdlmanlkçN³xuseK ???

k> kaer x> RtIekaNsm½gS K> RtIekaNEkg X> ctuekaNesµI

 : 002 GñksYrnrNamñak;nUvsMNYrxageRkam ehIy[eKeqøIyy:agrh½sbMput ³


etI«Bukekµkbgéfø´ RtUvCaGVInwg´ ???

 : 003 GñksYrnrNamñak;nUvsMNYrxageRkam ehIy[eKeqøIyy:agrh½sbMput ³


etI 2  2 2  b:unµan ???

 : 004 GñksYrnrNamñak;nUvsMNYrxageRkam ehIy[eKeqøIyy:agrh½sbMput ³


elxenaHminFMCag 2 ehIymintUcCag 2 etIelxenaHb:unµan ???

 : 005 bursmñak;mankUneQIEt 6 edIm b:uEnþKat;GacdaM)an 4 CYr EdlkñúgmYyCYr²mankUneQI


cMnYn 3 edImCanic©. etIKat;daMtamreboby:agNa cUrKUsrUbbBa¢ak;kardaM ???

 : 006 bursmñak;mankUneQIEt 10 edIm b:uEnþKat;GacdaM)an 5 CYr EdlkñúgmYyCYr²mankUneQI


cMnYn 4 edImCanic©. etIKat;daMtamreboby:agNa cUrKUsrUbbBa¢ak;kardaM ???

 : 007 bursmñak;mankUneQIEt 12 edIm b:uEnþKat;GacdaM)an 6 CYr EdlkñúgmYyCYr²mankUneQI


cMnYn 4 edImCanic©. etIKat;daMtamreboby:agNa cUrKUsrUbbBa¢ak;kardaM ???

 781 
 : 008 eKmanelx 1 , 2 , 3 , 4 , 5 , 6 , 7 , 8 , 9 . cUrykelxTaMgenH
eTAbMeBjkñúgRbGb;kaerxagsþaM edIm,I[plbUkCYredk esµIplbUk
CYrQr esµIplbUkGgÁt;RTUg esµInwg 15 .

 : 009 etIelxb:unµan EdlmanplbUkesµIplKuNénxøÜnÉg ???

 : 010 cUrKUsbnÞat; 6 edIm,I[)anRtIekaNcMnYn 8 .

 : 011 stVExVkTMelIEmkxVav )ak;EmkR)av gab; 3 rs; 2 cuHebIstV 120 gab;b:unµanrs;b:unµan ???

 : 012 etImanviFIsaRsþGVI EdleFVI[eKeCOeyIgfa 11 bUkEfm 2 eTotesµInwg 1 ???

 : 013 edayeRbIelx 1 , 2 , 3 , 4 , 5 , 6 , 7 , 8 cUrykelxTaMgenHdak;


kñúgRbGb;TaMg 8 xagsþaM ély:agNa kM[u elxEdlenAbnÞab;Kña
sßitenAkñúgRbGb;Cab;Kña b¤enAExVgKñak¾eday .

 : 014 etIeKemIly:agdUcemþcdwgfaqñaMNacUlqñaeM nAéf¶TI 13 ehIyqñaMNacUlqñaMéf¶TI 14 ???

 : 015 enAelIpøÚvRtg;mYymanLanebIkCaCYr ebIdwgfa LanmuxebIkmuxLanBIr LankNþal


ebIkkNþalLanBIr LaneRkayebIkeRkayLanBIr etILanTaMgGs;manb:unµan ???

 : 016 davImanGayudUc x ebIKit[xÞic x CaBhuKuNén 8 edaydwgfa x FMCag 10 nigtUcCag


20 etIsBVéf¶davIRsImanGayub:unµan ???

 782 
 : 017 enACMuvijRtIekaNxagsþaMeKeRbIcMnYnBI 2 dl; 10 edIm,IteRmób 3

[cMnYnEdlenAelIRCugnImYy²énRtIekaNmanplbUkesµI 21. 6 5

etIeKRtUvteRmóbcMnYnenHya:gNa edIm,I[cMnYnelIRCugnImYy² 10 9


2   4
manplbUkesµInwg 24 vijmþg ??? 7 8

 : 018 tamry³lMnaMrUbxageRkam etIRtUvykelxb:unµanmkbMeBjenAkñúgRbGb;sBaØa {?} ???


8 7 6

2 4 3 9 4 8
20 30 ?

 : 019 rkbIelxEdldUcKña éllkeFVIplbUkedIm,I[esµI 24 . bUkya:gNaeRsccitþ


[EtesµI 24 ehIybIelxenaHKWCaelxdUcKña ]TahrN_ 8  8  8  24 .

 : 020 eKmanb‘íccMnYn 10 edImerobmYy²enAdac;edayELkBIKña)anmYyCYrdUcrUbxageRkam.


cUrelIkb‘ícmþgmYy²eTAcab;KUKña edaykñúgkarelIkRtUvrMlgb‘íccMnYnBIKt; eTaHbICa
sßitenAkñúgsßanPaBEbbNak¾edayKWrMlgb‘ícBIrCanic© edIm,I[bí‘cTaMgGs;enACaKU².

 : 021 enAkñúgrUbctuekaNEkgxageRkammanGkSr A , B , C CaKU². cUrGñkKUsExSP¢ab;KUGkSr


EdldUcKña KWBI A eTA A / BI B eTA B nigBI C eTA C edaymin[ExSenaHkat;KñaeLIy
ehIyExSEdlKUsP¢ab;minGacenAeRkActuekaNEkg)aneLIy.
B
smÁal; ³ GñkRtUvcmøgrUbdak;RkdasepSgeTot
A C A C minRtUvKUsP¢ab;elIrUbxageqVgenHeLIg
B edIm,ITuk[GñkeRkaykmSanþbnþeTot .
 783 
 : 022 bursmñak;cg;cmøg xøa / eKa nig esµA eTAeRtIymçageToténTenømYy. TUkrbs;Kat;tUc
EdleFVI[Kat;Gaccmøg)anmþgmYyb:ueNÑaHkñúgcMeNam xøa / eKa nigesµA . eKdwgfaebI
bursenaHminenAKW xøasIueKa b¤ eKasIuesµA EtebIbursenaHenAKWKµankarsIuKñaekIteLIgeT.
etIKat;KYcmøg xøa / eKa nigesµA eTAeRtIymçageTotedayviFINaeTIbKµankar)at;bg; .

 : 023 GñksYrnrNamñak;nUvsMNYrxageRkam ehIy[eKeqøIyy:agrh½sbMput ³


mþayBUsn mankUnRbusbInak;KW kUnRbusb¥ÚneKmaneQµaHfa sux kUnRbusTI2maneQµaH
fa esA etIkUnRbusc,geKmaneQµaHfaGVI ???

 : 024 kaerFMmYyeKGacEckeTACakaertUcCagcMnYn 7 dUcrUbxagsþaM


cUrGñkEckkaerFMenHeTACakaertUcCagcMnYn 6 vijmþg .

 : 025 eKeRbIeQIcak;eFµjedIm,Ierob)ankaercMnYn 4 b:un²Kña. cUrbþÚrTItaMg


eQIcak;eFµjcMnYn 3 edIm,I[)ankaercMnYn 3 EdlmanTMhMb:un²Kña.

 : 026 eKmanEmkeQIBIrmanRbEvgRbEhlKña. kac;EmkeQITaMgBIrRtg;cMNuckNþaldUcKña


edaykac;mYy[dac; nigmYyeTotkMu[dac;. eKykEmkeQIEdldac;mYyeTAdak;Tl;
EmkeQIEdlminTan;dac; [mansßanPaBlMnwg ¬dUcrUbxageRkam¦ . eKnwgykEmk
eQIEdldac;mYyeToteTAelIkEmkeQITaMgGs;[eLIgputBIdIkñúgeBlEtmYy .
etIeKRtUvelIkdUcemþc ???

 784 
 : 027 eKeRbIeQIcak;eFµjedIm,Ierob)ankaercMnnY 5 b:un²Kña. cUrbþÚrTItaMg
eQIcak;eFµjcMnYn 2 edIm,I[)ankaercMnYn 4 EdlmanTMhMb:un²Kña.

 : 028 bursmñak;mandIERsragCakaer . Kat;cg;daMbEnøelIdIERsenH dUcenHKat;RtUveFVIrbg.


ebIdwgfaRCugnImYy²RtUvmanbegÁal 10 edImdUcKña rkcMnYnbegÁalTaMgGs;EdlKat;
RtUvkar ?

 : 029 cUrKUsGgát;bIbEnßmBIelIrUbxagsþaM edIm,I[vakøayCaRbGb;dIs.


¬sUmcmøgrUbecj kMuKUselIesovePAenH Tuk[GñkeRkaykmSanþbnþ¦

 : 030 cUrbMeBjRbGb;[)anRtwmRtUv ³ c6  30
s .

 : 031 tamlMnaMKRmUénrUbbIxagedIm cUrrkelxmkCMnYssBaØa {?} [)anRtwmRtUv ³


4 5 9 5 8 6 6 2
6 30 20 ?
2 4 3 5 2 4 5 3

 : 032 eK[lkçxNÐbI EdlbursbInak; A , B , C eTAsþIdNþwgnarIbInak; 1, 2 , 3 CaKUGnaKt


dUcxageRkam ³
k> ebIburs A CaKUnwgnarI 1 enaH burs B CaKUnwgnarI 2
x> ebIburs A CaKUnwgnarI 3 enaH burs C CaKUnwgnarI 1
K> ebIburs B minCaKUnwgnarI 3 enaH burs C CaKUnwgnarI 1 .
sMNYrsYrfa etIbursNa CaKUnwgnarINa ?

 785 
 : 033 rkBIrcMnYnedaydwgfa plbUk plKuN nigplEckrvagcMnYnTaMgBIr esµIKña .

 : 034 ksikmñak;mancMNIsRmab;pÁt;pÁg;eKarbs;Kat; 40 k,al)an 35 éf¶.


ebIKat;TijeKa 10 k,albEnßmeTot etIkat;GacpÁt;pgÁ ;cMNIdEdl[eKa)anb:unµanéf¶ ?

 : 035 bursmñak;man)arImUledayéd 10 edIm. Kat;Ck;)arIedayrkSaknÞúy)arI ebICk;Gs;bIedIm


enaHKat;GacykbnÞúy)arIEdlsl;mUl)anmYyedImfµvI ij .
etIKat;Ck;b:unµandgeTIbGs;)arITaMg 10 edImKµansl; ?

 : 036 kñúgkic©RbCMumYymanmnusS 6 nak; )ancUlrYm ehIyGñkTaMgenaH)ancab;édsVaKmn_Kña


eTAvijeTAmk. ebIdwgfaGñkTaMgGs;)ancab;édKñaRKb;²Kña. cUrrkcMnYnénkarcab;édKña
TaMgGs;.

 : 037 mFüménBIrcMnYnesµI 2012 nigmFüménbIcMnYnk¾esµInwg 2012 Edr. cUrrkcMnYnTI 3 .

 : 038 etIrUbxagsþamM ankaerTaMgGs;cMnYnb:unµan ?

 : 039 eKmanGkSrLataMg T , H , L,K , E , F etIGkSrNamanlkçN³xuseK ???

 : 040 kñúgfñak;eronmYy eQµaHrbs;sux enAelxerogTI 20 ebIrab;BIxagelI EtebIrab;BIxag


eRkamvij k¾enAEtelxerogTI 20 dEdl. etIfñak;eronenaHmansisSb:unµannak; ?

 786 
005
001

 

 

4
3 6

ÂÂCÂÂ ÂÂCÂÂ

3 3
002
006


   
 

ÂÂCÂÂ  

3 5
4 10
003
ÂÂCÂÂ
2  2 2  6 ”
2  2 2
2 2  4 42  6
3
8
007
ÂÂCÂÂ 

3 

 

004    

2
5
2 2
4 10
2
ÂÂCÂÂ ÂÂCÂÂ

3 3
 787 
008 011
15 120 72 48
2 7 6  15 5 3 2
9 5 1  15
4 3 8  15 120  3 120  2
120  72  48
15 15 15 15 5 5
ÂÂCÂÂ
15
3
.......
012
ÂÂCÂÂ 11 2

3 1
11 + 2 = 13
13 1
009 ÂÂCÂÂ

0 2
3
0  0  0 0 2  2  2 2 013
ÂÂCÂÂ

3 7
3
1
5
8 2
4 6
010
6 8
ÂÂCÂÂ

3
014

6 4
2 8 29 13
6 4
ÂÂCÂÂ 28 14

3 ÂÂCÂÂ

3
 788 
015 019
2, 2, 2

22 + 2 = 24
2, 2, 2
ÂÂCÂÂ ÂÂCÂÂ

3 3
016 020
x  16 ”

16 8
10 20
ÂÂCÂÂ

3
017 ÂÂCÂÂ

6

3
5 2
10 7
021
3   9
4 8
B
24 A C A C
ÂÂCÂÂ B
3 ÂÂCÂÂ


017
32
3
022
6

4 32 8

2  8  4  20
3 7  9  30 4  6  8  32
ÂÂCÂÂ ÂÂCÂÂ

3 3
 789 
023 027
” 2

ÂÂCÂÂ

3 3
ÂÂCÂÂ

024
6
028
36
8
   
 
 
ÂÂCÂÂ  
 

3
10   10
 
 
 
    

8
025
 10 10  8  8  36
ÂÂCÂÂ

3
029
ÂÂCÂÂ

3

026
ÂÂCÂÂ

3
030
c  30 
6 s
1

6 30
c  c  6 1 30s  30  1

ÂÂCÂÂ 6 c 6 s s 30
3 ÂÂCÂÂ

3
 790 
031 034
6 2 28
” 8
5 3
x

4 5 9 5
50
6 4 4  25  6 , 30 9  5  3 5  30 1
2 4 3 5  35
40  1  1
8 6  x    35    28
1  50  40
20 8  4  2  6  20 x
2 4 50

ÂÂCÂÂ ÂÂCÂÂ

3 3
032 035
A  3 , B  2 , C 1 15
10  10 10 =3 +1
6
3 +1 3 4 =1 +1
១. A  1 , B  2 , C  3
1 +1 1 2 + 1 =1 )
២. A  1 , B  3 , C  2
1 1 1
៣. A  2 , B  1 , C  3
10  3  1  1  15
៤. A  2 , B  3 , C  1
ÂÂCÂÂ
៥. A  3 , B  1 , C  2

៦. A  3 , B  2 , C  1
3
6
036
6 15
ÂÂCÂÂ

3 
 

 
ÂÂCÂÂ
033
1
1
3
2
1
1 1 1
037
  1    1  2 
2 2 1 2 2012 A, B, C
A B
 2012
2
A B C
A  B  4024  2012
ÂÂCÂÂ 3

3
4024  C
 2012 ឬ C  2012
3

 791 
038

14
ÂÂCÂÂ

3
039
K
T, H , L , E , F

ÂÂCÂÂ

3
040
39
20
20
 20  20 1  39
1
ÂÂCÂÂ

 792 
brimaRt épÞRkLa nigmaD énrUbFrNImaRt

1) RtIekaNsam½BaØ 5) ctuekaNEkg
-RtIekaNsam½BaØ CaRtIekaNKµanlkçN³Biess. -CactuekaNmanRCugb:unKñaBIr² nigmanmMuEkg .
-RkLaépÞ ³ S  ab A D
-RkLaépÞ ³ S  12 bh B

c a -brimaRt ³ P  2a  b a
-brimaRt ³ P  a bc
h
-Ggát;RTUg ³ d  a  b B
2 2
b
C
A b C
6) RbelLÚRkam
2) RtIekaNEkg
-CactuekaN EdlmanRCugRsbKñaBIr² .
-RtIekaNEkg CaRtIekaNmanmMuEkg manrgVas; 90 . o

-RkLaépÞ ³ S  bh  ab sin  A D

-RkLaépÞ ³ S  12 ab -brimaRt ³ P  2a  b B h


a

c b C
-brimaRt ³ P  a  b  c a

-GIub:Uetnus ³ c  a  b 2 2 b 7) ctuekaNesµI
-CactuekaNmanRCugTaMgbYnb:unKña nigKµanmMuEkg.
3) RtIekaNsm½gS -RkLaépÞ ³ S  bc2 a
c

- CaRtIekaNmanRCugTaMgbIb:unKña -brimaRt ³ P  4a
b
mMuTaMgbIesµIKañ esµInwg 60 .
o

8) ctuekaNBñay
a2 3
-RkLaépÞ ³ 1
S  ah 
2 4 -CactuekaNmanRCugRsbKñaEtBIr .
a

-RkLaépÞ ³ S  a 2bh
A a B

-km<s; ³ h
2
3
a
h
c h d

-brimaRt ³ P  3a -brimaRt ³ P  a  b  c  d D b C

9) rgVg; ¬fas¦
4) kaer -CasMNMucMNcu CaeRcInsßitenAesµIcm¶ay
-CactuekaNmanRCugTaMgbYnb:unKña nigmanmMuEkg. BIcMNucnwgmYy¬p©it¦.
r
-RkLaépÞ ³ S  a -RkLaépÞ ³ S  r  d4
2
2
A D 2
A  B
o
-brimaRt ³ P  4a a
d -brimaRt ³ P  2r  d
-Ggát;RTUg ³ d  a 2 B C -Ggát;p©it ³ d  2r d

793 -
10) KUb 15) ceRmókfas
-CarUbFrNImaRtkñúglMhmanRTnugb:un²Kña . -RkLaépÞ ³ S  12 r  2
r s

-RkLaépÞ ³ S  6a 2 H G

E F -RbEvgFñÚ ³ s  r r
-maD ³ V  a 3
a

-Ggát;RTUg ³ d  a 3
D
a C 16) EsV‘ R
A a B -RkLaépÞ ³ S  4 R 2

11) RbelBIEb:tEkg -maD ³ S  43  R 3

-CarUbFrNImaRtkñúglMhman
RTnugminb:unKña . H G 17) ekaN 
r

-RkLaépÞ ³ S  4ab  2aca E F


-RkLaépÞ ³ S  rl  r 2
 r l  r  h
D C l
-maD ³ V  abc
c
-maD ³ V  13 r h 2
A b B
CenRt
-Ggát;RTUg ³ d  a  b  c2 2 2

12) sIuLaMg 18) BIr:amIt A GabU:Etm a

R
-CarUbFrNImaRtkñúglMhman 
-RkLaépÞ ³ S  12  p  a h D
C

)atBIrCargVg;b:unKña . p a
Sb
I

- S  2 Rh
L
h -maD ³ 1
V  Sb h A B
3
-RkLaépÞ ³ S  S  2S T L B
C

-maD ³ V  S h   R h
B
2 SB
A

Edl S CaRkLaépÞxag , S
L B 19) RBIs B

13) eGlIb -maD ³ V  13 S h b


Sb
F

-(emIlniymn½yeGlIb) b
D
E

-RkLaépÞ ³ S  ab a
20) cMNit)a:r:abUl B

-brimaRt ³ P  2 
1 2
a  b2  a
2
-RkLaépÞ ³ 2
S  ab
3 A C
b

14) RkLaépÞcMNitfas
21) kaMrgVg;carwkeRkARtIekaN

-RkLaépÞ ³ S  12 r 2
  sin   r

r - R
abc
4 p p  a  p  b  p  c  a
c

R
P abc
Edl p 
2 2
b

794 -
22) ctumux A
29)
 r
- S  aR  r
L
-maD ³ V  13 S h b - S  aR  r  R r
T
2 2 h a

-maD³ V  3h  R  r  Rr  
D h
2 2
Sb
R
B
C

23) sIuLaMgeRTt 
r
30) kaMrgVg;carwkkñúgRtIekaN
-maD ³ V  R h  R l sin
2 2
l

h - : a
c
-RkLaépÞ ³ S  2Rl  2sinRh
 abc
 r
p
2 b
r
 p  a  p  b  p  c 
24) BhuekaNman n RCugcarwkkñúgrgVg; p
1 2 2 1 2 360 o r
S nr sin  nr sin
2 n 2 n 
 180 o 31)
h
P  2nr sin  2nr sin
n n - S  2 Rh

2
- V   R2h R
3
25) BhuekaNman n RCugcarwkeRkArgVg; - 1
V   h 2  3R  h 
 180 o 3
S  nr 2 tan  nr 2 tan r
n n 
 180 o 32) Sb
P  2nr tan  2nr tan
n n
- V
h
3
S B  Sb  S B S b  h

SB
26) eGlIbbrivtþ c


-manmaD 4
V  abc
3
a
b

27) cMNit)a:r:abUlbrivtþ ១២

a
1
-manmaD V  b 2 a
2  b

28) RbelBIEb:teRTt
-manmaD V  S h B

h 

795 -
n 1 100
n n2 n3 n4 n5 n 3
n 4
n 5
n
1 1 1 1 1 1 1 1 1
2 4 8 16 32 1.414214 1.259921 1.189207 1.148698
3 9 27 81 243 1.732051 1.442250 1.316074 1.245731
4 16 64 256 1024 2.000000 1.587401 1.414214 1.319508
5 25 125 625 3125 2.236068 1.709976 1.495349 1.379730
6 36 216 1296 7776 2.449490 1.817121 1.565085 1.430969
7 49 343 2401 16807 2.645751 1.912931 1.626577 1.475773
8 64 512 4096 32768 2.828427 2.000000 1.681793 1.515717
9 81 729 6561 59049 3.000000 2.080084 1.732051 1.551846
10 100 1000 10000 100000 3.162278 2.154435 1.778279 1.584893
11 121 1331 14641 161051 3.316625 2.223980 1.821160 1.615394
12 144 1728 20736 248832 3.464102 2.289428 1.861210 1.643752
13 169 2197 28561 371293 3.605551 2.351335 1.898829 1.670278
14 196 2744 38416 537824 3.741657 2.410142 1.934336 1.695218
15 225 3375 50625 759375 3.872983 2.466212 1.967990 1.718772
16 256 4096 65536 1048576 4.000000 2.519842 2.000000 1.741101
17 289 4913 83521 1419857 4.123106 2.571282 2.030543 1.762340
18 324 5832 104976 1889568 4.242641 2.620741 2.059767 1.782602
19 361 6859 130321 2476099 4.358899 2.668402 2.087798 1.801983
20 400 8000 160000 3200000 4.472136 2.714418 2.114743 1.820564
21 441 9261 194481 4084101 4.582576 2.758924 2.140695 1.838416
22 484 10648 234256 5153632 4.690416 2.802039 2.165737 1.855601
23 529 12167 279841 6436343 4.795832 2.843867 2.189939 1.872171
24 576 13824 331776 7962624 4.898979 2.884499 2.213364 1.888175
25 625 15625 390625 9765625 5.000000 2.924018 2.236068 1.903654
26 676 17576 456976 11881376 5.099020 2.962496 2.258101 1.918645
27 729 19683 531441 14348907 5.196152 3.000000 2.279507 1.933182
28 784 21952 614656 17210368 5.291503 3.036589 2.300327 1.947294
29 841 24389 707281 20511149 5.385165 3.072317 2.320596 1.961009
30 900 27000 810000 24300000 5.477226 3.107233 2.340347 1.974350
31 961 29791 923521 28629151 5.567764 3.141381 2.359611 1.987341
32 1024 32768 1048576 33554432 5.656854 3.174802 2.378414 2.000000
33 1089 35937 1185921 39135393 5.744563 3.207534 2.396782 2.012347
34 1156 39304 1336336 45435424 5.830952 3.239612 2.414736 2.024397
35 1225 42875 1500625 52521875 5.916080 3.271066 2.432299 2.036168
36 1296 46656 1679616 60466176 6.000000 3.301927 2.449490 2.047673
37 1369 50653 1874161 69343957 6.082763 3.332222 2.466326 2.058924
38 1444 54872 2085136 79235168 6.164414 3.361975 2.482824 2.069935
39 1521 59319 2313441 90224199 6.244998 3.391211 2.498999 2.080717
40 1600 64000 2560000 102400000 6.324555 3.419952 2.514867 2.091279
41 1681 68921 2825761 115856201 6.403124 3.448217 2.530440 2.101632
42 1764 74088 3111696 130691232 6.480741 3.476027 2.545730 2.111786
43 1849 79507 3418801 147008443 6.557439 3.503398 2.560750 2.121747
44 1936 85184 3748096 164916224 6.633250 3.530348 2.575510 2.131526
45 2025 91125 4100625 184528125 6.708204 3.556893 2.590020 2.141127
46 2116 97336 4477456 205962976 6.782330 3.583048 2.604291 2.150560
47 2209 103823 4879681 229345007 6.855655 3.608826 2.618330 2.159830
48 2304 110592 5308416 254803968 6.928203 3.634241 2.632148 2.168944
49 2401 117649 5764801 282475249 7.000000 3.659306 2.645751 2.177906
50 2500 125000 6250000 312500000 7.071068 3.684031 2.659148 2.186724

796 -
n n2 n3 n4 n5 n 3
n 4
n 5
n
51 2601 132651 6765201 345025251 7.141428 3.708430 2.672345 2.195402
52 2704 140608 7311616 380204032 7.211103 3.732511 2.685350 2.203945
53 2809 148877 7890481 418195493 7.280110 3.756286 2.698168 2.212357
54 2916 157464 8503056 459165024 7.348469 3.779763 2.710806 2.220643
55 3025 166375 9150625 503284375 7.416198 3.802952 2.723270 2.228807
56 3136 175616 9834496 550731776 7.483315 3.825862 2.735565 2.236854
57 3249 185193 10556001 601692057 7.549834 3.848501 2.747696 2.244786
58 3364 195112 11316496 656356768 7.615773 3.870877 2.759669 2.252608
59 3481 205379 12117361 714924299 7.681146 3.892996 2.771488 2.260322
60 3600 216000 12960000 777600000 7.745967 3.914868 2.783158 2.267933
61 3721 226981 13845841 844596301 7.810250 3.936497 2.794682 2.275443
62 3844 238328 14776336 916132832 7.874008 3.957892 2.806066 2.282855
63 3969 250047 15752961 992436543 7.937254 3.979057 2.817313 2.290172
64 4096 262144 16777216 1073741824 8.000000 4.000000 2.828427 2.297397
65 4225 274625 17850625 1160290625 8.062258 4.020726 2.839412 2.304532
66 4356 287496 18974736 1252332576 8.124038 4.041240 2.850270 2.311579
67 4489 300763 20151121 1350125107 8.185353 4.061548 2.861006 2.318542
68 4624 314432 21381376 1453933568 8.246211 4.081655 2.871622 2.325422
69 4761 328509 22667121 1564031349 8.306624 4.101566 2.882121 2.332222
70 4900 343000 24010000 1680700000 8.366600 4.121285 2.892508 2.338943
71 5041 357911 25411681 1804229351 8.426150 4.140818 2.902783 2.345588
72 5184 373248 26873856 1934917632 8.485281 4.160168 2.912951 2.352158
73 5329 389017 28398241 2073071593 8.544004 4.179339 2.923013 2.358656
74 5476 405224 29986576 2219006624 8.602325 4.198336 2.932972 2.365083
75 5625 421875 31640625 2373046875 8.660254 4.217163 2.942831 2.371441
76 5776 438976 33362176 2535525376 8.717798 4.235824 2.952592 2.377731
77 5929 456533 35153041 2706784157 8.774964 4.254321 2.962257 2.383956
78 6084 474552 37015056 2887174368 8.831761 4.272659 2.971828 2.390116
79 6241 493039 38950081 3077056399 8.888194 4.290840 2.981308 2.396213
80 6400 512000 40960000 3276800000 8.944272 4.308869 2.990698 2.402249
81 6561 531441 43046721 3486784401 9.000000 4.326749 3.000000 2.408225
82 6724 551368 45212176 3707398432 9.055385 4.344481 3.009217 2.414142
83 6889 571787 47458321 3939040643 9.110434 4.362071 3.018349 2.420001
84 7056 592704 49787136 4182119424 9.165151 4.379519 3.027400 2.425805
85 7225 614125 52200625 4437053125 9.219544 4.396830 3.036370 2.431553
86 7396 636056 54700816 4704270176 9.273618 4.414005 3.045262 2.437248
87 7569 658503 57289761 4984209207 9.327379 4.431048 3.054076 2.442890
88 7744 681472 59969536 5277319168 9.380832 4.447960 3.062814 2.448480
89 7921 704969 62742241 5584059449 9.433981 4.464745 3.071479 2.454019
90 8100 729000 65610000 5904900000 9.486833 4.481405 3.080070 2.459509
91 8281 753571 68574961 6240321451 9.539392 4.497941 3.088591 2.464951
92 8464 778688 71639296 6590815232 9.591663 4.514357 3.097041 2.470345
93 8649 804357 74805201 6956883693 9.643651 4.530655 3.105423 2.475692
94 8836 830584 78074896 7339040224 9.695360 4.546836 3.113737 2.480993
95 9025 857375 81450625 7737809375 9.746794 4.562903 3.121986 2.486250
96 9216 884736 84934656 8153726976 9.797959 4.578857 3.130169 2.491462
97 9409 912673 88529281 8587340257 9.848858 4.594701 3.138289 2.496631
98 9604 941192 92236816 9039207968 9.899495 4.610436 3.146346 2.501758
99 9801 970299 96059601 9509900499 9.949874 4.626065 3.154342 2.506842
100 10000 1000000 100000000 10000000000 10.000000 4.641589 3.162278 2.511886

Computer

797 -
sin  cos  tan  cot  sin  cos  tan  cot 
º 0.0000 0.0000 1.0000 0.0000 ------- 46º 0.8029 0.7193 0.6947 1.0355 0.9657
1º 0.0175 0.0175 0.9998 0.0175 57.2900 47º 0.8203 0.7314 0.6820 1.0724 0.9325
2º 0.0349 0.0349 0.9994 0.0349 28.6363 48º 0.8378 0.7431 0.6691 1.1106 0.9004
3º 0.0524 0.0523 0.9986 0.0524 19.0811 49º 0.8552 0.7547 0.6561 1.1504 0.8693
4º 0.0698 0.0698 0.9976 0.0699 14.3007 50º 0.8727 0.7660 0.6428 1.1918 0.8391
5º 0.0873 0.0872 0.9962 0.0875 11.4301 51º 0.8901 0.7771 0.6293 1.2349 0.8098
6º 0.1047 0.1045 0.9945 0.1051 9.5144 52º 0.9076 0.7880 0.6157 1.2799 0.7813
7º 0.1222 0.1219 0.9925 0.1228 8.1443 53º 0.9250 0.7986 0.6018 1.3270 0.7536
8º 0.1396 0.1392 0.9903 0.1405 7.1154 54º 0.9425 0.8090 0.5878 1.3764 0.7265
9º 0.1571 0.1564 0.9877 0.1584 6.3138 55º 0.9599 0.8192 0.5736 1.4281 0.7002
10º 0.1745 0.1736 0.9848 0.1763 5.6713 56º 0.9774 0.8290 0.5592 1.4826 0.6745
11º 0.1920 0.1908 0.9816 0.1944 5.1446 57º 0.9948 0.8387 0.5446 1.5399 0.6494
12º 0.2094 0.2079 0.9781 0.2126 4.7046 58º 1.0123 0.8480 0.5299 1.6003 0.6249
13º 0.2269 0.2250 0.9744 0.2309 4.3315 59º 1.0297 0.8572 0.5150 1.6643 0.6009
14º 0.2443 0.2419 0.9703 0.2493 4.0108 60º 1.0472 0.8660 0.5000 1.7321 0.5774
15º 0.2618 0.2588 0.9659 0.2679 3.7321 61º 1.0647 0.8746 0.4848 1.8040 0.5543
16º 0.2793 0.2756 0.9613 0.2867 3.4874 62º 1.0821 0.8829 0.4695 1.8807 0.5317
17º 0.2967 0.2924 0.9563 0.3057 3.2709 63º 1.0996 0.8910 0.4540 1.9626 0.5095
18º 0.3142 0.3090 0.9511 0.3249 3.0777 64º 1.1170 0.8988 0.4384 2.0503 0.4877
19º 0.3316 0.3256 0.9455 0.3443 2.9042 65º 1.1345 0.9063 0.4226 2.1445 0.4663
20º 0.3491 0.3420 0.9397 0.3640 2.7475 66º 1.1519 0.9135 0.4067 2.2460 0.4452
21º 0.3665 0.3584 0.9336 0.3839 2.6051 67º 1.1694 0.9205 0.3907 2.3559 0.4245
22º 0.3840 0.3746 0.9272 0.4040 2.4751 68º 1.1868 0.9272 0.3746 2.4751 0.4040
23º 0.4014 0.3907 0.9205 0.4245 2.3559 69º 1.2043 0.9336 0.3584 2.6051 0.3839
24º 0.4189 0.4067 0.9135 0.4452 2.2460 70º 1.2217 0.9397 0.3420 2.7475 0.3640
25º 0.4363 0.4226 0.9063 0.4663 2.1445 71º 1.2392 0.9455 0.3256 2.9042 0.3443
26º 0.4538 0.4384 0.8988 0.4877 2.0503 72º 1.2566 0.9511 0.3090 3.0777 0.3249
27º 0.4712 0.4540 0.8910 0.5095 1.9626 73º 1.2741 0.9563 0.2924 3.2709 0.3057
28º 0.4887 0.4695 0.8829 0.5317 1.8807 74º 1.2915 0.9613 0.2756 3.4874 0.2867
29º 0.5061 0.4848 0.8746 0.5543 1.8040 75º 1.3090 0.9659 0.2588 3.7321 0.2679
30º 0.5236 0.5000 0.8660 0.5774 1.7321 76º 1.3265 0.9703 0.2419 4.0108 0.2493
31º 0.5411 0.5150 0.8572 0.6009 1.6643 77º 1.3439 0.9744 0.2250 4.3315 0.2309
32º 0.5585 0.5299 0.8480 0.6249 1.6003 78º 1.3614 0.9781 0.2079 4.7046 0.2126
33º 0.5760 0.5446 0.8387 0.6494 1.5399 79º 1.3788 0.9816 0.1908 5.1446 0.1944
34º 0.5934 0.5592 0.8290 0.6745 1.4826 80º 1.3963 0.9848 0.1736 5.6713 0.1763
35º 0.6109 0.5736 0.8192 0.7002 1.4281 81º 1.4137 0.9877 0.1564 6.3138 0.1584
36º 0.6283 0.5878 0.8090 0.7265 1.3764 82º 1.4312 0.9903 0.1392 7.1154 0.1405
37º 0.6458 0.6018 0.7986 0.7536 1.3270 83º 1.4486 0.9925 0.1219 8.1443 0.1228
38º 0.6632 0.6157 0.7880 0.7813 1.2799 84º 1.4661 0.9945 0.1045 9.5144 0.1051
39º 0.6807 0.6293 0.7771 0.8098 1.2349 85º 1.4835 0.9962 0.0872 11.4301 0.0875
40º 0.6981 0.6428 0.7660 0.8391 1.1918 86º 1.5010 0.9976 0.0698 14.3007 0.0699
41º 0.7156 0.6561 0.7547 0.8693 1.1504 87º 1.5184 0.9986 0.0523 19.0811 0.0524
42º 0.7330 0.6691 0.7431 0.9004 1.1106 88º 1.5359 0.9994 0.0349 28.6363 0.0349
43º 0.7505 0.6820 0.7314 0.9325 1.0724 89º 1.5533 0.9998 0.0175 57.2900 0.0175
44º 0.7679 0.6947 0.7193 0.9657 1.0355 90º 1.5708 1.0000 0.0000 -------- 0.0000
45º 0.7854 0.7071 0.7071 1.0000 1.0000

798 -
sBVéf¶kareRbIR)as;xñatmaneRcInsn§wksn§ab;Nas; edIm,IbMeBjeTAnwgkarrIkceRmInénviTüasaRsþ .
minEt b:uNÑaHxñatmYycMnYnRtUv)anCMnYsedayxñatfµIepSgeTot edIm,I[RsbeTAnwgsþg;daGnþrCati. enHehIyCa
mUlehtuEdl naM[xñatFmµtamYycMnYn)an)at;bg;bnþicmþg² mYyvijeTot xñatmYycMnYnFMenAEteRbIR)as;enAkñúg
CIvPaBrs;enAsBVéf¶ b:uEnþKµanÉksarNa )anniyaylm¥it nigrebobeRbIxñatTaMgenaHeLIy. CaBiessedIm,I
kMu[sñaédExµr)at;bg; EdlbuBVburs )anbnSl;Tuk eTIb´)anerobcMeLIgvij GMBIxñatCaeRcIn CamYykarRsavRCav
Éksarmintic . kñúgEpñkenHnwgmankarbgðajxñatepSg²CaeRcInrYmman ³
1> xñatRbEvg

BhuKuNén m 1 =1
1 = 16 km 1 =2m
1Tm  1012 m
1 = 4 km 1 inch  1.54 cm
1Gm  109 m
1 = 40 m 1 foot  30.48 cm
1Mm  106 m
1 = 20 m 1 yard  0.9114 m
1Km  103 m
1 = 0.50 m 1 feet  0.3038 m
GnuBhuKuNén m 1 = 0.20 m 1 mile  1609.344 m
1mm  103 m 1 = 0.10 m 1 yard = 3 feet (
1μm  106 m 1 = 0.01 m 1 mile = 1760 yd
1nm  109 m 1 = 100 1 mill , mrin  1852m
1pm  1012 m 1 = 400 1 m  1010 Ao (

Km hm dam m dm cm mm
… …

799 -
2> xñatépÞRkLa

- m2 - m2 -

1 Tm 2  1024 m 2 1 mm 2  106 m 2 1 ha = 10 000 m 2


1 Gm 2  1018 m 2 1 μm 2  1012 m 2 1a ( = 100 m 2
1 Mm 2  1012 m 2 1 nm 2  1018 m 2 1 ca = 1 m2
1 Km 2  106 m 2 1 pm 2  1024 m 2

… Km2 hm2 dam2 m2 dm2 cm2 mm2 …


ha a ca

3> xñatmaD

1 Tm3  1036 m3 1 pm3  1036 m3 1 ml = 0.001


1 Gm  10 3 27
m 3
1  1 dm 3
1  1 000 ml
1 Mm  10 m 3 18
1 hl  100 1 mm3  1 cc
1 Km3  109 m3 1 cl = 0.01 1 ml  1 cm3
1 mm3  109 m3 1 dal = 10 1 = 30
18
1 μm  10
3
m 3
1 = 0.1 m 3
1 dl = 0.1
1 nm3  1027 m3

… Km3 hm3 dam3 m3 dm3 cm3 mm3 …


cc

800 -
4> xñateBl
s  j, h, mn , s

1 = 1000 1  h = 60  mn  1h = 3600 s (
1 = 100 1 mn ( ) = 60 s ( ) 1j( = 1440 mn
1 = 10 1s( ) = 60 ( ) 1j = 86400 s
1 = 365 6 1 ( ) = 60 1 = 8766 h

5> xñatm:as
1 kg = 1 000 g 1 = 15 kg ¬Ggár¦ 1 = 10
1 hg = 100 g 1 = 10
1 = 600 g
1 edkaRkam dag = 10 g 1 = 37.5 g
1 kg ( ) = 10

1 = 100 g
1 Rkam g = 0.001 kg 1 = 3.75 g
1 = 1000 kg
1 = 0.375 g
1 edsIuRkam dg = 0.1 g 1 = 907.18 kg
1 = 0.0375 g
1 sg;TIRkam cg = 0.01 g 1 =2
1 = 1016.05 kg

1 mIlIRkam mg = 0.001 g 1 =2
1  pound  = 0.453 kg

1 =4 1 N  100 g
1 = 60 kg
1 = 30 kg 1 = 16
1 = 30 kg 1 = 10

6> xñatsItuNðPaB
³  C
o
, K  ,  F
o

 K  t  273 ³K , t
180  oC
 o
F
100
 32 ³ F
o
,
o
C

7> xñatmMu
1o ( = 60 180º = 200 Grad 1º = 0.0174533 rad
1º = 1.111111 Grad
1 = 60 180º =  rad )
1 rad  63.662 Grad
1 D = 90º 1 rad  57 o1745  rad = 200 Grad

801 -
8> xñatepSg²
1 LÚ = 12 1 ry = 10 erol 1 )aj; = 100 g ¬fñaM¦
1 ekH ¬Rsaebo¦ = 24 kMb:ug 1 erol = 10 kak; 1eb ¬Ggár¦= 50 kg

1 b‘íc = 25 sug ¬)arI¦ 1 kak; = 10 esn 1 )avRkecA = 100 kg

1 sug = 10 kBa©b; ¬)arI¦ 1 dMbr = 4 ¬b¤5ebIKitk,ac;¦ 1 yYr¬TwksuT¦§ = 12 db

1 kBa©b; = 20 edIm¬)arI¦ 1 pøÚn = 10 dMbr 1 = 0.25 kg

1 sIu = 0.25 L lIt 1 søwk = 10 pøÚn

20

10

9> esckþIbEnßm

-xñatsemøgKitCa edsIuEbl
-xñatrBa¢ÜydIKitCa
-xñatfamBlGKÁisnIKitCa kw/h
-xñatkMlaMgKitCa jÚtun (N)
-xñatem:m:UrI KitCa éb
-cMnYnGaKuyKitCa RKab;
-xñatKitCa eRKÓg Qut sRmab; kMebø ér: >>> .

802 -

You might also like